Está en la página 1de 568

ÁLGEBRA

Ximena Carreño Campos


Ximena Cruz Schmidt
EDICIÓN Y PRODUCCIÓN:
Departamento Pedagógico Arrayán Editores S.A.
Actualmente compuesto por:
Dirección Editorial
Héctor Hidalgo González
Corrección de Estilo
Alejandro Cisternas Ulloa
Dirección Gráfica
Leonardo Vilches Robert
Diseño y Diagramación
Vinka Guzmán Tacla
Francisco Martínez Muñoz
Claudio Silva Castro

Participación Externa
Revisión de Contenidos:
Bernardita Cruz Schmidt

© Del texto: Ximena Carreño Campos y Ximena Cruz Schmidt.


© Arrayán Editores S.A.
Bernarda Morín 435, Providencia, Santiago de Chile. Teléfono: 431 4200
Fax: 431 4282 • e-mail: arrayan@arrayan.cl
Consultas: e-mail: editorial@arrayan.cl
Obra: Álgebra Arrayán.
I.S.B.N: 956-240-168-5. Inscripción: 87.879. Segunda edición, noviembre 2002.
Impreso en Chile por Morgan Impresores.
Prohibida su reproducción total o parcial, a través de cualquier sistema de reprografía o tratamiento
informático, bajo las sanciones establecidas por la ley.
Introducción

En este libro de ejercicios de ÁLGEBRA hemos querido proponer una


cantidad de trabajos que va desde los ejercicios más tradicionales
para el aprendizaje del álgebra hasta los problemas más modernos
y desafiantes que invitan al estudiante y al maestro a conversar y
discutir en torno a posibles soluciones.
Creemos sinceramente estar haciendo un aporte para colaborar con
aquellos estudiantes que se interesen en afianzar sus conocimientos
y sentar las bases de una sólida formación matemática.
Estimado lector: queremos invitarlo a recorrer estas páginas en
el orden que usted estime conveniente y de acuerdo con las
necesidades que se le vayan presentando. En estas líneas vamos
a tratar de darle una visión global del ámbito de trabajo de la
aritmética y del álgebra.
Nuestro mundo numérico se fue generando a lo largo de los
siglos según los hombres iban necesitando de diversos modos de
comunicación y de acuerdo con los requerimientos de otras áreas de
acción, como el comercio, la astronomía, la agricultura, el desarrollo
de las diversas ciencias, la matemática por sí misma y una infinidad
de actividades en que el hombre se ha interesado por crear su
expresión en términos numéricos.
En la página siguiente encontrará un esquema que contiene los
distintos conjuntos de números y la forma como los matemáticos los
han ido ordenando de acuerdo con distintos criterios; y más adelante
verá un gráfico de los diferentes conjuntos numéricos.
El objetivo que nos hemos propuesto al escribir esta introducción
y proponerle algunas actividades es que usted se forme una idea
global de los distintos ámbitos en que se mueve la aritmética y, como
consecuencia, el álgebra, que no es otra cosa que la descripción
de modelos matemáticos para representar múltiples situaciones de
la naturaleza y/o generaciones abstractas del matemático. Estos
modelos son las distintas relaciones entre variables, que al asignarles
los valores adecuados y haciendo los análisis pertinentes nos entregan
potentes herramientas para resolver problemas tradicionales, como
la trayectoria de un proyectil, que se puede describir a través de una
ecuación de segundo grado, u otros, como el uso de matrices para
organizar y manipular gran cantidad de información.

Introducción 3

1-6 (2003) 3 25/11/02, 10:58 AM


Le vamos a pedir que observe con mucha detención el esquema
titulado Conjuntos Numéricos y analice con sus compañeros
estudiantes o con sus profesores toda la información que pueda
obtener de él. No sería extraño que la primera vez no logre recoger
mucha información, pero con el tiempo, y conforme el avance
en sus conocimientos, debería servirle de gran ayuda para tener
una visión global de los ámbitos numéricos que el hombre ha ido
definiendo y entender por qué los ha ordenado de esta manera
y no de otra.
Lo invitamos a observar el esquema propuesto y a reflexionar en
torno a la información que contiene.

A continuación le entregamos la misma información pero con otra


presentación y lo invitamos a que usted ubique correctamente, en
el conjunto correspondiente del “ESQUEMA DE LOS CONJUNTOS
NUMÉRICOS”, los números que listamos más adelante. El profesor
podrá inventar una infinidad de actividades para determinar si sus
alumnos(as) se ubican bien en los distintos conjuntos numéricos.
Por ejemplo: ¿Cuáles fueron los primeros números inventados?;
¿para qué servían?; ¿cómo se expresa la ausencia de valor?; ¿qué
operaciones aritméticas están definidas en cada conjunto?; ¿por qué?;
¿qué conjuntos son subconjunto de otros?; ¿cuáles son disjuntos?;
¿qué necesidad del hombre inspiró la ampliación de los Naturales
a los Enteros?; ¿y a los Racionales?; ¿qué ejemplo concreto puede
dar de un número irracional?; ¿cómo lo puede ubicar en la recta
numérica?; ¿qué diferencia hay entre una fracción y una razón?;
¿cómo se generaron los números Complejos?; ¿dónde y para qué se
usan?; ¿cómo se grafican?; etc.

4 Introducción

1-6 (2003) 4 25/11/02, 10:58 AM


Aquí hay una cantidad de números y usted deberá determinar a qué
conjunto numérico pertenece y ubicarlo en el esquema siguiente.
1
a) 3; 2 ; 2,6; 8
; –3; 5; 12; 36; – 15 ; 2i; 0; –3,5;
3 4 3
32
1 ; 0,5; 12; –
8
1 12
b) 5; –1; 16; 32; – ; 25; (1,3); 2;
4
; 2 – 5i;
4
3 6 ; –1,32; 5 ; 12 ; 7
8;
7 4 10
1 1 22
c) ; 6; 5–1; 6 ; –15; 3 2 ; (4, –1); 3 ; 4 + 2i; – ;
3 3 11
5
8,3; (–1)4; ; – 144
6
1 18 3 3 25 1
d) –3,2; 9; ; ; 9 ; 0; 12 ; ; 32; 6–1; ;
9 5 9 5 2
3 2 ; 0.02; 3%
27 ; 12,3;
100

e) 12; 2–3; 3–2; –(2)5; –(3)12; 6 ; 3 1; 5 ; 12;


3 12 5
– 36; –5i; –1 ; 0,16; 100%

–1
f) 50%;
1
2
; 0,5;
1
4
; ( 4 )–1; 3
6
; ( 147 ) ; (( 2 ) ) –1 2 1
–2i 2 ;

(( ) )
2
2 1 2
; 2 ;
8 2 4

Esquema de los Conjuntos numéricos

C
Q R
Z
N I

Introducción 5

1-6 (2003) 5 25/11/02, 10:58 AM


Con el objeto de que el estudiante pueda formarse una idea completa
de lo que abarca el Álgebra abordada en el texto, le proponemos, a
continuación, un resumen esquemático que puede ayudar a tener una
idea general de los contenidos.

Mapa de contenidos del Algebra

FUNCIONES MATRICES
ÁLGEBRA
COMPLEJOS

CANTIDADES
REALES ESCALARES VECTOR EN R2

EXPRESIONES
ALGEBRAICAS CANTIDADES
VECTORIALES
TÉRMINO
ALGEBRAICO
POLINOMIO
COMPARACIONES

COEFICIENTE GRADO

IGUALDAD DESIGUALDAD
VARIABLE

ECUACIONES INECUACIONES
OPERACIONES
INTERVALO
EN R
LOGARITMO SISTEMAS DE SISTEMAS DE
ADICIÓN
ECUACIONES INECUACIONES

PRODUCTO RAÍCES
PROBLEMAS DE
POTENCIA OPTIMIZACIÓN

Obsérvelo, comente con sus compañeros y profesores lo que encuentre


en él; critíquelo y envíe sus observaciones al correo electrónico
ximenacs@entelchile.net.

En el texto hemos querido entregarle referencias para desarrollar sus


estructuras mentales, pero sin duda esto no se logrará si no se desea y
trabaja con esfuerzo y persistencia. Es probable que alguna vez haya
escuchado decir que el desarrollo del pensamiento es un proceso
interior de la persona. Efectivamente, el mundo circundante, cercano
o lejano físicamente, las inquietudes personales, las expectativas en
la vida, la disposición a trabajar son las únicas herramientas que lo
pueden llevar a desarrollar su capacidad de pensar y a enriquecer
sus estructuras mentales. Como usted sabe, el aprendizaje se produce
cuando relacionamos algo novedoso con algo que ya sabemos; por
eso es que la persona cada vez que aprende, potencia más aún su
capacidad de aprender. Ponemos en sus manos este texto con la ilusión
de que sea un medio eficaz para enriquecer sus estructuras mentales
y su aprendizaje en general. En la medida que ello suceda, el texto
estará sirviendo efectivamente como un medio para el aprendizaje,
y así estaremos colaborando en su crecimiento como persona en
este mudo globalizado.

6 Introducción

1-6 (2003) 6 25/11/02, 10:58 AM


CAPÍTULO 1

CAPÍTULO 1
A
lgebra
en los
números reales

Lenguaje algebraico 1.1

El lenguaje algebraico se basa en el uso de letras y relaciones


matemáticas para generalizar diferentes situaciones.
Ejemplos:
• El perímetro P de un cuadrado de lado a P = 4a.

• El área A de un cuadrado de lado a A = a2.

• El área A de un triángulo de base b y altura h A=b•h


2
Cada una de las letras involucradas en las fórmulas anteriores
es una variable; a cada variable se le pueden asignar diferentes
valores.
En general, una variable es cualquier letra involucrada en
una expresión algebraica.
Expresemos en lenguaje algebraico:
1. El doble de un número 2a, 2x, 2m, ...
2. El triple de un número 3x, 3y, 3b, ...
p q z
3. La mitad de un número , , , ...
2 2 2
4. El cuadrado de p p2
5. a aumentado en b a+b
6. a disminuido en b a–b
7. El producto entre a y b a•b

Si en alguna expresión no está especificado el término,


podemos asignar cualquier variable para representar el enunciado,
como se puede ver en los ejemplos 1, 2, 3 y 4.

Álgebra en los números reales 7

7-13.(2003) 7 18/11/02, 10:32 AM


En general,
• Son múltiplos de a:

el doble 2a
el triple 3a
el cuádruple 4a
el quíntuple 5a
:
:

• Son fracciones de a:
a o 1
un medio (o la mitad) • a
2 2
a o 1
un tercio (o la tercera parte) • a
3 3
a o 1
un cuarto (o la cuarta parte) • a
4 4
a o 1
un quinto (o la quinta parte) • a
5 5
:
:

• Son potencias de a:

el cuadrado a2
el cubo a3
la cuarta potencia (o a la cuarta) a4
la quinta potencia (o a la quinta) a5
:
:

• Otras expresiones algebraicas:


Un número par 2n
Un número impar 2n – 1

Ejercicios Expresemos en lenguaje algebraico:

resueltos 1. El doble de un número, aumentado en la mitad del mismo número.


Aquí el “número” no está determinado; asignémosle la variable
x; nos queda:
x
2x +
2
2. El doble de a, aumentado en b
2a + b

3. El doble de a aumentado en b
2 (a + b)
Observe los ejemplos 2 y 3. ¿Cuál es la diferencia?

8 Álgebra en los números reales

7-13.(2003) 8 18/11/02, 10:32 AM


CAPÍTULO 1

4. La mitad de a más el triple de b.


Aquí ya están asignadas las variables, son a y b. Nos queda:
a
+ 3b
2
5. El doble del cuadrado de a.
2 a2

6. El cuadrado del doble de a.


(2a)2
Observe la diferencia entre los ejercicios 5 y 6.

7. La cuarta parte del triple del cuadrado de b.


3 b2
4
8. El triple de la cuarta parte del cuadrado de b.
b2
3 ( )
4

9. El cuadrado de la cuarta parte del triple de b.


3b 2
(4)
Observe las diferencias entre los ejercicios 7, 8 y 9.

10. La diferencia entre el quíntuple de x y la mitad de y.


y
5x –
2
11. La suma de tres números pares consecutivos.
(2n) + (2n +2) + (2n + 4)
o
(2n – 2) + (2n) + (2n + 2)
Observe la diferencia entre ambas.

12. Tres impares consecutivos.


2n – 1, 2n + 1, 2n + 3
2n + 1, 2n + 3, 2n + 5
Observe la diferencia entre ambas y exprese esos tres números
de una manera distinta.

13. La semisuma entre a y b.


a+b
2
14. La semidiferencia entre a y b.
a–b
2
15. El producto entre un número y su antecesor.
x (x – 1)
16. El producto entre un número y su sucesor.
x (x + 1)

Álgebra en los números reales 9

7-13.(2003) 9 18/11/02, 10:32 AM


Ejercicios

I. Asigne variables y exprese en lenguaje algebraico:

1. La mitad de un número.

2. El triple de a, aumentado en el doble de b.

3. El doble del cociente entre a y b.

4. El cubo de la diferencia entre x e y.

5. La diferencia entre el cubo de x y el cuadrado de y.

6. El cuadrado de a equivale a la suma entre el cuadrado de x y el cuadrado


de y.

7. La suma de tres números consecutivos es 213.

8. La suma de tres pares consecutivos es 168.

9. El cubo del cuadrado de la diferencia entre x e y.

10. La cuarta parte del producto entre el cuadrado de a y el cubo de b.

11. El triple de un número equivale al doble del mismo número aumentado


en 15.

12. El volumen de una esfera de radio r equivale al producto entre cuatro tercios
de p y el cubo del radio.

13. La superficie de un rectángulo cuyos lados miden (a + 3) y (a – 3).

14. El volumen de un cubo de arista 2a – 1.

15. El volumen del paralelepípedo de la figura

16. La superficie lateral del paralelepípedo de la figura.

17. La suma de los cuadrados de tres números consecutivos.

18. El cuadrado de la suma de tres números consecutivos.

10 Álgebra en los números reales

7-13.(2003) 10 18/11/02, 10:32 AM


CAPÍTULO 1

Soluciones

a 2 3
1. 10. a • b
2 4
2. 3a + 2b 11. 3x = 2x + 15
a 4
3. 2
b 12. V = p • r3
3
4. (x – y)3 13. S = (a + 3) (a – 3)

5. x3– y2 14. V = (2a – 1)3

6. a2 = x2 + y2
15. V = 2a(2a + 3)(2a + 1)
7. (a – 1) + a + (a + 1) = 213
a + (a + 1) + (a + 2) = 213 16. S = 2(2a (2a + 3) + 2a(2a + 1))

8. (2n – 2) + 2n + (2n + 2) = 168 17. x2 + (x + 1)2 + (x + 2)2

9. [(x – y)2]3 18. [x + (x + 1) + (x + 2)]2

Definición: Se llama término (algebraico) a un con-


junto de números y letras que se relacionan entre sí
por medio de la multiplicación y/o división.

3a 5 2 2
Ejemplo: 2a2 b , p , – x y z.
7
El término algebraico consta de un FACTOR NUMÉRICO, un
FACTOR LITERAL y un GRADO.
El grado es la suma de los exponentes de las letras que aparecen
en el término.
12 6 4 2
Ejemplo: En el término – a b c el coeficiente numérico es
17
12
– ; el factor literal es a6b 4 c2 y el grado es 12 (6+4+2).
17

Observación 1: Si el coefi ciente numérico no está escrito,


entonces es 1.
Observación 2: Si el grado no está escrito, entonces es 1.
Se llama EXPRESIÓN ALGEBRAICA a cualquier suma o resta de
términos algebraicos. Si la expresión tiene dos términos, entonces es un
BINOMIO; si tiene tres términos se llama TRINOMIO; si tiene cuatro o
más, hablamos de POLINOMIOS. (El término POLINOMIO se puede
usar en forma general para cualquier expresión algebraica.)

Álgebra en los números reales 11

7-13.(2003) 11 18/11/02, 10:33 AM


1.2 Valorización de
expresiones algebraicas

Las expresiones algebraicas no representan valores en sí, sino


que pueden ser evaluadas para distintos valores que se les asignen a
las letras que las componen.

Ejercicios 1. El valor del monomio a2b cuando a = 2 y b = 5


resueltos es 22 • 5 = 20.
Reemplazamos directamente las letras a y b por los valores
asignados; en este caso, 2 y 5, y realizamos las operaciones
indicadas.
2. El valor del mismo monomio a2b cuando a = 3 y b = – 4 es:
32 • (– 4) = 9 • – 4 = – 36
3. Si x = – 2; y = 5 y z = 4, el valor de
2x + 3y – z es:
2•–2+3•5–4=
– 4 + 15 – 4 = 7
4. Si m es el doble de n, n es el cuadrado de p y p = 3,
determinemos m y n:
Aquí tenemos: m = 2n; n = p2 y p = 3, entonces n = 32 = 9
y m = 2n = 2 • 9 = 18.
Así; n = 9 y m = 18.

Ejercicios
2
Determine coeficiente numérico, factor literal y grado de 7. a b
I. los siguientes términos algebraicos: 2 4
3a b
8.
2 3 8 5
1. 3 ab 4. 17 p q z
12
2 m n
2. – a 5. – 0,3 c 9.
5 9
11
2 2 x y
3. 0,02 a b 6. a 10. –
4

12 Álgebra en los números reales

7-13.(2003) 12 18/11/02, 10:33 AM


CAPÍTULO 1

II. Si a = 3 y b = 2, IV. Si x = 4, y = –2 y z = 5, determine


determine el valor de: el valor de:

1. 2 ab 1. 2 x + y + z
2 2 2. x – y – 2z
2. a – b
2
3. b – a
2 3. x + y – x + z

2 2 2 2 2
4. a + ab + b 4. x x + y + z

1 1
5. – 2ab 5. –
x y
3 3 2 2
6. a – b 6. 2 x y – 2 x z
5 2
7. – b 7. x – 1

8. 1 + a + b + ab 2
8. z – 2 + z – 3
2

2 2
9. a + b – a – b 9. 3 – x yz + 2 – x yz
3
b 2 4 z
10. a – –6 10. x – y +
4 5

Si m = –2 y n = + 3,
III. determine el valor de: V.
1. 2 m – 3n
1. Si m + n = 3 y n = – 1, determine m.
2
2. m – m – 2n
2. Si m – 3 = 2p y p = – 2 determine m.
3. 1 + m
3. p + q – r = 12 , r – q = 5, determine p.
2 2
4. m – n 4. 2a – 9 = b y a = – 3, determine b.
5. m + n m – n
5. 1 + 2a = b – 2 y a = – 2, determine b.
2 2
6. m + 2 mn + n 6. Si a es el doble de b, b es un tercio de c
y c = 12, determine a y b.
7. – 5 mn
7. Si m es la cuarta parte de p y p es el
1 1 cuadrado de 2, determine m.
8. –
m n
8. La mitad de a es 1. ¿Cuál es el valor de a?
1
9.
m– n 9. La tercera parte del doble de m es 4. ¿Cuál
es el valor de m?
–1
10.
mn 10. Si p + q = 2r, q es el triple que p y p = 5,
¿cuál es el valor de r?

Álgebra en los números reales 13

7-13.(2003) 13 18/11/02, 10:33 AM


Soluciones

I. 1. 2. 3. 4. 5. 6. 7. 8. 9. 10.
Coeficiente 2 3 1 1
numérico 3. – 0,02 17 – 0,3 1 1 –
5 5 9 4
Factor literal ab a a2b2 p2q3z8 c a a2b a2b4 m12n x11y
Grado 2 1 4 13 1 1 3 6 13 12

II. 1. 12 2. 5 3. – 5 4. 19 5. – 12 6. 19 7. –32 8. 12 9. 8 10. –5


5 1 1
III. 1. – 13 2. – 12 3. – 1 4. – 5 5. –5 6. 1 7. 30 8. – 9. – 10.
6 5 6
3
IV. 1. 11 2. – 4 3. – 7 4. 180 5. 6. – 264 7. 15 8. 45 9. 85 10. 1
4
V. 1. m = 4 2. m = – 1 3. p = 17 4. b = – 15 5. b = – 1 6. a = 8 b = 4 7. m = 1

8. a = 2 9. m = 6 10. r = 10

1.3 Reducción de términos


semejantes y uso de paréntesis

Definición: Se llaman TÉRMINOS SEMEJANTES aquellos que


tienen el mismo factor literal (y por consiguiente el mismo
grado); sólo pueden diferir en el coeficiente numérico.

Ejemplo 1. Son términos semejantes:


a2
a2, 2a2, –3a2, 0,5a2,
4
Ejemplo 2. No son términos semejantes:

a2 b y ab2, –a y –a2, 2ab y ab2,


Vemos que en el ejemplo 1, el factor literal de todos ellos es a2;
por esta razón son todos semejantes.
En el ejemplo 2, en cambio, tenemos en los tres casos factores
literales diferentes entre sí.
En una expresión algebraica SÓLO podemos reducir aquellos
términos que son semejantes y esto se efectúa sumando (o restando) los
coeficientes numéricos y manteniendo el factor literal.
El uso de paréntesis es frecuente en álgebra. Sirve para separar
expresiones algebraicas y se elimina de acuerdo con las siguientes
reglas:
1. Si está precedido de un signo + o no tiene signo escrito, se
elimina sin hacer ningún cambio.
2. Si está precedido de un signo – se elimina después de cambiar
TODOS los signos de los términos del interior del paréntesis. (Es

14 Álgebra en los números reales

14-15. 14 18/11/02, 10:40 AM


CAPÍTULO 1

importante hacer notar que al eliminar el paréntesis también se


elimina el signo – que lo antecede.)
Si una expresión algebraica contiene paréntesis, es conveniente
eliminarlo antes de proceder a reducir los términos semejantes.

1. a + 2a + 3a Ejercicios
Los tres términos de la expresión son semejantes; por lo resueltos
tanto, sumamos sus coefi cientes numéricos y conservamos
el factor literal:
a + 2a + 3a = 6a
2. 2a + 3b – 5a + 6b
Aquí los términos 2a y – 5a son semejantes entre sí y lo mismo
ocurre con 3b y 6b; entonces los podemos agrupar entre sí
y obtenemos:
2a + 3b – 5a + 6b = (2a – 5a) + (3b + 6b) = – 3a + 9b

3. 3x6y – 5xy6 – 7x6y – x6y + 11xy6


Agrupamos los términos según su semejanza y obtenemos:
(3x6y – 7x6y – x6y) + (– 5xy6 + 11x y6) = – 5x6y + 6xy6
4. 5m + (3m – 7n) – 2n
Antes de proceder a la reducción de términos es necesario eliminar
el paréntesis; como éste está precedido de un signo +, lo eliminamos
sin hacer cambios y obtenemos:
5m + 3m – 7n – 2n = 8m – 9n

5. 3x2y – (x2y – 2xy2) + 3x2y


En este caso, al eliminar el paréntesis (y el signo que lo precede)
debemos cambiar los signos de los términos del interior; nos
queda:

3x2y – x2y + 2xy2 + 3x2y


(3x2y – x2y + 3x2y) + 2xy2 = 5x2y + 2xy2

6. a + a2 + a3 + a4
Aquí no es posible hacer ninguna reducción pues no existen
términos semejantes.
Si en una expresión nos encontramos con paréntesis dentro de otros
paréntesis, procedemos a eliminarlos desde dentro hacia afuera
atendiendo a la misma regla.

Álgebra en los números reales 15

14-15. 15 18/11/02, 10:40 AM


7. 2ab – [3a – (–2ab + 3a) – ab]
Eliminamos primero el paréntesis interior:
2ab –[3a + 2ab – 3a – ab]
Ahora eliminamos el exterior:
2ab – 3a – 2ab + 3a + ab
(2ab – 2ab + ab) + (– 3a + 3a) = ab

Ejercicios

I. Reduzca las siguientes expresiones:

1. m + 2m
2. a + 2a + 9a
3. m2 – 2m2 – 7m2
4. 6x2y2 – 12x2y2 + x2y2
5. 3a – 2b – 5b + 9a
6. a2 + b2 – 2b2 – 3a2 – a2 + b2
7. x2yz + 3xy2z – 2xyz2 – 3xy2z + xyz2 – x2yz
8. 2pq + 3p – 12q – 15q + 7pq – 13p
9. 2x – 6y – 2x – 3y – 5y
10. 15a + 13a – 12b – 11a – 4b – b

a a a
11. + +
2 3 4
2
a2b 2ab2 3ab2 6a b
12. – + –
5 3 2 5
m 2m m
13. m – + –
2 3 4
3a – b 3a – b
14. +
2 5
3 3
15. 2p + q – 7p + q
4 2
16. a + a2 + a3 + a4 – a – 2a2 + 3a3 – 4a4
17. 0,2 m – 0,02n + 1,07m – 1,03n – m – n
18. 0,5x2y – 0,4xy2 + 0,3x2y – 0,2xy2 + x2y
19. 1,17a – 2,15a – 3,25a + 4,141a

16 Álgebra en los números reales

16-17. 16 18/11/02, 10:41 AM


CAPÍTULO 1

20. 1 + x + xy – 2 + 2x – 3xy – 3 + 2xy – 3x


1 2 2 3 3 2 8
21. m n – mn– m2n + m n – mn
5 3 2 10 3
27 35 1 1
22. p– q+ p– q
4 6 4 6
2 2 2 2
23. u + uv + v – 2u + 3uv – v
11 3 2 1 5 1
24. s– t+ s– s– s+ t+ t
3 4 3 3 3 4
25. 0,117a – 0,35b – 2,25b – 1,1b + 3,04a
2 3 3 2
26. 10a + 5a – 13a – 2a – 9a + 16a + a
1 2 3 2 3 7 1
27. pt – p – t + pt – p + t + pt
6 5 4 3 5 4 6
2 2 2 2 2 2 2 2
28. x yz – xy z + xy z – x y z
3 2 2 2 2 2 1 2
29. a b – ab – a b – 3ab + ab
4 3 2
1 3
30. 0,7m – p – 0,04m + 0,3p – p
7 4

II. Elimine paréntesis y reduzca los términos semejantes:

1. a + b + a – b

2. a + b + b – a

3. a – b + a + b

4. a – b – a + b

5. 2a – 2a – 3b – b

6. 3x + 2y – x – x – y
7. 2m – 3n – – 2m + n – m – n

8. – a + b – c – – a – b + c + a – b + c
2 2 2 2 2 2 2
9. – x – y + 2x – 3y – x – 2x – 3y

10. – – a – 2b – a + 2b – – a – 3b

11. 3x + 2y – 2x – 3x – 2y – 3x – 2x – y

12. 3y – 2z – 3x – x – y – z – x – 2x
1 2 3 4
13. a – b– a – b
2 3 4 3

1 1 2
14. a – a– a–a
5 2 3

Álgebra en los números reales 17

16-17. 17 18/11/02, 10:41 AM


Ejercicios

15. 3 x + 2 y – x – 2y – 1 y – 2 x
4 5 5 3

16. a b
a– – –b
2 2

17. a – b – a – b + a + b
2 2
1 a b
18. 1 + a + b – + +
2 3 4

11 2 3 2 15 2 3 2 1 2 12 2 9 2
19. x – y – x – x – y – y – y
4 25 4 4 25 25 25
20. Si P = x2 + 3x – 2 y Q = 2x2 – 5x + 7, obtenga P + Q.

21. Si P = 3x – x2 y Q = 3x2 – x, obtenga Q – P y P – Q.

22. Si M = 2a2 + 3a3 + a4 y N = a4 – 3a2 + 2a,


obtenga M + N y M – N.

23. Si P = x3 – 5x2 – 1; Q = 2x2 – 7x + 3 y R = 3x3 – 2x + 2,


obtenga P + Q – R y P – (Q – R).

24. Si P = m6 + m3 – m; Q = m5 + 2m4 – 3m3 + 2m


y N = m6 + m5 – 2m3 + m, obtenga P + Q – N y N – P.

25. Si A = ab + 2b; B = a – ab y C = a + b + ab,


encuentre A + B + C ; A + B – C y A – (B + C).
a+b a–b
26. Si P = y Q= , entonces encuentre el valor de P + Q.
2 2
1 1 2 2 3 2
27. Si P = a – b– c y Q= a + b + c, encuentre Q – P.
2 3 4 3 2 4
28. Si A = 2x3 + 3x2 – 2x + 5 y A + B = x3 – 3x2 + x – 4, encuentre B.

29. Si A = 3x3 – 2x2 + 5x – 1; B = 2x3 – 3x – 3


y A – B + C = x3 – 2x2 – 3x – 2, encuentre C.

30. Si P = 1 – x3; Q = 1 – x2; R = 1 – x, determine P – (Q + R + 3).

Soluciones

I. 1. 3m 2. 12a 3. – 8m2 4. – 5x2y2 5. 12a – 7b 6. – 3a2 7. – xyz2


13a
8. 9pq – 10p – 27q 9. – 14y 10. 17a – 17b 11. 12. – a 2b + 5 ab2
12 6
11m 7 9
13. 14. 3a – b 15. – 5p + q 16. – a2 + 4a3 – 3a4 17. 0,27m – 2,05n
12 10 4
10
18. 1,8x2y – 0,6xy2 19. – 0.089 a 20. – 4 21. – m2n – mn 22. 7p – 6q
3
7 1
23. – u2 + 4uv 24. s+ t 25. 3,157a – 3,7b 26. 9a + 21a2 – 22a3
3 2
1 2 19 2 33 83
27. pt – p + t 28. x2yz – x2y2z2 29. – a b– ab 30. m– p
4 6 50 140

18 Álgebra en los números reales

18-19.(2003) 18 18/11/02, 10:43 AM


CAPÍTULO 1

II. 1. 2a 2. 2b 3. 2a 4. – 2b 5. 2b 6. 3x + y

7. 5m – 5n 8. a – b + c 9. 2x2+ y2 10. a – 3b 11. 5x + y 12. 4y – 3z – x


–19a 13 11 3a b
13. – 1 a + 2 b 14. 15. y– x 16. a + b 17. +
4 3 30 5 12 2 2 2 2
1 2 3
18. + a + b 19. – 7 x 2 – y 2 20. 3x2 – 2x + 5
2 3 4 4

21. Q – P = 4x2 – 4x 25. A + B + C = 2a + 3b + ab


P – Q = 4x – 4x2 A + B – C = b – ab
A – (B + C) = ab + b – 2a
22. M + N = 2a4 + 3a3 – a2 + 2a
26. P + Q = a
M – N = 5a2 + 3a3 – 2a
27. Q – P = 1 a + 11 b + c
23. P + Q – R = 2x3 – 3x2 – 5x 6 6
P – (Q – R) = 4x3 – 7x2 + 5x – 2 3 2
28. B = – x – 6x + 3x – 9

24. P + Q – N = 2m4 29. C = – 11x – 4


N – P = m5 – 3m3 + 2m 30. P – (Q + R + 3) = –x3 + x2 + x – 4

Multiplicación algebraica 1.4

Multiplicación de potencias.
La expresión an se llama potencia de base “a” y exponente
“n”. Se cumple:
an • am = an + m
(an)m = an • m
a0 = 1 con a 0
(ab)n = an • bn

Multiplicación de 2 o más monomios.


Multiplicamos los coeficientes numéricos y los factores literales
entre sí (hacemos uso de las propiedades asociativa y conmutativa
de la multiplicación).
Multiplicación de un monomio por un polinomio.
Multiplicamos el monomio por cada término del polinomio
(hacemos uso de la propiedad distributiva de la multiplicación
respecto de la adición).
Multiplicación de dos polinomios.
Multiplicamos cada término del primer polinomio por cada
término del segundo. Siempre que sea posible, es necesario reducir
términos semejantes.

Álgebra en los números reales 19

18-19.(2003) 19 18/11/02, 10:43 AM


Ejercicios 1. a6 • a7 = a 6 + 7 = a13
resueltos 2. (ab)4 = a4 • b4
3. x5 • x9 • x4 = x5 + 9 + 4 = x18
4. 2a2 • 3ab = 2 • 3 • a2 • a • b = 6a3b
5. – 5x2 y4 • – 3x6 • – 2y6 = – 5 • – 3 • – 2 • x2 • x6 • y4 • y6 = – 30x8 y10
6. – 4a2b (a2 + ab – b) = – 4a2b • a2 – 4a2b • ab – 4a2b • (– b)
= – 4a4b – 4a3b2 + 4a2b2

7. (3m5 – 2m4 – mp) • – 3m = 3m5 • (– 3m) – 2m4 • (– 3m) – mp • (– 3m)


= – 9m6 + 6m5 +3m2p

8. (2x + y) (3x + 2y) = 2x (3x + 2y) + y (3x + 2y)


= 2x • 3x + 2x • 2y + y • 3x + y • 2y
= 6x2 + 4xy + 3yx + 2y2
= 6x2 + 7xy + 2y2

(los términos 4xy y 3yx son semejantes, por lo tanto deben


reducirse).

Ejercicios
Efectúe las siguientes
I. operaciones:
14. m2p • – m

1. a2 • a3 15. abc • 2abc

2. m3 • m4 • m5 16. 3x2y • x3y6 • – y

3. x2 • x3 • x3 17. – 4abc • – 3a2b2 • 12ab5c7

4. a • ab 18. 2pr • 3pr5 • pr2 • 7p3r4

5. xy • x2y 19. – 6x3 • – 6x3

6. a • a2b • a3b2 20. – 2ax4 • – 3ax5 • – 3a2x4

7. 2a • ab6 21. an • an + 1

8. 3xy2 • 5x2y3 22. 2am • 3an

9. 2m • 5n 23. xp + 1 • xp – 1

10. ax • – axy 24. p2x • p3x – 2 • px + 9

11. – 2x • 3xy • – 2x 25. 2a • 2a – 3 • – 2a – 9

12. – 3a2b • – 5abc • c4 26. a2n – 3 • a3n – 2 • a2 – 3n

13. 7abc • – 2a2bc8 27. a2x – 5 • bx + 1 • a2x + 2 • bx – 1

20 Álgebra en los números reales

20-21. 20 18/11/02, 10:45 AM


CAPÍTULO 1

28. pa • pa + 2 • q2a – 3 • q5 – 3a 11. – 3a6 b2(– ab3 + ab + a4b6) – 3a7b3(b2 – 1)

29. ax – 4 • bx + 4 • c2x • ax • b2x • cx + 2 12. 20 abc(a + b – c)

30. (ab)5 • a4 • b2 13. a5b2 – a5(a2 – ab + b2)


14. 3x6y4(x2 + xy + y2)
31. (mp)3 • (mp)2 • mp
15. – 3b(2ab + b2 + 5bc)
32. (2x)x + 1 • (2x)x + 2 • (2x)x – 3
16. 7a6b8c9(2abc – 5a2b + 4ab2c2 – abc3)
33. (m2n)5 • m5 • n6
17. (x6y21 – 4xy11 – 9x10y2) • – 3x6y2
34. (a2)3 • (a3)4 • a6
1 3 2
18. 2 x 4 x – 3 y
35. 2x • (2x)6a – 2 • (2x)3a + 4
1 2 1 3 2
1 3 1 2 19. – a ab + ab
36. a • a • 5a6 3 2 5
2 3
37.
2 4 3 7
b • b •–
4 4
b 20. 3 x 2y 6 2 xy 4 + 4xy 2 – 1
3 8 3 4 5
6 3 2 15 6 5 21. 8 p2q 1 pq – 1 pq3 + 2pq
38. – x y • x y
5 4 3 4 5
8 6 4 2 2 3 3 2 5 11
39. – a b • ab c • – a b c 22. – 1 a 2 b3 c 6 abc – 8a 2 b2 c 2
9 5 4 8
40. 0,1a6b7c4 • 0,02abc4 • 0,1a2b
23.– 3 x 6y 2z 4 1 – xyz 4 + 2 x 4y 2z 6
41. 0,03a5b4 • 1,3a4b8 • 2,7ab6 5 3
3 7 2 6 2 4 2 6 2
42. 0,5xyz4 • 2,1x2yz • – 3,1x6 24. –
4
m n 14m n – mn – m n
3 9
43. 1,03a4b • – 1,3a3b4 2 2 2 2
25. x y x y – xy
5
44. 0,06m2n6p2 • 0,6mn6p4
2 6 12 26. – 1 a 6b4c 3 4 ab2 – 4 a 3b2 – 1 a
45. a • b • – 3a4b5 • 0,5a2b4 2 5 7 4
5 27. 0,03a6b2 (1 – a2b2 – 0,03ab3)
28. – 0,5m4n2 (– 0,5m6n – 2mn3 + 3,5mn3)
II. Monomio por polinomio:
29. 0,07a4b2 (100ab4 – 10ab3 – 2ab)
1. 3a (a – 2b) 30. 1,2x6y11 (2,1xy9 –1,1x2y2 + 2,1xy8)
2. – 5x (2 – 3x2 – 5x) 31. 0,5abc (a2 – b2 – c2) + 4,8abc (a2 – b2 – c2)

3. 7b (2a – b) 32. – 2,2x6y3z (1,1xyz – 1,2x2y2z2 + 3xyz3)

4. 3x2 (3x6 – 2x4 + x3 – 2x + 3) 33. 3 p2qr12 – 3 p2qr3 + 3 pqr6


4 5 4
5. – 6x5y3 (3x2y – 4xy4 – 2x2 y2) 2 11 10 2 35 6 2
34. – m n p 10m n – m n +2
5 8
6. (4xy – 5xy4) • – 6xy
17 8 6 3 6 11 27 6 8
35. – x y 1– x y – x y
7. (3m2 – 2mn + n6) • 13m4n2 9 4 34
8 2 4 2 3 4 11
8. – 15m2np4 (mn6p2 – m4n 4p2 + mnp) 36. – 3 x y x y – xy + y
9. 6m2(2m – 5n) – 3m(6m2 + 4n) 12 4 2 7 5 2 11 2
37. – a b c – a bc – 10 abc – 4ab
10. p2q4(2pq – pq3 – 1) + 3p3q2 (q3 – q5 + p2) 5 4

Álgebra en los números reales 21

20-21. 21 18/11/02, 10:46 AM


Ejercicios

III.

1. (x + y) ( x2 + y2) 18. (2p – 4) (2p + 7)

2. (2a + b ) (3a – 2b) 19. (2x – 3y – 4z) (x + y + z)

3. (1 – x) (1 – y) 20. (x2 + y2 – z2)(2x – 3y – 4z)

4. (2x – 6y) (x2 – 2xy) 21. (a + 1) (an + an + 1 + an + 2)

5. (x2 + 3x2y) (– 3xy2 + 4xy3) 22. (a – 1) (an – 1 + an + an + 1)

6. (4x + y) (– 2x – 5xy) 23. (u – v) (u2 – 3uv + v2)

7. (6a – 5b) (2b + 7a) 24. (x + y) (x 2 + 2xy + y2)

8. (a + b + 1) (a – b) 25. (– 3x + y2) (x2 – xy – y)

9. (2a – 3ab + b2) (b – b2) 26. (2y + 3x) (x2 – xy + 2y2)

10. (5x2y + 2xy2 – 3xy) (x – y2) 27. (– 3x – 2y + z) (x + y – 3z)

11. (m2 + n2 – mn) (2m – 3n) 28. (x – y) (x2 + xy + y2)

12. (– 3xy – 2xy2) (xy2 – 5xy) 29. (x + y) (x2 – xy + y2)

13. (2p2q + 3pq11 – 5pq4) (– 3pq + 2p) 30. (a + b) (a4 – a3b + a2b2 – ab3 + b4)

14. (x2 + 1) (x2 – 1) 31. (a – b) (a3 + a2b + ab2 + b3)

15. (a + b) (a – b) 32. (x + y) (xn – 1 + xn – 2 + xn – 3)

16. (x + 4) (x – 6) 33. (p2 – q2) (pn – pnqn – qn)

17. (a2 + 5) (a2 + 7)

Soluciones

I. 1. a5 2. m12 3. x8 4. a2b 5. x3y2 6. a6b3


7. 2a2b6 8. 15x3y5 9. 10 mn 10. – a2x2y 11. 12x3y 12. 15a3b2c5
13. – 14a3b2c9 14. – m3p 15. 2a2b2c2 16. – 3x5y8 17. 144a4b8c8
18. 42p6r12 19. 36x6 20. –18a4x13 21. a2n + 1 22. 6am + n 23. x2p
24. p6x + 7 25. – 23a – 12 26. a2n – 3 27. a4x – 3 b2x 28. p2a + 2 q2 – a
29. a2x – 4b3x + 4c3x + 2 30. a9b7

22 Álgebra en los números reales

22-23. 22 18/11/02, 10:47 AM


CAPITULO 1
CAPÍTULO

5 11
31. m6p6 32. (2x)3x 33. m15n11 34. a24 35. (2x)9a + 3 36. a
6
1 15 4 9 11 14
37. – b 38. – 9 xx9 y7 39. a b c 40. 0,0002a9 b9 c8 41. 0,1053a10 b18
3 2 15
3 12 21
42. – 3,255x9 y2 z5 43. – 1,339a7 b5 44. 0,036m3n12 p6 45. – a b
5

II. 1. 3a2 – 6ab 2. –10x + 15x3 + 25x2 3. 14ab – 7b2


4. 9x8 – 6x6 + 3x5 – 6x3 + 9x2 5. –18x7y4 + 24x6y7 + 12x7y5
6. – 24x2y2 + 30x2y5 7. 39m6n2 – 26m5n3 + 13m4n8
8. – 15m3n7p6 + 15m6n5p6 – 15m3n2p5 9. – 6m3 – 30m2n – 12mn
10. 5p3q5 – 4p3q7 – p2q4 + 3p5 q2 11. – 3a10b8 12. 20a2bc + 20ab2c – 20abc2
13. – a7 + a6b 14. 3x8y4 + 3x7y5 + 3x6y6 15. – 6ab2 – 3b3 – 15b2c
16. 14a7b9c10 – 35a8b9c9 + 28a7b10c11 – 7a7b9c12
3 2 1 1 1 2
17. – 3x12y23 + 12x7 y13 + 27x16y4 18. x – xy 19. – a3b – a3b
8 3 6 5
3 3 10 3 2 3 2 8 3 4 16 3 2
20. x y + 3x3 y8– x2 y6 21. pq – pq + pq
10 4 3 15 3
1 3 4 7 4 5 8 3 6 2 4 3 7 3 8 2 10 4 10
22. – a b c +a b c 23. x y z – x y z + x y z
8 5 5 5
21 13 3 1 8 6 1 13 4 2 4 2 2 3 3
24. – m n + m n + m n 25. xy – xy
2 2 6 5 5
2 7 6 3 2 9 6 3 1 7 4 3
26. – abc + abc + abc 27. 0,03a b – 0,03a8b4 – 0,0009a7b5
6 2
5 7 8
1 10 3 7
28. m n + m5n5 – m7n3 29. 7a5b6 – 0,7a5b5 – 0,14a5b3
4 4
30. 2,52x7y20 – 1,32x8y13 + 2,52x7y19 31. 5,3a3bc – 5,3ab3c – 5,3abc3
9 4 2 15 9 3 2 18
32. – 2,42x7y4z2 + 2,64x8y5z3 – 6,6x7y4z4 33. – pqr + pqr
20 16
13 11 7 17 12 4 11 10 17 8 6 17 14 17 3 14 14
34. – 4m n p + m n p – m n p 35. – xy + x y + x y
4 5 9 12 2
8 4 7 8 3 8 8 2 15 6 3 8 5 3 18 48 5 4 7
36. – xy + xy – xy 37. 3a b c + 24a b c + a b c
3 3 3 5
III. 1. x3 + xy2 + x2y + y3 2. 6a2 – ab – 2b2 3. 1 – x – y + xy
4. 2x3 – 10x2y + 12xy2 5. – 3x3y2 – 5x3y3 + 12x3y4 6. – 8x2 – 20x2y – 2xy – 5xy2
7. 42a2 – 23ab – 10b2 8. a2 – b2 + a – b 9. 2ab – 5ab2 + 3ab3 + b3 – b4
10. 5x3y – 5x2y3 + 2x2y2 – 2xy4 – 3x2y + 3xy3 11. 2m3 – 5m2n + 5mn2 – 3n3
12. 7x2y3 + 15x2 y2 – 2x2y4 13. – 6p3q2 + 4p3q – 9p2q12 + 6p2q11 + 15p2q5 – 10p2q4
14. x4 – 1 15. a2 – b2 16.x2 – 2x – 24 17. a4 + 12a2 + 35 18. 4p2 + 6p – 28
19. 2x2 – xy – 3y2 – 2xz – 7yz – 4z2
20. 2x3 – 3x2y – 4x2z + 2xy2 – 3y3 – 4y2z – 2xz2 + 3yz2 + 4z3
21. an + 2an+1 + 2an+2 + an+3 22. an+2 – an–1 23. u3 – 4u2v + 4uv2 – v3
24. x3 + 3x2y + 3xy2 + y3 25 . – 3x3 + 3x2y + 3xy + x2y2 – xy3 – y3
26. 3x3 – x2y + 4xy2 + 4y3 27. – 3x2 – 5xy + 10xz – 2y2 + 7yz – 3z2 28. x3 – y3
29. x3 + y3 30. a5 + b5 31. a4 – b4 32. xn + xn–1 + xn–2 + yxn–1 + yxn-2 + yxn–3
33. pn+2 – pn+2qn – p2qn – q2pn + qn+2pn + qn+2

Álgebra en los números reales 23

22-23. 23 18/11/02, 10:49 AM


1.5 Productos notables
Dentro de la multiplicación algebraica existen algunos productos
que pueden ser desarrollados en forma directa, es decir, sin multiplicar
término a término primero, y luego reducir. Éstos son:
Cuadrado de un binomio.
El desarrollo de este producto corresponde al cuadrado del primer
término, más (o menos) el doble del producto del primer término por el
segundo y más el cuadrado del segundo, es decir:
(a ± b)2 = a2 ± 2ab + b2
Suma por diferencia.
Es igual a la diferencia de los cuadrados de los términos, es decir:
(a + b) (a – b) = a2 – b2
Producto de binomios con un término común.
Es el cuadrado del término común más el producto del término
común por la suma de los términos no comunes y más el producto de
los términos no comunes, o sea:
(x + a) (x + b) = x2 + x • (a + b) + ab
Cubo de un binomio.
Corresponde al cubo del primer término, más (o menos) el triple
del cuadrado del primer término multiplicado por el segundo, más el
triple del primer término multiplicado por el cuadrado del segundo y
más (o menos) el cubo del segundo. Así:
(a ± b)3 = a3 ± 3a2b + 3ab2 ± b3
Para obtener otras potencias de un binomio podemos determinar
los coeficientes mediante el TRIÁNGULO DE PASCAL, que se obtiene
de la siguiente manera:
• Comienza y termina con 1.
• Cada coeficiente se obtiene sumando los dos correspondientes
según el orden en la fila anterior.
• La primera fila corresponde a los coeficientes de (a + b)0
• La segunda fila corresponde a los coeficientes de (a + b)1
• La tercera fila corresponde a los coeficientes de (a + b)2
Así, la fila n-ésima nos entrega los coeficientes de (a + b)n – 1.
Los factores literales se obtienen de la siguiente manera:
En (a + b)n debe haber (n + 1) términos.
El primer factor literal es an ; el segundo es an – 1 • b1 ; el tercero
es an – 2 • b2 y así sucesivamente. El grado del término “a” decrece a
medida que el grado de “b” aumenta hasta terminar en bn.
(Cada término se forma con el coeficiente numérico obtenido
del triángulo de Pascal y el factor literal señalado más arriba).
24 Álgebra en los números reales

24-25.(2003) 24 18/11/02, 10:52 AM


CAPÍTULO 1

Representación geométrica de expresiones algebraicas.


a) La expresión a•b representa el área del c) Observemos el producto de una
rectángulo de lados a y b. suma por su diferencia:
D C
a•b a D a I b C

A B
b a–b
b) Observemos el cuadrado del binomio
(a+b)2 = a2 + 2ab + b2 a
A a–b G b
D K C H B
b b2

a a2 a•b E F J

H I A(ABCD) = (a+b) (a–b)


b a •b b2 Tenemos A(EFGA) = A(HBCI)

A a J b B \ A(ABCD) = A(EFGHIDA) que es a2 – b2

1. (2 + x)2 = 22 + 2 • 2 • x + x2 Ejercicios
= 4 + 4x + x2
resueltos
2. (3a – 5b)2 = (3a)2 – 2 • 3a • 5b + (5b)2
= 9a2 – 30ab + 25b2
3. (2x – y) (2x + y) = (2x)2 – y2
= 4x2 – y2
a a a 2 2
4. + 5y – 5y = – 5y
2 2 2
2
a 2
= – 25y
4
5. (x + 8) (x + 5) = x2 + (5 + 8)x + 5 • 8
= x2 + 13x + 40
6. (2a + 3) (2a – 7) = (2a)2 + (3 – 7) • 2a + 3 • – 7
= 4a2 – 4 • 2a – 21
= 4a2 – 8a – 21
7. (p + 2)3 = p3 + 3 • p2 • 2 + 3 • p • 22 + 23
= p3 + 6p2 + 3p • 4 + 8
= p3 + 6p2 + 12p + 8
8. (2t – r)3 = (2t)3 – 3(2t)2 • r + 3(2t) • r2 – r3
= 8t3 – 3 • 4t2 • r + 6t • r2 – r3
= 8t3 – 12t2r + 6tr2 – r3
9. (a + b)4 = 1a4 + 4a3b + 6a2b2 + 4ab3 + 1b4
a4 + 4a3b + 6a2b2 + 4ab3 + b4
10. (2a + y)5 = 1(2a)5 + 5(2a)4 • y + 10 • (2a)3 • y2 + 10(2a)2 • y3 + 5(2a)y4 + 1 • y5
= (2a)5 + 5 • 16a4y + 10 • 8a3y2 + 10 • 4a2y3 + 10ay4 + y5
= 32a5 + 80a4y + 80a3y2 + 40a2y3 + 10ay4 + y5

Álgebra en los números reales 25

24-25.(2003) 25 18/11/02, 10:52 AM


Ejercicios

I. Cuadrado de binomio.

1. (x + y)2 14. (4pq – 3q)2

2. (p – q)2 15. (9x2 – 7y2)2

3. (2p + q)2 16. (8a2b + 7ab6)2

4. (3a + b)2 17. (15x2y – 3xy2z6)2

5. (2a – 3b)2 18. (2a – 3b)2 + (3a – 5b)2

6. (x + 1)2 19. (11x – 5y)2 – (13x + 3y)2 + (x – 2y)2


2 2
7. (a – 6)2 a b
20. + 2b + 2a –
2 2
8. (x + 9)2 21. 3a – b
2

5
9. (3p – 1)2
2
22. 2 2 3
10. (x + 5)2 x – yz
3 5
11. (6x – 5y)2 23. (0,1a2 – 0,2abc)2
12. (2m – 1)2 24. (1,5xy2 + 2,5x2y)2
13. (6x2y + 2x)2 25. 3 2 3 3 2
a b – ab6
4 5

II. Suma por diferencia.

1. (u – v) (u + v)
14. (a + 5x) (a – 5x)
2. (x + 2y) (x – 2y)
15. (– 9x2 + 5xy) ( – 9x2 – 5xy)
3. (3a – b) (3a + b)
16. (–13n5p2 + 1) (13n5p2 + 1)
4. (5x2 – 3y) (5x2 + 3y)
17. (1 – a) (1 + a) – (1 – 2a) (1 + 2a)
5. (2x – 3xy) (2x + 3xy)
18. (x2 – 2xy) (x2 + 2xy) + (x2 + 2xy)2
6. (6a + 1) (6a – 1)
19. (1 – w5) (1 + w5)
7. (9m2 – 3n) (9m2 + 3n)
20. 3 7 2 4 3 7 2 4
p – q p + q
4 5 4 5
8. (– 4a2b + 5b) (4a2b + 5b)
21. abc abc
9. (– 6m2n3 – 7m) (– 6m2n3 + 7m) + 4x – 4x
2x 2x

10. (10a2 – 1) (10a2 + 1) 22. (0,05x12 – 2) (0,05x12 + 2)

11. b2 – 1 b2 +
1 23. (6x5y2z3 – 1) (6x5y2z3 + 1)
2 2
q q
12. 2a 2a 24. 2p + 2p –
– 5b + 5b 4 4
3 3
13. (2a + b) (2a – b) – (2a + b)2 25. (0,3x2y – 2z) (0,3x2y + 2z)

26 Álgebra en los números reales

26-27. 26 18/11/02, 10:54 AM


CAPÍTULO 1

III. Producto de binomios con término común.

1. (a + 2) (a + 3) 10. (x + 6) (x – 2) 19. (3a2 – 2b) (3a2 – 5b)


2. (x + 5) (x + 4) 11. (x – 3) (x – 8)
20. (9a – 4) (9a + 11)
3. (t + 2 ) (t – 3) 12. (x – 13) (x + 2)
21. (6x2 – 2y) (6x2 – 7y)
4. (a + 5 ) (a – 9) 13. (a – 7) (a + 12)
5. (x – 8) (x – 1) 22. (4a2b – 3a) (4a2b + 9a)
14. (x2 + 5) (x2 + 3)
a a
6. (a – 7) (a – 9) 15. (a2 – 3) (a2 + 4) 23. – 2b – 6b
4 4
7. (x + 2) (x – 12) 16. (2b + 5) (2b + 9) 3a 3a
24. – 5b + 8b
8. (x + 3) (x + 8) 17. (6x – 3) (6x + 5) 5 5
3p 3p
9. (x – 4) (x – 6) 18. (2a + 3b) (2a + 5b) 25. + 3q +q
4 4

IV. Cubo de un binomio.


3
1
1. (a + b)3 10. (1 – 3y)3 19. –a
2
3
2. (p – q)3 11. (2 + 3t)3 1
20. x + 2y
2
3. (x + 2)3 12. (3a – 2x)3 3
2 1
21. a– b
4. (a – 3)3 13. (5a – 1)3 3 3
3
5. (t + 4)3 14. (3a2 – 2a)3 22. 5 p + 3 q
2 2
6. (2 – a)3 15. (t2 + t3)3 1 1 3
23. m– n
10 5
7. (2a – b)3 16. (1 + x4)3
3
24. a – a
8. (3a – 5b)3 17. (2t – 3a2)3 3
3
9. (2x + 3y)3 18. (u2 + 5v)3 25. 1 t + 2t 2
2

V. Otras potencias de binomios.


5
1. (2a + b)4 5. (3a + 2)6 8. 1
+a
2
2. (x – 2y)5 x y 44 2a 4
6. + 9. – 3a
3. (a + b)6 2 2 3

4. (2a – 1)7 7. (3a + 4)4 10. (x + 1)5

VI. Representación geométrica de expresiones algebraicas.


Investigar de qué manera se pueden representar como suma o resta de áreas los
siguientes productos.
3. (x–a) (x+b) = x2 + (b–a)x – ab

1. (a–b)2 = a2 – 2ab + b2 4. (x–a) (x–b) = x2 – (a+b)x + ab


2. (x+a) (x+b) = x2 + (a+b)x + ab 5. (a+b+c)2 = a2 + b2 + c2 + 2ab + 2bc + 2ac

Álgebra en los números reales 27

26-27. 27 18/11/02, 10:55 AM


Soluciones

I. 1. x2 + 2xy + y2 2. p2 – 2pq + q2 3. 4p2 + 4pq + q2 4. 9a2 + 6ab + b2


5. 4a2 – 12ab + 9b2 6. x2 + 2x + 1 7. a2 – 12a + 36 8. x2 + 18x + 81
9. 9p2 – 6p + 1 10. x2 + 10x + 25 11. 36x2 – 60xy + 25y2
12. 4m2 – 4m + 1 13. 36x4y2 + 24x3y + 4x2 14. 16p2q2 – 24pq2 + 9q2
15. 81x4 – 126x2y2 + 49y4 16. 64a4b2 + 112a3b7+ 49a2b12
17. 225x4y2 – 90x3y3z6 + 9x2y4z12 18. 13a2 – 42ab + 34b2
2 2 2
2 6 b
19. – 47x2 – 192xy + 20y2 20. 17a + 17b 21. 9a – ab +
4 4 5 25
4 4 4 2 9 2 2
22. x – x yz + y z 23. 0,01a4 – 0,04a3bc + 0,04a2b2c2
9 5 25
9 4 6 9 3 9 9 2 12
24. 2,25x2y4 + 7,5x3y3 + 6,25x4y2 25. a b – a b + a b
16 10 25

II. 1. u2 – v2 2. x2 – 4y2 3. 9a2 – b2 4. 25x4 – 9y2 5. 4x2 – 9x2y2


6. 36a2 – 1 7. 81m4 – 9n2 8. 25b2 – 16a4b2 9. 36m4n6 – 49m2

10. 100a4 – 1 11. b4 –


1 12. 4a2 2 13. – 4ab – 2b2 14. a2 – 25x2
4 – 25b
9
15. 81x4 – 25x2y2 16. 1 – 169n10p4 17. 3a2 18. 2x4 + 4x3y 19. 1 – w10
2 2 2
a b c
20. 9 p14 – 4 q8 21. 2
– 16x2 22. 0,0025 x24 – 4 23. 36x10y4z6 – 1
16 2
25 4x
2 q
24. 4p – 25. 0,09 x4y2 – 4z2
16

III. 1. a2 + 5a + 6 2. x2 + 9x + 20 3. t2 – t – 6 4. a2 – 4a – 45
5. x2 – 9x + 8 6. a2 – 16a + 63 7. x2 – 10x – 24 8. x2 + 11x + 24
9. x2 – 10x + 24 10. x2 + 4x – 12 11. x2 – 11x + 24 12. x2 – 11x – 26
13. a2 + 5a – 84 14. x4 + 8x2 + 15 15. a4 + a2 – 12 16. 4b2 + 28b + 45
17. 36x2 + 12x – 15 18. 4a2 + 16ab + 15b2 19. 9a4 – 21a2b + 10b2
20. 81a2 + 63a – 44 21. 36x4 – 54x2y + 14y2 22. 16a4b2 + 24a3b – 27a2
2 2
2 9a 9ab 9p
23. a – 2ab + 12b2 24. + – 40b2 25. + 3pq + 3q2
16 25 5 16
IV. 1. a3 + 3a2b + 3ab2 + b3 2. p3 – 3p2q + 3pq2 – q3 3. x3 + 6x2 + 12x + 8
4. a3 – 9a2 + 27a – 27 5. t3 + 12t2 + 48t + 64 6. 8 – 12a + 6a2 – a3
7. 8a3 – 12a2b + 6ab2 – b3 8. 27a3 – 135a2b + 225ab2 – 125b3
9. 8x3 + 36x2y + 54xy2 + 27y3 10. 1 – 9y + 27y2 – 27y3
11. 8 + 36t + 54t2 + 27t3 12. 27a3 – 54a2x + 36ax2 – 8x3

28 Álgebra en los números reales

28-29. 28 18/11/02, 10:57 AM


CAPITULO 1
CAPÍTULO

13. 125a3 – 75a2 + 15a – 1 14. 27a6 – 54a5 + 36a4 – 8a3

15. t6 + 3t7 + 3t8 + t9 16. 1 + 3x4 + 3x8 + x12 17. 8t3 – 36t2a2 + 54ta4 – 27a6

18. u6 + 15u4v + 75u2v2 + 125v3 19. 1 – 3 a + 3 a2 – a3


8 4 2
1 3 3 2 8 3 4 2 2 1 3
20. x + x y + 6xy2 + 8y3 21. a – a b+ ab2 – b
8 2 27 9 9 27
125 3 225 2 135 2 27 3 1 3 3 1 3
22. p + p q+ pq + q 23. m3 – m2 n + mn2 – n
8 8 8 8 1.000 500 250 125
8 3 1 3
24.
27
a 25. t3 + t 4 + 6 t 5 + 8 t6
8 2

V. 1. 16a4 + 32a3b + 24a2b2 + 8ab3 + b4

2. x5 – 10x4y + 40x3y2 – 80x2y3 + 80xy4 – 32y5

3. a6 + 6a5b + 15a4b2 + 20a3b3 + 15a2b4 + 6ab5 + b6

4. 128a7 – 448a6 + 672a5 – 560a4 + 280a3 – 84a2 + 14a – 1

5. 729a6 + 2.916a5 + 4.860a4 + 4.320a3 + 2.160a2 + 576a + 64

x4 x3 y 3x2 y2 xy3 y4
6. + + + + 7. 81a4 + 432a3 + 864a2 + 768a + 256
16 4 8 4 16
1 5 5 2 5 3 5 4 2.401 4
8. + a+ a + a + a + a5 9. a
32 16 4 2 2 81
10. x5 + 5x4 + 10x3 + 10x2 + 5x + 1

Factorización 1.6

Definición: Factorizar una expresión algebraica (o suma


de términos algebraicos) consiste en escribirla en forma de
multiplicación. Veremos los siguientes casos:

1.6.1 Factor común


(monomio y polinomio)
Aquí, todos los términos de la expresión presentan un factor
común, que puede ser un monomio o un polinomio, por el
cual se factoriza, es decir, el término común es uno de los
factores de la multiplicación. El otro se determina aplicando la
multiplicación algebraica.

Álgebra en los números reales 29

28-29. 29 18/11/02, 10:57 AM


Ejercicios 1. Factoricemos la expresión 2a + 6a2
resueltos Vemos que el término 2a está contenido en ambos términos del
binomio que queremos factorizar; por lo tanto, 2a es el factor
común y escribimos 2a + 6a2 = 2a (1 + 3a).
El segundo factor se obtiene buscando los términos por los cuales
hay que multiplicar el factor común (2a) para obtener los términos
de la expresión original.

2. Factoricemos la expresión
6xy2 – 15x2 y + 21x2 y2
El coeficiente numérico contenido en los tres términos de la
expresión es el tres y el factor literal es xy; por lo tanto, el factor
común es 3xy. Y escribimos:
6xy2 – 15x2y + 21x2y2 = 3xy (2y – 5x + 7xy).

3. Factoricemos la expresión
6 2 3
5a 10a 20a
2
– – 4
3b 21b 9b

El término o factor común de los numeradores es 5a2 y el de


los denominadores es 3b; por lo tanto, el factor común de la
2
expresión es: 5a y escribimos:
3b
6 2 3 2
5a 10a 20a 5a a4 2 4a
– – = – –
3b
2
21b 9b
4
3b b 7 3b3

4. Factoricemos la expresión m (2a + b) – 3n (2a + b).


Aquí podemos considerar el paréntesis (2a + b) como un solo
término y podemos factorizar por él. Entonces nos queda:
m (2a + b) – 3n (2a + b) = (2a + b) (m – 3n)

5. Factoricemos la expresión a (p – q) – p + q
Aquí no encontramos un término común en forma inmediata, pero
podemos hacer una asociación adecuada y nos queda:

a (p – q) – p + q = a (p – q) – (p – q)
= (p – q) (a – 1)

Observación 1: El proceso está completo si no es posible seguir


factorizando dentro de los paréntesis (o factores) obtenidos.

Observación 2: Por la propiedad conmutativa de la multiplicación no


importa el orden en que se entregue el resultado.

30 Álgebra en los números reales

30-31. 30 18/11/02, 10:59 AM


CAPÍTULO 1

Ejercicios
Factorice las siguientes expresiones: 30. 3a + 12a – 21a
2 3
b b b
1. m2 + 3m 2 2 3 3
31. p q + pq + p q
2. a2 + ab 2ab 2ac 2abc
5 4 3
3. 3a – 12ab 32. c – c – c
5 10 15
4. a2b2 + a3b3 – ab
2 2 3 3 2 2
33. a b + a b2 – a b3
5. 2pq2 – 3p2q x x x
20 10 5
6. 6x2y5 – 12x2y6 – 18x3y4 34.
m
+
m

m
20 10 5
7. 2ab + 2ac + 2ad 2
35. – p q + 2pq 2

8. 26x2y6 – 13x6y2 36. 3 (a – 2) – a (a – 2)


9. x2y2 – xy 37. a (x + 4) + b (x + 4) + c (x + 4)
10. 21a6 – 14a5 + 56a7
38. x (z2 + a2) + 2 (z2 + a2)
11. a + a2 + a3 + a4
39. m (a – c) + a – c
12. 3a2b – 6a 3 b – 12ab 3
40. m (a – c) – a + c
13. 15mn – 10m
41. a (x2 + y2 + z2) – x2 – y2 – z2
14. 2q + 2q2 + 2q6
42. 2a – b + 3a (2a – b)
15. 10q5 – 30pq5 – 15pq6
43. a + ax + ax2
16. 18gh5 – 4g2h2 – 8g3h3
44. c (3 – 5c) – 2d (3 – 5c)
17. 7y6x2 – 35yx4 – 28y4 2 2 2 2
a +c a +c 2 2
45. – –a –c
18. 2 – 2x 2b 2q
46. 3x (2x – y) – 2x + y
19. a + a2
47. (a + b) (a + c) – (a + b ) (a + d)
20. a6 – 7a5 – 5a4
48. (1 + a) (x – y) – (x – y)2
21. 4m5r6 – 6m4r5 – 16m5r3
49. (a2 + 6) (a2 + b) + a (a2 + b)
22. a2b2c6 – a3b5c2 + a7b3c2
50. (2 + a + c) (a – c) + (2 + a + c) (b – d)
23. x2 – x2y2 – x2y3 + x2y4
51. x2 + y2 + z2 + 2a (x2 + y2 + z2)
24. 2xyz – 2xy
52. a (b + x) + b (b + x) + c (b + x)
25. 6a + 36a6
2 4 16
53. a – ab – abc
26. t9 + t8 + t5 15 5 25
54. m (x + y – z) – n (x + y – z) – p (x + y –z)
27. 12ab6 – 12ab5

28. x6y9z12 + x6y8z6 + x5y8z10 55. 3 a 2 b – 3 a 2 b2 – 3 a 2 b3


4 2 8
2 3 4 2 2
a a a 56. x + y – x 2 – y 2
29. – –
2 2 2 9a

Álgebra en los números reales 31

30-31. 31 18/11/02, 10:59 AM


Soluciones

1. m (m + 3) 22. a2b2c2 (c4 – ab3 + a5b) 37. (x + 4) (a + b + c)


2. a (a + b) 23. x2 (1 – y2 – y3 + y4) 38. (x + 2) (z2 + a2)
3. 3a (1 – 4b) 24. 2xy (z – 1) 39. (a – c) (m + 1)
4. ab (ab + a2b2 – 1) 25. 6a (1 + 6a5) 40. (a – c) (m – 1)
5. pq (2q – 3p) 26. t5 (t4 + t3 + 1) 41. (x2 + y2 + z2) (a – 1)
6. 6x2y4 (y – 2y2 – 3x) 27. 12ab5 (b – 1) 42. (1 + 3a) (2a – b)
7. 2a (b + c + d) 28. x5y8z6 (xyz6 + x + z4) 43. a (1 + x + x2)
8. 13x2y2 (2y4 – x4) 44. (3 – 5c) (c – 2d)
2 1 1
9. xy (xy – 1) 29. a 1 – a – a2 45. a2 + c2 – –1
2 2b 2q
10. 7a5 (3a – 2 + 8a2)
46. (2x – y) (3x – 1)
11. a (1 + a + a2 + a3) 30. 3a 1 + 4 – 7 47. (a + b) (c – d)
12. 3ab (a – 2a2 – 4b2) b b b2
48. (x – y) (1 + a – x + y)
2 2
13. 5m (3n – 2) 31. pq pq + 1 + p q 49. (a2 + b) (a2 + 6 + a)
2a b c bc
14. 2q (1 + q + q5)
3
50. (2 + a + c) (a – c + b – d)
15. 5q5 (2 – 6p – 3pq) 32. c c2 – c – 1
5 2 3 51. (x2 + y2 + z2) (1 + 2a)
16. 2gh2 (9h3 – 2g – 4g2h) 52. (b + x) (a + b + c)
a2 b2 ab 1
17. 7y (y5x2 – 5x4 – 4y3) 33.
x 1 + – 2 2 1 8bc
x x 53. a – 2b –
5 3 5
18. 2 (1 – x)
m5 m15 m5 54. (x + y – z) (m – n – p)
19. a (1 + a) 34. + –1
5 4 2 3 2 1 1 2
55. a b – b– b
20. a4 (a2 – 7a – 5) 35. pq (–p + 2q) 2 2 4
21. 2m4r3 (2mr3 – 3r2 – 8m) 36. (a – 2) (3 – a) 1
56. x2 + y2 – 1
9a

1.6.2 Factor común compuesto


Muchas veces, no todos los términos de una expresión algebraica
contienen un factor común, pero haciendo una adecuada agrupación
de ellos podemos encontrar factores comunes de cada grupo. Veremos,
con ejemplos, cómo procederemos en estos casos.

Ejercicios 1. Factoricemos: ac + ad + bc + bd
resueltos Si observamos, vemos que el primer y el segundo término tienen el
factor común “a” y el tercer y el cuarto término tienen “b” como factor
común. Asociamos y factorizamos por parte:
ac + ad + bc + bd = (ac + ad) + (bc + bd)
= a(c + d) + b(c + d)

32 Álgebra en los números reales

32-33. 32 18/11/02, 11:01 AM


CAPÍTULO 1

Ahora nos queda (c + d) como factor común, por lo tanto, la


expresión original queda factorizada como sigue:
ac + ad + bc + bd = (c + d) (a + b)
2. Factoricemos: ax + bx + cx – ay – by – cy
Aquí podemos asociar el primer y el cuarto término, el segundo y
el quinto, el tercero y el sexto y nos queda:
ax + bx + cx – ay – by – cy = (ax – ay) + (bx – by) + (cx – cy)
= a(x – y) + b(x – y) + c(x – y)
= (a + b + c) (x – y)
3. Factoricemos: ax + bx + cx + ay + by + cy – az – bz – cz
Asociemos en el orden natural los tres primeros, los tres siguientes
y los tres últimos:
ax + bx + cx + ay + by + cy – az – bz – cz
= (ax + bx + cx) + (ay + by + cy) – (az + bz + cz)
= x(a + b + c) + y (a + b + c) – z(a + b + c)
= (a + b + c) (x + y – z)
• Observación: La forma de asociar no es única, pero la factorización
sí lo es.
En el primer ejemplo podríamos haber asociado el primer y
el tercer término y el segundo con el cuarto y el resultado habría
sido el mismo.

Ejercicios
Factorice las siguientes expresiones:

1. ac + ad + bc + bd 12. 3 + 15z + 4y + 20yz


2. ax – ay + bx – by + cx – cy 13. a2c2 + a2d2 + b2c2 + b2d2
3. pc + qc + pd + qd 14. 3ax3 – 2bx3 – 3ay3 + 2by3
4. rt + rv – st – sv 15. 1 + b + a + ab
5. 2ac – ad + 2bc – bd 16. a2x2y2 + b2x2y2 – 2a2 – 2b2
6. xu – xv – yu + yv 17. abc – 2abcz – xy + 2xyz
7. 2au + 2av – 3bu – 3bv 18. bd – 3bf + 2cd – 6cf
8. 3a2x + 3a2y + b2x + b2y 19. xp + 2xq – 2yp – 4yq + 4zp + 8zq
9. 2ac – 2ad + 3bc – 3bd 20. 4 + 2c + 2d + 2a + ac + ad + 2b + bc + bd
10. x + y + ax + ay 21. a2x2 + x2y2 – x2b + a2y2 + y4 – y2b – a2 – y2 + b
11. 2a – 2b + ax – bx

Álgebra en los números reales 33

32-33. 33 18/11/02, 11:01 AM


Ejercicios
22. a2x2 + b2x2 + c2x2 + a2y2 + b2y2 + c2y2 29. p4 + p2q2 + p2r2 + 2p2q + 2q3 + 2qr2 + p2r + q2r + r3
23. 12ac – 6ad – 2bc + bd 30. ax – bx – cx + 2ay – 2by – 2cy – az + bz + cz
24. aq – ar + bq – br 31. a2u – a2v + b2u – b2v + u – v
25. u + au – v – av – w – aw 32. 4 – 2a – 2b + 2x – ax – bx + 2y – ay – by
26. 2ax – 2ay – bx + by 33. x2y2w2 – x2y2z2 – xyw2 + xyz2
27. 3am2 – 3at2 – 5b2m2 + 5b2t2 34. ax + 2bx + 3cx – ay – 2by – 3cy
28. x – y + 2ax – 2ay + 3bx – 3by 35. 2ax + 2bx – ay – by – az – bz

Soluciones

1. (a + b) (c + d) 18. (b + 2c) (d – 3f)


2. (a + b + c) (x – y) 19. (x – 2y + 4z) (p + 2q)
3. (p + q) (c + d) 20. (2 + a + b) (2 + c + d)
4. (r – s) (t + v) 21. (x2 + y2 – 1) (a2 + y2 – b)
5. (a + b) (2c – d) 22. (x2 + y2) (a2 + b2 + c2)
6. (x – y) (u – v) 23. (6a – b) (2c – d)
7. (2a – 3b) (u + v) 24. (a + b) (q – r)
8. (3a2 + b2) (x + y) 25. (u – v – w) (1 + a)
9. (2a + 3b) (c – d) 26. (2a – b) (x – y)
10. (1 + a) (x + y) 2 2 2
* 27. (3a – 5b ) (m – t )
11. (2 + x) (a – b) 28. (1 + 2a + 3b) (x – y)
12. (3 + 4y) (1 + 5z) 29. (p2 + 2q + r) (p2 + q2 + r2)
13. (a2 + b2) (c2 + d2) 30. (x + 2y – z) (a – b – c)
3 3 31. (a2 + b2 + 1) (u – v)
* 14. (x – y ) (3a – 2b)
15. (1 + a) (1 + b) 32. (2 + x + y) (2 – a – b)
16. (x2y2 – 2) (a2 + b2) 2 2
* 33. xy (xy – 1) (w – z )
17. (abc – xy) (1 – 2z) 34. (a + 2b + 3c) (x – y)
35. (2x – y – z) (a + b)
NOTA:
Los ejercicios señalados con * son posibles de factorizar aún más con los métodos
que veremos a continuación.

1.6.3 Diferencia de cuadrados


Recordemos que el producto de una suma de dos términos por
su diferencia es igual a la diferencia de los cuadrados de ambos
términos.
Aplicamos este resultado en las factorizaciones siguientes:

34 Álgebra en los números reales

34-35. 34 18/11/02, 11:02 AM


CAPITULO 1
CAPÍTULO

1. Factoricemos a2 – b2 Ejercicios
Observamos que a2 y b2 son los cuadrados de a y b, resueltos
respectivamente.
Así: a2 – b2 = (a + b) (a – b)
2. Factoricemos 9m2 – 16p2
9m2 es el cuadrado de 3m y 16p2 es el cuadrado de 4p.
Entonces : 9m2 – 16p2 = (3m + 4p) (3m – 4p)

1 25
3. Factoricemos 2
– 2
a 4b
Usando el mismo razonamiento anterior vemos que la expresión se
1 25 1 5 1 5
factoriza: – = + –
a 2 4b2 a 2b a 2b
4. Factoricemos 6a2 – 24m4
En este ejemplo podemos factorizar primero por 6 (factor común
monomio).
6a2 – 24m4 = 6 (a2 – 4m4)
y ahora, el término (a2 – 4m4) es exactamente una diferencia de
cuadrados y por lo tanto la factorización correspondiente es:
6a2 – 24m4 = 6 (a2 – 4m4)
= 6 (a – 2m2) (a + 2m2)
• Observación: No es importante el orden en que uno presente los
factores, puesto que la multiplicación es conmutativa, es decir:
(a + b) (a – b) = (a – b) (a + b)

Ejercicios
9. a2b2 – c2d2 20. x2a – y2b
Factorice las siguientes
10. 1 – x10 21. m2an2b – 1
expresiones:
11. – b6 + a4 22. 25n16 – 16m4
1. x2 – y2 12. – 1 + a2 23. 40 – 90a4
2. a2 – 4b2 13. a5 – a3 24. – 24m2 + 54n12
3. 9m2 – 16n2 14. 8a4 – 2b2 25. m6n4p12 – a2b2c2
4. 9a2 – 25p2 15. p2q3 – q 26. 2x2 – 8y2z6
5. x2 – 0,01y2 16. 49a2b4c6 – 121m6n10 27. a10 – 100b10
6. 100a2 – 64b6 17. 12a6 – 75b8 28. 144b10 – 121c6
7. m2n2 – p2 18. 45m6 – 80p8 29. 81c4 – 9d4
8. m4n6 – z2 19. 27x4 – 48y2 30. 225 – a2

Álgebra en los números reales 35

34-35. 35 18/11/02, 11:02 AM


Ejercicios 1
37. 12 – 12 44. a12 –
a b 9b6
1 8a
4
2 4 25
31. – 121 + 2 38. – 45. –
2 6 6
y 45b 5 x y
32. – 64a2b4c6 + x8y2 39. 32m10 – 18p4q6 46. a2 – b2 – 2a – 2b
33. 16x4 – 4y16 40.
1
– a b
2 2
47. p2 – q2 – rp + rq
2 2
a b
34. 1 25
2 2
– 2 2 41. x2 – y2 – ax + ay 48. a 2 + ac – b2 – bc
4a b 9x y
4 1
35. 24x8 – 6 42. 25x – 49. m2 – n2 – pm – pn
25
6 2
36. 75m 27n 12 10
– m n 50. qr2 – q3s2
4 25 43.
2
– 4
c d

Soluciones

1.(x + y) (x – y) 2. (a + 2b) (a – 2b) 3. (3m + 4n) (3m – 4n)


4.(3a – 5p) (3a + 5p) 5. (x – 0,1y) (x + 0,1y) 6. (10a – 8b3) (10a + 8b3)
7.(mn + p) (mn – p) 8. (m2n3 – z) (m2n3 + z) 9. (ab – cd) (ab + cd)
10. (1 – x5) (1 + x5) 11. (a2 – b3) (a2 + b3) 12. (a – 1) (a + 1)
13. a3(a – 1) (a + 1) 14. 2 (2a2 – b) (2a2 + b) 15. q (pq – 1) (pq + 1)
16. (7ab2c3 – 11m3n5) (7ab2c3 + 11m3n5) 17. 3 (2a3 – 5b4) (2a3 + 5b4)
18. 5 (3m3 – 4p4) (3m3 + 4p4) 19. 3 (3x2 – 4y) (3x2 + 4y)
20. (xa – yb) (xa + yb) 21. (manb – 1) (manb + 1)
22. (5n8 – 4m2) (5n8 + 4m2) 23. 10 (2 – 3a2) (2 + 3a2)
24. 6 (3n6 – 2m) (3n6 + 2m) 25. (m3n2p6 – abc) (m3n2p6 + abc)
26. 2 (x – 2yz3) (x + 2yz3) 27. (a5 – 10b5) (a5 + 10b5)
28. (12b5 – 11c3) (12b5 + 11c3) 29. 9 (3c2 – d2) (3c2 + d2)
1 1
30. (15 – a) (15 + a) 31. y + 11 y – 11
32. (x4y – 8ab2c3) (x4y + 8ab2c3) 33. 4 (2x2 – y8) (2x2 + y8).

1 5 1 5 5m3 3n 5m3 3n
34. – + 35. 6 (2x4 – 1) (2x4 + 1) 36. 3 – +
2ab 3xy 2ab 3xy 2 5 2 5
2 2a2
37. 1

1 1
+
1 38. 2 2a – 1 +1 39. 2 (4m5 – 3p2q3) (4m5 + 3p2q3)
a b a b 5 3b 3b
1 1
40. – ab + ab 41. (x – y) (x + y –a) 42. 5x2 – 1 5x2 +
1
ab ab 5 5
m6 n5 m6 n5 2 5 2 5
43. – + 44. a6 + 1 a6 –
1 45. – 3 + 3
2
d2 3 x3 y x3 y
c d c 3b 3b3
46. (a + b) (a – b – 2) 47. (p – q) (p + q – r) 48. (a – b) (a+ b + c)
49. (m + n) (m – n – p) 50. q (r – qs) (r + qs)

36 Álgebra en los números reales

36-37.(2003) 36 18/11/02, 11:05 AM


CAPÍTULO 1

1.6.4 Trinomios ordenados

Definición: Llamamos trinomio ordenado (según el grado) a


una expresión de la forma ax2 + bx + c, donde a, b, c, y x
representan números reales.

En general, los trinomios pueden proceder:


• de la multiplicación de un binomio por sí mismo (o un cuadrado
de binomio); por ejemplo:
(a + 7)2 = a2 + 14a + 49
• de la multiplicación de dos binomios con un término común;
por ejemplo:
(a + 2) (a + 6) = a2 + 8a + 12
• o de la multiplicación de dos binomios de términos semejantes:
(2x + 1) (x + 2) = 2x2 + 5x + 2
Con estas consideraciones, resolvamos los ejercicios presentados
a continuación:

1. Factoricemos x2 + 10x + 25 Ejercicios


Observamos que el primer término (x2) y el último (25) son los resueltos
cuadrados de x y 5, respectivamente, y además el término central
(10x) corresponde al doble del producto de x y 5; entonces la
expresión es un cuadrado de binomio y así:
x2 + 10x + 25 = (x + 5)2
2. Factoricemos a2 – 8a + 16
Usando el mismo razonamiento anterior, observamos que el trinomio
corresponde al cuadrado del binomio (a – 4) y escribimos:
a2 – 8a + 16 = (a – 4)2
El signo del término central del trinomio indica el signo que corresponde
al segundo término del binomio.
3. Factoricemos y2 + 13y + 36
Aquí vemos que tanto el primer término como el tercero corresponden
a cuadrados exactos (de “y” y de 6, respectivamente), pero el término
central (13y) no corresponde al doble del producto entre “y” y 6 (es
decir, a 12y); en este caso, el trinomio puede corresponder al producto
de dos binomios con un término común, que sería “y”.
Buscamos entonces dos números cuyo producto sea igual a 36 (el

Álgebra en los números reales 37

36-37.(2003) 37 18/11/02, 11:05 AM


Ejercicios último término del binomio) y el producto del término común
(y) por la suma de estos números sea igual al término central
resueltos (13y). Los números son + 9 y + 4.
En efecto: + 9 • + 4 = 36 y 9 + 4 = 13
Entonces: y2 + 13y + 36 = (y + 9) (y + 4).
4. Factoricemos a2 – 2a – 48
Descartamos la posibilidad de cuadrado de binomio pues el último
término (– 48) no es cuadrado de ningún número.
Buscamos dos números cuyo producto sea – 48, y cuya “suma”
sea – 2, la que al multiplicarla por el término común “a” nos
da el término central – 2a.
Los números son – 8 y + 6 y la factorización correspondiente es:
a2 – 2a – 48 = (a – 8) (a + 6).
5. Factoricemos x2 – 5x + 6
No es cuadrado de binomio por la misma razón anterior (el + 6
no es cuadrado de un número entero). Corresponde entonces
al producto de dos binomios con un término común, que en
este caso es x. Buscamos dos números cuyo producto sea +
6 y cuya suma sea – 5.
Los números son – 2 y – 3. Por lo tanto, la factorización
correspondiente es:
x2 – 5x + 6 = (x – 2) (x – 3).
6. Factoricemos la expresión 2x2 – 3x – 2
En este ejemplo, ni siquiera el primer término es cuadrado exacto
de un término entero.
Amplifiquemos por el coeficiente de x2 (en este caso, por 2)
para obtener un primer término como en los ejemplos anteriores,
es decir, un cuadrado exacto.
2
2x – 3x – 2 2
/•
2 2
4x – 6x – 4
2

Podemos aplicar al numerador el razonamiento de los ejemplos


anteriores (porque el primer término ya es un cuadrado exacto)
y entonces trataremos de factorizar como producto de dos
binomios con un término común que en este caso es 2x.
Buscamos dos números que multiplicados sean igual a – 4 y
cuya suma sea igual a – 3 (pues al multiplicar la suma por el
término común 2x se debe obtener – 6x).
Los números son – 4 y 1 y así, la factorización de la expre-
sión amplificada es:
4x2 – 6x – 4 2x – 4 2x + 1
=
2 2

38 Álgebra en los números reales

38-39. 38 18/11/02, 11:06 AM


CAPÍTULO 1

Podemos factorizar el primer término por dos y luego simplificarlo


por el denominador, obteniendo:
(2x – 4) (2x + 1)
2 x2 – 3x – 2 =
2

2 (x – 2) (2x + 1)
2 x2 – 3x – 2 =
2

2 x2 – 3x – 2 = (x – 2) (2 x + 1)

7. Factoricemos 3x2 – 5x + 2
Siguiendo los pasos anteriores, obtenemos:

3 x2 – 5x + 2 3

3
9x2 – 15x + 6
3
(3x – 3) (3x – 2)
3
3 (x – 1) (3x – 2)
3
(x – 1) (3x – 2)

Ejercicios
Factorice las siguientes expresiones:

1. x2 + 14x + 49 12. 4x2 + 20x + 25 23. x2 – x – 6

2. x2 + 8x + 16 13. 9x2 – 6x + 1 24. x2 – 5x + 6

3. a2 + 18a + 81 14. a2 – 4ab + 4b2 25. a2 – 5a – 36

4. a2 – 6a + 9 15. y2 + 6xy + 9x2 26. a2 + a – 30

5. y2 – 24y + 144 16. 4t2 + 12t + 9 27. a2 + 8a + 7

6. x2 + 10x + 25 17. 4x2 + 12xy + 9y2 28. y2 + y – 56

7. t2 – 2t + 1 18. 9x2 – 30xy + 25y2 29. x4 – 6x2 + 9

8. z2 + 16z + 64 19. x2 + 14xy + 49y2 30. 4 + 20y2 + 25y4

9. x2 – 22x + 121 20. x4 + 2x2 + 1 31. x4 + 2x2y2 + y4

10. a2 – 12a + 36 21. x2 + 5x + 6 32. x6 + 2x3 + 1

11. 1 + 6a + 9a2 22. x2 + x – 6 33. a4 – 4a2b2 + 4b4

Álgebra en los números reales 39

38-39. 39 18/11/02, 11:07 AM


Ejercicios
45. 2x2 + 5x + 2 58. 6a2 + 13a + 6

34. 9m4 – 30m2p2 + 25p4 46. 2x2 + 5x – 3 59. 12a2 – 23a + 5

35. 9m2 – 30mp2 + 25p4 47. 3x2 + 14x + 8 60. 8a2 – 2a – 15


2 48. 3x2 + 11x – 4 61. 5x2 – 26x + 5
36. x – x + 1
4
2 1 49. 6x2 – 13x + 5 62. 18a2 – 18a + 4
37. a + a +
4
2 50. 2x2 +15x + 28 63. a4 + 5a3 + 6a2
38. a + ab + b2
4 51. 7x2 – 8x + 1 64. x3 – 3x2 – 40x
39. a2 – 23a + 132
52. 6x2 + 5x – 4 65. x4 – 3x2 + 2
40. a2 – 3a – 40
53. 8x2 – 2x –1 66. 2a3 + 6a2 + 4a
41. a4 + 5a2 + 6 54. 5x2 – 18x + 9 67. m3 – m2 – 30m
42. 4x2 – 22x + 30 55. 2x2 + 3x – 14 68. n4 + n2 – 2
43. 9x2 – 9x – 28 56. 3a2 – 7a + 2 69. p4 + 2p2 + 1

44. 25x2 – 15x + 2 57. 5a2 + 3a – 2 70. p3 – p2 – p + 1

Soluciones

1. (x + 7)2 2. (x + 4)2 3. (a + 9)2 4. (a – 3)2 5. (y – 12)2 6. (x + 5)2


7. (t – 1)2 8. (z + 8)2 9. (x – 11)2 10. (a – 6)2 11. (1 + 3a)2 12. (2x + 5)2
13. (3x – 1)2 14. (a – 2b)2 15. (y + 3x)2 16. (2t + 3)2 17. (2x + 3y)2
18. (3x – 5y)2 19. (x + 7y)2 20. (x2 + 1)2 21. (x + 3) (x + 2) 22. (x + 3) (x – 2)
23. (x – 3) (x + 2) 24. (x – 3) (x – 2) 25. (a – 9) (a + 4) 26. (a + 6) (a – 5)
27. (a + 7) (a + 1) 28. (y – 7) (y + 8) 29. (x2 – 3)2 30. (2 + 5y2)2 31. (x2 + y2)2
x 2
32. (x3 + 1)2 33. (a2 – 2b2)2 34. (3m2 – 5p2)2 35. (3m – 5p2)2 36. –1
2
1 2 a 2
37. a + 2 38. +b 39. (a – 12) (a – 11) 40. (a + 5) (a – 8)
2
41. (a2 + 2) (a2 + 3) 42. 2(2x – 5) (x – 3) 43. (3x + 4) (3x – 7) 44. (5x – 1) (5x – 2)
45. (2x + 1) (x + 2) 46. (2x – 1) (x + 3) 47. (3x + 2) (x + 4) 48. (3x – 1) (x + 4)
49. (3x – 5) (2x – 1) 50. (2x + 7) (x + 4) 51. (7x – 1) (x – 1) 52. (3x + 4) (2x – 1)
53. (4x + 1) (2x – 1) 54. (5x – 3) (x – 3) 55. (2x + 7) (x – 2) 56. (3a – 1) (a – 2)
57. (5a – 2) (a + 1) 58. (2a + 3) (3a + 2) 59. (3a – 5) (4a – 1) 60. (2a – 3) (4a + 5)
61. (x – 5) ( 5x – 1) 62. 2(3a – 2) (3a – 1) 63. a2(a + 2) (a + 3) 64. x(x + 5) (x – 8)
65. (x – 1) (x + 1) (x2 – 2) 66. 2a(a + 1) (a + 2) 67. m(m – 6) (m + 5)
68. (n – 1) (n + 1) (n2 + 2) 69. (p2 + 1)2 70. (p – 1)2 (p + 1)

40 Álgebra en los números reales

40-41.(2003) 40 18/11/02, 11:08 AM


CAPÍTULO 1

1.6.5 Sumas o diferencias de cubos


Los factores de una diferencia de cubos son:
x3 – y3 = (x – y) (x2 + xy + y2)
Los factores de una suma de cubos son:
x3 + y3 = (x + y) (x2 – xy + y2)

1. Factoricemos a3 – 8 Ejercicios
Observamos que a3 es el cubo de a y que 8 es el cubo de 2. Se resueltos
trata de una diferencia de cubos, por lo tanto:
a3 – 8 = (a – 2) (a2+ 2a + 4)
2. Factoricemos x3 + 27
El término x3 es el cubo de x y 27 es el cubo de 3. Aquí
tenemos una suma de cubos y por lo tanto:
x3 + 27 = (x + 3) (x2 – 3x + 9)
3. Factoricemos 27a3 – 125b3
El primer término es el cubo de 3a y el segundo término es
el cubo de 5b, entonces escribimos:
27a3 – 125b3 = (3a – 5b) (9a2 + 15ab + 25b2)
4. Factoricemos a6 – b6
Aquí tenemos primero una diferencia de cuadrados, la cual
factorizamos como una suma por su diferencia. Luego, cada
uno de los factores corresponde a una suma o diferencia de
cubos. Procedamos por pasos:
a6 – b6 = (a3 + b3) (a3 – b3)
= (a + b) (a2 – ab + b2) (a – b) (a2 + ab + b2)
y ésa es la factorización requerida.

Ejercicios
Factoricemos las siguientes expresiones:
1. m6 – n3 4. t3 – 64 v3 7. 1 – 125 a3 10. 216 a3 – 27 b3
1 1 8 27
2. x3 + p3 5. 27 x3 + y3 8. 3
+ 3 11. 3
– 3
x y z y
6
n3
3. a3 – 8 b3 6. 8m – 9. 16 x3 – 54 y3 12. 125 – 1
8 8a3

Álgebra en los números reales 41

40-41.(2003) 41 18/11/02, 11:09 AM


Ejercicios 20. 3 t3 – 3 27. a6 – 1 34.
x3
– 1
y3
13. 3 a3 – 81 b3 a6
21. 216 a3 + 8 b3 28. –1 – b3 35. 0,001 – 3
14. a2 b3 c6 + a2 d3 b
22. 8 t3 + 64 29. 8 – 27 36. 216 –
a3
15. m3 x3 + 1 t6 t3 b3
1 1 1
23. 125 t 3 – 3 30. p + q9
3
37. + 3
16. a3 b6 c9 + 8 z 125 z
2 16
24. – 3 31. m12 + 1 3
38. 64a –
1
17. x12 – y12 t3 y 216
1
18. m9 – 1 25. 3
8a + 3 32. a27 + b27 39. m3 n3 p6 – 8a3
b
26. –1 + a3 33. 1 – a9 1 1
19. a3 b12 – 27 40. +
8z3 27 y3

Soluciones

1. (m2 – n) (m4 + m2 n + n2) 2. (x + p) (x2 – px + p2)


4. (t – 4v) (t2 + 4tv + 16v2)
3. (a – 2b) (a2 + 2ab + 4b2)
n2 n4
6. 2m – 4m2 + mn2 +
5. (3x + y) (9x2 – 3xy + y2) 2 4
7. (1 – 5a) (1 + 5a + 25a2) 8. 1 1 1 1 1
x + y 2
– + 2
x xy y
9. 2(2x – 3y) (4x2 + 6xy + 9y2)
10. 27(2a – b) (4a2 + 2 ab + b2)
2 3 4 6 9
11.
z – y + + 2 12. 5 – 1 25 + 5 + 1
z2 yz y 2a 2a 4a2
13. 3(a – 3b) (a2 + 3ab + 9b2) 14. a2(bc2 + d) (b2c4 – bc2d + d2)
15. (mx + 1) (m2x2 – mx + 1) 16. (ab2c3 + 2) (a2b4c6 – 2ab2c3 + 4)
2 6 3
17. (x – y) (x + y) (x2 + y2) (x4+ x2 y2+ y4) (x4– x2 y2+ y4) 18. (m –1) (m + m + 1) (m + m + 1)
2
20. 3(t – 1) (t + t + 1)
19. (ab4 – 3) (a2 b8 + 3ab4 + 9)
22. 8(t + 2) (t2 – 2t + 4)
21. 8(3a + b) (9a2 – 3 ab + b2)
1 2 1 2 4
1 5t 1 24. 2 – + + 2
23. 5t – z 25t 2 + + 2 t y t 2
ty y
z z
26. (a – 1) (a2 + a + 1)
1 2a 1
25. 2a + 4a2 – + 2 28. – (1 + b) (1 – b + b2)
b b b
27. (a – 1) (a + 1) (a4 + a2 + 1) 30. (p + q3) (p2 – pq3 + q6)
32. (a + b) (a2 – ab + b2) (a6 – a3b3 + b6) (a 18 – a9b9 + b18)
1 2 4 6
29.
3 t
–3 2 + +9 x x2 x
t t t 34. y – 1 y2 + y + 1
31. (m4 + 1) (m8 – m4 + 1)
a 6a a2
33. (1 – a) (1 + a + a2) (1 + a3 + a6) 36. 6 + 36 – + 2
b b b
a2 0,1a2 a4
35. 0,1 – 0,01 + +
2a 1
b b b2 38. 4 a – 1 16a2 + +
6 3 36
1 1 1 1 1
37. + – + 2
5 z 25 5z z 40. 1 1 1 1 1
+ – +
39. (mnp2 – 2a) (m2n2p4 + 2a mnp2 + 4a2) 2z 3y 4z2 6yz 9y2

42 Álgebra en los números reales

42-43. 42 18/11/02, 11:11 AM


CAPÍTULO 1

Fracciones algebraicas 1.7

1.7.1 Simplificación
Para simplificar una fracción es necesario y suficiente que el
numerador y el denominador tengan un factor común.
En el caso de monomios, la simplificación se hace en forma
directa; en cambio, si el numerador o el denominador de la
fracción tienen dos o más términos, es necesario factorizar primero
y luego simplificar.

2a2
1. Simplifiquemos
3ab
Ejercicios
Aquí tanto el numerador (2a2) como el denominador (3ab) contienen el resueltos
término “a” como factor. Simplificamos, pues, por él y obtenemos:
2
2a 2a
=
3ab 3b 2 2
6m p q
2. Simplifiquemos 3 2
27 mp q
En este ejemplo, el término 3mp2q está contenido en el numerador y
en el denominador. Simplificando, nos queda:
2 2
6m p q 2m
=
3 2
27mp q 9pq
2
3. Simplifiquemos 2a + 2
4a
En este caso no es posible hacer una simplificación directa, pues en el
numerador hay un binomio (recordemos que no podemos simplificar
términos que se suman o restan).
Debemos entonces factorizar primero y después simplificar:
2 2 2
2a + 2 2(a + 1) a +1
= =
4a 4a 2a

Y ya no es posible seguir reduciendo porque el numerador no se


puede factorizar más.
2
4. Simplifiquemos a + ab
a+b
Usando el mismo razonamiento anterior, factorizamos primero y
luego simplificamos:
2
a + ab a(a + b)
= = a
a+b a+b
x2 + 5x + 6
5. Simplifiquemos
x2 + 3x + 2
Factorizando y luego simplificando obtenemos:
x2 + 5x + 6 (x + 2) (x + 3) x+3
= =
2
x + 3x + 2 (x + 2) (x + 1) x+1

Álgebra en los números reales 43

42-43. 43 18/11/02, 11:12 AM


x2 – 9
6. Simplifiquemos
x2 + 6x + 9
Procediendo como antes:
x2 – 9 (x + 3) (x – 3) x – 3
= =
x2 + 6x + 9 (x + 3) (x + 3) x + 3
3x3 – 3x y2
7. Simplifiquemos
x2 y – xy2
3x3 – 3xy2 3x (x2 – y2) 3x (x – y) (x + y) 3 (x + y)
= = =
x2 y – xy2 xy (x – y) xy (x – y) y

Ejercicios
Simplifique las siguientes expresiones:

3 p q3 2
1. 2a 13. 2 p q2 25. a + ab 37. ab + ac + xb + xc
5ab 2a b2 – c2
3a – 15 c7 d8 a2 b2 – a b x2 + 7x + 12
2. 14. 26. 38.
6b 35 ab c7 ab – 1 x2 + 5x + 6

a2 b – 17 m6 n11 2 x4 – y2
3. 15. 27. x – 25 39.
ab2 51 m4 n9 x+5 x2 + y

6m (a + b)2 1 – a2 x2 – 8x + 12
4. 16. 28. 40.
16pm (a + b) 1+a x2 – 4x – 12

5ad (p2 + 1)3 ab2 – ac2 2 2


5. 17. 29. 41. 2 a b – 8 a b
10d (p2 + 1)4 b+c 4 a2 b – 16 a b2
25 p2q a2 + b2 x+4 ax + bx – ay – by
6. 18. 30. 42.
15 pq3 2
(a + b ) 2 4 x2 + 8x + 16 x2 – y2

2 32 z4 y3 xy 10x2 – 15xy
7. 2 m np
2
19. 31. 43.
18mn p 96 z3 y4 x2 y2 – xy3 4x2 – 9y2
6 a 3a3 – 3a2 – 6a
8. 30 a b 20. 32. 3 abc
44.
21 a6 b 2 144a 2
6 a bc – 9 ab c 2
2a3 + 6a2 + 4a
– 125 x6 y5 z4 121 a 6x2 – 3xy x4 – x2
9. 5xyz
21. 33. 45.
11 ac 4x2 – y2 x3 + 2 x2 + x

2ab m4 n4 2 pq x2 – 11x + 30
10. 22. 34. 46.
8 a6 b6 4mn p2 q– pq2 x2 – 25
4 4
12 a12 b12 5xy + 10x x3 – 3xy2
11. a b 23. 35. 47.
4ab 6 a6 b6 y2 – 4 x 4 – 9y4
12 (a3 b2 )2 2 2 x8 – 9y4
12. 6 a 24. 36. m – 2mn + n 48.
12 a6 5a6 b8 m2 – n2 x4 – 3y2

44 Álgebra en los números reales

44-45. 44 18/11/02, 11:17 AM


CAPÍTULO 1

x2 – 7x + 6 2
58. x – 8x + 15
49.
5ab
52. 55. ac – ad – bc + bd
25a2 – 5ab x2 – 1 c2 – d2 x2 – 9

x2 + x – 2 2a2 b – 2ab2 2p2 x + 2px2 4p2 – 4p + 1


50. 53. 56. 59.
ax + 2a – x – 2 a2 – 2ab + b2 p2 – pq + xp – xq 4p2 – 1

x3 – x2 + x – 1 6 p2 q – 2 pq2 2 2m3 – 18m


51. 54. 57. x + 10x – 11 60.
x2 – 1 3 p2 q – p q 2 x2 + 9x – 10 2m2 – 6

Soluciones

3 5p
1. 2 2. a 3. a 4. 5. a 6. 7. m 8. 10
5b 2b b 8p 2 3q2 9n 7b
a3 b3 a6 3q 8
9. – 25x5y4z3 10. 1
11. 12. 13. 14. – 3d
4a5 b5 4 2 2 7ab
m2 n2 1 1 z 1
15. – 16. a + b 17. 18. 19. 20. 21. 11
3 p2 + 1 2 2 3 3y 144 c
(a + b )
m3 n3 1
22. 23. 2a6b6 24. 25. a + b 26. a b 27. x – 5 28. 1 – a
4 5b4 2

1 1 1 3x 2
29. ab – ac 30. 31. 32. 33. 2x + y 34. p – q
x+4 xy – y2 2a – 3b

5x x–2
35. 36. m – n 37. a + x 38. x + 4 39. x2 – y 40.
y–2 m+n b–c x+2 x+2

1 a+b 5x 3a – 6 x2 – x x–6
41. 42. x + y 43. 44. 45. 46.
2 2x + 3y 2a + 4 x+1 x+5

x b x2 + 1
47. 48. x4 + 3y2 49. 50. x – 1 51. 52. x – 6
x2 + 3y 2 5a – b a–1 x+1 x+1
a–b 2px 2p – 1
53. 2ab 54. 2 55. 56. 57. x + 11 58. x – 5 59.
a–b c+d p–q x + 10 x+3 2p + 1

m3 – 9m
60.
m2 – 3

1.7.2 Multiplicación y división


de fracciones algebraicas
Multiplicamos los numeradores y los denominadores entre sí y
hacemos todas las simplificaciones posibles.
En el caso de los monomios las simplificaciones pueden hacerse antes
o después de multiplicar; en el caso de los polinomios (expresiones con dos
términos o más) es conveniente hacer todas las simplificaciones primero
(factorizando por supuesto) y luego las multiplicaciones.
Para dividir fracciones, multiplicamos la primera por el recíproco
de la segunda.

Álgebra en los números reales 45

44-45. 45 18/11/02, 11:21 AM


Ejercicios 1. Efectuemos el siguiente producto:
resueltos 3ab •
2b
2a 3a2
Multiplicando en forma directa obtenemos:
3ab 2b 3ab • 2b 6 ab2 b2
• = = = 2
2a 3a2 2a • 3a2 6 a3 a
2. Efectuemos el producto:
3xy 6ab – 5z2
• •
2a 3xz 10b2 x
Multiplicando en forma directa obtenemos:
3xy • 6ab • – 5z2 – 90 x y a b z2 –3yz
2
= 2
=
2a • 3xz • 10b x 60 a x2 z b 2xb
3. Efectuemos el producto:
a+b a2 – 2ab + b2
• =
ax – bx a2 – b2
Aquí debemos simplificar antes de multiplicar (de lo contrario
complicamos mucho el ejercicio). Como sabemos, factorizamos
primero, obteniendo:
a+b a2 – 2ab + b2 a+b a–b a–b 1

2 2
= • =
ax – bx a –b x a–b a+b a–b x
Una vez hechas las factorizaciones podemos simplificar un
factor de cualquier numerador con un factor igual de cualquier
denominador.

4. a + 1 a2 – 4 a2 – 9
• •
a+2 a2 + 4a + 3 a2 – 4a + 4
Factoricemos primero:
a+1 a2 – 4 a2 – 9 a+1 a+2 a–2 a+3 a–3
• • = • •
a+2 a2 + 4a + 3 a2 – 4a + 4 a+2 a+1 a+3 a–2 a–2
el resultado es a – 3
a–2
5. Multipliquemos:
5a + b ab a
• •
2a – b a–b 5
Aquí no es posible efectuar ninguna simplificación; por lo tanto,
procedemos a multiplicar directamente.
5a + b • ab • a 5a + b a2 b 5a3 b – a2 b2
= =
2a – b a – b •5 2a – b 5a – 5b 10a2 – 10ab – 5ab + 5b2
Reduciendo términos semejantes obtenemos finalmente:
5 a3 b – a2 b2
10 a2 – 15 a b + 5 b 2
6. Efectuemos la siguiente división:
2ab 2a
3x
: 3x y
Cambiamos el signo de división : por el de multiplicación • e
invertimos la segunda fracción. Nos queda:

46 Álgebra en los números reales

46-47. 46 18/11/02, 11:24 AM


CAPÍTULO 1

2ab 2a 2ab 3xy


3x
: 3xy = 3x

2a

Hacemos las simplificaciones adecuadas y obtenemos:


2ab 3xy
• = by
3x 2a

7. Efectuemos la siguiente división:


a+b a2 – b2
2ab
: 6a2 b
Procediendo como en el ejemplo anterior:
2 2
6a2 b
a+b
: a6a–2bb =
a+b

2ab 2ab a2 – b2
a+b 6a2 b
= •
2ab a + b a –b
3a
=
a–b
(Aquí fue necesario factorizar el término a2 – b2 antes de simplificar).

Ejercicios
I. Simplifique las siguientes expresiones:
12x6 y10 6p + 12q x2 + 5x – 14
1. 3ab 12. 23. 34.
9a – 4x5 y11 p + 2q x2 + 9x + 14
2
2 a+b 1–a x4 – 2 x2 y2 + y4
2. ax 13. 24. 35.
bx a+b 3 1 – a2 x4 – y4
2ab p–q 3 ac + ad + bc + bd
3. 14. 25. 12x – 3x 36.
5ab 2p – 2q 3x ac – ad + bc – bd

3a + 3ab 4 – x2 2 2
4. 3a 15. 26. 37. 2 a b + 6 a b
18a2 b 1+b 2+x a2 + a b – 6 b2
– 16m2 2xy 2bx2 + 2bxy – axy – ay2
5. 16. 3x + 15 27. 38.
18n2 5x + 25 x2 y – xy2 6abx2 – 3a2 xy
b a2 + a 2 2
a4 + 3a3 + 2a2
6. 17. 28. 9x – 16y 39.
a b2
2
2a + 2 3x + 4y a3 – a
6m2 n 20x2 – 5xy a – 9b2
2
2t 2 – 2t – 12
7. 18. 29. 40.
15mn2 4x – y 3a + 9b 4t 2 – 16t + 12
7p2 qr 6 3x2 y – 3xy2 m– n 2 50 – 2y2
8. 19. 30. 41.
3pq2 r 5 2x2 – 2xy m2 – n2 4y2 + 44y + 120
– 3p6 q3 x2 – y2 a2 – 5a + 6 ab – ay – bx + xy
9. 20. 31. 42.
24p6 q2 x–y a2 – 4 b2 – by + bx – xy
a2 b2 c2 a2 – b2 2 a2 – 2 b2 2
10. 21. 32. 2 2
43. 2a – 10a + 12
2abc a+b a +2ab+b a2 + a – 6
2 2
9x2 y7 z11 3a – 3b 18 a b + 27a b 4u2 – v2
11. 22. 33. 44.
x2 y6 z10 a2 – b2 2
4 a + 12 a b + 9 b 2
6u2 v – 3uv 2

Álgebra en los números reales 47

46-47. 47 18/11/02, 11:27 AM


Ejercicios
p2 x + px 2 2 a2 b2 c2 x3 – y3
45. 47. 49.
p2 + 3px + 2x2 4 a2 b c + 2 a b2 c + 2 a b c2 x2 – y2

a3 – a2 – 30a x3 – y3 m2 – 4p2
46. 48. 50.
a – 11a3 + 30a2
4 x2 + xy + y2 m2 – 4mp + 4p2

II. Efectúe las operaciones indicadas (multiplicaciones):


1 2a + 4 a + 4 a2 – 8a + 16
1. a ab∑ 11. x2 + 2 x y + y2 • 21. • •
2b b x+y 3a – 12 a2 – 16 a+2

2x 1 2a3 a2 – 1 x2 – 6x + 5 x2 – 4
2. y ∑
x 12. • 22. •
a –1 a2 x+2 x2 – 1
2 x–6 x2 – x – 2 2x
3. 2m ∑ 3mn 13. 9x – 1 • 2x 23. • •
3n 4 2 x–2 x2 – 9x + 18 x+1
x 3x – 1
a2 ab m– n • m+n m2 – mp 4p 1
4. ∑ 14. 2 2 24. • •
b a m –n m– n 2 2p2 m2 – p2 2m

3x 10y 2 a2 – 3a – 18 a2 – 16 a+2
5. ∑ 15. a + a •
3a
25. • •
5y 6x a2 a2 + 2a + 1 2 2
a – 2a – 8 a – 5a – 6 a+3
2 2x2 + 6x 2xy 1 x3 – y3 3xy
6. 3u v • 2 u v 16. • 26. • •
2u 18 v 2 3y x+3 2x 3y x2 + xy + y2

10a2 bc 2b x2 + 5x + 6 x–2 a+b ab a2 – 2ab + b2


7. • 17. • 27. • •
4b3 5ac x2 – 4 x2 – 9 a2 – b2 a+b 3ab
2
7xy m 10m 1+x 1 – x2 2 2 2 2
8. •
2
• 18. •
2
28. a – 25b • a – 7b + 12b
2m 14x y 2x 1–x 1+x a – 3b a – 5b

a2 – ab a+b 2
9. a •
2b

4c 19. •
2
29. 2a – 4 ∑ a2 – 5 a + 6 ∑
a
b 3c a 2ab a2 –b 6a a –4a+3 a–2
x–y x2 3x – 6 x2 – 9 1 a3 + 2a2 + a a2 – 25 1
10. • 20. • • 30. • •
x x–y 2x – 6 x2 – 4 3 a2 + 7a + 10 a + 1 2a2

III. Efectúe las siguientes operaciones (divisiones):

15n2 p 3np2 2
b2 c2
1. a a 3
: 5. 2ab 6a
: 2ab 9. : 13. 15a bc : 25a
2 3 3b 2nz 4z 3ab2 bc
a2 x3 – x2 y x2 – y2
2. 1 : 2 6. x – 1 : x10
–1 10. –1
: aa +– 12 14. :
a a 5 a+2 2xy x+y
x2
: xy 2axy 2x 2
3. y 7. : 3y 11. x – 1 : ax2 –– xa 15. 2 a b : 2 x : 3 b x
3a a–1 8 b2 x – 1 2 x – 2
2 2x – 6 2
:x – 5x + 6
2
4. m n
: ax 8. a + b : a 2 – b2 12. a – b : a+b 16.
ax a–b a3 –b 3
a–b 3x2 y 6xy

48 Álgebra en los números reales

48-49. 48 18/11/02, 11:36 AM


CAPÍTULO 1

1
17.
a2 – 49
: a2 – 81a + 7 24. 1 : 1 : 1
x x x
2
1 2 a 22x y 11 x y
18. : :
2
25. : 14 : 2x
a a2 7
2 3 2 2
19. a – 1 : a2 – 1 26. a + 3 a : a + 2a
2 2
a–2 a –4 a –9 a – 5a + 6
2
20. a – 3 : a – 8a + 15 27. 1
: 1
2 3 2 2
a – 5 a – 11a + 30 x – 6x x – 12 x + 36
2 2 2
ac – ad – bc + bd c –d
21. 2a + 8 : a + 2a – 8 28. :
2 2 2 2 2
3a – 3 6a – 6 a –b a + 2 ab + b
3 2 3 2 2 2
22. a – 5a + 6a : a – 3a 29. x + 7 x + 10 x + 2
: •
x + 3x – 4
2 2 2 2
a + 7a + 12 a – 16 x + 2x – 3 x + 3 x – 25
2 2
a
23.
2
:a 30. 2x • x – 4 x + 2
2
: 2
b b x–2 x +x x –1

Soluciones
I. Simplificación
2
2. a x 8. 7 pr
– 8m
1. b 3. 2 4. 1 5. 2
6. 1
7. 2 m 9. – q
3 b 5 6 ab 9n a b
2
5n 3q 8

abc – 3x 1 1 3 a
10. 11. 9yz 12. 13. 14. 15. 3a 16. 17. 18. 5x
2 y a+b 2 5 2

1
19. 3 y 20. x + y 21. a – b 22. 3 23. 6 24. 25. 4 – x2 26. 2 – x
2 a+b 1+ a

a – 3b m– n
27. 2
28. 3x – 4y 29. 30. m+ n
31. a – 3 32. 2 a – 2 b
x–y 3 a+2 a+b
2 2 2
x–2
33. 9 ab
34. 35. x2 – y 2 36. c + d 37. 2 ab 38. x+y
39. a + 2 a
2a + 3b x+2 x +y c–d a – 2b 3a x a–1

5– y a–x 2a – 6 2u + v
40. t+ 2
41. 42. 43. a+3
44. 45. p x
2t– 2 12 + 2 y b+ x 3uv p+ 2x

a bc 2 2 m+ 2p
46. a+5
47. 48. x – y 49. x + x y + y 50.
2
a – 5a 2a + b + c x+y m– 2p

II. Multiplicación
2
a
2
2 2 u a 5m
2
8
1. 2. 3. m 4. a2 5. 1 6. 7. 8. 9. 10. x
2b y 2 6 b 4x
2 3

2
1 1
11. x + y 12. 2a2 + 2a 13. 6 x + 2 14. 15. 3
16. 4x
17.
x m– n
2 a+1 3 x–3

x+3 2
2x 1
18. 1 19. 1 20. 2x + 4
21. 2
22. x – 7 x + 10
23. 24.
2b 3 x+1 x–3 mp + p2

a+4 x–y a–b a–2 a2 – 4 a – 5


25. 26. 27. 28. a2 + ab – 20b2 29. 30.
a+1 2 3a + 3b 3a – 3 2 a2 + 4 a

Álgebra en los números reales 49

48-49. 49 18/11/02, 11:40 AM


Soluciones

III. División

2. 1
3
1. 3. x 4. m
5. 2 ab 6. 2 7. y2
2 2 n 9

1 10 a a–b 1
8. 9. 10. a + 1 11. 12. 13.
2 2 p x 2
a + ab + b
2
5b
2
a – 2a b + b
14. x 15. 3a 16. 4
17. a–1
18. 1 19. a + 2
2y 16 2
x – 2x a+7 a+1
2
1
20. a – 6 21. 4a + 4 22. a –2 6a + 8 23. 24. 1
x
25. 2
a–5 a–2 a + 3a ab
2
26. a – 2a 27. x–6 28. a + b 29. x+4
30. 2x – 2
a+2 x
2 c+d x–5

1.7.3 Adición y sustracción


de fracciones algebraicas
Si las fracciones tienen el mismo denominador, entonces sumamos (o
restamos) los numeradores y conservamos el denominador.
Si los denominadores son diferentes, entonces debemos buscar el
mínimo común múltiplo (m.c.m.) de ellos y amplificar cada fracción
por el factor necesario, de modo que todas queden reducidas a un
denominador común.
El mínimo común múltiplo de expresiones algebraicas es aquella que
las contiene, como factores, a todas.

Ejercicios 1. Encontremos el m.c.m. entre a y 2a.

resueltos Vemos que a está contenido (como factor) en 2a, por lo tanto,
el m.c.m. es 2a.
2. Encontremos el m.c.m. entre a, 2a y a2.
Aquí ninguno de los tres términos contiene a los otros dos. Buscamos
el m.c.m. entre los coeficientes numéricos, en este caso es 2, y entre los
factores literales, en este caso, como se trata de monomios de la misma
base, es el término que tiene el exponente más alto.
Así, el m.c.m. es 2a2.
3. Encontremos el m.c.m. entre 2x, 3xy, x2.
Usando el razonamiento anterior, determinamos el m.c.m. entre los
coeficientes numéricos, que es el 6, y entre los factores literales, que es
x2 y. Así, el m.c.m. entre 2x, 3xy, x2 es 6x2y.
4. Encontremos el m.c.m. entre a – b y a2 – b2.
Como sabemos, la factorización correspondiente de a 2 – b2
es (a – b) (a + b); por lo tanto, a – b está contenido en a2 – b2 y
así el m.c.m. es a2 – b2.

50 Álgebra en los números reales

50-51.(2003) 50 18/11/02, 11:46 AM


CAPÍTULO 1

5. Encontremos el m.c.m. entre a + 2 y a + 3.


Aquí ningún término está contenido en el otro; por lo tanto, el m. c. m.
es el producto de los dos, es decir, a2 + 5a + 6.
6. Efectuemos las operaciones indicadas.
a + 2 2a + 5
+
3 3
Se trata de una suma con igual denominador, así es que sumamos
los numeradores y conservamos el denominador.
a + 2 2a + 5 a + 2 + 2a + 5 3a + 7
+ = =
3 3 3 3

7. Efectuemos las operaciones indicadas:


4 5 3
+ –
2
x 2x 5x
Los denominadores son diferentes; por lo tanto, debemos
determinar el m.c.m. entre ellos, que será el denominador común.
Este es 10x2. Luego amplificamos cada fracción por el término
adecuado para obtener el m.c.m.
4 5 3 4 • 10x + 5 • 5x – 3 • 2
+ – 2
= 2
x 2x 5x 10x
40x + 25x – 6
= 2
10x
65x – 6
= 2
10x
8. Efectuemos las operaciones siguientes:
m+ 1 m+ 1 1
2
– 2
+
2m + 4m m –4 m– 2

Factoricemos los denominadores para encontrar el m.c.m.


m+ 1 m+ 1 1
– +
2m m + 2 m– 2 m+ 2 m– 2
El m.c.m. es 2m (m + 2) (m - 2)
Es conveniente mantener el m.c.m. factorizado, pues así facilita el
proceso de amplificación de cada fracción y el de simplificación,
si es posible, al final.
m+ 1 m+ 1 1
– + =
2m m + 2 m– 2 m+ 2 m– 2
m – 2 m + 1 – 2m m + 1 + 2m m + 2
=
2m m + 2 m – 2
2 2 2 2
m + m – 2m – 2 – 2m – 2m + 2m + 4m m + m– 2
=
2m m + 2 m – 2 2m m + 2 m – 2
Factorizamos el numerador y hacemos la simplificación corres-
pondiente:
m+ 2 m– 1 m– 1
=
2m m + 2 m – 2 2m m –2
m– 1
=
2
2m – 4m

Álgebra en los números reales 51

50-51.(2003) 51 18/11/02, 11:47 AM


Ejercicios

I. Determine el mínimo común múltiplo entre:

1. 2, 3, 5 16. 6m, 3m + 1, 6m + 2

2. 2, 2a, 3a 17. x + a, x2 – a2, x – a

3. 3x, 3xy 18. 1, x + 1, x + 2

4. 2x, 3xy, 2y 19. a, b, a + b

5. m2, n2 20. a2 – b2, a2 – 2ab + b2

6. m, mn, n 21. x + 3, x2 + 5x + 6, x + 2

7. x2, y2, xy 22. x – 3, 2x – 4, x2 – 5x + 6

8. 1, a, a2 23. a2 + a, a2 – 1, a2 + 2a + 1

9. x2yz, xy2z 24. a + 2, a2 + 4a + 4, a2 – 4

10. xy2z, xyz2 25. x2 + 9x + 14, x2 – 4, x2 + 5x –14

11. 4p2q, 5pq2 26. a – 1, a2 – 1, 3a2 – 3a

12. 5p6q6, 6p5q5 27. p, p + 5, p3 – 25p

13. a + b, a – b 28. 2x + 2, 4x + 4, x2 + 2x + 1

14. 2a + 4, a + 2 29. t – 5p, t2 – 25p2, 5t – 25p

15. 3a + 6, a2 – 4 30. x + y, x2 + 2xy + y2, x2 – y2

II. Efectúe las operaciones indicadas:

3 4 15 a
2
4a 4
1. – + 9. – +
11 11 11 2 2 2
a –4 a –4 a –4
3 5 21 3
2. + + +
16 16 16 16 10. 7p – 3q – 6p – 4q + 2p
2 2 2
p +1 p +1 p +1
3. 2 – 6 + 9 – 12
a a a a
11. 2a a + 4 – 3a a + 6 + 2a a – 5
2 2 2
4. 3a – 1 + 2a – 7 – 2 a – 20a a – 20a a – 20a
5 5 5 3a
2
3a a + 4 4a • 4b
9 2 1 12. – +
5. + + 3a – 4b 3a – 4b 3a – 4b
3x – 4 3x – 4 3x – 4 2 2
13. x – 7x + 1 2x x –3 x +x+2
a + 5b 2a + b 4a + 5b – +
– + 2 2 2
6. x + 5x + 6 x + 5x + 6 x + 5x + 6
a + 3b a + 3b a + 3b
2 2 2

7. 2x – 2 + 3x – 1 – 4x – 4 14. 5x – 7x – 1 + 2x + 4x – 1 – 3x – 6x – 2
2 2 2
x+6 x+6 x+6 x + 3x x + 3x x + 3x
2 2 2
3x + 5 5x + 8 x – 4
8. + – 15. 2x + 3x + 6 + x – 6x + 8 – 3x – 4x + 3
2x – 3 2x – 3 2x – 3 2
x – 121
2
x – 121
2
x – 121

52 Álgebra en los números reales

52-53. 52 18/11/02, 11:49 AM


CAPÍTULO 1

2 4 1
16. + – 34. 2a + b – 2a – b – a
3 5 2 3a 3b b
2a – 1 2a – 2 a
17. a + a 35. – 2

2 3(a – 2) a –4 3
a a a 2x 3x – y 2x – 4y
18. + +
2 3 4
36. – + 2 2
x+y x–y x – 2xy + y
19. 2b – b
a+7 a–6 2a – 4
37. – + 2
3 a–6 a–5 a – 25
2x x+4 2x – 3
20. 2x – x + x 38. 2
+ 2
+ 2
5 7 35 x + 3x + 2 x –4 x –x–2

21. a + a + a 39. x + 1 + x – 2 – x–3


+
x+4
2 3 3 2 2 2 2
x –1 x –9 x + 5x + 6 x – 4x + 3

22. a + a – a 40. 3x – 2 – 2
1
– 2
2x

6 – 3x
2
2 3 2x + 8 x – 16 x + 5x + 4 x –1
2
23. 3x – 2x + x 41. 2x – 1 + 3x + 1 – 5x2 + 19x – 2
5 5 10 x+3 x+5 x + 8x + 15
m 1 3m
24. a – 2 42. 2
– 2 3
– 2
9 m+ m m +m m –1

25. 1 + x 43. m + 1 + m + 2 + 2
2m
x m– 4 m– 5 m – 9m +20
x 2x x–2
26. 1 + x 44. – + 2
x 2 2x + 3 2x – 3 4x – 9

27. a – 7a – 2a
2 1 3
45. 2
+ 2
– 2
4 3 x + 7x + 12 x – 16 x –9

28. 3 4 5 46. x–4 2x – 4 4 – 2x


+ + 2
– 2
– 2
2a 3a 6a 3x + 12x x + x – 12 2x – 6x
2
3x 4x 2x – 3x
29. x – 1 + x – 2 + x 47. 2
+ 2
+ 2
4 3 2 3x + 15x x + 8x + 15 3x + 9x
1 1 2 1 2+ x 2– x
30. – – 48. + 2
– 2
2 3
a a a x+2 x + 4x + 4 x –4
3x 2x 11x 2x x–2 x–5
31. + – 49. 2
– 2
+ 2
x – 3 2x – 6 2 x – 2x – 24 x – 36 x – 16
1 1 1 2x x–3 x – 12 3x
32. + 2
– 2
50. – 2
– +
z z z –1 3x + 15 x – 25 6x x–5

33. 3x – 5 + 2x – 7 + x – 1
2
x–1 x –1 x+1

III. Efectúe las operaciones indicadas:

2a 6a a+3 1 ab 1 b
1. –
a – 3 a2 – 9

2a
2. 1+ : 1 – 1x 3. a+ • +
x a+b 2 2a

Álgebra en los números reales 53

52-53. 53 18/11/02, 11:51 AM


Ejercicios
2
3 5 3x – x
1 1 + 2+
4. +1 : –1 2 5 5
x x 17. 24.
2 2
x –1
15 3–
x–y 1– x 10
5. –1 •
x+y 2y a a ab
– a–
2 3 a+b
a x a+x 18. 25.
6. + : 2 2
a
+
a
1–
b
a+x a+x a + 2ax + x 2 3 a+b
7. 1 x–1
a–b : 1– –1 1
a+b –y
x+1 x
19. 26.
1 1 x–1 1
8. : 1+ +y
1–
x–1 x+1
+1 x
x+1
2 1
9. a + ab : 1 – 1 1 1 1
2a a – 27. 1 +
x–2 x+2 1
2
20. 1+
10. m + mn 1 1 x
m
: 1+ m 2
x –4 a–b a+b
2 –
11. 2a – 3b • b – 1 a

b 28. a a
5b b – 1 2 b a 1
21.
2 1 1 a
12. 2a • 4 – c 2+ c –
2+ c 4b
: 2c
–1 a b
x
2–
1 2
13. a+b a–b 2ab 22. 1 + 29.
– •
2 2 1 x
b a a +b 1+ 3–
x 3
14. b a a–b 2
a
+ –2
b
: 1
a 1 1–
23. x –1
2

x x 2 1 30.
15. + –1 • x –1 1– 1
x+1 x–1 1 1+
1– x+1
1 1
x

2 4
16.
1 1
+
2 4

Soluciones
I.
1. 30 2. 6a 3. 3xy 4. 6xy 5. m2n2
6. mn 7. x2y2 8. a2 9. x2y2z 10. xy2z2
11. 2
20p q 2 12. 30p6q6 13. a2 – b2 14. 2a + 4 15. 3a2 – 12
16. 2
18m + 6m 17. x – a2
2 2
18. x + 3x + 2
19. a2b + ab2 20. (a – b)2 (a + b) 21. x2 + 5x + 6
22. 2 (x – 2) (x – 3) 23. a (a – 1) (a + 1)2 24. (a – 2) (a + 2)2
25. (x + 2) (x – 2) (x + 7) 26. 3a (a2 – 1) 27. p (p2 – 25)
28. 4 (x + 1)2 29. 5 (t2 – 25p2) 30. (x – y) (x + y)2
II.
14
1. —– 2. 2 7
3. – — 4. a – 2 12
5. —–—
11 a 3x – 4

54 Álgebra en los números reales

54-55. 54 18/11/02, 11:55 AM


CAPITULO 1
CAPÍTULO

a–2 3p + q
6. 3 7. x + 1 8. 7x + 17 9. 10.
x+6 2x – 3 a+2 2
p +1
3 4x + 3 1
11. 1 12. – 4 13. 14. 15.
(x + 2) (x + 3) x+3 x – 11

16. 29 17. 3a
18. 13a 19. – b 20. 2x
30 2 12 3 7
2
a – 18
21. 7a 22. 7a
23. 3x 24. 25. 1 + x
6 6 9 x
10
2 2
26. 2 + x 27. – 17a 28. 11 29. 13x – 11 30. – a + a – 2
2x 12 3
3a 12 a
2 3 2
41x – 11x z –z–1
31. 32. 2 2
33. 4x – 2x – 11
2x – 6 z (z – 1) 2
x –1
2 2 3 2 3 2 2 2 2 3
34. 3ab + b – 5a 35. – a + 2 a + a + 4 36. – x – 3x y + 5xy – 2xy + 2x – 4y – y
3ab 2 2
3(a – 4) (x + y)(x – y)
2 2 3 2
5x + 2x – 2
37. 16a – 17a – 331 38. 3 2 39. 2x + 17x – 2x + 19
2 x + x – 4x – 4 2
( a – 25) (a – 6) (x – 1) (x – 9) (x + 2)
4 3 2
3x – 12x + 11x – 98x + 186
40. 41. 0
2 2
2 (x – 16)(x – 1)
3 2 2 2
– 2m – m – m + 1 2m – 4m – 13
42. 2 2
43. 44. – 2x – 8x – 2
(m – 4) (m – 5) 2
m ( m – 1) 4x – 9
– 14x + 63 2 3 2
45. 46. – 2x + 11x – 12 47. 2x + 22x – 6x
x2 – 16 x2 – 9 3x (x + 4) (x – 3) 3x (x + 3) (x + 5)
3 2 3 2
48. 3 49. 2x + x – 68x +148 50. 21x + 76x + 43x – 300
2 2 2
x+2 (x – 36)(x – 16) 6x (x – 25)
III.
a
1. —— 2. x——
+1 3. a———
+ 2b 4. 1
——+x
a–3 x–1 2 1–x
x–1
5. —— 6. a + x a2 – b2
7. ———–
2–x–2
8. x————
x+y a+b–1 x2 + x – 2
a2 + ab
9. ——— 10. m
2 + mn
———— 11. 4a – 15b3 – 15b2
——————— ac
12. —–
2a – 2 m+1 10b b

13. 2 1
14. —– 15. x2 + 1 1
16. —
b 3
75
17. —– 1
18. —– 1
19. – — 20. 4
4 5 x

2x + 1 2 – 2x + 20
21. – a – b 22. ——— 23. – x 24. 6x
—————–
x+1 31 – x2
25. a 1 – xy
26. ——– x+1
27. ——– 28. –2b
1 + xy 2x + 1
2
29. 12
———– 3x x –2
30. ————
18 – 2x x2 + x –2

Álgebra en los números reales 55

54-55. 55 18/11/02, 11:57 AM


Prueba de selección múltiple

Marque la alternativa correcta.


1. Si a = – 1 y b = – 2 5. –3p • 2pq =
el valor de a – ab es:
A. – 5p2q
A. – 1 B. 6p2q
B. – 2 C. – 6pq
C. 1 D. – 6p2q
D. – 3 E. 6pq2
E. 2
6. Si p = 1 y q = – 1 entonces
p + q + pq es:
2. Al reducir la expresión
a A. – 1
– a se obtiene:
2
a B. 1
A.
2 C. 0
a D. 2
B. –
2
C. – a E. – 2

D. 0
7. Si p + q = – 6 y q = 2 entonces el valor
E. – 1 de p es:
2
a A. 6
3. Al reducir 2a – a –
2 B. 8
se obtiene:
C. – 8
A. –
a D. – 4
2 E. 4
B. 1
2 8. Si m + 5n = 5 y n = – 2 entonces el
C. a valor de m es:
2 A. 15
D. 3a B. – 05
2
1 C. 0 5
E. –
2 D. – 15
E. – 10
4. Si m = 2 y p = 3 entonces
m2 – p2 es: 9. Si a = – 5 y a + b = 5 entonces el valor
de b es:
A. 5
A. 0
B. –5
B. 10
C. 13
C. 05
D. – 13
D. – 5
E. –2 E. – 10

56 Álgebra en los números reales

56-57. 56 18/11/02, 11:59 AM


CAPITULO 1
CAPÍTULO

n
10. Si m = y n = – 16 14. La expresión “el cuadrado
2
entonces el valor de m de la diferencia entre a obtiene:
es: y b” es:
A. 2a
A. 32 A. (a – b)2 B. – 2a
B. – 32 B. a2 – b2 C. 2b
C. 8 C. a – b2 D. – 2b
D. –8 D. 2(a – b) E. 0
E. – 4 a–b
E. 19. Al reducir
2
s 3m – [2m – (3p + m) – p]
11. Si q = – 2r, r = y
2 15. “El doble del producto
s = 9 entonces el valor se obtiene:
entre a y b” corresponde
de q es: A. 2m – 4p
a:
B. m + 2p
A. 9
A. 2a2b C. 2m + 4p
B. –9 B. 2ab2 D. 2m + 2p
9
C. C. 2a2b2 E. – 4p
2

D. 18 D. a2b2 1 2 1
20. a+ a– a–a
E. 2ab 2 3 3
9 es igual a:
E. –
2
16. Al reducir a
12. La expresión “el doble del A.
2a – [ a – (a – 2a) ] 6
cuadrado de a” corres- a
se obtiene: B. –
ponde a: 6
A. 2a a
A. (2a)2 C. –
B. 0 2
B. 2 (a2)2 a
D.
C. a 2
C. 2a2
D. 4a a
D. (2a2)2 E. –
3
. a2 E. – 4a
21. a • a2 • a– 2 =
13. La expresión “el cubo 17. Al reducir
A. a
de la mitad de a” corres- (a + b) – (a – b) se
B. a– 3
ponde a: obtiene:
C. a– 4
3a3 A. 2b
A. D. a3
2 B. – 2b
3
E. a5
B. a
C. 2a
2
a
2 D. – 2a 22. ab2 • – ab2 =
C.
3 E. 0
A. 0
D. a 3
2 18. Al reducir B. – a2b2
3a (a – b) – (a + b) se C. – a2b4
E.
2
Álgebra en los números reales 57

56-57. 57 18/11/02, 12:00 PM


Prueba de selección múltiple

D. a2b4 C. x2y D. a(a + b + 1)


E. – 2a2b4 D. 2xy2 E. (a + b) (a + b + 1)
E. – 2xy2
23. 2m • – 3m • – 4mp2 = 34. Para que la expresión
A. 24m3p3 29. Al factorizar m2–mn se
9a2 + 12ab + .....
B. – 24m3p3 obtiene:
sea un cuadrado de
C. 24m3p2 A. mn(m –1)
binomio falta:
D. – 24m3p2 B. m2(m – n)
A. 4b2
E. – 9m3p3 C. m(m – n)
B. 4b
D. m(1 – n)
24. x – [2x – 3y + (3y – 2x)] = C. 4
E. m2 (1 – n)
A. 3x – 6y D. b2
B. 4x – 6y 30. Al factorizar 4 – p2 se E. 9
C. 4x + 6y obtiene:
2
D. – x A. (2 – p)2 35. a b c =
abc
B. (2 – p) (2 + p)
E. x 1
C. (p – 2) (p + 2) A.
c
25. a (a2 + a3) = D. (4 – p)2 B. c
A. a6 E. 2p (2 – p) C. 1
B. 2a6
D. abc
31. La expresión 1 – p6 es
C. a7
equivalente a: E. 1
D. a2 + a3 abc
E. a3 + a4 A. (1 – p3) (1 – p2)
B. p3 (1 – p2) 36. a+ ab =
26. m(1 + m) – m(1 – m) = ab
C. (1 – p3) (1 + p3)
A. ab
A. – m2
D. (1 – p3)2
B. 2m2 B. a
E. (1 – p2)3
C. m – m2 C. a + 1
a
D. m + m2 32. Factorice
D. b + 1
E. 0 m2 – n2 – m – n = b
E. b
27. a(1 + a + a2) – a = A. (m – n) (m2 + n2)
B. (m + n) (m – n – 1)
A. a + a2 2 2
37. m – n =
B. a + a3 C. (m – n) (m – n – 1) m– n

C. a + a2 + a3 D. (m + n) (m – n + 1) A. m – n
D. a2 + a3 E. (m – n) (m – n + 1) 1
B.
E. 1 + a + a2 m– n
33. (a + b) + (a + b)2 =
C. m + n
28. xy (x + 2y) – 2xy2 = A. 3(a + b)
1
D.
A. x2y + xy2 B. 3(a + b)2 m+ n
m+ n
B. xy2 C. 3(a2 + b2) E.
m– n

58 Álgebra en los números reales

58-59. 58 18/11/02, 12:03 PM


CAPITULO 1
CAPÍTULO

2
38. a – 1 : a –1 = 41. x – 11x + 28 = D. – 1
B. – 3m
a a x–7 2 n
E. a – 2
A. a + 1 A. x – 4 C. 3m
x–7 n
B. a – 1 B. x – 7 3
x +y
3
3m
44. = D.
1
C. x – 4
2
x – xy + y
2 2n
C.
a +1

D. 1 D. x + 4 A. x + y E. –3m
a–1 1 2
E. x + 7 B.
x+y n+ 1 n
E. 1 47. a + a
=
a
2 C. x – y n
a
x+5 A. an
42. =
2 2 2
x – 25 1
39. 3xy – 3x y = D. B. a
3xy A. x + 5 x–y
C. a + 1
A. 3 ( x – y)
B. x – 5 E. x + y D. an+1
B. 3 (y – x) xy
E. an–1
C. y – x 1
C. a b
x+5 45. + = 48. 56 + 56 + 56 + 56 + 56 =
D. x – y a+b a+b
E. y – 3x 1 A. a A. 530
D.
x–5
B. (a + b) B. 57
E. 1 C. 256
2 2 C. (a + b)2
40. a – b = 2
x –5 D. 2530
4 4
a –b D. 1
1 2 n n+ 1
A.
49. 3 4 – 4
a –4 •
2 2 43. = E. a+b =
a –b 2 n
1 a + 3a + 2 2
a +b
2
4
B.
a +b
2 2
A. 3 – 4n+1
A. a – 2
C. a2 – b2 a+1 B. 2
46. m – 2m =
D. a2 + b2 2n n C. 1
B. a + 1
a–2 – 3m D. –1
1
E. A.
a–b C. – 2 2n E. 0

Soluciones

1. D 8. A 15. E 22. C 29. C 36. D 43. A


2. B 9. B 16. B 23. C 30. B 37. C 44. A
3. C 10. D 17. A 24. E 31. C 38. A 45. D
4. B 11. B 18. D 25. E 32. B 39. C 46. A
5. D 12. C 19. C 26. B 33. E 40. B 47. C
6. A 13. D 20. B 27. D 34. A 41. C 48. B
7. C 14. A 21. A 28. C 35. B 42. D 49. D

Álgebra en los números reales 59

58-59. 59 18/11/02, 12:05 PM


CAPÍTULO 2
E
cuaciones
e inecuaciones
de primer grado

2.1 Ecuaciones

Definición: Se llama ecuación a una igualdad que presenta


incógnitas y que es verdadera sólo para algunos valores
de la incógnita:

Ejemplos: 2x – 5 = 3
4x + 2y – 1 = 0
5x2 – 3x + 2 = 0
ax – by = 3 – ab
2x3 – 1 = x2 + 2
• Observación 1: La expresión de la izquierda del signo igual
se denomina primer miembro y la del lado derecho se llama
segundo miembro.
• Observación 2: Una ecuación puede tener una o más incógnitas.
• Observación 3: Se llama grado de una ecuación al grado del
término que presenta el grado más alto, después que se hayan reducido
los términos semejantes.
Ejemplos: 2x – 1 = 5 es ecuación de primer grado
x2 – x = 7 + x es ecuación de segundo grado
x4 – x + 2 = 0 es ecuación de cuarto grado
2xy + 5 – 3x + 2y = 0 es ecuación de segundo grado
x2 + 5x – 1 = (x – 2)2 es una ecuación de primer grado

60 Ecuaciones e inecuaciones de primer grado

60-61.(2003) 60 18/11/02, 12:29 PM


CAPÍTULO 2

• Observación 4: Se llama raíz o solución de una ecuación a todo


valor de la incógnita que verifique la igualdad.
Resolver una ecuación significa encontrar el o los valores de la o
las variables (incógnitas) para que la igualdad sea verdadera.
Para resolver una ecuación debemos tener presente las siguientes
propiedades de la igualdad.
• Propiedad 1: Al sumar o restar la misma cantidad en ambos
miembros de una igualdad, la igualdad persiste.
• Propiedad 2: Al multiplicar o dividir por una misma cantidad
distinta de 0 en ambos miembros de una igualdad, la igualdad
persiste.
• Propiedad 3: Al elevar a una potencia distinta de 0 ambos
miembros de una igualdad, la igualdad persiste.
• Propiedad 4: Al extraer la misma raíz, en ambos miembros de una
igualdad, la igualdad persiste.

2.1.1 Ecuaciones de primer grado con


coeficientes enteros

1. Resolver la ecuación 3x + 2 = 7 Ejercicios


Debemos despejar x. Para ello restamos 2 en ambos miembros resueltos
(Propiedad 1).
3x + 2 – 2 = 7 – 2
efectuamos las operaciones 3x = 5

3x 5
dividimos ambos miembros por 3 (Propiedad 2) =
3 3
5
efectuamos las operaciones x=
3
Verificamos:
3x + 2 = 7
5
3• +2 =7
3
5+2 =7

7≡7
2. Resolver la ecuación 3 – 2x = 5x – 9
Despejamos x. Para ello restamos 3 y restamos 5x en ambos miembros.
3 – 2x – 3 – 5x = 5x – 9 – 3 – 5x
efectuamos las operaciones
– 2x – 5x = – 9 – 3
– 7x = – 12

Ecuaciones e inecuaciones de primer grado 61

60-61.(2003) 61 18/11/02, 12:30 PM


Ejercicios multiplicamos por – 1
resueltos 7x = 12
dividimos por 7 7x = 12
7 7
12
x =
7
Verificamos: 3 – 2x = 5x – 9
12 12
3–2• =5• –9
7 7
24 60
3– = –9
7 7
21 24 60 63
– = –
7 7 7 7
–3 –3
7 7
3. Resolver la ecuación 2 (x – 1) = 3 (x + 2) – 5 (x + 3)
Primero resolvemos los paréntesis y reducimos términos
semejantes.
2x – 2 = 3x + 6 – 5x – 15
2x – 2 = – 2x – 9
sumamos 2 y sumamos 2x en ambos miembros
2x – 2 + 2 + 2x = – 2x – 9 + 2 + 2x
4x = – 7
dividiendo ambos miembros por 4
4x –7
=
4 4

7
x = –
4
Verificamos:
2 x– 1 =3 x+2 – 5 x+3
7 7 7
2 – – 1 =3 – +2 – 5 – +3
4 4 4
– 11 1 5
2• =3• – 5•
4 4 4
22 3 25
– = –
4 4 4
22 22
– ∫–
4 4

4. Resolver la ecuación
2x2 – 2 (x + 1) (x – 1) = (x – 3)2 – (x + 2) (x – 5) + 1
Primero resolvemos paréntesis y reducimos términos semejantes.
2x2 – 2x2 + 2 = x2 – 6x + 9 – x2 + 3x + 10 + 1
2 = – 3x + 20

62 Ecuaciones e inecuaciones de primer grado

62-63. 62 18/11/02, 12:35 PM


CAPÍTULO 2

restamos 2 y sumamos 3x en ambos miembros


2 – 2 + 3x = – 3x + 20 – 2 + 3x
3x = 18
dividimos por 3 en ambos miembros
3x 18
=
3 3
x = 6
Verificamos:
2x2 – 2 (x + 1) (x – 1) = (x – 3)2 – (x + 2) (x – 5) + 1
2(6)2– 2(6 + 1) (6 – 1) = (6 – 3)2 – (6 + 2) (6 – 5) + 1
2 • 36 – 2 • 7 • 5 = 32 – 8 • 1 + 1
72 – 70 = 9 – 8 + 1
2 ≡2

Ejercicios
1. x + 3 = 5 21. 2 (x + 3) = 5 (x – 1)
2. 2x – 5 = 7 22. (2x – 5) 2 = (3 + x) 5
3. 5 – 2x = x + 2 23. (x + 2) – (3x + 2) = 5 (x + 4) + 1
4. 2y + 1 = 3y + 4 24. 5 (1 – x) + (x – 3) 4 = (x – 1) – (1 – x)
5. 6z – 3 = 5 + 2z 25. 2[(3x + 1) – 2 (x + 4)] – (3x + 5) = 0
6. 4x – 5 + x = 3 + 2x + 4 26. 2x – 3 – (x + 1) = – [x + 3 (x + 2)] – (x + 4)
7. 4 + 2x – x = – 3x – 4 27. – 3 + x – 5 [(2x + 4) – (x + 2)] = x + 2
8. – y + 5y – 3 + 4 = y – 1 28. 2x – 10 – [2x – (x + 3) + 5] = 0
9. y + 2 = 5y – 4 + 3y – 1 29. – [2 (2 – y) – (2y – 3)] –5y = 4 (y + 3)
10. 4y + 9 – y – 2y = 16y + 42 30. 3 – [5y + 2 (y –1) + 4] = 5 –[2 (y–3) –3 (y–2)]
11. 17y – y + 9 = 32 – 19y + 82 31. – x + [12x – 3 (x + 1) – (3x + 2)] = 15x – 16
12. 45x – 33x + 19 = 25x + 17 32. 2x – [14x – 2 (x + 3) – (2x + 3)] = 16x + 9
13. z – 12 + 44z = 18 – 15z 33. – [2 (3x – 3) – (1 + x)] – [5 (3 – 2x) – (1+ x)] = 0
14. 132x + 25 – 33x = – 10 – x + 85
15. 49x – 105 + 16x = 48x – 301 – 8
16. 405x – 203 + 45x = 102 + 115
17. 18z – 42 + 15z = 10z – 3 + 32z – 39
18. 113x + 16 – 14x = – 12 + 27x + 19
19. 15x – 135 + 18x = 45 + 90 – 18x + 15x
20. 339x + 25 = 5 + 309x + 20

Ecuaciones e inecuaciones de primer grado 63

62-63. 63 18/11/02, 12:35 PM


Ejercicios
34. – [– 5 + 3x – 2 (x + 1) – (x + 3)] = [– 2x + 5 – (x + 1)] (– 2)
35. 2x + {3x – [5x + 2 (x – 1) – 3 (4x – 5)] + 3} 2 = 0
36. 5x – {1 + (3x – 5) 2 – [3 (x + 4) – 2x] – (x + 5)} = 17
37. 3 {x + 1 – 2 (x + 3)} – {6x – [2x + 3 (x – 1)] + 1}3 = x + 1
38. 1 – {– 3 [2x – (x + 4) – (6 – 2x) + 5] – 3x} + 2 = 0
39. 4 (x + 2) – {3 (x – 1) – [5 (1 – x) – 6x]}2 = [3 (x – 4) + 1]2
40. 26 – {32x – [1– x + 2 (18x – 3) + 6x]} = 12x – 4
41. (x + 1)2 = 12 + (x – 5)2
42. (x + 3) (x – 1) = 5 + (x – 2)2
43. (3 – x) (x + 4) + 16 = 12x – (x + 3)2
44. (x – 1) (x + 1) – (x + 2) (x + 3) = 5x – 1
45. 2 (y – 2) (y + 2) – (y – 5) (y + 3) = (y – 1)2 + 2
46. y (2y – 3) + (y – 1)2 = 2y (y – 4) + 3 + y2
47. 3x (x – 5) – (x – 3) (x + 3) = 2 (x + 5)2
48. (2x – 5) (4x + 3) – (2x – 1)2 = 3x2 + (x + 4) (x – 4)
49. – {– [– (x + 5)2 + (x + 1) (x – 1)] + (2x – 3) (2x + 3)} = – (2x + 1)2
50. x (x + 5)2 + 2x3 – 4x2 = 3x (1 + x)2 + 22

Soluciones

1. x=2 2. x=6 3. x=1 4. y = – 3 5. z=2

2 11
6. x=4 7. x=–2 8. y=– 9. y = 1 10. y=–
3 5
2 1 1
11. y=3 12. x= 13. z= 14. x = 2 15. x = –12
13 2
14 1 15
16. x= 17. z=0 18. x=– 8 19. x = 20. x=0
15 2
11 5
21. x= 22. x = – 25 23. x=–3 24. x = – 25. x = – 19
3 3
1
26. x=–1 27. x=–3 28. x = 12 29. y = – 19 30. y=–
5 2
11 7 10
31. x= 32. x=0 33. x= 34. x = 3 35. x=
10 6 9
23 25
36. x=–9 37. x=–4 38. x=1 39. x = 40. x=
15 3
37 3
41. x=3 42. x=2 43. x= 44. x = – 45. y=–1
7 5
2 41
46. y=
3
47. x=–
35
48. x=0 49. x = – 8 50. x=1
3

64 Ecuaciones e inecuaciones de primer grado

64-65.(2003) 64 18/11/02, 12:38 PM


CAPÍTULO 2

2.1.2 Ecuaciones de primer grado con


coeficientes fraccionarios

1. Resolver la ecuación Ejercicios


0,5x – 0,7 + 0,3x – 1,5 = 0,6x – 4 + 1,7x resueltos
Reducimos los términos semejantes
0,8x – 2,2 = 2,3x – 4
sumamos 2,2 y restamos 2,3x
0,8x – 2,2 + 2,2 – 2,3x = 2,3x – 4 + 2,2 – 2,3x
–1,5x = –1,8
dividimos por – 1,5
– 1,5x – 1,8
=
– 1,5 – 1,5

x = 1,2
Verificamos :
0,5x – 0,7 + 0,3x – 1,5 = 0,6x – 4 + 1,7x
0,5 • 1,2 – 0,7 + 0,3 • 1,2 – 1,5 = 0,6 • 1,2 – 4 + 1,7 • 1,2
0,6 – 0,7 + 0,36 – 1,5 = 0,72 – 4 + 2,04
– 1,24 ≡ 1,24
2. Resolver la ecuación
1 3 5 1
x + – x + 2 = x –3
3 4 6 5
El mínimo común múltiplo de los denominadores es 60.
Multiplicamos ambos miembros por 60 para dejar todos los coefi-
cientes enteros.
1 • 3 5 1
x 60 + • 60 – x • 60 + 2 • 60 = x • 60 – 3 • 60
3 4 6 5

20x + 45 – 50x + 120 = 12x – 180


reducimos términos semejantes
–30x + 165 = 12x –180
restamos 165 y restamos 12x en ambos miembros de la ecuación.
– 30x –12x = – 180 – 165
– 42x = –345
dividimos por – 42
–345
x=
–42

Ecuaciones e inecuaciones de primer grado 65

64-65.(2003) 65 18/11/02, 12:39 PM


Ejercicios simplificamos por –3
115
resueltos x=
14
Verificamos: 1 3 5 1
x+ – x+2 = x– 3
3 4 6 5
1 115 3 5 115 1 115
• + – • +2 = • –3
3 14 4 6 14 5 14

115 3 575 23
+ – +2 = –3
42 4 84 14
230 63 575 168 138 252
+ – + = –
84 84 84 84 84 84
114 114
– =–
84 84

3. Resolver la ecuación
3 – x 2x + 1 x+2
– =2–
5 3 10

Para eliminar los denominadores multiplicamos por el mínimo


común múltiplo de 5 , 3 y 10, que es 30

(3 – x) 6 – (2x + 1) 10 = 60 – (x + 2) 3
Resolvemos los paréntesis y reducimos términos semejantes.

18 – 6x – 20x – 10 = 60 – 3x – 6
– 26x + 8 = 54 – 3x

sumamos 3x y restamos 8

– 26x + 3x = 54 – 8
– 23x = 46

dividimos por –23


46
x=
– 23

x=–2

Verificamos: 3 – x 2x + 1 x+2
– =2–
5 3 10
3 + 2 –4 + 1 –2 + 2
– =2–
5 3 10
5 3
+ =2– 0
5 3

2∫2

66 Ecuaciones e inecuaciones de primer grado

66-67. 66 18/11/02, 12:43 PM


CAPÍTULO 2

Ejercicios

3 x
1. x=2 2. =1
4 7

2x x
3. =4 4. =0
5 8

5. x + 1 = 4 2x 3x x 1
6. – + =–
4 3 5 4 10 4

5x 2x x 9 1 1 1
7. + = +5 8. x– x+ =1
3 5 4 20 2 3 2
x 3x 1 31 7x 5x 6x 5 9
9. – + – =x– 10. – + + =–
3 2 5 30 4 3 5 6 20

11. 3 8x
– +
7x
+
3x
=
7 12. 3x – x + 1 = x + 1
4 3 5 2 12 4 2 8 3 12
1 x 2x 3x 2
13. 4x + 3x – x + 11 = 0 14. + + = –
5 4 30 10 5 7 10 175

15. 2 x – 6 x + 3 x + 5x = 1 – 77 16. 0,5x + 3,2x - 5,4x + 0,9x + 1,6 = 0


3 5 4 2 240

17. (x + 3,5) - (4,2 - 2,3x) - (4,8x + 2,1) = -1,3 18. 0,7x + 0,3x - 0,1x + 2,6x + 10,5 = 0

19. 3 x + 0,5x – 1 = 3 x – 0,3x + 0,9 20.


5 2
x + x + 0,6 = 0,5x + 1,3x
4 5 4 5

21. x – 3 – 1 = x – 1 + 1 22. 3 – x + 2 x – 1 + 1 = x + 1 + 2 x + 6 + 2
4 3 3 8 4 5 3 4 5

23. 2 x – 3 – 4 x – 1 + 1 = 3x + 2 – x – 1 + 17 24. 8 – 2 x – x + 3 – 26 = 3x + 2 – 1 – x
4 3 3 2 300 3 6 27 3 9
4 – 3x 2x – 3 1 3x 1
25. + + = – 26. x – 1 + 3x + 2 – 5x – 2 = – 11
5 15 2 10 20 3 7 3 105

27. 1 + 4x – 3 – 5x + 2 + 1 = 3x + 5 + 6x + 1 – 2 28. 4x – 2 x + 1 – 3 = 3x – x – 1 + 11
2 4 9 12 3 9 3 5 15
x x 3x 7
29. 3– + x– +7= 1– – x 4x
4 3 2 12 2+ 5–
30. 3 – 2x 2 11 3
+ + =
31. 3 x + 1 – 4 x – 5 – 13 = 5 – 2 x – 4 – x + 11 5 3 60 4
2 3 4 3 12
2x x
4– 13 +
2 4 32. 12 – x – 5 4 1
x+ 8x – + =
33. 3 5 5 6 9 18 9
– + =0
5 3 6

x 2 x 2 11
34. (x – 5) (x + 5 ) – x + = 3x – + x – 27
4 3 2 12

35. 3 x + 1 – 4 x – 5 = 2 + 5 x + x
4 2 3 3 4 3 2 4

3 2x x
– 4–
36. 5 – 3x 3 4 8 207 1 4
• – + = 8 –
3 4 30 120 2 5

Ecuaciones e inecuaciones de primer grado 67

66-67. 67 18/11/02, 12:45 PM


Ejercicios
37. 12 – 13x + 5 – 15x + 17 – 21x = – 3 1
3 4 3 156
38. 5 – 4x + 3 + 2x – 5 – 3x = 67 1
3 4 1 12
2 5 2
3x 1 4x 2 4 2
+ – 1–x x+9
39. 4 2 5 3 5 3 3
5
– 4 = 2
– 3
+ 20

40. 4 x+1 13 3 3–x 1 5 2 2x + 5


3 2
– 90 – 5 6
+ 4 = 72 – 9 4

41. 18 1 – 3x + 42 5 + 2x = 25 3x + 9 – 61 x + 1
15 3 45 3 30 8 80 3
13 x + 2 14 x – 3 27 x + 4 143
42. + – + 33 =0
7 5 4 560

43. 5x – 7 – 6x + 2 = 27x – 9 – 12x + 7 + 2 2


3 5 6 15 15

44. x+2 x+3 2x – 1 5x + 3


2
– 20
= –14
2
2x + 5 x+1 x–1 17 x – 1 x + 2 2
45. + – = 12 3
5 3 15
2
3x – 1 x + 4 2x – 5 4x + 9
46. – =
3 4 12
2 3

47.
x x+5 x–1 7x + 3 x – 1 51 6 4 – x3
4
– 3
= 2
+ 2
+ 12
3 3
x+1 x–1 x+7 27 x + 2 6x – 1
48. – = 2 + 10 +
5 5 5
2 3 2

49. 2 x –1 x–2 x+1 1 – 5x2 x3 – 1 7


3
+ 4
– 4
= 12
+ 4 – 12
2 2

50.
x –2 x–3 9 x2 + 3x – 5 81
5
+ 4
= 20
– 20

Soluciones
8
1. x = 3
2. x = 7 3. x = 10 4. x = 0 5. x = 44
3
6. x = 1 7. x = 3 8. x = 3 9. x = – 5 10. x = – 1
5 1 2 3
11. x = –
7
12. x = 2 13. x = –
3
14. x = –
5
15. x = 1
4
16. x = 2 17. x = – 1 18. x = – 3 19. x = 2 20. x = 4
21 12 1 3
21. x = –
2
22. x = – 1 23. x = –
25
24. x = 3
25. x = 2
4
26. x = 5
27. x = 0 28. x = 8 29. x = – 5 30. x = – 2
1
31. x = 17 32. x = 20 33. x = 2
34. x = 1 35. x = 3
68
12 245
36. x = – 6 37. x = 13
38. x = 12 39. x = – 3 40. x = –
316
3 87
41. x = 2 42. x = 43. x = –
97
44. x = – 7 45. x = 2
4
46. x = 1 47. x = 4 48. x = – 2 49. x = 1 50. x = 3

68 Ecuaciones e inecuaciones de primer grado

68-69.(2003) 68 18/11/02, 12:48 PM


CAPÍTULO 2

2.1.3 Ecuaciones fraccionarias


de primer grado
Se denominan ecuaciones fraccionarias aquellas que presentan
incógnita en el denominador.
Para resolverlas usamos la técnica de multiplicar por el mínimo
común múltiplo de las expresiones que son denominadores o
simplificar al máximo las fracciones.
Debemos poner especial atención en que la solución que
aparezca no vaya a indeterminar alguna expresión, es decir, no vaya
a hacer cero algún denominador. En un caso así la ecuación no
tiene solución. Ver ejemplo 5.

1 3 1 13
1.
x
+ 2 x = 3x + 12 Ejercicios
Multiplicamos por el mínimo común múltiplo m.c.m. que es 12x resueltos
12 + 18 = 4 + 13x
despejando x
13x = 26
x=2
Verificamos:

1 3 1 13
+ = +
x 2 x 3x 12

1 3 1 13
2
+ 4 = 6 + 12

5 15
4
= 12

5 5

4 4
2. Resolver la ecuación
1 3 5
– = 2
x + 3 2x – 1 2 x + 5x – 3
Primero tratamos de factorizar al máximo las expresiones del
denominador. Observamos que 2x2 + 5x – 3 = (x + 3) (2x – 1)
Así, el m.c.m. es 2x2 + 5x – 3
multiplicamos ambos miembros de la ecuación por el m.c.m.
(2x – 1) – 3 (x + 3) = 5

Ecuaciones e inecuaciones de primer grado 69

68-69.(2003) 69 18/11/02, 12:49 PM


Ejercicios resolvemos paréntesis y reducimos términos semejantes
resueltos 2x – 1 – 3x – 9 = 5
– x = 15
x = – 15
Verificamos:
1 3 5
– =
x +3 2 x – 1 2 x2 + 5 x – 3
1 3 5
– = 2
– 15 + 3 2 (– 15) – 1 2 (– 15) + 5 (– 15) – 3
1 3 5
– =
– 12 – 31 372
5 5

372 372
3. Resolver la ecuación
2x – 1 x+3 x2 – 3
– =
x+2 x+5 x2 + 7x + 10
Primero factorizamos todas las expresiones posibles de factorizar
2x – 1 x + 3 x2 – 3

x+2 x+5
= x+2 x+5
m.c.m. : (x+2)(x+5). Multiplicamos toda la ecuación por el m.c.m.

(2x – 1) (x + 5) – (x + 3) (x + 2) = x2 – 3

desarrollamos los paréntesis y reducimos términos semejantes.


(2x2 + 9x – 5) – (x2 + 5x + 6) = x2 – 3
2x2 + 9x – 5 – x2 – 5x – 6 = x2 – 3
4x = 8
x=2
Verificamos:
2x – 1 x+3 x2 – 3
– =
x+2 x+5 x2 + 7x + 10

3 5 1
– =
4 7 28
1 1

28 28
4. Resolver la ecuación
1
11x2 –
4x +2 3x + 1 5x – 1 2
12
– 2 x – 3 = 4 – 12x – 18
Vemos que 12x – 18 = 6 (2x–3) y que 4 está contenido en 12, luego,
el m.c.m. de los denominadores es 12 (2x – 3).
Multiplicamos por este m.c.m.
(4x + 2) (2x – 3) – (3x + 1) • 12 = (5x – 1) (2x – 3) • 3 – (11x2 – 1) • 2
2

70 Ecuaciones e inecuaciones de primer grado

70-71. 70 18/11/02, 12:52 PM


CAPÍTULO 2

Resolvemos los paréntesis y reducimos términos semejantes.


(8x2 – 8x – 6) – (36x + 12) = (30x2 – 51x + 9) – 22x2 + 1
8x2 – 8x – 6 – 36x – 12 = 30x2 – 51x + 9 – 22x2 + 1
7x = 28
x=4
Verificamos:
1
11x2 –
4x + 2 3x + 1 5x – 1 2
12
– 2x – 3
= 4

12x – 18

1
176 –
18 13 19 2
12
– 5 = 4 – 30
3 13 19 176 1
– = 4 – 30 + 60
2 5
11 11
– ∫–
10 10
5. Resolver la ecuación
2 2–x 3–x 4
+ +3= 2
x–1 x+1 x –1
Para eliminar los denominadores multiplicamos por el m. c. m / x2 – 1
2 (2 – x) (x + 1) + (3 – x) (x – 1) + 3 (x2 – 1) = 4
4x + 4 – 2x2 – 2x + 3x – 3 – x2 + x + 3x2 – 3 = 4
reducimos términos semejantes
6x = 6
x=1
pero ocurre que si x = 1 se indeterminan dos de los términos
de la ecuación: 2 2 – x y 4 ; por lo tanto, la ecuación plan-
x–1 x2 – 1
teada no tiene solución.

Ejercicios
1 3 4 5 3
1. 1 = 3 1 6. 2 x – 5 + 4 3 – 2 x = 0 11. – =
x 3x – 1 2 x + 3 6 x2 + 7x – 3
3 1 –3 4 2 1 x+4
2. = 1 7. =
5x – 1 6 – 7x
12.
3 x+4
+ 4 x–5 = 2
x 2 6 x – 6 x – 120
4
3. –3=0 1 8. 1 + 3 = 11 13. 1

4
=
3x – 1
5x 2x 2 5x 3x – 2 5x + 1 15 x2 – 7x – 2
x
1 3 6 3 3 4 6
4. =3 1 9. – – =0 14. – =
2x + 1 4x 5x x2 x + 2 x – 2 x2 – 4
2 3 2
5. – =0 10. x + 3 = 2 x 15. 2 2 – x + 3 – x + 3 = 1
x–1 x+3 2x – 1 x–1 x+1 x2 – 1

Ecuaciones e inecuaciones de primer grado 71

70-71. 71 18/11/02, 12:53 PM


Ejercicios
16. 6 3 2 5
+ 2
= – 2
3x – 5 x – 2x + 1 x–1 3x – 8 x + 5
17. 2 x + 3 6x+4
= 15x + 2
5x – 1
18. 3 – x + 2
=
2 x–2
–3
2
x+1 x –1 x–1

19. 4x – 7 = x – 16
12x + 3 3x + 5

20. 3 – 16 + 2 2 – x = x – 3
2
x –1 x–1 x+1

21. 4x – 5 – 8 x – 3 = 0
3x + 2 6x + 5

22. 3 – 5x – 5 x + 2 = 0
1 – 3x 3x + 20
23. x3 – 125 x–1
2
– x+3 = 1 + x
x – 2 x – 15
2
24. x + 2 x + 1 + x – 1 = x3 3
2
+ 2
x+1 x+3 x – 2 x – 15 2 x – 4x – 30
2
25. x x+1 5x + x + 4
+
3x + 2 2 x – 1
= 2
6x + x – 2
26. 2 3 1 5 7
+ – + =–
3x – 3 4x – 4 2x – 2 6x – 6 16
27. 2 2 x – 13
– =
2
x+2 3x – 5 3x + x – 10
28. x 1 4x2 + 3x – 1
+ =
2x – 6 6x + 3 8 x2 – 20x – 12 3
3 2 x+1
29. 6x x+2 3x – 1 2
3 2
– 2
= +
4x – 12x – x + 3 4x – 1 2 x2 – 7x + 3 2 x2 – 5x – 3
2
2+
2
1 1 1 2+

x 2 x 33. x 36.
2 2–x 3–x 8
=3 + x+1 + 3 =
30. = 2–
2 x–1 1 – x2
3 1 2

x 3 5 + 3x 2–
2
2 2 2
x 37. x + 1 – 3 = x + 3 – 11x
x+2 2 x–2 2
2x – 8
1 1 1 1 1
– –
2 x x 3 1
31. = 1– 38. 5x – 1 12x2 3 – 2x
1 3 2 34. x 8 – = 4x + 1

1 1
= – 15 2 x – 6 4x2 – 11x – 3
3 x 3x + 5 +
4 1 12x2 – 17 5x – 1 2x – 3
1 1+ 39. = 2 x – 6 + 4x + 1
1+ x 2
x 4x – 11x – 3
32.
1
=x 2 1 1
1– – 1–
1 3x 6x x 40. x 2x 5x2 + 125
1+ 35.
1 4
= 1 2x + 3
+ –
x – 5 2 x2 – 7x – 15
=0
x
– 1+
2x 6x x

72 Ecuaciones e inecuaciones de primer grado

72-73. 72 18/11/02, 12:59 PM


CAPITULO 2
CAPÍTULO

Soluciones
1 4 1
1. x = 3
2. x = 6 3. x = 15
4. x = – 3
5. x = 9
20 3
6. x = – 9 7. x = 14 8. x = 10 9. x = – 3
10. x = 5

11. x = 2 12. x = 4 13. x = 1 14. x = – 20 15. x = 1


2
4 2 13
16. x = 3
17. x = – 7
18. x = 0 19. x = – 188 20. x = 3
19 25 No tiene 1 2
21. x = – 17
22. x = 33
23. solución 24. x = – 25. x =
2 3
1 Ecuación 13
26. x = – 3 27. x = – 28. x = 9 29. de 2º grado 30. x = – 2
3 16
45 1 7 1
31. x = – 16 32. x = – 33. x = – 2 34. x = – 35. x = –
2 23 2
36. No tiene 37. x = – 1 38. x = 1 39. No tiene 40. x = 125
solución solución

2.1.4 Ecuaciones literales


de primer grado
Se denominan ecuaciones literales de primer grado aquellas que
presentan expresiones literales y/o numéricas además de la incógnita.
Generalmente para la incógnita se utilizan las letras x, y o z y para los
coeficientes u otros términos, las letras a, b, c, m, n, s, t . . .
Resolver una ecuación literal es despejar una de las letras que en
ella intervienen, generalmente x, y o z en función de las otras y/o los
números que aparezcan. Para ello se usan los mismos procedimientos
que se utilizan para resolver ecuaciones numéricas.

1. Resolver la ecuación Ejercicios


a (x + 1) = a (a + 1) – x resueltos
Resolvemos los paréntesis y reducimos términos semejantes
ax + a = a2 + a – x
trasponemos términos dejando todos los que contengan x en el
primer miembro y los que no la contienen en el segundo.
ax + x = a2 + a – a
ax + x = a2
factorizamos por x y dividimos toda la expresión por (a + 1)
x (a + 1) = a2
a2
x=
a+1

Ecuaciones e inecuaciones de primer grado 73

72-73. 73 18/11/02, 1:01 PM


Ejercicios Verificamos:
resueltos a (x + 1) = a ( a + 1) – x
a2 a2
a a + 1 + 1 = a a+1 – a + 1
a3 a2
+ a = a2 + a – /• a + 1
a+1 a+1
a + a (a + 1) = (a + a) (a + 1) – a2
3 2
a3 + a2 + a = a3 + a2 + a2 + a – a2
a3 + a2 + a ≡ a3 + a2 + a
2. Resolver la ecuación
x + b (bx + 1) = (1 + b) – b (2x – 1)
Resolvemos los paréntesis y reducimos términos semejantes.
x + b2x + b = 1 + b – 2bx + b
x + b2x + b = 1 + 2b – 2bx
trasponemos los términos agrupando los que contienen x en el
primer miembro y los que no la contienen en el segundo.
x + b2x + 2bx = 1 + 2b – b
factorizamos ambos miembros
x (1 + 2b + b2) = 1 + b
x (1 + b)2 = 1 + b
dividimos ambos miembros por (1 + b)2
1+b
x= 2
1+b
1
x = 1+b
Verificamos:
x + b (bx + 1) = (1 + b) – b (2x – 1)
1 b 2
1+b
+ b 1+b + 1 = 1+b –b –1
1+b
1 b2 2b
+
1+b 1+b
+ b = 1 + b – 1+b + b
1 + b2 1 + b 2 – 2b
=
1+b 1+b
1 + b2 1 + b2

1+b 1+b

3. Resolver la ecuación
1+x 1+x 1+a 1–b
b
+ a = b + a
Eliminamos los denominadores multiplicando la ecuación com-
pleta por el m.c.m. = ab
a (1 + x) + b (1 + x) = a (1 + a) + b (1 – b)
resolvemos paréntesis y trasponemos términos
a + ax + b + bx = a + a2 + b – b2
ax + bx = a2 – b2

74 Ecuaciones e inecuaciones de primer grado

74-75. 74 18/11/02, 1:16 PM


CAPÍTULO 2

factorizamos por x y dividimos por a + b


x (a + b) = a2 – b2
a+b a–b
x =
a+b
x = a–b
Verificamos:
1+x 1+x 1+a 1–b
b
+ a = b + a
1+a–b 1+a–b 1+a 1–b
b
+ a
= b + a
2 2
a + a – ab + b + ab – b a + a2 + b – b2
ab
= ab
a + a2 + b – b2 a + a2 + b – b2

ab ab
4. Resolver la ecuación
m+x n+x
n
– m =0

Para eliminar los denominadores multiplicamos toda la ecuación


por el m.c.m. que es m n
m (m + x) – n (n + x) = 0
m2 + mx – n2 – nx = 0
asociamos los términos agrupando los que contienen a x
mx – nx = n2 – m2
factorizamos por x en el primer miembro y factorizamos el
segundo miembro
x (m – n) = (n – m) (n + m)
x (m – n) = – (m – n) (n + m)
– m–n n+m
x =
m–n
x = –n–m
Verificamos:
m+x n+x
n
– m = 0

m+ –n–m n+ –n–m
– = 0
n m
–n –m
– m = 0
n
(–1) – (–1) = 0
–1 + 1 = 0
0 ≡ 0
5. Despejar h en la ecuación
1
A= (b1 + b2) • h
2
En este tipo de ecuaciones literales que generalmente se trata
de fórmulas que se usan en algún cálculo de geometría, física
o química, se especifica cuál de las variables se debe despejar
en función de las otras.

Ecuaciones e inecuaciones de primer grado 75

74-75. 75 18/11/02, 1:16 PM


Ejercicios Aquí debemos despejar h.
resueltos La ecuación o fórmula puede ser escrita así:

multiplicamos por 2
2 A = (b1 + b2) • h
dividimos por (b1 + b2)
2A
b1 +b2
=h

con esto quedó despejada h, tal como se pedía.

Ejercicios

I.
8. ax – 1 = bx + 1
x 9. (a – 1) x + (b – 1) x = (x – 1) a + (x – 1) b
1. =b
a
2. ax = ab 10. (1 – a)2 – (a + x)2 = (1 – b)2 – (b + x)2
3. ax – 1 = b 11. a2 (x – 2) – b2 (x – 2) = b (a2 – b2)
4. 2ax – a = a + 2x 12. m2x = n (1 + n) + x (m2 – n2)
5. abx = a – x (a2 + b2) – b (ax – 1) 13. (m – 4) x + (m – 5) x = (x – 5) m + (x – 4) m
6. ax – 3 = bx – 5 14. (a – 4x) (2a – x) – (2x – a)2 = (a + x) (a – 1)
7. a2b + bx = ab2 + ax 15. (x + m)2 – (x + n)2 = (m – n)2

16. (a + b + c) x – (a + b + x) c = (a + x + c) b – (x + b + c) a
17. (x + 1) a – (x + 1) b – (x + 1) c = (a + b + c) x + a
18. 2 (x – a) + 3 (x – b) – 4 (x – c) = 3a + 2b – c
19. (x + a) (x – b) – (x – a) (x + b) = a2 – b2
20. (a + b + c) (x – a) + (a + b + c) (x – b) + (a + b + c) (x – c) = 0

x
21. x – b = 25. 3 – a + 1 = 0
a x 2 a

26. x – a – x – b = 2ab – 2b2


22. x + x = 1 x–b x–a x2 – ax – bx + ab
a b ab 2
9a
– 6a + b
x x b
23. ax – b x = 1 + 1 27. – =
b a a b b 3a 6a

24. 1 – a = 2 28. 3 a – x – 2 b – x = 2b – 6 a
2 2
x x a b a ab

76 Ecuaciones e inecuaciones de primer grado

76-77. 76 18/11/02, 1:23 PM


CAPÍTULO 2

29. a – x b–x c–x 1 33. 3 x + 2 b – 3 = 4b2


a
+ b + c = 3 + ab c x – 3b x2 – 9 b2

x–a x–b
30. + =2 34. 3x + 5a 6x – 2b
= 4x – 2a
x+b x+a 2x + b

bx a x 2x 1
31. =b x–a +a 35. + – =0
a 2m 3n mn

32. bx a
– b x–a =b 36. 3

2
=
3b
a 2x – b 2x + b 4x2 – b2

II. En los ejercicios siguientes se trata de despejar la variable indicada.


2
1. Si d = v • t , despejar v. 18. Si T – m v = 0, despejar m.
R
dv dm dv
2. Si F = m • a , despejar m. 19. Si F = m + dt , despejar dt .
dt
v2
3. Si P = 2 π r , despejar r. 20. Si x = , despejar g.
2mg
d 4
4. Si v = , despejar t. 21. Si F = , despejar t.
t 1+t 2
5. Si S = R • ϕ, despejar R. 22. Si L = m • r2 • w, despejar r.

w 1
6. Si P = , despejar w. 23. Si d = 2
a t2, despejar a.
t
1 24. Si I = M R2, despejar R.
7. Si Ec = m v2, despejar m.
2
m
1
8. Si Ec = m v2, despejar v. 25. Si T = 2k k , despejar m.
2
9. Si Ep = m • g • h, despejar m. 26. Si L = m k r , despejar r.

2
10. Si m2 = a1 , despejar a1. 27. Si Z = x 1 + R2 , despejar x .
1
m1 a2
11. Si F = m • g, despejar g. 28. Si Ec = h (ƒ – ƒo), despejar ƒo.

12. Si T = P + m • a, despejar m. 29. Si 1 – 1 = h • (1 – cos a), despejar f.


f f’ mcc2

13. Si Fr = µ • m • a , despejar a. 30. Si P • V = n • R • T, despejar T.

14. Si F + Fr – mg sen α = m a, despejar m. 31. Si Q = C (T2 – T1), despejar T2.


Q2
15. Si N – mg cos α – F sen α = 0, despejar F. 32. Si E1 = , despejar Q2.
Q1 – Q2
33. Si Q1 – Q2 = 0, despejar T .
2
16. Si Fc = m v , despejar R. 1
R T1 T2
2 4p
17. Si v = µ g, despejar v. 34. Si T2 = g • l , despejar l.
R

Ecuaciones e inecuaciones de primer grado 77

76-77. 77 18/11/02, 1:25 PM


Ejercicios
x1 + x2 + .... + xn
35. Si X = n , despejar n. 38. Si S = So + gt , despejar t.
x–m
36. Si Z = , despejar s. 39. Si 1 = 1 _ 1 , despejar s.
s/ n f s´ s
40. Si 1 = 1 + 1 + 1 , despejar R.
37. Si Z = t –s x , despejar t. R R1 R 2 R 3

Soluciones
b+1 a
I. 1. x = a b 2. x = b 3. x = a 4. x =
a–1
1 2 2
5. x = 6. x = 7. x = a b 8. x =
a+b b–a a–b
a+b 1+n
9. x = 10. x = – 1 11. x = b + 2 12. x =
2 n
a bc
13. x = m 14. x = 15. x = – n 16. x = a
6a–1
a+b
17. x = – 1 18. x = 5(a + b + c) 19. x = 20. x = a+b+c
2 2 3
2
ab 1 1 a–a
21. 22. x = 23. x = 24. x =
a–1 a+b a–b 2

6a a–b 3a – b
25. 2
26. x = 27. x = 28. x = 3a + 2b
a –2 2 2
2
-1 a+b a
29. 30. x = – 31. x = 32. x = a
ab + bc + ac 2 a+b
2 2
29 5a – b 6
33. x = – b 34. x = 35. x = 36. x = – b
11 7a – b 3n + 4m
II. d
3. r = P
d F
1. v = 2. m = 4. t =
t a 2p v
s 2 Ec 2 Ec
5. R = j 6. w = P • t 7. m = 2
8. v =
v m
Ep a 2 m2 F T–P
9. m = 10. a1= 11. g = m
12. m =
g•h m1 a
N – m g cos a 2
13. a = Fr 14. m = a +Fg+sen
Fr
15. F = 16. R =
mv
mm a sen a Fc
dm
TR F– v2
17. v = m g R 18. m = 19. dv = dt 20. g =
v
2
m 2 mx
dt
2 L 2d I
21. t = –1 22. r = 23. a = 24. R = M
F mw t
2

2 2
T L 2 E
25. m = 26. r = 27. x1 = Z –R2 28. fo = f – c
4k mk h
mc c2 f ’ PV Q E1 Q1
29. f = ’ 30. T = 31. T2 = + T1 32. Q2 =
h f 1 – cos a + mc c2 nR C 1 + E1
Q 1 T2 g T2 x1 + x2 + ... + xn n x–m
33. T1 = 34. l = 35. n = 36. s =
Q2 4p X z

S – So R1 R2 R3
37. t = zs + x 38. t = 39. s = s f 40. R =
g ’ R2 R3 + R1 R3 + R1 R2
f–s

78 Ecuaciones e inecuaciones de primer grado

78-79.(2003) 78 20/11/02, 9:49 AM


CAPÍTULO 2

2.1.5 Ecuaciones con


valor absoluto
Definición: x = x si x ≥ 0
– x si x < 0

1. | x + 2 | = 5 Ejercicios
Solución resueltos
i i) Si x + 2 ≥ 0 ⇒ | x + 2 | = x + 2
x+ 2 = 5 ⇒ x = 3
ii) Si x + 2 < 0 ⇒ | x + 2 | = – (x + 2)
–x – 2 = 5 ⇒ x = – 7
2. | 7x + 3 | – 2 = 1 – x
Solución
ii) Si 7x + 3 ≥ 0 ⇒ | 7x + 3 | = 7x + 3
7x + 3 – 2 = 1 – x ⇒ 8x = 0 ⇒ x = 0
ii) Si 7x + 3 < 0 ⇒ | 7x + 3 | = – ( 7x + 3 )
– 7x – 3 – 2 = 1 – x ⇒ – 6x = 6 ⇒ x = – 1
3. – 3 = 8x – | 9x + 4 |
Solución
i)) Si 9x + 4 ≥ 0 entonces | 9x + 4 | = 9x + 4
Así, – 3 = 8x – | 9x + 4 | ⇒ – 3 = 8x – ( 9x + 4 )
⇒ x = –1 contradicción
pues 9x + 4 ≥ 0; por lo tanto x = – 1 no es solución
ii) Si 9x + 4 < 0 entonces | 9x + 4 | = – ( 9x + 4 )
Así, – 3 = 8x – | 9x + 4 | ⇒ – 3 = 8x + 9x + 4
7
⇒ x = – 17 contradicción
7
pues 9x + 4 < 0; por lo tanto x = – no es solución
17

La ecuación no tiene solución

Ejercicios
1. | 2x – 4 | = 3 4. – 3 = 7x – | 10x + 4 |
2. 8 (– 5 + | 4x + 9 | ) = – 32 5. – 13 + | – 6x + 7 | = – 13
3. 15 = – 15 (12 – | – 2x + 9 | ) 6. – 4 = 8 – | – 6x + 7 |

Ecuaciones e inecuaciones de primer grado 79

78-79.(2003) 79 20/11/02, 9:50 AM


Ejercicios 14. – 8x = 2 – | 9x + 4 |
7. | – 3x – 3 | = 16 15. | 19x + 3 – 8x | – 4 = 7x
8. 4 + | 6x + 7 | + 9x = 11x + 10 16. | 22x + 6 – 10x | – 1 = 8x
9. – 1 = – 1 – | – 6x – 6 | 17. 9 = 9 – | 6x + 3 |
10. | 7x + 2 | – 1 = 2x 18. 11 (– 6 + | – 7x – 8 | ) = 44
11. – 3 (– 4 + |– 4x + 1 | ) = 12 19. 2 + | 6x + 8 | = 14
12. 10 (2 + |– 2x –5 | ) = 80 20. | 21x + 6 – 7x | – 5 = 10x
13. – 55 = 11 (4 – |– 3x + 8 | ) 21. – 6 = 2x – | 8x + 7 |

Soluciones
1 7 5
1. , 2. – 2, – 3. – 2, 11
2 2 2
1 7 7 19 5
4. – , – 5. 6. ,–
3 17 6 6 6
13 –19
7. , 8. – 1 , – 13 9. – 1
3 3 4 8
1 1 1
11 1
10. – ,– 11. 12. – ,
5 3 4 2 2
1 17 1 –7
13. – 3 , 3 14. No tiene solución 15. ,
4 17
1 18 2
16. No tiene solución 17. – 18. – ,
2 7 7
2 10 – 1 – 11 1 13
19. ,– 20. , 21. – , –
3 3 4 24 6 10

2.2 Problemas

Hay un sinnúmero de problemas que pueden resolverse planteando,


con sus datos, ecuaciones de primer grado.
Para resolver un problema recomendamos los siguientes pasos:
1. Leer atenta y comprensivamente el enunciado del problema.
2. Identificar la incógnita y los datos que se utilizarán en la so-
lución.
3. Relacionar los datos con la incógnita planteando una ecuación.
4. Resolver la ecuación.
5. Analizar la solución de la ecuación cuidando que tenga rela-
ción con el enunciado del problema.
6. Dar la respuesta.

80 Ecuaciones e inecuaciones de primer grado

80-81.(2003) 80 18/11/02, 1:40 PM


CAPITULO 2
CAPÍTULO

1. Escribir en forma de expresión algebraica el siguiente enunciado. Ejercicios


a) La mitad de los asistentes a la reunión, más uno. resueltos
Sea x el número de asistentes a la reunión.
x
Entonces la mitad más uno es: +1
2
b) El doble de un número menos el 30% de él.
Sea n el número.
30
2n – n
100
c) Si la suma de las edades de Juan y María es 63 años, escribir
una expresión para la edad de cada uno:
Sea x la edad de Juan, entonces 63 – x es la edad de María.
d) La suma de dos números multiplicada por su diferencia.
Sean x e y los números, entonces el producto de su suma
por su diferencia se expresa:
(x + y) (x – y)
e) Sumar el cuadrado de un número con el doble de su cubo.
Si a es el número, la situación descrita se escribe así:
a2 + 2a3
f) Sumar el cuadrado de un número con su duplo al cubo.
Si a es el número, la situación ahora se escribe así:
a2 + (2a)3
g) Escribir tres números enteros consecutivos.
Si n es un número entero, la situación descrita se puede escribir:
n, n + 1, n + 2
n – 1, n, n + 1
h) Un viajero debe recorrer m km. y ha recorrido n km. ¿Cuánto
le falta por recorrer?
Le falta (m – n) km por recorrer.
i) Pedro recibe por un trabajo $ a y de mesada $ b. Con todo el
dinero alcanza a comprar (m + 2) revistas. Si todas las revistas
tienen el mismo valor, ¿cuánto le costó cada una?
Cada revista le costó:
a+b
$
m+2

2. En un gallinero hay 5 pavos más que gallinas y 3 patos más


que pavos. Si en total hay 49 aves, ¿cuántas gallinas, pavos
y patos hay?
Sea x el número de gallinas, entonces:
x + 5 será el número de pavos y
x + 5 + 3 será el número de patos

Ecuaciones e inecuaciones de primer grado 81

80-81.(2003) 81 18/11/02, 1:40 PM


Ejercicios Como en total hay 49 aves podemos decir que:
resueltos x + x + 5 + x + 5 + 3 = 49
3x + 13 = 49
3x = 36
x = 12
Respuesta: Hay 12 gallinas, 17 pavos y 20 patos.
3. La suma de tres números pares consecutivos es 102. Hallar los
tres números.
La expresión 2n representa un número par, el siguiente es 2n + 2
y el que sigue es 2n + 4.
Como los tres números suman 102 podemos decir que:
2n + 2n + 2 + 2n + 4 = 102
6n + 6 = 102
6n = 96
96
n=
6
n = 16
Si n = 16, entonces
2n = 32
2n + 2 = 34
2n + 4 = 36
Los números pedidos son 32, 34 y 36.
4. El perímetro de un rectángulo es de 40 m. Si el largo se aumenta
2m y el ancho se disminuye 2 m, su área disminuye en 12m2.
Calcular sus dimensiones.
El perímetro es 40 m, luego el semiperímetro es 20 m.
Si llamamos x al largo, entonces 20 – x representa el ancho.
Área del rectángulo x (2 – x)
El nuevo rectángulo tiene:
largo x + 2, ancho 20 – x – 2 y área (x + 2) (20 – x – 2)
Pero el área disminuye en 12 m2, luego
x (20 – x) – (x + 2) (20 – x – 2) = 12
resolviendo la ecuación
20x – x2 – (x + 2) (18 – x) = 12
20x – x2 – 18x + x2 – 36 + 2x = 12
4x = 48
x = 12
El largo es 12 m y el ancho es 8 m.
5. La edad de Pedro es el doble de la edad de María. Si en cinco
años más la suma de sus edades será 43 años, ¿qué edad tienen
actualmente?

82 Ecuaciones e inecuaciones de primer grado

82-83. 82 18/11/02, 1:42 PM


CAPÍTULO 2

En este tipo de problemas es adecuado hacer un cuadro en el tiempo.


actual 5 años más
Edad de María x x+5
Edad de Pedro 2x 2x + 5
En cinco años más sus edades sumarán 43 años:
(x + 5) + (2x + 5) = 43
resolviendo la ecuación
3x + 10 = 43
3x = 33
x = 11
Actualmente María tiene 11 años y Pedro tiene 22 años.

6. Se tiene una herencia a repartir entre sus herederos y la


1
instrucción es que el mayor debe recibir de ella, el segundo
4
3 1
de ella y el tercero de lo que queda, entregando la otra
8 2
mitad a una institución de beneficencia. Si esta institución
recibe $ 815.625, hallar el monto de la herencia y cuánto
recibe cada heredero.
Sea x el monto de la herencia, entonces:
1
El mayor recibe 4
x
El segundo recibe 3 x, con esto ya se han repartido
8
1 3 5 3
x + x = x, luego quedan x
4 8 8 8

El tercero recibe
La institución de beneficencia recibe también x, lo que
equivale a $ 815. 625.
x = 815.625
3
x = 815. 625
16
x = $ 4.350.000
que es el monto de la herencia, así:
1
El mayor recibe x = $ 1.087.500
4
El segundo recibe 3 x = $ 1.631.250
8
El tercero recibe x = $ 815.625
7. Un estanque se llena con la llave A en 4 horas y con la
llave B en 8 horas. ¿Cuánto tardará en llenarse si se abren
simultáneamente las dos llaves?
Si con la llave A tarda 4 horas en llenarse, en 1 hora se habrá
1
llenado sólo de estanque.
4

Ecuaciones e inecuaciones de primer grado 83

82-83. 83 18/11/02, 1:43 PM


Si con la llave B tarda 8 horas en llenarse, en 1 hora se habrá
Ejercicios 1
llenado sólo de estanque.
resueltos 8
Llamemos x al tiempo que tarda en llenarse con ambas llaves
1
abiertas, entonces en 1 hora se habrá llenado sólo de
x
estanque.

Podemos entonces plantear la ecuación 1 + 1 = 1


4 8 x
Resolviendo:
2x + x = 8
3x = 8

x= 8
3
2
Demora 2 3 horas en llenarse el estanque con las dos llaves
abiertas, es decir, 2 horas 40 min.
8. Determinar a qué hora, entre las 5 y las 6 horas, los punteros de
un reloj forman un ángulo de 90º.
o
360 o
En cada minuto, el puntero avanza =6
60

En cada minuto, el horario avanza


A las 5 horas el minutero está en las 12 y el horario en las 5 y el
ángulo que forman es de 150º (arco AB).
A
12

C
9 O. 3

5
6
DB
Se pide que el arco CD corresponda a un ángulo de 90º.
Si llamamos O al centro del reloj, por cada minuto OA avanza
6º y OB avanza 0,5º; por lo tanto, por cada minuto el ángulo
AOB se achica 5,5º.
Sea x la cantidad de minutos que deben transcurrir para
que el arco AB corresponda a un ángulo de 90º. Entonces
podemos decir:
150º – 5,5ºx = 90º
60 = 5,5x
10
x = 10
11
Luego, el ángulo formado por los punteros del reloj es de 90º
10
a las 5 horas 10 min.
11
El ángulo de 90º ocurrirá nuevamente después que el minutero
haya pasado sobre el horario.
Veamos primero a qué hora el ángulo se hará 0º.

84 Ecuaciones e inecuaciones de primer grado

84-85.(2003) 84 18/11/02, 1:44 PM


CAPÍTULO 2

A
12

150º – 5,5ºx = 0
9 O . 3
C 150 = 5,5x
3
5 x = 27
6 B 11
D
3
Esto es a las 5 horas 27 min.
11
A partir de este momento el ángulo se va agrandando a razón
90 4
de 5,5º por minuto; luego en = 16 minutos más, el ángulo
5,5 11
entre los punteros volverá a ser de 90º, es decir, a las 5 horas
7
43 min.
11

Ejercicios
7. En el triángulo ABC los lados
1
AB = 3 BC y BC = 2 AC. Si su perímetro
1. Un número multiplicado por 5 sumado es 84 m. ¿Cuánto mide cada lado?
con el mismo número multiplicado por
seis da 55. ¿Cuál es el número?
8. Si el lado de un cuadrado se duplica,
su perímetro aumenta 40 m. Calcular la
2. ¿Qué número se debe restar de p+2 medida del lado del cuadrado.
para obtener 5?

9. Las dimensiones de un rectángulo están


3. El doble de un número aumentado en en la razón 3:5 y su perímetro es 140 m.
12 es igual a su triple disminuido en 5. Calcular el largo y el ancho.
¿Cuál es el número?

10. Una figura cerrada de seis lados


4. Tres números impares consecutivos tiene cinco de sus lados en la razón
suman 81. ¿Cuáles son los números? 1:2 :3 :6:4 y sus ángulos son rectos,
excepto el formado por los lados que
están en la razón 2: 3, que vale 270°. Si
5. El doble de un número más el triple de su perímetro es 144 m, hallar el sexto
su sucesor, más el doble del sucesor de término de la proporción y la medida
éste es 147. Hallar el número. de cada uno de los lados.

6. La diferencia entre los cuadrados de dos 11. Si el lado de un cuadrado es aumentado


números consecutivos es 103. ¿Cuáles en 8 unidades, su perímetro se triplica.
son los números? ¿Cuánto mide el lado?

Ecuaciones e inecuaciones de primer grado 85

84-85.(2003) 85 18/11/02, 1:45 PM


Ejercicios
12. Con una cuerda de 70 cm de longitud 20. Repartir $ 952 entre tres personas
se pide formar un triángulo cuyos lados de modo que reciban cantidades
estén en la razón 3 : 4 : 7. ¿Cuánto proporcionales a 4, 6 y 18.
medirá cada lado? ¿Se podrá construir?
Y si la longitud de la cuerda es 84 cm,
¿se podrá construir? 21. Se compran 25 lápices, 32 cuadernos
y 24 gomas de borrar y se cancela por
ello $16.990. Si cada cuaderno cuesta
13. Un padre tiene 20 años más que su el triple de cada goma, más $ 20 y
hijo. Dentro de 12 años, el padre tendrá cada lápiz cuesta el doble de cada
el doble de la edad del hijo. ¿Cuántos goma, más $ 8, ¿cuánto cuesta cada
años tiene cada uno actualmente? material?

14. Las edades de un matrimonio suman 62


22. Hernán tiene el doble de dinero que
años. Si se casaron hace 10 años y
Gladys y el triple que María. Si Hernán
la edad de la novia era 3 de la edad
4 regalara $14 a Gladys y $35 a María,
del novio, ¿qué edad tienen actual-
los tres quedarían con igual cantidad.
mente?
¿Cuánto dinero tiene cada uno?
15. La edad de Pedro excede a la de su
amigo Santiago en 4 años y a la de su
23. Entre Gastón y Javier juntan $ 2.340.
amigo Juan en 2 años. Hace 6 años la
Gastón reparte su capital entre cuatro
razón entre sus edades era 2:3:4. ¿Qué
amigos y Javier lo hace entre tres. Los
edad tienen actualmente?
amigos de Javier reciben cada uno
$10 más que los amigos de Gastón.
16. La edad de María es el triple de la de Hallar el dinero que tenían Gastón
Ester y excede en 5 años a la edad y Javier.
de Isabel. Si las edades de Ester e
Isabel suman 23 años, hallar la edad
de cada una. 24. Una persona puede pintar una muralla
en 5 horas, otra lo hace en 6 horas y una
tercera persona tarda 12 horas en pintar
17. Guido tiene la cuarta parte de la edad la misma muralla. ¿Cuánto tardarían si
de su padre Andrés y el triple de la la pintaran entre las tres?
edad de su hermano David. ¿Qué
edad tiene cada uno, si sus edades
suman 48 años? 25. Una llave puede llenar un estanque
en 6 horas. Otra puede hacerlo en
7 horas. Estando lleno, el desagüe
18. Hace 6 años un padre tenía el cuádruplo puede vaciarlo en 10 horas. ¿En cuánto
de la edad de su hijo. En 10 años más tiempo se llenará el estanque si estando
tendrá sólo el doble. Hallar la edad vacío y con el desagüe abierto, se
actual de padre e hijo. abren las dos llaves?

19. Un padre tiene 52 años y su hijo 16. 26. ¿A qué hora entre las 11 y las 12 los
¿Hace cuántos años el hijo tenía la punteros del reloj formarán un ángulo
séptima parte de la edad del padre? de 90°? ¿A qué hora coincidirán?

86 Ecuaciones e inecuaciones de primer grado

86-87. 86 18/11/02, 1:46 PM


CAPÍTULO 2

27. ¿A qué hora entre las 3 y las 4 33.Una prueba de matemática contiene
los punteros de un reloj formarían ítemes de álgebra y geometría y está
un ángulo extendido? ¿A qué hora estructurada con un 80% de preguntas
coincidirán? de selección múltiple. De ellas, el 50%
son de álgebra. De las preguntas con
respuestas abiertas, el 75% son de
28. El numerador de una fracción excede álgebra. El número de preguntas de
en dos unidades al denominador. Si al geometría que no son de selección
numerador se le suma 3, la fracción múltiple es 2.
4
queda equivalente a . Hallar la frac-
3
ción. a) Calcule el número de preguntas de
geometría que tiene la prueba.
29. La cifra de las decenas excede en b) Calcule el número total de preguntas
5 a la cifra de las unidades de un de la prueba.
número de dos cifras. Si el número se
divide por la suma de sus dígitos, da c) ¿Qué fracción de las preguntas de
8. Hallar el número. geometría son de selección múlti-
ple?
30. Descomponer el número 564 en tres d) Si las preguntas de selección múltiple
sumandos que sean inversamente valen 0,5 puntos; de las preguntas
proporcionales a 3, 4 y 5. abiertas, las de geometría valen 0,8
puntos y las de álgebra 0,4 puntos,
calcule el puntaje máximo de la
31. Considere el conjunto de alumnos
prueba.
y alumnas de su colegio desde 1°
Básico hasta 4° Medio.
e) En una escala de 1 a 7, ¿cuál sería la
Identifique: nota que proporcionalmente le corres-
a) El tamaño de la población pondería a un alumno que contesta
b) 3 variables cualitativas para estudio correctamente la mitad de las pregun-
tas de selección múltiple y todas las
c) 3 variables cuantitativas para estudio
preguntas abiertas?
d) Los rangos de cada una de las
variables identificadas en b y c. 34. Una persona pinta una piscina en r
horas y otra lo hace en s horas. ¿ Cuánto
32. En las siguientes situaciones: se demoran en pintar la piscina ambas
personas trabajando juntas?
a) Identifique las variables relevantes
b) Elija parejas de variables que se 35. Para cortar el césped de una cancha de
relacionen en forma inversa y otras que fútbol una persona tarda 4 horas y otra
se relacionen en forma directa. lo hace en 6 horas. Calcule cuánto se
c) Identifique el campo de variación de demorarían si trabajaran juntas.
cada una de las variables elegidas.
• El curso quiere organizar una despe- 36. Si un cuadrado de área a aumenta su
dida a un compañero que se va de lado al doble, halle en qué proporción
intercambio. aumenta su área.

• El curso decide realizar un regalo 37. Haga un análisis de qué podría ocurrir
a la profesora de ciencias. con el perímetro de un rectángulo si su
• El orientador avisa que viene un área se duplica. ¿El perímetro del nuevo
curso de visita para intercambiar rectángulo, depende de la forma como
experiencias en la clase de álgebra. se duplica su área?

Ecuaciones e inecuaciones de primer grado 87

86-87. 87 18/11/02, 1:46 PM


Ejercicios
Observación: Considere las siguientes ¿Habrá otras posibilidades?
alternativas:
38. En un cubo, la longitud de su arista se
a) x x triplica. ¿Cómo varía su volumen?

y y y
39. Se tiene un triángulo de base b y altura
x
→ x h. Si se disminuye su base a la mitad y
b) se aumenta su altura al doble. ¿Qué
y x ocurre con su área?
y
y y
40. En un paralelogramo ABCD, P es un
c) x → x punto cualquiera de la diagonal AC. Por
P se trazan EF å AD y GH å AB. Demos-
y trar que el área del paralelogramo EBHP
es igual al área del paralelogramo GPFD.
x

Soluciones

1. 5. 15. Pedro 14 años; Juan 12 años, y Santiago


10 años.
2. p – 3.
16. Ester 7 años; Isabel 16 años, y María
21 años.
3. 17.
17. Andrés 36 años; Guido 9 años, y David
4. 25, 27 y 29. 3 años.

5. 20. 18. Hijo 14 años y padre 38 años.

19. Hace 10 años.


6. 51 y 52.
20. $136, $204 y $612.
7. AB = 42 m, BC = 14 m y AC = 28 m.
21. Lápiz $198, cuaderno $305, goma $95.
8. 10 m.
22. Hernán $126; Gladys $63, y María $42.
9. Largo = 43,75 m; ancho = 26,25 m.
23. Gastón $ 1.320, y Javier $1.020.

10. l1 = 6m; l2 = 12 m; l3 = 18 m; 24. 2 horas 13 minutos 20 segundos.


l4 = 36 m; l5 = 24 m; l6 = 48 m.
25. 4 horas 46 minutos 21,8 segundos.
11. 4 u. 10
26. A la 11 horas 10 11 minutos y 11
horas 43 7 minutos.
12. No se puede construir un triángulo cuyos 11
lados están en la razón 3:4:7 porque Coincidirán a las 12 horas.
3 + 4 > 7. 27. A las 3 horas 49
1
minutos.
11
4
Coincidirán a las 3 horas 16 11
minutos.
13. 8 y 28 años.
28. 17
15
14. 28 y 34 años. 29. 72

88 Ecuaciones e inecuaciones de primer grado

88-89.(2003) 88 18/11/02, 1:47 PM


CAPÍTULO 2

30. 240, 180 y 144. 33. a)18 b) 40 c) 8 d) 20 e) 4.2


9
31. a) n = 487 (número de estudiantes 34. r s horas
matriculados. r+s
b) Lugar de nacimiento, color de 35. 2 horas 24 minutos
ojos, sexo. 36. Se cuadriplica
c) Talla, peso, N° de hermanos.
d) {Ciudades del mundo}; {café, negro, 38. Aumenta en un factor 27.
gris, celeste, verde}; {masculino, 39. Se mantiene; el área de un triángulo
femenino} varía en forma directamente proporcio-
{x E R/ 50 ≤ x ≤ 210 cm} nal a su base y a su altura.
{x E R/ 10 kg ≤ x ≤ 130 kg};
{x E Z / 0 ≤ x ≤ 15} 40. Observe las áreas de los triángulos ABC
y CDA; AEP y EGA; PHC y CFP.

Desigualdades e inecuaciones 2.3

El conjunto de los reales R es un conjunto ordenado; por lo tanto,


podemos comparar sus elementos mediante una relación de orden
y podemos decir que:
Para a, b E R se tiene
a<b¤a–b<0
a>b¤a–b>0
a=b¤a–b=0
El signo “<” se lee menor que.
El signo “>” se lee mayor que.
También se usa la combinación de un signo de desigualdad
con el signo igual:
“≤” se lee menor o igual que,
“≥” se lee mayor o igual que.
Una relación entre números o letras que representan números en que
se usan los signos <, >, ≤ o ≥ se llama desigualdad.
Son desigualdades numéricas o literales las siguientes:
1. 3 + 2 < 7
2. x + 5 ≥ 2
3. (x – 1)2 > 0
4. 2 – 5 ≥ 4
Una desigualdad puede ser verdadera o falsa.
En los ejemplos:
1. es verdadera
2. es verdadera para x = 2 y es falsa para x = – 4
3. es verdadera para x 1 y falsa para x = 1
4. es falsa
Cuando una desigualdad presenta una incógnita se denomina
inecuación y su valor de verdad (verdadero o falso) dependerá del valor
que le asignemos a la incógnita.

Ecuaciones e inecuaciones de primer grado 89

88-89.(2003) 89 18/11/02, 1:47 PM


Resolver una inecuación es encontrar el intervalo de números
reales para el cual la inecuación se transforma en una desigualdad
verdadera.
Dependiendo del grado que presenta la incógnita, las inecuaciones
pueden ser de primer, segundo, tercer... grado y dependiendo del
número de incógnitas diferentes, pueden ser inecuaciones de una,
dos o más variables.
Aquí demostraremos la veracidad de algunas desigualdades y
resolveremos las inecuaciones de primer grado.
Para desarrollar estos ejercicios hay que tener presentes las siguientes
propiedades de las desigualdades. Las enunciaremos usando el signo <,
pero son válidas también para >, ≤ y ≥.

Propiedades de las desigualdades.


Coinciden a, b, c, d E R
1. Al sumar una misma cantidad en ambos miembros de una desigualdad,
la desigualdad se mantiene
a<b⇔ a+c<b+c
2. Al restar una misma cantidad en ambos miembros de una desigualdad,
la desigualdad se mantiene
a<b ⇔ a–c<b–c
3. Al multiplicar o dividir una desigualdad por una cantidad mayor que
cero, la desigualdad se mantiene
a < b ⇔ ac < bc c>0
a b
a<b ⇔ < c>0
c c
4. Al multiplicar o dividir una desigualdad por una cantidad menor que
cero, la desigualdad se invierte
a < b ⇔ ac > bc c<0
a b
a<b ⇔ > c<0
c c
5. Al invertir ambos miembros de una desigualdad, ésta cambia
de signo
1 1
0<a<b ⇔ >
a b
6. Al elevar a la misma potencia ambos miembros de una desigualdad,
ésta se mantiene
0 < a < b ⇒ an < bn n E R+
7. Al sumar miembro a miembro desigualdades del mismo signo, la
desigualdad se mantiene
a<b
c<d }⇒a + c < b + d
90 Ecuaciones e inecuaciones de primer grado

90-91.(2003) 90 20/11/02, 4:03 PM


CAPÍTULO 2

8. La regla de los signos para el producto se expresa en términos


de desigualdad así
a>0^b>0 ⇒a•b>0
a<0^b<0 ⇒a•b>0
a>0^b<0 ⇒a•b<0
• Observación: Nótese que las propiedades 5 y 6 sólo se cumplen si
los términos que se comparan son ambos mayores que cero (positivos).

2.3.1 Desigualdades

1. Demostrar que si n > 0 entonces

n+
1
≥2
Ejercicios
n
Sabemos que el cuadrado de un número es siempre mayor que
resueltos
cero, luego
(n – 1)2 ≥ 0
1
n2 – 2n + 1 ≥ 0 • > 0 (prop. 3)
n
1
n–2+ ≥ 0 +2 (prop. 1)
n
1
n+ ≥ 2
n
1 1 2
2. Si x > 0 e y > 0, demostrar que + >
x y x+y
Esto es verdadero si y sólo si
1 1 2
+ – >0
x y x+y
Entonces 1 1 2
+ – =
x y x+y
x+y 2
– =
xy x+y
2
x + y – 2xy
=
xy x + y
2 2
x + 2xy + y – 2xy
=
xy x + y
2 2
x +y
>0
xy x + y
ya que por dato del problema
x>0 e y>0 ⇒ x+y >0
xy > 0
∴ xy (x + y) > 0
además x2 > 0
e y2 > 0
∴ x + y2 > 0
2

Ecuaciones e inecuaciones de primer grado 91

90-91.(2003) 91 20/11/02, 4:04 PM


2 2
Ejercicios luego x +y
>0
xy x + y
resueltos
y 1 1 2
+ – >0
x y x+y
1 1 2
luego + >
x y x+y

Ejercicios
1 1. Demuestre que si x > 0 e y > 0 entonces x > y ⇔ x 2 > y2

1 2. Demuestre que, para a y b positivos, su media aritmética es


a+b ab
mayor que su media geométrica, es decir >
2
1 3. Demuestre que si 0 < x < 1 entonces x2 < x

1 4. Demuestre que x2 + 4 > 4x

1 5. Demuestre que x2 + y2 + 1 > 2 (x + y – xy)

1 6. Demuestre que a2 + 2a + b2 – 2b > 2ab – 1

1 7. Si b > 0 y d > 0 entonces a < a + c < c con a > c


b b+d d
b d
1 8. Demuestre que si a > 0 y b > 0, entonces a b + ab3 < a4 + b4
3

1
1 9. Demuestre que si a > 0 entonces a2 + 2
≥2
a
10. Determine para qué valores de a la desigualdad del ejercicio
anterior es igual a 2

Soluciones

1. Datos del ejercicio x > 0 , y > 0


Debe demostrar hacia ambos lados la implicación:
⇒) Dato : x > y . Por dem : x2 > y2

x>y⇒x–y>0 /• (x + y) > 0 (por dato)

(x – y) (x + y) > 0
x2 – y2 > 0
x2 > y2

92 Ecuaciones e inecuaciones de primer grado

92-93. 92 18/11/02, 1:50 PM


CAPITULO 2
CAPÍTULO

fi) Dato: x2 > y2 Por dem.: x > y


x2 > y 2 ⇒ x2 – y2 > 0
(x + y) (x – y) > 0 pero x + y > 0 (por dato)
x–y>0
x>y
2
2. a– b >0
a – 2 ab + b > 0
a + b > 2 ab
a+b
> ab
2
3. 0 < x < 1 ⇒ x > 0 y x – 1 < 0
x (x – 1) < 0
x2 – x < 0
∴ x2 < x
4. (x – 2)2 > 0
x2 – 4x + 4 > 0
x2 + 4 > 4x
5. x2 + y2 + 1 – 2 (x + y – x y) =
x2 + y2 + 1 – 2x – 2y + 2xy =
(x + y)2 – 2 (x + y) + 1=
(x + y – 1)2 > 0
∴ x + y + 1 – 2(x + y – x y) > 0
2 2
x2 + y2 + 1 > 2 (x + y – x y)
6. a2 + 2a + b2 – 2b –2ab + 1 =
(a – b)2 + 2 (a – b) + 1 =
(a – b + 1)2 > 0
∴ a + 2a + b – 2b – 2ab + 1 > 0
2 2
y a2 + 2a + b2 – 2b > 2ab – 1

7. Por hipótesis bd > 0

a a+c ab + ad – ab – bc
entonces – = 2
b b+d b + bd
ad – bc
= 2
<0
b + bd
2
1
∴ 9. a– a ≥0

8. a3 b + a b3 – a4 – b4 = a3 (b – a) + b3 (a – b) a2 – 2 +
1
≥0
2
= a3 (b – a) – b3 (b – a) a
= (b – a) (a3 – b3) a2 +
1
≥2
2
= (b – a) (a – b) (a2 + ab + b2) a

= – (b – a)2 (a2 + ab + b2) < 0


∴ a3 b + ab3 – a4 – b4 < 0 10. para a = 1 y a = –1
a3b + ab3 < a4 + b4

Ecuaciones e inecuaciones de primer grado 93

92-93. 93 18/11/02, 1:51 PM


2.3.2 Inecuaciones

Ejercicios 1. Si a < b. Escribir como intervalo y gráficamente los siguientes


resueltos conjuntos.
a) {x E R / x ≥ a}
b) {x E R / x > a}
c) {x E R / x ≤ b}
d) {x E R / x < b}
e) {x E R / a < x < b}
f) {x E R / a ≤ x ≤ b}
g) {x E R / a < x ≤ b}
h) {x E R / a ≤ x < b}

a) [a, + ∞) a

b) (a, + ∞) a

c) (– ∞, b] b

d) (– ∞, b) b

e) (a, b) a b

f) [a, b] a b

g) (a, b] a b

h) [a, b) a b

2. Resolver la inecuación.
2x – 5 < x + 2
aplicando las propiedades
2x – x < 2 + 5
x < 7 o (– ∞, 7)
Lo que gráficamente es
7
En efecto, cualquier valor de x menor que 7 hace que la
desigualdad sea verdadera, por ejemplo, si x = 3.
2 •3 – 5 < 3 + 5
1<8 lo que es verdadero

94 Ecuaciones e inecuaciones de primer grado

94-95. 94 18/11/02, 1:55 PM


CAPÍTULO 2

3. Resolver la inecuación
x 2x
4– ≥ –1
3 5
Aplicando las propiedades

x 2x
4– ≥ –1 / • 15
3 5
60 – 5x ≥ 6x – 15
1
– 11x ≥ – 75 / •–
11
75

11

Nótese que al multiplicar por un número negativo, la desigualdad


se invierte.
La solución se puede entregar como intervalo real – ∞,
gráficamente
(75
11
o ]
75

11

75
Verifiquemos para x = – 1 <
11
–1 –2
4– ≥ –1
3 5

13 – 7

3 5 lo que es verdadero.

4. Resolver la inecuación

(x – 1)2 ≤ x2 – +3

Resolviendo paréntesis y aplicando las propiedades

x2 – 2x + 1 ≤ x2 – +3

– 2x + 1 ≤ – +3 /•2

– 4x + 2 ≤ – x + 6

– 3x ≤ 4 /•
4
x ≥–
3
4
[
como intervalo – , + •
3 ) y gráficamente

4
–—
3

Ecuaciones e inecuaciones de primer grado 95

94-95. 95 18/11/02, 1:56 PM


Ejercicios
Escriba los siguientes conjuntos Resuelva las siguientes inecuacio-
I. como intervalos: III. nes y escriba su solución como
intervalo:
1. {x E R / 2 < x ≤ 3}
1. x2 – 4x – 1 ≥ (x – 5)2
2. {x E R / –7 ≤ x < – 5}

3. {y E R / y > – 4} 2. (x + 1) (x – 5) ≤ (x – 2) (x + 5)

4. {x E R / x < – 2 V x > 4} 3. 4x – (x2 – 1) < (5 – x) (1 + x)

5. {x E R / x ≤ 3 V x ≥ – 2} 4. (3x – 1) (x + 2) < x + 3 x2
6. {x E R / x < –1 A x ≥ – 3}
5. (x + 2) (x – 2) > (5 + x) (x + 1)
7. {x E R / x ≤ – 2 A x ≥ 2}
6. (2x – 1) (x + 4) – (x – 1) (2x + 3) ≥ 0

7. (3x – 2) (x + 5) + (1 – x) (4 + 3x) < 0


II. Resuelva las siguientes inecua-
ciones y grafique su respuesta: 8. (x + 2) (x – 3) < (x + 4) (x – 5)
1. 5x + 2 < 2x – 1 9. (2x – 1) x ≤ (x + 3) 2x
2. 3 – 4x ≥ – 3 + 2x
10. (3x + 1) x + (5 – x) 3x > 0
3. 2x – 1 > 3

4. 5 – 3x ≤ 12 IV.
5. 2 x – 1 > 0 1. Encuentre los números enteros positivos
2
6. 4x – 2 x + 1 + 1 < 0 tales que su quinta parte más 3 sea
3 mayor que la mitad de su triple.
4 – 2x 5 – 3x
7. ≥
3 4
8. 2 – 3x – 6x + 1 ≥ 0 2. Encuentre los números naturales cuya
2 3 tercera parte sea mayor que su mitad,
9. 1 – x 2 x – 1 4x + 2 más 1.
+ <
2 3 6

Soluciones
I
1. (2, 3] 2. [– 7, – 5) 3. ( – 4, + ∞)
4. (– ∞, – 2) K (4, + ∞) 5. R 6. [ – 3, – 1) 7. ∅
II
1. x < – 1 2. x ≤ 1
–1 1
–7
3. x > 2 4. x ≥
3
2 –7
3

96 Ecuaciones e inecuaciones de primer grado

96-97. 96 20/11/02, 4:57 PM


CAPITULO 2
CAPÍTULO

1 1
5. x > 6. x < –
2 1
2
5
_1
7. x ≥ – 1 5
–1
1
4 9. x > –
3
8. x £ –
1
21 4 3
21
III

1. [ 13
3
, +∞) 2. [ , + ∞)
5
7
3. R (
4. – ∞, 1
2 ) (
5. – ∞, – 3
2 )
6. [ 1
, + ∞) 7. ( – ∞, ) 1
8. ∅ 9. [ 0, + ∞ ) 10. ( 0, + ∞ )
6 2
IV
1. 1 y 2 2. No hay.

2.3.3 Inecuaciones simultáneas


Se llaman inecuaciones simultáneas aquellas que se satisfacen
simultáneamente. Se considera como solución de ellas aquel intervalo
para el cual se satisfacen todas.

1. Resolver simultáneamente las siguientes inecuaciones. Ejercicios


x–1> 2 resueltos
x – 2 < 2x + 3

Resolviendo la primera
x–1> 2
x> 3
Resolviendo la segunda
x – 2 < 2x + 3
–x< 5
x> –5
Comparando ambas soluciones en la recta numérica vemos que
ambas se satisfacen sólo para x > 3

–5 3
Luego la solución es x > 3 o (3, + ∞)
2. Resolver las inecuaciones simultáneas.
3x – 1 > x + 2
x x
≤ 4
+3
2

Ecuaciones e inecuaciones de primer grado 97

96-97. 97 20/11/02, 4:58 PM


Resolviendo la primera
Ejercicios
3x – 1 > x + 2
resueltos 2x > 3
3
x > 2

Resolviendo la segunda
x x
£ +3
2 4
2x ≤ x + 12
x ≤ 12
Comparando ambas soluciones en la recta numérica, observamos
que sólo se satisfacen ambas inecuaciones en la intersección de
3
ellas, es decir, para < x ≤ 12.
2

3 12
2
Luego la solución es
( ]
3
2
, 12

3. Resolver las inecuaciones simultáneas


5x – 1 > 0
3x + 3 < x + 1

Resolviendo la primera
5x – 1 > 0
5x > 1
x>1
5
Resolviendo la segunda
3x + 3 < x + 1
2x <– 2
x <–1
Comparando ambas soluciones en la recta numérica vemos
que no hay ninguna solución común, luego no existe x que
las satisfaga a ambas.

–1 1
5
Luego la solución es vacía (∅)

4. Resolver simultáneamente las siguientes inecuaciones


x + 3 < 2x – 1
x + 3 < 2x + 1
2 4 3
5x – 1 > x + 2
4

98 Ecuaciones e inecuaciones de primer grado

98-99. 98 20/11/02, 9:54 AM


CAPÍTULO 2

En este caso debemos resolver las tres inecuaciones y comparar


sus resultados.
Resolviendo la primera
x + 3 < 2x – 1
–x<–4 /• –1
x>4
Resolviendo la segunda
x 3 1
+ <2x+ /• 12
2 4 3
6x + 9 < 24 x + 4
–18 x < – 5
5
x>
18
Resolviendo la tercera
x
5x – 1 > 4 +2 /• 4
20x – 4 > x + 8
19 x > 12
12
x>
19
Comparando las tres soluciones vemos que la solución para todas
simultáneamente es x > 4

5 12 4
18 19
Luego la solución es (4, + ∞).

Ejercicios
Resuelva las inecuaciones simultáneas siguientes:
5. 6x ≥ 2x – 1
1. 2x – 1 > x + 3 – 2x x
x
4x – 5 < x + 2 < +3
4 3
x 5x
2. + –1≥ 0 9x
2 3 6. – x ≥x–2
4x – 3 ≤ x + 2 5 3
3x 2x
+ ≤x+1
3. 2x + 1 – x < x + 3 2 3
3x 2x
– <0
5 4 1 2x 4x
7. – < +1
2 3 3
2x
4. 2x + 1 ≥ x + 3 1
3 – x ≥ 2x + 4
3x + 1 ≤ 2 4 5

Ecuaciones e inecuaciones de primer grado 99

98-99. 99 20/11/02, 9:56 AM


Ejercicios
12. 2x – 3 > – x
8. 2 13 + 3 1
4
x≤ 1
3
– x
4
x+2– x
≥0 1 – 2x > 5
5
4x – 1 < 3
9. (x – 1) (2x + 3) ≤ (x – 5) (2 + 2x)
(2 – x) (3 – x) ≥ (8 – x) (1 – x) 13. (x – 1) x ≤ x2 + 2
1 + x2 ≥ 2x + x2 – 1
10. (x – 1) (2x + 3) ≥ (x – 5) (2 + 2x)
3x – 5 < 2x + 4
(2 – x) (3 – x) ≤ (8 – x) (1 – x)

11. 2x + 1 < 3 14. 5 – 2x + 1 > 0


x + 2 > 2x – 1 2x – 3 + x < 0
1 – 3x < 0 2x + 3 – 1 < 0

Soluciones
6 5
4. – 3, 13
4 7
1. ,
3 3
2. , 3. (– ∞, 0)
13 3

5. [ _ 41 , + ∞ ) 6. – 30
7 7
,
6
(
1 5
7. – 4 , 22 ] 5 4
8. – 2 , – 7

9. ∅ 10. – 7 , 1 11. 1
,1 12. ∅
9 2 3

13. [– 2, 1] 14. (– ∞, – 1)

2.3.4 Inecuaciones
con valor absoluto
El valor absoluto de un número real x representa la distancia
a que éste se encuentra del origen y se denota por | x |

|x|= { x si x ≥ 0
–x si x < 0

Propiedades de la función valor absoluto

1. | x | ≥ 0 IxER
2. | x | = 0 ⇔ x = 0
3. | x |2 = x2 I x E R
4. ÷x2 = | x | I x E R
5. – | x | ≤ x ≤ | x | I x E R
Teorema 1: Sean x, a E R, a > 0, entonces:
ii) | x | ≤ a ⇒ – a ≤ x ≤ a
ii) | x | ≥ a ⇒ x ≥ a V x ≤ – a

100 Ecuaciones e inecuaciones de primer grado

100-101. 100 20/11/02, 9:58 AM


CAPITULO 2
CAPÍTULO

1. Demostrar el teorema 1 que dice: Sean x, a E R, a > 0, entonces


Ejercicios
ii) | x | ≤ a ⇒ – a ≤ x ≤ a
resueltos
ii) | x | ≥ a ⇒ x ≥ a V x ≤ – a
Demostración:
i)) si x ≥ 0 ⇒ | x | = x ≤ a 1
si x < 0 ⇒ | x | = – x ≤ a ⇒ x ≥ – a 2
de donde 1 y 2 – a ≤ x ≤ a
ii) si x ≥ 0 ⇒ | x | = x ≥ a 1
si x < 0 ⇒ | x | = – x ≥ a ⇒ x ≤ – a 2
de donde 1 y 2 x ≥ a V x ≤ – a

2. Resolver la inecuación
| 5x – 3 | < 2
Por teorema 1 tenemos
–2 < 5x – 3 < 2 / sumamos 3
1 < 5x < 5 / dividimos por 5
1
< x<1
5
1
Solución ,1
5 0 1 1
5
3. Resolver la inecuación
| 5 + 2x | ≥ 3
Por teorema 1
5 + 2x ≥ 3V 5 + 2x ≤ – 3
2x ≥ – 2 V 2x ≤ – 8
x≥–1Vx ≤–4 –4 –1
Solución: (– ∞, – 4] K [– 1, + ∞)

4. Resolver la inecuación
|x+1|+|x|≤3
Como se trata de una suma de valores absolutos, no podemos aplicar
el teorema anterior.
Vamos a eliminar las barras de valor absoluto conociendo el signo de
la expresión que está entre barras. Analizaremos la recta numérica a
tramos, teniendo en cuenta en qué puntos (números reales) la expresión
entre barras cambia de signo (o se hace cero).
x + 1 = 0 para x = – 1
x = 0 para x = 0
Dividimos la recta numérica en tres tramos:

1 0

Ecuaciones e inecuaciones de primer grado 101

100-101. 101 20/11/02, 9:58 AM


Ejercicios i) Si x E ( – ∞, – 1), entonces x + 1 < 0
x<0
resueltos
Al aplicar la definición de valor absoluto:
| x + 1 | = – (x + 1) |x|=–x
entonces la inecuación queda:
– (x + 1) – x ≤ 3
–x – 1 – x ≤ 3
– 2x ≤ 4
x≥–2
Luego del intervalo (– ∞, – 1) es solución el intervalo [ – 2, – 1)

ii) Si x E [ – 1, 0), entonces x + 1 ≥ 0


x<0
| x + 1| = x + 1 y |x|=–x
entonces la inecuación queda:
x+1–x≤3
1≤ 3 verdadero, lo que indica que todo el intervalo
considerado es solución: [ – 1, 0)

iii) Si x E [ 0, + ∞) entonces x + 1 > 0


x≥0
luego | x + 1 | = x + 1 y |x|=x
donde la inecuación queda:
x+1+x≤3
2x ≤ 2
x≤1
Luego del intervalo [ 0, + ∞) sólo es solución el intervalo [ 0, 1].
Uniendo las soluciones de i, ii, iii tenemos que la solución de la
inecuación es S = [ – 2, – 1) K [ – 1, 0 ) K [ 0, 1 ] = [ – 2, 1 ]

Ejercicios
Resuelva las siguientes inecuaciones: 6. | 1 – 4x | – 2 ≥ 0

1. | 2x + 5 | ≤ 3 7. 1 – | 2x + 5 | + 3 ≤ 0
3
2. | 5 + 3x | < 3 8. 1– x +2≤5
4 4

3. | 6 – 4x | ≤ 2 9. | 3x + 1 | + 5 ≤ 0

4. | 2x + 1 | > 5 10. | 1 – 2x | + 3 ≥ 0

5. | 7x – 4 | ≥ 3 11. 3 < | x + 2 – 3x |

102 Ecuaciones e inecuaciones de primer grado

102-103. 102 20/11/02, 10:00 AM


CAPÍTULO 2

12. 5 > | 2x – 3 | 25. – 11 £ 2 – | – x – 1 |

13. 2+x 2
£3 26. | 19x | ≥ 0

14. x2 – 10x + 25 ≥ 2 27. 5 ≥ 5 – | 9x |

15. | x + 3 | ≤ x 28. – 36 > 4 (– 9 – | 6x + 3 | )

16. | 2x – 1| – | x + 2 | < 3 29. | 19x | > 0

17. 1 – | 3 – x | + | 2x + 3 | ≤ 0 30. | – 4x – 3 | ≤ 12

18. | x | + | 3x – 6 | > 5 31. 9 + | – 3x – 8 | < –1

19. | x – 2 | + | x + 3 | < | 2x – 2 | 32. 9 > 9 – | – 9x + 5 |

20. | 2x – 1 | – | 3x + 2 | ≤ 4 33. | 3x + 4 | > 0

21. | 3x + 2 | – | 2x – 1 | ≤ 4 34. – 2 + | x – 2 | ≤ – 2

22. – 3 > 11 – | – 9x – 3 | 35. – 9 ≤ – 9 – | – 6x – 4 |

23. 3 (6 + | – 3x + 4 | ) ≤ 63 36. 7 (2 + | x – 7 | ) ≤ 119

24. – 63 ≥ 9 ( – 7 – | 9x – 2 | ) 37. – 2 > 3 – | 5x + 6 |

Soluciones
13. [ – 5, 1 ]
25. [ – 14, 12]
1. [ – 4, – 1 ]
14. ( – ∞, 3 ] K [ 7, + ∞)
26. R
2. – 23, – 17
12 12 15. ∅
27. R
3. [ 1, 2 ]
4 1
16. – , 6 28. R – –
4. ( – ∞, – 3) K (2, + ∞) 3 2

17. – 5, – 1 29. R – {0}


( 17] K [1, + ∞)
5. – ∞, 3
30. [ – 3 75 ; 2,25 ]
,

6. ( – ∞, ] K [ , + ∞) 18. – •, 1 K 2 3 , + •
1 3
4 4
2 4
31. No tiene solución
7. ( – ∞, – ] K [– , + ∞) 19. – •, – 1 1
9 1
5
2 2 2 32. R – {0,5} o R – 9
– 8 16 20. R 4
8. , 33. R – –
3 3 3
9. ∅ 21. [ – 7, 1] 34. {2}
17 11
22. – •, – 9 K 9 ; + • –2
10. R 35.
3
11 19
1 5 23. [ – 3,67; 6,33] = – ,
11. –•,–
2
K
2
,+• 3 3 36. [ – 8, 22 ]

12. ( – 1, 4) 24. R 37. (– ∞ ; – 2,2) K (– 0,2 ; + ∞)

Ecuaciones e inecuaciones de primer grado 103

102-103. 103 20/11/02, 10:01 AM


Prueba de selección múltiple
1. En la expresión 3x – 7 = 2, x vale: B. 1
3
A. 1 C. 2
B. 2 D. – 1
3
C. 3 E. – 1
D. 4
E. 5 7. En la ecuación
0,2x – 0,3x + 3,1 = x – 3,5 x vale:
2. La solución de la ecuación
5y + 2 = 4y – 5 es: A. 0,06
A. –7 B. 0,6
B. –5 C. 6
C. 0 D. – 0,6
D. 5
E. – 0,06
E. 7
2 x 3x x 1
8. La expresión + – = –
3. La expresión 3(x + 2) – 2(x + 3) = x 3 2 4 2 4
es verdadera para x igual:
es verdadera para:
A. x=0 A. 9
11
B. x=1
9
C. x = –1 B. –
11

D. ningún x 11
C.
9
E. todo x real
11
D. –
4. En la ecuación (x–1)2+ 2 = x2 – 1, 9

x vale: 1
E.
9
A. –2
B. –1 9. El valor de x en la ecuación
C. 0 4x – 3 5x + 2 3 6x + 1 3x + 5 2
– + = + – es:
4 9 2 3 12 9
D. 1
A. – 2
E. 2 B. – 1
C. 0
5. La solución de la ecuación
D. 1
(2z – 1) (z + 2) = 1 + 2z2
E. 2
A. 2
B. 1 10. La solución de la ecuación
C. 0 1 2
2x– 4x +
2 3 , es:
D. –1 =
3 4
E. –2 A. – 2
B. – 1
6. En la ecuación x(1–x2)+3 = 2 – (x2 + 2)x,
C. 0
x vale:
D. 1
A. 1 E. 2
104 Ecuaciones e inecuaciones de primer grado

104-105. 104 20/11/02, 10:04 AM


CAPITULO 2
CAPÍTULO

11. La expresión 15. El valor de x que hace D. 2


13x 4x verdadera la expresión E. – 2
–5 x+2 = –2 x+1 2
3 3 3x + 5
x–3= es:
es verdadera para: 3x + 1 19. La solución de la ecua-
A. –2 ción
A. x = 0 1 3 –4
B. –1 – =
B. x = 1 x + 2 2 x – 1 2 x2 + 3x – 2
C. 0 es:
C. x = – 1
D. 1 A. –1
D. ningún x
E. 2
E. todo x real B. –2
C. –3
16. La solución de la ecuación
12. En la ecuación x–3 3x + 1 D. –4
– = 0 es:
2 3 3 2x + 5 6x – 4 E. –5
z z+1 z+1 3–z+z
– = 7
2 3 6 A. –
el valor de z es:
39 20. La solución de la ecuación
B. 7
39 1 2 1 + 2x
3 + =
A. C. –
39 2 – 2 x 3 – 3x 6x – 6
2 7
5 es:
B. D. 39
2
7
A. –1
C. 1 E. 5
3
B. –2
7
5
D. – C. –3
2

E. 3 17. En la ecuación D. –4

2
1 2 1 E. –5

x x x
13. En la ecuación = , 21. En la expresión
1 2 2
+
1 1 x 3 x ax + bx = a + b, x vale:
– = 0, x vale:
3x – 4 x + 2 el valor de x es: A. a
A. 1
A. 4,0 B. b
B. 2
B. 4,5 C. – a
C. 3
D. –b
C. 5,0
D. 4 E. 1
E. 5 D. – 4,5

E. – 4,0 22. En la ecuación


a b
14. La solución de la ecuación 2ax – 3bx = – ,
3 2
2 1 1 18. En la ecuación x vale:
es: + 2 =
x x 2x 1
A. 1 1– A. 6
x+1 1 , x vale:
=
B. 2 1 3 B. a
3 1+
1
C. 0 1+ C. b
x
D. –1 A. 1 D. 1
6
2
B. – 1 E. 1
E. – a
3 C. 0
Ecuaciones e inecuaciones de primer grado 105

104-105. 105 20/11/02, 10:05 AM


Prueba de selección múltiple
23. La ecuación 27. En la fórmula
2
mx + x = m2 – 1 B. 4b
es verdadera para x = n a 1 + an c
Sn = ,n=
A. m + 1 2 – 4b
2
C.
B. m–1 2Sn c
A. 2
a1 – an b
C. 1–m Sn
D.
B. 4c
D. m a1 – an
E. 4b2c
E. m2 C.
2Sn
gt
2

a 1 + an 31. Si E = , entonces
2
24. En la expresión t2 es:
Sn
p2 x – 2pq = (p – q)2, D.
a1 + an A. 2 Eg
x vale: Eg
E. 2Sn B.
p+q 2
A. 2
an–a1
p
5x
2
C. 2E
B. 1+
p 28. Si y = h – 2
,
q v g
q
C. 1+ entonces x2 = D. E
p
2 2g
p
D. 1+ A.
1
(h – y) v2
q 5 E. g
2
q 2E
E. 1+
p
B. 5 (h – y) v2

25. El valor de x en la 32. En la expresión


C. 5 (y – h) v2
ecuación 5 = π r2, r2 =
px + p2 = 1 + x es: D. 1
(y – h) v2
5
A. 1+p A. 5π
5
E. h–y
B. 1–p v
2
p
B.
C. p–1 5
29. En la expresión C. 5
D. –1 – p 2p
E. 1 y = mx + k, m = D. 5
p
26. En la fórmula A. y E. 5
–k rp
x
an = a1 + (n – 1) d, d =
y–k
B.
x 33. La frase “el doble de un
an + a1
A. k número menos su cuarta
n+1 C. y–
x parte” se expresa:
an – a1 D. y+k
B. n
n–1 x A. n–
k 4
an – a1 E. y+
C. x B. 2n – 4n
1–n
a1 – an C. 8n – n
D. 30. En la ecuación 2
n–1
D. 2n
an – a1
E. b2 – 4ac = 0 , a = 4
n+1 7n
E.
–b
2 4
A.
4c

106 Ecuaciones e inecuaciones de primer grado

106-107. 106 20/11/02, 10:10 AM


CAPITULO 2
CAPÍTULO

2
34. El 20% de un número sumado con el 38. Un poste está enterrado 5 de su
2
doble de él se expresa: longitud, del resto está bajo
7
agua y sobresalen 3 m. ¿Cuál es la
A. n longitud del poste?
B. 2 n A. 6 m.

C. 2,2 n B. 7 m.

D. 22 n C. 9 m.

E. 120 n D. 9,5 m.

35. El 20% de x menos el 50% de y lo E. 10 m.


podemos expresar como: 39. Si un niño tiene el triple de la edad que
tenía hace 6 años, ¿cuántos años tiene
A. 2x – 5y en la actualidad?

B. 2 x – 5y A. 3
100
B. 6
C. 2 x + 5y
10 C. 9

D. 0,02x – 0,05y D. 12

2 x – 5y E. 18
E.
10 40. Un comerciante compró 25 juguetes. Si
36. Un número sumado con su quinta parte hubiera comprado 5 juguetes más por
es 12. La ecuación que representa esta el mismo valor, cada juguete le habría
situación es: costado $ 10 menos. ¿Cuánto le costó
cada juguete?
A. x + 12 = x
5 A. $ 10
x
B. x + = 12
5 B. $ 30
x
C. 12 + = x
5 C. $ 50
x
D. x – = 12
5 D. $ 60
x
E. x – 12 = E. $ 80
5

37. “La suma de dos números pares con- 41. Son soluciones de la inecuación
secutivos es 106”. Ésta se representa 2x – 3 ≤ 5 los números:
mediante la ecuación:
I. 4 II. 5 III. 3
A. 2n + (2n + 1) = 106 A. Sólo I
B. 4n + 1 = 106 B. Sólo II

C. 4n + 2 = 106 C. Sólo III

D. n + n + 1 = 106 D. Sólo I y III

E. 2n + 1 = 106 E. I, II y III

Ecuaciones e inecuaciones de primer grado 107

106-107. 107 20/11/02, 10:10 AM


Prueba de selección múltiple
42. No son solución de la inecuación 47. Al resolver la inecuación 2 – 3X < 1 – 4x
se obtiene que: 5 2
2x – 1 ≤ x + 3
1
A. x >
I. 5 II. 3 III. 8 14
1
A. Sólo I B. x ≥
14
1
B. Sólo II C. x <
14
C. Sólo III D. x ≤ 1
14
D. Sólo I y III 1
E. x =
14
E. I, II y III
48. La inecuación 2 – x – x – 1 ≥ 3 – x es
43. Si x distinto de cero, de las expresiones 3 2 4
equivalente a:
siguientes son verdaderas siempre:
A. 5x≥7
I. x2 > 0 II. x2 > x III. | x | > 0 B. 5x≥–7
A. Sólo I C. 7x≥5
D. 7x≤5
B. Sólo II
E. 7x≤–5
C. Sólo III
D. I, II y II
E. Sólo I y III 49. La inecuación (x – 1) (x + 3) ≤ (x – 2)2
es equivalente a:
44. De las desigualdades siguientes son siempre
verdaderas: A. 6≤7x
1 B. 6x ≤7
I. x 2+y 2 ≥ 2xy II. x + >2
x C. 6≥7x
III. x 2 + 4 ≥ 4 x
A. Sólo I D. 6 x ≥ 7
B. Sólo II E. –6 ≤7x
C. Sólo I y III
50. La solución de la inecuación
D. Sólo III
(2x – 1)(2 – x) + (1 + 2x) (x + 3) ≥ 13 es:
E. I, II y III
A. (– ∞,1]
B. (– ∞, 1)
45. La solución de la inecuación 3 – x ≥ 1 es:
C. (1, + ∞)
A. [– 2, + ∞)
D. [1, + ∞)
B. [2, + ∞)
E. (– 1, 1)
C. (– ∞, –2]
D. (– ∞, 2] 51. ¿Cuántos números naturales no cumplen
E. [– 2, 2] la condición de que su tercera parte más
8 sea menor que su quíntuplo?
46. El intervalo solución de la inecuación
5x – 3 A. 1
< 0 es: 3
–2 D. 5
,+• B. 2
A. (3, 5)
E. 3 C. 3
–•,
3 – ,+•
B. 5 5
D. Ninguno
C. 3
–•,– E. Todos
5

108 Ecuaciones e inecuaciones de primer grado

108-109. 108 20/11/02, 10:13 AM


CAPITULO 2
CAPÍTULO

52. ¿Cuántos números naturales cumplen la A. (– ∞, – 5)


condición de que su décima parte es B. (– ∞, – 5]
mayor o igual que su mitad disminuida
1
en 2? C. – 5,
4
A.
B.
1
2
D. ( 1
4
,+•
)
C. 3 E. [ 1
4
,+• )
D. 4 57. Son solución simultánea de ambas inecua-
E. 5 ciones: 5x≥x–8

53. “La quinta parte de un número disminuido ≤2


en 3 es mayor que el doble de él”. Esta
proposición se escribe algebraicamente I. –1 II. 0 III. 3
como: A. Sólo I
x–3
A. >2x B. Sólo II
5
x C. Sólo III
B. –3>2x
5
x–3 D. Sólo I y II
C. <2x
5 E. I, II y III
x
D. –3<2x
5

E. x – 10 < 2 x 58. La solución simultánea de las siguientes


inecuaciones es:
54. El doble de un número natural se aumenta
(x – 1) (x + 2) ≤ (x – 3) ( x + 1)
en 3. El doble de esta expresión resulta
igual a 12. ¿Cuál es el número? 2x–3≤2–3x
x (x + 2) ≥ (1 + x) (x – 3)
A. 1
B. 2 1 3
A. – ,
3 4
C. 3 1
B. – ,1
3
D. 4
3
C. ,1
E. No existe 4

D. –3 1
55. Los números enteros tales que su cuarta ,–
4 3
parte es menor que su mitad, disminuida
E. R
en 2, son los números:
59. La solución de |3 – 2 x | ≤ 5 es:
A. Menores que – 8
A. [1, 4]
B. Menores que 8 B. [1, – 4]
C. Mayores que – 8 C. [– 1, 4]
D. [– 1, – 4]
D. Mayores que 8
E. (– 1, – 4)
E. No hay
60. La solución de |2 x + 3 | ≥ 7 es:
56. La solución de las inecuaciones A. (– ∞, – 5) K (2, + ∞)
simultáneas siguientes es: B. (– ∞, – 5] K [2, + ∞)
x–2≥2x+3 C. [– 5, 2]
D. (– 5, 2)
3x–1≤1–5 x
E. (– 5, + ∞)

Ecuaciones e inecuaciones de primer grado 109

108-109. 109 20/11/02, 10:14 AM


Soluciones

1. C 2. A 3. E 4. E 5. B
6. D 7. C 8. C 9. C 10. B
11. D 12. D 13. C 14. E 15. B
16. B 17. D 18. A 19. C 20. D
21. E 22. D 23. B 24. E 25. D
26. B 27. C 28. A 29. B 30. D
31. C 32. D 33. E 34. C 35. E
36. B 37. C 38. B 39. C 40. D
41. D 42. D 43. E 44. C 45. D
46. D 47. C 48. D 49. B 50. D
51. A 52. E 53. A 54. E 55. D
56. B 57. E 58. D 59. C 60. B

110 Ecuaciones e inecuaciones de primer grado

110. 110 20/11/02, 10:15 AM


CAPÍTULO 3
R elaciones
y funciones

Lógica 3.1

Una expresión del lenguaje a la cual puede aplicarse con


sentido uno y sólo uno de los calificativos “verdadera” o “falsa”
se denomina proposición.
Es decir, una proposición es una expresión susceptible de ser
verdadera o falsa.
p: El ser humano es mortal
q: El perro tiene dos patas
Si una proposición es verdadera, diremos que su valor de verdad es
V y si es falsa, diremos que su valor de verdad es F.
Se llama función proposicional o proposición abierta a una
proposición en que el sujeto está dado en forma de símbolo y
puede ser reemplazado por alguno de los elementos de un conjunto
fijado con anterioridad.
p (x): x es un número natural x E N
Cada vez que el símbolo o variable (x) sea reemplazado por un
elemento del conjunto (en este caso N) la función proposicional pasa
a ser proposición y tiene su valor de verdad.
Si x = 2 “2 es número natural” es V
Si x = 0,5 “0,5 es número natural” es F
Al conjunto al que pertenece la variable se le llama dominio o
universo de la función proposicional.
Las funciones proposicionales pueden tener más de una variable.
q (x, y) : x e y viajaron en el buque Esmeralda el año 1992.
La negación de una proposición es aquella que modifica la
proposición dándole sentido contrario.
p : 5 es mayor que 2

Relaciones y funciones 111

111. 111 08/11/2001, 15:12


La negación de p se denota por ~ p
~ p : 5 no es mayor que 2 o
~ p : es falso que 5 es mayor que 2.
Axioma de la negación: p y ~p tienen valores de verdad contrarios.

p ~p
V F
F V

Las proposiciones (proposiciones simples) que hemos definido dan


origen a nuevas proposiciones (proposiciones compuestas) si éstas se
conectan a través de los conectivos lógicos siguientes:
V:o (disyunción)
A:y (conjunción)
fi: si ................, entonces (condicional)
¤: si y sólo si (bicondicional)
Ejemplo:
Si a • b = 0 entonces a = 0 o b = 0
p:a•b=0
q:a=0
r :b=0
Son proposiciones simples que dan origen a la proposición
compuesta enunciada y que simbólicamente se escribe:

p fi (q V r)
es decir (a • b = 0) fi (a = 0 V b = 0)
El condicional p fi q se puede expresar de las siguientes
maneras:
p implica q
si p, entonces q
q es condición necesaria para p
p es condición suficiente para q.
El bicondicional p¤q se puede expresar de las siguientes
maneras :
p si y sólo si q
q si y sólo si p
p es condición necesaria y suficiente para q
q es condición necesaria y suficiente para p
p es equivalente a q
q es equivalente a p
El valor de verdad de una proposición compuesta depende del
valor de verdad de las proposiciones simples que la forman según las
siguientes tablas de verdad.

112 Relaciones y funciones

112-113. 112 08/11/2001, 15:15


CAPÍTULO 3

Disyunción Conjunción
p q pVq p q pAq
V V V V V V
V F V V F F
F V V F V F
F F F F F F

Condicional Bicondicional
p q pfiq p q p¤q
V V V V V V
V F F V F F
F V V F V F
F F V F F V

El conectivo disyunción V se usa con una variante si se considera en


forma excluyente. Se denota Y y su tabla de verdad es:
Disyunción excluyente
p q pYq
V V F
V F V
F V V
F F F

Se llama tautología a una proposición compuesta cuyo valor de


verdad es verdadero (V) para cualquier combinación de los valores de
verdad de las proposiciones simples que la componen.
Se llama contradicción a una proposición compuesta cuyo valor
de verdad es falso (F) para cualquier combinación de los valores de
verdad de las proposiciones simples que la componen.
Los símbolos ∀ y ∃ se llaman cuantificador universal y
cuantificador existencial, respectivamente, y se leen “para todo”
y “existe”.
El cuantificador existencial presenta una variante que es ∃!, que
significa “existe un único”.
Ejemplos:
Si p (x) es una proposición abierta (o función proposicional)
y E es el conjunto en el cual se define (dominio o universo de
la función), entonces:
1. (∀ x ∈ E), p(x) se lee: “para todo x en E, tal que p (x)” o “para
cada x de E, p (x)”.

2. (∃ x ∈E), p(x) se lee: “existe x en E tal que, p (x)” o “existe por lo


menos un x en E tal que, p (x)”.

3. (∃! x ∈ E), p(x) se lee: “existe un único x en E tal que, p (x)” o


“existe sólo un x ∈ E tal que p (x)”.

Relaciones y funciones 113

112-113. 113 08/11/2001, 15:15


así podemos decir que:
1. (∀ x E E) p (x) ¤ [p (x1 ) ∧ p (x2 ) ∧ p (x3 ) ∧.....]
2. (∃ x ∈E) p (x) ¤ [p (x1 ) V p (x2 ) V p (x3 ) V.....]
3. (∃! x ∈ E) p (x) ¤ [p (x1 ) Y p (x2 ) Y p (x3 ) Y.....]
Si a una función proposicional se le agrega un cuantificador, ésta
pasa a ser una proposición, puesto que tiene un valor de verdad.
Ejemplo:
Sea p (x): x es un múltiplo de 2
E = {x ∈N / x < 10} = {1, 2, 3, 4, 5, 6, 7, 8, 9}
(∀ x ∈E), p (x) es falsa, puesto que hay elementos de E que no
son pares.
(∃ x ∈ E), p (x) es verdadera, puesto que hay al menos un elemento
de E que es par.
(∃! x ∈ E), p (x) es falsa, ya que existe más de un elemento de
E que es par.
Leyes de Morgan para cuantificadores.
1. ~ (∀ x ∈E), p (x) ¤ (∃ x ∈E), ~ p (x)
2. ~ (∃ x ∈E), p (x) ¤ (∀ x ∈E), ~ p (x)
En palabras:
1. “Es falso que para todo x en E se cumple p (x)” es equivalente a
“existe algún x en E tal que no se cumple p (x)”.
2. “Es falso que existe x en E tal que se cumple p (x)” es equivalente a“
para todo x en E, no se cumple p (x)”.

Ejercicios 1. Determinar cuáles de las siguientes expresiones del lenguaje son


resueltos proposiciones y determinar su valor de verdad.
a) ¿Qué hora es?
b) El árbol pertenece al reino vegetal
c) El queso es un subproducto de la leche
d) En Chile en invierno la temperatura ambiente pasa de 35º
e) Voy a salir, vuelvo más tarde
f) Cierra la puerta
Solución:
a) No es proposición, no podemos asignarle un valor de verdad
b) Es proposición y es verdadera
c) Es proposición y es verdadera
d) Es proposición y es falsa
e) No es proposición, no podemos asignarle un valor de verdad
f) No es proposición

114 Relaciones y funciones

114-115. 114 08/11/2001, 15:53


CAPÍTULO 3

2. Determinar cuáles de las siguientes expresiones son proposiciones


y cuáles son funciones proposicionales.
a) Los números mayores que 2 son negativos.
b) El número entero x es mayor que 10.
c) Los múltiplos de 3 son infinitos.
d) Los enteros x e y son factores de 12.
Solución:
a) Proposición; su valor de verdad es falso.
b) Función proposicional; según el valor que tome x, su valor
de verdad será verdadero o falso.
c) Proposición; su valor de verdad es verdadero.
d) Función proposicional; según los valores que tomen x e y
será su valor de verdad.

3. Sean p : x • y > 0 , q: x > 0 , r: y > 0


x, y ∈ R, x, y π 0.
a) Explicar qué significa ~ q , ~ r , ~ p
b) Escribir en símbolos la siguiente proposición: “el producto
de dos números reales es mayor que cero si y sólo si ambos
son positivos o ambos son negativos”.
c) Escribir en palabras la siguiente proposición:
~p ¤ (~q ∧ r) V (q ∧ ~ r)
Solución:
a) Como x, y ∈R y x, y π 0 entonces
~q:x<0 y ~r:y<0
~p: x • y<0
b) p : x • y > 0 : el producto de dos números x e y reales es
mayor que cero.
q : x > 0 : x es mayor que cero
r : y > 0 : y es mayor que cero
∴ p ¤ (q ∧ r) V (~q ∧ ~ r)
c) ~ p ¤ (~q ∧ r) V (q ∧ ~ r)
El producto de dos números reales es negativo si y sólo si el
primero es negativo y el segundo positivo o el primero es positivo
y el segundo negativo.

4. Demostrar que si p, q y r son proposiciones, entonces:


(p ⇒ q) ∧ (q ⇒ r) ⇒ (p ⇒ r)
Para hacer esta demostración haremos la tabla de verdad de
la proposición compuesta:

Relaciones y funciones 115

114-115. 115 08/11/2001, 15:53


Ejercicios 1 2 3 4

resueltos p q r p fiq q fir 1 A 2 p fir 3fi 4
V V V V V V V V
V V F V F F F V
V F V F V F V V
V F F F V F F V
F V V V V V V V
F V F V F F V V
F F V F V F V V
F F F F V F V V

Observamos que 3 fi 4 es una manera de escribir más


brevemente la proposición.
(p fi q) ∧ (q fi r) fi (p fi r) es verdadera para cualquier
combinación de valores de verdad de las proposiciones simples
que la componen.

5. Sean las siguientes proposiciones


p: ∀ x ∈ N, x + 2 > 0
q: ∃ x ∈N, x – 1 ∉ N
a) Determinar su valor de verdad
b) Escribir ~p y ~q
Solución:
a) p es verdadera, ya que para todo número natural x, x + 2 > 0
q es verdadera, porque existe un número natural x tal que x – 1 no es
natural. Ese número es 1, ya que 1 – 1 = 0 ∉ N
b) ~p : ∃ x ∈N, x + 2 ≤ 0
c) ~q : ∀ x ∈ N, x – 1 ∈ N

6. Sea A = {x ∈N / x < 5} y B = { y ∈N / y < 4}


Sean las proposiciones:
p: (∀ x ∈ A) (∃ y ∈ B), x + y < 6
q: (∃ x ∈ A) (∃ y ∈ B), x • y = 15
a) Determinar el valor de verdad de cada proposición
b) Escribir ~p y ~q
Solución: Vemos que A = {1, 2, 3, 4} y B = {1, 2, 3}
a) p es verdadero, ya que para todo número de A existe un
número de B tal que la suma es menor que 6.
q es falso, ya que no existe ningún número en A ni ningún
número en B tal que su producto sea 15.
b) ~p: (∃ x ∈A) (∀ y ∈ B) x + y ≥ 6
~q: (∀ x ∈ A) (∀ y ∈ B) x • y π 15

116 Relaciones y funciones

116-117. 116 08/11/2001, 15:56


CAPÍTULO 3

Ejercicios
1. Determine si las siguientes expresiones son o no proposiciones.
Si lo son, determine su valor de verdad.
a) p: Rembrandt es un pintor famoso.
b) q: La velocidad se define como distancia recorrida en un tiempo dado.
c) r: El 25% de 400 es 200.
d) s: Las gallinas son mamíferos
e) t: Vengan a tomar el té.
f) u: 13 es un número par.
g) v: Pintemos esa casa.
h) w: 25 es la décima parte de 250.
2. Determine cuáles de las siguientes expresiones son proposiciones y cuáles son
funciones proposicionales.
a) El conjunto de los números naturales es parte de los números reales.
b) Un número racional se puede escribir de muchas maneras.
2
c) Los racionales y 5 son equivalentes.
3 8 3
d) x en Q es equivalente a
4
e) El número entero x se puede escribir como un racional.
f) α es un ángulo agudo.
g) 60o es el suplemento de 120o.
h) Los enteros x e y son factores de 15.
3. Usando cuantificadores, transforme las funciones proposicionales del ejercicio
anterior en proposiciones verdaderas.
4. Sean p, q, r, proposiciones simples, T tautología y C contradicción. Demuestre que
las siguientes proposiciones son tautologías.
a) ~(p V q) ¤ ~p ∧ ~q
b) ~(p ∧ q) ¤ ~p V ~q } Leyes de Morgan

c) (p ⇒ q) ¤ (~p V q)
d) p V T ¤T
p ∧T ¤ p
pVC¤p
p∧C¤C
} Leyes de Identidad

e) p V p ¤ p
p∧p¤p } Leyes de Idempotencia

f) [(p ∧ q) ∧ r] ¤ [p ∧ (q ∧ r)]
[(p V q) V r] ¤ [p V (q V r)] } Leyes de Asociatividad

g) p ⇒ p V q ; p ∧ q ⇒ p
h) ~(~p) ¤ p
i) pVq¤qVp
p∧q¤q∧p } Leyes de Conmutatividad

Relaciones y funciones 117

116-117. 117 08/11/2001, 15:56


Ejercicios
j) [p V (q ∧ r)] ¤ [(p V q) ∧ (p V r)]
[p ∧ (q V r)] ¤ [(p ∧ q) V (p ∧ r)] } Leyes de Distributividad
k) (p ⇒ q) ¤ (~ q ⇒ ~ p)
l) (p ¤ q) ¤ (~ p ¤ ~ q)

5. Sea A el conjunto de los seres humanos.


Sean las siguientes proposiciones:
a) p : (∀h1 ∈ A) (∃!h2 ∈ A) (h2 es padre de h1)
b) q : (∃h3 ∈ A) (∀h4 ∈ A) (h3 es hermano de h4)
c) r : (∀h ∈ A) (h es mortal)
d) s : (∃h ∈ A) (h es mortal)
Escriba cada proposición en palabras y determine su valor de verdad.

6. Sea N el conjunto de los números naturales.


Sean las siguientes proposiciones:
a) p : (∀x ∈N ), (∃y ∈ N ) x + y > 10
b) q : (∀x ∈ N), (∀y ∈ N) x • y ∈ N
c) r : (∃x ∈ N), (∃y ∈ N) x • y = 3
d) s : (∃x ∈ N), (∀y ∈ N) x • y = y
Escriba cada proposición en palabras y determine su valor de verdad.

7. Escriba la negación de las cuatro proposiciones del ejercicio anterior. Determine su


valor de verdad, y si es falsa, muestre un contraejemplo.

8. Sea p (x): x es solución de la ecuación x2 – 4 = 0


Sea E = {– 2, 0, 2}
Determine el valor de verdad de las siguientes proposiciones:
a) p : ∀x ∈ E, p (x)
b) q : ∃x ∈ E, p (x)
c) r : ∃!x ∈ E, p (x)
d) s : ∀x ∈ E, ~ p (x)
e) t : ∃x ∈ E, ~ p (x)

9. Sean las siguientes proposiciones:


p : (∀x ∈ {1, 3, 5}) (∃y ∈ {2, 4, 6}), (x es mayor que y)
q : (∃x ∈ {1, 3, 5}) (∀y ∈ {2, 4, 6}), (x es mayor que y)
a) Escriba p y q en lenguaje corriente.
b) Determine el valor de verdad de p y q.
c) Escriba simbólicamente ~ p y ~ q.

10. Sean p (x) : x es mayor que – 3 y menor que 2


q (x) : x es mayor que – 1 y menor que 4
r (x) : x es mayor que 0 y menor que 1
s (x) : x es mayor que 3 y menor que 5

118 Relaciones y funciones

118-119. 118 08/11/2001, 15:57


CAPÍTULO 3

ParaPara
las siguientes proposiciones,
las siguientes determine
proposiciones, el valor
determine el de verdad
valor y dé yun ejemplo de las
de verdad
verdaderas.
dé un ejemplo de las verdaderas
a) (∃x ∈ N), p (x) ∧ q (x) e) (∀x ∈ N), ~ (r (x) ∧ s (x))
b) (∃x ∈ N), p (x) ∧ q (x) ∧ r (x) f) (∃x ∈ N), ~ (r (x) V s (x))
c) (∃x ∈ R), p (x) ∧ q (x) ∧ r (x) g) (∃x ∈ N), ~ p (x) ∧ r (x)
d) (∃x ∈ Z), p (x) V s (x) h) (∀x ∈ Z), ~ r (x)

Soluciones
1. a) proposición verdadera b) proposición verdadera
c) proposición falsa d) proposición falsa
e) no es proposición f) proposición falsa
g) no es proposición h) proposición verdadera
2. a) proposición b) proposición
c) proposición d) función proposicional
e) función proposicional f) función proposicional
g) proposición h) función proposicional
3
3. d) (∃ x ∈ Q) x =
4
e) (∀x ∈ Z) x se puede escribir como fracción
f) (∀ α < 90º) α es agudo
h) (∃x, ∃ y ∈ Z), x • y = 15
4. a) p q pVq ~(p V q) ~p ~q ~p∧~q
V V V F F F F
V F V F F V F
F V V F V F F
F F F V V V V

Comparando ambas columnas se ve que ~(p V q) ¤ ~p ∧ ~ q

b) p q p∧q ~(p ∧ q) ~p ~q ~pV~q


V V V F F F F
V F F V F V V
F V F V V F V
F F F V V V V

Comparando ambas columnas se ve que ~(p ∧ q) ¤ ~p V ~ q

c) p q p⇒q ~p ~pV q
V V V F V
V F F F F
F V V V V
F F V V V

Comparando ambas columnas se ve que (p ⇒ q) ¤ (~p V q)

Relaciones y funciones 119

118-119. 119 08/11/2001, 15:57


Soluciones

d) p T p VT p T p ∧T
V V V V V V
F V V F V F

p VT ¤T p∧T¤p

p C p VC p C p∧C
V F V V F F
F F F F F F

pVC¤p p∧C¤C

e) p pVp p p∧p
V V V V
F F F F

pVp¤p p∧p¤p

f) p q r p ∧ q (p ∧ q) ∧ r q∧r p ∧ (q ∧ r)
V V V V V V V
V V F V F F F
V F V F F F F
V F F F F F F
F V V F F V F
F V F F F F F
F F V F F F F
F F F F F F F

Comparando ambas columnas observamos que (p ∧ q) ∧ r ¤ p ∧ (q ∧ r)

p q r p V q (p V q) V r qVr p V (q V r)
V V V V V V V
V V F V V V V
V F V V V V V
V F F V V F V
F V V V V V V
F V F V V V V
F F V F V V V
F F F F F F F

Comparando ambas columnas observamos que (p V q) V r ¤ p V (q V r)

120 Relaciones y funciones

120-121.(2003) 120 20/11/02, 10:45 AM


CAPITULO 3
CAPÍTULO

g) p q p V q p⇒(pVq ) p q p ∧ q (p∧q)⇒p
V V V V V V V V
V F V V V F F V
F V V V F V F V
F F F V F F F V

Observando la última columna en cada caso se ve que


p ⇒ (p V q) y (p ∧ q) ⇒ p son siempre verdaderas.

h) p ~p ~(~p) i) p q p vq q v p p q p∧ q q ∧p
V F V V V V V V V V V
F V F V F V V V F F F
F V V V F V F F
F F F F F F F F
p ¤ ~ (~ p)
(p v q) ¤ (q v p) (p ∧ q) ¤ (q ∧ p)

j) p q r q∧ r p V q p Vr p V (q∧ r) (pVq) ∧ (p Vr)


V V V V V V V V
V V F F V V V V
V F V F V V V V
V F F F V V V V
F V V V V V V V
F V F F V F F F
F F V F F V F F
F F F F F F F F

p V (q ∧ r) ¤ (p V q) ∧ (p V r)

p q r q Vr p ∧ q p ∧ r p∧ (q V r) (p∧q)V(p∧r)
V V V V V V V V
V V F V V F V V
V F V V F V V V
V F F F F F F F
F V V V F F F F
F V F V F F F F
F F V V F F F F
F F F F F F F F

p ∧ (q V r) ¤ (p ∧ q) V (p ∧ r)

Relaciones y funciones 121

120-121.(2003) 121 20/11/02, 10:45 AM


Soluciones

k) l)
p q p⇒q ~q ~p ~q⇒~p p q p¤q ~p ~q ~p¤~q
V V V F F V V V V F F V
V F F V F F V F F F V F
F V V F V V F V F V F F
F F V V V V F F V V V V

(p ⇒ q) ¤ (~q ⇒ ~p) (p ¤ q) ¤ (~p ¤ ~q)

5. a) p: para todo hombre existe un único padre. Verdadero


b) q: existe un hombre que es hermano de todos los hombres. Falso
c) r: todo hombre es mortal. Verdadero
d) s: existe un hombre mortal. Verdadero

6. a) p: para todo número natural existe algún número natural tal que su suma es
mayor que 10. Verdadero
b) q: para todo par de números naturales, su producto es un número natural. Verdadero
c) r: existen dos números naturales tal que su producto es 3. Verdadero
d) s: existe un número natural tal que su producto con cualquier número natural da este
cualquier número natural. Verdadero

7. a) ~p : (∃ x ∈ N) (∀ y ∈ N) x + y ≤ 10. Falso
Si y = 10 no hay ningún natural que sumado con 10 sea menor que 10
b) ~q : ∃x ∈ N, ∃ y ∈ N, x • y F N. Falso
No existe ningún par de números naturales tales que su producto no sea
número natural
c) ~r : ∀x ∈ N, ∀y ∈ N, x • y π 3. Falso. 3 • 1 = 3
d) ~s : ∀x ∈ N, ∃y ∈ N, x • y π y. Falso. 1 • 5 = 5

8. a) p es falso, 0 no es solución de x2 – 4 = 0
b) q es verdadero, 2 es solución de x 2 – 4 = 0
c) r es falso, 2 y – 2 son soluciones
d) s es falso, 2 es solución de x 2 – 4 = 0
e) t es verdadero, 0 no es solución de x2 – 4 = 0

9. a) p: para cada x en {1, 3, 5} existe un y en {2, 4, 6} tal que x es mayor que y


q: existe x en {1, 3, 5} tal que para cada y en {2, 4, 6} x es mayor que y
b) p es falso (1 no es mayor que ningún y)
q es falso (no hay ningún x que sea mayor que todos los y)
c) ~p : (∃ x ∈ {1, 3, 5}) (∀ y ∈ {2, 4, 6}) (x ≤ y)
~q : (∀ x ∈ {1, 3, 5}) (∃ y ∈ {2, 4, 6}) (x ≤ y)

10. a) Verdadero, ejemplo: el 1


b) Falso.
1
c) Verdadero, ejemplo
2
d) Verdadero, ejemplo – 2
e) Verdadero, r (x) A s (x) es vacío

122 Relaciones y funciones

122-123.(2003) 122 20/11/02, 10:46 AM


CAPITULO 3
CAPÍTULO

f) Verdadero, ejemplo 2
g) Falso
h) Verdadero, todos los enteros son menores o iguales que 0 o
mayores o iguales que 1

Conjuntos 3.2

3.2.1 Conceptos básicos


Una teoría matemática se fundamenta y se va construyendo a
partir de esos fundamentos, encadenando los nuevos conocimientos
o proposiciones que se basan en las anteriores. Es así como se
parte de términos no definidos o conceptos fundamentales. Luego
hay proposiciones que relacionan estos conceptos fundamentales
y que son tan evidentes que se aceptan como verdaderas. Estas
proposiciones se denominan axiomas de la teoría. Siguiendo
con la construcción, aparecen las proposiciones, cuya veracidad
debe ser probada o demostrada. Son los llamados teoremas, que
según su importancia se pueden denominar: proposición, lema,
corolario o teorema.
En la teoría de conjuntos aceptamos como términos no definidos
las ideas de “conjunto”, “elemento” y “pertenencia”; son tres palabras
que usamos, que entendemos, pero que no definimos.
Los conjuntos se denotan por letras mayúsculas, sus elementos
por letras minúsculas y se escriben entre corchetes.
A = {a, e, i, o, u} B = {1, 2, 3, 4, 5, 6, 7, 8}
La relación de pertenencia se simboliza por ∈ y la negación
de ella es ∉.
Así a∈A Verdadero
i ∈B Falso
3∉ A Verdadero
5∉ B Falso
Los conjuntos se definen por extensión (nombrando todos
sus elementos) o por comprensión (indicando la característica
que poseen sus elementos y que no poseen los elementos que
no son del conjunto).
A = {0, 1, 2, 3, 4, 5, 6, 7, 8, 9} B = {x/x es dígito}
P = {–2, –1, 0, 1, 2} Q = {x∈Z / –3 < x <3}
A y P están definidos por extensión
B y Q están definidos por comprensión
Conjunto vacío: es el conjunto que no contiene elementos. En
símbolo {} o Δ.

Relaciones y funciones 123

122-123.(2003) 123 20/11/02, 10:46 AM


Conjunto universo: es el conjunto que contiene a todos los
elementos. Se puede definir de acuerdo con el contexto en que se esté
trabajando. Se denota por U.
Subconjunto: dado un conjunto no vacío A, se llama subconjunto
de A a todo conjunto B tal que todo elemento de B está en A.
Se anota B C A.
En símbolos:
B C A ¤ (∀x ∈ B) x ∈ A
El conjunto vacío es subconjunto de todo conjunto.
Conjunto potencia: se llama conjunto potencia de A y se denota
P(A) al conjunto cuyos elementos son todos los subconjuntos
del conjunto A.
P(A) = {B/B C A}
Ejemplo: si A = {1, 2, 3}
P(A) = { {1}, {2}, {3}, {1,2}, {1,3}, {2,3}, {1,2,3},Δ }
Se llama cardinalidad de un conjunto al número de elementos
que el conjunto tiene.
# A representa la cardinalidad de A.
En el ejemplo anterior # A = 3 y # P(A) = 8
En general, la cardinalidad del conjunto potencia de un conjunto
dado es igual a 2 elevado a la cardinalidad del conjunto. Es decir:
Si # A = n entonces # P(A) = 2n
Igualdad de conjuntos: dos conjuntos A y B son iguales si y sólo
si tienen los mismos elementos. En símbolos:
(A = B) ¤ (∀x ∈A), x ∈B ∧ (∀x ∈ B), x ∈A
En otras palabras y de acuerdo con la definición de subconjunto:
(A = B) ¤ (A C B) A (B C A)
Equivalencia de conjuntos: dos conjuntos son equivalentes si y
sólo si tienen el mismo número de elementos (igual cardinalidad).
En símbolos:
(A ~ B) ¤ (# A = # B)

Ejercicios 1. Escribir por extensión los siguientes conjuntos:


resueltos a) B = {x ∈ Z / –5 £ x £ 5}
b) C = {x / x es dígito del número 4.552.361}
Solución:
a) B = {–5, –4, –3, –2, –1, 0, 1, 2, 3, 4, 5}
b) C = {4, 5, 2, 3, 6, 1}

124 Relaciones y funciones

124-125. 124 08/11/2001, 16:09


CAPÍTULO 3

2. Escribir por comprensión los siguientes conjuntos:


a) A = {2, 4, 6, 8}
b) B =
{ 12 , 13 , 14 , 15 , 16 , 17 }
Solución:
a) A = {x ∈N / x es par ∧ x < 10}

b) B = {
1 / n ∈ N ∧ 2 ≤ n ≤ 7}
n
3. Hay conjuntos que no se pueden escribir por extensión porque
contienen infinitos elementos. Ejemplo:
A = {x ∈ R / –2 ≤ x ≤ 5}
Este tipo de conjuntos se puede escribir como intervalo y graficar como
subconjunto de R en la recta numérica. Ver cap. 2 (2.3.2).
Graficar los siguientes conjuntos en la recta numérica y escribirlos
como intervalo.
a) A = {x ∈ R / x > 3}
b) B = {x ∈ R / x ≥ –5}
c) C = {x ∈ R / x < 8}
d) D = {x ∈ R / x £ –1}
e) E = {x ∈ R / 0 £ x £ 5}
f) F = {x ∈ R / 2 < x < 3}
g) G = {x ∈ R / –1 £ x < 2}
h) H = {x ∈ R / –3 < x £ 1}

Solución:

A = (3, + ∞)
3
B = [–5, + ∞)
-5
C = (–∞, 8)
8
D = (–∞, –1]
-1
E = [0, 5]
0 5

F = (2, 3)
2 3

G = [–1, 2)
-1 2

H = (–3, 1]
-3 1

4. Dados los conjuntos A = {dígitos de 125} y B = {vocales de la


frase “hace frío”}

Relaciones y funciones 125

124-125. 125 08/11/2001, 16:09


Ejercicios Determinar el valor de verdad de las siguientes proposiciones:
resueltos Solución:
a) 2 ∈ A a) verdadero porque 2 pertenece a A
b) 5 ∈ B b) falso porque 5 no pertenece a B
c) i ∉ B c) falso porque i pertenece a B
d) u ∈ B d) falso porque u no pertenece a B
e) e ∈ B e) verdadero porque e pertenece a B
f) f ∈ B f) falso porque f no pertenece a B
g) 125 ∉ A g) verdadero porque 125 no pertenece a A
h) 1 ∉ A h) falso porque 1 pertenece a A
i) a ∉ A i) verdadero porque a no pertenece a A
5. Sea A = {2, 4, 6}. Encontrar todos los subconjuntos de A y formar
el conjunto potencia de A.
Solución:
Son subconjuntos de A los siguientes conjuntos:
{2}, {4}, {6}, {2,4}, {2,6}, {4,6}, A, Δ.
P(A) = {{2}, {4}, {6}, {2,4}, {2,6}, {4,6}, A, Δ.}
6. Dados los siguientes conjuntos, determinar cuáles de ellos son
iguales y cuáles son equivalentes.
A = {a,e,i,o,u}
B = {x ∈N / x ≤ 5}
C = {vocales de la frase “hace mucho frío”}
D = {x ∈N / x + 1 ≤ 6}
Solución:
#A = 5 , #B = 5 , #C = 5 , #D = 5
por lo tanto, todos son equivalentes, pero sólo son iguales
A=C y B=D
Nota: A = B ⇒ A ~ B, pero el recíproco no se cumple.

Ejercicios
1. Escriba por extensión los siguientes conjuntos:
a) A = {múltiplos de 5 menores que 40}
b) B = {divisores de 36}
c) C = {números primos menores que 20}
n
d) D = {x / x = , n ∈N, n ≤ 10}
n+1
e) E = {n2 + 1 / n ∈N}
f) F = {2n / n ∈N}
g) G = {2n – 1 / n ∈N}
h) H = {n(n + 1) / n ∈N, n < 5}
i) I = {factores primos de 36}

126 Relaciones y funciones

126-127. 126 08/11/2001, 16:10


CAPÍTULO 3

2. Escriba por comprensión los siguientes c) B⊂A


conjuntos: d) {∅} ⊂ A
e) {x ∈ N / –1 ≤ x ≤ 1} ⊂ B
a) A = {3, 6, 9, 12, 15}
f) ADB
b) B = 1, { 1 1 1 1
, , , ...
2 3 4 5
} g) BDA
c) C = {1, 3, 9, 27, 81...} h) {1, 2, 3} D A
d) D = {0, 3, 8, 15, 24, 35, 48...} i) A⊂ A
j) B⊂ B
{ 1 1 1
e) E = 1, , , , ...
2 4 8 16
1
}
6. Sea A = {x / x es divisor de 25}
f) F = {0, 1 2 3 4 5 6
, , , , , ... }
3 4 5 6 7 8 a) Encuentre todos los subconjuntos
g) G = {1, 2, 4, 8} de A.
h) H = {1, 2, 5, 10} b) Escriba el conjunto potencia de A.
i) I = {4, 8, 12, 16, 20, 24}
7. Sea M un conjunto tal que # M = 8
3. Grafique en la recta numérica cada ¿Cuántos subconjuntos de M se pueden
uno de los siguientes conjuntos y formar?
escríbalos como intervalo:
8. Sea la expresión “n|m” que significa que n
a) A = {x ∈R / x ≥ 3}
divide a m. Sean los conjuntos:
b) B = {y ∈R / y ≤ 5}
M = {x ∈ N / x | 10}
c) C = {x ∈R / x > –4 }
N = {x ∈ N / x | 5}
d) D = {x ∈R / x < 2}
P = {x ∈ N / x | 2}
e) E = {y ∈R / –1 < y < 7}
f) F = {x ∈R / 3 ≤ x ≤ 5} a) Escriba M, N y P por extensión.
g) G = {x ∈R / –5 < x ≤ 0} b) Determine cuáles son equivalentes.
h) H = {x ∈R / –3 ≤ x < 3} c) Determine el número de subconjuntos
de cada uno.
4. Dados los siguientes conjuntos: 9. Sean los siguientes conjuntos:
A = {factores de 36} A = {x ∈ R / –2 ≤ x ≤ 2}
B = {factores de 24} B = {x ∈ R / x ≤ 1}
C = {x ∈ R / x ≥ –1}
Determine el valor de verdad de las
siguientes proposiciones: a) Grafique A, B y C en la misma recta
numérica.
a) 6∈A∧ 6∉B b) Escriba A, B y C como intervalo.
b) 4∈ A∧ 4∈B c) Escriba el conjunto de elementos
c) 12 ∉ A V 12 ∈ B comunes a los tres conjuntos dados.
d) 9∈ A∧ 9∉B d) Escriba el conjunto de elementos
e) 36 ∈ A V 36 ∈ B comunes a A y B.
f) 8∉A∧8∉B e) Escriba el conjunto de elementos
g) 1∈ A∧ 1∈B comunes a A y C.
h) 1∈ AV1∉B f) Escriba el conjunto de elementos
5. Dados los conjuntos siguientes: comunes a B y C.
A = {x ∈N / x < 5} 10. Sean los siguientes conjuntos:
B = {x ∈Z / –2 ≤ x ≤ 2}
M = {x ∈ R / –2 ≤ x ≤ 2}
Determine el valor de verdad de las N = {x ∈ R / x > 2}
siguientes proposiciones:
Encuentre el conjunto de elementos
a) A C B
que pertenecen a M y a N.
b) ∅ C A

Relaciones y funciones 127

126-127. 127 08/11/2001, 16:10


Soluciones
1. a) A = {5, 10, 15, 20, 25, 30, 35} b) B = {1, 2, 3, 4, 6, 9, 12, 18, 36}
c)
e)
C
E
= {1, 2, 3, 5, 7, 11, 13, 17, 19}
= {2, 5, 10, 17, 26, 37, 50, 65,.....}
d) D = { 1 2 3 4 5 6 7 8 9 10
, , , , , , , , ,
2 3 4 5 6 7 8 9 10 11 }
g) G = {1, 3, 5, 7, 9, 11.....} f) F = {2, 4, 6, 8, 10.....}
i) I = {1, 2, 3} h) H = {2, 6, 12, 20}

1
2. a) A = {múltiplos de 3 menores que 18} b) B = { / n E N}
n
c) C = {3n / n E Z, n ≥ 0}
d) D = {n2 – 1 / n E N}
{ }
n
1
e) E = n E Z, n ≥ 0
2
g) G = {factores de 8} f) F = { nn +– 11 / n E N}
i) I = {múltiplos de 4 menores que 28} h) H = {factores de 10}

3. a) A = [3, + ∞) 3

b) B = (– ∞, 5]
5

c) C = (– 4, + ∞)
-4

d) D = (– ∞, 2)
2

e) E = (–1, 7)
-1 7

f) F = [3, 5]
3 5

g) G = (– 5, 0] -5 0

h) H = [ –3, 3)
-3 3

4. a) falso, porque 6 E B b) verdadero c) verdadero


d) verdadero e) verdadero f) falso, porque 8 E B
g) verdadero h) verdadero

5. a) falso, 3 E A A 3 F B b) verdadero c) falso, 0 E B A 0 F A


d) falso, Δ F A e) verdadero f) verdadero
g) verdadero h) falso, {1, 2, 3}C A i) verdadero
j) verdadero

6. a) {1}, {5}, {25}, {1,5}, {1,25}, {5,25}, A, Δ


b) P (A) = {{1}, {5}, {25}, {1,5}, {1,25}, {5,25}, A, Δ}

7. 256

8. a) M = {1, 2, 5, 10} N = {1, 5} P = {1, 2}


b) N B P
c) # P(M) = 16 , # P(N) = 4 , # P(P) = 4

128 Relaciones y funciones

128-129.(2003) 128 20/11/02, 10:48 AM


CAPITULO 3
CAPÍTULO

9. a) B A C d) {x E R /– 2 ≤ x ≤ 1}
-2 -1 1 2 e) {x E R /– 1 ≤ x ≤ 2}
b) A = [– 2, 2] ; B = (– ∞, 1] ; C = [– 1, + ∞)
c) {x E R /– 1 ≤ x ≤ 1} f) {x E R /– 1 ≤ x ≤ 1}
10. Δ

3.2.2 Operaciones entre conjuntos


Los conjuntos en general y sus operaciones suelen graficarse a
través de una figura llamada diagrama de Venn-Euler. A continuación
definiremos las operaciones más usuales y las graficaremos según el
diagrama de Venn-Euler. Consideraremos el conjunto Universal como
el rectángulo que contiene a todos los demás conjuntos.

Complemento: Dado un conjunto A, lla-


maremos complemento de A y lo denotaremos A
por A', Ac o A al conjunto que contiene A
a todos los elementos del universo que no
están en A.
A' = {x E U / x F A}

Unión de conjuntos: Dados dos conjuntos A


y B, se llama unión de A y B y se denota por
A K B al conjunto que contiene los elementos A B
de A, los elementos de B y los elementos que
están en ambos conjuntos.
A K B = {x / x E A V x E B}

Intersección de conjuntos: Dados dos


conjuntos A y B, se llama intersección de A y
B y se denota A J B al conjunto que contiene A B
a los elementos que están simultáneamente
en ambos conjuntos:
A J B = {x / x E A A x E B}

Dos conjuntos cuya intersección es vacía


se denominan conjuntos disjuntos.

Diferencia de conjuntos: Dados dos conjuntos A B


A y B, se llama Diferencia de A y B, se denota
A–B al conjunto que contiene a los elementos
de A que no están en B.
A – B = {x / x E A A x F B}
Otra forma de definir el complemento de A
es según la diferencia.
A' = U – A = {x / x E U A x F A}

Relaciones y funciones 129

128-129.(2003) 129 20/11/02, 10:49 AM


Diferencia simétrica de conjuntos:
Dados dos conjuntos A y B se llama di-
A B ferencia simétrica de A y B y se denota
A Δ B al conjunto que contiene a los elementos
que están en A y no están en B más los elementos
que están en B y no están en A.

A Δ B = {x / (x E A A x F B) V (x E B A x F A)}
A Δ B = (A – B) K (B – A)
Propiedades de la operaciones entre conjuntos.
Sean A, B, C conjuntos contenidos en el universo U.
1. (A')' = A
2. U' = ∅ ∅' = U
3. A – A = ∅ A–∅=A
4. A K ∅ = A AJ∅=∅
AKU=U AJU=A
AKA=A AJA=A Idempotencia
A K A' = U A J A' = ∅
5. (A K B) K C = A K (B K C)
(A J B) J C = A J (B J C) } Leyes de la
Asociatividad
6. A K B = B K A
AJB=BJA } Leyes de la
Conmutatividad
7. A K ( B J C) = (A K B) J (A K C)
A J ( B K C) = (A J B) K (A J C) } Leyes de la
Distributividad
8. (A K B)' = A' J B'
(A J B)' = A' K B' } Leyes de Morgan

Propiedades de la relación de inclusión entre conjuntos.


Sean A, B, C conjuntos en un universo U.
1. Si A C B entonces A K B = B A A J B = A
2. Si A C B entonces A–B= ∅
3. (I A E U) A C A prop. reflexiva
4. A C B A B C A fi A = B prop. antisimétrica
5. A C B A B C C fi A C C prop. transitiva
6. A C B fi A K C C B K C A
AJC C BJC
Nota: Las propiedades 3, 4 y 5 dan a la relación de inclu-
sión la característica de relación de orden (ver punto 3.3).
Observación: Si A y B son conjuntos disjuntos, entonces:
AJB=∅ ; A–B=A ; B–A=B

130 Relaciones y funciones

130-131. 130 08/11/2001, 15:17


CAPITULO 3
CAPÍTULO

1. Sean A = {1, 2, 3} , B = {3, 4, 5} y C = {4, 5, 6} Ejercicios


Hallar: a) AJB resueltos
b) AKB
c) AKBKC
d) AJBJC
e) A–B
f) BΔC

Solución:
a) A J B = {x / x E A A x E B} = {3}
b) A K B = {x / x E A V x E B} = {1, 2, 3, 4, 5}
c) A K B K C = {x / x E A V x E B V x E C} = {1, 2, 3, 4, 5, 6}
d) A J B J C = {x / x E A A x E B A x E C} = Δ
e) A – B = {x / x E A A x F B} = {1, 2}
f) A Δ B = {x / (x E A A x F B) V (x E B A x F A)} = {1, 2, 4, 5}

2. Sean A = {x E R / – 2 £ x £ 5} , B = {x E R / x ≥ 4}
C = {x E R / x < – 2}
Hallar a) AJB
b) BKC
c) A'
d) C'
e) (A J B)'
f) (A K B) – B
Solución:
Representemos los tres conjuntos en la recta numérica. U = R

C A B
-2 4 5
a) A J B = {x E R / 4 £ x £ 5}
b) B K C = {x E R / x < – 2 V x ≥ 4}
c) A' = {x E R / x < – 2 V x > 5}
d) C' = {x E R / x ≥ – 2}
e) (A J B)' = {x E R / x < 4 V x > 5}
f) (A K B) – B = {x E R / – 2 £ x < 4}

3. Sean P = {1, 2, 3, 4}, Q = {3, 4, 5, 6} considérese el universo


U = {0, 1, 2, 3, 4, 5, 6, 7, 8, 9}
Verificar que: a) (P K Q)' = P' J Q'
b) (P J Q)' = P' K Q'
Solución:
Tenemos que P' = {0, 5, 6, 7, 8, 9}
Q' = {0, 1, 2, 7, 8, 9}
a) P K Q = {1, 2, 3, 4, 5, 6} luego (P K Q)' = {0, 7, 8, 9}
P' J Q' = {0, 7, 8, 9} \ (P K Q)' = P' J Q'

Relaciones y funciones 131

130-131. 131 08/11/2001, 15:17


Ejercicios b) P J Q = {3,4} luego (P J Q)' = {0, 1, 2, 5, 6, 7, 8, 9}
resueltos P' K Q' = {0, 1, 2, 5, 6, 7, 8, 9} \ (P J Q)' = P' K Q'
4. En el diagrama de Venn-Euler adjunto verificar:
a) listar los elementos
A b) A J (B K C) = (A J B) K (A J C)
a B c) A J (B – C) = (A J B) – C
c f
Solución:
d b
i g a) A = {a, b, c, d} B = {b, c, e, g, i, f} C = {b, e, d, h, i, j, k}
h e b) B K C = {b, c, e, d, f, g, h, i, j, k}, luego
j k
C A J (B K C) = {b, c, d}
por otro lado
A J B = {b, c} y A J C = {b, d} entonces
(A J B) K (A J C) = {b, c, d}
\ A J (B K C) = (A J B) K (A J C)
c) B – C = {c, f, g}, luego A J (B – C) = {c}
por otro lado
A J B = {b, c}, luego (A J B) – C = {c}
\ A J (B – C) = (A J B) – C. Se hace notar que esta igualdad
no siempre es verdadera.
5. Mostrar en un diagrama de Venn-Euler que si A J B = Δ,
entonces A J B’ = A.
Solución:
A B En el diagrama se observa claramente que si se intersecta A con
B’, la resultante es A, ya que A C B’
6. Demostrar que A J B’ = A – B.
Solución:
Para demostrar igualdades de conjuntos podemos hacerlo por
doble inclusión, es decir:
1º A J B’ C A – B y 2º A – B C A J B’
1º Sea x E A J B’ fi x E A A x E B’ fi x E A A x F B fi x E (A – B)
\ A J B’ C A – B
2º Sea x E (A – B) fi x E A A x F B fi x E A A x E B’ fi x E (A J B’)
\ A – B C A J B’
Luego A J B’ = A – B.
7. Probar que (P K Q) J Q’ = P si y sólo si P J Q = ∅ .
Solución:
Aplicando las propiedades:

132 Relaciones y funciones

132-133. 132 08/11/2001, 16:14


CAPÍTULO 3

(P K Q) J Q’ = (P J Q’) K (Q J Q’) = (P J Q’) K ∅ = P J Q’


Luego el problema se transforma en probar que P J Q’ = P si y
sólo si P J Q = ∅.
Como es una bicondicionalidad, demostraremos primero la implicación
a la derecha y luego la implicación a la izquierda.
1º fi
Hip.: P J Q’ = P
Tesis: PJQ=∅
Dem.: P J Q’ = P fi P C Q’ fi P J Q = ∅
2º ‹
Hip.: PJQ=∅
Tesis: P J Q’ = P
Dem.: P J Q = ∅ fi P C Q’ fi P J Q’ = P
\ P J Q’ = P ¤ P J Q = ∅

8. Probar que A C B ¤ B’ C A’
1º fi
Hip.: ACB
Tesis: B’ C A’
Hip.
Dem.: Sea x E B’ fi x F B fi x F A fi x E A’
\ B’ C A’
2º ‹
Hip.: B’ C A’
Tesis: ACB
Hip.
Dem.: Sea x E A fi x F A’ fi x F B’ fi x E B
\ACB
Luego, de 1º y 2º A C B ¤ B’ C A’

9. Demostrar que la diferencia simétrica es conmutativa, es decir:


A Δ B = B Δ A.
Solución:
A Δ B = (A – B) K (B – A) = (B – A) K (A – B) = B Δ A.

10. Demostrar la asociatividad de la intersección:


(A J B) J C = A J (B J C).
Solución:
Demostraremos por doble inclusión:
1º Por dem.: (A J B) J C C A J (B J C).

Relaciones y funciones 133

132-133. 133 08/11/2001, 16:14


Ejercicios Sea x E (A J B) J C fi x E A J B ∧ x E C
fi (x E A ∧ x E B) ∧ x E C
resueltos
fi x E A ∧ (x E B ∧ x E C)
fi x E A ∧ x E (B J C)
fi x E A J (B J C)
∴ (A J B) J C C A J (B J C)
2° Por dem.: A J (B J C) C (A J B) J C
Sea x E A J (B J C) fi x E A ∧ x E (B J C)
fi x E A ∧ (x E B ∧ x E C)
fi (x E A ∧ x E B) ∧ x E C
fi x E (A J B) ∧ x E C
fi x E (A J B) J C
∴ A J (B J C) C (A J B) J C
Luego, de 1 y 2 tenemos A J (B J C) = (A J B) J C

Ejercicios
1. Sean A = { 1, 3, 5, 7} c) A J (B K C)
B = { 1, 3, 5, 7, 9} d) (A K B) J C
C = { 2, 4, 6, 8} e) (A K C) J (B K C)
Considere f) (A J C) K (B J C)
U = {0, 1, 2, 3, 4, 5, 6, 7, 8, 9}
Determine: 4. Sean A = { x ∈ R / 2 £ x £ 4}
a) A K B e) A – B i) (A J B)' B = { x ∈ R / 3 £ x £ 6}
C = { x ∈ R / x ≥ 5}
b) A J C f) C – B j) C' J A
Encuentre:
c) A K C g) A Δ B k) (A – C)'
a) A Δ B d) B – (A K C)
d) B J C h) A' l) (B K C)'
b) B – C e) B – (A J C)
2. Sean P = { x ∈ R / 1 < x £ 5} c) (A K B) – C f) A Δ C
Q = { x ∈ R / x ≥ –2 }
R = { x ∈ R / –4 ≤ x < 1} 5. Sean P = [–6 , 0], Q = [–3 , 4] y
R = [3, 5] subconjuntos de R.
Determine: Determine:
a) PJQ e) RKP i) Q–P a) (P K R)'
b) QJR f) RKQ j) R–P b) (P J Q)'
c) RJP g) PJQJR k) R–Q c) (P K Q K R)'
d) (P – Q) J R'
d) PKQ h) P–Q l) Q–R
e) (R – Q) J P'
3. Sean A = {x ∈ R / –6 ≤ x ≤ –2} f) (Q – P) Δ (Q – R)
B = {x ∈ R / 1 ≤ x ≤ 6} 6. Sean U = {n ∈ N / n < 10}
C = {x ∈ R / |x| ≤ 7}
A = {n ∈ U / n < 5}
Encuentre: B = {n ∈ U / 4 < n < 8}
a) (A J B) K C b) (A K C) J B C = {n ∈ U / 3 < n < 10}

134 Relaciones y funciones

134-135. 134 08/11/2001, 16:16


CAPÍTULO 3

a) Liste los elementos de A, B, C y U 9. Muestre en un diagrama de Venn-Euler y


b) Haga un diagrama de Venn-Euler dé un ejemplo en que se vea que si
c) Verifique que (A J B)’ = A’ K B’ A C B entonces B’ C A’.
d) Verifique que
10. Demuestre que A J B = B – A’
(A J B) J C = A J (B J C )
7. En una encuesta en la Región Metropo- 11. Demuestre que si
litana se consulta a 1.000 personas y A J B = A J C A A K B = A K C,
entonces B = C
se obtiene que:
990 personas hablan castellano 12. Demuestre que si P C Q, entonces
626 personas hablan inglés PJRCQJR
134 personas hablan francés
620 personas hablan castellano e inglés 13. Demuestre que A – B = B’ – A’
130 personas hablan castellano y francés 14. Demuestre que si A C B, entonces
100 personas hablan los tres idiomas A K (B – A) = B
a) Haga un diagrama de Venn-Euler
que refleje la situación. 15. Demuestre que P – Q C P K Q
b) ¿Cuántas personas hablan sólo 16. Haga un diagrama de Venn-Euler para
castellano?, ¿cuántas sólo inglés? y los conjuntos no vacíos A, B y C que
¿cuántas sólo francés? cumplan las siguientes propiedades:
c) ¿Cuántas hablan inglés y no hablan a) A C B, B C C
francés? b) A C B, C C B, A J C ≠ ∅
d) ¿Cuántas hablan inglés o francés? c) A J C = ∅, B J C ≠ ∅ , A J B ≠ ∅
8. En el siguiente diagrama de Venn- Euler d) A C B, C J B ≠ ∅ , C J A ≠ ∅,
CDB
verifique que:
a) (A J B) – D = (B – D) J A 17. Demuestre que A – (A – B) = A J B
b) (B K C)' = D – (B K C)
c) (B K C) – (A K D) = (A K D)' 18. Demuestre que
A J (B – C) = (A J B) – (A J C)
B C
3 5 19. Demuestre que
1 6 7
A 4 A J (B K C) = (A J B) K (A J C)
9
11 12
10 8
20. Demuestre que
2 D A K (B J C) = (A K B) J (A K C)

Soluciones

No se incluye la respuesta de las demostraciones.


1. a) A K B = {1, 3, 5, 7, 9} = B b) A J C = ∅ c) A K C = {1, 2, 3, 4, 5, 6, 7, 8}
d) B J C = ∅ e) A – B = ∅ f) C – B = {2, 4, 6, 8} = C g) A Δ B = {9}
h) A' = {0, 2, 4, 6, 8, 9} i) (A J B)' = {0, 2, 4, 6, 8, 9} j) C' J A = {1, 3, 5, 7}
k) (A – C)' = {0, 2, 4, 6, 8, 9} l) (B K C)' = {0}
2. a) P porque P C Q b) {x E R / – 2 £ x < 1} c) Δ (1 F R A 1 F P)
d) Q porque P C Q e) {x E R / – 4 £ x £ 5 A x ≠ 1} f) {x E R / x ≥ - 4}

Relaciones y funciones 135

134-135. 135 08/11/2001, 16:16


Soluciones
g) Δ h) Δ porque P C Q i) {x E R / – 2 ≤ x ≤ 1 V x > 5}
j) R porque R y P son disjuntos k) {x E R / – 4 ≤ x < – 2} l) {x E R / x ≥ 1}
3. a) C b) B c) A d) A K B e) C f) A K B
4. a) [2, 3) K (4, 6] b) [3, 5) c) [2, 5) d) (4, 5) e) [3, 6] = B
f) A K C, ya que A J C = Δ
5. a) (– •, – 6) K (0, 3) K (5, + •) b) (– •, – 3) K (0, + •)
c) (– •, – 6) K (5, + •) d) [– 6, – 3) e) (4, 5] f) [– 3, 0] K [3, 4]
6. a) A = {1, 2, 3, 4} b) U
A 1 2
B = {5, 6, 7}
C = {4, 5, 6, 7, 8, 9} 3 4 75
6 B
U = {1, 2, 3, 4, 5, 6, 7, 8, 9} C 89
7. a) Los números representan la C
cardinalidad de cada conjunto. J
520 6
340
b) Sólo castellano 340, sólo inglés 100
6 y sólo francés 4. 30
c) 526 4
d) 660 F

8. a) ambos son el conjunto {1, 3} b) ambos son el conjunto {2}


c) ambos son el conjunto {5, 6, 7, 12}
9. Ej.: Sea U = {0, 1, 2, 3, 4, 5}
B'
B A = {1, 2, 3} B = {1, 2, 3, 4}
A
A' = {0, 4, 5} B' = {0, 5}
A'
aquí claramente A C B A B' C A'
16. a) b) B c) d) B
A C B
A B A
C A C
C

3.3 Relaciones

3.3.1 Conceptos básicos


Sean A y B conjuntos no vacíos. Se define la operación producto
cartesiano de los conjuntos A y B que se denota A ¥ B al conjunto
de pares ordenados.
A × B = {(a, b) / a E A A b E B}
Una relación R de un conjunto A en un conjunto B es un
subconjunto del conjunto A × B.

136 Relaciones y funciones

136-137. 136 08/11/2001, 16:19


CAPÍTULO 3

(R relación de A en B) ¤ R C A × B
• Observación 1: Una relación es un conjunto de pares ordenados.
• Observación 2: Una relación R de A en B se denota R : A Q B.
• Observación 3: (a, b) E R ¤ a R b
(a, b) F R ¤ a R b

Sea R : A Q B una relación y (a, b) E R


1. a se denomina preimagen.
2. b se denomina imagen de a según la relación R. Se denota
b = R (a).
3. Im. (a) es el conjunto de todas las imágenes del elemento a.
Sea R : A Q B una relación. Se denomina.
Dominio de la relación al conjunto de todas las preimágenes.
Dom. R = {a E A / H b E B A (a, b) E R } C A
Rango o recorrido de la relación, al conjunto de todas las
imágenes.
Rang. R = {b E B / H a E A A (a, b) E R} C B.
Una relación se puede graficar usando un sistema cartesiano o
un diagrama sagital (diagrama de flechas).
Si A = {a, b, c, d, e} B = {a, b, c, d}
R : A Q B, R = {(a, a), (a, b), (b, b), (b, c), (d, c), (e, d)}

GRÁFICO CARTESIANO GRÁFICO SAGITAL


B A R B
d
a
c a
b b
b
c
a c
d
d
e
a b c d e A

Si la relación se define en conjuntos numéricos, sus gráficos


pueden ser figuras geométricas.
Cada relación R : A Q B tiene una relación inversa R –1 : B Q A
R–1 = {(b, a) / (a, b) E R}

1. Sean A = {1, 2, 3, 4} y B = {1, 5, 3}. Escribir tres relaciones Ejercicios


de A en B. resueltos
Se tiene que A ¥ B tiene cardinalidad 12, luego hay
12
2 = 4.096 subconjuntos de A ¥ B y por lo tanto el mismo
número de relaciones que se pueden formar. Tenemos que:

Relaciones y funciones 137

136-137. 137 08/11/2001, 16:19


Ejercicios R : A Q B = {(a, b) / a E A A b E B}
resueltos R1 = {(1, 1), (1, 5), (1, 3)}
R2 = {(1, 1), (2, 5), (3, 3), (3, 5), (4, 3)}
R3 = {(4, 5), (3, 5), (2, 5), (1, 5)}
y así podríamos formar 4.093 relaciones distintas a estas tres.

2. Sean A = {1, 3, 5, 7} y B = {2, 4, 6}. Sea R : A Q B una relación definida


por R = {(x, y), / y = x + 1}. Escribir R por extensión.

R está formada por todos los pares (x, y) de A ¥ B tales que su segunda
coordenada y es igual a la primera coordenada x más 1.

R = {(1, 2) (3, 4) (5, 6)}

3. Sea R : Z Q Z una relación definida por

R (x) =
{ x–1
3
x+1
si
si
si
x>0
x=0
x<0
Hallar R (2), R (0), R (– 3)
Ésta es una relación definida a tramos, es decir, tiene una fórmula para
las imágenes de los números mayores que 0 y otra para los números
menores que 0. La imagen de 0 está dada y es 3. Así:
R (2) = 2–1 =1 (2 > 0)
R (0) = 3
R (– 3) = –3+1=–2 (– 3 < 0)

4. Sea P una relación de N Q N definida por


P = {(x, y) / y > x}
Hallar las imágenes de dos elementos cualesquiera del dominio.
Ésta es una relación donde la imagen “y” debe cumplir con
ser mayor que la preimagen “x”, luego, cada “x” tiene infinitas
imágenes:
Im 1 = {2, 3, 4, 5, ...... }
Im 10 = {11, 12, 13, 14, 15, ...... }

5. Dada la relación R = {(1, 5) (5, 1) (3, – 2) (4, 1)}


Hallar: a) Dom. R b) Rang. R
a) Dom. R = {x / x es preimagen}
= {1, 5, 3, 4}
b) Rang. R = {y / y es imagen}
= {5, 1, – 2}
6. Sea R : N Q N una relación definida por
R = {(x, y) / x + 2 y – 10 = 0}
Hallar: a) Dom. R. b) Rang. R.
Primero escribamos R por extensión. ¿Cuáles son los pares ordenados
de números naturales que cumplen con que la primera coordenada
“x” más dos veces la segunda “y” menos diez es cero?

138 Relaciones y funciones

138-139. 138 08/11/2001, 16:21


CAPÍTULO 3

x = 10 – 2y
si y = 1 Q x = 8 Q (8, 1) E R
y = 2 Q x = 6 Q (6, 2) E R
y = 3 Q x = 4 Q (4, 3) E R
y = 4 Q x = 2 Q (2, 4) E R
y=5Qx=0 que no es natural
Luego R = {(8, 1) (6, 2) (4, 3) (2, 4)}
Si hubiéramos despejado y en la fórmula que define R tendríamos:
10 – x
y=
2
9
Si x = 1 Q y = 2
FN
x = 2 Q y = 4 Q (2, 4) E R
7
x=3Qy= 2
FN
x = 4 Q y = 3 Q (4, 3) E R
5
x=5Qy= FN
2
x = 6 Q y = 2 Q (6, 2) E R
3
x=7Qy= 2
FN
x = 8 Q y = 1 Q (8, 1) E R
1
x=9Qy= 2
FN
x = 10 Q y = 0 F N
y no hay más pares en R
Luego R = {(8, 1) (6, 2) (4, 3) (2, 4)}
Así Dom. R = {8, 6, 4, 2}
Rang. R = {1, 2, 3, 4}

7. Sea M : A Q B una relación definida por


M = {(3, 2) (0, 1) (3, 1) (1, 1) (2, 4)}
Hallar a) A b) B c) graficar en un diagrama sagital
d) graficar en un diagrama cartesiano.
a) A = {3, 0, 1, 2}
b) B = {2, 1, 4}
c) M d)
A B
4
0 1 3
1 2
2 1
2
3 4
0 1 2 3

Relaciones y funciones 139

138-139. 139 08/11/2001, 16:21


Ejercicios 8. Dadas las siguientes relaciones definidas gráficamente, escribirlas
resueltos señalando sus pares por extensión.
a) b)

R Y
A B
6
1 5
a
3 4
b
3
c 5 2
1

1 2 3 4 5 6 7 8 9 10 X
Solución:
a) R : A Q B R = {(a, 1), (a, 3), (b, 3), (c, 5)}
b) S : X Q Y S = {(2, 1), (2, 3), (3, 3), (4, 4), (5, 2), (7, 4), (8, 6)}

Ejercicios
1. Sea A = {p, c, d, m} y B = {b, a} 5. En la siguiente figura
Se define T : AQ B tal que Abuelo - Abuela Abuelo - Abuela
T = {(u, v) / u E A A v E B} Aníbal Carmen Felipe Tamara
Determine si los siguientes pares perte- Óscar María
necen o no a la relación T.
José Ruth Luis Rosa Hugo
a) (p, b) d) (a, d)
b) (m, a) e) (c, a) Se definen las relaciones siguientes:
c) (b, c) f) (m, b) a) R1 : “ser padre de”
b) R2 : “ser hijo de”
2. Sea S = {– 8, – 7, – 6, – 5, – 3, – 2, 1, 4, 7, 8}
y T = {– 8, – 7, – 6, – 3, – 2} c) R3 : “ser hija de”
d) R4 : “ser esposo de”
Se define la relación R : S Q T
Escriba cada relación por extensión.
tal que R = {(x, y) / y = x – 1}
Escriba R por extensión. 6. Sea H : Z Q Z una relación definida por

3. Sea A = {b, s, i, h} y B = {n}


Sea T : A Q B
una relación definida por
T = {(u, v) / u E A A v E B}
H (x) =

Encuentre:
{ –5
–1–x
– 5x + 4
si x < – 2
si x = – 2
si x > – 2

Escriba T por extensión.


H (10), H (6), H (0), H (– 2), H (– 10)
4. Sea P = {– 5, – 9, – 4, – 8} y
7. Sea H : Z Q Z definida por:
Q = {2, – 2, 4, 1, 3, 5}
Se define S : P Q Q
tal que S = {(u, v) / u – 2 = v}
Escriba S por extensión.
H (x) =
{ –7
–1–x
–7x–4
si x < – 2
si x = – 2
si x > – 2

140 Relaciones y funciones

140-141. 140 08/11/2001, 16:22


CAPÍTULO 3

Determine: 17. Dada la relación


H (- 14), H (- 6), H (1), H (0), H (3) F = {(0, 3), (- 3, - 6), (3, - 1), (0, 0), (1, 6)}
8. Sea H : Z Q Z una relación definida por Determine:

{
4 si x < 3 a) Dom F b) Rang F
H (x) = 4-x si x = 3
4x + 5 si x > 3 18. Dada la relación
Determine: F = {(3, 3), (- 8, - 8), (- 5, 2), (0, 6), (- 6, 8)}
H (14), H (- 5), H (1), H (3), H (0) Determine:
a) Dom F b) Rang F
9. Sea R : Z Q Z una relación definida por
R = {(x, y) / y = 4x} Busque: 19. Sea F = {(- 6 , 3), (- 7, 8), (- 5, - 7), (0, - 12),
R (- 11), R (6), R (3), R (0), R (1) (- 5, - 8)} una relación.
Determine:
10. Sea P : R Q R una relación definida por a) Dom F b) Rang F

{
P = (x, y) / y = 6x – 7
4 }
Determine: 20. Sea R : N Q N una relación definida por
P (0), P (5), P (- 1), P (3), P (- 2) R = {(u, v) / 2u + 3v - 12 = 0}
Determine:
11. Sea R : Z Q Z una relación definida por a) Dom R b) Rang R
R = {(x, y) / y = 2x} Halle:
R (- 7), R (3), R (14), R (0), R (5) 21. Sea R : N Q N una relación definida por
R = {(s, t) / 3t + 3s = 12}
12. Sea G : N Q N una relación definida Halle:
por G = {(x, y) / y < x} a) Dom R b) Rang R
Encuentre:
a) una imagen de 7 22. Sea R : N Q N una relación definida por
b) una imagen de 20
c) una imagen de 1
{
R = (p, q) / q =
10 – 3p
2 }
Determine:
13. Sea G : N Q N una relación definida a) Dom R b) Rang R
por G = {(x, y) / y < x}
23. Escriba las siguientes relaciones por
Encuentre:
extensión:
a) Im (4) b) Im (14) c) Im (10)

14. Sea S : Z Q Z una relación definida por a) R


S = {(- 4, - 2), (- 2, - 4), (- 4, - 5), A B
(- 5, - 4), (- 4, 1), (1, - 4)} r 1
Determine: q 3
a) Im (- 4) b) Im (- 5) c) Im (- 2) p 5
u
0
15. Sea S : Z Q Z una relación definida por
S = {(6, 8), (8, 6), (6, 3), (3, 6), (6, 2),
b) T
(2, 6)} E F
Encuentre: Im (6)
9
16. Sea S : Z Q Z una relación definida por
3
S = {(3, 5), (5, 3), (3,8), (8,3), (3,7), (7,3)} 0
6
Encuentre: Im (3) 0

Relaciones y funciones 141

140-141. 141 08/11/2001, 16:22


Ejercicios a) R = {(x, y) / x2 + y2 = 25}
b) S = {(x, y) / x – y = 5}
c) N c) T = {(x, y) / 2x – y = 1}
5
4 26. Determine la relación inversa R–1 dada
3 • • la relación R.
2 •
a) R = {(1, 2), (1, 3), (1,4), (2,1),
1 •
(2, 2), (2, 3)}
1 2 3 4 5 6 7 N b) R = {(a, b), (a, c), (a, a)}
c) R = {(1, 1), (2, 2), (3,3), (4,4),
(3, 5), (5, 3)}
d) N
5 27. Determine la relación inversa R–1 dada
4 • • • • la relación R.
3 • • a) R = {(x, y) E N ¥ N / x = 2y}
2 •
b) R = {(x, y) E N ¥ N / x + 2y = 22}
1 •
c) R = {(x, y) E R ¥ R / y = 4x – 2}
1 2 3 4 5 6 7 N d) R = {(x, y) E R ¥ R / x + y – 4 = 0}
28. Dado el conjunto U = {– 2, 2, 4}
24. Grafique las siguientes relaciones de N
Halle la relación inversa R–1 de cada
en N usando un gráfico cartesiano.
relación R en U y proceda a listarla.
a) R = {(x, y) / x + y = 9}
a) R = {(x, y) / x ≠ y}
{
b) S = (x, y) / y =
10 – x
2
} b) R = {(x, y) / x es múltiplo de y}
c) T = {(x, y) / 5y + x = 26} c) R = {(x, y) / x es la mitad de y}
d) R = {(x, y) / x mayor o igual que y}
25. Grafique las siguientes relaciones reales
e) R = {(x, y) / y = x }
en un gráfico cartesiano.

Soluciones
1. a) sí b) sí c) no d) no e) sí f) sí
2. R = {(– 6, – 7), (– 2, – 3), (– 5, – 6), (– 7, – 8)}
3. T = {(b, n), (s, n), (i, n), (h, n)}
4. S = Δ. No hay ningún par de P ¥ Q que satisfaga la relación definida.
5. a) R1 = {(Aníbal, Óscar), (Felipe, María), (Óscar, José), (Óscar, Ruth), (Óscar, Luis),
(Óscar, Rosa), (Óscar, Hugo)}
b) R2 = {(Óscar, Aníbal), (Óscar, Carmen), (José, Óscar), (José, María), (Luis, Óscar),
(Luis, María), (Hugo, Óscar), (Hugo, María)}
c) R3 = {(María, Felipe), (María, Tamara), (Ruth, Óscar), (Ruth, María), (Rosa, Óscar),
(Rosa, María)}
d) R4 = {(Aníbal, Carmen), (Felipe, Tamara), (Óscar, María)}
6. H (10) = – 46 H (6) = – 26 H (0) = 4 H (– 2) = 1 H (– 10) = – 5
7. H (– 14) = – 7 H (– 6) = – 7 H (1) = – 11 H (0) = – 4 H (3) = – 25
8. H (14) = 61 H (– 5) = 4 H (1) = 4 H (3) = 1 H (0) = 4
9. R (– 11) = – 44 R (6) = 24 R (3) = 12 R (0) = 0 R (1) = 4

142 Relaciones y funciones

142-143. 142 08/11/2001, 16:24


CAPÍTULO 3

7 23 13 11 19
10. P (0) = – 4
P (5) = 4
P (– 1) = – 4
P (3) = 4
P (– 2) = – 4

11. R (– 7) = – 14 R (3) = 6 R (14) = 28 R (0) = 0 R (5) = 10


12. a) cualquier número natural menor que 7. Ejemplo el 5.
b) cualquier número natural menor que 20. Ejemplo el 3.
c) cualquier número natural menor que 1. No hay.
13. a) {3, 2, 1} b) {13, 12, 11, 10, 9, 8, 7, 6, 5, 4, 3, 2, 1} c) {9, 8, 7, 6, 5, 4, 3, 2, 1}
14. a) {– 2, – 5, 1} b) {– 4} c) {– 4}
15. {8, 3, 2}
16. {5, 8, 7}
17. a) {– 3, 0, 1, 3} b) {3, – 6, – 1, 0, 6}
18. a) {3, – 8, – 5, 0, – 6} b) {3, – 8, 2, 6, 8}
19. a) {– 6, – 7, – 5, 0} b) {3, 8, – 7, – 12, – 8}
20. a) {3} b) {2}
21. a) {1, 2, 3} b) {1, 2, 3}
22. a) {2} b) {2}
23. a) {(r, 0), (q, 1), (p, 3), (u, 5)} b) {(9, 0), (3,0), (6, 0), (0, 0)}
c) {(1, 3), (2, 2), (3, 1), (3, 3)} d) {(1,1), (1, 2), (1, 3), (2, 3), (2, 4), (3, 4), (4, 4), (5, 4)}

24. a) b)
8 • 8
7 • 7
6 • 6
5 • 5
4 • 4 •
3 • 3 •
2 • 2 •
1 • 1 •

1 2 3 4 5 6 7 8 9 1 2 3 4 5 6 7 8 9

c)
5 •
4 •
3 •
2 •
1 •

1 2 3 4 5 6 7 8 9 10 11 12 13 14 15 16 17 18 19 20 21

25. a) y b) c) y
y
5
1
–– x
5 x 2

–5 5 x –1

–5
–5

Relaciones y funciones 143

142-143. 143 08/11/2001, 16:24


Soluciones
26. a) R– 1 = {(2, 1), (3, 1), (4, 1), (1, 2), (2, 2), (3, 2)}
b) R– 1 = {(b, a), (c, a), (a, a)}
c) R– 1 = R
27. a) R– 1 = {(x, y) E N ¥ N / x es la mitad de y}
22 – y
b) {
R– 1 = (x, y) E N ¥ N / x = 2 }
R– 1 = {(x, y) E R ¥ R / y = }
x+2
c) 4

d) R– 1 = {(x, y) E R ¥ R / y = 4 – x} = R
28. a) R– 1 = {(x, y) / x ≠ y} = {(– 2, 2), (– 2, 4), (2, – 2), (2, 4), (4, – 2), (4, 2)} = R
b) R– 1 = {(x, y) / x es factor de y} = {(2, 2), (2, – 2), (2, 4), (– 2, 4), (– 2, – 2), (4, 4)}
c) R– 1 = {(x, y) / x es el doble de y} = {(4, 2)}
d) R– 1 = {(x, y) / x ≤ y} = {(– 2, – 2), (– 2, 2), (2, 2), (2, 4), (4, 4), (– 2, 4)}
e) R– 1 = {(x, y) / y = x2} = {( 2, 4), (– 2, 4)}

3.3.2 Relación de equivalencia y de orden


Una relación definida de un conjunto A en sí mismo se denomina
relación en A.

(R relación en A) ¤ R C A ¥ A
• Observación 1
A El gráfico sagital de este tipo de relaciones se hace dibujando una
sola vez el conjunto y uniendo mediante flechas los elementos
a relacionados.
b
c Si R = {(a, b) (a, c) (a, a)} es una relación definida en el conjunto
A = {a, b, c}, su gráfico sagital es el adjunto.
• Observación 2
Una relación definida en el conjunto R se denomina relación real
y se grafica en el sistema cartesiano.

Propiedades de una relación en A

Sea R una relación en A.


(R es refleja) ¤ (I x E A, x R x)
(R es simétrica) ¤ (x R y fi y R x)
(R es antisimétrica) ¤ ((x R y A y R x) fi x = y)
V ((x ≠ y A x R y) fi y R x)
(R es transitiva) ¤ ((x R y A y R z) fi x R z)
• Observación 3
Una relación R en A se llama relación de equivalencia si y sólo si
R es refleja, simétrica y transitiva.

144 Relaciones y funciones

144-145. 144 08/11/2001, 16:26


CAPÍTULO 3

• Observación 4.
Una relación R en A se llama relación de orden si y sólo si R es
refleja, antisimétrica y transitiva.
Sea R una relación de orden en un conjunto A. Sean “x” e “y”
elementos de A tal que (x, y) ∈R (x R y). En este caso se dice que
“x” e “y” son comparables según la relación R.
Si en un conjunto A todos los pares de elementos son comparables
según una relación de orden R, el conjunto A se dice totalmente
ordenado y la relación R se dice de orden total. Si al menos un
par de elementos de A son no comparables, entonces A se dice
parcialmente ordenado y R es una relación de orden parcial.

1. Sea A = {1, 2, 3, 4, 5} y R la relación. Ejercicios


R = {(1, 2) (1, 3) (3, 1) (2, 3) (2, 1) (1, 1) (2, 2) (3, 3) (4, 4) (5, 5) resueltos
(3, 4) (1, 4) (2, 4)}
Determinar si R es refleja, simétrica, antisimétrica y/o transitiva.
a) R es refleja porque I x E A; x R x
b) R no es simétrica porque 2 R 3 y 3 R 2
c) R no es antisimétrica porque 1 R 3 A 3 R 1 pero 3 ≠ 1
d) R no es transitiva porque 3 R 1 A 1 R 2 pero 3 R 2
2. Sea A = {alumnos de un colegio}. Se define la relación R en A tal
que: x R y ¤ x está en el mismo curso que “y”. Probar que R es
una relación de equivalencia en A.
a) R es refleja porque x está en el mismo curso que x. Esto se
cumple para todos los alumnos del colegio.
b) R es simétrica porque si x está en el mismo curso que y, esto
implica que y está en el mismo curso que x.
c) R es transitiva porque si x está en el mismo curso que y, e y
está en el mismo curso que z, entonces tenemos que x está en
el mismo curso que z.
Observaciones:
R clasifica a todos los alumnos del colegio en diferentes cursos.
Cada curso es una clase de equivalencia.
Esta es una clasificación porque:
a) Cada alumno está en un curso.
b) Ningún alumno está en dos cursos.
c) Todo curso tiene algún alumno.
3. Sea A = {1, 2, 3, 4, 5} y R la relación en A definida por
R = {(a, b) / a – b es múltiplo de 2}
a) Listar los elementos de R.
b) Demostrar que R es relación de equivalencia.

Relaciones y funciones 145

144-145. 145 08/11/2001, 16:26


Ejercicios c) Se llama clase de a y se denota [a] al conjunto:
[a] = {x / (x, a) ∈R}
resueltos Εscribir la clase de cada elemento de A.
d) Escribir la clasificación que la relación R determina en el
conjunto A.
Solución:
a) R = {(1, 1) (2, 2) (3, 3) (4, 4) (5, 5) (1, 3) (3, 1) (1, 5) (5, 1) (2, 4)
(4, 2) (3, 5) (5, 3)}
b) R es refleja porque (a, a) ∈ R ∀ a ∈ A.
R es simétrica porque si a–b es múltiplo de 2, b – a también lo
es.
R es transitiva porque:
a – b múltiplo de 2 ⇒ a – b = 2n
b – c múltiplo de 2 ⇒ b – c = 2m
sumando: a – c = 2 (n + m)
∴ a – c, también es múltiplo de 2. Luego, por ser refleja, simé-
trica y transitiva, R es relación de equivalencia.
c) [1] = {x ∈ A / (x, 1) ∈ R} = {1, 3, 5}
[2] = {x ∈ A / (x, 2) ∈ R} = {2, 4}
[3] = {x ∈ A / (x, 3) ∈ R} = {3, 1, 5}
[4] = {x ∈ A / (x, 4) ∈ R} = {4, 2}
[5] = {x ∈ A / (x, 5) ∈ R} = {5, 1, 3}

d) Se observan dos clases:

1
2
2
4
5

[1] = [3] = [5] y [2] = [4]


Nota: Cualquier elemento es representativo de su clase.

Antes de resolver los siguientes ejercicios diremos que una relación


de orden se llama así porque ordena los elementos del conjunto
donde se define:
Representaremos el orden según la relación R en un diagrama así:
y cuando (x, y) ∈R
x c
b
cuando (a, b), (b, c), (a, c) ∈R
a
4. Sea A = {a, b, c, d, e} y R la relación en A definida por:
R = {(d, b), (d, a), (b, e), (b, c), (a, c), (d, c), (d, e), (a, a), (b, b),
(c, c), (d, d), (e, e)}
a) Determinar si R es una relación de orden.
b) Si R es relación de orden hacer el diagrama de orden.

146 Relaciones y funciones

146-147..indd 146 01-02-2006 16:20:10


CAPÍTULO 3

Solución:
a) R es refleja porque x R x ∀ x E A.
R es antisimétrica porque si x ≠ y ∧ (x, y) ∈R entonces ( y, x) FR.
R es transitiva porque (x, y) ∈R ∧ (y, z) ∈R ⇒ (x, z) ∈R, luego, R
es relación de orden.
b) El diagrama de orden según R es:
c e Se observa que d es el menor elemento
a b según R en el conjunto A y no hay un
d elemento que sea mayor que todos.

Éste es un orden parcial.

5. Sea A = { x ∈IN / x es par ∧ x < 13}


R = {(x, y) / x es divisor de y}
a) Determinar si R es relación de orden.
b) Hacer el diagrama de orden.
Solución:
Escribamos A y R por extensión.
A = {2, 4, 6, 8, 10, 12}
R = {(2, 4), (2, 6), (2, 8), (2, 10), (2, 12), (4, 8), (4, 12), (6, 12),
(2, 2), (4, 4), (6, 6), (8, 8), (10, 10), (12, 12)}
a) R es relación de orden porque es refleja, antisimétrica y transi-
tiva.
b) 12 8 10 no es menor que 12 porque 10 no
divide a 12. 4 no es menor que 6. 2 es
10 6 4 menor que todos (los divide a todos), no
hay un elemento mayor que todos (que
sea dividido por todos).
2
Éste es un orden parcial.

6. Sea A = {(8, 6, 2, 7} y S la relación definida por


S = {(8, 6) (6, 2) (7, 7) (8, 2) (6, 6) (2, 2) (8, 8) (8, 7) (6, 7) (2, 7)}
a) Determinar si S es relación de orden.
b) Hacer el diagrama de orden en A según S.
Solución:
a) S es relación de orden porque es refleja, antisimétrica y transitiva.
b) El diagrama de orden en A según la relación S es:

7 8 es el menor de todos, 7 es el mayor de


2 todos y todos los pares de elementos de A
6 son comparables.
8 Éste es un orden total.

Relaciones y funciones 147

146-147..indd 147 01-02-2006 16:20:13


Ejercicios

1. Sea A = {1, 2, 3, 4} se definen las relaciones siguientes. Determine si éstas son


reflejas, simétricas, antisimétricas y/o transitivas.
R1 = {(1, 1) (2, 2) (3, 3) (4, 4) (2, 3) (3, 2)}
R2 = {(1, 2) (3, 2) (4, 2) (2, 2) (2, 1) (2, 3) (2, 4)}
R3 = {(1, 1) (2, 2) (3, 3) (4, 4)}
R4 = {(1, 2) (1, 3) (1, 4) (2, 3) (2, 4) (3, 4)}
R5 = {(2, 1) (2, 2) (3, 1) (3, 2) (3, 3) (4, 1) (4, 2) (4, 3) (4, 4) (1,1)}

2. Dadas las siguientes relaciones en Z, determine si éstas son reflejas, simétricas,


antisimétricas y/o transitivas.
a) x R y ¤ x es mayor o igual a y
b) x R y ¤ x • y≤0
c) x R y ¤ x = y
d) x R y ¤ x – y = 1
e) x R y ¤ x – y es un número par
f) x R y ¤ x + y = 10
g) x R y ¤ | x – y | > 2
h) x R y ¤ | x – y | < 3

3. Sea A = {0, 1, 2, 3, 4, 5, 6, 7, 8, 9}. Se defi nen en A las siguientes relaciones.


Determine las propiedades que tienen dichas relaciones.
a) x S y ¤ x – y = 2
b) x S y ¤ x divide a y
x
c) x S y ¤ el cociente tiene resto 2
y
d) x S y ¤ y – x < 3
e) x S y ¤ x · y = 36
f) x S y ¤ x + y = 9

4. Determine las propiedades que posee la relación de inclusión entre conjuntos.


ARB ¤ A C B A y B conjuntos no vacíos.

5. Sea R una relación simétrica que cumpla la condición


aRb ∧ bRc ⇒ cRa. Demuestre que R es transitiva.

6. En el conjunto de países A = {Japón, Sudán, India, China} se define la relación


S = {(Japón, Japón), (Sudán, Sudán), (India, India), (China, China), (Sudán,
Japón), (Japón, Sudán)}
a) ¿Es S relación de equivalencia?
b) Si lo es, haga la clasificación correspondiente.

7. Dados los siguientes conjuntos A y la relación R definida en ellos


a) Determine si R es relación de equivalencia
b) Si lo es, haga la clasificación correspondiente.

148 Relaciones y funciones

148-149. 148 08/11/2001, 16:28


CAPÍTULO 3

ii i) A= {9, 2, 6, 5} R = {(2, 9), (9, 9), (6, 6), (9, 2), (2, 2), (5, 5)}
ii) A = {3, 9, 1, 8} R = {(9, 3), (3, 3), (1, 1), (3, 9), (9, 9), (8, 8)}
iii) A = {1, 6, 9, 2} R = {(6, 1), (1, 1), (9, 9), (1, 6), (6, 6), (2, 2), (9, 2), (2, 9)}
iv) A = {4, 1, 9, 7} R = {(1, 4), (4, 4), (9, 9), (4, 1), (1, 1), (7, 7)}
v) A = {Alhué, Rapel, Purén, Maule}
R = {(Alhué, Alhué), (Rapel, Rapel), (Purén, Purén), (Alhué, Rapel), (Maule,
Maule), (Rapel, Alhué), (Alhué, Maule), (Maule, Rapel), (Rapel, Maule),
(Maule, Alhué)}
vi) A = {Coatí, Koala, Hiena, Lirón}
R = (Coatí, Koala), (Koala, Hiena), (Lirón, Lirón), (Coatí, Hiena), (Koala, Koala),
(Coatí, Coatí), (Hiena, Hiena), (Lirón, Hiena)}
vii) A = {Coatí, Lemur, Koala, Okapi}
R = {(Lemur, Okapi), (Okapi, Okapi), (Koala, Koala), (Lemur, Lemur), (Okapi,
Lemur), (Coatí, Coatí)}

8. Dados los siguientes conjuntos A y las relaciones S definidas en A


a) Determine si S es b) Haga el diagrama c) Determine si el or-
relación de orden. de orden en A según S. den es total o parcial.
i) A = {Sirio, Aries, Urano, Virgo}
S = {(Aries, Sirio), (Sirio, Sirio), (Urano, Urano), (Aries, Aries), (Sirio,
Aries), (Virgo, Virgo)}
ii) A = {5, 2, 4, 7}
S = {(5, 4), (5, 7), (2, 4), (2, 7), (4, 7), (5, 5), (2, 2), (4, 4), (7, 7)}
iii) A = {2, 5, 4, 1}
S = {(2, 4), (2, 1), (5, 4), (5, 1), (4, 1), (4, 2), (2, 2), (5, 5), (4, 4), (1, 1)}
iv) A = {7, 6, 8, 3}
S = {(7, 6), (6, 8), (3, 3), (7, 8), (6, 6), (7, 7), (8, 8), (3, 8)}
v) A = {Talca, Ancud, Alhué, Achao}
S = {(Talca, Alhué), (Talca, Achao), (Talca, Talca), (Ancud, Alhué), (Ancud,
Achao), (Alhué, Achao), (Alhué, Alhué), (Ancud, Ancud), (Achao,
Achao)}
vi) A = {5, 1, 6, 7}
S = {(5, 5), (6, 6), (1, 1), (5, 6), (7, 7), (6, 5), (5, 7), (7, 5), (6, 7), (7, 6)}
vii) A = {plomo, verde, caoba, rojo}
S = {(plomo, verde), (verde, caoba), (rojo, rojo), (plomo, caoba), (verde, verde),
(plomo, plomo), (caoba, caoba), (rojo, caoba)}
viii) A = {5, 7, 4, 2}
S = {(5, 5), (5, 4), (4, 4), (7, 7), (7, 4), (2, 4), (2, 2)}
ix) A = {7, 9, 6, 4}
S = {(7, 6), (7, 4), (9, 6), (9, 4), (6, 4), (7, 7), (9, 9), (6, 6), (4, 4)}
x) A = {4, 8, 5, 2}
S = {(4, 4), (4, 5), (5, 5), (8, 8), (8, 5), (2, 5), (2, 2)}

Relaciones y funciones 149

148-149. 149 08/11/2001, 16:28


Ejercicios
xi) A = {Libra, Virgo, Orión, Urano}
S = {(Libra, Orión), (Libra, Urano), (Libra, Libra), (Virgo, Orión),
(Virgo, Urano), (Orión, Urano), (Orión, Orión), (Virgo, Virgo), (Urano,
Urano)}
xii) A = {4, 1, 8, 9}
S = {(4, 1), (1, 8), (9, 9), (4, 8), (1, 1), (4, 4), (8, 8), (4, 9),
(1, 9), (8, 9)}
xiii) A = {1, 8, 6, 4}
S = {(1, 8), (6, 4), (1, 1), (8, 1), (6, 6), (4, 6), (8, 8)}

9. Sea el conjunto de rectas en el plano.


Determine qué propiedades posee la relación “ser perpendicular a”.
10. En un conjunto de personas determine las propiedades de la relación “tener
los mismos años que”.
11. En un conjunto de circunferencias de un plano se define cRc' si y sólo si c es
tangente con c'. Determine las propiedades de esta relación.
12. Determine las propiedades de la relación “ser hermano de” en un conjunto
de personas.
13. Determine las propiedades de la relación “ser hijo de” en un conjunto de
personas.
14. En un conjunto de jóvenes determine las propiedades de la relación “ser
más alto que”.
15. En el conjunto de polígonos determine las propiedades de la relación “tener
igual número de lados que”.

Soluciones

1. R1: Refleja, simétrica, transitiva c) Antisimétrica


R2: Simétrica d) Refleja
R 3 :Refleja, simétrica, antisimétrica, e) Simétrica
transitiva f) Simétrica
R4: Antisimétrica, transitiva 4. Refleja, antisimétrica y transitiva
R5: Refleja, antisimétrica, transitiva
2. a) Refleja, antisimétrica, transitiva China Sudán
b) Simétrica 6. a) Sí b)
India Japón
c) Refleja, simétrica, antisimétrica,
transitiva
d) Antisimétrica 5 2
e) Refleja (0 es par), simétrica, transi- 7. i) a) Sí b)
6 9
tiva
f) Simétrica 8 9
ii) a) Sí b) 1 3
g) Simétrica
h) Refleja, simétrica
3. a) Antisimétrica iii) a) Sí b) 6 9
b) Refleja, antisimétrica, transitiva 1 2

150 Relaciones y funciones

150-151.(2003) 150 20/11/02, 10:51 AM


CAPÍTULO 3

iv) a) Sí b) 4 9 Caoba
vii) a) Sí b) c) Orden
1 7 rojo
verde parcial.
plomo
v) a) Sí b) Alhué
Purén Rapel 4
viii) a) Sí b) c) Orden
Maule parcial.
5 2 7
vi) a) No, no es simétrica.
b) No se puede clasificar. ix) a) Sí b)
4
c) Orden
vii) a) Sí b) Lemur Coatí 6 parcial.
Okapi Koala 7 9
5
x) a) Sí b) c) Orden
4 2 8 parcial.
8. i) a) No
7
ii) a) Sí b) c) Orden
Urano
4 parcial. xi) a) Sí b) c) Orden
Orión parcial.
5 2
Libra Virgo
iii) a) Sí b) 1 c) Orden
parcial. 9
4 xii) a) Sí b) c) Orden
8
total.
2 5 1
4
8
iv) a) Sí b) c) Orden xiii) a) No es de orden, no es refleja.
6 3 parcial.
9. Simétrica.
7
10. Refleja, simétrica, transitiva. (Es relación
v) a) Sí b) Achao c) de equivalencia, las personas quedan
Orden
Alhué parcial. clasificadas según su edad.)
Talca Ancud 11. Simétrica.
12. Simétrica, transitiva.
vi) a) No es relación de orden, no es 13. Antisimétrica.
antisimétrica. 14. Antisimétrica, transitiva.
15. Refleja, simétrica, transitiva.

Funciones 3.4

3.4.1 Conceptos básicos


Dada una relación F : A Q B, esta relación es función si y sólo
si cada elemento de A tiene imagen única en B.
Dom F = A
(
(F : A Q B función) ¤ (F(x) = y A F(x) = z ⇒ y = z )
En un gráfico sagital, una relación es función si de todos los
elementos del primer conjunto sale una sola flecha.

Relaciones y funciones 151

150-151.(2003) 151 20/11/02, 10:51 AM


a 1 a 1
b 2 b 2
c 3 c 3
es función no es función

En un gráfico cartesiano una relación es función si al trazar


cualquier paralela al eje y ésta corta en un solo punto al gráfico
de la relación.
y y

x x

es función no es función

Composición de funciones.
Sea f : A Q B y g : B Q C funciones
(g o f) (x) = g (f (x)) ver ejercicio 5.
Algunas funciones reales interesantes.
Función constante Función idéntica
f (x) = k I (x) = x I(x)

f(x) 2 .
k
1 .
y=k

x
–2 –1
. –1
1 2 x

. –2

Dom f = R Dom I = R
Rang f = {k} Rang I = R

Función valor absoluto Función parte entera

y = |x| =
{ x si x ≥ 0
–x si x < 0
y = [x] = parte entera de x
[x] es el entero que cumple
x – 1 ≤ [x] ≤ x

y=|x | y = [x]
3

2 2

1 1

x -3 -2 -1 1 2 3 4 x
-2 -1 1 2
-1

-2

-3

Dom |x| = R Dom [x] = R


Rang |x| = R+ K {0} Rang [x] = Z

152 Relaciones y funciones

152-153. 152 08/11/2001, 15:18


CAPITULO 3
CAPÍTULO

1. Sean A = {1, 2, 3} y B = {4, 5, 6}. Dadas las relaciones de A en


B, determinar cuáles son funciones.
Ejercicios
a) R = {(1, 4) (2, 5) (3, 6)}
resueltos
b) R = {(1, 4) (2, 4) (3, 4)}
c) R = {(1, 4) (1, 5) (1, 6) (2, 4) (3, 6)}
d) R = {(1, 5) (2, 4) (1, 6) (2, 6)}
e) R = {(1, 6) (2, 4) (3, 6)}
Solución:
a) R es función porque todos los elementos de A tienen una
sola imagen en B.
b) R es función porque todos los elementos de A tienen una sola
imagen en B, no importa que ésta sea la misma para todos.
c) R no es función porque 1 ∈ A tiene más de una imagen en B.
d) R no es función porque 3 ∈ A no tiene imagen en B.
e) R es función porque todos los elementos de A tienen una
única imagen en B.

2. Dada la función f = {(1, 3) (2, 5) (3, 7) (4, 5)}


Determinar:
a) Dom f
b) Rang f
c) f (1); f (3)
d) el diagrama sagital
Solución:
a) Recordemos que el dominio de una función es el conjunto
de las preimágenes.
Dom f = {1, 2, 3, 4}
b) Recordemos que el rango de una función es el conjunto de
las imágenes.
Rang f = {3, 5, 7}
c) f (1) = 3 , f (3) = 7
d) 1
2 3
3 5
4 7

3. Dada la función: f : R → R definida por:


1
f (x) = | x–2|–1
3
a) Graficar f (x). Hallar Dom f y Rang f.
b) Determinar f (– 3), f (6), f (2)
c) Determinar x si f (x) = 1
Solución:
a) Como se trata de una función valor absoluto, veremos para

qué valor de x, | 1 x – 2 | se hace cero.


3

Relaciones y funciones 153

152-153. 153 08/11/2001, 15:19


Ejercicios 1
=0⇒
1 x–2=0⇒ 1 x=2⇒x=6
x–2
resueltos 3 3 3
∴ f (6) = 6
3
–2 –1=0 –1=–1
Como es un valor absoluto menos 1, la gráfica se traslada
una unidad hacia abajo.
Encontraremos, además, la imagen de dos valores simétricos
con respecto a 6, por ejemplo, 5 y 7.
5 1 2
f (5) = –2 –1=
–1=– 3
3 3
1 2
f (7) = 7 – 2 – 1 = –1=– 3
3 3
Para determinar en qué valor de x f (x) es 0, hacemos:
1
x–2 –1=0⇒ 1
x–2 =1⇒
3 3
x –2=1 V x –2 =–1
3 3
x =9 V x=3
Luego el gráfico es:
f(x)

3 6 9 12 x
-1

Dom f = R, ya que cada número real tiene imagen en R.


Rang f = {y ∈ R / y ≥ – 1}, ya que el menor valor de la
función es – 1.
1
b) f (– 3) = 3
–3 – 2 – 1 = | – 1 – 2| – 1 = |– 3| – 1 = 2
1
f (6) = 6 –2 – 1 = |2 – 2| – 1 = 0 – 1 = – 1
3
1 2 6 4 1
f (2) = 3
2 –2 –1= – –1= –1=
3 3 3 3
1 1
c) f (x) = 1 ⇒ 3
x–2 –1=1 fi x–2 =2
3


x
–2 = 2 V x – 2 = – 2
3
3
x x
= 4 V =0
3 3
x = 12 V x =0
Así, si f (x) = 1 entonces x = 12 y x = 0
x+3
4. Sea f : R Q R definida por f (x) =
x–3
a) ¿Es f una función?
b) Si no lo es, cómo podemos hacer que lo sea.
c) Hallar f (1), f (0), f (3), f (– 3), f (5)
Solución:
a) f no es función, porque 3 no tiene imagen, ya que al
calcular f (3) aparece una indeterminación.

154 Relaciones y funciones

154-155. 154 08/11/2001, 16:31


CAPÍTULO 3

b) Si f la definimos de R – {3} en R, entonces f es función, porque cada


número real distinto de 3 tiene una imagen única en R.
1+3 4
c) f (1) = = = –2
1–3 –2
0+3
f (0) = =–1
0–3
f (3) = no existe
–3+3 0
f (– 3) = = =0
–3–3 –6
5+3 8
f (5) = = =4
5–3 2

5. Sean f y g funciones reales definidas por


f (x) = 2x – 3 y g (x) = 4 – 5x. Hallar :
a) (f o g) (3)
b) (g o f) (– 1)
c) Una fórmula para f o g y para g o f

Solución:
a) (f o g) (3) = f (g (3)) = f (4 – 5 · 3) = f (– 11) = – 25
b) (g o f) (– 1) = g (f (– 1)) = g (2 · (– 1) – 3) = g (– 5) = 29
c) (f o g) (x) = f (g (x)) = f (4 – 5x) = 2 (4 – 5x) – 3 = 5 – 10x
(g o f) (x) = g (f (x)) = g (2x – 3) = 4 – 5 (2x – 3) = 19 – 10x

Ejercicios
1. Sean A = {1, 3, 5, 7} y B = {2, 4, 6}. b) R = {(0, – 3), (3, 0), (– 3, 3)}
Determine si las siguientes relaciones de c) R = {(0, – 3), (0, 3), (0, 0)}
A en B son o no funciones. d) R = {(– 3, 0), (3, 0)}
Justifique la respuesta.
e) R = {(– 3, 0), (0, 0), (3, 0)}
a) R = {(1, 2), (3, 4), (5, 6), (7, 6)} f ) R = {(– 3, – 3), (3, 3), (0, – 3), (0, 3)}
b) R = {(1, 2), (3, 4), (5, 6)} g) R = {(3, 0), (3, – 3), (3, 3)}.
c) S = {(3, 4), (5, 6), (7, 2), (7, 4)} 3. Sean A = {6, 7, 8} y B = {3, 6, 9}.
d) S = {(1, 4), (3, 4), (5, 4), (7, 4)} Sea f una relación de A en B definida
e) T = {(7, 6), (5, 4), (3, 2), (1, 2)} en los diagramas dados. Determine
f ) T = {(1, 6), (3, 4), (5, 2), (5, 4), (7, 6)} en qué casos f es función. Justifique
la respuesta.
g) R = {(3, 2), (3, 4), (3, 6)} f
a) A B
2. Sea A = {– 3, 0, 3}. Determine si las
6 3
siguientes relaciones en A son o no
7 6
funciones. Justifique la respuesta. 8 9
a) R = {(– 3, – 3), (0, 0), (3, 3)}

Relaciones y funciones 155

154-155. 155 08/11/2001, 16:31


Ejercicios 6. Sea F = {(2,3) (3,5) (4,7) (5,9)} una
f
b) A B función. Determine:
6 3 a) Dom F
7 6 b) Rang F
8 9
c) F(2), F(3), F(5)
d) El diagrama sagital
f
A B
7. En cada una de las siguientes funciones
c)
6 3 encuentre las imágenes pedidas.
7 6
8 9 a) f : NQN, f(n) = 5 + n
f(1), f(5), f(10), f(50)
f b) f : ZQZ, f(x) = 6 – 4x
A B
f(–3), f(0), f(5), f(9)
d)
6 3
7 6 c) h : RQR, h(x) = 3x – 1
8 9 1
h –2 , h ( 13 ), h(6), h(a + 1)

f d) f : RQR, f(x) = 3x2 + 2x – 1


A B
e) f(–3), f – 1 , f(0), f(0,2)
6 3 2
7 6 e) f : RQR, f(x) = (x – 2) (x + 3) (x – 5)
8 9 f(–1), f(0), f(1), f(2)
f) g : RQR, g(x) = |x2 – 3|
f
A B g(4), g(1), g(–1), g(3)
f) g) f : RQR, f(x) = 4x
6 3
7 6 f(a), f(2a), f(a + ε), f(a – 3)
8 9
8. Sea f : NQN la función definida por
4. Dadas las relaciones siguientes, deter-
mine si son o no funciones. Justifique
f(n) =
{
n + 2 si n ≤ 4
n – 2 si n > 4

la respuesta en caso de que no lo sean. Encuentre:


12 – x f(1), f(3), f(4), f(5), f(10)
a) R = {(x, y) E N ¥ N / y = 3
}
9. A = {–4, –3, –2, –1, 0, 1, 2, 3, 4}
b) F = {(x, y) E N ¥ N / y = x + 1}
Sea f : AQR la función definida por
c) R = {(x, y) E N ¥ N / y = x – 1} 2
x –1
f(x) =
d) F = {(x, y) E N ¥ N / y es 2
múltiplo de x} Determine Rang f.

e) G = {(x, y) E N ¥ N / y = x2 + x} 10. Sea g : RQR una función definida por

f) G = {(x, y) E N ¥ N / y = x2 + 1}
g) R = {(x, y) E N ¥ N / y = 7 – x}
g(x) =
{ x+3
2
–x + 3
si x ≤ 1
si 1 < x < 3
si x ≥ 3
5. De las relaciones del ejercicio anterior a) Grafique la función g. Hallar
que no son funciones de N en N, Dom g y Rang g.
¿cuáles serían funciones si estuvieran b) Determine
definidas de N en Z? g(0), g(1), g(–1), g(2), g(2,5), g(5)

156 Relaciones y funciones

156-157. 156 08/11/2001, 16:32


CAPÍTULO 3

11. Sea f : RQR una función definida por: c) Determine x si g(x) = 1


–2 si x ≤ –1 d) Determine x si g(x) = 0
f(x) =
{ x–1
1
si –1 < x < 2
si x ≥ 2
17. Sea f: RQR una función definida por
f(x) = 1 + [x]
a) Grafique la función f. Halle: Dom f a) Grafique f. Halle Dom f y Rang f
y Rang f. b) Determine:
b) Encuentre f(1), f(1,3), f(1,5), f(0) f(0,2), f(–3),
7 4 f(–3,5), f(–3,8)
f –
3
, f(–1), f(0), f(1), f(2), f
3 c) Determine: x tal que f(x) = 2
12. Sea g: RQR una función definida por d) Determine: x tal que f(x) = –4

g(x) =
{ 2x + 6
3
x+2
si x < 2
si x ≥ 2
a) Grafique la función g. Halle: Dom g
18. Sea f: RQR una relación definida por

f(x) =
x–3
x+3
a) ¿Es f una función de R en R?
y Rang g
b) Si no lo es, ¿cómo puede hacer que
b) Determine
lo sea?
g(–3), g(–1), g(0), g(2), g(3), g(10)
c) Determine: f(3), f (–3), f(0), f (2)
13. Sea g: RQR una función definida por
19 . Sea f: RQR una relación definida por
g(x) = |2x –1|
1
a) Haga un gráfico. f(x) = .
x
Halle Dom g y Rang g.
b) Determine g(2), g(–2), g(5) a) ¿Es f una función de R en R?
c) Determine x si g(x) = 9 b) Si no lo es, ¿cómo puede hacer que
d) Determine x si g(x) = 0 lo sea?

14. Sea f: RQR una función definida por c) Determine: f(0), f(1), f(2), f
1
2 (1)
1
f(x) = | x + 3| + 1
2
f( –1), f(–2) y f –
2 ( )
a) Haga un gráfico de f(x). d) Bosqueje un gráfico de f
Halle Dom f y Rang f. 20. Sea f: RQR una relación definida por
b) Determine f(2), f(–5), f(–6), f(–7), f(0)
1
c) Determine x si f(x) = 5 f(x) =
x–2
d) Determine x si f(x) = 0
a) ¿Es f una función de R en R?
15. Sea f: RQR una función definida por b) Si no lo es, ¿cómo puede hacer que
f(x) = 1 – |x – 3| lo sea?
a) Haga un gráfico de f(x). c) Determine: f(2), f(3), f(4), f(1),
Halle Dom f y Rang f.
b) Determine f(3), f(2), f(4), f(0), f(6)
f ( 32 ), f(0 )
c) Determine x si f(x) = 0 d) Bosqueje un gráfico de f
d) Determine x si f(x) = 2 21. Sean f(x) = 2x – 1 y g(x ) = x + 2
16. Sea g: RQR una función definida por dos funciones reales.
g(x) = 2 – |2x + 1| Halle:
a) Haga un gráfico de g. a) (f o g) (2)
Halle Dom g y Rang g. b) (g o f) (2)
b) Determine c) (f o f) (3)
1 d) Una expresión para (f o g) (x) y
g ( 2 )
, g(0 ), g(–1), g(1), g(–2)
para (g o f) (x).

Relaciones y funciones 157

156-157. 157 08/11/2001, 16:33


Ejercicios

22. Sean f(x) = 2x, g(x) = x2 y h(x) = x+2 Responda las siguientes preguntas:
tres funciones reales. Determine: a) Una familia consumió 25 m3 en el
a) (f o g o h) (3) mes. ¿Cuánto debe cancelar?
b) (g o h o f) (–2) b) Al mes siguiente se queda una llave
c) (g o h o g) (1) goteando y el medidor arroja 52
d) una fórmula para (h o g o f) (x) m3. De acuerdo con el gráfico de la
1 x+2
tarifa, ¿pagará poco más que el doble
23. Sean f(x) = , g(x) = x – 2 dos fun- que el mes anterior o mucho más
x
ciones reales. Determine: que el doble? ¿A qué se debe?
a) Dom f y Dom g c) ¿Cuánto debe cancelar por metro
b) (f o g) (3) cúbico una familia que gasta entre
c) (f o g) (–2) 70 y 80 m3 al mes?
d) (g o f) (1) d) ¿Hasta cuántos metros cúbicos
1 puede gastar una familia para no
e) (g o f) ( )
2 cancelar más de 1 US$ por m3?
f) una expresión para f o g
e) Si en la cuenta se lee un cobro fijo de
g) una expresión para g o f US$ 3 y el total a cancelar es de US$
h) Dom f o g y Dom g o f 105. ¿Qué cantidad de m3 consumió
24. Sean f(x) = 2x2 – x + 3 y g(x) = x2 – 1 esa familia en el mes?
dos funciones reales. Determine: f) Defina la función del gráfico a
a) (f o g) (–5) través de una fórmula y calcule su
b) (g o f) (2) dominio y su recorrido.
c) (g o g) (–3)
1
d) (f o f) (1) 27. Si f(x) = x+1
2
e) una expresión para f o g y g o f a) Grafique f(x)
b) Calcule y grafique f(x + 1)
25. Sean h(x) = x3 + 1 y g(x) = x2 dos c) Calcule y grafique f(x – 1)
funciones reales. Determine: d) Calcule y grafique f(x) + 1
a) (h o g) (–1) e) Calcule y grafique f(x) – 1
b) (g o h) (–3) f) Compare los resultados y este-
c) (h o g) (x) blezca alguna conjetura.
d) (g o h) (x)
28. Escriba algunas funciones y = f(x) e
26. Suponiendo que el gráfico siguiente investigue qué ocurre con f(x ± k) y
representa los valores en US$ por f(x) ± k para k E N.
metro cúbico del consumo de agua 1 1
potable en una ciudad desértica. Ej. 1: f(x) = ; f(x+2) = ;
x x+2
[US$] 1
f(x) + 2 = +2
3 x
2,4
2 Ej. 2: f(x) = x + 3 ; f(x–5) = x – 2 ;
1,6 f(x) – 5 = x + 3 – 5
1
0,6 Grafique y determine intersecciones con
los ejes, y determine dominios y rangos.
10 20 30 40 50 60 70 80 90 100 110 120130140 [m3]

158 Relaciones y funciones

158-159. 158 08/11/2001, 16:34


CAPÍTULO 3
Soluciones

1. a) R es función porque cada elemento d) No es función, cada número


de A tiene imagen única en B. natural tiene infinitas imágenes.
b) R no es función porque 7 E A no e) Es función.
tiene imagen en B. f) Es función.
c) S no es función porque 1 E A no g) No es función, los números naturales
tiene imagen en B y además 7 E A mayores que 6 no tienen imagen en
tiene más de una imagen. N.
d) S es función porque cada elemento 5. c, y g,
de A tiene imagen única en B. 6. a) Dom F = {2, 3, 4, 5}
e) T es función porque cada elemento b) Rang F = {3, 5, 7, 9}
de A tiene imagen única en B. c) F (2) = 3; F (3) = 5; F (5) = 9
f) T no es función porque 5 E A tiene
d)
más de una imagen en B.
g) R no es función porque hay elementos 2 3
de A que no tienen imagen en B 3 5
y además 3 E A tiene más de una 4 7
imagen en B. 5 9
2. a) R es función porque cada elemento 7. a) f (1) = 6, f (5) =10, f (10) = 15,
de A tiene imagen única en A. f (50) = 55
b) R es función porque cada elemento b) f (–3) = 18, f (0) = 6, f (5) = –14
de A tiene imagen única en A. f (9) = –30
c) R no es función porque 0 E A tiene
1 5 1
más de una imagen en A y además c) h – =– ,h = 0, h (6) = 17
2 2 3
hay elementos de A que no tienen h (a + 1) = 3a + 2
imagen.
d) R no es función porque 0 no tiene d) f (–3) = 20, f – 1 = – 5 , f (0) = –1
2 4
imagen. f (0,2) = –0,48
e) R es función porque cada elemento e) f (–1) = 36, f (0) = 30, f (1) = 16
de A tiene una única imagen en A. f (2) = 0
f) R no es función porque 0 E A tiene f) g (4) = 13, g (1) = 2, g (–1) = 2
más de una imagen en A. g (3) = 6
g) R no es función porque hay elementos
g) f (a) = 4a, f (2a) = 8a, f (a + E) = 4a+4E
de A que no tienen imagen en A y f (a – 3) = 4a – 12
además el 3 E A tiene más de una
8. f (1) = 3, f (3) = 5, f (4) = 6, f (5) = 3,
imagen en A.
f (10) = 8
3. a) Es función.
15 3 1
b) Es función. 9. Rang f = , 4, , 0, –
2 2 2
c) No es función, 6 no tiene imagen. 10. a) Y
d) No es función, 7 tiene más de una
4
imagen, 6 y 8 no tienen imagen.
e) Es función.
f) Es función.
4. a) No es función, hay números natu- -1 1 3 X
rales que no tienen imagen en N,
ejemplo: Dom g = R
11 Rang g = {y E R / y £ 4}
si x = 1 entonces y = F N.
3
b) Es función. b) g (0) = 3, g (1) = 4, g (–1) = 2
c) No es función, 1 no tiene imagen en N. g (2) = 2, g (2,5) = 2, g (5) = –2

Relaciones y funciones 159

158-159. 159 08/11/2001, 16:34


Soluciones
f(X)
15. a) 1
11. a) Y
1 1 2 3 4 5 6 X
-1
-2
2 X
-2

Dom f = R Dom f = R
Rang f = { y E R / –2 ≤ y ≤ 1} Rang f = { y E R / y ≤ 1}
7
b) f – ( ) 3
= – 2, f (– 1) = – 2, f (0) = – 1, b) f (3) = 1, f (2) = 0, f (4) = 0,
4 1
f (1) = 0, f (2) = 1, f
3
=
3 ( ) f (0) = –2, f (6) = –2
12. a) c) x = 4 V x = 2
Y
d) no existe x ya que f (x) = 2 F Rang f.
4

g(x)
2 16. a)
2
1
-3 2 X
-3
-1 -1 1 x
2 2 2

Dom g = R
10
Rang g = { y E R / y £ V y ≥ 4} Dom g = R
3
Rang g = {y E R / y ≤ 2}
b) g(– 3) = 0, g (–1) = 4 , g (0) = 2,
3
1
g (2) = 4, g (3) = 5, g (10) = 12 b) g –( 2 )
= 2, g (0) =1, g (–1) = 1,
13. a) g (1) = –1, g (–2) = –1
g (X)
c) x = 0 V x = –1
1 3
d) x = V x=–
2 2
1 X
17. a)
Dom g = R f(x)
Rang g = { y E R / y ≥ 0}
b) g (2) = 3, g (–2) = 5, g (5) = 9
c) x = 5 A x = –4 x
f(X)
d) x = 1
2 4
14. a)

1
Dom f = R
–6 X
Rang f = Z
Dom f = R
Rang f = { y E R / y ≥ 1} b) f (1) = 2, f (1, 3) = 2, f (1, 5) = 2,
3 3 f (0) = 1, f (0, 2) = 1, f (–3) = – 2,
b) f(2) = 5, f(–5) = , f(–6) = 1, f(–7) =
2 2 f (–3, 5) = –3, f (–3, 8) = –3
f(0) = 4
c) x = –6 c) x E [1, 2)
d) H x E R d) x E [– 5, – 4)

160 Relaciones y funciones

160-161.(2003) 160 20/11/02, 10:53 AM


CAPÍTULO 3

18. a) f no es función de R en R porque 21. a) 7 b) 5 c) 9 d) (f o g) (x) = 2 x + 3


f (–3) no existe. (g o f) (x) = 2 x + 1
b) definiendo f de R – {– 3} en R.
22. a) 50 b) 4 c) 9 d) 4 x2 + 2
c) f (3) = 0, f (–3) = no existe,
1
f (0) = – 1, f (2) = –
5 23. a) Dom f = R – {0}, Dom g = R – {2}

19. a) f no es función de R en R porque b) 1 c) no existe d) – 3 e) no existe


5
f (0) no existe. x–2 1+2x
f) g)
b) definiendo f de R – {0} en R. x+2 1–2x
1 h) Dom f o g = R – {– 2}
c) f (0) = no existe, f (1) = 1, f (2) = ,
2
f ( 12 ) = 2, f (–1) = – 1, f (–2) = – 1
2
, Dom g o f = R – { 12 }
1
f (–
2)
=–2
24. a) 1.131 b) 80 c) 63 d) 31
d)
f(x) e) (f o g) (x) = 2 x4 – 5 x2 + 6;

2
(g o f) (x) = 4 x4 – 4 x3 + 13 x2 – 6x + 8

1
1
25. a) 2 b) 676 c) x6 + 1 d) x6 + 2 x3 + 1
2

-1 1 1 2
2 x
26. a) US$ 15
b) Debe cancelar US$
52, lo que repre-
senta mucho más
del doble, porque
el consumo cayó
20. a) f no es función de R en R porque
en el tramo entre
f (2) no existe. 30 y 60 m3, cuyo
b) definiendo f de R – {2} Q R valor es de US$ 1
1 por m3.
c) f (2) = no existe, f (3) = 1, f (4) = ,
2 c) US$ 1,6
3 1
f (1) = – 1, f ( )
2
= – 2, f (0) = –
2
d) 60 m3
e) 63,75 m3
d)
f(x)

{
0,6 si 0 < x ≤ 30
1 si 30 < x ≤ 60
f) f(x) =
1,6 si 60 < x ≤ 100
2,4 si 100 < x ;

2 3 4 x Dom f = R+ K {0}
Rang f = {0,6 , 1 , 1,6 , 2,4}

Relaciones y funciones 161

160-161.(2003) 161 20/11/02, 10:53 AM


Soluciones
27. f(X)+1= 12 x + 2
f(X+1)= 12 x + 32
f(X)= 12 x + 1 f(X)= 12 x + 1
f (X–1)= 12 x + 12 2 f(X)–1= 12 x
1
f) f(x+1) está desplazada 1
0
–3 –2 –1 unidad a la izquierda de f(x).
f(x) + 1 está desplazada 1
unidad hacia arriba de f(x).

3.4.2 La función de primer grado


(Ecuación de la recta)
y = f (x) = a x + b, a ≠ 0, a, b E R
Dom f = R Rang f = R
Su gráfico es una recta y depende de los valores que tomen a y b.
b se llama coeficiente de posición y (0, b) es el punto donde
la recta intersecta al eje y.
a, coeficiente de x, es el valor de la pendiente de la recta.
Si en una recta (x1, y1) y (x2, y2), con x1 ≠ x2 son puntos
de ella, entonces.
y2 – y1 Δy
Su pendiente es m = =
x2 – x1 Δx
y2 – y1
Su ecuación es: (y – y1) = (x – x1)
x2 – x1
la cual se puede escribir en la forma: y = m x + k, donde m es
la pendiente y k el coeficiente de posición.
Si los puntos (x1, y1) y (x2, y2) son tales que x1 = x2, entonces
la recta es paralela al eje y, su pendiente es indeterminada y
su ecuación es x = x1
Se llama ángulo de inclinación (a) de una recta al ángulo que la
recta forma con la parte positiva del eje x.

Relación entre el ángulo de inclinación (α) y la pendiente (m) de una recta L


α α=0 0º < α < 90º α = 90º 90º < α < 180º
m m=0 m>0 Indeterminada m<0
G Y Y Y Y
R
Á L L
α α
F
X X X X
I
C L
O

162 Relaciones y funciones

162-163.(2003) 162 20/11/02, 10:56 AM


CAPÍTULO 3

Relación entre las pendientes y la posición de dos rectas en el plano.


• Dos rectas L1 y L2 son paralelas si y sólo si sus pendientes son iguales.
(L1 // L2) ¤ (m1 = m2)

• Dos rectas L1 y L2 son perpendiculares si y sólo si el producto


de sus pendientes es – 1
(L1 ⊥ L2) ¤ (m1 • m2 = – 1)

1. Dada una ecuación de primer grado en dos variables, siempre es


posible despejar una de las variables en función de la otra.
Ejercicios
Así, dada la ecuación: ax + by + c = 0 (b ≠ 0) resueltos
podemos escribir:

y = – ax – c
b
a c
y =– x–
b b
a c
si hacemos – =m y – =k
b b
escribimos y = mx + k, que es una función de primer grado y
representa una recta de pendiente m y coeficiente de posición k.
a se llama coeficiente de x.
b se llama coeficiente de y.
c se llama término libre.
Dada la ecuación 5x + 3y – 6 = 0
Hallar:
a) el coeficiente de x.
b) el coeficiente de y.
c) el término libre.
d) la pendiente m de la recta que representa.
e) el coeficiente de posición k de la recta que representa.
Solución:
a) coeficiente de x = 5
b) coeficiente de y = 3
c) término libre = –6
d) debemos escribir la ecuación en forma de función de primer
grado:
5
y = – x+2
3
5
luego m = –
3
e) En la misma función del punto d) k = 2
2. Escribir la ecuación 2x – 1 = 0 como una función de primer
grado y = f (x).
Solución:
No es posible escribir la ecuación 2x – 1 = 0 como una función
de primer grado y = f (x) porque la variable y no aparece en la
ecuación y esto significa que su coeficiente es 0.

Relaciones y funciones 163

162-163.(2003) 163 20/11/02, 10:56 AM


Ejercicios 3. Dada la ecuación 5x – 3y + 8 = 0 encontrar dos puntos que
pertenezcan a la recta que representa y graficarla.
resueltos
Solución:
Recordemos que un punto pertenece a una recta si y sólo si
satisface su ecuación.
Dando valores a una de las variables podemos obtener el valor
correspondiente a la otra, de modo que el par (x, y) sea un punto
de la recta representada por la ecuación 5x – 3y + 8 = 0.
Así, si x = 2 fi 5 • 2 + 3y + 8= 0
3y = – 18
y = –6 Y
2
∴ (2, –6) es punto de la recta cuya 3

-2 2
ecuación es 5x + 3y + 8 = 0 X

Si x = –2 fi 5 • (–2) + 3y + 8 = 0
3y = 2
2
y =
3
2 -6
∴ (–2, ) es punto de la recta cuya
3
ecuación es 5x + 3y + 8 = 0
Sabemos que dados dos puntos de una recta, ésta queda definida
(ver gráfico).
4. Graficar las rectas representadas por las siguientes ecuaciones:
a) x – 2y + 6 = 0
b) x –1 = 2
c) 1 –2y = 5
Solución:
a) Para graficar la recta cuya ecuación es x –2y + 6 = 0 buscaremos
dos puntos de ella.
Si x = 2 fi 2 – 2y + 6 = 0 Y

–2y = –8 5
y = 4 4

∴ (2, 4) es un punto de la recta


Si x = 4 fi 4 – 2y + 6 = 0
–2y = –10 2 4 X
y = 5
\ (4, 5) es un punto de la recta
(ver gráfico).
b) La ecuación x – 1 = 2 es Y

equivalente a la ecuación x = 3;
representa todos los puntos del
plano cuya abscisa vale 3 (es 3 X
paralela al eje Y) .
c) La ecuación 1 – 2y = 5 es equi- Y
valente a la ecuación y = –2; repre-
senta todos los puntos del plano
X
cuya ordenada es –2 (es paralela -2
al eje X).

164 Relaciones y funciones

164-165. 164 08/11/2001, 16:39


CAPÍTULO 3

5. Hallar los puntos en que la recta cuya ecuación es 4x – 6y + 8 = 0


intersecta a los ejes coordenados. Graficarla.
Solución:
Intersección al eje X: Como todos los puntos del eje x son de
la forma (x, 0), debemos encontrar el punto que satisface la
ecuación de la recta y tiene ordenada y = 0
Si y = 0 ⇒ 4x + 8 = 0 ⇒ x = –2
∴ La recta intersecta al eje X en (–2, 0)
Intersección al eje Y: Como todos los puntos del eje y son Y
de la forma (0, y), debemos encontrar el punto que satisface
la ecuación de la recta y tiene abscisa x = 0
4 4
Si x = 0 ⇒ – 6y + 8 = 0 ⇒ y = 3
3

∴ La recta intersecta al eje Y en 0,


4
3
-2 X

Con los dos puntos de intersección podemos trazar


su gráfico.
6. Hallar la ecuación de la recta que pasa por los puntos (3, 2) y
(1, –1). Graficarla.
Solución:
Si (x1, y1) y (x2, y2) son puntos de una recta, su ecuación es:
y2 – y1
y – y1 = (x – x1) (*)
x2 – x1
y2 – y1 Y
donde = m (pendiente).
x2 – x1
2
Sea (x1, y1) = (3, 2) y (x2, y2) = ( 1, –1)

y2 – y1 –1 –2 –3 3 1 3 X
= = = -1
x2 – x1 1–3 –2 2
Luego en (*) queda:
3
y–2 = (x – 3) /•2
2
2y – 4 = 3x – 9
3x – 2y – 5 = 0 es la ecuación pedida (ver gráfico).
7. Hallar la ecuación de una recta cuya pendiente es –3 y pasa
por el punto (2, –5)
Solución:
m = –3 y (x1, y1) = (2, –5)
La ecuación pedida es:
y – y1 = m ( x – x1)
y + 5 = –3 ( x – 2)
y + 5 = –3x + 6
3x + y – 1 = 0 es la ecuación buscada.
8. Determinar si los tres puntos (–3, 2), (–2, 0) y (1, –6) son colineales.
Solución:
Para que tres puntos sean colineales, la pendiente de la recta
definida por dos de ellos debe ser igual a la pendiente de la recta

Relaciones y funciones 165

164-165. 165 08/11/2001, 16:39


Ejercicios definida por el tercer punto y cualquiera de los dos anteriores:
resueltos Pendiente m1 entre (–3, 2) y (–2, 0)
0–2
m1 = = –2
–2+3
Pendiente m2 entre (–2, 0) y (1, –6)
–6–0
m2 = = –2
1+2
Como m1 = m2, los tres puntos son colineales.

9. Determinar si las rectas determinadas por los siguientes pares de


ecuaciones son paralelas, perpendiculares o sólo son secantes.
a) L1: 2x –3y + 6 = 0 L2 : 9y –6x = 0
b) L1: 2x + y – 1 = 0 L2 : x –2y + 3 = 0
c) L1: 1 –4x = y L2 : 2x + 5y – 3 = 0

Solución:
Para responder hay que calcular y comparar las pendientes
de las rectas.
2
a) L1: 2x –3y + 6 = 0 ⇒ y = x + 2
3
∴ mL1 = 2
3
6 2
L2: 9y – 6x = 0 ⇒ y = 9
x⇒y= x
3
∴ mL2 = 2
3

luego, como mL1 = mL2, entonces L1 // L2

b) L1: 2x + y – 1 = 0 ⇒ y = –2x + 1
∴ mL1 = –2
1 3
L2: x – 2y + 3 = 0 ⇒ y = x+
2 2
∴ mL2 = 1
2

luego, como mL1 • mL2 = –2 •


1
2
= –1, entonces L1 ⊥ L2

c) L1: 1 – 4x = y ⇒ y = –4x + 1
∴ mL1 = –4
2 3
L2: 2x + 5y – 3 = 0 ⇒ y = – x+
5 5
∴ mL2 = – 25
Como mL1 ≠ mL2, las rectas no son paralelas.
Como mL1 • mL2 ≠ –1, las rectas no son perpendiculares.
Luego podemos decir que son rectas secantes. Con ayuda de
algún concepto de trigonometría podríamos calcular el ángulo
formado por las rectas.
10. Encontrar la ecuación de la familia de rectas que pasan por el
punto (5, 3). Dibujar 3 de ellas.

166 Relaciones y funciones

166-167. 166 08/11/2001, 16:45


CAPÍTULO 3

Solución:
Para definir una recta necesitamos conocer dos parámetros, los que
pueden ser dos puntos de ella o un punto y su pendiente.
En este caso sólo tenemos un parámetro, el punto (5,3), por ello
sólo podemos encontrar la ecuación de la familia de rectas (todas
las rectas) que pasan por el punto (5,3) dándole a la pendiente
un valor m.
Entonces la pendiente es m y (x1, y1) = (5,3). Y
Así, la ecuación de la familia es:
8 m=1
y – y1 = m (x – x1)
y – 3 = m (x – 5)
mx – y – 5m + 3 = 0
3 m=0
Ahora, para dibujar 3 de ellas debemos asignar
3 valores distintos a su pendiente m.
si m = 1 ⇒ x – y – 2 = 0 2 5 8 X
si m = 0 ⇒ – y + 3 = 0 2 m = -1
si m = –1 ⇒ – x – y + 8 = 0

Ejercicios
1. Determine coeficiente
l) 0,5x = 1 + 2,5y 3. Escriba dos soluciones
de x, coeficiente de y,
de cada una de las ecua-
y término libre en cada m) 3x = 2y + x – 1
ciones siguientes:
una de las siguientes
n) ax = 2by + c
ecuaciones. a) 4x – y + 2 = 0
o) ax – bx + aby – b = 0
a) 4x + 6y + 1 = 0 b) 6x – 2 = y
p) y = mx + k
b) 3x – y + 5 = 0 c) 3x – 1 = 5
q) ax – by = 2a + b
c) 2x = 5 + y d) 6x – y = 0
r) x = 1 – y + x – 3k
d) 6y – 2x = 3 e) x – 2y + 9 = 0
s) 2x – ky + y – 3 = 1
e) 4x + y = 0 f) x + 1 = 0
t) x + 2y – 4 = y + k g) y = 2
f) 6x = 2y
g) 4x – 1 = 0 1 3
h) x– y=1
2. Exprese cada una de las 2 4
h) 1 – 5x = 0 ecuaciones del ejercicio 1
i) 9x – y – 0,5 = 0
anterior como una fun- 2
i) 1 = 6y
ción de primer grado j) 12x – y + 1 = 0
j) 2y – 3 = 0
y = f (x). Si no es posible,
k)
1 x+ 1 y–1=0 indique por qué. 4. Determine cuáles de los
3 4 pares de valores indica-

Relaciones y funciones 167

166-167. 167 08/11/2001, 16:45


Ejercicios c) x + ky – 5 = 1
q) x + y = x – 3
(–3, –1)
dos satisfacen la ecuación dada. r) 2x + 3y = 3y – 6
d) kx – y = 2k
a) 6x – y + 3 = 0
(–5, 1) 8. Determine la intersección
(1, 1), (0, 3), (1, –3)
con cada uno de los ejes
e) kx + 2ky + k = 0
b) y + 2x – 5 = 0 coordenados de las rectas
(1, –1)
representadas por las ecua-
( 12 ,4,) ,(2, 1), (3, –1) f) kx + 2ky + k = 0 ciones siguientes:
c) 6y = 12 (1, 1)
(2, 1), (1, 2), (2, 2) a) x + y = 5
g) 3x – k + 2y = 5
d) 4x – y = 0 (0, 0) b) 6x – 2y + 3 = 0
(0, 0), (1, 4), (2, 7) c) x – 5y + 10 = 0
h) 1 – 2k + x – y = 0
e) x = 2y – 1 (3, –1) d) 2x – y + 6 = 0
(1, 1), (3, 2), (2, 3)
i) 3kx + y – 1 = 0 e) 3x – 5 = 0
f) 1 = x + 2y (4, 1)
(1, 1), (1, –1), (–1, 1) f) 4y + 2 = 0
7. Grafique cada una de las g) 2x – 5y = 0
5. Determine cuáles de las siguientes ecuaciones:
siguientes ecuaciones repre- h) 4y – x = y + 3x – 1
sentan una recta que pasa a) 3x – y = 1
i) 6x – 5 = 0
por el origen (el pun to
b) x + 2y – 5 = 0 j) 4y = 2
(0, 0)).
c) – 6x + 2y = 0 k) ax – by = ab, a, b ≠ 0
a) 6x – y = 0
d) 3x = 5 l) 4x – 3y + 1 – k = 0
b) 4x + 1 = y
e) 4y – 1 = 0 m) 1 – 2y = 4 – x
c) x – 2y + 1 = 1
x + 2y –1 n) x – 1 = y – 2
d) 1 – x – 5y + 3 = 4 f) =2
3
1 o) x + 2y –1 = 0
e) 2x – 5y = x – y g) 1 – x = y x+y y
3 p) =1+
f) 1 – 2y = x + 2 3 1 2 2
h) x + y – 1 = 0 q) 3x – 4 = 6y – 1
g) 6x – 2y = 5 4 3
x y
3 r) –1= +2
h) 1 + 2y = 1 – 3x i) 0,5x + y – 0,2 = 0 3 2
4
i) x – 2y + 4 = y + 1 x+2 y–1 9. Sin hacer cálculos, indique
j) = cuál(es) de las siguientes
3 2
j) x – y – 1 = 1
k) 3x + y – 1 = 0 ecuaciones corresponde(n)
a rectas que son paralelas
6. Determine el valor de k l) 24x + 8y – 16 = 0 al eje X, paralelas al eje
en la ecuación para que la
Y, pasan por el origen o
recta que representa pase m) y + 1 = 3x – 2
intersectan a ambos ejes
por el punto dado.
n) 4x + 2y – 5 = x + y en puntos distintos.
a) x – 2y + k = 0
(1, 1) o) 2x – 3 = 2y – 4 a) 4x – y = 0

b) 3x + 2y – k = 0 6x –1 + 3y b) 2x – y = 1 – y
p) =0
(2, 3) 4

168 Relaciones y funciones

168-169. 168 08/11/2001, 16:47


CAPÍTULO 3

de cada una de las rectas j) (7, 1) , (–7, –1)


c) x + 2y – 5 = 0 representadas por las
k) (–3, –4) ,(–4, –3)
ecuaciones siguientes:
d) 4x – 5y = 6 l) (0, 0), (1, 5)
e) 2x – 1 = 0 a) 3x + y – 2 = 0
13. Encuentre la ecuación de
f) 3y – 4 = 0 b) x + 5y – 5 = 0
la recta dada su pendiente
g) x + 2y = 1 + x c) x + y – 3 = 0 m y un punto de ella.
h) x + 1 = –3y + 1 d) 3x = y + 5 a) m = 1 (2, 3)
i) x + y – 2 = 2 3
e) –x + 5y = 2 b) m = (1, –1)
4
j) 4 – 6x = y + 4 c) m = –3 (2, 5)
f) x + 2 = y – 3
k) 2x – 3y + 5 = 5 x y 1 d) m =
1 (3, 1)
g) + – =0
2 3 6 2
l) 4x – 2y + 1 – k = 0
h) 0,3x – 0,5y = 0 e) m = 0 (12, 5)
m) y + 2x = 10 1
i) 2x – 3 = y – 3 f) m = – (3, –1)
2
n) 3x – y = 1 + y
j) 5x = x – 4 g) m = 5 (– 6, 0)
o) 4y – 6 = x
k) x – 1 = 1 – y h) m = 2 (0, 3)
p) x + 3 = 0
l) y = y + 2x – 3 14. Encuentre la ecuación
q) 2 (x – 1) = 3 (x + y)
de la recta que corta al
r) 3 (x + 2) = 2 ( y + 3) m) y – 1 = 0 eje X en (5, 0) y al eje
n) 3x – 2y + 1 = x + y – 1 Y en (0, –3).
10. Determine la pendiente
de la recta que pasa por x 2y 1 15. Encuentre la ecuación
o) + – =0
4 3 2
los puntos dados. de la recta cuya pen-
p) y = 6x – x + 2
diente es 3 y pasa por
a) (4, 1) , (2, 3) 4
el origen.
b) (5, 1) , (–1, 4) 12. Encuentre la ecuación
de la recta que pasa
16. Encuentre la ecuación
c) (–2, 5) , (2, 5) de la recta que pasa
por los puntos dados.
d) (–3, 8) , (–3, 3) Graficarla. por el punto (3, –1) y
que corte al eje X en un
e) (0, 0) , (1, 1) a) (1, 1) , (3, 3) ángulo de 45º.
f) (0, 0) , (–1, 1)
b) (1, 5) , (3, –2) 17. Encuentre la ecuación
g) (3, 5) , (6, 8) de la recta que pasa por
c) (6, 6) , (6, –1)
h) (–1,5 ; 2,5), (–3,5 ; – 6,5) el punto (4, 2) y forma
3 4 6 3
d) (4, 3) , (3, 4) un ángulo de 150º con
i) , , ,
4 3 3 2
e) (3, 9) , (0, 9) el eje X.
j) (6, 1) , (1, –1)
f) (0, 0) , (–5, –3) 18. Encuentre la ecuación de
k) (–3, 5) , (5, –3) la recta cuya pendiente
l) (0, 5) , (3, 0) g) (–3, 1) , (5, –1) es igual a la pendiente de
la recta cuya ecuación es
h) (4, 3) , (4, 7)
11. Determine la pendiente y 2x – 5y – 3 = 0 y que
el coeficiente de posición i) (1, 12) , (–5, 12) pasa por el origen.

Relaciones y funciones 169

168-169. 169 08/11/2001, 16:47


Ejercicios
b) m = 2 k=–3 a) (5,1) (3, 2) (– 5, 6)
1
19. Encuentre la ecuación de c) m = – 5 k= –
2 b) (2,1) (3, 5) (6, 3)
la recta cuyo coeficiente 3
c) (1, 6) (2, 5) (3, 4)
d) m = – 1 k=
de posición es –2 y forma 4
1
un ángulo de 135º con e) m = k=6 d) (– 1, 3) (0, 1) (2, 0)
2
el eje X. 4 e) (4, 0) (4, 1) (4, –3)
f) m= k=–2
3
g) m = –
3
k=2 f) ( – 7, 8) ( –1, 4) (8, 10)
20. Encuentre la ecuación de 5
la recta que pasa por el h) m = 0 k=1 g) ( – 1, 8) ( – 7, 4) (2, – 10)
origen y forma un ángulo
i) m=5 k=0 h) (0, 0) (1, 1) (1, 2)
de 30º con el eje X.
1 1
j) m= k=
2 2 28. Determine la ecuación y
21. Encuentre la ecuación de 3
k) m = – k=–3 grafique la recta paralela
la recta que pasa por el 4
origen y forma un ángulo l) m=–1 k=–1 al eje X que pase por el
de 60º con el eje X. punto (3, 5).

25. Determine la pendiente,


22. Encuentre la ecuación de la ecuación y el coefi- 29. Encuentre la ecuación de la
la recta que pasa por el recta que sea paralela al eje
ciente de posición de la
origen y forma un ángulo X y que pase por el punto
recta que pasa por los
de 120º con e l eje X. (7, –3). Grafíquela.
puntos (3, 2) y (2, – 5).
Grafique.
23. Grafique la recta dados un 30. Determine la ecuación de la
punto y su pendiente. recta que sea paralela al eje
26. Dada la ecuación de una
Y y que pase por el punto
a) (3, –2) m = 1 recta, determine su pen-
(–5, –8). Grafíquela.
diente, su coeficiente de
b) (6, –4) m = – 2
posición y grafíquela.
2
c) (–3, 1) m = 3 31. Encuentre la ecuación de la
4
a) 2 x – y + 1 = 0 recta que sea perpendicular
d) (–2, –3) m = 5 al eje X y que pase por el
e) (1, 1) m = –1 b) 4 x –3 y – 2 = 0
punto (–1, 3).
f) (4, 0) m = – 1 c) x + 5 = 1
2
g) (0, 3) m = 5
32. Encuentre la ecuación de la
h) (0, 0) m = 0 d) 3 x – y + 6 = 0
recta que sea perpendicular
i) (1, 5) m = 0
e) 1 – x = y + 2 al eje Y y que pase por el
j) (3, 4) m = – 3
2 punto (6, 16).
k) (4, –1) m = 5
1 f) 1 = 2 – y

l) (6, 1) m = indefi- g) x + 6 = y 33. Determine si las rectas L1


nida y L2 son o no paralelas.
h) 3 x –1 = y – 1
Justifique la respuesta.
24. Grafique la recta conocida 27. Determine si los siguien- a) L1 : 2 x – 3 y + 4 = 0
su pendiente m y su coefi- tes tríos de puntos son L2 : – 4 x + 6 y – 1 = 0
ciente de posición k. colineales; justifique la b) L1 : 5 x – y = 1
a) m = 3 k=1 respuesta. L2 : 10 x – 1 = 2 y

170 Relaciones y funciones

170-171. 170 08/11/2001, 16:48


CAPÍTULO 3

c) L1 : 2 x + 5 y – 1 = 0 37. Halle la ecuación de (3, –1) y (4, 2) es paralela


L2 : 2 x – 5 y + 3 = 0 la recta que pasa por a la recta que une los
el punto (4, –2) y es puntos (–1, – 4) y (2, 5).
d) L1 : 1 – 3 x = y perpendicular a la recta
cuya ecuación es: 44. Pruebe que una recta
L2 : 3 y = x + 2
2x – 3y + 1 = 0. de pendiente m = 4 es
3

e) L1 : 4 x – y + 1 = 3 paralela a la recta que


L2 : y – 4 x = 0 38. Determine la ecuación de la pasa por los puntos
recta que intersecta al eje X (–1, 4) y (3, 7).
f) L1 : 1 – 3y = x en (3, 0) y es perpendicular
a la recta cuya ecuación
L2 : 4y – x = 5 45. Demuestre que la recta
es: x + y = 3.
que une los puntos
34. Determine si las rectas
(2, 3) y (4, –1) es perpen-
L1 y L2 son perpendicu- 39. Determine el valor de
k en la ecuación dicular a la recta que pasa
lares. Justifique la res-
5x – (1 + k) y – 3 = 0 por los puntos (5, 2) y
puesta.
para que represente una (3, 1).
a) L1: 2x – y + 3 = 0 recta paralela a la recta
L2: x + 2y – 3 = 0 cuya ecuación es: 46. Demuestre que la recta
x –5y + 8 = 0. cuyo coeficiente de posi-
b) L1: 5x + 2y – 1 = 0
ción es 2 y corta al eje X
L2: 0,2x – 0,5y = 0
40. Determine el valor de k en (2, 0) es perpendicular
en la ecuación a la recta de igual coefi-
c) L1: 3x + y – 6 = 0
2kx – 3y + 1 = 0 para ciente de posición pero
L2: 3x – y + 6 = 0
que represente una recta corta al eje X en (–2, 0).
paralela a la recta cuya
d) L1: 2x – 3y + 1 = 0
ecuación es:
L2: 4x – 6y + 5 = 0 47. Determine la ecuación
– 2x + 5y – 3 = 0.
de la familia de rectas
e) L1: x + y = 0 que tiene pendiente –3.
L2: x – y = 0 41. Encuentre el valor de k
Grafique tres de ellas.
en la ecuación
f) L1: x + y + 5 = 0 (2 – k) x + 3y – 4 = 0
L2: x – y + 5 = 0 para que represente una 48. Determine la ecuación
recta perpendicular a de la familia de rectas
la recta cuya ecuación perpendiculares a la
35. Determine la ecuación
es: – 6x + y – 9 = 0. recta cuya ecuación es:
de la recta que pasa por
3x – 2y + 1 = 0.
el punto (2, –1) y es
paralela a la recta cuya 42. Encuentre el valor de k
ecuación es: en la ecuación 49. Determine la ecuación de
2x + 6y – 1 = 0. Grafique. x – 2ky – 3 = ky + k la familia de rectas que
para que represente una pasa por el punto (3, –1).
36. Determine la ecuación recta perpendicular a la Grafique tres de ellas.
recta cuya ecuación es:
de la recta que tiene
2y + 3 = 9x.
coeficiente de posición 50. Determine la ecuación de la
–3 y es paralela a la familia de rectas que tienen
recta x + y – 1 = 0. 43. Demuestre que la recta coeficiente de posición 3.
Grafique. que pasa por los puntos Grafique tres de ellas.

Relaciones y funciones 171

170-171. 171 08/11/2001, 16:48


Ejercicios
51. En un diagrama cartesiano (sistema de ejes 52. En la figura siguiente fije la escala
coordenados), grafique los siguientes de valores en los ejes coordenados
segmentos de rectas en los intervalos y determine las ecuaciones correspon-
indicados. Grafique además los puntos dientes a los segmentos que forman
P y Q. la figura.
P (4,2 ; 4,8)
Q (5,8 ; 4,8)

y=1 3,5 ≤ x ≤ 4
6 ≤ x ≤ 6,5
x=4 1 ≤ y ≤ 3
x=6 1 ≤ y ≤ 3
y = 3,6 4,5 ≤ x ≤ 5,5
y=3 2 ≤ x ≤ 8
y=6 4 ≤ x ≤ 6
3
y= x 2 ≤ x ≤ 4
2
3 6 ≤ x ≤ 8
y = – x + 15 53. Haga un diseño compuesto por segmen-
2
y = 2x – 5 5,5 ≤ x ≤ 6 tos de rectas y determine las ecuaciones
correspondientes. Comparta el trabajo
y = –2x + 15 4,0 ≤ x ≤ 4,5
con sus compañeros y compañeras.
x=5 6 ≤ y ≤ 7

Soluciones

1. a) 4, 6, 1 b) 3, –1, 5 c) 2, –1, – 5
d) – 2, 6, –3 e) 4, 1, 0 f) 6, –2, 0
g) 4, 0, –1 h) – 5, 0, 1 i) 0, – 6, 1
1 1
j) 0, 2, –3 k) 3
, , –1 l) 0,5 , –2,5 , –1
4
m) 2, –2, 1 n) a, – 2b, – c o) a – b, ab, – b
p) – m, 1, – k q) a, – b, – 2a – b r) 0, 1, 3k – 1
s) 2, 1 – k, – 4 t) 1, 1, – 4 – k

2. a) y = – 2 x – 1
b) y = 3x + 5 c) y = 2x – 5
3 6
1 1
d) y = x+ 2
e) y = – 4x f) y = 3x
3
1
g) No se puede porque el h) No se puede porque el i) y=
6
coeficiente de y es 0 coeficiente de y es 0

172 Relaciones y funciones

172-173.(2003) 172 20/11/02, 10:58 AM


CAPÍTULO 3

3 4
j) y = k) y = – x + 4 l) y = 0, 2x + 0, 4
2 3
o) y = b – a x + 1
1
m) y = x + 2
n) y = – a x + c
2b 2b ab a
a 2a + b
p) y = mx + k q) y = x+ r) y = 1 – 3k
b b
2 4
s) y= x+ t) y=–x+4+k
k–1 1–k

3. a) Por ejem: (0, 2), (1, 6) b) Por ejem: (1, 4), (2, 10) c) Por ejem : (2, a), (2, b)
d) Por ejem : (1, 6), (2, 12) e) Por ejem: (1, 5), (3, 6) f) Por ejem : (–1, 2), (–1, 3)

g) (a, 2), (k, 2) h) (2, 0) , 4, 4 i) (1, –17), (–1, –19)


3
1 1
j) ,4 , ,5
4 3

1
4. a) (0, 3) b) ,4 , (2, 1), (3, –1) c) (1, 2), (2, 2)
2

d) (0, 0), (1, 4) e) (1, 1), (3, 2) f) (–1, 1)

5. a, c, d, e, h

1
6. a) 1 b) 12 c) –9 d) – e) k puede tomar cualquier valor
7

f) Cualquiera que sea el valor de k, la recta no pasa por (1, 1)


5
g) –5 h) 2
i) 0

3 1
8. a) (0, 5) (5, 0) b) 0, – ,0 c) (0, 2) (–10, 0)
2 2
5 1
d) (0, 6) (–3, 0) e) 3
,0 No corta al eje Y f) 0, –
2
No corta al eje X
1 1 5
g) (0, 0) h) 0, –
3 4
,0 i) 6
,0 No corta al eje Y
1 1–k k–1
j) 0, No corta al eje X k) (0, – a) (b, 0) l) 0, ,0
2 3 4
3 1
m) 0, –
2
(3, 0) n) (0, 1) (–1, 0) o) 0,
2
(1, 0)
1
p) (2, 0) No corta el eje Y q) 0, – (1, 0) r) (0, – 6) (9, 0)
2

9. a) pasa por el origen b) paralela al eje Y c) corta a ambos en puntos distintos


d) corta a ambos ejes en puntos distintos e) paralela al eje Y
f) paralela al eje X g) paralela al eje X h) pasa por el origen
i) corta a ambos ejes j) pasa por el origen k) pasa por el origen.

l) Si k = 1 pasa por el origen. Si k ≠ 1 corta ambos ejes en puntos distintos


m) corta a ambos ejes en puntos distintos n) corta a ambos ejes en puntos distintos
o) corta a ambos ejes en puntos distintos p) paralela al eje Y
q) corta a ambos ejes r) pasa por el origen
1
10. a) –1 b) – c) 0 (recta å eje X) d) Indeterminada (recta å eje Y) e) 1
2
2 2 5
f) –1 g) 1 h) 4, 5 i) 15
j) 5
k) –1 l) –
3

Relaciones y funciones 173

172-173.(2003) 173 20/11/02, 11:00 AM


Soluciones

1
11. a) m = –3 k=2 b) m = – k=1 c) m = –1 k=3
5
1 2
d) m = 3 k = –5 e) m = k= f) m=1 k=5
5 5
3 1 3
g) m = –
2
k= 2
h) m = 5
k=0 i) m=2 k=0

j) m = Indeterminada k) m = – 1 k=2 l) m = Indeterminada


k = No existe. k = No existe
2 2 3 3
m) m = 0 k=1 n) m = k= 3
o) m = – k= 4
3 8

p) m = 5 k=2

12. a) x – y = 0 b) 7x + 2y – 17 = 0 c) x – 6 = 0 d) x + y – 7 = 0
e) y – 9 = 0 f) 3x – 5y = 0 g) x + 4y – 1 = 0 h) x – 4 = 0
i) y – 12 = 0 j) x – 7y = 0 k) y + x + 7 = 0 l) y – 5x = 0

13. a) x – y + 1 = 0 b) 4y – 3x + 7 = 0 c) 3x + y – 11 = 0 d) 4x + 3y – 15 = 0
e) y – 5 = 0 f) x + 2y – 1 = 0 g) 5x – y + 30 = 0 h) 2x – y + 3 = 0

14. 3x – 5y – 15 = 0 15. 3x – 4y = 0 16. x – y – 4 = 0

17. x + 3 y – 2 3 – 4 = 0 18. 2x – 5y = 0 19. x + y + 2 = 0

20. x – 3 y = 0 21. 3 x – y = 0 22. 3 x + y = 0

25. m = 7 7x – y – 19 = 0 k = – 19

c) m = Indeterminada
4 2
26. a) m = 2 k=1 b) m = 3
k=– d) m = 3 k = 6
3 k = No existe
e) m = –1 k = –1 f) m=0 k=1 g) m = 1 k=6 h) m = 3 k = 0

27. a) Sí, los tres satisfacen la ecuación x + 2y – 7 = 0 b) No son colineales


c) Sí, tomados de dos en dos producen la misma pendiente d) No son colineales
e) Sí, los tres satisfacen la ecuación x – 4 = 0 f) No son colineales
g) No son colineales h) No son colineales
28. y – 5 = 0 29. y + 3 = 0 30. x + 5 = 0 31. x + 1 = 0 32. y – 16 = 0
2
33. a) L1 å L2 porque m1 = m2 = 3
b) L1 å L2 porque m1 = m2 = 5
2 2 1
c) L1 ç L2 porque m1 = – y m2 = 5
d) L1 ç L2 porque m1 = –3 y m2 =
5 3
1 1
e) L1 å L2 porque m1 = m2 = 4 f) L1 ç L2 porque m1 = – y m2 =
3 4

1 5 0,2
34. a) L1 ß L2 porque m1 • m2 = 2 •– = –1 b) L1 ß L2 porque m1 • m2 = • = –1
2 2 0,5
c) L1 ® L2 porque m1 • m2 = –3 • 3 = –9 ≠ –1
2 4 8
d) L1 ® L2 porque m1 • m2 = 3

6
= 18
≠ –1

174 Relaciones y funciones

174-175.(2003) 174 20/11/02, 11:04 AM


CAPITULO 3
CAPÍTULO

e) L1 ß L2 porque m1 • m2 = –1 • 1 = –1

f) L1 ß L2 porque m1 • m2 = –1 • 1 = –1

35. x + 3y + 1 = 0 36. x + y + 3 = 0 37. 3x + 2y – 8 = 0 38. y – x + 3 = 0


3 3 3
39. k = 24 40. k = 5
41. k = 2
42. k = –
2

43. Se demuestra calculando las pendientes (m1 = m2).

44. Se demuestra calculando la pendiente de la recta que pasa por (–1, 4) y (3, 7).

45. Se demuestra calculando las pendientes y verificando que su producto es –1.

46. Ídem 45. 47. y + 3x – k = 0 48. 2x + 3y – k = 0 49. y – mx + 1 + 3m = 0.


50. y – mx – 3 = 0.

3.4.3 Tipos de funciones


Función inversa
• Función inyectiva o uno a uno
Una función f : AQB se dice inyectiva o uno a uno si y sólo si
elementos distintos en A tienen imágenes distintas en B.

((f : AQB es uno a uno


) (
• Función epiyectiva o sobre
¤
(a ≠ b ⇒ f (a) ≠ f (b)
o
f (a) = f (b) ⇒ a = b )
Una función f : AQB se dice epiyectiva o sobre si y sólo si todo
elemento de B es imagen de algún elemento de A.
(f : AQB es sobre) ¤ (Rang f = B).
• Función biyectiva
Una función f : AQB se dice biyectiva si y sólo si f es uno
a uno y sobre.
• Función inversa
Toda función f : AQB posee una relación inversa f –1 : BQA.
Esta relación inversa f –1 : BQA es función si y sólo si f es
función biyectiva.
Observación: (f o f –1) (x) = I(x)
(f –1 o f) (x) = I(x)
La composición de una función con su inversa da la función
idéntica.

Relaciones y funciones 175

174-175.(2003) 175 20/11/02, 11:04 AM


Ejercicios 1. Sean fi las funciones definidas por los siguientes diagramas.
resueltos Determinar si fi es uno a uno, sobre o biyectiva.
f1 A f2 B
a) A B b
1 5 1 5
2 6 2 6
3 7 3
7
4 8 4

c) f3 d A f4 B
A B
1 5 1 5
2 6 2 6
3 7 3 7
8 4 8

Solución:
a) f1 no es uno a uno porque 7 es imagen de dos elementos
distintos, el 2 y el 3.
f1 no es sobre porque 6 E B no pertenece al Rang de f1.
f1 no es biyectiva porque no es uno a uno ni sobre.
b) f2 no es uno a uno ya que f2(3) = 7 y f2(4) = 7 y 3 ≠ 4.
f2 es sobre porque Rang f2 = B.
f2 no es biyectiva porque no es uno a uno.
c) f3 es uno a uno porque imágenes distintas corresponden
a pre-imágenes distintas.
f3 no es sobre porque Rang f3 = {5, 6, 7} ≠ B, el 8 no es
imagen de ningún elemento de A.
f3 no es biyectiva porque no es sobre.
d) f4 es uno a uno porque imágenes distintas corresponden
a pre-imágenes distintas
f4 es sobre porque
Rang f4 ={5, 6, 7, 8} = B.
f4 es biyectiva porque es uno a uno y sobre.

2. Sea f : ZQZ la función definida por f(x) = 4x – 2. Determinar si f


es biyectiva, si no lo es, redefinirla para que lo sea.
Solución:
f es uno a uno: Sean a, b E Z.
f(a) = f(b) ⇒ 4a – 2 = 4b – 2 /+2
⇒ 4a = 4b /: 4
⇒ a =b
luego f es uno a uno.
f no es sobre ya que f(x) = 4x – 2 ⇒ 4x = f(x) + 2

x = f(x) + 2
4
luego 1 E Z no es imagen de ningún entero ya que 1 sería imagen

176 Relaciones y funciones

176-177. 176 08/11/2001, 16:56


CAPÍTULO 3

1+2 3
de x = = ∉Z
4 4
\ f no es biyectiva porque no es sobre.
Para que f sea biyectiva debe definirse sobre su rango y con eso
aseguramos que es sobre:
y+2
Rang f = {y E Z / E Z} = {y E Z / y = ± 2(2n – 1), n E N}
4
∴ f : Z Q {± 2 (2n – 1), n E N} definida por f(x) = 4x – 2 es sobre y
como era uno a uno, ahora podemos decir que es biyectiva.

3. Sea f : RQR una función definida por :

{
x – 1 si x < 2
f(x) = x + 2
2 si x ≥ 2
Determinar si f es biyectiva, si no lo es restringir su dominio
y/o rango para que lo sea. Graficar f(x).
Solución:
f es uno a uno.
i) Sean a, b < 2
f(a) = f(b) ⇒ a – 1 = b – 1 ⇒ a = b
ii) Sean a, b ≥ 2
a+2 b+2
f(a) = f(b) ⇒ = fia=b
2 2
iii) a < 2 ⇒ a – 1 < 2 – 1 ⇒ f(a) < 1

b ≥ 2 ⇒ b + 2 ≥ 2 + 2 ⇒ f(b) ≥ 2
2 2 f(x)
∴ a ≠ b ⇒ f(a) ≠ f(b)
3
luego f es uno a uno.
2
f no es sobre porque como veíamos: 1
si x < 2 ⇒ f(x) < 1
1 2 4 x
si x ≥ 2 ⇒ f(x) ≥ 2
Luego [1, 2) D Rang f
∴ f no es biyectiva.
Para que la función sea biyectiva debemos definirla de RQR – [1, 2).

4. Sea f una función real definida por


si x ≤ 4

{
x+1
f(x) = 2 4 – 3x
2 si x > 4
Determinar si f es biyectiva. Si no lo es restringir su dominio y/o
su recorrido para que lo sea. Graficar f.
Solución:
f no es uno a uno.
i) Sean a, b ≤ 4
f(a) = f(b) ⇒ a + 1 = b + 1 ⇒ a = b
ii) Sean a, b > 4

Relaciones y funciones 177

176-177. 177 08/11/2001, 16:57


2 4 – 3a 24 – 3 b
Ejercicios f (a) = f (b) ⇒ 2
=
2 ⇒a=b
resueltos
iii) a £ 4 ⇒ a + 1 £ 5 ⇒ f (a) £ 5
b > 4 ⇒ –3b < – 12 / + 24
24 – 3b < 12 /:2
24 –3b
< 6 ⇒ f (b) < 6
2
∴ a ≠ b © f (a) ≠ f (b)
en efecto, sea f (a) = 4 ≤ 5 ∧ f (b) = 4 < 6
f (a) = a + 1 = 4 ⇒ a = 3
24 –3b 16
f (b) = 2 = 4 ⇒ 24 – 3b = 8 ⇒ –3b = – 16 ⇒ b = 3

hemos probado que f (a) = 4 ⇒ a = 3


16
f (b) = 4 ⇒ b = 3

así con f (a) = f (b) tenemos a ≠ b


luego f no es uno a uno.
f no es sobre, ya que si x ≤ 4 Q f (x) ≤ 5
si x > 4 Q f (x) < 6
f(x) ∴ Rang f = {y E R / y < 6} ≠ R
6 para que f fuera biyectiva podríamos definirla de
5
(– ∞, 4] Q (– •, 5] o de (4, + ∞) Q (– •, 6)

4 8 x

5. Sea f una función real definida por

{
4 –x
2 si x < 2
f (x) =
3 – x si x ≥ 2
Determinar si f es biyectiva. Si no lo es, redefinirla, de modo que lo
sea. Hallar una fórmula para f –1. Graficar f y f –1.
Solución: f es uno a uno:
i) Sean a, b < 2
4 –a 4 –b
f (a) = f (b) ⇒ = ⇒a=b
2 2
ii) Sean a,b ≥ 2
f (a) = f (b) ⇒ 3 – a = 3 – b ⇒ a = b
iii) a < 2 ⇒ – a > – 2 /+ 4
4–a>2 /: 2
4 –a 4 –b
=> 1 ⇒ f(a) > 1
2 2

178 Relaciones y funciones

178-179..indd 178 01-02-2006 9:24:33


CAPÍTULO 3

b≥2⇒–b≤–2 /+ 3
3 – b ≤ 1 ⇒ f (b) ≤ 1
\ a ≠ b ⇒ f (a) ≠ f (b)
luego f es uno a uno.
f es sobre ya que si x < 2 ⇒ f (x) > 1
y si x ≥ 2 ⇒ f (x) ≥ 1
Así todos los elementos de R son imagen de algún elemento de R.
∴ f es biyectiva, lo que implica que f tiene función inversa f –1.

Encontremos ahora una fórmula para la función f –1.


Si x < 2 ⇒ f (x) > 1
llamemos y = f (x)
4 –x
y= 2 ⇒ 2y = 4 – x ⇒ x = 4 – 2y

x = f –1 (y) = 4 – 2y si y > 1
Si x ≥ 2 ⇒ f (x) ≤ 1
llamemos y = f (x)
y=3–x⇒x=3–y
x = f –1 (y) = 3 – y si y ≤ 1
Entonces la fórmula para f es

f –1 (y)=
{ 4 – 2y si

3 – 2y si
y>1

y≤1

que usando la variable x queda:


f
f –1

2
f(x)

{
1
4 – 2x si x>1
f –1 (x)= 1 2 3 x
3 – x si x≤1
y = f (x)
Nótese que los gráficos de f y f –1 son simétricos res- I(x)
y = f -1(x)
pecto de la recta x = y.

6. Sea f una función real biyectiva definida por:


5x+3
f (x) = 2

Hallar una fórmula para f –1.


So lución:
Sabemos que (f o f –1) (x ) = I (x).
Entonces: (f o f –1) (x ) = f (f –1 (x) = x
–1
5f x + 3
2 =x

5f–1 (x) + 3 = 2x
5f–1 (x) = 2x – 3
2x– 3
f –1 (x) =
5
Ésta es la fórmula pedida.

Relaciones y funciones 179

178-179..indd 179 01-02-2006 9:24:37


Ejercicios 7. Sea f una relación real definida por f (x) =
x–1
resueltos x+3
Hallar dominio y recorrido de f para que sea una función
biyectiva y encontrar una fórmula que defina f –1.
Solución:

Dom f = {x E R / x ≠ – 3} Si x = – 3, f (x) no existe, ya que – 4


0
está indeterminado.
Para determinar el rango de f vamos a despejar x en función de f (x)
para identificar posibles f (x) que indeterminan la expresión:
x–1
f (x) =
x+3
x f (x) + 3 f (x) = x – 1
x f (x) – x = – 1 – 3 f (x)
x (1 – f(x) ) = 1 + 3f (x)
1 + 3 f (x)
x =
1 – f (x)
1 + 3y
si f (x)= y Q x =
1–y
Es decir, el 1 no es imagen, luego: Rang f = {x E R / x ≠ 1}
así f : R - {– 3} Q R - {1}
Encontremos ahora una fórmula para f –1.
(f o f –1) (x) = I (x) = x
f (f –1(x)) = x
–1
f (x) – 1
=x
–1
f (x) + 3
f –1 (x) – 1 = x f –1 (x) + 3x
f –1 (x) – x f –1 (x) = 3x + 1
f –1 (x) (1 – x) = 1 + 3x
1 + 3x
f –1 (x) =
1–x
lo que nos da la misma expresión que encontramos para hallar
el rango de f. Esto se debe a que f –1 está definida del Rang f al
Dom f, luego:
f –1 : R - {1} Q R - {–3}

Ejercicios

1. Sean A = {1, 2, 3, 4} y B = {a, b, c, d}


Determine cuáles de las funciones defi nidas en los
siguientes diagramas son uno a uno, sobre, y/o biyectivas.
Justifi que las respuestas.

180 Relaciones y funciones

180-181. 180 08/11/2001, 15:23


CAPÍTULO 3

f A f B
a) A B b)
1 a
1
2 b
2 a
3 c
3
4 d

f A f B
b) A B c)
1 a 1
a
2 b 2
b
3 c 3
c
4 d 4

f
c) A B d) A f B
1 a a
2 b b 1
3 c c 2
4 d d 3

A f B
d) e) A f B
1 a a
1
2 b b
2
3 c c
3
4 d

A f B f
e) f) A B

1 a 1 2
2 b 3 4
3 c 5 6
4 d

A f B 3. Sean A = {1, 2, 3, 4} y B = {5, 6, 7}


f)
¿Se puede definir una función biyectiva
1 a de A en B? Represéntelo mediante un
2 b
c
diagrama.
3
4 d 4. Sean A = {a, e, i, o, u} y B = {1, 2,
3, 4, 5} ¿Se puede definir una función
2. Dadas las funciones defi nidas en biyectiva de A en B? Represéntelo
los siguientes diagramas, determine mediante un diagrama.
cuáles son biyectivas. Justifi que las
respuestas. 5. Sea f : Z Q Z una función definida por
a) f
f (x) = 2x – 1. Indique si f es uno a uno,
A B sobre o biyectiva.
a 6. Sea f : R Q R una función definida
a
b
b 5x – 3
c por f (x) = . Si f es biyectiva,
c d 2
encuentre una fórmula para f –1.

Relaciones y funciones 181

180-181. 181 08/11/2001, 15:23


Ejercicios A f B
7. Sea g : RQR una función definida
por g(x) = 4x + 3. a 1
b 2
Si g es biyectiva, encuentre una
fórmula para g –1. c 3
d 4
8. Sea A = {x E Z / – 5 ≤ x ≤ –2}
B = {x E Z / – 2 < x < –2}
13. De las funciones definidas en los
C = {x E Z / 3 ≤ x < –5}
diagramas siguientes, determine cuáles
y sean las funciones: poseen función inversa.
f1 : A Q Z f1 (x) = x2 – 1
A f1 B
f2 : B Q Z f2 (x) = x2 + 2 a)
f3 : C Q Z f3 (x) = 32 – x2 a 1
b 2
Determine cuáles de estas funciones c 3
son uno a uno (inyectivas).
d 4
9. Sea A = {x E R / – 3 ≤ x ≤ 3}
A f2 B
Se dan las funciones de A en A b)
definidas por : x 1
f(x) = x2 – 2 y g(x) = x3 + 1.
y 2
Determine cuáles de las funciones z 3
dadas son uno a uno y cuáles son
sobre. Intente bosquejar un gráfico
calculando valores. A f3 B
c)
10. Sea f : RQR definida por: a
1
b
– 2 si x ≥ –1 2

{
c
f(x) = 3 d
– 2x si x < –1
A f4 B
Grafique y determine si f es uno a d)
uno y/o sobre. x u
y v
11. Dadas las siguientes funciones, deter-
z w
mine dominio y recorrido de cada
una. En cada caso determine la
relación inversa e indique si ésta f5
es función. A B
e)
f1 = {(1, 3) (2, 4) (3, 5) (4, 6)} 1
2 1
f2 = {(1, 5) (2, 5) (3, 5) (4, 5) (5, 5)} 3
f3 = {(a, 1) (b, 2) (c, 3) (d, 4) (e, 5)}
A f6 B
f4 = {(b, 2) (a, 1) (r, 2) (c, 2) (o, 1)} f)
1 1
2 2
12. Dada la función f : A Q B definida
por el diagrama siguiente. 3 3
¿Es f –1 función?
Halle f –1(2), f –1(3) y f –1(4). 14. Sea f : ZQZp, Zp = {números pares}
Haga un diagrama de f –1. definida por f(n) = 2n.

182 Relaciones y funciones

182-183. 182 08/11/2001, 17:00


CAPÍTULO 3

a) Determine si esta función tiene 16. Sea A = {x E N / 3 < x < 9} y f: AQN


función inversa. definida por f(x) = x2 – 1.
b) Si f se definiera de ZQZ, ¿tendría a) Escriba f por extensión.
función inversa? b) Escriba f –1 por extensión.
15. Dadas las funciones reales definidas por c) Escriba f –1 (15) y f –1 (63).
los siguientes gráficos. Determine cuáles
d) ¿Es f –1 una función?
de ellas poseen función inversa.
17. Dada la función real f(x) = 4x – 8.
Determine si f posee función inversa
a)
3 y encuentre una fórmula que la defina.
2

1 18. Dada la función real definida por

{
x–2
si x ≤ 3
–3 –2 –1 1 2 3 2
–1 f(x) =
–2
x – 2 si x > 3
–3 a) Grafique y determine Rang f.
b) Demuestre que f es uno a uno.
c) Demuestre que f no es sobre.
b)
3
19. Dada la función real definida por:
2
2x + 5 si x ≤ – 1.

–3 –2 –1
1

–1

–2
1 2 3
f(x) =
3{ si x > – 1.

a) Grafique y encuentre Rang f.


–3
b) Determine si f es uno a uno.
c) Determine si f es sobre.
c) d) Encuentre, si es posible,
3
la función f –1.
2
20. Dada la función real definida por:
1

{
9 – 2x
–3 –2 –1 1 2 3
si x < 3.
f(x) = 3
–1

–2 4 – x si x ≥ 3.
–3 a) Grafique.
b) Demuestre que f es biyectiva.

d) c) Encuentre la fórmula que define f–1.


3
21. D ada la función real definida por:
2

{
1 2x + 4 si x ≤ – 3.
f(x) =
–3 –2 –1 1 2 3 2x – 4
–1
si x > – 3.
5
–2

–3
a) Grafique.
b) Demuestre que f es biyectiva.

Relaciones y funciones 183

182-183. 183 08/11/2001, 17:00


Ejercicios a) Determine dominio y rango para
que f sea una función biyectiva.
c) Encuentre la fórmula que define
b) Encuentre una fórmula para f –1
f –1.
22. Dada la función real definida por: 24. Dada la relación f : RQR definida por

f(x) = 2 x + 1

{
x+2 si x < – 1. x–3
f(x) =
2–x
si x ≥ – 1. a) Determine dominio y recorrido de f
3
para que sea función.
a) Grafique y determine Rang f. b) Encuentre una fórmula para f –1.
b) Demuestre que f es uno a uno.
25. Dada la relación g : RQR defini -
c) Demuestre que f no es sobre. 1 – 2x
da por g(x) =
x
23. Dada la relación f : RQR definida por
x–1 a) Determine dominio y recorrido de
f(x) = g para que sea función.
x+2
b) Encuentre una fórmula para g –1.

Soluciones
biyectiva de A en B porque para ello se
1. a) f no es uno a uno, b es imagen de necesita que ambos conjuntos tengan
1, 2 y 3. la misma cardinalidad (cantidad de
f no es sobre, a, c no son imagen. elementos).
b) f es uno a uno, cada elemento de B es 4. Es posible. Ej.:
imagen de un solo elemento de A.
f es sobre, Rang f = B, f es biyectiva. A f B
c) f no es uno a uno. a 1
f(1) = a A f(2) = a A 1 ≠ 2. e 2
f no es sobre, b y d no son imagen. i 3
o 4
d) f no es uno a uno. u 5
f(3) = f(4) = d A 3 ≠ 4.
f no es sobre, a no es imagen.
5. f es uno a uno.
e) f es uno a uno, f(a) ≠ f(b) ⇒ a ≠ b.
f es sobre, Rang f = B. f no es sobre (4 no es imagen de ningún
f es biyectiva. elemento de Z).

f) f es uno a uno, f es sobre, f es f no es biyectiva.


biyectiva. 2x + 3
6. f es biyectiva f –1(x) =
2. a) f es uno a uno, f no es sobre. 5
b) f no es uno a uno, f es sobre. x–3
7. g es biyectiva g –1(x) =
c) f no es uno a uno, f no es sobre. 4
d) f no es uno a uno, f es sobre. 8. f1 y f3
e) f es uno a uno y sobre, f es biyectiva. 9. f no es uno a uno ni sobre.
f) f es uno a uno y sobre, f es biyectiva. g es uno a uno y no es sobre.
3. No es posible definir una función 10. f no es uno a uno, f no es sobre.

184 Relaciones y funciones

184-185. 184 08/11/2001, 17:01


CAPITULO 3
CAPÍTULO

11. Dom f1 = {1, 2, 3, 4} 17. f posee inversa porque es biyectiva.


Rang f1 = {3, 4, 5, 6} f –1 (x) = x + 8
4 f(x)
f1 es biyectiva. 18. a)
f1–1 = {(3, 1) (4, 2) (5, 3) (6, 4)}
1
es función.
2 3 4 x
Dom f2 = {1, 2, 3, 4, 5}
Rang f2 = {5} f2 no es biyectiva.
–1
f2 = {(5, 1) (5, 2) (5, 3) (5, 4) (5, 5)} 1
Rang f = {f(x) E R / f(x) ≤ 2 V x > 1}
no es función. 19. a) f(x)
3
Dom f3 = {a, b, c, d, e}
Rang f3 = {1, 2, 3, 4, 5} f3 es biyectiva.
f3–1 = {(1, a) (2, b) (3, c) (4, d) (5, e)}
–3 –1 x
–1
es función.
Dom f4 = {b, a, r, c, o}
Rang f = {f(x) E R / f(x) ≤ 3}
Rang f4 = {1, 2}
b) f no es uno a uno.
f4 no es biyectiva.
c) f no es sobre.
f4–1 = {(2, b) (1, a) (2, r) (2, c) (1, 0)}
d) no hay función inversa porque f no
no es función. es biyectiva.
12. f –1 es función porque f es biyectiva.
20. a)
f –1(2) = a f –1(3) = b f –1(4) = c f(x)
–1 3
B f A
1
1 a
2 b 3 4 x

3 c
c)

{
4 d 9 – 3x
si x > 1
f –1(x) = 2
13. f1, f4, f6 4-x si x ≤ 1
14. a) f tiene inversa.
21. a) f(x)
b) f no tendría inversa porque no sería
sobre. –4 2 x

15. a) tiene inversa. –2


b) tiene inversa.
c) no tiene inversa.
x–4

{
d) no tiene inversa. c) si x ≤ 2
2
16. a) f = {(4, 15), (5, 24), (6, 35), (7, 48), f –1(x) =
5x + 4
(8, 63)} si x > –2
2
b) f –1= {(15, 4), (24, 5), (35, 6), f(x)
(48, 7), (63, 8)} 22. a)
2

c) f –1(15) = 4; f–1(63) = 8
d) f –1 no es función de N Q A pero sí es –2 –1 2 x
función de {15, 24, 35, 48, 63} en A.
Rang f = {f(x) E R / f(x) ≤ 1}

Relaciones y funciones 185

184-185. 185 08/11/2001, 17:01


Soluciones

23. a) Dom f = R – {–2} Rang f = R – {1} 25. a) Dom g = R – {0} Rang g = R – {–2}
2x+1
b) f –1 (x) = 1–x b) g –1 (x) = x +1 2
24. a) Dom f = R – {3} Rang f = R – {2}
3x+1
b) f –1 (x) =
x–2

3.4.4 Funciones de primer grado simultáneas.


Sistema de ecuaciones de primer grado
Una función y = f (x) de primer grado representa una recta. Cualquier
ecuación de primer grado en dos variables ax + by + c = 0 donde b ≠ 0
se puede escribir como una función y = f (x). Si b = 0 estamos frente
a una ecuación de primer grado de la forma ax + c = 0 con a ≠ 0, que
representa una recta paralela al eje Y.
Rectas secantes son aquellas representadas por ecuaciones que
tienen una solución común. Se cortan en un punto.
Rectas paralelas son aquellas representadas por ecuaciones que
no tienen ninguna solución común. No se cortan. Sus ecuaciones
constituyen un sistema inconsistente.
Rectas coincidentes son aquellas representadas por ecuaciones que
tienen todos sus puntos en común, es decir, infinitas soluciones
comunes. Sus ecuaciones forman un sistema indeterminado.

Definición:
Sean L1 y L2 dos rectas en el plano representadas por las ecuaciones
a1 x + b1 y + c1 = 0 y a2 x + b2 y + c2 = 0.
Las ecuaciones a1 x + b1 y + c1 = 0 y a2 x + b2 y + c2 = 0 se
llaman linealmente dependientes (L.D.) si y sólo si
a1 x + b1 y + c1 = k (a2 x + b2 y + c2) con k ≠ 0, k E R
En caso contrario las ecuaciones se denominan linealmente
independientes (L.I.).

RECTAS ECUACIONES SISTEMA 2 x 2 SOLUCIONES


secantes L.I.Simultáneas determinado única
coincidentes L.D. equivalentes indeterminado infinitas
paralelas L.I. incompatibles inconsistente no tiene

Para resolver geométricamente un sistema de ecuaciones se


grafican ambas rectas y luego se leen las coordenadas del punto de
intersección (si tiene solución única).

186 Relaciones y funciones

186-187. 186 08/11/2001, 15:25


CAPÍTULO 3

Para resolver algebraicamente un sistema hay varios métodos,


entre ellos: eliminación por reducción, eliminación por sustitución y
eliminación por igualación (ver problemas resueltos 5, 6 y 7).

Dos sistemas se llaman equivalentes si tienen la misma solución.

Una función de primer grado en dos variables z = f(x, y)


representa geométricamente un plano. Así, una ecuación de
la forma ax + by + cz + d = 0 con c ≠ 0 , puede ser escrita
en forma de función z = f (x , y); luego, toda ecuación ax + by + cz +
d = 0 representa un plano.
Tres ecuaciones con tres variables forman un sistema 3 x 3
y sus soluciones serán:

iii) Solución única si las tres ecuaciones son L.I. y los planos
tienen un punto en común.

iii) Infinitas soluciones si las ecuaciones que lo forman son L.D. y


los planos tienen infinitos puntos en común.

a) los tres planos coinciden.

b) los tres planos se intersectan formando una recta.

iii) Ninguna solución si las ecuaciones son L.I. y los tres planos son
paralelos, es decir, no tienen ningún punto en común.

Para resolver este tipo de sistemas se elimina una incógnita


combinando adecuadamente dos ecuaciones distintas dos veces y
con ello se obtiene un sistema 2 x 2 . Conociendo el valor de dos
variables, en cualquiera de las tres ecuaciones se puede encontrar
el valor de la variable eliminada.

Hay otro método, llamado método de eliminación de Gauss.

1. Determinar si las ecuaciones siguientes son L.I. o L.D. Ejercicios


a) L1 : 5x + 2y – 3 = 0 resueltos
L2 : 6x + 3y – 4 = 0
5 2 –3
Las ecuaciones son L.I., ya que ≠ ≠
6 3 –4
Las rectas L1 y L2 se intersectan en un punto, puesto que sus
pendientes son distintas. mL1 = – 5 mL2 = – 2
2
b) L1 : 4x – 3y + 2 = 0

L2 : –8x + 6y – 5 = 0

Relaciones y funciones 187

186-187. 187 08/11/2001, 15:25


4 –3 2
Ejercicios Las ecuaciones son L.I. porque – 8 = 6 π –5
resueltos Las rectas L1 y L2 no se intersectan, ya que representan rectas
paralelas; sus pendientes son iguales.
4 8 4
mL1 = 3
mL2 = =
6 3

c) L1 : 4x + 6y – 8 = 0
L2 : 6x + 9y – 12= 0

Las ecuaciones son L.D., ya que


4 6 8 3
= = y 6x + 9 y – 12 = (4x + 6y –8)
6 9 12 2
Las rectas L1 y L2 son coincidentes.
2. Determinar el o los valores de k en los siguientes pares de
ecuaciones para que sean linealmente independientes.

a) L1 : 5x + 2y – 1 = 0
L2 : kx – 2y – 1 = 0
Para que las ecuaciones sean L.I.
5 2 2 1
π o π
k –2 –2 1
Como la segunda desigualdad es verdadera, las ecuaciones
serán L. I. para cualquier valor real de k.
b) L1 : x + 2y – 6 = 0

L2 :– 3x – 6y + k = 0

Para que las ecuaciones sean L. I.,


1 2 2 –6
π o π
–3 –6 6 k
Como la primera desigualdad es falsa, debemos asegurarnos
de que la segunda sea verdadera; luego k debe ser distinto
de 18.

3. Determinar el o los valores de k para que el sistema dado sea


determinado. (Tenga solución única)
a) 2x – ky + 1 = 0
3x + 2y – 3 = 0

Para que el sistema tenga solución única 2 π – k


3 2
\k ≠ – 4
3

b) 4x + ky = 2
kx + 4y = 3

Para que el sistema tenga solución única 4 π k


k 4
∴ k ≠ 16 ⇒ k ≠ ± 4
2

188 Relaciones y funciones

188-189. 188 08/11/2001, 17:03


CAPÍTULO 3

4. Determinar si los pares de números reales dados son soluciones


del sistema dado:
1
a) 3x – 6y + 1 = 0 (2, – 1) , (0, )
6
x–y = 3

(2, – 1) no es solución porque 3 • 2 – 6 • (– 1) + 1 ≠ 0 no satisface la


primera ecuación, aunque sí satisface la segunda.
1
(0, ) no es solución porque no satisface la segunda ecuación,
6
aunque sí satisface la primera.
b) 2x – 4y + 1 = 0 (1, 3 ), (3, 7 ) , (9, 9)
4 4
–4x + 8y = 2

(1, 3 ) es solución porque satisface ambas ecuaciones.


4
(3, 7 ) es solución porque satisface ambas ecuaciones.
4
(9, 9) no es solución, ya que no satisface ninguna de las ecuaciones.

5. Resolver usando el método de eliminación por reducción el sistema.

3x–5y+4=0 (E1)
2x+6y–1=0 (E2)

Este método consiste en multiplicar ambas ecuaciones por factores


tales que una de las incógnitas quede con coeficientes iguales y de
signo contrario, y luego sumar miembro a miembro.

Multipliquemos: E1 • (– 2) y E2 • (3)

– 6 x + 10 y – 8 = 0
6 x + 18 y – 3 = 0

Sumamos miembro a miembro 28 y – 11 = 0


11
y =
Multipliquemos ahora E • 6 y E • 5 28
1 2

18 x – 30 y + 24 = 0
10 x + 30 y – 5 = 0

Sumamos miembro a miembro 28 x + 19 = 0


– 19
x=
28
Luego la solución del sistema es:
–19 11
,
28 28

Relaciones y funciones 189

188-189. 189 08/11/2001, 17:03


Ejercicios Comprobamos si el par
–19 11
, satisface ambas ecuaciones:
resueltos 28 28
E1: 3x–5y+4 = 0 E2 : 2x+6y–1 = 0
– 19 11 – 19 11
3 –5 +4 = 0 2 +6 –1 = 0
28 28 28 28
57 38 66
– – 55
28
+4 = 0 – + –1 = 0
28 28 28
57 55 112 28
– – 28
+ = 0 –1 = 0
28 28 28
112 112
– + = 0 1–1 = 0
28 28
0 ≡ 0 0 ≡ 0

6. Resolver usando el método de eliminación por sustitución el


sistema:
5x – 3 y + 2 = 0 (E1)
x – 4 y + 5 = 0 (E2)

Este método consiste en despejar de una de las ecuaciones


una de las incógnitas en función de la otra y sustituir su valor
en la otra ecuación, obteniendo así una ecuación con una
sola incógnita.
Despejemos en E2 el valor de x e introduzcamos su valor en E1.

x – 4 y + 5 = 0 ⇒ x = 4 y – 5 (E3)

Sustituyendo x en E1 5 (4 y – 5) – 3 y + 2 = 0

resolviendo: 20 y – 25 – 3 y + 2 = 0
17 y = 23
23
y = 17
Reemplazando y en E3 obtenemos el valor de x:
23
x=4 –5
17
92 85
x= –
17 17
7
x=
17
7 23
Luego, la solución del sistema es ,
17 17
7 23
Comprobemos si el par , satisface ambas ecuaciones:
17 17
E1: 5x–3y +2 = 0 E 2: x–4y+5 = 0
7 23 7 23
5 –3 +2 = 0 –4 +5 = 0
17 17 17 17
35 69 34 7 92 85
– + = 0 – + = 0
17 17 17 17 17 17
34 34 85 85
– + = 0 – + = 0
17 17 17 17
0≡0 0≡0

190 Relaciones y funciones

190-191. 190 08/11/2001, 15:29


CAPÍTULO 3

7. Resolver por el método de eliminación por igualación el sistema:


x + 2 y – 1 = 0 (E1)
5x – 3 y + 4 = 0 (E2)

Este método consiste en despejar la misma incógnita de las dos


ecuaciones e igualar los valores así obtenidos, consiguiendo con
ello una ecuación con una sola incógnita.
Despejemos x en E1 y E2 obteniendo las ecuaciones E3 y E4.

E1 Q x=–2y+1 (E3)
3y–4
E2 Q x = (E4)
5
3y–4
Igualando E3 y E4 –2y+1 = /• 5
5
– 10 y + 5 = 3 y – 4
– 13 y = – 9
9
y =
13
9
Reemplazando en E3 (o E4) x = –2 +1
13
18 13
x= – +
13 13
5
x = –
13
5 9
Luego la solución es – ,
13 13
5 9
Comprobemos que el par – , satisface ambas ecuaciones:
13 13

E1 : x+2 y–1 = 0 E2: 5x – 3y + 4 = 0


5 9 –5 9
– +2 –1 = 0 5 –3 +4 = 0
13 13 13 13
5 18 13 25 27 52
– + – = 0 – + = 0
13 13 13 13 13 13
13 13 52 52
– = 0 – + = 0
13 13 13 13
0≡0 0≡0

8. Resolver el siguiente sistema:


4x : (y – 2) = 3 : 4 (E1)
(x + 2) : (y + 3) = 4 : 8 (E2)
Transformando cada una de las ecuaciones formamos un sistema
equivalente:
4x 3 x+2 4
E1 : = E2: =
y–2 4 y+3 8
16x = 3y – 6 8x + 16 = 4y + 12
16x – 3y + 6 = 0 8x – 4y + 4 = 0 /: 4
2x - y + 1 = 0 (*)

Relaciones y funciones 191

190-191. 191 08/11/2001, 15:31


Ejercicios 16x – 3y + 6 = 0
resueltos 2x – y + 1= 0 /• (– 3)

16x – 3y + 6 = 0
– 6x + 3y – 3 = 0 sumando

10 x + 3 = 0
–3
x=
10
Reemplazando x en (*)

–6
–y+1=0
10
6
–y= –1
10
4 2
y= =
10 5
–3 2
Luego, la solución es ,
10 5
Verifiquémoslo:

E1: 4x : (y – 2) = 3 : 4 E2: (x + 2) : (y + 3) = 4 : 8
–3 2 –3 2
4 : –2 = 3 :4 +2 : +3 = 4 :8
10 5 10 5
– 12 –8 17 17
10
: 5
= 3 :4 : = 4 :8
10 5

= 3 :4 =4 :8
3 3 1 4
∫ =
4 4 2 8
9. Resolver el siguiente sistema: 1 1

2 2
ax – (1 + b) y + ab = 0 (E1)
(1 – a) x + by – ab = 0 (E2)

Despejemos y en ambas ecuaciones:

E1 : (1 + b) y = ax + ab
a x + ab
y=
1+b (E3)
E2 : by = ab – (1 – a) x
ab – 1 – a x
y= (E4)
b
Igualando E3 con E4:
ax + ab a b – 1 – a x
=
1+b b

192 Relaciones y funciones

192-193. 192 08/11/2001, 15:42


CAPÍTULO 3

Multiplicando cruzado
abx + ab2= ab – x + ax + ab2 – bx + abx
x (1 – a + b) = ab
ab
x=
1– a + b
En (E3)
ab
a + ab
1–a+b
y=
1+b
2 2 2
a b + ab – a b + ab ab 1 + b
y= 2
=
1 – a + b + b – ab + b 1+b–a 1+b +b 1+b
ab
y=
1–a+b
ab ab
Así la solución es: ,
1–a+b 1–a+b
Verifiquémoslo:
E1: a x – (1 + b) y + ab = 0
ab ab
a – 1+b + ab = 0
1–a+b 1–a+b
2 2 2 2
a b ab + ab ab – a b + ab
– + =0
1–a+b 1–a+b 1–a+b
0 =0
E2 : (1 – a) x + by – ab = 0
ab ab
1–a +b – ab = 0
1–a+b 1–a+b
2 2 2 2
ab – a b ab ab – a b + ab
+ – =0
1–a+b 1–a+b 1–a+b
0 ≡0
10. Resolver el siguiente sistema:
x + 3y – 3z = –16 (E1)
– 3x + 2y – 2z = 4 (E2)
12x – 25y – 2z = – 2 (E3)

Con E1 y E2 vamos a eliminar x


3E1: 3x + 9y – 9z = – 48
E2: – 3x + 2y – 2z = 4 sumando
11y – 11z = – 44 ⁄ : 11
y – z =– 4 (E4)
Con E1 y E3 vamos a eliminar x
– 12 E1:: –12x – 36y + 36z = 192
E3: 12x – 25y – 2z = – 2 sumando
– 61y + 34z = 190 (E3)

Relaciones y funciones 193

192-193. 193 08/11/2001, 15:43


Combinamos E4 con E5 para calcular el valor de y A z.
Ejercicios
y–z= –4 (E4)
resueltos
– 61y + 34z = 190 (E5)

34E4 : 34y – 34z = –136


– 61y + 34z = 190

– 27y = 54
y=–2

En E4 si y=–2 ,–2–z=–4
z= 2
En E1 si y = – 2, z = 2 , x + 3 (– 2) – 3 • 2 = – 16
x = – 16 + 6 + 6
x =–4
Luego la solución es (– 4, – 2, 2)
Verifiquemos:

E1: x + 3y – 3z = – 16
– 4 + 3 (– 2) – 3 • 2 = – 16
– 16 ≡ – 16
E2: – 3x + 2y – 2z = 4
– 3 (– 4) + 2 (– 2) – 2 • 2 =4
12 – 4 – 4 = 4
4≡4
E3: 12x – 25y – 2z = – 2
12 (– 4) – 25 • (– 2) – 2 • 2 = – 2
– 48 + 50 – 4 = – 2
2 – 4= – 2
–2≡–2
11. Resolver el mismo sistema del problema 10 pero usando el
método de eliminación de Gauss.
x + 3y – 3z = – 16 E1
– 3x + 2y – 2z = 4 E2
12x – 25y – 2z = – 2 E3

Llamaremos operaciones por fila a los siguientes cambios que


podemos hacer con las ecuaciones sin alterar la solución
del sistema.
• Intercambiar dos ecuaciones.
• Multiplicar una ecuación por un factor distinto de cero.
• Cambiar una ecuación por ella misma sumada con otra
que previamente ha sido multiplicada por algún factor
distinto de cero.

194 Relaciones y funciones

194-195.(2003) 194 20/11/02, 11:07 AM


CAPÍTULO 3

El método se trata de ir ordenando el sistema de modo que los


coeficientes bajo la primera incógnita de la primera ecuación sean
cero, luego los coeficientes bajo la primera incógnita de la segunda
ecuación sean cero y así sucesivamente.
Este método es válido para resolver sistemas de n ecuaciones
de primer grado con n incógnitas y es de fácil implementación
computacional.

Los explicaremos aplicándolo.

E1 la dejamos igual, ya que el coeficiente de x es 1.


E2 Q E2 + 3 E1 x + 3 y – 3 z = – 16
E3 Q E3 + (– 12) E1 11 y – 11 z = – 44
– 61 y + 34 z = 190

E1 Q E1 x + 3 y – 3 z = – 16
1
E2 Q 11 E2 y–z = – 4
E3 Q E3 – 61 y + 34 z = 190

E1 Q E1 x + 3 y – 3 z = – 16
E2 Q E2 y – z = – 14
E3 Q E3 + (61) E2 – 27 z = – 54

En E3: z = 2
En E2: y – 2 = – 4 fi y = – 2
En E1: x + 3 (– 2) – 3 • 2 = – 16 ⇒ x = – 4

Luego, la solución es (– 4, – 2, 2).

12. Resolver el sistema


x+2y–z = 3 (E1)
x – y + 2z = 0 (E2)
2 x – 3 y + 5z = 7 (E3)

Aplicando el método de eliminación de Gauss.

E1 Q E 1 x+2y –z = 3
E2 Q E2 + (– 1) E1 – 3 y + 3 z =– 3
E3 Q E3 + (– 2) E1 – 7 y + 7z = 1

Relaciones y funciones 195

194-195.(2003) 195 20/11/02, 11:07 AM


E1 Q E1
Ejercicios
x + 2y – z = 3
resueltos E2 Q –
1
E2
3 y–z = – 1
1 1
E3 Q – E3 y–z = –
7 7

E1 Q E1 x + 2y – z = – 3
E2 Q E2 y–z = – 1
8
E3 Q E3 + (–1) E2 0 = –
7

Vemos que en la última ecuación se nos produce una


contradicción, lo que quiere decir que el sistema no tiene
solución. No hay ningún punto común a los tres planos.

13. Resolver el sistema:

x + 2y – z = 3 (E1)
x – y + 2z = 0 (E2)
2x – 3y + 5z = –1 (E3)

Aplicando el método de eliminación de Gauss.


E1 Q E1 x + 2y – z = 3
E2 Q E2 + (– 1) E1 – 3y + 3z = – 3
E3 Q E3 + (– 2) E1 – 7y + 7z = – 7

E1 Q E1
x + 2y – z = 3
E2 Q – 1 E2
3 y–z= 1
1
E3 Q – E3 y–z= 1
7

E1 Q E1 x + 2y – z = 3
E2 Q E2 y–z=1
E3 Q E3 + (–1) E2 0=0

196 Relaciones y funciones

196-197. 196 08/11/2001, 17:05


CAPÍTULO 3

Ya veíamos en el paso anterior que había dos ecuaciones equivalentes,


lo que significa que el sistema tiene una variable libre y por lo tanto
tiene infinitas soluciones.
Si z = 1 (variable libre), en E2 : y = 2 en E1 : x = 0. Luego (0, 2,
1) es una solución. Dando otros valores a z se pueden encontrar
otras soluciones.
Si z = a, y = 1 + a x = 1 – a
Así la solución general es (1 – a, 1 + a, a)

14. Resolver el sistema:

2x – y + z – 3u = –7 (E1)
x + 2y – z + u = 8 (E2)
3x + y – 2z – u = 6 (E3)
– x + y – 4z – 2u = –3 (E4)

Reordenando
x + 2y – z + u = 8
–x+y–4z–2u = –3
2x–y+z–3u = –7
3x+y–2z–u = 6
E2 Q E2 + E1
E3 Q E3 + (– 2) E1
E4 Q E4 + (– 3) E1 x + 2y – z + u = 8
3y – 5z – u = 5
– 5y + 3z – 5 u = – 23
– 5y + z – 4 u = – 18

1
E2 Q E2 x + 2y – z + u = 8
3
5 1 5
y– 3
z– 3
u =
3
– 5y + 3z – 5u = – 23
– 5y + z – 4u = – 18

E3 Q E3 + 5 E2

E4 Q E4 + 5 E2 x + 2y – z + u = 8
5 1 5
y– z– u = 3
3 3
16 20 44
– z– u =
3 3 3
22 17 29
– z– u =–
3 3 3

Relaciones y funciones 197

196-197. 197 08/11/2001, 17:06


Ejercicios E3 Q – 3 E3 x + 2y – z + u = 8
resueltos 2
1
E4 Q – 3 E4 y–5 z–3 u = 5
3 3
8z + 10u = 22

22z + 17u = 29

E3 Q 1 E3 x + 2y – z + u = 8
8
5 1 5
y–3 z–3 u = 3
z+5 u = 11
4 4
22z + 17u = 29

E4 Q E4 + (– 22) E3 x + 2y – z + u = 8
y–5 z–1 u = 5
3 3 3
z+5 u = 11
4 4
– 21 u = –63
2 2

E4 : u = 3

E3 : z = 11 – 5 u fi z = – 1
4 4
E2 : y = 5 + 5 z + 1 u fi y = 1
3 3 3
E1 : x = 8 – 2y + z – u fi x = 2
Luego la solución es (2, 1; – 1, 3)
15. Determinar el valor de k para que el sistema tenga:
i) solución única
ii) ninguna solución
iii) infinitas soluciones
x + 2y – z = – 6
4x – y + z = 9
x – ky + 2z = 5

E2 Q E2 + (– 4) E1
E3 Q E3 + (– 1) E1 x + 2y – z = – 6
– 9y + 5z = 33
– (k + 2) y+ 3z = 11

E2 Q –1 E2 x + 2y – z = – 6
9
y – 5 z = – 33
9 9
– (k + 2) y + 3z = 11

198 Relaciones y funciones

198-199. 198 08/11/2001, 15:51


CAPÍTULO 3

E3 Q E3 + (k +2) E2 x+2y–z = –6
5 33
y– z = –
9 9
(17 – 5 k) z = 33 –33k

396
Si k = 17 E3: 0 = – y el sistema no tiene solución.
5 5
Si k 17 el sistema tiene solución única.
5
No existe k para el cual el sistema tenga infinitas soluciones, ya que
17–5k y 33–33k no pueden ser ambas cero.

Ejercicios
1. Determine si los siguientes pares de j) – 3y – 3 = 0
ecuaciones son L. I. o L. D. 3y + 3 = 0

a) 2x + y + 3 = 0
2. Determine el valor de k para que las
5x + y – 9 = 0 ecuaciones dadas sean L. I.

b) 4x + y – 7 = 0 a) 2x + 3y – k = 0 x+y–1 = 0
6x + 5y + 7 = 0 b) x + y – 3=0 kx = 3 – y
c) 1 – 2x = y x + 2y = k
c) – x + y – 12 = 0
5x + 3y + 20 = 0 d) x + ky – 2 = 0 x–y = 2

d) – 2x + y + 1 = 0 3. Determine el valor de k en el sistema


2x – y + 3 = 0 para que tenga solución única.

e) 2x + 3y – 1 = 0 a) x + ky = 2

4 x + 6y – 2 = 0 2x + y = 4

f) 2x + 3y – 1 = 0 b) 8x + 4y – 6 = 0
2x – y = 0 12x + ky = 9

g) 2x + 3y – 1 = 0
c) kx – y = 2
4x + 6y + 7 = 0
x – ky = 3
h) 4x + 4y – 8 = 0
4. En los casos del ejercicio Nº 1 en que
6x + y – 22 = 0
las ecuaciones sean L. I., determine
si forman un sistema de ecuaciones
i) – 2y – 3 = 0
simultáneas o incompatibles (determi-
–y–1 = 0 nado o inconsistente, respectivamente).

Relaciones y funciones 199

198-199. 199 08/11/2001, 15:52


Ejercicios b) –x+y–4 =0
5. Determine, sin resolver, si los siguientes – 4x + y – 25 = 0
sistemas son determinados, indetermi- (1, 5) (– 7, – 3) (1, 29)
nados o inconsistentes.
c) 2x + 7y – 1 = 0
a) – 2x + 2y + 5 = 0
– 2x + 7y – 13 = 0
–x+y+7 = 0
(– 24, – 5), (– 3, 1) (– 38, – 9)
b) 5x + y – 17 = 0
4x – 8 = 0 d) 3x – 5y – 25 = 0
– 6x + 10y + 50 = 0
c) – 3x + 3y – 1 = 0
–x+y+2 = 0 (– 30, – 23), (20, 7) (15, 9)

e) 4x + y + 23 = 0
d) 3x + y + 3 = 0 5x + 7y + 23 = 0
9x + 3y + 9 = 0
(– 13, 6), (– 6, 1), (8, – 9)
e) 5x – 3y = 0
f) – x + 6y + 32 = 0
25x – 15y = 0
– 3x + 6y + 36 = 0

f) – 2x + y – 3 = 0 (2, – 5), (50, 19), (26, 7)


– 5x + 4y – 15 = 0
7. Determine gráficamente la solución de
g) 4x + y – 10 = 0 cada uno de los siguientes sistemas:
5x + y – 11 = 0 a) 5x = 4y – 36
9x = y – 40
h) 7x – 5y – 4 = 0
– 14x + 10y + 8 = 0 b) 8x + 11y – 4 = 0
3x – 2y + 23 = 0
i) x–2 y = 0
– 2x + 4y + 1 = 0 c) x + 12y – 31 = 0
4x = – 3y – 11
j) y+2 = 0
– 5x + 4y – 7 = 0 d) –x=–y–2
4x – y – 17 = 0

6. Determine si los pares de números reales


e) 2x = – 5y + 8
dados son solución del sistema dado.
4x = – 8y + 8
a) 3x + y – 4 = 0
6x + 2 y – 8 = 0
f) 4x + 4y + 12 = 0
(5, – 11) (1, 1) (2, – 2) x + 2y – 1 = 0

200 Relaciones y funciones

200-201. 200 08/11/2001, 17:07


CAPÍTULO 3

8. Resuelva usando el método de eliminación


g) – 9x – 8y + 5 = 0 por reducción los siguientes sistemas
– 6x – 7y – 5 = 0 de ecuaciones:
a) x + 5y + 21 = 0
h) 11y + 11 = 0
– 2x + 3y – 3 = 0
– 4x + 6y – 10 = 0

b) 5x + 3y + 17 = 0
i) – 2x – 8y – 16 = 0
4x + y + 15 = 0
– x = 7y + 17

j) – 5x = 7y + 35 c) 3x + 5y + 9 = 0
– 8x – 2y – 10 = 0 4x + 2y – 2 = 0

k) 3x + 8y – 6 = 0 d) – x + 3y – 13 = 0
2x = – 6y + 6
– 4x + 2y – 22 = 0

l) 5x = – y – 21
e) – x + 2y – 1 = 0
7x – 2y + 43 = 0
2x + 5y – 25 = 0
m) x = – 14y + 21
4x = – 12y – 4 f) – 3x + 6y – 39 = 0
6x + 6y – 12 = 0
n) – 4x = – 3y – 8
– 3x = 7 y + 31
g) 3x + 8y – 6 = 0
– 2x + 2y – 18 = 0
o) – 2y + 6 = 0
7x + 2y – 41 = 0
h) 3x + 3y + 30 = 0
p) – 4x = – 6y + 2 5x + y + 34 = 0
– 7x = – 11y + 2
i) 2x + 5y + 19 = 0
q) 7x + 12y + 32 = 0
5x + 2y – 5 = 0
x + 13y – 18 = 0

j) x + 5y + 6 = 0
r) x = – 8y + 12
– 5x + 4y – 1 = 0
7x + 5y + 18 = 0

k) 4x + 4y – 24 = 0
s) 0=y+4
– 6x – 11y – 26 = 0 5x + 3y – 24 = 0

t) x + 6y – 9 = 0 l) 4x + 7y – 26 = 0
– x = – 7y + 17 2x + 5y – 16 = 0

Relaciones y funciones 201

200-201. 201 08/11/2001, 17:07


Ejercicios
9. Resuelva con el método de elimina- 10. Resuelva por el método de elimina-
ción por sustitución cada uno de los ción por igualación cada uno de los
siguientes sistemas de ecuaciones: siguientes sistemas de ecuaciones:
a) 4x + 3y – 3 = 0 a) –x + 2y + 1 = 0
2x + 7y – 7 = 0 x + 2y + 15 = 0

b) x + 4y + 4 = 0 b) x + 5y – 6 = 0
– 4x + 6y + 28 = 0 5x + 6y – 11 = 0

c) 4x + y – 30 = 0 c) x + 8y – 39 = 0
2x – y – 12 = 0 5x + y = 0

d) x + y – 10 = 0 d) –2x + 2y – 6 = 0
–x + 2y – 8 = 0 6x + 3y – 18 = 0

e) 2x + 5y + 37 = 0 e) x+y–3=0
x + 2y + 16 = 0 – 4x – y – 6 = 0

f) –x + 2y + 3 = 0 f) –2x – y – 3 = 0
4x + 5y + 14 = 0 – 6x + 5y + 15 = 0

g) –2x + 4y + 12 = 0 g) x + 2y = 0
–3x + 8y + 20 = 0 – 4x + 7y = 0

h) x + 2y + 1 = 0 h) x + 5y – 3 = 0
– 6x + 2y – 48 = 0 2x – y + 1 = 0

i) –3x + 5y – 22 = 0 i) x + 2y + 3 = 0
–5x + 8y – 36 = 0 –x + y – 3 = 0

j) x + 2y + 1 = 0 j) 4x + 7y + 15 = 0
4x + y + 4 = 0 3x + 8y + 25 = 0

k) –x + 3y – 7 = 0 k) –x + 3y – 15 = 0
–3x + y – 5 = 0 –2x + y – 15 = 0

l) –x + 6y + 2 = 0 l) 6x + 4y – 28 = 0
–2x + 3y – 5 = 0 –x + 2y – 6 = 0

202 Relaciones y funciones

202-203. 202 08/11/2001, 17:07


CAPÍTULO 3

11. Resuelva los siguientes sistemas de m) –3x + 4y – 38 = 0


ecuaciones usando el método que x+ y+1=0
estime más conveniente:

a) 3x – y – 12 = 0
12. Resuelva los siguientes sistemas de
x + 4y – 5 = 0 ecuaciones:
a) 4x + 5y + 9 = 0
b) –5x + 7y + 8 = 0 20x + 6y – 12 = 0
6x + 2y + 32 = 0
b) 20y – 5 = 0
c) x + 2y – 1 = 0 – 4x + 12y – 15 = 0
3x + 5y = 0
c) 32y + 8 = 0
d) –x + 4y + 2 = 0 10x + 4y – 9 = 0
–2x + 6y + 4 = 0
d) –20x + 6y – 8 = 0
–8x + 21y + 3 = 0
e) x + 2y + 12 = 0
–x + 3y + 13 = 0 e) –12x + 20y – 7 = 0
4x + 8y + 6 = 0
f) y–6=0
–x + 6y – 36 = 0 f) 12x + 10y + 2 = 0
3x + 4y + 2 = 0

g) x + 5y – 3 = 0
g) 4x + 5y + 11 = 0
–3x + 4y + 9 = 0
– 4x + 3y + 13 = 0

h) 7y + 7 = 0 h) –x + 4y + 1 = 0
6x + 5y – 25 = 0 x + 8y – 10 = 0

i) x + 3y – 16 = 0 i) –3x + 9y – 2 = 0
– 6x + 5y – 42 = 0 –15x + 21y + 6 = 0

j) 2y + 2 = 0 j) 8x + 2y – 16 = 0
– 4x + 2y + 2 = 0 10x + 3y – 22 = 0

k) y =0 k) –3x + 5y + 14 = 0
–5x + y – 2 = 0 – 4x + 2y + 14 = 0

l) 4x + 6y – 4 = 0 l) – 4x + 3y – 6 = 0
–5x + 4y + 28 = 0 –16x + 21y – 33 = 0

Relaciones y funciones 203

202-203. 203 08/11/2001, 17:07


Ejercicios
7x + 8y –7x – 7y
13. Resuelva los siguientes sistemas: h) =
6 6
3y – 1 = 0
a) px + y = 1
x – qy = 0 i) 5x – 4 (7y + 5) = – 2
– 7x – 4 (– y – 2) = 3
b) px – y = 0
pqx – 1 = 0 j) 6 (– 3x – 7) – 5 (4y + 8) = 9
(– 6x + 2y) : (– 9x + 2y) = – 5 : 2
c) ax – b = 0
ax – by = 1
k) – 6 (5 – x) – 8 (4y + 2) = 2
(8x + 6) + 2 (5y – 4) = 3
d) ax + by = 1
bx + ay = 0
6x – 5y 6x + 4y
l) – =
2 3
6x + 8 (3 – 8y) = 0
14. Resuelva los siguientes sistemas de
ecuaciones:
8y – x – 6y 15. Un cuarto de la suma de dos números
a) 6x + –4 = +2 es 152 y un tercio de su diferencia es
6 6
x – 5 ( – 3y – 4) = – 6 66. ¿Cuáles son los números?

16. Si el doble de un número se suma


7y x + 8y con el triple de otro se obtiene 45.
b) – 6x + –4= +8 Si al triple del primero se le resta el
4 4
– (– 4x – 2) – 7 (– 5y – 7) = – 4 segundo se obtiene 29. Determine
los números.
c) 7x = 6 17. La suma de las cifras de un número
– 4 (– 5x + 1) – 2 (y – 8) = – 2 de dos cifras es 7. Si se invierten las
cifras, el nuevo número es igual a dos
veces el número anterior, más dos
d) 8x : (– 7) = – 3y : (– 1) unidades. Calcule el número.
4 (– 8x – 4) – 8 (2 – y) = – 7
18. Si un número de dos cifras se divide
por la suma de sus cifras, el cociente
– x + 7y 8 es 7 y el resto es cero. Si a la cifra
e) 5x – 4y
= –
7
de las decenas se resta la cifra de
8 (– 8x – 7) + 3 (6 – 9y) = – 5 las unidades, se obtiene 4. ¿Cuál es
el número?
f) 5x + 7 (6y – 8) = 9 19. Dos números están en la razón 5 : 5.
(– x – 4y) : (7x – 3y) =3 :6 Si el primero se aumenta en 12 y el
segundo se disminuye en 3, quedan
en la razón de 9 : 4. ¿Cuáles son
4y 5x + 2y los números?
g) – 6x – +3 = +3
8 8
8x = 6y – 3 20. La suma de dos números es 208 y

204 Relaciones y funciones

204-205. 204 08/11/2001, 17:08


CAPÍTULO 3

su diferencia es 122. ¿Cuáles son los la edad de Marcos. ¿Cuál es la edad


números? actual de cada uno?

21. Sergio tiene $ 1.950 en monedas de 29. El doble de la edad de Ángela sobre-
$ 100 y de $ 50. En total tiene 24 pasa en 14 años la edad de Sergio. Y
monedas. Determine cuántas son de un quinto de la edad de Sergio es 13
$ 100 y cuántas de $ 50. años menos que la edad de Ángela.
Calcule ambas edades.
22. La suma de las cifras de un número de
2 cifras es 4. Si se invirtieran las cifras, 30. Dos números están en la razón 10 : 5. Si
el nuevo número sería igual al doble se resta 20 al primero y se suma 20 al
del número anterior, más 5 unidades. segundo, la razón de ellos se invierte.
¿Cuál es el número? ¿Cuáles son los números?

23. Si un número de dos cifras se divide 31. La suma de dos números es 169 y
por la suma de sus dígitos, el cociente su diferencia es 132. ¿Cuáles son los
es cinco y el resto es cero. Si a la números?
cifra de las decenas se le resta la cifra
de las unidades se obtiene –1. ¿Cuál 4
32. de la suma de dos números es
es el número? 5 10
igual a 32 y de su diferencia es 10.
9
1 Encuentre los números.
24. La edad de Eliana es de la edad
5
de Miguel y hace 5 años, la edad de 33. Andrés le pagó a Carlos $ 1.550 en
1 monedas de $ 100 y de $ 50. En total
Eliana era de la edad de Miguel.
10 le dio 21 monedas. ¿Cuántas eran de
Determine sus edades actuales. $ 100 y cuántas de $ 50?

25. Un cuarto de la suma de dos números 34. Si un número de dos cifras se divide
es 81 y un tercio de su diferencia es por la suma de sus cifras el cociente
54. ¿Cuáles son los números? es cinco y el resto es trece. Si a la
cifra de las decenas se resta la cifra
26. Si el doble de un entero se suma con el de las unidades se obtiene 1. ¿Cuál
triple de otro se obtiene 26. Si al triple del es el número?
primero se le resta el segundo se obtiene
28. Determine los números. 35. La edad de Adolfo es 15 años menos
que el doble de la edad de Teresa. Y la
27. Si una de dos llaves de agua queda séptima parte de la edad de Adolfo es
abierta durante 40 min. y la otra 20 años menos que la edad de Teresa.
durante 17 min., entregan ambas 1.163 Calcule ambas edades.
litros de agua. Si la primera queda
abierta durante 21 min. y la otra 36. Dos números están en la razón de
durante 41 min., entregan 1.220 litros 6:4. Si se resta 6 del primero y se
de agua. ¿Cuántos litros de agua por suma 6 al segundo, quedan en la
minuto entrega cada llave? razón 2 : 3.
¿Cuáles son los números?
28. Hace 5 años la edad de Manuel era 9
veces la edad de Marcos. En 5 años 37. La suma de la cifra de las decenas y la
más la edad de Manuel será 4 veces cifra de las unidades de un número de

Relaciones y funciones 205

204-205. 205 08/11/2001, 17:08


Ejercicios
i) 8z + 32 = 0
dos cifras es 4. Si al número se le resta 18, las – x – 52y + 30z + 17 = 0
cifras se invierten. ¿Cuál es el número? 2x + 13y + 7z + 52 = 0

38. Hace 4 años la edad de Ximena era


j) – 21x – 14y – 13z – 73 = 0
8 veces la edad de Matías. En cuatro
– 14x – 21y – 6z – 115 = 0
años más la edad de Ximena será 4
4x – 4y – 12 = 0
veces la de Matías. ¿Cuál es la edad
de cada uno?
k) – 10x + 18y – 40z – 84 = 0
39. Resuelva los siguientes sistemas: – 3x + 30y = – 22z – 92
5x + 6y = – 17z – 66
a) – 5x – 3y – 5 = 0
– 8x + 8y = – 4z
– 2x – y + 2z + 2 = 0 l) – 4x + 20y = – 20z – 12
3x + 9z – 27 = 0
– 7x + 26y – 2z + 77 = 0
b) 9x + 14y + 25z + 22 = 0
– 6x + 2y = 5z – 19
m) 3y – z + 5 = 0
– 7x + 4y – 3z + 26 = 0
16x – 20y + 12z + 52 = 0
13x – 46y – 14z – 142 = 0
c) 10x – 4y + 12z – 28 = 0
– 22x + 31y = – z – 29
n) x – 2y – z = 0
4x + 3y – 6z + 15 = 0
19x – 29y – 12z + 35 = 0
– 18x + 24y + 15z – 15 = 0
d) 2x + 2y + 11z + 23 = 0
y+z =0
ñ) 7x + 5y + 10z + 21 = 0
– 3x + 4y + 7z + 15 = 0
– 4x + 6y + 20z + 82 = 0
– 4x – 5y – 24z – 83 = 0
e) 5x + 25y + 26z + 49 = 0
– 18x + 20y – 51z + 1 = 0
o) 2x + z – 3u = 8
15x – 9y + 7z + 50 = 0 x – 2y + z + u = – 4
– x + y – 3u = 4
f) 3x + 9y = 9z – 48 y – 4z + u = – 2
– 6x + 4y – 4z – 8 = 0
12x – 25y – 2z + 2 = 0 p) x – 2y + 5z –u =–6
x+y–z+u =6
g) x–y–4 =0 x + 2y – z – u =2
– 3x + 3y = – 4z – 4 2x + y + u =7
– 14x – y + 6z – 31 = 0
q) 5x – 3y + z – 2u = 11
h) – 15x – 8y – 11z – 9 = 0 x – 5y + 3z – u = 10
– 5x – 3y + 27z + 29 = 0 2x + y – z + 3u = – 3
x + y + 5z + 3 = 0 3x – 2y + 2z + u = 6

206 Relaciones y funciones

206-207. 206 08/11/2001, 17:09


CAPÍTULO 3

3 1 2
r) x + 2y – z + u = – 3 g) x + y – z
=–2
2x – y + 2z – u = 8 2 3
– x + z =3
– x + 3y – 2z + u = – 7
3x – 4y + 3z – 2u = 13 1 1 2
– x – y – z =1

40. Resuelva los siguientes sistemas:


2 4
h) – x – y = –4
a) – 1 1 2
x + y + z
=2
3 4 4
x – y –z
= –1
3 3 3
– x – y – z =–3
3 32
2 1 3 – x + y –z = –4
x – y – z =3

1 2 4
1 2 =–1 i) – x – y –z = 2
b)
x + z
4 3
1 3 3 – x +z = –2
x + y +z =0
4 2 3
3 3 – x –y+z = –1
y + z =2

2 1
j) y – z = –1
c) 3 – 1 + 3 = – 4
x y z
1 3
2 2 – x – z = 0
– x +z =–1
3 2 2
2 1 3
=–2 – x – y –z = –4
x + y –z

3 3
d) 1 1 2
=2 k) x – y = 0
x +y–z
2 3 3 1 2 = –1
– x +y =–1 x – y +z
4 4 1 3 3
– x –z =–1 x – y –z
= 2

3 2 2
e) 2 1
x – y – z
=–1 l) y + z
= 2
3 2 4 1 1 2
=2
x – y – z x + y – z = –4
1 2 4 2 2 1
– x – y – z =3 x + y – z = 3

1 4 3 3 1 2 = –4
f) – x – y – z = 1 m)
x – y – z
2 4 4 1 3 1
x – y – z =0 – x – y + z = –2
1 3 2 2 1 1
– x + y – z =–1 – x + y – z = –3

Relaciones y funciones 207

206-207. 207 08/11/2001, 17:11


Ejercicios 44. Resuelva el sistema.
3 1 3
n) x – y – z =–3 a) x – y + 5z = 3
2 3 1 – 3x + 3y – 15z = 8
– x – y + z = 2
2x – y + z = 2
4 1
– x – z =–1
1
b) 2x – y – z =2
2
3 4 x + y – 2z = 3
o) y – z =–1 1 1
x– y– z =5
3 1 2 4
– y + z = 0
2 1 4
x – y – z =–4 45. La suma de las tres cifras de un número
es 16. Al sumarle 9 se intercambia la cifra
41. Encuentre los valores de k en el sistema de las unidades con la de las decenas
siguiente para que éste tenga: y al sumarle 99 se intercambia la cifra
de las unidades con la de las centenas.
iii) Solución única. Encuentre este número.
iii) Infinitas soluciones.
iii) Ninguna solución. 46. Andrés, Arturo y Carlos juntan $ 50.000.
Si Andrés tiene la mitad del dinero de
a) x + 2y – z = 1 Arturo y tres veces el de Carlos, determine
x+y+z =0 el dinero que posee cada uno.
x + ky – 2z = 2
47. Encuentre la medida de los ángulos
interiores de un triángulo si el doble
b) x – 2y + z = 3 del primero menos el segundo es 81° y
el triple del segundo, menos el tercero
kx – y + 2z = 3
es 9º.
2x – y – 2z = 1
48. Entre Matías y Javier reúnen $ 9.000
menos que lo que tiene Daniel. Si Javier
c) 2x – 3y + kz = 5
le diera $ 2.000 a Matías, tendrían lo
x – 2y + 4z = 2 mismo y si Daniel tuviera $ 6.000 más,
– x + ky – 9z = 1 tendría el triple de lo que tiene Matías.
Encuentre el capital de cada uno.
42. Resuelva el sistema.
49. Determine la medida de los ángulos
x–y =2 interiores de un triángulo si la suma del
primero con el segundo, menos el tercero
2x – z = 1
es 72º y el doble del primero menos
y+z =3 la suma del segundo con el tercero es
168º.
43. Resuelva el sistema. 50. Encuentre tres números sabiendo que su
x – 2y + 1= 0 suma es 8. Si a la suma del doble del
x + 2y – 1= 2 primero más el segundo se le resta el
tercero da 3 y la suma de los dos primeros
2x – 4y + 2= 0
menos el triple del tercero es 4.

208 Relaciones y funciones

208-209. 208 08/11/2001, 17:13


CAPÍTULO 3

51. La suma de las edades de tres amigas el triple del primero es – 36 y la suma
es 80 años. Si la edad de Regina es el de los tres es 26.
doble de la edad de Gloria más tres 54. Encuentre la medida de tres ángulos
años y la edad de Susana es el doble sabiendo que están en la razón 3 : 6 : 4 y
de la edad de Gloria menos tres años, los dos primeros son suplementarios.
encuentre las edades de las tres.
55. Tres compradores pagaron las sigu-
52. Determine tres números tales que al ientes facturas por compra de ropa
sumar el primero con el doble del en una tienda: el primero canceló $
segundo se obtiene 4. El doble del 60.700 por la compra de 5 camisas, 3
tercero incrementado en 38, más el chalecos y dos pares de calcetines. El
triple del primero es 4 y la suma de segundo canceló $ 17.700 y compró
ellos es 22. una camisa, un chaleco y un par de cal-
cetines. El tercero adquirió 2 camisas, 1
53. Determine tres números enteros tales chaleco y dos pares de calcetines y pagó
que al sumar el primero con el tercero $ 22.700.
se obtiene 1. El doble del tercero más Determine el valor de cada prenda.

Soluciones

1. a) L. I. b) L. I. c) L. I d) L. I. e) L. D. f) L. I. g) L. I. h) L. I.
i) L. I. j) L. D.
2. a) I k E R b) k 1 c) I k E R d) k – 1
1
3. a) k b) k 6 c) k – 1, 1
2
4. Determinado: a, b, c, f, h; inconsistente: d, g, i.

5. a) Inconsistente b) Determinado c) Inconsistente d) Indeterminado


e) Indeterminado f) Determinado g) Determinado h) Indeterminado
i) Inconsistente j) Determinado
6. a) (5, – 11) Sí b) (1, 5) No c) (– 24, – 5) No d) (– 30, – 23) Sí
(1, 1) Sí (– 7 – 3) Sí (– 3, 1) Sí (20, 7) Sí
(2, –2) Sí (1, 29) No (– 38, – 9) No (15, 9) No
e) (– 13, 6) No f) (2, – 5) Sí
(–6, 1) Sí (50, 19) No
(8, – 9) No (26, 7) No
7. a) (– 4, 4) b) (– 5, 4) c) (– 5, 3) d) (5, 3) e) (– 6, 4) f) (– 7 , 4)
g) (5, – 5) h) (– 4, – 1) i) (4, – 3) j) (0, – 5) k) (– 6, 3) l) (– 5, 4)
m) (– 7, 2) n) (– 1, – 4) o) (5, 3) p) (– 5, – 3) q) (– 8, 2) r) (– 4, 2)
s) (3, – 4) t) (– 3, 2)
8. a) (– 6, – 3) b) (– 4, 1) c) (2, – 3) d) (– 4, 3) e) (5, 3) f) (– 3, 5)
g) (– 6, 3) h) (– 6, – 4) i) (3, – 5) j) (– 1, – 1) k) (3, 3) l) (3, 2)
9. a) (0, 1) b) (4, – 2) c) (7, 2) d) (4, 6) e) (– 6, – 5) f) (– 1, – 2)
g) (4, – 1) h) (– 7, 3) i) (– 4, 2) j) (– 1, 0) k) (– 1, 2) l) (– 4, – 1)
10. a) (– 7, – 4) b) (1, 1) c) (– 1, 5) d) (1, 4) e) (– 3, 6) f) (0, – 3)

Relaciones y funciones 209

208-209. 209 08/11/2001, 17:13


Soluciones
g) (0, 0) h) – ( 2 7
,
11 11 ) i) (–3, 0) j) (5, –5) k) (– 6, 3) l) (2, 4)

11. a) ( 5313 , 133 ) b) (– 4, – 4) c) (–5, 3) d) (2, 0) e) (–2, –5) f) (0, 6)

g) (3, 0) h) (5, –1) i) (–2, 6) j) (0, –1) k) – , 0 ( 2


5 ) l) (4, –2)
m) (– 6, 5)

12. a) (64, –3) ( 14 )


b) –3, ( 14 )
c) 1, – d) – , –( 1
2
1
3 ) e) – 1, – 1
4
( ) (3 )
f) 2, – 1
3
h) (4, ) i) ( , ) l) ( – , 1)
3 4 2
g) (1, –3) j) (1, 4) k) (3, –1)
4 3 3 4

13. a) ( q
,
1
1 + pq 1 + pq ) b) ( l , l )
pq q
c) a ,
b
(b b – 1) d) ( 2
a
2
–b
, 2 2
a –b a –b )
14. a) ( 904 998
,–
541 541 ) b) ( –1.625 –1.183
871
,
871 ) c) ( 6 109
,
7 7 ) d) (–525 25
,
736 92 )
e) (
–561 1.089
,
197 197 ) f) ( –325 585
,
353 353 ) g) ( –3 53
,
61 122 ) h) ( ,)
–5 1
14 3

i) (17 –101
,
44 176 ) j) ( –637 –1.729
,
696 464 ) k) ( 160 –177
,
79 158 ) l) ( )
28 120
,
313 313
15. 403 y 205 16. 12 y 7 17. 25 18. 84
19. Ambos números son iguales a 15. 20. 165 y 43
21. Tiene 15 monedas de $ 100 y 9 de $ 50 22. 13 23. 45
24. Eliana tiene 9 años, Miguel tiene 45 años. 25. 243 y 81 26. 10 y 2
27. 21 y 19 28. Manuel tiene 59 años, Marcos tiene 11 años.
29. Ángela tiene 17 años, Sergio tiene 20 años. 30. 40 y 20
31. 150,5 y 18,5 32. 24,5 y 15,5 33. 10 de $ 100 y 11 de $ 50
34. 98 35. Teresa tiene 25 años, Adolfo tiene 35 años.
36. 18 y 12 37. 31 38. Ximena tiene 52 años, Matías tiene 10 años.
39. a) (–1, 0, –2) b) (1, – 4, 1) c) (0, –1, 2) d) (2, 3, –3) e) (–5, –2, 1)
f) (– 4, – 2, 2) g) (–1, –5, 2) h) (–2, 4, –1) i) (1, –2, – 4) j) (–2, –5, 3)
k) (– 4, –2, –2) l) (3, –2, 2) m) (– 4, –3, – 4) n) (–5, 0, –5) ñ) (2, 1, – 4)
o) (2, 3, 1, –1) p) (1, 2, 0, 3) q) (1, –1, 1, –1) r) (1, 0, 2, –2)

40. a) – , ( 1 1
,–
6 18 11
1
) ( )
b) – , 6, 2
1
2 (
c) – , –
5
6
10 –10
,
47 17 ) d) ( 22 11 44
,
17 2
,–
23 )
e) (– 4,
–8 8
,
41 15 ) f) (– )
29 –29 29
,
8 9 5
, g) (– 2 2
, ,–
9 15 2
1
) h) ( 14 28 56
, ,
17 11 43 )
i)
19
5 ( –2 –
, ,
19
6 ) j) ( )
2 –12 –6
, ,
5 11 5
k) (2, 2, –1) l) ( 1 –3 3
, ,
6 8 11 )
(
23 23 23
m) 19 , 29 , 60 ) n) (– 51
–17 51
,
5 55), o)
–18
23
, 9, 3 ( )
41. a) iii) I k 5 b) iii) k – 2 sol. única c) iii) Si k 3 o k 7 sol. única
2 5
iii) no existe k iii) no existe iii) no existe k
iii) k = 5 iii) k = – 2 iii) k = 3 o k = 7 no tiene sol.
2 5

210 Relaciones y funciones

210-211.(2003) 210 20/11/02, 11:13 AM


CAPÍTULO 3

42. (2, 0, 3)

43. El sistema tiene infinitas soluciones porque las ecuaciones 1 y 3 representan el mismo
1
plano. Una solución es (0, , 1)
2
44. a) El sistema no tiene solución porque b) El sistema no tiene solución porque
las ecuaciones 1 y 2 representan las ecuaciones 1 y 3 representan
planos paralelos. planos paralelos.
45. 556 46. Andrés: $ 15.000, Arturo: $ 30.000 y Carlos: $ 5.000
47. 57°, 33° y 90° 48. Matías $ 19.000, Javier $ 23.000 y Daniel $ 51.000
49. 116°, 10° y 54° 50. – 3, 10 y 1
51. Gloria 16 años, Regina 35 años, Susana 29 años.
52. – 56, 30 y 48 53. – 38, 25 y 39 54. 60°, 120° y 80°
55. Camisa $ 4.200, Chaleco $ 12.700 y Calcetín $ 800

3.4.5 Inecuaciones con dos variables.


Sistemas y problemas de
programación lineal.
Una inecuación de la forma
f(x, y) > 0, f(x, y) < 0, f(x, y) ≥ 0, f(x, y) ≤ 0
es una relación que geométricamente representa un semiplano
determinado por la recta, cuya ecuación es f(x, y) = 0.
Para determinar el semiplano solución se aplica el siguiente
teorema:
Teorema:
Sea P un punto de uno de los semiplanos en que la recta
representada por la ecuación f(x, y) = 0 divide al plano.
Si f(x, y) > 0 en P, entonces f(x, y) > 0 para todos los puntos que
pertenecen al mismo semiplano de P.
En forma análoga se aplica para
f(x, y) < 0, f(x, y) ≥ 0 o f(x, y) ≤ 0
Si consideramos más de una relación de las formas señaladas
estamos frente a un sistema de inecuaciones de primer grado y
su solución será la intersección de los semiplanos, solución de las
distintas inecuaciones que lo formen.
Una interesante aplicación de los sistemas de inecuaciones de
primer grado son los problemas de optimización o de programación
lineal. (Ver problema.)

Relaciones y funciones 211

210-211.(2003) 211 20/11/02, 11:13 AM


Ejercicios 1. Graficar el conjunto solución de la inecuación 2x – y + 1 < 0
Solución:
resueltos Consideramos la ecuación 2x – y + 1 = 0 y trazamos la recta
que determina.
Consideremos el punto P = (1, 1) en la
y inecuación dada:
2x – y + 1 < 0
ón

1
ci

2•1–1+1<0
lu

x 2 <0 F
So

–1
2
Luego es solución el semiplano contrario
al que pertenece P.
NOTA: Achuramos el semiplano que no es solución. Los puntos
sobre la recta 2x – y + 1= 0, no son solución. Se usa dibujar
esa recta discontinua.
2. Graficar el conjunto solución del siguiente sistema de inecuaciones.
2x + 5y – 1 ≥ 0
x – 3y + 5 ≥0
3x + 2y – 7 £ 0
Consideremos las rectas
L1: 2x + 5y – 1 = 0
L2: x – 3y + 5 = 0
L3: 3x + 2y – 7 = 0
y
Las graficamos y de la misma forma que
elegimos el semiplano solución en el
ejercicio anterior lo hacemos acá. Luego
L2
señalamos la intersección de los tres
2
So
semiplanos.
1 lu
ci
ón

-2 3 x
-1
L1

L3

3. Determinar el polígono que se forma al graficar las inecuaciones


dadas y encontrar sus vértices.
5x – 4y – 28 £ 0
2x + 3y – 25 £ 0
2x – 3y + 5 ≥ 0
5x + 2y – 16 ≥ 0
Solución:
Consideremos las rectas
L1: 5x – 4y – 28 = 0 L3: 2x – 3y + 5 = 0
L2: 2x + 3y – 25 = 0 L4: 5x + 2y – 16 = 0

212 Relaciones y funciones

212-213. 212 08/11/2001, 17:17


CAPÍTULO 3

Graficando las rectas y eligiendo


Y L2
el semiplano solución como en el L4 L3
problema 1 nos encontramos con el
cuadrilátero P Q R S. 5 .R

Resolviendo el sistema

L1: 5x – 4y – 28 = 0
3 S . . Q

L2: 2x + 3y – 25 = 0

Encontramos el punto Q = (8, 3)


2 4 5 8 X
Resolviendo el sistema
L2: 2x + 3y – 25 = 0
L3: 2x – 3y + 5 = 0
–2
P
.
L1
Encontramos el punto R = (5, 5)
Resolviendo el sistema
L3: 2x – 3y + 5 = 0
L4: 5x + 2y – 16 = 0 Encontramos el punto S = (2, 3)

Resolviendo el sistema
L4: 5x + 2y – 16 = 0
L1: 5x – 4y – 28 = 0 Encontramos el punto P = (4, –2)

4. Se tiene una región del plano definida por las inecuaciones.


1–x– y£0 (1)
0 £ x£4 (2)
0 £ y£3 (3)

Determinar para qué punto (x, y) de la región dada, la función


F(x, y) = 4x + 3y es máxima y para qué valores es mínima.
y
Solución:
Si graficamos la región obtenemos:
1 1–x–y£0
S R
2 0 £x£4 3

3 0 £y£3
4x

Todos los puntos de la región P Q R S T T


+
3y

cumplen con las tres condiciones planteadas


=

Q
0

3
en el sistema de inecuaciones. Si evaluamos P x

la función F(x, y) = 4x + 3y en los puntos P,


Q, R, S, T y cualquier otro de la región F, es
máxima para R y mínima para T.
En efecto: F(x, y) = 4x + 3y 2 2 1

Relaciones y funciones 213

212-213. 213 08/11/2001, 17:17


P = (1, 0) F (1, 0) = 4
Ejercicios Q = (4, 0) F (4, 0) = 16
resueltos R = (4, 3) F (4, 3) = 4 •4 + 3 • 3 = 25
S = (0, 3) F (0, 3) = 9
T = (0, 1) F (0, 1) = 3
∴ 4x + 3y es máxima si x=4 e y=3
\ 4x + 3y es mínima si x=0 e y=1
Observamos que la recta 4x + 3y = 0 pasa por el origen. Para
maximizar la función F(x, y) = 4x + 3y debemos hallar la recta
paralela a 4x + 3y = 0 que intersecta al eje y en el mayor valor
posible y que pase por la región dada. Ésta es la recta paralela a
4x + 3y = 0 que pasa por R.
Para minimizar F(x, y) = 4x + 3y debemos hallar la paralela a
4x + 3y = 0 que intersecte al eje y en el menor valor posible y ésta
es la recta paralela a 4x + 3y = 0 que pasa por T.
5. En una fábrica se producen refrigeradores de dos tipos: corrientes
y de lujo. Se trabaja en dos secciones, una de montaje, la cual
dispone de un máximo de 120 horas de trabajo al día, y una de
acabado, que dispone de 180 horas de trabajo diario. Para producir
un refrigerador corriente se necesitan 3 horas de montaje y 3 de
acabado. Para producir uno de lujo debe disponerse de 3 horas
de montaje, pero 6 horas de acabado. La ganancia al producir un
refrigerador corriente es de $ 30.000 y al producir uno de lujo es
de $ 40.000. ¿Cuántos refrigeradores de cada tipo deben producirse
diariamente para obtener ganancia máxima?
Solución:
La función que debemos maximizar es la función que expresa la ganancia.
Supongamos que se producen x refrigeradores corrientes e y
refrigeradores de lujo diariamente.
Luego, la ganancia será: 30.000x + 40.000y
F (x, y) = 30.000x + 40.000y
Veamos ahora qué restricciones debemos observar:
horas de montaje 3x + 3y £ 120 1
horas de acabado 3x + 6y £ 180 2
naturalmente x ≥ 0, y ≥ 0 3
y Graficando las restricciones (ver gráfico).
40
La recta F (x, y) = 30.000x + 40.000y = 0 que
30 se puede escribir 3x + 4y = 0 nos da la dirección
P
de maximización. Así vemos que en P la función
F (x, y) es máxima.

3x 40 60 x
+
4y
=
0
➀ ➁
214 Relaciones y funciones

214-215. 214 08/11/2001, 17:19


CAPÍTULO 3

Para hallar P debemos resolver el sistema.


3x + 3y = 120
3x + 6y = 180

x + y = 40
x + 2y = 60

y = 20 A x = 20
Luego para que la ganancia sea máxima deben fabricarse 20
refrigeradores corrientes y 20 refrigeradores de lujo.
Si evaluamos la función, observamos que la ganancia es de $ 1.400.000
F (x, y) = 30.000 • 20 + 40.000 • 20 = 1.400.000.
Se puede comprobar fácilmente que usando cualquier otro punto
de la región, definida por las sustracciones, el valor de la función
ganancia resultaría inferior a $ 1.400.000.
6. En una plantación se ha detectado una enfermedad y para combatirla
se necesita una mezcla que contenga como mínimo 15 partes de
una sustancia A y 20 partes de otra sustancia B. En el mercado sólo
se encuentran dos productos que pueden ser usados mezclados.
Uno tipo “x”, que contiene 1 parte de A y 5 partes de B y que
cuesta $ 1.000 el litro. Otro tipo “y”, que contiene 5 partes de A
y 2 partes de B y su valor es de $ 3.000 el litro. ¿Qué cantidad
se debe mezclar de cada uno para satisfacer las necesidades
con un costo mínimo?
Solución:
Sea x la cantidad de producto “x”
Sea y la cantidad de producto “y”
La función que queremos minimizar es la función que expresa
el costo de la mezcla.
F (x, y) = 1.000x + 3.000 y.
Las restricciones que nos impone el
10
problema son:
1 Debe haber a lo menos 15 partes de
sustancia A en la mezcla.
x + 5y ≥ 15 3
2 Debe haber a lo menos 20 partes de 2,4 . 8

sustancia B en la mezcla.
5x + 2y ≥ 20 3,1 4 15 ➀
x + 3y
=0
3 Naturalmente x ≥ 0 e y ≥ 0
Graficando las restricciones.

Relaciones y funciones 215

214-215. 215 08/11/2001, 17:19


Ejercicios La función F (x, y) = 1.000x + 3.000y debe ser mínima.
Vemos que la ecuación 1.000x + 3.000y = 0 o x + 3y = 0 nos
resueltos da la dirección de minimización y la función se hace mínima
en el punto P.
Para hallar las coordenadas de P debemos resolver el sistema:
x + 5y = 15
5x + 2y = 20

70
cuya solución es x= ≈ 3,1
23
55
y= ≈ 2,4
23
Luego para que el costo de la mezcla sea mínimo se debe adquirir 3,1
litros del producto tipo “x” y 2,4 litros del producto tipo “y”.
Su costo es F (3,1 ; 2,4) = 1.000 • 3,1 + 3.000 • 2,4
= 3.100 + 7.200 = 10.300
Si se evalúa la función costo en cualquier otro punto de la región
definida por las restricciones, se verá que el costo es mayor
que $ 10.300.

Ejercicios
n) x – 3y < 9
1. Trace la gráfica del conjunto solución
o) x–y–2>0
de las siguientes inecuaciones con dos
variables. 2. Grafique la solución de cada uno de los
a) 2x + y – 10 < 0 siguientes sistemas de inecuaciones.
b) x > 3,1 a) 4x + 7y < – 5
c) 4x + 3y + 21 < 0 2x – 9y + 1 < 0
d) x < 1,3 x > – 2,7
e) 2x – 3y > 18
f) x < 4,3
g) – 7x + y – 9 < 0 b) 8x + 13 y – 9 > 0
h) x – 2y – 4 > 0 9x – 5y + 16 < 0
i) x > – 1,9 x < 2,5
j) 2x – 8y > – 8
k) x < 3 c) 5x + 10y < – 15
l) – 3x + 6y £ – 24 3x – 6y – 9 < 0
m) – 10x + y ≥ 25 x<3

216 Relaciones y funciones

216-217. 216 08/11/2001, 17:20


CAPÍTULO 3

c) x+y <0
d) 6x + 5y – 7 > 0 7x – 10y > – 15
9x – 4y > – 2 y+2 >0
x > – 2,9

4. Determine cuál o cuáles de los puntos


e) 6x + 11y – 14 < 0
dados pertenecen al conjunto solución
5x – 7y + 6 > 0
del sistema dado:
x < 3,1

f) 3x + 5y + 1 > 0 a) 10x + 10y + 20 > 0


5x – 12y – 23 > 0 3x – 13y + 5 < 0
x + 4,1 > 0
y+3 >0

(1, 0), (0, 1), (1, –1), (–1, –1)


g) 6x + 6y + 12 > 0
(–1, 1) (3, –3)
9x – 8y – 4 > 0
x – 3,3 < 0
b) 4x + 13y + 11 < 0
h) 10x + 11y – 5 ≥ 0 7x – 9y + 3 > 0
9x – 8y – 4 < 0
y ≤2 (0, 0), (2, 2), (1, –3), (–1, 5),
(–1, –1), (3, 0)

i) 2x + 3y – 2 > 0 c) 7x + 4y > 6
4x – 3y + 8 > 0 9x – 9y + 27 < 0
2x + y – 2 < 0
(0, 0), (2, 1), (3, 7), (3, –7), (0, 5),
(–3, –1)
j) 6x + 8y < 0
9x – 11y – 8 < 0 5. Determine el polígono que se forma
al graficar las inecuaciones dadas y
x+3 >0
encuentre sus vértices.

2x – y – 3 £ 0
3. Escriba a lo menos dos puntos que sean x + 2y – 4 £ 0
solución de cada uno de los siguientes
x – 2y + 4 ≥ 0
sistemas de inecuaciones:
x + 5y + 4 ≥ 0
a) 6x + 10y < – 6
6. Determine el polígono que se forma
5x – 11y > 13
al graficar las inecuaciones dadas y
x + 3,1 > 0 determine sus vértices.

b) 6x + 6y + 18 > 0 5x – 6y – 26 £ 0
5x – 3y > – 4 3x + 2y – 10 £ 0
x <4 2x – y – 2 ≥ 0

Relaciones y funciones 217

216-217. 217 08/11/2001, 17:20


Ejercicios
10. Maximice la función F(x, y) = 2x + 3y en
7. Considere la región del plano definida la región definida por las restricciones
en el siguiente gráfico: siguientes:
y
– 6x + 5y – 10 £0
4 7x – 3y – 15 £0
2x – 3y + 10 ≥ 0
x ≥ 0, y ≥0

11. Un artesano fabrica dos tipos de anillos.


2 5 x
Para los del tipo A requiere 1 gr. de
oro y 1,5 gr. de plata y los vende a
a) Determine el sistema de inecua-
$ 4.000 cada uno. Para los del tipo B
ciones que definen la región.
necesita 1,5 gr. de oro y 1 gr. de plata
b) Maximice la función y los vende a $ 5.000 cada uno. Si
F (x, y) = 4x – 2y dispone de 750 gr. de oro y 750 gr.
de plata, ¿cuántos anillos de cada tipo
8. Maximice la función F (x, y) = 3x + 2y deberá fabricar para obtener el máximo
sujeta a las siguientes condiciones. de dinero por su venta?
5y – 7x £ 10
7x – 3y £ 15 12. Se desea contratar movilización para
3y – 2x £ 10 trasladar a 400 personas y se dispone
x 0, y ≥ 0 de las siguientes alternativas. Hay 8
buses con capacidad para 40 personas
y cada uno cuesta $ 12.000, y 10 buses
9. Maximice y minimice la función
con capacidad para 50 personas, con
F(x, y) = 4x + 5y en la región definida
un valor de $ 16.000 cada uno. Si se
por:
dispone sólo de 9 conductores para
12x + 3y £ 120 esa oportunidad, ¿cuántos buses de
3x + 2 y £ 90 cada tipo convendría arrendar para
x + 2y £ 50 que el viaje resulte lo más económico
x ≥ 0, y ≥ 0. posible?

Soluciones

1. a) b) c)
Solución

10
– 21
4
So
Sol

lu
ció
uci

n
ón

5 3,1 – 21
3

218 Relaciones y funciones

218-219.(2003) 218 20/11/02, 11:16 AM


CAPÍTULO 3

d) e) f)

Solución
9
So

i ón
uc
lu

-6
l
So
ci

4,3
ón

1,3

9
g) h) i)

Solución
Solución 4
ó n
ci
-9 -1,9
7 u
-2 S ol

j) k) l)
S
o

1
lu

8
c

-4 3

n

n

-4 c
Solución lu
So
Solución

m) n) o)

i ón
l uc 9
So
n

2

c
lu

-3
o

-2,5 -2
S

Solución

2. a) 2
b) 2 c)
1
So

1
luc
ión

-3 -2 -1 1 2 -3 3
-1
-1 Solución
-2

Relaciones y funciones 219

218-219.(2003) 219 20/11/02, 11:16 AM


Soluciones

d) e) y f) y

2
1 1
1 Solución
-1 2 x -1 1 2 3 4 5 x
-3 -1 1 2 -1
uc ión
Solución -2 Sol

y y y
g) h) i) 3

2 2

So
n

luc
1 luc

ión
o S
-2 1 3 x -1 1 x -2 1 x
-1 -1
Solución
-2

j) y

1
ión
Soluc

-1 1 x
-1

3. a) Entre otros: (1, – 2) (0, – 5) b) Entre otros: (0, 0) (3, 1)

c) Entre otros: 1, – 3 (– 2, – 1)
2
4. a) Sólo (0, 1) (– 1, + 1) b) (1, – 3), (– 1, – 1) c) (3, 7) , (0, 5).
5. Se forma un cuadrilátero cuyos vértices son (1, – 1), (2, 1), (0, 2), (– 4, 0).
6. Se forma un triángulo cuyos vértices son: (– 2, – 6), (4, – 1) y (2, 2).
7. 2x – y ≥0
4x – 5y £ 0 F (x, y) = 4x – 2y es máxima para (5, 4) y vale 12.
0£y£4

8. F (x, y) = 3x + 2y y lo máximo para x = 5, y = 20


3
9. F (x, y) = 4x + 5y es máxima para x = 4,3 y = 22,8 y es mínima para x = 0, y = 0.
20
10. F (x, y) = 2x + 3y es máxima para x = 5, y =
3
11. 300 de cada tipo.
12. Hay que contratar 5 buses de capacidad 40 y 4 buses de capacidad 50.

220 Relaciones y funciones

220-221.(2003) 220 20/11/02, 11:19 AM


CAPITULO 3
CAPÍTULO

Prueba de selección múltiple


1. Dadas las siguientes expre- I) pfiq 8. Sea
siones del lenguaje, son II) qfip A = {x E N / x > 1 A x £ 4}.
proposiciones. III) Bp fi q Determinar cuál de las
III) Cuént ame un cuento A. Sólo I siguientes proposiciones es
III) Gagarin voló a la Luna B. Sólo II falsa:
III) La Tierra está en la C. Sólo I y II
A. 1 F A
vía láctea. D. Sólo I y III
E. Sólo II y III B. 2 E A
A. Sólo I C. 4 F A
B. Sólo II 5. Sea p (x) : x es solución D. 5 F A
C. Sólo III de la ecuación x2 – 1 = 0. E. 3 E A
D. Sólo I y II Sea E = {–1, 0, 1}. De las
E. Sólo II y III proposiciones siguientes
9. Sean
son falsas:
2. La función proposicional A = {factores de 12}
“x no es múltiplo de 3” es I) (Ix E E), p(x) B = {factores de 18}
falsa si x vale: II) (Hx E E), p(x)
Determinar cuál de las
III) (Hx E E), Bp(x)
A. 1 siguientes proposiciones
B. 2 A. Sólo I es verdadera:
C. 3 B. Sólo II
C. Sólo III A. 4E A A 4 F B
D. 4
E. 5 D. Sólo I y II B. 2E A A 2 F B
E. Sólo II y III C. 9E A A 9 F B
3. Sea A= {1, 2, 3, 4} D. 6E A A 6 F B
6. Sean
y B= {1, 2, 3}. E. 9 E A A 9 E B
p(x) : x es menor que 3
De las siguientes proposi- q(x) : x es mayor que 1.
ciones son verdaderas: 10. A = {1, 2, 3}
De las proposiciones
siguientes son verdade- La cardinalidad del con-
p: (In E A) (H m E B)
ras: junto potencia de A es:
(m + n) E A
I) (Hx E N), p(x) A q(x) A. 2
q: (In E A) (Im E B)
II) (Ix E N), p(x) V q(x) B. 3
(n – m + 1) E A
III) (Hx E N), Bp(x) A B q(x) C. 4
r: (In E A) (Hm E B) D. 8
(n – m + 1) E A A. Sólo I
B. Sólo II E. 16
A. Sólo p C. Sólo III
B. Sólo q D. Sólo I y II 11. El conjunto
C. Sólo r E. Sólo I, II y III F = {n (n+1) – 1 / n E N,
D. Sólo p y r
n < 6}, escrito por exten-
E. p, q y r 7. Dado el conjunto
A = {1, 2, 3}, entonces: sión es:
4. Sean p = n + m es par y
A. {1, 5, 11, 19, 29, 41}
q = m – n es par, dos fun- A. 1 F A
ciones proposicionales en B. {1, 5, 11, 19, 29}
B. 2 C A
el conjunto de los números C. {0, 1, 5, 11, 19}
C. {3} E A
enteros. De las propo- D. {0, 1, 5, 11, 19, 29}
D. Δ E A
siciones siguientes son E. {–1, 1, 5, 11, 19}
verdaderas: E. {1} C A

Relaciones y funciones 221

220-221.(2003) 221 20/11/02, 11:19 AM


Prueba de selección múltiple
12. Si A = {7, 13, 19, 25, 31}. entonces #A K B es: A. 4
¿Cuál de los siguientes A. 4 B. 8
conjuntos es distinto de B. 7 C. 10
A? C. 8 D. 14
D. 9 E. 16
A. {7n – (n –1)
E. 11
/ n E N, n £ 5} 19. Sean A = {1, 2, 3}
B. {6n + 1 / n E N, n £ 5} 16. Sea B = {5, 6, 7}
C. {8n – (2n – 1) A = {x E R De las siguientes relacio-
/ n E N, n £ 5} / – 2 £ x £ 2} nes, ¿cuáles son de A
D. {5n + (n + 1) B = {x E R en B?
/ 0 £ x £ 5}
/ n E N, n £ 5} I {(1, 5) (2, 7) (1, 7)}
C = {x E R
E. {4n + (2n –1) II {(5, 1) (6, 1)}
/ 1 < x £ 7}
/ n E N, n £ 5} III {(3, 5) (3, 6) (3, 7)}
El conjunto (A J B) – C es:
13. ¿Cuál de las siguientes A. Sólo I
proposiciones representa A. {x E R / 0 £ x < 1} B. Sólo II
la parte coloreada del B. {x E R / 0 £ x £ 1} C. Sólo III
diagrama? C. {x E R / 1 < x £ 2} D. Sólo I y II
I) (A J B) – (B J C) D. {x E R / 1 £ x < 2} E. Sólo I y III
II) [B J (A K C)] – (A J C) E. {x E R / 0 < x < 1}
20. Sean A = {2, 4, 6}
III) [(B J C) K (A J B)] –
17. Si A C B y #B = 12 y B = {1, 3, 5, 7}.
(A J C)
y #A = 6, entonces La relación R : A Q B
se define por:
A B #A J B es:
R = {(x, y) / y = x – 1}.
A. 6 Entonces R=
B. 12
A. {(3, 2) (5, 4) (7, 6)}
C. 18
C D. 24 B. {(2, 1) (4, 5) (6, 7)}
E. No se puede C. {(2, 1) (4, 3) (6, 5)}
A. Sólo I
determinar D. {(2, 3) (4, 5) (6, 7)}
B. Sólo II
E. {(1, 2) (3, 4) (5, 6)}
C. Sólo III 18. Se ha consultado a 28
D. Sólo I y II personas por el consumo 21. Sea R la relación en
E. Sólo II y III de 3 productos A, B y C, Z x Z definida por
14. La expresión obteniéndose el siguiente R = {(x, y) / x ≥ 2y}
resultado: Son pares de esta
(A J B) – (B J C)
es equivalente a: relación:
3 consumen A, B y C
A. A' J B J C 7 consumen A y C I) (5, 3)
B. A J B' J C 6 consumen B y C II) (5, 1)
5 consumen A y B III) (5, 2)
C. A J B J C'
D. A K (B J C) 5 consumen sólo A A. Sólo I
E. (A K B) J C 7 consumen sólo B B. Sólo I y II
C. Sólo I y III
15. Si #A = 4 y #B = 7 ¿Cuántas personas con- D. Sólo II y III
y #A J B = 3, sumen el producto C? E. I, II y III

222 Relaciones y funciones

222-223. 222 08/11/2001, 17:25


CAPITULO 3
CAPÍTULO

22. En la relación N → N 25. De las siguientes relacio- (c, b) (a, b) (d, d)


definida por R = {(x, y) / nes en N, cuál de ellas (c, d) (a, d)}
2x + y £ 10}, el domi- es refleja.
El diagrama de orden
nio y el recorrido son A. x Ry ¤ x • y £ 0 definido por S es:
respectivamente:
B. x Ry ¤ x • y = x2
A. {1, 2, 3, 4} d b
{2, 4, 6, 8} C. x Ry ¤ x – y = 2 c
A.
D. x Ry ¤ x + y es impar a
B. {1, 2, 3, 4} d
{1, 2, 3, 4, 5} E. x Ry ¤ x + y = 12 c
B. b
C. {1, 2, 3, 4} a
{1, 2, 3, 4, 5, 6, 7, 8} 26. De las siguientes relaciones
en N, determine cuál de c
D. {2, 4, 6, 8} b d
ellas es de equivalencia C. a
{1, 2, 3, 4}
A. x Sy ¤ x + y es par d
E. {1, 2, 3, 4, 8, 6, 7, 8}
{1, 2, 3, 4} B. x Sy ¤ y = 2x b
D.
C. x Sy ¤ x > y a c
23. Si R = {(3, 1), (4, 1), D. x Sy ¤ x < y d
(5, 1), (6, 1)} E. x Sy ¤ 2x < y
entonces R–1 es E. a b c
A. {(3, 1), (4, 1), (5, 1), 27. Sea A = {2, 5, 6, 9} y
30. La relación R en
(6, 1)} R = {(9, 2), (9, 9), (6, 6),
(2, 9), (2, 2), (5, 5)} A = {1, 2, 3} definida por
B. {(3, 2), (4, 2), (5, 2),
(6, 2)} una relación de equiva- R = {(3,2) (2,3) (3,3)} es:
lencia en A. Entonces la
C. {(1, 3), (1, 4), (1, 5), A. Refleja y transitiva.
clase del 9 es:
(1, 6)} A. {2} B. Refleja y simétrica.
D. {(2, 3), (2, 4), (2,5), B. {9} C. Simétrica y transitiva.
(2, 6)} C. {2, 9} D. Refleja y
E. {(3,3), (4, 4), (5, 5), D. {5, 6}
antisimétrica.
(6, 6)} E. {2, 5}
E. Transitiva y
24. Si R = {(x, y) E N x N 28. De las siguientes relacio-
antisimétrica.
/ x es el doble de y}, nes en N, determine cuál
de ellas es una relación 31. De las relaciones siguien-
entonces R–1 es:
de orden. tes en A = {1, 2, 3},
A. {(x, y) E N x N cuáles de ellas son fun-
/ x es el doble de y} A. x Ry ¤ x divide a y ciones:
B. x Ry ¤ x + y = 2
B. {(x, y) E N x N f1 = {(1,1) (2,2) (3,3)}
C. x Ry ¤ x + y es par
/ x es la mitad de y}
D. x Ry ¤ x = 2y f2 = {(1,1) (2,1) (3,1)}
C. {(x, y) E N x N E. x Ry ¤ x es menor f3 = {(1,2) (2,3) (3,1)}
/ y es la mitad de x} que y
D. {(x, y) E N x N A. Sólo f1
29. Sea A = {a, b, c, d} y S la
/ y es x más 2} B. Sólo f1 y f2
relación de orden en A
E. {(x, y) E N x N definida por: C. Sólo f2 y f3
/ y es x menos 2} D. Sólo f1 y f3
S = {(a, a) (c, c) (b, b) (a, c)
E. f1, f2 y f3

Relaciones y funciones 223

222-223. 223 08/11/2001, 17:25


Prueba de selección múltiple
define una función real; B. 2x –y+3=0
32. En la función de 1 es imagen de: C. 2x –y–3= 0
A = {1, 2, 3} en
A. Sólo 1 D. 2x +y–4=0
B = {4, 5, 6,}
definida por B. Sólo 3 E. 2x +y+4=0
f = {(1,4) (2,4) (3,4)}, C. 1y3 41. De los tríos de puntos
el dominio y el rango D. Sólo 0 siguientes son colineales:
E. –1 y –3
son: I (3, 2) (1, 0) (–1, –2)
A. {1, 2, 3}, {4, 5, 6} 37. Si II (6, 1) (3, 2) (3, 4)
B. {1, 2, 3}, {4, 5} 2 si x ≤ –1 III (3, 1) (4, 3) (5, 7)
C.
D.
{1, 2, 3}, {5, 6}
{1, 2, 3}, {4, 6}
{
f (x) = –x + 1 si –1 < x <1
–1 si x ≥ 1
A.
B.
Sólo I
Sólo II
E. {1, 2, 3}, {4} C. Sólo III
define una función real;
33. Dada la función real su rango es: D. Sólo I y II
f (x) = 2x2– 3x + 1, E. Sólo II y III
A. (– •, –1]
la imagen de 1 es: 42. De las ecuaciones siguien-
B. [–1, 0]
A. 2 C. [–1, 2] tes, la que representa una
B. 1 D. [0, 2] K {–1} recta paralela a la recta
C. 0 E. [2, + •) x – 2y + 3 = 0 es:
D. –1 A. 2y +x+3=0
38. La recta que pasa por
E. –2 B. 2y –x–6= 0
los puntos (1, 2) y (-3, 1)
tiene por ecuación: C. 2y +x+6=0
34. Dada la función
D. y+2x –3=0
x – 1 si x < 2

{
A. x – 4y + 9 = 0
E. y+2x +6=0
f (x) = 3 si x = 2 B. x + 4y – 9 = 0
x + 1 si x > 2 C. x + 4y + 7 = 0 43. Una recta perpendicular
D. x – 4y – 7 = 0 a la recta de ecuación
¿Cuál de las siguientes
E. x – 4y + 7 = 0 3x – y + 1 = 0 es la
proposiciones es falsa?
representada por:
A. 2 E Dom f 39. La recta cuya ecuación es
A. 3x + y + 3 = 0
B. 2 E Rang f 2y –x + 1 = 0 intersecta
B. 3y – x – 2 = 0
C. f (2) = 3 al eje x Y al eje y En los
C. 3y – x + 2 = 0
D. f (1) = 0 puntos:
D. 3y + x + 2 = 0
E. f (3) = 4 A. –1 y –1 E. 3x + y – 1 = 0
35. Dadas las funciones rea- B. –1 y 1 44. En kx – x + y + 3 = 0
les f (x) = x2 – 1 y C. – 1 y – 1 el valor de k para que
g (x) = 2x +3. La fórmula 2 2
la ecuación represente a
que define (g o f) (x) es: D. 1 y – 1
2 una recta que pasa por el
A. 2x2 – 1 E. – 1 y 1 punto (1, –3) es:
2
B. 2x2 + 3 A. 0
40. La recta cuya pendiente
C. 2x2 + 1 B. 1
es –2 y que pasa por el
D. x2 + 3x + 2 punto (1, 1) tiene por C. 2
E. 4x2 + 12x + 8 ecuación: D. – 1
36. Si f (x) = x2 – 4x + 4 A. 2x + y – 3 = 0 E.–2

224 Relaciones y funciones

224-225. 224 20/11/02, 11:22 AM


CAPITULO 3
CAPÍTULO

45. La pendiente de la recta B. 3x – 2y + k =0


A. (–1, –3)
que pasa por los puntos C. 2x – 3y + k =0
(3, 5) y (– 2, 1) es: D. –2x – 3y + k =0 B. (1, –3)
E. 3x + 2y + k =0 C. (–1, 3)
A. 4
D. (3, 1)
B. 5 50. De las siguientes funcio-
E. (–3, –1)
C. 4 nes en A = {4, 6, 8}
5 determinar cuál o cuáles 54. Los valores de p y q para
D. 5 son biyectivas. que la solución del sistema
4
4 I) f1 = {(4, 4) (6, 6) (8, 8)} x – 2py – 5 = 3
E. –
5 II) f2 = {(4, 6) (6, 6) (8, 6)} qx + 1 – 2y = 3
46. De las siguientes rectas, III) f3 = {(4, 6) (6, 8) (8, 4)} sea (4, 3) son respecti-
pasan por el origen: vamente:
A. Sólo f 1
I) x + 2y – 1 = 1 A. 2 y 2
B. Sólo f 3
II) 3x + 2y + 2 = 2 3
C. Sólo f 1 y f 2 B. 2 y 2
III) x – 5y = 0
D. Sólo f 2 y f 3 3
2
A. Sólo I C. – y –2
E. Sólo f 1 y f 3 3
B. Sólo II 2
D. – 2 y –
C. Sólo II y III 3
51. La función inversa de la 2
D. Sólo I y III E. – y 2
función real 3
E. I, II y III
f(x) = 2x – 1 55. El valor de k para que
x+3 el sistema
47. La ecuación de la recta 3x – 1
A. f –1(x) = 5x – y +3 = 0
paralela al eje X que 2–x
2kx + 3y – 1 = 0
pasa por el punto (4, –1) B. f –1(x) = 3x + 1
es: 2–x no tenga solución es:
3x + 1
A. x–1 =0 C. f –1(x) =
x–2 A. –15
B. x+1=0 3x – 1
D. f –1(x) = B. 15
C. y–1 =0 x–2
x–3 C. 2
D. y+1=0 E. f –1(x) =
E. x+y=1 x–2 D. –2
52. La función inversa de la E. – 15
48. La ecuación de la recta función real
2
perpendicular al eje Y 56. La solución (x, y) del
f(x) = 3x + 5 es:
que pasa por el punto sistema
(– 3, – 4) es: A. f –1 (x) = 3 (x – 5) 2ax – by + 2 = 0
A. y – 4 = 0 B. f –1 (x) = 3x – 5 ax + 2by + 1 = 0 es:
B. y+4=0 x–5
C. f –1 (x) = 1 1
C. x + 4 = 0 3 A. ,
a b
D. x – 4 = 0 D. f –1 (x) = x – 5 1
3 B. a , 0
E. x–y=4 x+5
E. f –1 (x) = C. –
1
, –
1
49. La ecuación de la fami- 3 a b
lia de rectas que tiene 53. La solución del sistema 1
D. –
a , 0
pendiente 2 es: 2x – y – 5 = 0
3
x+y–4=0 es: E. 1 1
A. 2x + 3y + k = 0 – ,
a b

Relaciones y funciones 225

224-225. 225 20/11/02, 11:23 AM


Prueba de selección múltiple
57. La solución del sistema A. Sólo I C. Sólo III
B. Sólo II D. Sólo I, y III
x + y – z + 2u = 2
x + 2y – z – u = –1 C. Sólo III E. I, II y III
2x – y + 2z – 3u = 1 D. I, II y III
– x – 2y + z + 3u = 3 E. ninguno 60. Considerando las restric-
ciones
es: 59. Son soluciones del sis- 2x + y – 1≥ 0
A. (0, 1, 1, 1) tema –x + 2y £ 0
B. (1, 0, 1, 1)
2x + y – 1 ≥ 0 x£3
C. (1, 1, 0, 1)
x – 2y £ 0
D. (1, 1, 1, 0)
x–3£ 0 la función F (x, y) = x + y
E. (1, 1, 1, 1)
es máxima para:
58. ¿Son soluciones de la ine- I) (2, 1)
II) (3, – 5) A. 3 1,5
cuación x – 2y + 5 ≥ 0
los siguientes puntos? III) (2, 3) B. 3 0
C. 2 3
I) (1, 2)
A. Sólo I D. 3 2
II) (3, 4)
III) (–1, 2) B. Sólo II E. 1,5 3

Soluciones

1. E 2. C 3. C 4. C 5. A 6. D 7. E 8. C
9. A 10. D 11. B 12. E 13. E 14. C 15. C 16. B
17. A 18. D 19. E 20. C 21. D 22. C 23. C 24. B
25. B 26. A 27. C 28. A 29. A 30. C 31. E 32. E
33. C 34. B 35. C 36. C 37. D 38. E 39. D 40. A
41. A 42. B 43. D 44. B 45. C 46. C 47. D 48. B
49. C 50. E 51. A 52. C 53. D 54. E 55. E 56. D
57. B 58. D 59. D 60. A

226 Relaciones y funciones

226. 226 08/11/2001, 17:29


4
E
CAPÍTULO

cuaciones e
inecuaciones
de segundo grado

Ecuación cuadrática 4.1

La expresión ax2 + bx + c = 0, donde a, b y c son números


reales cualesquiera y a π 0, se llama ecuación cuadrática o
ecuación de segundo grado.
La solución de esta ecuación puede obtenerse por factorización
o aplicando la fórmula general.
Todas las ecuaciones de segundo grado tienen dos soluciones.

4.1.1. Solución de la ecuación


por factorización
Aplicamos aquí la siguiente propiedad:
a •b = 0 P a = 0 o b = 0
(si el producto de dos números reales es cero, entonces al menos
uno de ellos es cero).

1. Resolvamos la ecuación: x2 – 3x = 0
Ejercicios
Factorizando obtenemos: x2 – 3x = 0 resueltos
x (x – 3) = 0
y aplicando la propiedad indicada, nos queda:
x=0 o x–3=0
de donde obtenemos las soluciones x1 = 0
x2 = 3

Ecuaciones e inecuaciones de segundo grado 227

227. 227 8/11/01, 12:50


Ejercicios 2. Resolvamos 5x2 + 11x = 0

resueltos Factoricemos y apliquemos la propiedad:


5x2 + 11x = 0 fi x (5x + 11) = 0
fi x = 0 o 5x + 11 = 0

fi x1 = 0 ; x2 = – 11
5
3. Resolvamos x2 – 64 = 0
La factorización correspondiente es:
x2 – 64 = 0 fi (x – 8) (x + 8) = 0
fix–8=0 o x+8=0
fi x1 = 8 ; x2 = – 8
4. Resolvamos x2 – 5 = 0
Factorizando como suma por diferencia nos queda:
x2 – 5 = 0 fi (x – 5) (x + 5) = 0
fix– 5=0 o x+ 5=0

fi x1 = 5 ; x2 = – 5
5. Resolvamos x2 – x – 30 = 0
Procediendo como antes:
x2 – x – 30 = 0 fi (x – 6) (x + 5) = 0
fix–6=0 o x+5=0
fi x1 = 6 ; x2 = – 5

Ejercicios
Resuelva aplicando factorización:

1. x2 – 7x = 0 10. 5x2 + 24x = 0 19. x2 – 121 = 0 28. 9x2 – 16 = 0

2. x2 – 13x = 0 11. –9x2 + x = 0 20. x2 – 4 = 0 29. x2 – 15 = 0

3. x2 + 20x = 0 12. x2 + x = 0 21. x2 – 9 = 0 30. x2 – 3 = 0

4. x2 + 19x = 0 13. –x2 + x = 0 22. x2 – 100 = 0 31. x2 – 11 = 0

5. –x2 + 6x = 0 14. 11x2 – x = 0 23. x2 – 49 = 0 32. 6x2 – 24 = 0

6. –x2 – 9x = 0 15. x2 – 25 = 0 24. 2x2 – 50 = 0 33. 2x2 – 6 = 0


7. 7x2 – 5x = 0 16. x2 – 36 = 0 25. 3x2 – 12 = 0 34. 4x2 – 3 = 0
8. 13x2 + 2x = 0 17. x2 – 1 = 0 26. 5 – 5x2 = 0 35. 49x2 – 1 = 0
9. 20x2 – 4x = 0 18. x2 – 16 = 0 27. 4x2 – 1 = 0 36. x2 – 5x + 6 = 0

228 Ecuaciones e inecuaciones de segundo grado

228-229. 228 8/11/01, 12:54


CAPÍTULO 4

37. x2 – 6x + 5 = 0 49. x2 + 10x + 21 = 0 61. x2 – 6x + 9 = 0


38. x2 – x – 12 = 0 50. x2 + 14x + 45 = 0 62. x2 – 8x + 16 = 0
39. x2 + 7x – 18 = 0 51. x2 + 9x – 36 = 0 63. x2 + 18x + 81 = 0
40. x2 – 11x + 30 = 0 52. x2 – 5x – 36 = 0 64. x2 – 10x + 25 = 0
41. x2 – 9x – 22 = 0 53. x2 + 15x – 16 = 0 65. 4x2 + 4x + 1 = 0
42. x2 + 5x – 24 = 0 54. x2 – 9x + 20 = 0 66. 9x2 – 12x + 4 = 0
43. x2 + 3x – 28 = 0 55. y2 – y – 2 = 0 67. 9x2 – 6x + 1 = 0
44. x2 – 9x + 8 = 0 56. y2 – 13y + 40 = 0 68. 4x2 + 20x + 25 = 0
45. x2 + 15x + 36 = 0 57. y2 + 8y + 12 = 0 69. 9x2 + 24x + 16 = 0
46. x2 + 11x + 30 = 0 58. y2 + 10y + 24 = 0 70. 16x2 – 24x + 9 = 0
47. x2 – x – 20 = 0 59. x2 – 12x + 36 = 0

48. x2 – 13x + 42 = 0 60. x2 + 2x + 1 = 0

Soluciones
1. x1 = 0 x2 = 7 2. x1 = 0 x2 = 13 3. x1 = 0 x2 = – 20
4. x1 = 0 x2 = – 19 5. x1 = 0 x2 = 6 6. x1 = 0 x2 = – 9
2 1
7. x1 = 0 x2 = 5 8. x1 = 0 x2 = – 9. x1 = 0 x2 =
7 13 5
24 1
10. x1 = 0 x2 = – 11. x1 = 0 x2 = 12. x1 = 0 x2 = – 1
5 9
13. x1 = 0 x2 = 1 14. x1 = 0 x2 = 1 15. x1 = 5 x2 = – 5
11
16. x1 = 6 x2 = – 6 17. x1 = 1 x2 = – 1 18. x1 = 4 x2 = – 4
19. x1 = 11 x2 = – 11 20. x1 = 2 x2 = – 2 21. x1 = 3 x2 = – 3
22. x1 = 10 x2 = – 10 23. x1 = 7 x2 = – 7 24. x1 = 5 x2 = – 5
1 1
25. x1 = 2 x2 = – 2 26. x1 = 1 x2 = – 1 27. x1 = x2 = –
2 2
4 4
28. x1 = x2 = – 29. x1 = 15 x2 = – 15 30. x1 = 3 x2 = – 3
3 3
31. x1 = 11 x2 = – 11 32. x1 = 2 x2 = – 2 33. x1 = 3 x2 = – 3
3 3 1
34. x1 = x2 = – 35. x1 = 1 x2 = – 36. x1 = 2 x2 = 3
2 2 7 7
37. x1 = 5 x2 = 1 38. x1 = 4 x2 = – 3 39. x1 = 2 x2 = – 9
40. x1 = 5 x2 = 6 41. x1 = 11 x2 = – 2 42. x1 = 3 x2 = – 8
43. x1 = – 7 x2 = 4 44. x1 = 8 x2 = 1 45. x1 = –12 x2 = – 3
46. x1 = – 6 x2 = – 5 47. x1 = 5 x2 = – 4 48. x1 = 6 x2 = 7
49. x1 = – 3 x2 = – 7 50. x1 = – 9 x2 = – 5 51. x1 = 3 x2 = – 12
52. x1 = 9 x2 = – 4 53. x1 = 1 x2 = – 16 54. x1 = 4 x2 = 5
55. y1 = 2 y2 = – 1 56. y1 = 8 y2 = 5 57. y1 = – 6 y2 = – 2

Ecuaciones e inecuaciones de segundo grado 229

228-229. 229 8/11/01, 12:55


Soluciones

58. y1 = – 6 y2 = – 4 59. x1 = 6 x2 = 6 60. x1 = –1 x2 = –1


61. x1 = 3 x2 = 3 62. x1 = 4 x2 = 4 63. x1 = –9 x2 = –9
1 2 2
64. x1 = 5 x2 = 5 65. x1 = – x2 = – 1 66. x1 = x2 =
2 2 3 3
1 1 5 5
67. x1 = x2 = 68. x1 = – x2 = – 69. x1 = – 4 x2 = – 4
3 3 2 2 3 3
3 3
70. x1 = x2 =
4 4

4.1.2. Solución de la ecuación


cuadrática aplicando la
fórmula general
A partir de la ecuación general de segundo grado
ax2 + bx + c = 0
podemos obtener las soluciones x1 y x2 aplicando la fórmula:

x = – b ± b – 4ac
2

2a

Ejercicios 1. Resolvamos la ecuación x2 + 3x – 10 = 0 aplicando la fórmula.

resueltos Primero determinamos los coeficientes que son:


a = 1 ; b = 3 y c = – 10

y luego reemplazamos estos valores en la fórmula.

x = – 3± 9 + 40
2
– 3± 7
x=
2
obteniendo x1 = 2 y x2 = – 5
2. Resolvamos la ecuación 4x2 + 4x + 1 = 0
Los coeficientes son a = 4, b = 4 y c = 1

Reemplazando en la fórmula obtenemos:


– 4± 16 – 16
x=
8
– 4± 0
x=
8
1
lo cual nos da las soluciones iguales a – , es decir,
2
1
x1 = – y x2 = – 1
2 2

230 Ecuaciones e inecuaciones de segundo grado

230-231.(2003) 230 20/11/02, 11:41 AM


CAPÍTULO 4

3. Resolvamos la ecuación 2x2 + 3x – 1 = 0


Aplicando la fórmula para los valores a = 2 ; b = 3 y c = – 1

– 3± 9+8
obtenemos: x= 4
– 3± 17
x=
4
– 3+ 17 – 3 – 17
y obtenemos x1= y x2 =
4 4
Nota: Si la cantidad subradical no es un cuadrado exacto, la dejamos
expresada tal cual aparece, así como en el ejemplo anterior.

4. Resolvamos la ecuación x2 + x + 2 = 0
Los coeficientes en este caso son a = 1 ; b = 1 y c = 2,
aplicando la fórmula obtenemos:
– 1± 1– 8
x= 2
– 1± – 7
x= 2
y las soluciones son:

– 1+ – 7 – 1– – 7
x1 = y x2 =
2 2
Nota: Si la cantidad subradical es un número negativo, entonces las
soluciones son números complejos. El capítulo de números complejos
está estudiado más adelante, pero aquí podemos definir:

– 1 = i unidad imaginaria

Ej. : –2 =i 2
– 25 = – 1 25 = 5 i .......etc.

entonces en el ejemplo anterior, las soluciones pueden ser


expresadas por:
– 1+ i 7 – 1– i 7
x1 = y x2 = 2
2

5. Resolvamos la ecuación x2 + 2x + 5 = 0

Apliquemos la fórmula directamente:


– 2± 4 – 20 – 2 ± – 16 – 2 ± 4i
x= = = = – 1 ± 2i
2 2 2

y las soluciones son x1 = – 1 + 2i y x2 = – 1 – 2i

Ecuaciones e inecuaciones de segundo grado 231

230-231.(2003) 231 20/11/02, 11:42 AM


Ejercicios

Aplique la fórmula para resolver las siguientes ecuaciones:

1. x2 + x = 0 20. x (x + 5) – 3 = 2x (x – 6)

2. 3x2 – 2 = 0 21. 3x (x + 2) = (x + 5) (x – 5)

3. x2 + 2x + 1 = 0 22. (x – 6) (2 – x) = (x + 3)2 – (x – 2)2

4. x2 – x – 30 = 0 23. 5x (x + 2) = 2x (x + 1)

5. 2x2 + 3x – 1 = 0 24. x (x – 6) + 2x (x – 1) – x (x – 3) = 0

6. 3x2 – x – 2 = 0 25. (1 + x)2 + (2 + x)2 = (3 – x)2

7. x2 + 2x + 3 = 0 26. (x – 8)2 + (x – 5)2 = (x – 9)2

8. x2 – 5x – 4 = 0 27. (x + 6) (x – 6) – (x – 5)2 = 0

9. 4x2 + 4x + 1 = 0 28. (3x – 1) (x + 2) – x (x – 4) = 0

10. 2x2 + x – 2 = 0 29. a (x – a) + b (x – b) = x (x – a) + x (x – b)

11. x2 + 6x + 5 = 0 30. (a + x)2 + (b + x)2 = a2 + b2

12. x2 – 6x + 5 = 0 31. x2 + ax + b = 0

13. 3x2 + x – 2 = 0 32. x2 – 3abx = – 3ab (x – 3ab)


14. 2x2 + x – 1 = 0
33. 1 1
– = a–b
15. 6x2 +x+5=0 x–a x–b

34. 1 1
16. 3x2 – x – 1 = 0 + =1
x–2 x–3
17. 9x2 – 2x + 3 = 0 35. 1 + x – 1 – x = 3
1– x 1+ x
18. (2x – 3) (x + 1) = (x – 3) (x + 2)
36. 3 1
– =2
19. (x – 7)2 + 2x = (2x – 1) (x – 2) 2x – 1 2x + 1

Soluciones
6 – 6
1. x1 = 0 x2 = – 1 2. x1 = 3 x2 = 3. x1 = – 1 x2 = – 1
3
– 3 + 17 – 3 – 17 –2
4. x1 = 6 x2 = – 5 5. x1 = x2 = 6. x1 = 1 x2 =
4 4 3
5 + 41
7. x1 = – 1 + i 2 x2 = – 1 – i 2 8. x1 = x2 = 5 – 41
2 2
1 1 – 1 + 17 – 1 – 17
9. x1 = – x2 = – 10. x1 = x2 = 11. x1 = – 5 x2 = – 1
2 2 4 4
2 1
12. x1 = 5 x2 = 1 13. x1 = – 1 x2 = 14. x1 = – 1 x2 =
3 2

232 Ecuaciones e inecuaciones de segundo grado

232-233. 232 8/11/01, 13:05


CAPÍTULO 4

–1– i 119 1 + 13
15. x1 = –1+ i 119 x2 = 16. x1 = x2 = 1 – 13
12 12 6 6
1 + i 26 1 – i 26
17. x1 = x2 = 18. x1 = i 3 x2 = –i 3
9 9

–7 + 237 –7 – 237 17 + 277 17 – 277


19. x1 = x2 = 20. x1 = x2 =
2 2 2 2
–3+ i 41 –3– i 41
21. x1 = x2 = 22. x1 = –1 + 4i x2 = –1 – 4i
2 2
–8
23. x1 = 0 x2 = 24. x1 = 0 x2 = 5
3 2

25. x1 = –6 – 2 10 x2 = –6 – 2 10 26. x1 = 4 + 2 2 x2 = 4 – 2 2

– 9 + 97 –9 – 97
27. x1 = 61 x2 = No hay 28. x1 = x2 = 4
10 4
a+b+i a–b a+b–i a–b
29. x1 = x2 = 30. x1 = 0 x2 = –(a + b)
2 2
– a + a2 – 4b –a – a2 – 4b
31. x1 = x2 = 2
32. x1 = 3ab x2 = –3ab
2
a+b+ a– b 2+4 a+b– a– b 2+4 7+ 5 7– 5
33. x1 = x2 = 34. x1 = x2 =
2 2 2 2

–2+ 13 1 + 13 1 – 13
35. x1 = x2 = –2– 13 36. x1 = x2 =
3 3 4 4

4.1.3 Ecuaciones bicuadráticas


Estas ecuaciones tienen la forma
ax4 + bx2 + c = 0
y podemos resolverlas haciendo el siguiente cambio de variables
y = x2
Con este cambio, la ecuación original se transforma en una
ecuación cuadrática en la variable y:
ay2 + by + c = 0
y aplicando la fórmula general o factorizando podemos obtener los
dos valores de y, que son soluciones de la ecuación transformada.
A partir de cada valor obtenido para y, usando el cambio
de variable efectuado al comienzo, obtenemos dos valores para
la variable original x, y de este modo las 4 soluciones de la
ecuación original.
Nota: La ecuación original es de grado 4 y por lo tanto tiene
4 soluciones.

Ecuaciones e inecuaciones de segundo grado 233

232-233. 233 8/11/01, 13:07


Ejercicios 1. Resolvamos la ecuación: x4 – 5x2 + 4 = 0
resueltos Haciendo el cambio de variable y = x2 obtenemos:
y2 – 5y + 4 = 0
Resolviendo esta ecuación (por factorización o aplicando fórmula)
obtenemos las siguientes soluciones: y1 = 1 ; y2 = 4
Pero como y = x2 (recordemos que “y” es variable auxiliar, nosotros
debemos buscar los valores para la variable original “x”).
y1 = 1, esto implica x2 = 1, es decir x1 = 1
x2 = – 1
y2 = 4, es decir x2 = 4, entonces x3 = 2
x4 = – 2
y las cuatro soluciones de la ecuación original son:
x1 = 1 ; x2 = – 1 ; x3 = 2 ; x4 = – 2

2. Resolvamos la ecuación: x4 – 11x2 + 18 = 0


Hacemos el cambio de variable y = x2 y reemplazamos; nos
queda:
y2 – 11y + 18 = 0
Podemos factorizar (y – 9) (y – 2) = 0
y obtenemos las soluciones auxiliares: y1 = 9 ; y2 = 2
Volvemos a nuestra variable original del siguiente modo:
y1 = 9 implica x2 = 9, es decir x1 = 3
x2 = – 3
y2 = 2 implica x2 = 2, es decir x3 = 2
x4 = – 2

3. Resolvamos la ecuación x4 – 3x2 – 4 = 0


Haciendo y = x2, reemplazando y factorizando obtenemos:
y2 – 3y – 4 = 0
(y – 4) (y + 1) = 0
las soluciones auxiliares son: y1 = 4; y2 = – 1
y= 4 implica x2 = 4, es decir x1 = 2
x2 = – 2
y = – 1 implica x2 = – 1, es decir x3 = i
x4 = – i
Las soluciones pedidas son:
x1 = 2 ; x2 = – 2 ; x3 = i ; x4= – i

234 Ecuaciones e inecuaciones de segundo grado

234-235.(2003) 234 20/11/02, 11:44 AM


CAPÍTULO 4

Ejercicios
Resuelva las siguientes ecuaciones:

11. x4 + 10x2 + 9 = 0 12. x4 – 16 = 0 13. x4 – 7x2 + 10 = 0

14. x4 – 5x2 – 36 = 0 15. x4 – 7x2 + 12 = 0 16. x4 – 13x2 + 42 = 0

17. x4 – 14x2 + 33 = 0 18. x4 + 5x2 – 6 = 0 19. x4 – 4x2 + 4 = 0

10. x4 – 10x2 + 25 = 0 11. 4x4 – 5x2 + 1 = 0 12. 9x4 – 10x2 + 1 = 0

13. 2x4 – 9x2 + 4 = 0 14. 3x4 – 8x2 + 4 = 0 15. 8x4 – 6x2 + 1 = 0

Soluciones
1. x1 = i; x2 = – i; x3 = 3i; x4 = –3i 9. x1 = 2; x2 = 2; x3 = – 2; x4 = – 2
2. x1 = 2; x2 = – 2; x3 = 2i; x4 = –2i 10. x1 = 5; x2 = 5; x3 = – 5; x4 = – 5
1 1
3. x1 = 2; x2 = – 2; x3 = 5; x4 = – 5 11. x1 = 1; x2 = – 1;
; x4 = x3 = –
2 2
1 1
4. x1 = 3; x2 = – 3; x3 = 2i; x4 = –2i 12. x1 = 1; x2 = – 1; x3 = ; x4 = –
3 3
1 1
5. x1 = 2; x2 = – 2; x3 = 3; x4 = – 3 13. x1 = 2; x2 = – 2; x3 = ; x4 =–
2 2
6. x1 = 6; x2 = – 6 ; x3 = 7 ; x4 = – 7 2 2
14. x1 = 2; x2 = – 2; x3 = 3 ; x4 = – 3
7. x1 = 3; x2 = – 3; x3 = 11; x4 = – 11 1 1
15. x1 = ; x2 = – ; x3 = 1 ; x4 = – 1
8. x1 = 1; x2 = –1; x3 = i 6; x4 = –i 6 2 2 2 2

4.1.4 Relación entre los coeficientes de


una ecuación cuadrática y sus
raíces o soluciones y la
naturaleza de ellas

Sean x1 y x2 las soluciones de la ecuación:


ax2 + bx + c = 0

–b c
Se verifica: x1 + x2 = y x1 • x2 =
a a
Las soluciones de la ecuación ax2 + bx + c = 0 están dadas
por:
– b ± b2 – 4ac
x=
2a
Llamamos discriminante de la ecuación a la expresión denotada
por Δ y definida por:
Δ = b2 – 4ac.

Ecuaciones e inecuaciones de segundo grado 235

234-235.(2003) 235 20/11/02, 11:47 AM


El signo Δ determina la naturaleza de las soluciones de la
ecuación.
Se verifica:
Si Δ > 0, entonces las soluciones son números reales y distintos.
Si Δ = 0, entonces las soluciones son números reales e iguales.
Si Δ < 0, entonces las soluciones son números complejos.

Ejercicios 1. Determinemos la suma de las soluciones de la ecuación


3x2 – 9x – 16 = 0
resueltos
Notamos que no es necesario obtener las soluciones para
determinar su suma, pues podemos aplicar directamente la
propiedad
–b
x1 + x2 = a para este caso a = 3 y b = –9
9
Entonces tenemos x1 + x2 = =3
3

2. Determinemos el producto de las soluciones de la ecuación


2x2 + x – 15 = 0
Aquí también podemos aplicar directamente la propiedad
c
x1 • x2 = a para a = 2 y c = –15

y obtenemos: x1 • x2 = –15
2

3. ¿Qué valor(es) debe tomar k en la ecuación


9x2 – kx + 1 = 0
para que sus soluciones sean números reales e iguales?
La condición para que las raíces sean reales e iguales es que el
discriminante Δ sea igual a cero. En este ejemplo tenemos a = 9;
b = –k; c = 1
entonces: Δ = 0 ⇒ b2 – 4ac = 0
k2 – 36 = 0
k2 = 36
k=±6
y los valores que puede tomar k son +6 y –6

4. ¿Qué condición debe cumplir t en la ecuación


tx2 + 2x + 1 = 0
para que sus raíces sean números complejos conjugados?

236 Ecuaciones e inecuaciones de segundo grado

236-237. 236 8/11/01, 13:18


CAPÍTULO 4

Para que las raíces de una ecuación sean números complejos


conjugados se debe cumplir que el discriminante sea negativo.
Aquí a = t ; b = 2 y c = 1
entonces: Δ < 0 ⇒ b2 – 4ac < 0
4 – 4t < 0
4 < 4t
1<t
y por lo tanto la condición pedida es que t sea mayor que 1.

5. Determine una ecuación cuadrática sabiendo que sus raíces son:


x1 = 5 y x2 = – 6
Solución 1:
Aplicando las propiedades que relacionan los coeficientes de una
ecuación cuadrática con sus soluciones obtenemos:
–b –b
x1 + x2 = a ⇒ –1 = a
c c
x1 • x2 = a ⇒ – 30 = a
Podemos asignar a “a” cualquier valor; en particular, hagamos
a = 1 y entonces obtenemos b = 1 y c = – 30 y la ecuación
pedida es:
x2 + x – 30 = 0
Solución 2:
Si x1 y x2 son las raíces de la ecuación, entonces ésta se puede
factorizar por (x – x1) (x – x2) = 0
Aquí x1 = 5 y x2 = – 6,
entonces la ecuación factorizada es (x – 5) (x + 6) = 0
y la ecuación pedida es: x2 + x – 30 = 0
NOTA: Cualquier amplificación que hagamos a una ecuación cuadrática
nos dejará invariables las soluciones. Ésta es la razón que nos permitió
“elegir” a = 1 en la solución 1 del ejemplo anterior.

Ejercicios
1. ¿Cuál es la suma de las soluciones 3. ¿Cuál es la suma de las raíces de
de la ecuación: la ecuación:
3x2 – 5x – 2 = 0? 3x2 – 5x – 1 = 7(x – 3)?

2. ¿Cuál es el producto de las soluciones 4. ¿Cuál es el producto de las raíces de


de la ecuación: la ecuación:
3x2 + 5x + 2 = 0? (x – 5)2 = (x – 5) (x + 5)?

Ecuaciones e inecuaciones de segundo grado 237

236-237. 237 8/11/01, 13:18


Ejercicios
con coeficientes enteros e irreductibles
15. Determine la suma y el producto de cuyas raíces sean:
las raíces de la ecuación: 1
x1 = 3 y x2 =
2ax2 – bx + a2b2 = 0 2
17. Determine una ecuación cuadrática
16. Determine la suma y el producto de
con coeficientes enteros e irreductibles
las raíces de la ecuación:
cuyas soluciones sean:
(a – x)2 + (b – x)2 = 0
1
x1 = – y x2 = 2
17. Determine una ecuación cuadrática- 2
cuyas raíces sean: 18. Determine una ecuación cuadrática
x1 = – 2 y x2 = – 5 con coeficientes enteros e irreductibles
(Esta ecuación debe tener coeficientes cuyas raíces sean:
enteros e irreductibles). 3 2
x1 = y x2 =
5 5
18. Determine una ecuación cuadrática
cuyas raíces sean: 19. Determine una ecuación cuadrática
x1 = 0 y x2 = 1 con coeficientes enteros que tenga
como soluciones:
19. Determine una ecuación cuadrática 2 3
x1 = – y x2 =
cuyas raíces sean: 7 2
x1 = 0 y x2 = 0
20. Determine una ecuación de segundo
10. Determine una ecuación cuadrática grado con coeficientes enteros que
cuyas raíces sean: tenga como soluciones:
x1 = 2 y x2 = – 2 5 3
x1 = y x2 = –
11 4
11. Determine una ecuación cuadrática
cuyas raíces sean: 21. Sin resolver la ecuación
x1 = 3 y x2 = – 3 2x2 + 3x – 5 = 0
determine la naturaleza de sus
12. Determine una ecuación cuadrática soluciones.
cuyas raíces sean:
22. Sin resolver la ecuación
x1 = 5 y x2 = – 5
x2 + x + 1 = 0
13. Determine una ecuación cuadrática determine la naturaleza de sus raíces.
cuyas raíces sean: En los ejercicios 23 Q 30, determine
x1 = 2 y x2 = – 2 la naturaleza de las raíces sin resolver
las ecuaciones.
14. Determine una ecuación cuadrática
cuyas raíces sean: 23. 2 (x – 3)2 – 3 (x + 1)2 = 0
x1 = 6 y x2 = – 6 24. (x – 6) ( x + 5) – 2 (x – 7)2 = (x + 3)2
15. Determine una ecuación cuadrática 25. 3x2 – 5x – 2 = 3 (x – 3) + 2 (x – 1)
con coeficientes enteros e irreductibles
cuyas raíces sean: 26. (1 + x)2 = (1 – 2x)2
2 27. 6x2 + 7x + 4 = 0
x1 = 2 y x2 =
3
16. Determine una ecuación cuadrática 28. 2x (x + 4) – x (x – 1) = (x – 3) (2x – 1)

238 Ecuaciones e inecuaciones de segundo grado

238-239. 238 8/11/01, 13:29


CAPÍTULO 4

x + 5 2x – 3 x – 3
29. + =
x x x–2
35. 3x2 – x – 2k = 0
30. x – 2 x + 3
+ =1 Soluciones reales y distintas.
x+3 x–2
En los ejercicios 31 Q 40 determine qué 36. x2 + x + 3k = 0
valores debe tomar k o qué condiciones
Soluciones reales y distintas.
debe cumplir k para que las soluciones
sean como se requiere en cada caso. 37. 4x2 – 12x – k = 0
31. 2x2 + kx – 3 = 0 Soluciones reales e iguales.
Soluciones reales y distintas. 38. 3kx2 + 2x – 1 = 0
32. 3x2 – kx + 3 = 0 Soluciones complejas conjugadas.
Soluciones reales e iguales. 39. 3x2 – 2kx + 2 = 0
33. kx2 + kx – 2 = 0 Soluciones reales e iguales.
Soluciones reales e iguales. 40. 3kx2 – 2x + 5 = 0
34. 5x2 + 2x + k = 0 Soluciones reales e iguales.
Soluciones complejas conjugadas.

Soluciones

1. x1 + x2 = 5 14. x2 – 6 = 0 29. Δ > 0 Reales y distintas.


3
2 15. 3x2 – 8x + 4 = 0 30. Δ < 0 Complejas conju-
2. x1 • x2 =
3 16. 2x2 – 7x + 3 = 0 gadas.
3. x1 + x2 = 4 17. 2x2 – 3x – 2 = 0 31. k2 > – 24; cualquier k
4. Tiene 1 sola raíz. real.
18. 25x2 – 25x + 6 = 0
32. k = ± 6
5. x1 + x2 =
b 19. 14x2 – 17x – 6 = 0
2a 33. k = 0 o k = – 8;
20. 44x2 + 13x – 15 = 0
ab2
x1 • x2 =
2 21. Δ > 0 Reales y distintas. k2 + 8k > 0
6. x1 + x2 = a + b 22. Δ < 0 Complejas conju- 1
34. k >
a2 + b2 gadas. 5
x1 • x2 =
2 23. Δ > 0 Reales y distintas. 35. k > – 1
24
7. x2 + 7x + 10 = 0
24. Δ < 0 Complejas conju-
36. k < 1
8. x2 – x = 0 gadas. 12
9. x2 = 0 25. Δ < 0 Complejas conju- 37. k = – 9
gadas.
10. x2 – 4 = 0 38. k < – 1
26. Δ > 0 Reales y distintas. 3
11. x2 – 9 = 0
27. Δ < 0 Complejas conju- 39. – 6 < k < 6; k 2 < 6
12. x2 – 25 = 0 gadas.
1
13. x2 – 2 = 0 40. k =
28. Δ > 0 Reales y distintas. 15

Ecuaciones e inecuaciones de segundo grado 239

238-239. 239 8/11/01, 13:30


4.2 La función cuadrática
Corresponde a la expresión y = ax2 + b x + c, donde x es la
variable independiente; y es la variable dependiente; a,b, y c son
los coeficientes de la función.
La gráfica de la función cuadrática es una parábola y puede
tener una de las siguientes seis posiciones.

y y y

x x x

1 2 3
y y y

x x x

4 5 6
Es decir, se puede abrir hacia arriba (figuras 1-2-3) o hacia abajo
(figuras 4-5-6) y puede intersectar al eje x en 2 puntos (figuras 1 y 4);
en 1 punto (figuras 2 y 5) o en ningún punto (figuras 3 y 6).
La concavidad de la parábola o la posición en que se abre,
(hacia arriba o hacia abajo) está determinada por el signo del
coeficiente de x2 en la función y = ax2 + bx + c , es decir, está
determinada por el signo de "a". Así:
• si a > 0, entonces la concavidad es positiva y la parábola
se abre hacia arriba.
• si a < 0, entonces la concavidad es negativa y la parábola
se abre hacia abajo.
NOTA: “a” no puede tomar el valor 0 (cero) pues entonces la
función sería lineal y no cuadrática.
Las intersecciones de la gráfica con el eje X corresponden
a las soluciones de la ecuación cuadrática asociada; es decir a;

240 Ecuaciones e inecuaciones de segundo grado

240-241. 240 8/11/01, 13:31


CAPÍTULO 4

ax2 + bx + c = 0 (cuando y toma el valor cero la gráfica está


sobre el eje x).
Como sabemos, los tipos de soluciones de la ecuación dependen
del signo del discriminante Δ = b2 – 4ac.
Si Δ > 0, entonces las soluciones son reales y distintas y
por lo tanto hay dos intersecciones con el eje x; éstas son los
puntos x1 y x2.
Si Δ = 0, las soluciones son reales e iguales y hay una sola
intersección con el eje x. Aquí x1 = x2.
Si Δ < 0, las soluciones son complejas conjugadas y entonces
no hay intersección con el eje x.
La intersección de la parábola con el eje Y se obtiene haciendo
x = 0 y corresponde por supuesto a y = c.
Todas las parábolas tienen un vértice que corresponde al valor
mínimo (si la parábola se abre hacia arriba) o al valor máximo
(si se abre hacia abajo).
Las coordenadas del vértice son: x = –b
2a

b b2 – 4ac
V – ,–
2a 4a
b
La recta x = – es el eje de la parábola.
2a

El dominio de la función cuadrática es R


(no hay restricción).
El recorrido depende de la concavidad: x1
. .x
2

Si a > 0 entonces c
.
[( ) [
2 V
b – 4ac
Rec.(f) = – ,+•
4a
Si a < 0 entonces

Rec.(f) = ] 2
(
– • , – b – 4ac
4a
)]

1. Determinemos la concavidad y el número de intersecciones con


Ejercicios
el eje x de la gráfica de la función:
y = 2x2 + 3x – 1
resueltos
En esta función tenemos: a = 2, b = 3, c = –1.
Para la concavidad nos basta con analizar el signo de a.
a = 2; a > 0 implica concavidad positiva, la parábola se
abre hacia arriba.

Ecuaciones e inecuaciones de segundo grado 241

240-241. 241 8/11/01, 13:32


Ejercicios Para determinar las intersecciones con el eje X analizamos el signo
del discriminante Δ = b2 – 4ac
resueltos Δ=9+8
Δ = 17; Δ > 0, es decir, las soluciones son reales y distintas, por lo
tanto hay dos intersecciones con el eje X.

2. Determinemos la concavidad y el número de intersecciones de la


gráfiica de la función: (con el eje X)
y = – 3 x2 – x + 2
De inmediato; a = – 3; a < 0 implica concavidad negativa y la
parábola se abre hacia abajo.
Δ = b2 – 4ac
Δ = 1 + 24 = 25
Δ > 0 , hay dos intersecciones con el eje X.

3. Determinemos concavidad e intersecciones con el eje X en la


función
y = – x2 + 6x – 9
a = – 1 , concavidad negativa, por lo tanto la parábola se abre
hacia abajo.
Δ = 36 – 36
Δ = 0, es decir, hay un solo punto de intersección con el eje X.

4. Determinemos, en la función
y = x2 – 4x – 32
la concavidad, las intersecciones con ambos ejes, las coordenadas
del vértice, el dominio y el recorrido y esbocemos la gráfica.
Tenemos:
y = x2 – 4x – 32 a = 1 ; b = – 4 ; c = – 32
a) concavidad
a=1,a>0fi
b) intersecciones
con eje X : Δ = 144
Δ > 0 fi 2 intersecciones.
Solucionamos la ecuación para determinar los puntos de
intersección, que están dados por las soluciones x1 y x2
x2 – 4x – 32 = 0 fi (x – 8) (x + 4) = 0
fi x1 = 8 y x 2 = – 4
con eje Y:
hacemos x = 0 en la función y = x2 – 4x – 32 y obtenemos
y = – 32
c) Coordenadas del vértice.
b b2 – 4ac
V ,–
2a 4a
reemplazando obtenemos: V (2 , – 36)

242 Ecuaciones e inecuaciones de segundo grado

242-243.(2003) 242 20/11/02, 4:09 PM


CAPITULO 4
CAPÍTULO

d) Dom (f) = R
Rec (f) = [– 36, + • [ x2 = –4 2 x1 = 8

e) Gráfico

–32
–36 vértice

5. Dada la siguiente gráfica, determinemos


la función correspondiente.
Debemos determinar a, b y c. . 28
Tenemos x1 = – 7
x2 = 2
c = 28
(c es la intersección de la gráfica con el
eje y)
.
–7
.2
b
Sabemos que x1 + x2 = – a
c
y x1 • x2 = a
c 28
x1 x2 = 2 = a ⇒ – 14a = 28 ⇒

a :–7 • a=–2

b b
x1 + x2 = – ba : – 7 + 2 = – a ⇒ – 5 = ⇒ b = – 10
2
entonces la función pedida es: y = – 2x2 – 10x + 28

Ejercicios
1. Dados los siguientes gráficos, determine signo de ‘a’, (concavidad)
y tipos de soluciones de la ecuación asociada:
a) b) c) d)

Ecuaciones e inecuaciones de segundo grado 243

242-243.(2003) 243 20/11/02, 4:10 PM


Ejercicios
c) coordenadas del
En los ejercicios 2 → 10, 33.
vértice
determine la concavidad y
el número de interseccio- d) recorrido de la
nes con el eje X. función
5
2. y = x2 –1 e) gráfico

3. y = x2 + 1 19. y = x2 + 4x + 3
2 5
20. y = – x2 + 5x
4. y = – 2x2 – 3x + 1
21 y = x2 – 6x + 5 34.
5. y = 3x2 + x + 1
22. y = – x2 + 2x + 24
6. y = – 5x2 + 2x
23. y = – x2 + 6x + 16
7. y = – 3x2 +4 4
24. y = 3x2 – 5x – 2
8. y = 6x2 – 2x – 3
25. y = 4x2 – 9x + 2 –8 –2
9. y = x2 +x+1
26. y = – 4x2 + 9
10. y = – 5x2 27. y = 2x2 + 5x + 4 35.

Determine las coorde- 28. y = x2 + 5x


nadas del vértice de la 4
gráfica de las funciones 29. y = 6x2 – 13x – 5
dadas en los ejercicios 30. y = – 3x2 – 5x – 6
11 Q 18. –2 2
En los ejercicios 31 Q 42
11. y = x 2 – 5
determine la función corres-
12. y = x2 + 2x + 1 pondiente de acuerdo con
los datos dados:
13. y = 4x2 – 3x – 2
31. 36.
14. y = – 2x2 + x + 1

15. y = 3x2 – 3x + 2 6 .
–2
. .4 3
3x 1
16. y = – 3x2 + – 2
4 32
–8 .
17. y = – x2 +1
1 2

3x2
18. y = – 2x + 5
2
32. 37.
En los ejercicios 19 → 30 9
determine:

a) concavidad de la
parábola –3 3

–6
. .–2
b) intersección con el –1
eje X

244 Ecuaciones e inecuaciones de segundo grado

244-245. 244 8/11/01, 15:16


CAPÍTULO 4

38. 40. 42.


–7
. –1
.
–4 4

. –8 . –14

39. 41.

6 .
–1
. .4
–2 .
3

Soluciones
7. Concavidad negativa; 19. a) positiva
1. a) a < 0 ; 2 intersecciones. b) x1 = – 1; x2 = – 3
soluciones reales 8. Concavidad positiva; c) V( – 2, – 1 )
y distintas. 2 intersecciones. d) Rec: [ – 1, • )
b) a > 0 ;
soluciones 9. Concavidad positiva; 20. a) negativa
complejas 0 intersecciones.
b) x1 = 0 ; x2 = 5
conjugadas. 10. Concavidad negativa;
c) a > 0 ;
soluciones reales
1 intersección. c) V ( 5 25
,
2 4 )
11. V (0, – 5)
y distintas.
d) a < 0 ;
d) Rec: ( – ∞ ,
25
4
]
12. V (– 1, 0)
soluciones 21. a) positiva
reales e
iguales. 13. V ( ) 3
8
,–
41
16
b) x1 = 5
c) V ( 3, – 4 )
; x2 = 1

2. Concavidad positiva;
2 intersecciones. ( )
14. V 4 , 8
1 9 d) Rec: [ –4, • )

3. Concavidad positiva; 22. a) negativa


0 intersecciones. 15. V ( ) 1 5
,
2 4
b) x1 = 6
c) V (1, 25)
; x2 = – 4
4. Concavidad negativa;
2 intersecciones.
5. Concavidad positiva;
16. V
1
( 1)
,
8 64 d) Rec: (– •, 25]

0 intersecciones. 17. V (0, 1) 23. a) negativa


b) x1 = 8 ; x2 = –2
6. Concavidad negativa;
2 intersecciones.
18. V ( 2 13
,
3 3 ) c) V (3, 25)

Ecuaciones e inecuaciones de segundo grado 245

244-245. 245 8/11/01, 15:16


Soluciones

d) Rec: ( – ∞ , 25] 5 7
c) V – , 31. y = x2 – 2x – 8
4 8
24. a) positiva 7
d) Rec: [ , ∞) 32. y = – x2 + 9
8
b) x1 = 2 ; x2 = – 1 1 2 7
3 28. a) positiva 33. y = x – x+5
5 49 2 2
c) V , – 4 2 40 64
6 12 b) x1 = 0 ; x2 = – 5 34. y = – x – x–
– 49 9 9 9
d) Rec: [ , ∞) 5 25
12 c) V – , – 35. y = x2
2 4
25. a) positiva
– 25 36. y = x2 – 4x + 6
1
d) Rec: [ , ∞)
4
b) x1 = 2 ; x2 = 1 2
4 29. a) positiva 37. y = x + 2x + 3
4
c) V 9 – 49
, 5 2
8 16 b) x1 = ; x2 = – 1 38. y = – x – 4x – 8
2 3 2
d) Rec: [ – 49 , ∞ ) 1 2 3
16 c) V 13 289 39. y = x – x – 2
, – 2 2
26. a) negativa 12 24
3 3 d) Rec: [ – 289 , ∞ ) 40. (Faltan datos)
b) x1 = ; x2 = – 24
2 2
30. a) negativa
c) V (0, 9) 41. y = 2x2 – 4x + 6
b) no hay 3
d) Rec: ( – ∞ , 9 ]
5 47 42. y = –2x2 – 16x – 14
c) V – , –
27. a) positiva 6 12
b) no hay 47
d) Rec: ( – ∞ , – ]
12

4.3 Inecuaciones de segundo grado

Resolveremos aquí inecuaciones que pueden ser expresadas


en la forma:
ax2 + bx + c ≥ 0 o ax2 + bx + c ≤ 0
(por supuesto que las desigualdades también pueden ser estrictas,
es decir > ).
Usaremos la siguiente propiedad de los números reales:
a • b > 0 Pa > 0 A b > 0 o
a<0 A b<0
a • b < 0 Pa > 0 A b < 0 o
a<0 A b>0
es decir, un producto de dos factores es positivo si ambos tienen el
mismo signo y es negativo si ambos tienen distinto signo.
Entonces para resolver una inecuación cuadrática, la factorizamos
primero (esto es siempre posible determinando las raíces) y luego
aplicamos la propiedad señalada.

246 Ecuaciones e inecuaciones de segundo grado

246-247. 246 8/11/01, 15:23


CAPÍTULO 4

1. Resolvamos la inecuación: x2 – 5x + 6 > 0


Ejercicios
Factorizándola nos queda: (x – 2) (x – 3) > 0
resueltos
Aplicando la propiedad, tenemos las siguientes condiciones:
ii) x–2>0 A x–3>0 o
ii) x–2<0 A x–3<0
De i) obtenemos x–2>0⇒x>2
x–3>0⇒x>3
Como deben cumplirse simultáneamente, la solución S1 es la
intersección de ambas soluciones parciales, es decir Si = ] 3, ∞[
De ii) tenemos el siguiente sistema x – 2 < 0 y
x–3<0
con lo cual obtenemos las condiciones x < 2 A x < 3

la intersección de ambas es S2 = ] – ∞, 2 [
La solución final es la unión de S1 y S2 (puesto que i) e ii) son
situaciones independientes), es decir;
S = ] – ∞, 2 [ K ] 3, + ∞ [
En forma gráfica:

2 3

2. Resolvamos la inecuación x(2x + 4) – (x2 + 2x) – 35 £ 0


Factorizando tenemos: (x + 7) (x – 5) ≤ 0
Aplicando la propiedad tenemos dos sistemas, que son:

i) x + 7 £ 0 y ii) x + 7 ≥ 0
x– 5≥0 x–5£0
De i) obtenemos x £ – 7 A x ≥ 5 ,
lo cual es una contradicción pues no hay ningún número que
cumpla simultáneamente ambas condiciones.
De ii) obtenemos x ≥ – 7 y x ≤ 5
lo que nos da como solución el intervalo [– 7, 5]
la solución gráfica es:

–7 5

3. Resolvamos la inecuación 2x2 + 9x – 5 > 0

Las raíces de la ecuación 2x2 + 9x – 5 = 0 son

x1 = 1 y x2 = – 5,
2
entonces podemos escribir (la ecuación) en la forma:
1
x– x+5 =0
2

Ecuaciones e inecuaciones de segundo grado 247

246-247. 247 8/11/01, 15:24


Ejercicios Amplificándola por 2, nos queda la factorización correspondiente a
la inecuación original, es decir, estudiamos:
resueltos
(2x – 1) (x + 5) > 0

i) 2x – 1 > 0 Q x > 2
1 { 1
fi x > 2 (S1)
x+5>0 Qx>–5

ii) 2x – 1 < 0 Q x < 1 {


2
fi x < – 5 (S2)
x+5<0Qx<–5

S = S1 K S2, es decir
1
S = ] - •, – 5 [ K ] 2 , + • [
en forma gráfica:

–5 1
2

4. Resolvamos la ecuación 3x2 + 20x – 7 ≥ 0


Procedamos aquí de un modo diferente. Factorizando la expresión
nos queda (3x – 1) (x + 7) ≥ 0

Las raíces de la ecuación correspondiente son x1 = – 7 y x2 = 1


3
Ubicamos estos puntos en el eje real, obteniendo tres intervalos.

I II III

–7 1
3

Los signos que se obtienen al reemplazar la variable x de la inecuación


por un número real, van intercalados, es decir, cambian de un intervalo
al intervalo siguiente. La razón es obvia.
Por esto sólo basta reemplazar la variable x por un valor cualquiera;
esto nos determinará el signo del intervalo donde se encuentra ese valor
y por consiguiente, el signo de los otros intervalos.
Veamos qué pasa con x = 0, (x pertenece al segundo intervalo).
x = 0 Q 3x2 + 20x – 7 < 0
Entonces si x pertenece al segundo intervalo, la expresión es negativa
allí y por lo tanto es positiva en el primer y tercer intervalo.
1
(y es igual a cero en los puntos –7 y 3 )
+ – +

–7 1
3

Así, la solución para 3x2 + 20 x – 7 ≥ 0 es


1
S = ] – •, – 7] K [ 3 , + • [

248 Ecuaciones e inecuaciones de segundo grado

248-249.(2003) 248 20/11/02, 4:22 PM


CAPITULO 4

5. Resolvamos la inecuación 3x2 – 11 x – 4 < 0.


1
Las raíces de la ecuación asociada son x1 = – 3
y x2 = 4
por lo tanto la factorización correspondiente es
(3x + 1) (x – 4) < 0.
Ubicamos las raíces en la recta real (en este caso estos valores no
deben estar incluidos en la solución pues se trata de una desigualdad
estricta) y analizamos lo que pasa para cualquier valor de la variable,
por ejemplo para x = 0 :

I II III Nota:
+ – +
Si x = 0, que pertenece al segundo
1 x=0 4 intervalo, la inecuación queda:
3
3 • 02 – 11 • 0 – 4 < 0
(x = 0 pertenece al segundo intervalo) –4 < 0
1
Por lo tanto la solución pedida es: S = ] – 3 , 4 [ como esto es verdadero el intervalo
II es solución.

Ejercicios

Resuelva las siguientes inecuaciones:


1. x2 – 1 ≥ 0 12. 3 (2x2 + 1) > 11x
2. 8x2 + 5x ≥ 0 13. x (3x – 4) > 7
3. x (x – 3) – 2x (x – 2) + 3x < 0 14. 5x2 + 4x – 1 £ 0
4. 4x2 < 1 15. (x – 2)2 £ 2 (x2 + 2)
5. 3x2 – 5x < 0 16. x2 – 10x + 25 < 0
6. x (x – 5) – 2x (x + 3) + 6 £ x2 – 11x 17. 4x (x – 4) + 7 ≥ 0
7. x2 – 13x + 40 < 0
x+2 x
8. 2x2 + 3 £ 7x 18. 2x – 1 – x – 2 + 2 ≤ 0
9. 2x2 – 3x – 36 > x2 + 2x 2x x 5
19. – + ≥0
10. 3x2 + 16x – 12 < 0 x + 12 x + 3 x + 12 x + 3
x+1 x+2 x+3
11. 4x (x + 3) ≥ – 5 20. + ≤
x – 1 2x + 1 x – 1

Soluciones

–1 1
1. S = ]– •, –1] K [1, + •[

5 0

] 5
]
2. S = – •, – 8 K [0, + •[ 8

Ecuaciones e inecuaciones de segundo grado 249

248-249.(2003) 249 20/11/02, 4:22 PM


Soluciones

3. S = ] – •, 0 [K] 4, + • [ 0 4

4. S = ] –1 1
,
2 2 [ 1
2
1
2

5. S = 0,] 5
3
[ 0 5
3

6. S = ]– •, – 3] K [ 3, + •[
– 3 3

7. S = ] 5, 8 [ 5 8

8. S = [ 12 , 3] 1
2
3

9. S = ]– •, – 4 [ K ] 9, + •[ –4 9

10. S = – 6, ] 2
3
[ –6 2
3

]
11. S = – •, – 2
5
]K[ –1
2
,+ • [ 5
2
1
2

]
12. S = – •, 3
1
[ K ] 32 , + •[ 1
3
3
2

13. S = ] – •, – 1[ K ] 73 , + •[ –1 7
3

14. S = – 1, [ 1
5
] –1 1
5

15. S = ] – •, – 4] K [0, + •[ –4 0

16. S = Δ

]
17. S = – •, 1
2
]K [ 7
2
,+• [ 1
2
7
2

1
18. S = [ 0, [ K ] 2, 3 ] 0 1 2 3
2 2

19. S = ] – •, – 12 [ K ] – 3, 1 ] K [ 5, + • [
– 12 –3 1 5

1
20. S = [ –1, – [ K ] 1, 4 ] –1
2 –1 1 4
2

250 Ecuaciones e inecuaciones de segundo grado

250-251.(2003) 250 20/11/02, 12:00 PM


CAPÍTULO 4

Sistemas de ecuaciones de 4.4


segundo grado
Un sistema de ecuaciones en dos variables es de segundo grado si
alguna de las ecuaciones contiene alguno de los términos x2, y2 o xy
(suponiendo que las variables son x e y por supuesto).
No hay métodos generales que puedan ser aplicados en forma
práctica a todos los sistemas.
Veremos aquí algunos tipos de ellos.

4.4.1 Sistemas que contienen


una ecuación lineal y una
ecuación cuadrática
Para resolverlo, despejamos una de las variables de la ecuación
lineal y la sustituimos en la ecuación cuadrática.

1. Resolvamos: 2x + y = 10 Ejercicios
2 x2 – y2 = 12 resueltos
Despejemos la variable “y” de la primera ecuación:
y = 10 – 2x
Reemplacemos en la segunda ecuación la variable “y” despejada.
Obtenemos:
x2 – (10 – 2x)2 = 12
x2 – (100 – 40x + 4x2) = 12
Ordenando los términos tenemos la siguiente ecuación cuadrática:
3x2 – 40x + 112 = 0
28
cuyas soluciones son: x1 = 4 y x2 =
3
x1 = 4 Q y1 = 2
Si 28 26
x2 = Q y2 = – 3
3
La solución de la ecuación es el conjunto

S= { (4, 2), ( 283 , – 263 )}.


Notemos que la solución de este tipo de sistemas puede estar
formada por 2 puntos, 1 punto o ninguno (geométricamente representa
la intersección de una línea recta con una cónica, o bien, la intersección
de dos cónicas).

Ecuaciones e inecuaciones de segundo grado 251

250-251.(2003) 251 20/11/02, 12:01 PM


Ejercicios 2. Resolvamos el sistema: x + y =6
resueltos x2 + y2 = 16

Despejamos la variable x (o la variable “y”) de la primera ecuación


y obtenemos:
x=6–y
y la reemplazamos en la segunda ecuación:
(6 – y)2 + y2 = 16
2y2 – 12y + 20 = 0 o y2 – 6y + 10 = 0
las soluciones algebraicas de esta ecuación son los puntos
y1 = 3 + i y y2 = 3 – i
y por lo tanto
x1 = 3 – i ; y1 = 3 + i
x2 = 3 + i ; y2 = 3 – i
geométricamente el sistema no tiene solución.
3. Resolvamos el sistema: x – y = – 7
xy = – 10
Despejando la variable “y” de la primera ecuación:
y=x–7
y reemplazándola en la segunda: x (x – 7) = – 10
x2 – 7x + 10 = – 0
las soluciones de la ecuación son: x1 = 2 y x2 = 5.
Si x = 2 entonces y=– 5
x = 5 entonces y=– 2
y la solución del sistema es: S = {(2, – 5), (5, – 2)}

Ejercicios

Resuelva los siguientes sistemas:

1. x – y = 2 4. 2x + y2 = – 1 7. 3x – 2y = 6
x2 + y2 = 20 2 x – y2 =–8 2x2 – y2 = 23

2. 2x + y = 4 5. 2x – y2 = 29– 8. 2x + 4y = – 18
x2 + y2 = 5 2xy = – 40 4xy = – 40

3. x – 2y = 7 6. 2x + y = – 6 9. 5x – y = 8
x2 – y = 26 x2 + 2y = – 0 – 2xy = 6

252 Ecuaciones e inecuaciones de segundo grado

252-253. 252 8/11/01, 15:31


CAPÍTULO 4

10. x + 5y = – 1 14. 2x + y = 11
x2 + 3xy = – 27 x2 – xy = 4 18. 2x + 3y = 5
61
11. 3x +9y2 = – 2 15. 3x + y2 – 5 = 0 x2– xy + y2= 36
9x2 – 9y2 = – 8 x2 + y2 – xy = 3
16
19. x – 2y = –
12. –x – 2y = 0 16. 2x + y2 =– 8 15
2xy = – 1 2 x2 – y2 =– 5 x2 + 3xy = – 6
25
17. 5x +9y 2= 17
13. 3x + 4y = 0 20. x + y = – 12
x2 – y2
1 5 =1 x + y – xy = – 8
– 4xy = 5
3

Soluciones

1. (4, 2) ; (–2, –4) 2. (1, 2) ; ( 11 – 2


5
,
5 ) 3. (5, –1) ; ( – 9 – 23
2
,
4 )
(
4. (1, –3) ; – 7 ,
4 2
5
) 5. (5, –4) ; (4, –5) 6. (–2, –2) ; (6, –18)
7. (4, 3) ; (32, 45) –1 3
8. (–2, 5) ; (20, ) 9. (1, –3) ; ( 5 , –5)
2
10. (9, –2) ; ( – 15 13
,
2 10 ) 1
( ) ( )
11. , 1 ; ,
3
7 –3
6 2 ( ) (1
12. 1, 2 ; – 1,
–1
2 )
(
1 –1
13. 3, 4 ;
–1 1
,
3 4 ( ) ( )
14. (4, 3) ;
– 1 35
,
3 3 (
15. (1, 2) ;
22 – 1
,
3 13 )
16. (–3, –2) ; ( – 23 22
,
3 3 ) ( , )
17. (3, 2) ;
49 – 41
12 12 18. (, ) (
3 2
2 3
; 13 , 82
38 57 )
(
19. 5,
2 –1
) (
6 – 31
3 ; 25
,
75 ) 20. (2, 10) ; (10, 2)

4.4.2 Sistemas en que ambas ecuaciones


son de la forma ax2 + by2 = c
(No hay términos de primer grado, ni el término xy).
Lo más práctico en estos casos es proceder por reducción de variables y como
sabemos, esto se logra con una adecuada amplificación de las ecuaciones:

1. Resolvamos el sistema: x2 + y2 = 41
Ejercicios
x2 – y2 = 9
resueltos
Podemos reducir la variable “y” en forma inmediata sumando
ambas ecuaciones.

Ecuaciones e inecuaciones de segundo grado 253

252-253. 253 8/11/01, 15:33


Ejercicios Nos queda: 2x2 = 50
x2 = 25
resueltos
y las soluciones para la variable x son: x1 = 5 y x2 = –5
Si x1 = 5 entonces, reemplazando en la primera ecuación,
obtenemos: y2 = 41 – 25
y2 = 16
y = ±4
Lo mismo ocurre si x = – 5.
La solución del sistema, entonces, consiste en 4 puntos que son:
5 = {(5, 4), (5, – 4), (– 5, 4), (– 5, – 4)}

2. Resolvamos el sistema: 2x2 – y2 = 1


3x2 + 2y2 = 5

Aquí para eliminar la variable y2 podemos amplificar la primera


ecuación por 2, y luego sumamos ambas ecuaciones:

+
{ 4x2 – 2y2 = 2
3x2 + 2y2 = 5

7x2 = 7

Y las soluciones para la variable x son: x1 = 1 y x2 = –1


Si x = 1 entonces, y ± 1
Si x = – 1 entonces, y ± 1

y la solución del sistema está dada por:


S = {(1,1), (1, – 1), (– 1, 1), (– 1, –1)}

3. Resolvamos el sistema: 3x2 – 2y2 = 3


x2 – 3y2 = –13

Amplificando la segunda ecuación por (– 3) y sumando ambas


ecuaciones obtenemos:

+
{ 3x2 – 2y2 = 3
–3x2 + 9y2 = 39

7y2 = 42
y2 = 6
Las soluciones para la variable y son: y1 = 6 ; y2 = – 6
Y sustituyendo estos valores en cualquier ecuación del sistema
obtenemos para x los valores x1 = 5 x2 = – 5
La solución del sistema es, entonces:
5 = {( 5, 6), ( 5, – 6), (– 5, 6), (– 5, – 6)}

254 Ecuaciones e inecuaciones de segundo grado

254-255. 254 8/11/01, 15:36


CAPÍTULO 4

Ejercicios
Resolver los siguientes sistemas:

1. x2 + y2 = 5 6. –5x2 + 3y2 = 172 11. 6x2 – 5y2 = 3x2 – 2y2


x 2 – y2 = 3 x2 – y2 = – 60 x2 = 2y2

12. x2 + y2 = 7
2. x2 + 2y2
= 72 7. 2x2 – 3y2
= 194 4x2 – 7y2 = 6
x2 – y2 = 60 3x2 + y2 = 379

13. 6x2 – 2y2 = x2 + y2 – 4


3x2 – y2 = 4
3. 2x2 + y2 = 22 8. 4x2 – 5y2 = – 8
x2 – 2y2 = 1 x2 + 3y2 = 49
x2 + y2
14. = 17
2
4. 3x2 + y2 = 124 9. 2x2 – y2 = 1 x2 – y2
2x2 + 3y2 = 120 2x2 + y2 = 7 = 4
4

5. –5x2 + y2 = – 20 10. 2x2 + 3y2 = 27 3x2 + 2y2 7


15. 4
=
– 3y2 = – 75 x2 – y2 = 1 4
x2 + y2 =3

Soluciones
1. S = {(2, 1), (2, – 1), (– 2, 1), (– 2, – 1)}
2. S = {(8, 2), (8, – 2), (– 8, 2), (– 8, –2)}
3. S = {(3, 2), (3, – 2), (– 3, 2), (– 3, – 2)}
4. S = {(6, 4), (6, – 4), (– 6, 4), (– 6, – 4)}
5. S = {(3, 5), (– 3, – 5), (3, – 5), (– 3, 5)}
6. S = {(2, 8), (2, – 8), (– 2, 8), (– 2, – 8)}
7. S = {(11, 4), (11, – 4), (– 11, 4), (– 11, – 4)}
8. S = {( 13 , 12 ), ( 13 , – 12 ), (– 13 , 12 ), (– 13 , – 12 )}

9. S = {( 2, 3), ( 2, – 3), (– 2, 3), (– 2, – 3)}


10. S = {( 6, 5), ( 6, – 5), (– 6, 5), (– 6, – 5)}

11. S = {(0, 0)}


12. S = {( 5, 2), ( 5, – 2), (– 5, 2), (– 5, – 2)}
13. S = {(2, 8), (2, – 8), (– 2, 8), ( – 2, – 8)}

14. S = {(5, 3), (5, – 3), (– 5, 3), (– 5, – 3)}


15. S = {(1, 2), (1, – 2), (– 1, 2), (– 1, – 2)}

Ecuaciones e inecuaciones de segundo grado 255

254-255. 255 8/11/01, 15:39


4.4.3 Sistemas formados por una ecuación de
– x2 + y2 = a y la otra ecuación
la forma
de la forma x y = b.

Una manera práctica de resolver estos sistemas es completando


cuadrados de binomio y reduciendo la solución del sistema de 2º
grado a sistemas lineales (o de 1er grado).

Ejercicios 1. Resolvamos el sistema: x2 + y2 = 34

resueltos y=

Amplificando la segunda ecuación por 2, y sumando las


ecuaciones, y luego restando, obtenemos:

+
{ x2 + y2 = 34
2xy = 30

{ x2 + y2 = 34
2xy = 30

x2 + 2 xy + y2 = 64 x2 – 2xy + y2 = 4
fi (x + y)2 = 64 fi (x – y)2 = 4
x+y = ±8 x–y =±2
y nos quedan cuatro sistemas, de solución casi inmediata,
que son:
x+y=8 x+y=–8 x+y=–8 x+y=–8
x–y=2 x–y=–2 x–y=2 x–y=–2

y las soluciones son respectivamente:

(5, 3); (3, 5); (– 3, – 5); (– 5, – 3)

La solución del sistema es la unión de todas ellas.


S = {(5, 3); (3, 5); (– 3, – 5); (– 5, – 3)}

2. Resolvamos el sistema: x2 + y2 = 15
xy =

Repitiendo el procedimiento anterior tenemos:

+
{ x2 + y2 = 15
2 xy = 12

{ x2 + y2 = 15
2 xy = 12

x2 + 2 xy + y2 = 27 x2 – 2xy + y2 = 3
(x + y)2 = 27 (x – y)2 = 3
x+y= ± 27 = ± 3 3 x–y = ± 3

256 Ecuaciones e inecuaciones de segundo grado

256-257.(2003) 256 20/11/02, 12:04 PM


CAPÍTULO 4

Los sistemas lineales asociados al sistema original son:


x+y=3 3 x+y =3 3 x+y =–3 3 x+y= –3 3
x–y= 3 x–y =– 3 x–y = 3 x–y = – 3

y las soluciones son, respectivamente:

x1 = 2 3 x2 = 3 x3 = – 3 x4 = – 2 3

y1 = 3 y2 = 2 3 y3 = – 2 3 y4 = – 3

Y la solución del sistema es:

S = {(2 3 , 3); ( 3, 2 3 ); (– 3, – 2 3 ); (– 2 3 , – 3)}

Observación: Las soluciones de estos sistemas están en el conjunto de


los números reales, es decir, no se aceptan como soluciones raíces de
números negativos.

3. Resolvamos el sistema: x2 + y2 = – 4
y =

Si procedemos en forma análoga a los ejemplos anteriores, obtenemos:

+
{ x2 + y2 = – 4
2 xy = 4

{ x 2 + y2 = – 4
2 xy = 4

x2 + 2 xy + y2 = 0 x2 – 2xy + y2= – 8
(x + y)2 = 0 (x – y)2= – 8

Y los sistemas asociados serían:

x+y = 0 x+y = 0
x –y = –8 x – y = – –8

Y sus soluciones estarían dadas por:


– –8
x1 = – 8 x2 =
2 2
– –8 –8
y1 = y2 =
2 2
las cuales no son números reales. Decimos entonces que el sistema
no tiene solución en R.

Nota: Una simple inspección en el sistema original nos habría determinado


de inmediato la no existencia de solución real del sistema, pues una
propiedad elemental de los números reales es:

x2 ≥ 0 I x E R.

¿Qué conclusión se obtiene?

Ecuaciones e inecuaciones de segundo grado 257

256-257.(2003) 257 20/11/02, 12:06 PM


Ejercicios

Resolver los siguientes sistemas:

1. x2 + y2 = 26 8. x2 + y2 = 97 14. x2 + y2 = – 2
xy = 5 xy xy = 6
= 6
6

2. x2 + y2 = 20 15. x2 + y2 = 0
xy = 6 9. 5 x2 +5 y2 = 185 xy = 2
– xy
= –2
3
3. x2 + y2 = 30 16. 3 x2 + y2 = 11
2xy = 18 xy = 0
10. – x2 – y2 = – 61
xy
= 6
4. x2 + y2 = 85 5 17. x2 + y2 = 153
3xy = 54 – xy
11. x2 + y2 = – 6 = –9
4
xy
5. 2 x2 + 2 y2 = 50 = –8
3 18. x2 + y2 = – 1
xy = 12
xy = – 1
x2 + y2
12. = 15
6. x2 + y2 = 40 3
xy = 18
5 xy = 60

7. x2 + y2 = 58 13. – 2x2 – 2 y2= – 100


– 2 xy = – 42 xy = 25

Soluciones
1. S = {(5, 1); (1, 5); (– 5, – 1); (– 1, – 5)}
2. S = {( 18 , 2 ); (– 18 , – 2); ( 2, 18 ); (– 2,– 18 )}
3. S = {( 3, 27 ); (– 3, – 27 ); ( 27 , 3); (– 27 ,– 3)}
4. S = {(9, 2); (– 9, – 2); (2, 9); (– 2, – 9)}
5. S = {(3, 4); (– 3, – 4); (4, 3); (– 4, – 3)}
6. S = {(6, 2); (– 6, – 2); (2, 6); (– 2, – 6)}
7. S = {(7, 3); (– 7, – 3); (3, 7); (– 3, – 7)}
8. S = {(9, 4); (– 9, – 4); (4, 9); (– 4, – 9)}
9. S = {(1, 6); (– 1, – 6); (6, 1); (– 6, – 1)}
10. S = {(6, 5); (5, 6); (– 6, – 5); (– 5, – 6)}
11. S = Δ 12. S = {(3, 6); (6, 3); (– 3, – 6); (– 6, – 3)} 13. S = {(5, 5); (– 5, – 5)}
11 11
14. S = Δ 15. S = Δ 16. S = {(0, 11 ); (0, – 11 ); ( – , 0)}
3 , 0); ( 3
17. S = {(12, 3); (– 12, – 3); (3, 12); (– 3, – 12)} 18. S = Δ

258 Ecuaciones e inecuaciones de segundo grado

258-259. 258 8/11/01, 15:50


CAPÍTULO 4

4.4.4 Sistemas homogéneos formados por


ecuaciones cuyos términos son todos
de segundo grado

(Es decir, contienen términos x2, y2, xy)


Para resolver estos sistemas usamos el cambio de variable y = λ x, y así
reducimos el problema de resolver un sistema de segundo grado (con 2
variables) en una ecuación de segundo grado (con una variable).

1. Resolvamos el sistema: x2 + y2 = 50 Ejercicios


x2 + xy = 56
resueltos
Hagamos el cambio de variable indicado: y = λ x,
y sustituyamos en ambas ecuaciones.
x2 + λ2 x2 = 50
x2 + λ x2 = 56
Factorizando ambas ecuaciones por x2 y luego dividiéndolas
obtenemos:

:
{
x2 (1+ λ2 ) = 50
x2 (1 + λ ) = 56
1 + λ2 50
=
1+ λ 56
lo cual da origen a la siguiente ecuación cuadrática en la
variable λ:
56 + 56λ2 = 50 + 50 λ
56 λ – 50 λ + 6 =
2 0
o 28 λ2 – 25 λ + 3 = 0
resolviendo la ecuación, obtenemos para λ las siguientes
soluciones:
3 1
λ1 = y λ2 =
4 7
3
Para λ =
1 tenemos y = 3 x
4 4
sustituyendo en la segunda ecuación (o en la primera),
3
obtenemos: x2 + x • x = 56
4
4 x2 + 3 x2 = 224
7 x2 = 224
x2 = 32
x =± 32 = ± 4 2
Si x1 = 4 2 Q y1 = 3 2 (y = 3 x)
4
Si x2 = – 4 2 Q y2 = – 3 2

Ecuaciones e inecuaciones de segundo grado 259

258-259. 259 8/11/01, 15:51


1 1
Ejercicios Para l2 =
7
tenemos y = x.
7
resueltos Sustituyendo en la 2a ecuación, obtenemos:
1
x2 + x2 = 56
7
7x2 + x2 = 392
x2 = 49
x = ±7
1
Si x3 = 7 y3 = 1 (y = x)
7
x4 = – 7 y4 = – 1
La solución del sistema es, entonces:
S = {(4 2, 3 2) , (– 4 2, – 3 2), (7, 1), (– 7, – 1)}

2. Apliquemos el mismo procedimiento para resolver:

2x2 + y2 = 33
y2 – xy = 15

Primero hacemos la sustitución y = l x y reemplazamos:

2x2 + l2 x2 = 33
l2x2 – lx2 = 15

Factorizamos por x2 y dividimos ambas ecuaciones:

x2 (2 + l2) = 33
x2 (l2 – l) = 15
2 + λ2
= 33
λ2 – λ 15
Formamos la ecuación cuadrática en la variable l:
30 + 15 l2 = 33 l2 – 33l
18 l2 – 33 l – 30 = 0
6 l2 – 11 l – 10 = 0
2
las soluciones son l1 = 5 y l2 = –
2 3
Para l1 = 5 tenemos y= 5x
2 2
sustituyendo en la segunda ecuación:
25 2 5 2
x – x = 15
4 2
25x2 – 10 x2 = 60
15 x2 = 60
x2 = 4 x1 = 2
x2 = – 2
x1 = 2 y1 = 5
x2 = –2 y2 = – 5

260 Ecuaciones e inecuaciones de segundo grado

260-261. 260 20/11/02, 12:13 PM


CAPÍTULO 4

2 2
Para l2 = – 3 tenemos: y=– 3x

4 2 2 2
sustituyendo: 9
x + 3x = 15
4x2 + 6x2 = 135
10x2 = 135
3
135 27 x3 = 3 2
x2 = = 2 3
10
x4 = – 3 2

3 3
x3 = 3 2 y3 = – 2 2
3 3
x4 = – 3 2 y4 = 2 2

y la solución está dada por el conjunto:

{
S = (2, 5), (– 2, – 5), 3 ( 3
2,–2
3
)(
2 , –3
3
2,2
3
2 )}

Ejercicios
Resolver los siguientes sistemas: 5. 2x2 + 3y2 = – 4
x2 – xy = 12
1. x2 – y2 = –7
xy = 12
6. x2 + xy + y2 = 26
x2 – y2
2. x2 + y2 = 41 = 4
4
x2 – xy = – 4

7. 3x2 – 2y2 = – 24
3. 2x2 – y2 = – 34
2xy + y2
xy = 6 = 14
6

4. x2 – 3y2 = 13 8. x2 – 2xy – y2 = 41
xy = 10 x2 + 3xy – y2 = 131

Soluciones

1. S = {(3, 4), (– 3, – 4)} 2. S = {(4, 5), (– 4, – 5) ( 0,5, 40,5); (– 0,5, – 40,5)}

3. S = {(1, 6), (– 1, – 6)} 4. S = {(5, 2), (– 5, – 2)}

5. S = Δ 6. S = {( 18 , 2), (– 18 , – 2)}

7. S = {(4, 6), ( – 4, – 6)} 8. S = {(9, 2), (– 9, – 2)}

Ecuaciones e inecuaciones de segundo grado 261

260-261. 261 20/11/02, 12:15 PM


4.4.5 Otros sistemas y problemas
Resolveremos aquí algunos problemas cuyo planteamiento
corresponde a un sistema de ecuaciones de segundo grado.
Hay oprtunidades en que las ecuaciones de 2º grado son la
expresión de dos rectas que se intersectan y por lo tanto la solución
del sistema puede ser única si el punto en que se intersectan las
cuatro rectas coincide; o vacío en el caso en que la cuatro rectas no
concurran en el mismo punto. (ver ejercicios 4 y 5)

1. La suma de dos números es 11 y la suma de sus cuadrados es 65.


Ejercicios Determine dichos números.
resueltos Sean x e y los números pedidos. Tenemos entonces:
x + y = 11
x2 + y2 = 65
Resolviendo el sistema por sustitución, nos queda: y = 11 – x.
x2 + (11 – x)2 = 65
x + 121 – 22x + x2
2
= 65
2x2 – 22x + 56 = 0
x2 – 11x + 28 = 0
las soluciones para x son; x1 = 7 y x2 = 4
Si x1 = 7 Q y1 = 4
x2 = 4 Q y2 = 7
y los números pedidos son x = 7, y=4
(por la naturaleza del problema, no es necesario en este caso tomar
en cuenta la segunda solución).
2. El perímetro de un rectángulo es 20 cm y su área mide 24 cm2.
Determine sus dimensiones.
Llamemos x al largo del rectángulo e y al ancho. Tenemos el
sistema:
2x + 2y = 20 cm (Perímetro)
2
xy = 24 cm (Área)
Despejamos “y” de la primera ecuación: y = (10 – x) cm
y lo reemplazamos en la segunda:
x (10 – x) = 24
10x – x2 = 24 o x2 – 10x + 24 = 0
las soluciones para x son; x1 = 6 y x2 = 4
Si x = 6 Q y = 4
x=4 Q y=6
Por lo tanto las dimensiones del rectángulo son 6 cm de largo
y 4 cm de ancho.
7
3. La suma de dos números es 10 y la suma de sus recíprocos es 7.
Determine dichos números.

262 Ecuaciones e inecuaciones de segundo grado

262-263. 262 10/11/2001, 14:28


CAPÍTULO 4

Sean x e y los números. Entonces:


x + y = 7
10
1 1
x + y = 7

Procediendo algebraicamente tenemos:


7
x+y= 7 Q x+y= Q
10 10
x+y
xy = 7 x+y=7xy

7 7
x+y= Q y= –x
10 10
1
xy =
10

7 1 7 1
x 10 – x = Q x – x2 = Q 7x –10x2 = 1
10 10 10

10x2 – 7x + 1 = 0 x1 = 1
2
x2 = 1
5

1 7 1 1
Si x1 = entonces y1 = – = 5
2 10 2
7 1 1
x2 = 1 entonces y2 = – =
5 10 5 2
1
y los números pedidos son; x = 1 e y = 5
2

4. Resolvamos el sistema:

x2 + 4y2 = 0 (1)

9x2 + 16y2 = 0 (2)

Algebraicamente vemos que en (1) x2 = 4y2 y reemplazando en (2):


9 • 4y2 – 16y2 = 0
32y2 – 16y2 = 0
16y2 = 0
y2 = 0
y =0

En (1) x2 – 4 • 0 =0
x2 = 0
x =0

Por lo tanto, la única solución es (0, 0).

Ecuaciones e inecuaciones de segundo grado 263

262-263. 263 10/11/2001, 14:29


Analíticamente vemos que:
Ejercicios (1): x2 – 4y2 = 0 =
0
– 4y
resueltos (x + 2y) (x – 2y) = 0 3x y = 0
3 2
representa las rectas x–
x + 2y = 0 y x – 2y = 0 1
–4 –2 2 4
x+
(2): 9x2 – 16y2 = 0 3x 2y = 0
(3x + 4y) (3x – 2y) = 0 +4
y=
representa las rectas 0
3x + 4y = 0 y 3x – 4y = 0

Las cuatro rectas se intersectan en el punto (0, 0)

5. Resolvamos el sistema:

4x2 – y2 + 6y – 9 = 0 (1)
x2 – 4y2 + 24y – 36 = 0 (2)

Observamos que para eliminar “y” debemos amplificar la primera


ecuación por –4 y sumamos:

– 16x2 + 4y2 – 24y + 36 = 0 (1)


x2 – 4y2 + 24y – 36 = 0 (2)

–15x2 = 0
x2 = 0
x=0

Sustituyendo x por su valor 0, en cualquiera de las ecuaciones del


sistema obtenemos que y = 3
Luego, la única solución del sistema es (0, 3)

Otra forma: Analíticamente vemos que si


despejamos “y“ en la primera ecuación:

y2 – 6y + 9 – 4x2 = 0

6+ 36 – 4(9 – 4x2) 6+ 36 – 36 + 16x2)


y= =
2 2
+ y1 = 3 + 2x
= 6 4x
2 y2 = 3 – 2x
ecuaciones que corresponden a dos rectas que se intersectan
en el punto (0, 3).

En forma análoga, podemos ver que la segunda ecuación,


x2 – 4y2 + 24y – 36 = 0 corresponde a las rectas
y = 1 x + 3 y y = – 1 x + 3 , las cuales también se
2 2
intersectan en el punto (0, 3).

264 Ecuaciones e inecuaciones de segundo grado

264-265. 264 8/11/01, 16:07


CAPÍTULO 4

Ejercicios

1. La suma de dos números es 12 y la diferencia de sus


cuadrados es 24. Determine dichos números.

2. La diferencia de dos números es –5 y la suma de sus


cuadrados es 97. Determine dichos números.

3. La diferencia de dos números es 5 y la diferencia de sus


cuadrados es 55. Determínelos.

4. La suma de dos números es 25 y la diferencia de sus


cuadrados es 25. Determínelos.

5. El cuadrado de la suma de dos números es 100 y el producto


de ellos es 24. ¿Cuáles son?

6. El cuadrado de la suma de dos números es 225 y su cociente


es 4. ¿Cuáles son esos números?

7. El cuadrado de la diferencia de dos números es 25 y el


producto de ellos es 36. ¿Cuáles son?

8. Dos números están en la razón 1:3 y la diferencia de sus


cuadrados es – 200. ¿Cuáles son?

9. Dos números están en la razón 2:3 y el triple del cuadrado


del primero menos el cuadrado del segundo es 27. ¿Cuáles
son los números?

10. La suma del cuadrado de un número más el quíntuple del


cuadrado de otro es 49, y la diferencia entre el triple del
cuadrado del primero y el cuadrado del segundo es 3.
¿Cuáles son los números?

11. El doble del cuadrado de un número menos el triple del


cuadrado de otro es 23 y el producto de ambos es 15.
Determine dichos números.

12. Determine las dimensiones de un rectángulo sabiendo que


su perímetro es 40 cm y su área mide 91 cm2.

13. Determine las dimensiones de un rectángulo sabiendo que el


largo es 4 veces el ancho y que el área mide 25 cm2.

14. ¿Cuáles son las dimensiones de un rectángulo cuyos lados


están en la razón 1:3 y cuya superficie mide 6,75 cm2?

Ecuaciones e inecuaciones de segundo grado 265

264-265. 265 8/11/01, 16:07


Ejercicios
15. El área de un triángulo rectángulo mide 10 cm2 y la hipotenusa mide 41 cm.
Determine la medida de los catetos.
16. La suma de las áreas de dos cuadrados es 74 cm2 y la diferencia de sus perímetros
es 8 cm. Determine el lado de cada uno.
17. Las áreas de dos círculos están en la razón 1:4 y sus radios suman 9 cm.
Determine los radios.
18. Los catetos de un triángulo rectángulo están en la razón 5:3 y su superficie mide
120 cm2. Determine la medida de sus 3 lados.
19. Resolver los siguientes sistemas:
a) 9x2 – y2 + 2y – 4 = 0
4x2 – 4y2 + 16y + 16 = 0

b) x2 – 4y2 + 2x + 16y – 15 = 0
2x2 – y2 + 4x + 4y – 2 = 0

c) (x + 1)2 – 2y – 2 = 8
2 6 9
2x2 + 4x + 2 – y – 1 = 2
3 5

20. Una piscina rectangular cuyas dimensiones son 5 por 10 metros tiene un borde de
ancho uniforme. Si el área del borde es 16 m2, calcule el ancho del borde.

Soluciones

1. x = 7 , y = 5 2. (x1 = 4 , y1 = 9); (x2 = – 9 , y2 = – 4)


3. x = 8 , y = 3 4. x = 13 , y = 12
5. (x1 = 4 , y1 = 6); (x2 = – 4 , y2 = – 6) 6. (x =12 , y = 3); (x2 = – 12; y2 = – 3)
7. (x1 = 4 , y1 = 9); (x2 = – 4 , y2 = – 9) 8. (x1 = 5 , y1 = 15); (x2 = – 5 , y2 = – 15)
9. (x1 = 6 , y1 = 9); (x2 = – 6 , y2 = – 9) 10. (2, 3), (– 2, – 3), (– 2, 3), (2, – 3)
11. (5, 3) , (– 5, – 3) 12. 7 cm y 13 cm
13. 2,5 cm y 10 cm 14. 1,5 cm y 4,5 cm
15. a = 4 cm , b = 5 cm 16. a = 7 cm , b = 5 cm
17. r1 = 3 cm , r2 = 6 cm 18. a = 20 cm , b = 12 cm
19. a) (0, 2) b) (–1, 2) c) (1, 13 ) (–3, 13 ) 20. 50 cm
3 3

266 Ecuaciones e inecuaciones de segundo grado

266-267. 266 20/11/02, 12:17 PM


CAPITULO 4
CAPÍTULO

Prueba de selección múltiple


1. La suma y el producto sus raíces sean complejas 9. La intersección de la
de las raíces de la ecua- conjugadas? parábola cuya ecuación
ción ax2 + x + c = 0 son es y = 2x2 + 3x – 2 con el
9
respectivamente: A. k= eje x es en los puntos.
40
1 c
A. y
a
B. – 1 y
a
c
B. k<
9
40
A. ( 12 , –2)
a a 9
B. ( , 0) y (– 2, 0)
C. k <– 1
1 c 40
C. y –a 2
a
c 9
C. (0, ) y (0, – 2)
D. x
y D. k >– 1
a a 40
2
c 9
E. – x y E. k>
D. (0, – ) y (0, 2)
a a 40 1
6. La ecuación cuyas raíces 2
2. Las soluciones de la ecua-
son 0 y – 2 es:
ción x2 + x – 20 = 0 son:
A. x2 – 2 = 0
E. (–
1 ,0
2 ) y (2, 0)
A. – 5 y 4 10. El vértice de la parábola
B. x2 + 2 = 0
B. 5 y–4 C. x2 – 2x = 0 cuya ecuación es
C. – 4 y–5 D. x2 + 4x = 0 y = x2 – 2x – 24 tiene por
D. 4 y 5 E. x2 + 2x = 0 coordenadas:
E. 10 y–2 A. (1, – 25)
7. Una de las raíces de la
B. (1, 25)
ecuación
3. La ecuación cuyas raíces C. (– 1, 25)
ax2 – 2x – 3 = 0
son x1 = 4 y x2 = – 6 D. (– 1, – 25)
es: – 3
es: E. (0, –24)
¿Cuál es el valor de a?
A. x2 – 4x – 6 = 0 11. La función
B. x2 + 2x + 24 = 0 1 y = – x2 + 2x + 15
A.
9 alcanza su máximo valor
C. x2 – 2x + 24 = 0
1 para:
D. x2 + 2x – 24 = 0 B. –
9
E. x2 – 2x – 24 = 0 A. x = 5
C. – 1 B. x = –3
3
4. Para que las raíces de la 1 C. x = –1
D. D. x = 1
ecuación 4x2 + 12x – k = 0 3
sean reales e iguales el valor E. No se puede deter- E. x = –5
de k debe ser: minar.
12. La solución de la inecua-
A. 9 8. El producto de las raíces ción x2 – 2x > 0 está
B. – 9 de la ecuación representada por:
C. 36 2ax2 + 3abx + 4ab2 = 0 es:
A.
A. – 3 b
0 2
D. – 6
2 B. 0 2
E. 6
B. – 2b2 C. 0 2
5. ¿Qué condición debe C. 4ab2
cumplir k en la ecuación D. 2b2 D.
0 2
2kx2 + 3x + 5 = 0 para que E. 4ab E.
0 2

Ecuaciones e inecuaciones de segundo grado 267

266-267. 267 20/11/02, 12:19 PM


Prueba de selección múltiple
13. La solución de la ine- C. x = – 3 y = 3 A. 2
cuación x2 – 5x + 6 £ 0 D. x = – 3 y = – 3 B. – 2
está representada por: E. x = 6 y = 0 C. – 5
D. 7
A. 2 3 18. Dado el sistema:
E. – 7
B. x+y=9
2 3
x–y=1 22. x = – 3 es solución de la
C.
2 3 ecuación x2 – 9 = 0. La
D. el valor de 2x es: otra solución es:
–2 3

E. –3 2
A. 10 A. 9
B. 5 B. – 9
14. La solución de la ine-
C. 8 C. 3
cuación x2 – 1 ≥ 0 es:
D. 4 D. – 3
A. x ≥ 1 E. otro. E. 0
B. x £ – 1
19. Dado el sistema: 23. La suma de las solucio-
C. ] – •, – 1] K [1, •[ 2x + y = 7 nes de la ecuación
D. [ – 1, 1] x+y=4 2x2 + 5x – 1 = 0 es:
E. ] – 1, 1[ 1
el valor de –x es: A. 5
15. El conjunto [– 3, 3] es A. 3 1
solución de la inecuación: B.
B. – 3 2
C. 1 1
A. x2 + 9 < 0 C. – 2
D. – 1
B. x2 – 9 ≥ 0 5
E. 4 D. 2
C. x2 – 6x + 9 ≥ 0
20. Si x2 + y2 = 26 5
D. x2 – 9 £ 0 E. –
2
x+y =6
E. x2 + 6x + 9 < 0 24. El producto de las solu-
ciones de la ecuación
entonces son soluciones
16. La solución de la inecuación x2 + ax + b = 0 es:
del sistema:
x2 – 4 < 0 está dada por:
I. x = 5, y = 1 A. a
A. [ – 2, 2] II. x = 1, y = 5 B. b
B. ] – •, – 2[ III. x = y = 5 C. – a
C. ] – •, – 2[ K ]2, •[ D. – b
Son verdaderas:
D. ] – 2, 2[ A. Sólo I E. – b
a
E. ] – 2, •[ B. Sólo II
C. I y II 25. La condición para que
17. Una solución del sistema las soluciones de la
D. I y III
ecuación
3x + y = 6 E. Todas.
kx2 + 3x + 2 = 0 sean
x2 + y2 = 18 21. Si x = 5 es solución de complejas conjugadas es:
la ecuación 9
es: A. k >
8
x2 – 7x + k = 0
A. x = 3 y=–3 8
entonces la otra solu- B. k >
B. x = 3 y=3 9
ción es:

268 Ecuaciones e inecuaciones de segundo grado

268-269. 268 8/11/01, 16:14


CAPÍTULO 4

9 A. 0 C. 42 cm
C. k < –
8 D. 56 cm
B. 1
9 C. –1 E. 70 cm
D. k <
8
3
34. El área de un triángulo
8 D. rectángulo es 24 cm2
E. k < – 2
9
1 y la hipotenusa mide
26. Para que las soluciones E. –
2 10 cm. ¿Cuál es el perí-
de la ecuación
30. Una de las raíces de la metro?
12x2 + kx + 3 = 0 ecuación A. 24 cm
sean iguales se debe 2x2 + 17x – 9 = 0 B. 34 cm
cumplir: es – 9 C. 40 cm
A. k > 12 ¿Cuál es la otra raíz? D. 60 cm
B. k < 12 A. 9 E. 30 cm
C. k > – 12 B. – 2 35. El perímetro de un rec-
D. k < – 12 C. 2 tángulo es 28 cm y su
E. k = ± 12 1 área mide 33 cm2. El
D.
2 lado menor mide:
27. La suma y el producto
de las raíces de una E. – 1 A. 11 cm
2
ecuación cuadrática son B. 5 cm
31. La suma de dos números
3 y –10 respectivamente. C. 3 cm
es 21 y su producto es
La ecuación es: D. 6 cm
90. ¿Cuál es el número
A. x2 – 3x – 10 = 0 mayor? E. 7 cm
B. x2 – 3x + 10 = 0 A. 15 36. La suma de dos números
C. x2 + 3x – 10 = 0 B. 18 es 28 y la diferencia de
D. –x2 – 3x + 10 = 0 C. 9 sus cuadrados es 56. La
E. x2 + 3x + 10 = 0 diferencia de ellos es:
D. 6
E. 12 A. 2
28. Las raíces de una ecua-
ción de segundo grado B. 1
32. Dos números están en la
están en la razón 3:1 y C. 4
razón 3:2 y la diferen-
son ambas positivas. Si D. 8
cia de sus cuadrados es
la ecuación es: E. 6
20. ¿Cuál es el número
x2 + ax + 12 = 0 mayor? 37. La función que repre-
el valor de "a" es: A. 4 senta la curva dada es:
A. 2 B. 6
B. 4 C. 8
C. 8 D. 10
D. – 8 E. 2
E. no se puede deter-
. –2
33. La superficie de una
minar. jaula rectangular es de
29. ¿Qué valor debe tener k 48 cm 2 . Si los lados A. y = x2 + 2
en la función están en la razón 3:4. B. y = x2 – 2
y= 2x2 – 3x + k – 1 ¿Cuál es su perímetro?
C. x = y2 + 2
para que el punto (0, 0) A. 14 cm D. x = y2 – 2
pertenezca a ella? B. 28 cm E. y = – x2 – 2
Ecuaciones e inecuaciones de segundo grado 269

268-269. 269 8/11/01, 16:14


Prueba de selección múltiple
38. A partir del siguiente 41. La función asociada al 45. La función cuya gráfica
gráfico, podemos afir- gráfico es: es la siguiente cumple las
mar que la ecuación siguientes condiciones:
cuadrática asociada:
3

–1 3

A. y = – x2 – 2x + 3 A. Δ = 0 a>0
B. y = – x2 – 2x – 3 B. Δ = 0 a<0
A. Tiene solución ima- C. y = – x2 + 2x + 3 C. Δ = 0 a=0
ginaria. D. y = – x2 + 2x – 3 D. Δ > 0 a>0
E. y = x2 + 2x + 3 E. Δ < 0 a<0
B. Tiene una raíz nega-
tiva. 42. El recorrido de la función 46. La gráfica de la función
C. Tiene raíces reales y = 16x2 – 1 es: y = 3x2 – 2x intersecta
iguales. A. ] –•, 1] al eje x en:
D. Tiene raíces reales y B. ] – •, – 1] A. 0 y 2
distintas. C. [1, • [
B. 0 y 3
D. [ – 1, •+[
E. No tiene solución. 2
E. [ – 1, 1] C. 0 y
3
3
39. La gráfica de la función D. 0 y
43. El recorrido de la fun- 2
cuadrática ción E. 0 y –2
y = 3x2 – 2x – 5 inter- y = – x2 + 2x + 15 es: 3
secta al eje y en: 47. La gráfica de la función
A. [16, • + [
y = x2 – x + 1 intersecta
A. – 3 B. [– 16, • + [ al eje x en:
C. ] – •, – 16] A. x = 1
B. – 2
D. ] – •, 16] B. x = 0
C. 2
E. [– 16, 16]
C. x = – 1
D. – 5
44. La función cuya gráfica D. x = – 2
E. 5 es la siguiente cumple las
E. No lo intersecta.
siguientes condiciones:
40. La gráfica de la función 48. Las coordenadas del
y = 3x2 – 8x – 3 vértice de la parábola
intersecta al eje x en: cuya función es
y = 9x2 + 6x – 8 son:
1
A. 3 y– 3
1
A. ( , 9)
1
3
B. – 3 y 3
1 A. Δ > 0; a>0
B. (– , 9)
1
3
C. – 3 y – 3
1
B. Δ = 0; a<0 C. ( , – 9)
1
3
D. 3 y 3
E. 3 y –3
C.
D.
Δ > 0;
Δ < 0;
a<0
a<0
D. (– , – 9)
1
3
E. Δ = 0; a>0 ,
E. (– 3 – 9)

270 Ecuaciones e inecuaciones de segundo grado

270-271. 270 20/11/02, 12:21 PM


CAPITULO 4
CAPÍTULO

Soluciones

1. B 7. C 13. A 19. B 25. A 31. A 37. B 43. D


2. A 8. D 14. C 20. C 26. E 32. B 38. C 44. C
3. D 9. B 15. D 21. A 27. A 33. B 39. D 45. A
4. B 10. A 16. D 22. C 28. D 34. A 40. A 46. C
5. E 11. D 17. A 23. E 29. B 35. C 41. C 47. E
6. E 12. C 18. A 24. B 30. D 36. A 42. D 48. D

Ecuaciones e inecuaciones de segundo grado 271

270-271. 271 20/11/02, 12:22 PM


CAPÍTULO 5
P
olinomios
y Teoría de
ecuaciones

5.1 Definición y
operaciones con polinomios

Definición: Decimos que P es un polinomio en el conjunto de


los números reales R si y sólo si P es una función de R en R, tal
que P(x) admite una representación de la forma

P(x) = an xn + an – 1 x n – 1 + …… + a2 x2 + a1 x + a0

donde an, an – 1, …… a2, a1, a0 E R

y se denominan coeficientes del polimonio, y n E N

Ejemplo: P(x) = 2 x2 – 3x + 1
Q(x) = –5x3 – 2x2 + x – 1
R(x) = x5 – 3x + 2
C(x) = 4x2 – 1
son polinomios.

Definición: Llamamos grado de un polinomio P al mayor


exponente que presenta la variable (frecuentemente x) con
coeficiente distinto de cero.

En el ejemplo anterior:
grado P (x) =2
grado Q (x) =3
grado R (x) =5
grado C (x) =2

272 Polinomios y Teoría de ecuaciones

272-273. 272 20/11/02, 12:48 PM


CAPÍTULO 5

• Observación 1: Un número real puede entenderse como un


polinomio de grado 0.
• Observación 2: Al coeficiente del término de mayor grado del
polinomio se le llama coeficiente principal.
• Observación 3: Dos polinomios P(x) y Q(x) son iguales si y sólo
si Ix E R, P(x) = Q(x).

OPERACIONES CON POLINOMIOS.


Sean P(x) = an xn + am xm +...+ a2 x2 + a1 x + a0
y Q(x) = bm xm + .....+ b2 x2 + b1 x + b0

dos polinomios de grados n y m, E N, con n > m.


Estos polinomios podemos escribirlos usando sumatoria
n m
P(x) = Σ a ix i ;
i= 0
Σ
Q(x) = i = 0 b i x
i

5.1.1 Suma. (Ver problema 2, pág. 275)


P(x) + Q(x) = anxn + … + (am + bm) xm + … + (a2 + b2)x2 +
(a1 + b1) x + (a0 + b0)
o bien, usando sumatoria:
n
P(x) + Q(x) = Σ
i= 0
ai + bi x i

5.1.2 Resta. (Ver problema 2, pág. 275)


Previo:
(–1) P(x) = – anxn – … amxm – … – a2 x2 – a1x– a0
luego
P(x) – Q(x) = P(x) + (–1) Q(x)

5.1.3 Producto. (Ver problema 3, pág. 275)


P(x) • Q(x) = este producto se obtiene aplicando la propiedad distributiva
del producto sobre la suma en forma reiterada y considerando las leyes
de multiplicación de potencias de igual base.
m+n i
P(x) • Q(x) = Σ ci x i,
i= 0
donde ci = Σ a k bi–k
k=0

Teorema 1: El conjunto de los polinomios con coeficientes reales


para la suma y el producto de polinomios tiene una estructura
de anillo:

Polinomios y Teoría de ecuaciones 273

272-273. 273 20/11/02, 12:49 PM


Es decir:

{
+ es cerrada
+ es asociativa
(P, +) es grupo abeliano + es conmutativa
+ tiene elemento neutro
+ tiene elemento inverso
• es asociativa
• es distributiva sobre la suma.

Teorema 2. Sean P(x) y Q(x) dos polinomios con coeficientes


reales.
Si P(x) • Q(x) = 0 I x E R,
entonces P(x) = 0 V Q(x) = 0

Teorema 3. Sean P(x), Q(x) y R(x) tres polinomios en R, tales


que P(x) π 0.

Si P(x) • Q(x) = P(x) • R(x) I x E R,


entonces Q(x) = R(x).

5.1.4 División. (Problemas 4 y 5, pág. 276)


En toda división de dos polinomios se verifica que
Dividendo = Divisor • cociente + resto.
P(x) = Q(x) • C(x) + R(x),
lo que también se puede escribir como
P(x) R(x)
= C(x) +
Q(x) Q(x)
• Observación 1: Para efectuar la división de P(x) por Q(x), Q(x) debe
tener grado inferior o igual a P(x).
• Observación 2: La división de polinomios se termina cuando el
resto es de grado inferior al divisor.
Regla de Ruffini o división sintética: Para efectuar la división de
polinomios por un divisor de la forma x – a es posible trabajar sólo
con los coeficientes y mediante este procedimiento determinar el
cociente y el resto. Ver problema 7, pág. 277.
Teorema 4. (Teorema del Resto) Al dividir un polinomio P(x) por
(x – a), el resto es P(a).
Demostración: Recordemos que al dividir un polinomio se obtiene un
cociente C(x) y un resto R(x), donde el resto tiene grado inferior que
el grado del divisor. En este caso el grado del divisor (x – a) es 1, por
lo tanto el grado del resto es cero, es decir, el resto es una constante
real que denotaremos por r, R(x) = r.
P(x) r
x – a = C(x) + x – a o bien
P(x) = C(x) • (x – a) + r

274 Polinomios y Teoría de ecuaciones

274-275. 274 20/11/02, 12:50 PM


CAPÍTULO 5

Si hacemos x = a nos queda P(a) = C(a) • 0 + r

P(a) = r.

Es decir, al dividir P(x) por (x – a) el resto está expresado por P(a).

1. Dado el polinomio P(x) = 5x4 – 3x2 + x – 3


Ejercicios
determinar su grado y hallar su valor para x = 1, x = –1 y x = 0. resueltos
Solución:

El grado de P(x) es 4.

Como P(x) = 5x4 – 3x2 + x – 3


Se tiene que P(1) = 5 • 14 – 3 • 12 + 1 – 3
= 5–3+1–3=0
P(–1) = 5 • (–1)4 – 3 • (–1)2 + (–1) – 3
= 5 – 3 – 1 – 3 = –2
P(0) = 5 • 04 – 3 • 02 + 0 – 3 = – 3

Observar que el valor de un polinomio en x = 0 es equivalente al término


independiente de la variable que presente el polinomio.

2. Dados los polinomios P(x) = 6x5 – 3x2 + x – 2


y Q(x) = x4 – x3 + 2x2 – x + 3.
Hallar P(x) + Q(x); P(x) – Q(x).

Solución:

P(x) + Q(x) = (6x5 – 3x2 + x – 2) + (x4 – x3 + 2x2 – x + 3)


= 6x5 + x4 – x3 – x2 + 1
P(x) – Q(x) = P(x) + (–1) Q(x)
= (6x5 – 3x2 + x – 2) + (–x4 + x3 –2x2 + x – 3)
= 6x5 – x4 + x3 – 5x2 + 2x – 5

Observar que para obtener (–1) Q(x) se cambia el signo de todos


los términos de Q(x).

3. Dados los polinomios P(x) = x3 – x2 + 2x – 1


y Q(x) = x2 – x + 3.
Hallar P(x) • Q(x).

Solución:

Para obtener el producto de ambos polinomios debemos multiplicar


cada término de P(x) por cada término de Q(x) y luego reducir
términos semejantes.

Polinomios y Teoría de ecuaciones 275

274-275. 275 20/11/02, 12:50 PM


Ejercicios P(x) • Q(x) = (x3 – x2 + 2x – 1) • (x2 – x + 3)
resueltos = x5 – x4 + 2x3 – x2 – x4 + x3 – 2x2 + x + 3x3 – 3x2 + 6x – 3

= x5 – 2x4 + 6x3 – 6x2 + 7x – 3

4. Si P(x) = 2x4 – x3 + 3x – 1
y Q(x) = x2 – x + 3
P(x)
Hallar el cociente C(x) = y el resto R(x)
Solución: Q(x)
Para realizar una división de polinomios, primero se deben ordenar
dividendo y divisor de acuerdo al grado de sus términos. Seguidamente
buscamos el primer término del cociente, que es el que multiplicado
por el primer término del divisor da el primer término del dividendo.
Este término se multiplica por cada término del divisor y ese producto
se resta del dividendo.
Se obtiene así un nuevo polinomio, el cual se divide nuevamente
por el divisor, con el mismo procedimiento, obteniendo el segundo
término del cociente. Análogamente se obtienen los demás términos
del cociente.
La división se termina cuando al restar aparece un polinomio cuyo grado
es inferior al grado del divisor.

(2x4 – x3 + 3x – 1) : (x2 – x + 3) = 2x2 + x – 5


–) 2x4 – 2x3+ 6x2
x3 – 6x2 + 3x – 1
–) x3 – 6x2 + 3x
– 5x2 – 1
–) – 5x2 + 5x – 15
– 5x + 14
2
Luego C(x) = 2x + x – 5 es el cociente y
R(x) = – 5x + 14 es el resto.

5. Sea P(x) = x6 – 64
y Q(x) = x3 – 8
P(x)
Hallar el cociente C(x) = y el resto R(x).
Q(x)
Solución: (Ver explicación ejercicio anterior)

(x6 – 64) : (x3 – 8) = x3 + 8


–) x6 – 8x3
8x3 – 64
–) 8x3 – 64
0 0
C(x) = x3 + 8
R(x) = 0

276 Polinomios y Teoría de ecuaciones

276-277. 276 20/11/02, 12:51 PM


CAPÍTULO 5

6. Realizar el cociente entre x5– 243 y x – 3

Solución: (Ver explicación ejercicio 4)


(x5– 243) : (x – 3) = x4 + 3x3 + 9x2 + 27x + 81
–) x5 – 3x4
3x4 – 243
–) 3x4 – 9x3
9x3 – 243
–) 9x3 – 27x2
27x2 – 243
–) 27x2 – 81x
81x – 243
–) 81x – 243
0 0

Luego (x5– 243) : (x – 3) = x4 + 3x3 + 9x2 + 27x + 81

7. Efectuar el cociente entre P(x) = 6x4 – 10x3 – 4x2 – 3x + 6


y Q(x) = x – 2.
Solución:
Efectuaremos la división aplicando la regla de Ruffini o división
sintética.
– Copiamos los coeficientes del dividendo teniendo especial cuidado
de anotar 0 si algún término no aparece.
– Copiamos al margen derecho el término independiente del divisor
con signo contrario (2).
– Al dividir un polinomio por (x – a) el cociente tendrá como grado
una unidad inferior al dividendo y el coeficiente principal del cociente
será el mismo del dividendo. Copiamos, según muestra el diagrama el
coeficiente principal (6) bajo la línea horizontal.
6 –10 –4 –3 6 2

6
– Este número (6) es el coeficiente principal del cociente. Ahora este
coeficiente principal se multiplica por el factor (2) que está al margen y
su resultado se escribe sobre la línea horizontal bajo el coeficiente del
segundo término del dividendo. Se suman estos dos números obteniendo
así el coeficiente del segundo término del cociente (2).
6 –10 –4 –3 6 2
12

6 2
– Este número (2) se multiplica por el factor (2) que está al margen
y se procede en la misma forma anterior hasta llegar al último
número (0).

Polinomios y Teoría de ecuaciones 277

276-277. 277 20/11/02, 12:52 PM


Ejercicios 6 –10 – 4 –3 6 2
resueltos 12 4 0 –6
6 2 0 –3 0

Bajo la línea horizontal se obtienen los coeficientes del cociente, cuyo


grado es inferior en una unidad al grado del dividendo y el último número
representa el resto de la división. En este caso es 0.

Así, el cociente es 6x3 + 2x2 – 3 y el resto es 0.

8. Efectuar la división entre los siguientes polinomios:


P(x) = 5x5 – 3x2 + 6x + 12 y Q(x) = x + 1
Solución:
Usaremos división sintética (ver explicaciones del ejercicio anterior).
Dividendo P(x) = 5x5 – 3x2 + 6x + 12
Divisor Q(x) = x + 1
Coeficientes del dividendo

5 0 0 –3 6 12 –1 O término
independiente del
–5 5 –5 8 –14 divisor con signo
5 –5 5 –8 14 –2 contrario.

X
coeficientes del cociente resto

Luego cociente C (x) = 5x4 – 5x3 + 5x2 – 8x + 14


y el resto R (x) = – 2

Observar que el resto es independiente de x pues su grado debe ser


menor que el del divisor que en este caso es uno.

9. Determinar el resto que se produce al dividir


P(x) = x6 – 3x2 + 2x – 5 por x – 1
Solución:
Para hallar la solución basta con evaluar P(1). (Solución de la ecuación
x – 1 = 0, divisor igual a cero).
P(1) = 16 – 3(1)2 + 2(1) – 5
=1–3+2–5=–5

10. Sean P(x) = x3 – ax2 + x – b


y Q(x) = x3 – 2x2 + ax + b
dos polinomios. Determinar a y b para que P(x) + 2 sea divisible por
x – 1 y Q(x) – 3 sea divisible por x + 1.
Solución:
P(x) + 2 = x3 – ax2 + x – b + 2
Dividiendo P(x) + 2 por x – 1:

278 Polinomios y Teoría de ecuaciones

278-279. 278 20/11/02, 12:53 PM


CAPÍTULO 5

1 –a 1 –b + 2 1
1 1–a 2–a
1 1–a 2–a –a–b+4

–a–b+4=0 (1) El resto debe ser cero.

Q(x) – 3 = x3 – 2x2 + ax + b – 3

Dividiendo Q(x) – 3 por x + 1

1 –2 a b–3 –1
–1 3 –a–3
1 –3 a+3 –a+b–6

–a+b–6=0 (2) El resto debe ser cero.

Resolviendo el sistema (1) y (2).


a+b=+4
fi a=–1yb=5
a–b=–6

Ejercicios
1. Dados los siguientes polinomios, determine su valor para el número
real indicado:
a) P(x) = 5x6 – 3x4 + 2x2 – 1 x=2
b) P(x) = –x5 – 2x4 + x3 + x2 – 3x – 1 x = –1
c) P(x) = 4x4 – 3x + 4 x=0
1
d) P(x) = 36x6 – 2x5 – x – 3 x=
2
e) P(x) = 2x4 – x3 + 3x2 – x + 2 x=4
f) P(x) = x3 – x + 2 x = –3
g) P(x) = 3x3 + x2 + 5x x = –2

h) P(x) = 2x2 – x + 5 x= 5
1 2
i) P(x) = x4 – x2 + x – 1 x= 2
2 3
j) P(x) = x5 – x4 – x3 + x2 + x – 1 x=1
k) P(x) = x5 – x4 + x3 – x2 + x – 1 x = –1
l) P(x) = x5 + x4 + x3 + x2 + x + 1 x=1
m) P(x) = x5 + x4 + x3 + x2 + x + 1 x = –1

Polinomios y Teoría de ecuaciones 279

278-279. 279 20/11/02, 12:53 PM


Ejercicios
2. Dados los polinomios P(x) = x2 + 2x – 1 y Q(x) = x3 – x + 2
Determine:
a) P(x) + Q(x) b) P(x) – Q(x) c) 2P(x) – 3 Q(x)
d) P(x) • Q(x) e) P(x) + 2Q(x)

3. Dados los polinomios P(x) = x3 – 2x2 + x – 2 y Q(x) = x – 3


Encuentre:
a) P(x) + Q(x) b) P(x) – Q(x) c) P(x) + [Q(x)]2
d) P(x) + x Q(x) e) P(x) + P(x) • Q(x)
4. Dar ejemplos de sumas y productos de polinomios donde se verifiquen
todas las propiedades del teorema 1.

5. Si P(x) = 3x3 – 2x2 + x – 5 Q(x) = x3 + 3x


R(x) = x2 – x + 2 S(x) = – x3 + 2x2 –3.
Determine:
a) [P(x) + Q(x)] – [R(x) + S(x)]
b) P(x) – [Q(x) + R(x) + S(x)]
c) [P(x) – Q(x)] + [R(x) – S(x)]
6. Realice la división de los siguientes polinomios; obtenga cociente
C(x) y resto R(x).
a) (5x3 + 2x – 3) : (x2 – x + 1) =
b) (x5 – 2x4 + x3 – 3x2 + 5) : (2x3 – 2x2 + x – 1) =
c) (4x5 + x3 – 6x + 1) : (x3 – 2x + 3) =
d) (x4 – 3x3 + 2x2 – x + 4) : (x2 – 2x + 1) =
e) (x5 + x4 – 4x3 + 4x2 – 5x + 3) : (x2 + 2x – 3) =
f) (2x 6 – 2x 5 + x 4 + 4x 3 – 6x 2 + 6x – 5) : (2x 3 – x + 5) =
g) (x7 – 3x6 + 2x4 + 2x3 – 8x2 + 3x – 3) : (x3 – 3x2 + x – 1) =
h) (x7 – 3x5 + x4 – 2) : (x5 + x2 – 3) =
7. Realice la división de los siguientes polinomios obteniendo cociente
C(x) y resto R(x).
a) (x4 – 14x2 + 17x – 6) : (x – 3)
b) (x3 + 3x2 – 5x + 2) : (x – 1)
c) (x5 + 3x4 + 2x3 – 3x2 – 7x – 2) : (x + 2)
d) (x4 + x3 – 3x – 1) : (x + 1)
e) (x6 + 3x5 –2x4 – 5x3 + 2x2 – 3x) : (x + 3)

280 Polinomios y Teoría de ecuaciones

280-281. 280 20/11/02, 12:54 PM


CAPÍTULO 5

f) (x5 – 2x3 + x2 – x) : (x – 1) =
g) (x4 + x3 – x2 ) : (x2) =
h) (x5 – 2x3 + x2 – x) : (x + 1) =
i) (x5 – 3x4 – 4x3 – 3x + 12) : (x – 4) =
j) (x5 – 2x4 – 4x3 – 3x + 12) : (x – 3) =
k) (x5 – 3x4 – 4x3 – 3x + 12) : (x – 1) =
l) (x4 – 2x3 – 34x2 + 4x – 5) : (x + 5) =
m) (x3 – 3x2 + x – 1) : (x – 1) =
8. Encuentre el resto que se produce al dividir cada uno de los
polinomios dados por x + 3.
a) x4 – 2x3 + x2 – 1
b) 3x5 – x + 6
c) x3 + x2 – 8
d) –3x3 + 4x2 – 9
9. Determine el resto que se produce al realizar las siguientes
divisiones.
a) x5 – 2x + 3 por x+1
b) x4 – x3 + 2x2 – x + 3 por x–1
c) 4x3 – 2x2 + x – 5 por x+2
d) –2x2 – 4x + 6 por x–3
10. ¿Qué valor debe tomar a, para que al dividir x4– 6x2 + x – a por
x – 2, el resto sea cero?

11. ¿Qué valor debe tomar a, para que al dividir x3– 4x2 + ax – 3 por
x + 2, la división sea exacta?

12. ¿Qué valor debe tomar a, para que al dividir 3x4– x3 + 2x2 – ax + 1
por x – 1, la división dé como resto 5?

13. Encuentre un polinomio P(x) tal que dividido por x2 – x + 1 dé como


cociente x + 2 y como resto x – 3.

14. Determine los valores de a, b y c para que


ax2 + bx– c
=3 IxER
3x2 + 5x – 1
15. Determine el valor de a E R de modo que los polinomios
P(x)= x4 – 2 ax3 + 3x2 – 2 ax + 1 y
Q(x) = x2 – x + 1
verifiquen la condición P(x) = [Q(x)]2

16. Determine un polinomio P de segundo grado de modo que


P(0) = 1, P(–1) = 0, P(–2) = 1

Polinomios y Teoría de ecuaciones 281

280-281. 281 20/11/02, 12:54 PM


Ejercicios
17. Pruebe que si x3 + ax – b es divisible por x2 – x – b,
entonces a + b = –1.
18. Sea P(x) = x3 + 2x2 + ax + b. Determine a y b tales que P(x) + 1 sea
divisible por x + 1 y P(x) – 1 sea divisible por x – 1.
19. Sea P(x) = x3 – 2x2 + ax – b. Determine a y b tales que P(x) + 2 sea
divisible por x – 2 y P(x) + 1 sea divisible por x – 1.
20. Determine a y b para que P(x) = x3 – (a + b) x + 2 y
Q(x) = x2 – x + (a – b) sean ambos divisibles por 1 – x.
21. Determine un polinomio P de segundo grado de modo que
P(1) = 0, P(–1) = 6 y P(0) = 1.
22. Encuentre un polinomio de segundo grado P de modo que
P(1) = – 1, P(2) = – 8 y P(–1) = – 5.
23. Encuentre los valores de a y b en el polinomio
4x3 – 3x2 + ax + b para que éste sea divisible por x2 – 1.
24. Al dividir ax4 + bx3 – 12x2 + 16x – 5 por x2 + 2x – 3 se obtiene como
resto –32x + 40. Pruebe que a = 5 y b = 4.

Soluciones

1. a) 279 b) 1 c) 4 d) – 3 e) 494 f) – 22 g) – 30 h) 10

i) 2 2 – 3 j) 0 k) – 6 l) 6 m) 0
3
2. a) P(x) + Q(x) = x3 + x2 + x + 1 b) P(x) – Q(x) = –x3 + x2 + 3x – 3
c) 2P(x) – 3Q(x) = –3x3 + 2x2 + 7x – 8 d) P(x) • Q(x) = x5 + 2x4 – 2x3 + 5x – 2
e) P(x) + 2Q(x) = 2x3 + x2 + 3

3. a) P(x) + Q(x) = x3 – 2x2 + 2x – 5 b) P(x) – Q(x) = x3 – 2x2 + 1


c) P(x) + [Q(x)]2 = x3 – x2 – 5x + 7 d) P(x) + x Q(x) = x3 – x2 – 2x – 2
e) P(x) + P(x) • Q(x) = x4 – 4x3 + 5x2 – 4x + 4

5. a) 5x3 – 5x2 + 5x – 4 b) 3x3 – 5x2 – x – 4 c) 3x3 – 3x2 – 3x

6. a) C(x) = 5x + 5 R(x) = 2x – 8

b) C(x) = 1 x2 – 1 x – 1 R(x) = 5 x2 – 1 x + 19
2 2 4 2 4 4
c) C(x) = 4x2 + 9 R(x) = –12x2 + 12x – 26
d) C(x) = x2 – x – 1 R(x) = –2x + 5
e) C(x) = x3 – x2 +x–1 R(x) = 0
f) C(x) = x3 – x2 +x–1 R(x) = 0
g) C(x) = x4 – x2 +3 R(x) = 0
h) C(x) = x2 – 3 R(x) = 6x2 – 11

282 Polinomios y Teoría de ecuaciones

282-283.(2003) 282 20/11/02, 12:56 PM


CAPITULO 5
CAPÍTULO

7. a) C(x) = x3 + 3x2 – 5x + 2 R(x) = 0


b) C(x) = x2 + 4x – 1 R(x) = 1
c) C(x) = x4 + x3 – 3x – 1 R(x) = 0
d) C(x) = x3 – 3 R(x) = 2
e) C(x) = x5 – 2x3 + x2 – x R(x) = 0
f) C(x) = x4 + x3 – x2 – 1 R(x) = – 1
g) C(x) = x2 + x – 1 R(x) = 0
h) C(x) = x4 – x3 – x2 + 2x – 3 R(x) = 3
i) C(x) = x4 + x3 – 3 R(x) = 0
j) C(x) = x4 + x3 – x2 – 3x – 12 R(x) = – 24
k) C(x) = x4 – 2x3 – 6x2 – 6x – 9 R(x) = 3
l) C(x) = x3 – 7x2 + x – 1 R(x) = 0
m) C(x) = x2 – 2x – 1 R(x) = – 2
8. a) 143 b) – 720 c) – 26 d) 108
9. a) 4 b) 4 c) – 47 d) – 24
27
10. a = – 6 11. a = – 12. a = 0 13. x3 + x2 – 1
2
14. a = 9 b = 15 c=3
15. a = 1 16. x2 + 2x + 1 18. a = 0 , b = –2

19. a = b = 2 20. a = b = 3 21. P(x) = 2x2 – 3x + 1


2
22. P(x) = – 3x2 + 2x 23. a = – 4 y b = 3

Teoría de ecuaciones 5.2

5.2.1 Cálculo de las raíces


de un polinomio.
Factorización.

Definición: Un número a es raíz de un polinomio P(x) si


y sólo si P(a) = 0.

Teorema 5: Si a es una raíz de la ecuación polinómica P(x) = 0, es


decir, P(a) = 0, entonces (x – a) es divisor de P(x).
Si (x – a) es divisor de P(x), entonces a es una raíz de la
ecuación P(x) = 0.
Ver ejercicio 2, (pág. 285).
Teorema 6: Sea P(x) un polinomio no constante, entonces, P(x) tiene
a lo menos una raíz, real o compleja.

Polinomios y Teoría de ecuaciones 283

282-283.(2003) 283 20/11/02, 12:57 PM


Teorema 7: Sea P(x) un polinomio de grado n ≥ 1, entonces P(x)
tiene a lo más n raíces distintas.
Algunas raíces de una ecuación polinómica pueden ser iguales;
esto es, tienen multiplicidad mayor que 1.
Por ejemplo:
La ecuación de 6º grado P(x) = (x – 2)3 (x – 1) (x + 3)2 = 0
tiene por raíces 2 de multiplicidad 3,
1 de multiplicidad 1
– 3 de multiplicidad 2,
luego las 6 raíces son 2, 2, 2, 1, –3 y –3.

RAÍCES COMPLEJAS Y RAÍCES IRRACIONALES.


Si un número complejo a + bi es raíz de un polinomio P(x)
de coeficientes reales, el complejo conjugado a – bi también
es raíz de P(x).
De aquí se deduce que una ecuación P(x) = 0 de grado impar
tiene por lo menos una raíz real.
Si un número real de la forma a + b , con b irracional, es raíz de
una ecuación P(x) = 0 con coeficientes racionales, entonces el número
a– b también es raíz de la ecuación P(x) = 0.

RAÍCES RACIONALES.
p
Si q es una fracción irreducible que es raíz de la ecuación
P(x) = an xn + an –1 xn – 1 + …… + a2 x2 + a1 x + a0 = 0
entonces p es divisor de a0 y q es divisor de an.

Ejemplo:
Dada la ecuación 2x4 + 5x5 + x2 + 10x – 6 = 0 pueden ser raíces
racionales de ellas los números formados por un divisor de 6 (± 1,
± 2, ± 3, ± 6) partido por un divisor de 2 (± 1, ± 2).
Así pueden ser solución o raíz los números

1 3
± 1, ± , ± 2, ± 3, ± , ± 6.
2 2
1
En efecto, son soluciones racionales y – 3.
2

5.2.2 Relación entre los coeficientes


de una ecuación p(x) = 0
y sus raíces
Sea P(x) = an xn + an – 1 xn – 1 + . . . + a2 x2 + a1 x + ao = 0, an π 0
dividiendo la ecuación por an tenemos:
an– 1 a2 2 a1 a0
xn + xn– 1 + …… + x + x+ = 0
an an an an

284 Polinomios y Teoría de ecuaciones

284-285. 284 20/11/02, 12:58 PM


CAPÍTULO 5

donde:
an – 1
– a = suma de las raíces.
n
an – 2
= suma de los productos de las raíces
an
tomadas de dos en dos.
an – 3
– a = suma de los productos de las raíces
n
tomadas de tres en tres y así sucesivamente hasta
a0
(–1) n a = producto de todas las raíces.
n
Un caso particular:
Si P(x) = ax2 + bx + c = 0
y sus raíces son r1 y r2

entonces: r1 + r2 = – b
a
c
r1 • r2 = a
Si P(x) = ax3 + bx2 + cx + d = 0
y sus raíces son r1, r2 y r3

entonces: r1 + r2 + r3 = – b
a
r1 r2 + r1 r3 + r2 r3 = c
a
r1 r2 r3 = – d
a
Ver ejercicios 6 y 7, (págs. 287, 288).

1. Determinar si 3, – 2, 1 son o no raíces del polinomio Ejercicios


x4 + x3 – 7x2 – 13x – 6 resueltos
Solución:
a es raíz de P(x) si P(a) = 0
P(3) = 34 + 33 – 7 • 32 – 13 • 3 – 6 = 0
luego 3 es raíz del polinomio.
P(– 2) = (– 2)4 + (– 2)3 – 7(– 2)2 – 13 (– 2) – 6 = 0
luego – 2 es raíz del polinomio
P(1) = (1)4 + 13 – 7 • 12 – 13 • 1 – 6 = – 24
luego 1 no es raíz del polinomio.

2. Demostración teorema 5.
a) P(a) = 0 fi x – a divisor de P(x)
P(x) = (x – a) • Q(x) + P(a) (5. 1. 4)
como P(a) = 0
P(x) = (x – a) • Q(x)

Polinomios y Teoría de ecuaciones 285

284-285. 285 20/11/02, 12:59 PM


Ejercicios Luego, (x – a) es divisor de P(x).
resueltos b) x – a divisor de P(x) fi P(a) = 0
P(x) = (x – a) Q(x) + P(a)
como (x – a) es divisor de P(x), al dividir se tiene resto 0,
luego P(a) = 0

3. Escribir un polinomio que tenga por raíces 2, 3 y –1 de multiplicidad 2.


Solución:
Como este polinomio tiene cuatro raíces; 2, 3, –1, –1, su grado
debe ser a lo menos cuatro y (x – 2), (x – 3) y (x + 1)2 son
factores del polinomio pedido.
Luego un polinomio que cumple las exigencias es
P(x) = (x – 2) (x – 3) (x + 1)2
= (x2 – 5x + 6) (x2 + 2x + 1)
P(x) = x 4 – 3x3 – 3x2 + 7x + 6

4. Determinar las raíces racionales de la ecuación


2x 4 – 3x3 – 2x2 – 18x – 9 = 0
Solución:
Divisores de 9 = {±1, ±3, ±9}
Divisores de 2 = {±1, ±2}
Posibles soluciones racionales de la ecuación propuesta:

{ 1 3
± 1, ± , ± 3, ± , ± 9, ±
2 2
9
2 }
Para determinar cuáles son las soluciones debemos probar ya
sea usando división sintética o evaluando la ecuación para los
distintos valores posibles.

Para – ( 12 ): 2 (– 12 )
( 12 ) – 2 (– 12 )
4
–3 –
3 2
( 12 ) – 9 = 0
– 18 –

luego – 1 es raíz y ( x + ) es factor de


1
2 2
2x4 – 3x3 – 2x2 – 18x – 9
para 3: 2 (3)4 – 3 (3)3 – 2 (3)2 – 18 (3) – 9 = 0
luego 3 es raíz y (x – 3) es factor del polinomio.
La ecuación 2x4 – 3x3 – 2x2 – 18x –9 = 0 no tiene más raíces
racionales.

5. Factorizar el polinomio x 4 – 4x3 – 6x2 + 7x – 10.


Solución:
Son posibles raíces racionales del polinomio los divisores de
10 = {±1, ±2, ±5, ±10}
Si aplicamos división sintética y probamos con 5 obtenemos:

286 Polinomios y Teoría de ecuaciones

286-287. 286 20/11/02, 1:03 PM


CAPÍTULO 5

1 –4 –6 7 –10 5 Q (x – 5) es factor
5 5 –5 10

1 1 –1 2 0 Q resto

x3 + x2 – x + 2 es el cociente que se obtiene al dividir


el polinomio dado por (x – 5) y resto 0.
Luego podemos escribir:
x 4 – 4x3 – 6x2 + 7x – 10 = (x3 + x2 – x + 2) (x – 5)

Ahora buscamos algún factor del polinomio x3 + x2 – x + 2.


Son posibles soluciones los divisores de 2 = {±1, ±2}.
Aplicando división sintética y probando con –2 obtenemos

1 1 –1 2 –2 Q (x + 2)
– 2 2

1 –1 1 0 Q resto

x2 – x + 1 es el cociente que se obtiene al dividir


x3 + x2 – x + 2 por x + 2 y resto 0.
luego:
x4 – 4x3 – 6x2 + 7x – 10 = (x2 – x + 1) (x + 2) (x – 5)
Observamos que x2 – x + 1 no es factorizable en R ya que sus raíces
son complejas conjugadas (discriminante menor que cero).

Así podemos decir que: (x2 – x + 1) (x + 2) (x – 5) es la factorización


del polinomio x4 – 4x3 – 6x2 + 7x – 10
6. Resolver la ecuación x3 – 16x2 + 79x – 120 = 0 sabiendo que una
de sus raíces es 7 unidades menor que el producto de las otras dos y
que las tres raíces son racionales.
Solución:
De acuerdo con la relación entre coeficientes y raíces de una
ecuación sabemos que:

r1 + r2 + r3 = 16
r1 • r2 • r3 = 120
y por dato del problema podemos decir que:
r3 = r1 • r2 – 7
así resolviendo el sistema:
r1 + r2 + r3 = 16
r1 • r2 • r3 = 120
r3 + 7 = r1 • r2

Polinomios y Teoría de ecuaciones 287

286-287. 287 20/11/02, 1:03 PM


Ejercicios r32 + 7r3 – 120 = 0
(r3 + 15) (r3 – 8) = 0
resueltos r3 = –15 o r3 = 8

Si r3 = –15
r1 + r2 = 31 fi r1 = 31 + 993 y r2 = 31 – 993
2 2
r1 • r2 = – 8 que no son racionales.

Si r3 = 8
r1 + r2 = 8 fi r1 = 3 y r2 = 5
r1 • r2 = 15

Por lo tanto, la solución es r1 = 3, r2 = 5 y r3 = 8


7. Si la ecuación x3 – 3x2 – x – 6 = 0 tiene como raíces los valores a, b
y γ. Hallar el valor de las siguientes relaciones:
a) a2 + b2 + γ2
b) a3 + b3 + γ3
c) 1 + 1 + 1
α β γ
Solución:
Sabemos que:
a+b+γ = 3 (Relación entre coeficientes
ab + aγ + bγ = –1 y raíces de una ecuación)
abγ = 6

a) (a + b + γ)2 = a2 + b2 + γ2+ 2ab + 2aγ + 2bγ = 9


= a2 + b2 + γ2 + 2 (ab + aγ + bγ) = 2

–1
= a2 +b2 + γ2 –
2=2
Luego: a2 +b +γ =4
2 2

b) La ecuación es x3 – 3x2 + x – 6 = 0 y se satisface para todas


sus raíces, luego:

a: a3 – 3a2 + a – 6 = 0
b: b3 – 3b2 + b – 6 = 0
γ: γ3 – 3γ2 + γ – 6 = 0
Sumando:

a3 + b3 + γ3 – 3 (a2 + b2 + γ2) + (a + b + γ) – 18 = 0

4 3
a3 + b3 + γ3 – 12 + 3 – 18 = 0
Luego:

a3 + b3 + γ3 = 27

288 Polinomios y Teoría de ecuaciones

288-289. 288 20/11/02, 1:05 PM


CAPÍTULO 5

c) Sabemos que:
αβγ ( α1 + β1 + 1γ ) = βγ + αγ + αβ
6 –1

6 ( α1 + β1 + 1γ ) = – 1
luego: 1 1 1 1
+ + =–
α β γ 6

Ejercicios
1. Dados los polinomios y los números 4. Factorice los siguientes polinomios:
siguientes, determine cuál de ellos es
(o son) raíz(ces). a) x4 + 10x3 + 35x2 + 50x + 24

a) x3 – 2x2 – 5x + 6 –2, 3, 0 b) x4 + 2x3 – 7x2 – 8x + 12

b) x2 + 3x – 10 2, 3, 5 c) 4x4 – 28x3 + 47x2 + 7x – 12

c) x4 + x3 – 7x2 – x + 6 2, –3, 1 d) x5 + x4 + 5x2 – x – 6

d) x4 + x3 – 6x2 – x + 5 1, 2, –1 e) 2x4 – 5x3 – 20x2 – 22x – 15


f) x3 – 6x2 + 8x – 3
e) x4 – 1 1, –1, 2
2. Escriba un polinomio con coeficientes g) 36x4 – 13x2 + 1
enteros cuyas raíces sean: h) x4 + 2x3 – 3x2 – 4x + 4
a) 2, 3, 1 i) x4 – 5x3 + 6x2 + 4x – 8
b) 3, –1, 2 de multiplicidad 2 j) –x5 + 3x4 – 5x3 + 7x2 – 6x + 2
c) 1 de multiplicidad 2 y –1 de 5. Encuentre las soluciones de la ecuación
x3 – 7x + 6 = 0, sabiendo que el producto
multiplicidad 3
de dos de ellas es 2.
d) 2, – 2, 5
6. Resuelva la ecuación
e) 1 + 2, 1 – 2, –1 x3 – 19x2 + 114x – 216 = 0, sabiendo que
1 el cuadrado de una de sus raíces es igual
f) 0, 2
, 2 de multiplicidad 3
al producto de las otras dos.
g) 3 – 2i, 5, 3 + 2i
7. Resuelva la ecuación
3. Determine las raíces racionales de las
2x3 – x2 – 18x + 9 = 0, sabiendo que la
siguientes ecuaciones:
suma de dos de sus raíces es cero.
a) x4 – 6x2 + 7x – 6 = 0
8. Resuelva la ecuación
b) 2x5 + 7x4 + 6x3 + 3x2 + 4x – 4 = 0 x3 – 4x2 – 17x + 60 = 0, sabiendo que la
c) x4 – 8x2 – 4x + 3 = 0 suma de dos de sus raíces es 1.
d) 3x4 – 7x3 + 5x2 – 7x + 2 = 0 9. Resuelva la ecuación
e) 2x4 + x3 – 8x2 – x + 6 = 0 x3 – 4x2 – 11x + 30 = 0, sabiendo que
la suma de dos de sus raíces es igual
f) x3 + x2 – 5x + 3 = 0 a la tercera.

Polinomios y Teoría de ecuaciones 289

288-289. 289 20/11/02, 1:06 PM


Ejercicios
19. Determine el valor de k en la ecuación
10. Resuelva la ecuación x3 –11x2 + 38x + k = 0, para que una
x3 – 15x2 + 71x –105 = 0, sabiendo que solución sea el doble de otra.
una de sus raíces es la mitad de la suma
20. Determine el valor de k en la ecuación
de las otras dos.
x3 – 3x2 + k = 0, para que tenga una
11. Encuentre las soluciones de la ecuación raíz de multiplicidad 2.
x3 – 6x2 – 37x + 90 = 0, sabiendo que
una de sus raíces es dos unidades menor 21. Determine el valor de k en la ecuación
que la suma de las otras dos. x3 – 2x2 – 4x + k = 0, para que tenga
dos raíces opuestas.
12. Resuelva la ecuación
x3 – 17x2 + 82x –120 = 0, sabiendo que 22. Pruebe que si a + bi es raíz de un
el producto de dos de sus raíces es 2 polinomio P(x), entonces (x – a)2 + b2
unidades mayor que la tercera y que las divide a ese polinomio.
tres raíces son racionales.
23. Sabiendo que 2 – i es raíz del polinomio
13. Si a, b y g son las raíces de la ecuación
x3 – 7x2 + 15x – 9, encuentre un
x3 – 2x2 – 6x – 8 = 0, encuentre el
polinomio de grado 2 que lo divida
valor de:
(que sea factor). (Aplique ejercicio
a) a + b + g anterior.)
b) a b + a g + b g
c) a b g 24. Si 1 + 3i es raíz del polinomio
P(x) = x4 – x3 + 6x2 + 14x – 20,
d) a2 + b2 + g2
encuentre un polinomio primo de
e) a3 + g3
+ b3
segundo grado que lo divida (Aplique
1 1 1 ejercicio 22).
f) + +
α β γ
NOTA: Polinomio primo es aquel no
g) 1 + 1 + 1 factorizable en R.
αβ αγ βγ
14. Encuentre la suma y el producto de las
25. Si i es raíz del polinomio
raíces de la ecuación
P(x) = x5 – 4x4 + 7x3 – 8x2 + 6x – 4,
3x3 – 4x2 + x – 6 = 0
factorice este polinomio en sus factores
15. Encuentre la suma y el producto de las primos. (Aplique ejercicio 22.)
raíces de la ecuación
5x5 – 2x4 + 3x + 20 = 0 26. Determine k en la ecuación
x3 – kx2 + 72x – 108 = 0
16. Determine la suma y el producto de las sabiendo que dos raíces son iguales y
raíces de la ecuación todas son enteras.
3x6 – 2x5 + x – 1 = 0
27. Determine la multiplicidad de la raíz r
17. Escriba la ecuación de menor grado
dada en la ecuación:
de coeficientes reales que tengan por
solución 1 + i, 2 y 5 a) x8 + 5x6 – 6x2 = 0; r = 0

18. Encuentre la ecuación de menor grado b) 2x5 – 17x4 + 51x3 – 69x2 + 47x – 14 = 0;
que tenga una raíz igual a 2 – 3 r=1

290 Polinomios y Teoría de ecuaciones

290-291. 290 20/11/02, 1:07 PM


CAPITULO 5
CAPÍTULO

Soluciones

1. a) –2, 3 b) 2 c) 2, –3, 1 d) 1, –1 e) 1, –1
2. a) x3 – 6x2 + 11x – 6 b) x4 – 6x3 + 9x2 + 4x – 12
c) x5 + x4 – 2x3 – 2x2 + x + 1 d) x3 – 5x2 – 2x + 10
e) x3 – x2 – 3x – 1 f) 2x5 – 13x4 + 30x3 – 28x2 + 8x
g) x3 – 11x2 + 43x – 65
1 1
3. a) 2, –3 b) , –2 de multiplicidad 2 c) – 1, 3 d) – 3, 2
2
3
e) –1, 1, –2, 2 f) –3, 1 de multiplicidad 2
4. a) (x + 1) (x + 2) (x + 3) (x + 4) b) (x – 1) (x + 2) (x + 3) (x – 2)
c) (x – 3) (x – 4) (2x + 1) (2x – 1) d) (x + 1) (x – 1) (x + 2) (x2 – x + 3)
e) (x – 5) (2x + 3) (x2 + x + 1) f) (x – 1) (x2 – 5x + 3)
g) (2x + 1) (2x – 1) (3x +1) (3x – 1) h) (x – 1)2 (x + 2)2
i) (x – 2)3 (x + 1) j) (x2 – 2) (1 – x)3
1
5. 1, 2 y –3 6. 4, 6 y 9 7. 3, –3 y 8. 5, 3 y – 4 9. 5, 2 y –3
2
10. 7, 5 y 3 11. 2, 9 y –5 12. 3, 4 y 10
1
13. a) 2 b) –6 c) 8 d) 16 e) 68 f) – 3 g) 4
4 2
14. Suma = producto = 2 15. Suma = producto = – 4
3 5
2 1
16. Suma = producto = – – 3 17. x – 7x4 + 10x3 + 4x2 – 24x + 20 = 0
5
3
18. x4 – 10x2 + 1 = 0 19. k = –40 20. k = 4 21. k = 8
23. x2 – 4x + 5 24. x2 – 2x + 10 25. (x2 – 2x + 2) (x2 + 1) (x – 2)
26. k = 15 27. a) 2 b) 1

Prueba de selección múltiple


1. Si P(x) = 4x5 – 6x2 – x + 3 C. 6
es un polinomio, su grado es: D. 4
A. –1 E. –1
B. 3
3. Dados P(x) = 4x2 – 5x + 3
C. 4
y Q(x) =2x3 – x2 + 5, entonces
D. 5
E. 6 P(x) + Q(x) es:

2. En el polinomio A. 6x5 – 6x3 + 8


Q(x) = 25x6 – 14x4 + x2 – 1 B. 2x4 + 3x3 – 5x + 8
el coeficiente principal es: C. 2x3 + 3x2 – 5x + 8
A. 25 D. 2x3 + 4x2 – 5x + 8
B. 14 E. 2x3 – 3x2 – 5x + 8

Polinomios y Teoría de ecuaciones 291

290-291. 291 20/11/02, 1:08 PM


Prueba de selección múltiple
4. Sean P(x) = 5x4 – 3x + 1 C. 80
y Q(x) = x4 – 3, entonces P(x) – Q(x) = D. 84
E. 152
A. 6x4 – 3x – 2
B. 4x4 – 3x + 4
9. Si Q(x) = 4x 4 – 3x 2 + 2, entonces
C. 4x4 – 3x – 2
Q( 2 ) es:
D. 4x4 + 3x – 2
E. 4x4 + 3x + 4 A. 10
B. 12
5. Si P(x) = x4 – x3 +3 C. 14
y Q(x) = x4 + x3 – 3, D. 16
entonces 2 P(x) – Q(x) es: E. 18

A. 2x4 10. Cuál (o cuáles) de las siguientes expresio-


B. –2x3 +6 nes es(son) polinomio(s) en R.
C. x4 – 3x3 +9 I) x–2 – x–1 + 1 II) 3x2 – x–1
D. x4 –3x3 – 9
1
III) x – 2x + 3
2

E. x4 – 3x3 +6 A. Sólo I
B. Sólo II
6. Si P(x) = 6x5 – x4 + 2x3 – x2 – 1, entonces
C. Sólo III
P(–1) es:
D. Todas
A. 1 E. Ninguna
B. –1
11. Al dividir x4 – 2x2 – 6 por x + 3, el
C. 11
resto es:
D. –11
E. 6 A. 69
B. 62
7. Si P(x) = x5 + 2x4 – x – 2 son soluciones C. 59
o raíces del polinomio D. 57
I) i II) –i III) –2 E. 54

A. Sólo I 12. Al dividir x5 – 6x4 – 2x3 – x + 1 por


B. Sólo II
x3 – 3x2 + 1 el cociente y el resto son
C. Sólo III
respectivamente:
D. Sólo I y II
E. I, II y III A. x2 + 3x – 11 y –34x2 – 2x + 12
B. x2 – 3x + 11 y –34x2 + 2x – 12
8. Si P(x) = 6x3 – x2 + 2x – 1, entonces
C. x2 – 3x – 11 y –34x2 + 2x + 12
P(3) es:
A. 156 D. x2 + 3x + 11 y –34x2 – 2x –12

B. 158 E. –x2 + 3x – 11 y 34x2 – 2x – 12

292 Polinomios y Teoría de ecuaciones

292-293. 292 20/11/02, 1:10 PM


CAPÍTULO 5

13. Cuál es el polinomio que dividido por 18. Para que x3 – ax – x + b sea divisible
x2 + 1 da como cociente x + 2 y por x2 + x – a debe ser:
resto x – 3. A. a = –b
A. x3 – 2x2 + 2x – 1 B. a=b
B. x3 + 2x2 + 2x – 1 C. a+b=1
C. x3 + 2x2 – 2x – 1 D. a–b=1
D. x3 + 2x2 + 2x + 1 E. a + b = –1
E. x3 – 2x2 + 2x + 1
19. Sea P(x) = x3 + 2x2 – ax + b – 3.
14. Al dividir x4 – 2x3 + 4x2 – x + 1 por x – Determinar la relación que debe cumplirse
2 el resto es: entre a y b para que P(x) + 6 sea divisible
A. 3 por x + 1.
B. 9
A. a+b–3=0
C. 15
B. a+b–4=0
D. 51
C. a+b+3=0
E. 61
D. a+b+4=0
15. ¿Cuál es el valor que debe tener k en el E. a–b+4=0
polinomio 4x5 – 2x3 + kx – 2 para que
sea divisible por x – 2? 20. Qué valores deben tener a y b para que
5x3 – 2x2 + ax – b sea divisible por x2 + 1.
A. 5
B. 25 A. 5 y 2
C. 50 B. 5 y –2
D. –50 C. –5 y 2
E. –55 D. –5 y –2
E. 4 y 2
16. ¿Qué valor debe tener k en el polinomio
6x3 – kx2 + x – 1 para que al dividirlo por 21. El polinomio de segundo grado tal que
x2 – 3, el resto sea 19x – 7? P(1)1 = 6, P(2) = 14 y P(–2) = 18 es:
A. –1
A. 3x2 – x + 4
B. 0
C. 1 B. 3x2 – x – 4
D. 2 C. 3x2 + x + 4
E. 3 D. –3x2 + x + 4

17. Qué valores deben tomar a y b para que E. –3x2 + x – 4


se verifique la igualdad
1 22. El polinomio con coeficientes reales de
2ax3 – bx2 + 1 = 2 (bx3 – 2x2 + )
2 menor grado cuyas raíces son 3, 4 y 5 es:
I x E R.
A. x3 – 12x2 + 47x + 60
A. a=4 b=–4
B. x3 – 12x2 – 47x – 60
B. a=2 b= 2
C. a=4 b= 4 C. x3 + 12x2 + 47x + 60
D. a=–4 b=–4 D. x3 + 12x2 – 47x – 60
E. a = –2 b = –2 E. x3 – 12x2 + 47x – 60

Polinomios y Teoría de ecuaciones 293

292-293. 293 20/11/02, 1:10 PM


Prueba de selección múltiple
23. El polinomio con coeficientes reales 27. El polinomio x3 – 7x – 6 es equivalente a:
de menor grado cuyas raíces son 1 y A. (x + 2) (x – 3) (x – 1)
2 es: B. (x + 2) (x – 3) (x + 1)
A. x3 – x2 + 2x + 2 C. (x – 2) (x + 3) (x + 1)
B. x3 – x2 – 2x – 2 D. (x – 2) (x + 3) (x – 1)
C. x3 + x2 + 2x – 2 E. (x + 2) (x + 3) (x + 1)
D. x3 – x2 – 2x + 2 28. Si las raíces de la ecuación
E. x3 + x2 + 2x + 2 x3 – 2x2 + x – 3 = 0 son
a, b y g, entonces el valor de
24. El polinomio con coeficientes reales de a+b+g=
menor grado cuyas raíces son
A. 1
1 – 3, i y 2 es: B. 2
C. 3
A. x5 + 4x4 + 3x3 + 2x + 4
D. –3
B. x5 – 4x4 – 3x3 + 2x + 4
E. –2
C. x5 – 4x4 + 3x3 – 2x + 4
D. x5 – 4x4 + 3x3 + 2x – 4 29. Si las raíces de la ecuación
E. x5 – 4x4 + 3x3 + 2x + 4 2x3 – 6x2 + 8x – 10 = 0
son a, b y g, entonces el valor de la
25. Las raíces racionales del polinomio expresión a2 + b2 + g2 es:
x5 – 2x4 + x – 4 son: A. 3
A. 2, –2 y 4 B. 4
B. 1, 2 y –2 C. 5
C. 1, 4 y – 4 D. 1
D. 1, –1 y 4 8
E.
E. no tiene 9

30. El valor de k en la ecuación


26. Las raíces racionales del polinomio
x5 – 4x4 – 5x3 + 20x2 + 4x – 16 son: x3 – kx2 + 48x – 36 = 0
para que ésta tenga una raíz de multipli-
A. 1, –1, 2, –2 y 4 cidad 2 es:
B. 1, –1, 2, 4 y –4 A. 10
C. 1, –1, –2, 4 y –4 B. 12
C. 13
D. 1, 2, –2, 4 y –4
D. 24
E. –1, 2, –2, 4 y –4 E. 32

Soluciones

1. D 2. A 3. C 4. B 5. C 6. D 7. E 8. B
9. B 10. E 11. D 12. C 13. B 14. C 15. E 16. D
17. C 18. B 19. D 20. A 21. A 22. E 23. D 24. E
25. E 26. A 27. B 28. B 29. D 30. C

294 Polinomios y Teoría de ecuaciones

294. 294 20/11/02, 1:11 PM


CAPÍTULO 6
Potencias
y Raíces

Potencias 6.1

6.1.1. Potencias de exponente natural


Definimos an = a • a • a • • • • a (n veces)
La expresión an se llama potencia n-ésima de a.
“a” es la base de la potencia.
“n” es el exponente de la potencia.

6.1.2. Potencias de exponente cero


y exponente entero negativo
De las propiedades que estudiaremos más adelante se deduce que:
a0 = 1 para todo valor de “a”. (con a π 0)
1
a–n = para todo valor de a : a π 0 ; n ∈ N
an

1. Calculemos el valor de (–2)3


Ejercicios
Aplicando la definición tenemos:
resueltos
(–2)3 = (–2) • (–2) • (–2) = –8
2. Calculemos el valor de –34
Observamos aquí que la base de la potencia es 3 ( y no –3),
expresándola en forma de producto nos queda:
–34 = –3 • 3 • 3 • 3 = –81
3. Calculemos (–3)4
Aquí la base es (–3) y por lo tanto:
(–3)4 = (–3) (–3) (–3) (–3) = 81
4. Calculemos 2 – 5

Potencias y raíces 295

295.(2003) 295 20/11/02, 1:13 PM


Ejercicios Aplicando la definición (exponente negativo):
resueltos 1 1 1
2–5 = = =
25 2 • 2 • 2 • 2 • 2 32
0
5
1 17 –7
5. Calculemos 2 • – • –3
2 4
Como el exponente es cero ( y la base es distinta de cero ), aquí no es
necesario hacer ningún cálculo. El valor es 1.

Ejercicios
I. Aplique la definición de potencias para calcular :
4 4
3 1
1. 22 13. (–11)2 25. (3,5)0 35. 43. – 1
4 2
2. 32 14. (–5)4 26. (–1,7)0 –3 6 3
36.
4 44. 2 5
3. 52 15. (3)5 7
27. (2,1)1 3 2
3
4. (–2)3 16. (–3)5 37. 3
5 45. –3
28. –(0,8)2 4
5. 33 17. 63 11 4
38. 46. (– 0,27)1
6. 42 18. (–6)3 29. (–0,8)2 2
2
7. 101 19. 25 30. (2,5)3 2
39. 3 47. (0,08)2
3
8. 122 20. (–2)5 2 3
4 3
31. 1 1
9. (–3)2 21. (0,5)2
3 40. 1 48. –1
3
5 16
10. (–3)3 22. (–1,1)3 32. 3 3 4
2 1 3
2 41. 2 49. –2
3
11. (8)3 23. (0,3)3 33. 2 4
5 2
4 2 3
12. (–12)3 24. (–2)2 34. – 3 42. 3 1
5 50. –1
4 4

II. Aplique la definición para calcular :


–1 –4
3 1
1. 2 –1 9. (–1)–5 17. 22. 1 28. (0,06) –1
4 3
2. 2 –2 10. (–1)–6 –2 29. – (0,04)–2
2 23. –(– 0,25)0
18. –1
3. (–2)–2 11. (–0,5)– 4 3 3
30. – – 2 4
24. (–0,25)0
4. – (–2)–2 12. (0,5)–3 –3 –2 –2
19. 4 –2 31. – 1 1
5. (–5)– 3 2 15
13. (1,7)– 2 25. – 5
3
3 –2 32. –(–0,06)– 1
6. (5)– 4 14. – (0,2)– 2 20. – –4
2 26. 3 1 3 –2

7. – (5)–3 15. – (2,1)–3


4 33. – 3
–3 –3 5
3
21. 2 27. 2 4
8. 2 –3 16. (2,05)0 4 5 34. – (0,71)–2

296 Potencias y raíces

296-297. 296 08/11/2001, 10:45


CAPÍTULO 6

1 0
35. – – 1 1 –2 37. (– 0,04) – 1 39. –6
2 16
3 –6 3 0
36. (0,04) – 1 38. 2 40. 2
2 5

III. Calcule el valor de: 0


1 1 2
1 3 4 1
1– + – 31. 3 +2
1. 3 + 32 18.
2 2 2 3 2

2. 23 – 22 2
+
3
19. 1 – 2–1 + 2–2 – 2–3 32.
3–2 2–1
3. 20 + 21 + 22 + 23 –1 –2 3
3 3 4 0
20. – + 2 1
2
4. 3 • 35 4 4 3 33. –
5 7
5. (2 • –3)2 21. (0,02)2 + (0,02)–2 –1 –2
1 1
34. 3 + 2
6. 2 • (–3)2 –1 –1 2 2
1 2
22. +
7. – (–2)2 + (–3)2 4 3 –1
5 –1
2
3 2
3 2 35. +
23. + – 5 2
8. (0,2)2 – (0,1)2 2 2
0 –1
1 1
9. –5 • (–3)2 3 2
3 2
36. + + 2–1
24. – – 2 2
2 2
10. –5 • (–3)3 –1 –2 –1 1
–2
1
2
2 3 1 37. 3–2 + +
25. + 2 – 1 3 3
11. (–5)2 • (–2)3 3 2 5
–3
4 7 3
12. (–3)1 + (–3)3 26. – 2 3 2
7 4 1 1 1
38. 1+ – +
13. 3 • 43 3 4
2 4 2 2 2
27. •
2 3 0
14. 23 • 32 • 41 –3
1 1 1
28. 3–2 : 2–3 39. 3 – 2
7
+2
15. 26 • 32 – 25 • 32 – 1 0
–3 5 5
–1 –2
29. 4 1
16. (12)–1 + (–12)–1 3
5
+
2 –1 –2 3
2 5 2
3 –3
4 3
2 –1
0 40. • •
17. – + 30. –2 5 2 5
3
4 3 3 11
5

IV. Determine el valor numérico de las siguientes expresiones para los valores
de las variables indicadas:
1. x2 + y2 si x = 1 e y =2
2. x3 – 3y2 + 2x si x = 2 e y = –1
3. 2a2b – 3b2 si a = 3 y b =2
4. 5a – 5b si a = –2 y b = –3
5. 3 (a2 + 2ab + b2) si a = 4 y b =3
6. –2 (1 – x + x2 – x3) si x = –2
7. x –3 + y –3 si x = –1 e y =–2

Potencias y raíces 297

296-297. 297 08/11/2001, 10:46


Ejercicios
8. 2x – 4 – 3y – 5 si x = –1 e y = –2
9. a(2a2 – a–2) si a = –3
10. (2 + x – y)–2 si x = –2 e y = –3
11. (–3x2y4)–1 si x = –3 e y = –3
12. (3xy)–x + y –2 si x = –2 e y = –1
13. (22 + 33)–x si x = –3
14. (a– b + b– a)–1 si a = –2 y b =3
15. 2a(2a3 – 3a4)–2 si a = –2
16. (3x –2 + y –3 – 2y – 4)–1 si x = –2 e y = –1
17. 2a–b + a2 – b2 si a =5 y b =3
18. x – 4 + 4 –x si x = –1
19. 2x–1 – 3y–3 + xy si x = –1 e y = –1
20. x –2 + y –3 si x = –2 e y = –3

Soluciones
I. 1. 4 2. 9 3. 25 4. –8 5. 27 6. 16 7. 10 8. 144 9. 9 10. –27 11. 512
12. –1.728 13. 121 14. 625 15. 243 16. –243 17. 216 18. –216 19. 32
20. –32 21. 0,25 22. –1,331 23. 0,027 24. 4 25. 1 26. 1 27. 2,1 28. –0,64
8 3 4 81 81 729
29. 0,64 30. 15,625 31. 27 32. 3 33.
35. 256 36. 4.096 34.
8 25 256
9 1 4 46 19 14 1 342
37. 38. 915 39. 13 40. 2 41.12 42. 11 43. 5 44. 19 343
25 16 9 625 27 25 16
47 817 49 61
45. –52 46. – 0,27 47. 0,0064 48. –1 49. 57 50. – 1
64 4.096 256 64

1 1 1 1 1 1
II. 1. 2. 3. 4 4. – 5. – 6. 625 7. – 1 8. 1 9. –1 10. 1
2 4 4 125 125 8
4
19. 4
100 1.000
11. 16 12. 8 13. 14. –25 15. – 16. 1 17. 18. 9
289 9.261 3 4 9
4 64 81 9 256 125
20. – 21. 22. 23. –1 24. 1 25. – 26. 27.
9 1.331 256 361 28.561 2.744
50 4 – 225 50 25 10.000 4
28. 29. – 625 30. 31. 256 32. 3 33. 324 34. – 35. –
3 11 5.041 9
64
36. 25 37. –25 38. 39. 1 40. 1
117.649

III. 1. 12 2. 4 3. 15 4. 729 5. 36 6. 18 7. 5 8. 0,03 9. – 45 10. 135


2 5 5
11. –200 12. –30 13. 192 14. 288 15. 287 16. 0 17. 18. 19.
3 8 8
52 25 1 1 110
20. =1 21. 2.500,0004 22. 5 23. 4 24. 0 25. 26. 0
27 27 2 2 147

298 Potencias y raíces

298-299. 298 08/11/2001, 10:53


CAPITULO 6
CAPÍTULO

8 78 9 1
27. 1 28. 29. 1 30. 1 31. 1 32. 24 33. 1 34. 35. 2 36. 3 2
9 175 10
2 57 16
37. 9 38. 1 39. 1 40.
9 64 625
4 1 3 1
IV. 1. 5 2. 9 3. 24 4. 5. 147 6. –30 7. – 1 8 8. 2 9. – 53 2 10.
125 32 3 9
1 8 1 4
11. – 12. 37 13. 29.791 14. 15. – 16. – 17. 20 18. 5
2.187 71 1.024 9
23
19. 2 20.
108

Propiedades de las potencias 6.2

6.2.1 Multiplicación de
potencias de igual base
Para multiplicar potencias de igual base mantenemos la base y
sumamos los exponentes, es decir:
an • am = am+n

6.2.2 División de potencias


de igual base
En este caso, mantenemos la base y restamos los exponentes,
es decir:
an : am = an–m

6.2.3 Elevación de potencia a potencia


Aquí debemos elevar la base a la multiplicación de los
exponentes.
(am)n = an • m

6.2.4 Multiplicación de potencias


de igual exponente
Elevamos el producto de las bases al exponente común.
an • bn = (ab)n
Potencias y raíces 299

298-299. 299 08/11/2001, 10:53


6.2.5 División de potencias
de igual exponente
Elevamos el cociente de las bases al exponente común.
n
an : bn = a
b ( )
Los recíprocos de las propiedades 6.2.4 y 6.2.5 también son
válidos, es decir:

6.2.6 Potencia de un producto


Se eleva cada factor del producto al exponente:
(a • b)n = an • bn

6.2.7 Potencia de un cociente


n
( )
a
b
n
= an
b

1. Expresemos en forma de potencias:


Ejercicios
1 1 1 1 1
resueltos – – – – –
2 2 2 2 2
Aquí tenemos el producto del término – 1 cinco veces (el
2
término se repite 5 veces).
Así es que :
5
1 1 1 1 1 1
– – – – – = –
2 2 2 2 2 2
2. Efectuemos los productos:
3 5 2 6
a • a • 15 a11
4 5
La multiplicación es una operación conmutativa, por lo tanto :
3 5 2 6• 3 2 3
a • a 15 a11 = • • 15a5 • a6 • a11
4 5 4 5
2

= 9 a22
2
3. Desarrollemos (a2 + a6)2
Se trata de un cuadrado de binomio, por lo tanto,
(a2 + a6)2 = a4 + 2a8 + a12
4. Efectuemos los productos indicados :
am – 3 • b2 • a4 • bn + 2
conmutamos los términos agrupando bases iguales y luego multi-
plicamos.

300 Potencias y raíces

300-301.(2003) 300 20/11/02, 1:23 PM


CAPÍTULO 6

am – 3 • a4 • b2 • bn + 2 = am + 1 • bn + 4
5. Efectuemos las operaciones:
25 • 5a +1 – 27 • 3a + 4
Observamos que el 25 es potencia de 5 (25 = 52) y que 27 es
potencia de 3 (27 = 33). Entonces:
25 • 5a +1 – 27 • 3a + 4 = 52 • 5a + 1 – 33 • 3a + 4
= 5a + 3 – 3a + 7
6. Multipliquemos: 162 • 4n + 2
Podemos expresar el 16 como potencia de 4; 16 = 42
162 • 4n + 2 = 44 • 4n + 2 = 4n + 6

Ejercicios

I. 19. 5 • 54
1. a3 • a5 = 20. 16 • 24 + a
2. x2 • x3 • x6 = 2 5 • – 3 9 • 10 12
21. p p p
3 5 7
3. – 6 a4 • – 5 a3 • – 2a8 22. 0,07a–3 • 0,5a–2 • 11,1a–1
4. (a – b2)4 • (a – b2)3
23. 4 m2 p • 3 m2 p2
5 5
5. 2 ab (a2 + b2)
24. 2 (a + b)7 • 5 (a + b)8 • – 4 (a + b)– 6
6. nk – 3 • n4 – k
25. 254 + p • 1253 – p
7. n– 2 • n– k • n3 + k
26. 9 • 3n – 2 • 3n + 1
8. 10 c8 • 0,25 c –4 • 2 c6
27. 125 • 5– 2 • 5 – 4
9. pn + 1 • pn – 2
28. 3c4 • 9c6 • 81c – 4
10. (2 a2 – 3b2)4
29. 2 • 4n • 83n
11. (a2 + a3 + a4) –2

30. 64 • 2 – 6 • 22
12. (1 + a + a2) • a6
1 2n – 2 • 1 2n – 3
31. p2n – 1 • p p
13. (na – 1 – 2na – 2 + n) • na + 3 2 4
32. am – 3 (am – 2 – a3 – m)
14. (y – 1 – y6 + y9) • 2 y2

15. (3an – 2 – 2an – 3) a3 33. 128 • 24n – 1


3 –n • 4 – 2n
16. (m6 + n6) (m6 – n6) 34. m– p m– p
4 5
17. 32 • 2k – 2 35. 0,4 • 4 – 1 + 0,3 • 3 – 2 + 0,1 • 10 – 3

18. 27 • 3m + 3 36. (105 + 106) 10 – 4

Potencias y raíces 301

300-301.(2003) 301 20/11/02, 1:24 PM


Ejercicios

3 n– 5 3 5– n
37. ax + 1 • ax – 1 • a2x 39. p • p
4 5
– 3 –7 – 2 –1
38. – 12n8 • n • n 40. – 3an – 2 • bn – 3 • 6a3 • b – 4
4 5

II.

1. x6 : x2
18. (u – 4 : 4u) : u – 6
2. a4 :a
19. (a – 2 : 3a4) : (a6 : a – 6)
3. m16 : m6 2 –2
2 –3

4. (2p – 3q)5 : (2p – 3q)3


20.
3
: 3
21. (ex – e – x) : ex
5. (216 : 24) : 28
22. – 3a2 : 6a3
6. a11 : (a3 : a5)
23. m6 – c : mc – 6
a+b 3
7. 24. x2n – 1 : xn – 1
a+b
8. x7 : x4 25. 82 – 3x : 2x + 2
6 5 26. a – 2x : ax
9. x5 + x4
x +x a –2
a –3

10. (a –8 –a – 3) :a – 11 27.
4
: 2
11. (p – a + p – 2a + p – 3a) : p – 4a
1– n n
28. a : a
bn b1 – n
12. (a : ab) : b
29. (m –a – m – b – m – c) : m
13. (abc : bc) : a
30. (a4 – b4) : (a2 – b2)
14. 81 : 3a – 3
31. (a6 – a5) : a5
–1 –2
2 3 32. (16a8 – 8a4 – 4a2) : 2a2
15.
3
: 2
16. a – 6 : a – 8 33. [a8 : (a4 : a2)] : a3

17. (2p – 2 q – 3) : 6p – 3 q – 5 34. [mp + 1 : 2mp + 2] : 2mp – 2

III.

5 5
1. 34 • 24 4. 2 p •
3
q 7. (2x + y)3 • (2x – y)3
3 2
am • bm –1 • (1,4) – 1
2. 5. (1,04) 8. (m + n)6 • (m – n)6
6–n –4
3. (– 2a)4x • (3b)4x 6. 36 – n • 1 9. 2rs –4 2
3 rs

302 Potencias y raíces

302-303. 302 20/11/02, 1:26 PM


CAPÍTULO 6

25– 3
10. (0,2)5 • 105 17. 0,6 a – 4 • 0,2 b – 4 24.
75– 3
4 4
11. 2 •
3
18. 27 : 37 (3m)a
25.
3 4 ma
–1 162
12. – 3a2 2 –1
19. 63
• 82 26.
2 a2 6a 3
35
13. 2 –6 :3 –6 20. 5
27.
2 ay
9
x x 6 ay
a 2y 212
14.
y
• 21.
492
28. (a2 – b2)2 : (a – b)2
3a
4 4
15. (b2 – 4ac)2 • (b – 2)2 22. 2 : 4 29. – 5p – 2 : – 6q – 2
3 9
2a + b
6x – 3y
16. (a – 3b) –3 • (a + 3b) – 3 23. 3,2 – 2 : 1,6 – 2 30. 2a + b
36x2 – 9y2

IV.
01. (22)3 02. (32)3 03. [(–2)2]4

04. ((– 6)3) –1 05. [(8,5)–1]–2 06. (2a2b)3


1
07. (mamb) ab 08. (2–3)–2 09. (1002)–1 • 104
10. 105 : (104 : 102) 11. [(a – b)2 : (a – b)]–1 12. (x2)m

13. (x2 y – 3) –1 • x2 y – 2 14. [px + 2 • qx + 2]


–3
3 –1 3 –2
15. a • a 16. (a – a2 + a3 – a4) • a–1
2 2
17. (a – 1 + a – 2 + a – 3 + a – 4) : a – 5 18. [(– z – 4) : (– z)4]3

19. [125 x6 : (25x3 : 5x)] – 2 20. (5a3) –3 : (5a3)– 4


21. [(x + y) : (x2 – y2)] – 2 22. [(a2 – b2) : (a – b)] – 1

23. [(– 0,117)0 : (– 3,15)2] – 1 24. [(0,03)2 • (0,3)2] – 1

25. 9x5 : [2x : x5]–3 26. [16a3 : 4a2] –2

27. [ax + 1 • bx – 2]3 – x • (ab)x 28. [xu + yv] – 1


–1
2 2
29. [a6 : b5] – 2 30. –
2 • –
3

20
5 5 3

Soluciones

I. 1. a8 2. x11 3. – 60 a15 4. (a – b2)7 5. 2a3b + 2ab3 6. n 7. n 8. 5 • c10


1
9. p2n - 1 10. 16a8 – 96a6 b2 + 216a4 b4 – 216a2 b6 + 81 • b8 11.
a4 + 2 a5 + 3 a6 + 2 a7 + a8
12. a6 + a7 + a8 13. n2a + 2 – 2n2a + 1 + na + 4 14. 2y – 2y8 + 2y11 15. 3an + 1 – 2an
4
16. m12 – n12 17. 2k + 3 18. 3m + 6 19. 55 20. 28 + a 21. – p26 22. 0, 3885 • a–6
7

Potencias y raíces 303

302-303. 303 20/11/02, 1:28 PM


Soluciones

12 4
23. m • p3 24. – 40 (a + b)9 25. 517 – p 26. 32n + 1 27. 5 – 3 28. 37 c6
25
3
29. 211n + 1 30. 22 31. 1 p6n – 6 32. a2m – 5 –1 33. 24n + 6 34. m– p – 3n
8 5
18 9
35. 0,13343 36. 10 + 102 37. a4x 38. – 39. 40. –18 • an + 1 • bn – 7
5 20

II. 1. x4 2. a3 3. m10 4. (2p – 3q)2 5. 24 6. a13 7. (a + b)2 8. x3 9. x


3
3 1
10. a3 – a8 11. p3a + p2a + pa 12. b – 2 13. 1 14. 37-a 15. 2 16. a2 17. • p • q2
3
1 1
18. 4 u 19. a– 18 20. 2 21. 1 – e– 2 x 22. – 1 a– 1 23. m12 – 2c 24. xn 25. 24 – 10x
3 3 2
26. a – 3x 27. 2a 28. a1 – 2n • b1 – 2n 29. m – a – 1 – m – b – 1 – m – c – 1 30. a2 + b2

31. a – 1 32. 8 a6 – 4 a2 – 2 33. a3 34. 2 – 2 • m1 – p

III 1. 64 2. (ab)m 3. (– 6 ab)4x 4. (p •q)5 5. (1, 456) – 1 6. 1 7. (4x2 – y2)3 8. (m2 – n2)6
6 x
3 2
9. 4 –4 10. 25 11. 2 – 4 12. – 1 13. 14. 15. (b3 – 2b2 – 4abc + 8ac)2
3 2 3
7 2

()
2 3 3 4
16. (a2 – 9b2) – 3 17. 0,12 (ab) – 4 18. 19. 4 20. 3 – 5 21. 22.
3 7 2
2
1 5 q
23. 2 – 2 24. 33 25. 3a 26. a – 3 27. 28. (a + b)2 29. 30. (6x + 3y) – 2a – b
3 6 p
1 1
IV 1. 26 2. 36 3. (– 2)8 4. (– 6) – 3 5. (8,5)2 6. 8a6 • b3 7. mb • na 8. 26
3
9. 1 10. 103 11. (a – b) – 1 12. x2m 13. y 14. (pq)x + 2 15. 4 a9 16. 1 – a + a2 – a3
9
17. a4 + a3 + a2 + a 18. – z – 24 19. 5 – 4 • x – 8 20. 5 a3 21. (x – y)2 22. (a + b) –1

23. (– 3, 15)2 24. 12.345,679 25. 72x – 7 26. (4a) – 2


2 + 3x + 3 • b –x2 + 6x –6
1 125
27. a–x 28. xu + yv 29. a – 12 • b10 30. –
48

6.3 Ecuaciones exponenciales


Son aquellas ecuaciones que presentan las variables en el
exponente. Para resolverlas aplicamos la siguiente propiedad
de potencias: ax = ay ↔ x = y, para todo a, a ≠ 0, a ≠ 1.
Si en una ecuación no resulta posible igualar las bases, la
solución se obtiene aplicando LOGARITMOS, tema que no
abordaremos en este capítulo.

Ejercicios 1. Resolvamos la ecuación 2x = 4.


resueltos Debemos expresar ambos miembros de la igualdad como potencias
de la misma base; en este caso claramente la base es 2.

304 Potencias y raíces

304-305. 304 08/11/2001, 11:15


CAPÍTULO 6

Así tenemos: 2x = 4 Q 2x = 22
y aplicando la propiedad indicada al comienzo obtenemos la
solución x = 2.
2. Resolvamos la ecuación 3x + 2 = 27.
Procediendo como en el ejercicio anterior tenemos:
3x + 2 = 33 Q x + 2 = 3 Q x = 1
3. Resolvamos la ecuación 16x = 32
Aquí, tanto el 16 como el 32 son potencias de 2. Entonces:
16x = 32 Q (24)x = 25 Q 24x • 25
Q 4x = 5

Q x= 5
4
4. Resolvamos la ecuación 7x – 3 = 1
Recordemos la propiedad de potencias: a0 = 1, es decir, podemos
representar el 1 como una potencia de cualquier base (distinta de
cero) con exponente cero.
Así: 7x – 3 = 1 Q 7x – 3 = 70 Q x – 3 = 0
Qx=3

Ejercicios
I.
13. 5x + 4 = 125x – 4
1. 2x = 16
14. a2x – 1 = a2
2. 2x-5 = 32
15. mx – 3 = (m2)2x
3. 36 – x = 27x – 2
16. (3a)2x – 5 = 9a2
4. 32x – 2 = 2
17. (p – 3)5x = p2 – 6p + 9
5. 642x – 5 = 16x – 2
18. (a2 + 2ab + b2)2 = (a + b)2x
6. 125x – 3 = 25x – 3
19. (1 + a2)x = 1 + a2
7. 3 • 27x – 2 = 9x
20. (a2 + 2a – 9)x – 3 = 1
8. 5x – 3 = 1
21. 81x – 6 = 3x – 4
9. 162x – 4 = 1
10. 2x + 1 = 8 22. 322x – 3 = 2x + 3

11. 2x + 1 = 16 23. 125y + 2 = 52y

12. 2x + 1 = 128 24. 256y = 4 • 42y – 3

Potencias y raíces 305

304-305. 305 08/11/2001, 11:15


Ejercicios
25. m 2 y – 5 = (m5)y + 4 37. ux – 5y = u0
v3y = 1
26. (2p)6y + 1 = 16p4
x–2
2 4
y 38. =
27. 2x + 1 =4 3 9
y
22x = 4 + 1 2x – 3
4 5
39. =
5 4
28. ax + 1 = a2
x+y 2x
b =1 1 9
40. 2 =
3 49
29. m4x – 2 = m2 x–6
px + y = p3 1 216
41. 1 =
5 125
30. u2x – 1 = u5 42. (0,1)2x – 4 = 10
v2y = vx
43. (0,001)x – 4 = (0,01)x
31. mx = my + 1
n2x = ny – 3 44. (0,5)x – 6 = 1
16
2x – 3 0
32. 53x – 2 = 125
y
45. 2 3
=
7 4
32x – 2 = 16
46. (0,0001)2x = 0,1
33. (a + b)6x = (a2 + 2ab + b2)y 47. ax • ay = a4
(2a)x – y = 32a5 xy = 3

34. 2x – y = 2 48. m2x + 1 •m2y + 1 = m18


16y = 32x mx : my = m

49. 23x • 22x – y = 16


35. 25x – 3 = 1
32x + y = 81
px – y = p2
1
50. 128 x = 1
36. m2y = m4
mx + y = m4

Soluciones
11 11
I. 1. x = 4 2. x = 10 3. x = 3 4. 5. 6. x = 3 7. x = 5
5 4
8. x = 3 9. x = 2 10. x = 2 11. x = 3 12. x = 6 13. x = 8
3 7 2
14. x = 15. x = – 1 16. x = 17. x = 18. x = 2 19. x = 1
2 2 5
20 – 25
20. x = 3 21. x = 22. x = 2 23. y = – 6 24. y = – 1 25. y =
3 3
1
26. y = 27. x = 3 ; y=2
2

306 Potencias y raíces

306-307 306 08/11/2001, 11:24


CAPÍTULO 6

28. x = 1 ; y = –1 29. x = 1 ; y = 2 30. x = 3 ; y = 3


2
14 32 –5 – 15
31. x = – 4 ; y = –5 32. x = ; y= 33. x = ; y=
5 15 2 2
34. x = – 4 ; y = –5 35. x = 3 ; y = 1 36. x = 2 ; y = 2

37. x = 0 ; y = 0 38. x = 4 39. x = 1 40. x = –1 41. x = 9


3
42. x = 43. x = 12 44. x = 10 45. x = 3 46. x = 1
2 2 8
47. (x = 3 ∧ y = 1) V (x = 1 ∧ y = 3) 48. x = 9 ; y = 7
2 2
49. x = 8 ; y = 12 50. No hay solución en R.
7 7

Raíces 6.4
n
Definición: a = b ↔ bn = a
“n” es el índice de la raíz
“a” es la cantidad subradical

Observaciones:
n
1. Si a > 0 y n es par, entonces a representa un número real, es
decir, n
a ∈ R.
2. Si a < 0 y n es par, entonces n a representa un número complejo,
conjunto que estudiaremos más adelante.
Es decir, a < 0 y n es par → n a ∉ R.
3. Las operaciones definidas para las raíces verifican las propiedades
que se cumplen en los números reales (R).

Propiedades 6.5

6.5.1 Potencia de
exponente fraccionario
Toda potencia de exponente fraccionario se puede expresar como
raíz cuyo índice es el denominador del exponente
m
a n = n am
Potencias y raíces 307

306-307 307 08/11/2001, 11:24


6.5.2. Multiplicación de raíces
de igual índice
Multiplicamos las cantidades subradicales y conservamos el
índice.
n n n
a • b = ab

6.5.3. División de raíces


de igual índice
Dividimos las cantidades subradicales y conservamos el índice.
n n n
a : b= a :b

6.5.4. Raíz de una raíz


Conservamos la cantidad subradical y multiplicamos los
índices.
m n
a = mn a

Ejercicios 1. Determinemos el valor de


3
216
resueltos Aplicando la definición tenemos:

3
216 = 6, ya que 63 = 216
5
2. Expresemos la raíz m3 como potencia de exponente fraccionario.
Aplicando directamente la propiedad tenemos:

3
5
m3 = m5
3 3
3. Obtengamos el siguiente producto: 15a • 5a
Se trata de multiplicación de raíces de igual índice;

3 3 3
15a • 5a = 75a2

4. Obtengamos la siguiente división 18 5 m4 : 3 5 m


Se trata de división de raíces de igual índice;

18 5 m4
5
= 6 5 m3
3 m

308 Potencias y raíces

308-309. 308 08/11/2001, 11:29


CAPÍTULO 6

5. Simplifiquemos 75a3b 4
Aplicando las propiedades tenemos:

75a3b4 = 25 • 3 •a2 • a •b2 • b2 = 5ab2 3a

5 3
6. Expresemos en forma de una sola raíz, x2y

Podemos directamente multiplicar los índices; nos queda:


5 3 15
x2y = x2y

3
7. Expresemos como una sola raíz a b
Aquí es necesario introducir el término “a” dentro de la raíz cuadrada
antes de multiplicar los índices. Así:
3 3 6
a b = a2 b = a2 b

8. Realicemos las operaciones siguientes:


2 +3 3 +4 2 –5 3

Recordemos que sólo podemos sumar o restar raíces que tengan


el mismo índice y la misma cantidad subradical; nos queda
entonces:
2 +3 3 +4 2 – 5 3= 2 +4 2 +3 3 – 5 3
=5 2 – 2 3

Ejercicios

I. Determine el valor de:


4 3
1. 4 8. 100 15. 256 23. 8.000
4
3 16. 81 5 1
2. 25 9. 8 24. –
32
3 1
3 17. 8 5
3. 64 10. –27 25. 1
5
3
18. – 32 26. 9,61
3
4. 64 11. –216 3 1 121
19. 27 27. 196
3 3
5. 1.000 12. 0,001 81
20. 49 28. 0,09
3
6. 121 13. –125 3 29. 0,16
21. –512
4 4 16
7. 196 14. 625 22. 841 30. 81

Potencias y raíces 309

308-309. 309 08/11/2001, 11:31


Ejercicios

II.
1. Escriba los cuadrados de los números naturales del 1 al 20.
2. Escriba los cubos de los números naturales del 1 al 30.
3. Exprese los números naturales del 1 al 20 como raíces cuadradas.
4. Exprese los números naturales del 1 al 10 como raíces cúbicas.

III. Exprese las siguientes potencias como raíces:


3 3
4
1 2a 7
2
1. a 17. mn2 3 12. 2 16. 23
3b
1
2 2 1
2. m 18. 3pq 5 1 3
5 6 17. 5a2bc5
4 13.
3. 3
5
3
2a
p
1 19. 5a2 4
18. 4m2n q
6 1
4. 2
14. 1
3
1 y
3
4 10. m n 6 7 8 2 3z z
5. p 19.
2y
1 1
2 p
1 2 2
6. 11.
a 5 15. 1 20.
4m q
2 3 5n6
b

IV. Exprese las siguientes raíces como potencias de exponente fraccionario.


6
z 5t
1. a3 5. 5a7 19.
5
x2y6 13. 3u
11 x p+q p
2. 5m 6. p10 10. xy 14. 2a
4 n ab
3. 2p2 3 7. 2m4 11. 81 15. 3xy 2a
p 2a
6
4. 2x5 8. 3p6q3 12. 5 16. b
5a2 a

V. Aplique la definición para despejar la incógnita indicada en cada caso:

4
1. a =5 (a) 6. a+1 = b2 (a) 11. x–2 =2 (x)
3 4
2. z =2 (z) 7. 2t + 5 =3 (t) 12. a– 1 =4 (a)
5 n 1 1
3. m =n (m) 8. 2x + 1 = m (x) 13. = (a)
2a 2
3 x
4. 3p + 1 = 4 (p) 9. y–1 =4 (y) 14. 16 a = n (a)

2n = 1 (n)
n
5. 10. 2p + q = 3q (p) 15. p+2 = 2p (p)
2

310 Potencias y raíces

310-311.(2003) 310 20/11/02, 1:34 PM


CAPÍTULO 6

16.
3
18.
6
2a – 1 = b (a) 20. 1 (x)
t+5 =3 (t) x –1 =
6
5
17. a–2 =b (a) 19. a– 2 =1 (a)

VI. Señale qué condición se debe cumplir en cada caso para que las expresiones
representen números reales

1. x –1 2. x+1 3. 2a + 1 4. a–3

5. 1 – 2x 6. 3 – 2x 7. a2 – 1 8. x2 – 25

1
9. 9 – y2 10. 2x 11. x 12. 1+ a • 1– a

1 x–1 –1
13. 2 – x2 14. 15. 16. x
x–1 x+1

–6 x+4
17. – 2a2 18. 19. 20. 1 + a2
6– a x+1

VII. Señale a qué conjunto pertenecen las siguientes raíces.


(R : números reales; k : números complejos)
3 3
1. 2 2. 2 3. –2 4. –3

3 4 1 4 1
5. –3 6. 7. – 8. 1 + 2
3 3
3 3 3
9. 2+ 3 10. 2+ –3 11. 2 + –3 12. 6 + 3 + – 3

3 4 3 3 3 3
13. 2 – 3 14. 1 + 2 5. –1– 1 16. –3+ –2

VIII. Simplifique las siguientes expresiones:

6 5
1. 9b2 6. p6 q12 r18 11. m20 n15 t 10

4 6 64 a6
2. 16x2 7. 81 m4 n12 12. +a
b12
7
3. 25a2 b2 c2 8. a21 b7 c14 13.
1
– t4
t4
5
4. 81a4 b2 9. 32 m25 a8 a6
14. 6
+
b b8
3
5. 125x3 y6 10. x2 – 2x + 1

Potencias y raíces 311

310-311.(2003) 311 20/11/02, 1:34 PM


Ejercicios
a2 + 2ab + b2 a2 + 2a + 1
15. 4a2 + 4a + 1 17. a2 – 2a + 1

25a6 3
16. 18. x3 – 3x2 y + 3xy2 – y3
a2 – 12a + 36

IX. Ubique las siguientes raíces entre dos números enteros consecutivos:
3 4 3
1. 3 5. – 100 9. 112 13. 334

3 6 4
2. 5 6. 1.125 10. 1.156 14. 112

3 3
3. 20 7. 4.810 11. – 124

3 4 3
4. – 35 8. 34 12. – 1.149

X. Reduzca a términos semejantes:

1. 2 +2 2 +3 2 11. 3q a –2q b + 5q b – q a

2. 3 3 – 5 3 – 6 3 + 9 3 12. 3 7 + 2 28 – 6 63

3. 5 – 3 5 – 11 5 + 2 5 13. 11 2 + 3 8 + 13 12

4. 3 a – 4 a + 6 a – a 14. 3 3 – 22 75 – 5 27

5. 3a 2 + 2a 2 – a 2 15. 4 2 – 3 12 + 3 75 – 6 8

6. 3 p – 53 p + 23 p 16. 2 5 – 13 20 + 5 45 – 11 5

17. 3 28 – 2 20 + 5 80 – 4 63
7. 3 –2 2 +5 3 –6 2 + 2

18. 45 + 3 20 – 11 112
8. 4 6 – 3 5 – 5 6 + 2 5
4
19. a b – 3a b + 3 a2 b – a 4 b2
9. a – b –3 a – a –3 b
1 2 3 1
10.
n
p – 2 n p + 18 n p – 4n p 20. a– a+ a– a
2 5 4 3

XI. Efectúe las siguientes multiplicaciones:

1. 2• 3 3. 3a • 2a • 6
3
2. 3 • 12 4. 3x • 3 2x • 3 16x2

312 Potencias y raíces

312-313. 312 08/11/2001, 11:52


CAPÍTULO 6

2
4 4 4
5. 2p3 • 5p7 • 7p6 13. 1 + 2 + 3

6. a–1• a–1 14. 6– 2 • 6+ 2

7. 3a + 2 • 3a – 2 15. 16 – 3 • 16 + 3
2 2
3 3
8. 3x2 yz • 2x2 y2 z 16. 3– 5 + 3+ 5
2 2
9. 3x + 1 3x – 1 17. 1 + 2 2 + 2 3– 6
2 2
10. 2 + 2 2– 2 18. 3–1 + 3– 2
2 2
11. 1 + 5 – 1– 3 19. a + b a – b
2
12. 2 – 3 3 20. a– b • a+ b

XII. Efectúe las siguientes divisiones:

1. 18 : 2 11. 30 6a – 27 18a + 18 12a :6 6a

2. 125 : 5 12. x2 – 4 : x – 2 : x+2


3 3
3. 9a6 b12 : ab5 13. 96x3 : 24x

4.
4
x6 y2 z4 :4 xyz 14. 2x + 3 : 2x

n
5.
5
a2 + 5 a6 + 5 a9 : 5 a2 15. am + 6 : n am – 6
6.
3
x2 y :3 x y2 16. x2 – 8x + 7 : x – 7

7. 26a : 2a 17. a2 a2 – 121 : a a – 11

3x 3x
8. 3 128a4 : 6 64a2 18. 22a : 2a – 1

9. 444a3 : 111a 19. x 2 – 25 : x – 5

10. 12 20 – 18 15 :6 5 20. a2 – 6a + 9 : a – 3

XIII. Exprese en forma de una sola raíz los siguientes términos:

3 4
1. 3 2. 2 3. 5a 4. 2 2

Potencias y raíces 313

312-313. 313 08/11/2001, 11:54


Ejercicios
3 x x
5. 2 3 11. 4+ 2 • 4– 2 16.
5 4
3 2

4
6. a a 12.
2x 2x
17.
6
3 2
3 4
4
3 3x • 2 3 •

8 3 12
7. a a a 13. 15 : 5 18.
5
2 3•
4 10
3 2

3 4 4 3 3
8. 5 2 14. 3 5: 2 2 19. 2 3• 3 2

n m
a b 3
mn m
9. 15. 3 2 :6 5 20.

10. 3 3 3

XIV. Exprese las siguientes raíces con un índice común:

3 3 x y xy
1. 2 y 2 6. a+1 y a+1 11. 4 , 3 y 6

4 6 4 2a a2 a
2. 2 y 3 7. p+2 y p+1 12. 3n , 3m , mn

3 m n 8a 6a 12 a
3. a y a 8. a y a 13. m6 , m3 , m2

a a2 2a 2a 2b
4. 4 9. 14.
5 y 10 2 , 2 , 2 b, a

a b 4 3
5. 9 y 5 10. a , a , a

XV. Efectúe las siguientes operaciones:

1. 3 6 + 5 8 – 2 2 : 4 2 7. 3 – 2 2+ 3+ 2 2

2. 3 +5 3–5 2 8. 2 a a – 3 b a + ab a • 2 ab a

a+b 4
3. a – b 9. a2 + 2ab + b2 • a+b
a2 – 2ab + b2

4
4. a b + b a : ab 10. x2 – 2xy + y2 + x – y

3 3
5. a+b • a2 + 2ab + b2 11. 3
3a • 2a

4
6. 1 + 2 + 5 1 + 2 – 5 12. 3 2x • 2 3x

314 Potencias y raíces

314-315. 314 08/11/2001, 11:59


CAPÍTULO 6

3 3
13. 5 2 : 10 5 17. 5• 5+ 5

3 3
14. 4 4 a – 2 3 b : 6 ab 18. 3+ 3 3– 3

3
15. 3 6 2x + 2 4 3x – x •
3
x 19. 3
x2 – 2x + 1 • x – 1 • x–1

3 4 4
16. 2• 2• 2 20. 2 x + 2 • 3 x + 3

Soluciones
I. 1. 2 2. 5 3. 8 4. 4 5. 10 6. 11 7. 14 8. 10 9. 2 10. – 3
1
11. – 6 12. 0,1 13. –5 14. 5 15. 4 16. 3 17. 1 18. – 2 19. 3
2
9 11
20. 21. – 8 22. 29 23. 20 24. –1 25. 1 26. 3,1 27. 28. 0,3
7 2 14
2
29. 0,4 30.
3
II. 1. 1, 4, 9, 16, 25, 36, 49, 64, 81, 100, 121, 144, 169, 196, 225, 256, 289, 324, 361,
400.

2. 1, 8, 27, 64, 125, 216, 343, 512, 729, 1.000, 1.331, 1.728, 2.197, 2.744, 3.375, 4.096,
4.913, 5.832, 6.859, 8.000, 9.261, 10.648, 12.167, 13.824, 15.625, 17.576, 19.683,
21.952, 24.389, 27.000.

3. 1, 4 , 9, 16, 25 36, 49, 64, 81, 100, 121, 144, 169, 196, 225,

256, 289, 324, 361, 400.


3 3 3 3 3 3 3 3 3 3
4. 1 , 8 , 27 , 64 , 125 , 216 , 343 , 512 , 729 , 1.000

6 5 3
III. 1.
4
a3 5. 4
p3 09. 4
5a2 3
13. 2a 17. 5a2bc5

1 3 1 q
8 p
2. m 6. 2 10. m6 n7 14. 2 18. 4m2n
y
5 a 2
1 z 3z
3 2y
3. 5
3 4
7. m n2 11. b 15. 3 19.
3 p
7 2a q 4m
6 5 2 3
4. 2 8. 3pq 12. 3b2 16. 22 20. 5n6
3 1 3 1 5 1 7 10 1
IV. 01. a2 2. 5m 2 3. 2 p2 4 4. 2 6 x 6 5. 5 6 • a 6 6. p 11 7. 2 2 m2
2 6 1 1
1 3 1 1 1 2a p 5t z
08. 3 2 p3 q 2 9. x 5 y 5 10. x x y x 11. 81 n 12. 5 13. 3 v
p 2 a
p+q b
14. 2 a 15. 3xy 16. 5 a2 b

Potencias y raíces 315

314-315. 315 08/11/2001, 12:02


Soluciones
1
V. 1. a = 25 2. z = 8 3. m = n5 4. p = 21 5. n = 6. a = b4 – 1 7. t = 38
8
n
mn – 1 3q –q
8. x = 9. y = 4x + 1 10. p = 11. x = 18 12. a = 1 13. a = 2
2 2 16
n2 1 ± 33 b6 + 1
14. a = 15. p = 8
16. t = 22 17. a = b5 + 2 18. a =
16 2

19. a = ± 1 20. x = 36

1 3
VI. 1. x ≥ 1 2. x ≥ – 1 3. a ≥ – 4. a ≥ 3 5. x £ 1 6. x £
2 2 2
7. a E ] – • – 1 ] K [ 1, + • [ 8. x E] – •, – 5] K[5, + •] 9. y E [–3,3] 10. x ≥ 0
11. x > 0 12. a E [–1,1] 13. x ∈ – 2, 2 14. x > 1 15. x E ] – •, – 1[ K [ 1, + • [
16. x < 0 17. a = 0 18. a > 6 19. x E ] – •, – 4 [ K ] – 1, + • [ 20. I a E R

VII. 1. R 2. R 3. R 4. k 5. R 6. R 7. k 8. R 9. R 10. k
11. R 12. R 13. R 14. R 15. R 16. R

VIII. 1. 3b 2. 4x 3. 5abc 4. 9a2b 5. 5xy2

6. pq2r3 7. 3mn3 8. a3bc2 9. 2m5 10. x–1


2a 1 a4 a3 a+b
11. m4n3t2 12. +a 13. – t2 14. + 15.
b2 t2 b3 b4 2a + 1
5 a3 a+1
16. 17. a–1
18. x – y
a–6

I X. 1. 1 y 2 4. – 4 y – 3 7. 69 y 70 10. 3 y 4 13. 6 y 7
2. 2 y 3 5. – 5 y – 4 8. 2 y 3 11. – 5 y – 4 14. 3 y 4
3. 2 y 3 6. 10 y 11 9. 3 y 4 12. – 11 y – 10

X. 1. 6 2 6. – 2 3 p 11. 2q a + 3q b 16. – 20 5

2. 3 7. 6 3 – 7 2 12. – 11 7 17. 16 5 – 6 7

3. – 11 5 8. – 5 – 6 13. 17 2 + 26 3 18. 9 5 – 44 7

4. 4 a 9. – 3 a – 4 b 14. – 122 3 19. 0


31
5. 4a 2 10. 13 n p 15. 9 3 – 8 2 20. a
60

3 4
XI. 1. 6 2. 6 3. 6a 4. 96x4 5. 70 • p4
6. a – 1 7. 9a2 – 4 8. y 3 6x4 z2 9. 3x – 1

10. 2 11.2 + 2 3 + 2 5 12. 31 – 12 3

316 Potencias y raíces

316-317.(2003) 316 25/11/02, 9:46 AM


CAPITULO 6
CAPÍTULO

13. 6 + 2 2 + 2 3 + 2 6 14. 2 15. 253 16. 16

17. 27 – 8 2 18. 9 – 2 3 – 2 6 19. a2 – b 20. a–b

5 5 3
XII. 1. 3 2. 5 3. 3
9a5 b7 4. 4
x5 y z3 5. 1 + a4 + a7 6. x y –1

2 9
7. 13 8. • a 9. 2 • a 10. 4 – 3 3 11. 5 + 3 2 – 3 12. 1
2 2
n
13. 2x 14. 23 15. a12 16. x–1 17. a a + 11 18.
3x
2a+1
19. x+5 20. a–3

4 6 n• m 24 12
XIII. 1. 3 5. 12 9. am • b 13. 3 17. 72
8 45
6 8 4 20
2. 2 6. a3 10. 2.187 14. 32 18. 864
x
6 18
8 8 14 6
3. 5a 7. a7 11. 15. 5 19. 216
40 2n
4.
4
8 8.
6
250 12.
4x
81 • 22x • x 16. 18 20. mn + 1

6 6 mn mn
XIV. 1. 8 , 4 8. an , am

4 4 2a 2 2a 2 2a 2
2. 4 , 3 9. 22a , 4 , 2a
6 6 12 12 12
3. a2 , a3 10. a3 , a4 , a6

4 4 xy xy xy
4. 5 , 100 11. 4y , 3x , 6

2a 2 2a 2 2a 2
5. ab
9b ,
ab
5a 12. 3n a , 3m 2 , mn 2a

6 6 12a 12a 12a


6. a+1 3 , a+1 2 13. m9 , m6 , m2

12 12 2ab 2ab
7. p+2 2 , p+1 3 14. bb , aa

3
XV. 1. 2 + 4 3 2. 484 3. a+b 4. a + b 5. a + b 6. 2 2 – 2
4
7. 10 8. 4a3b – 6a2b2 + 2a3b2 9. a+b 3 10. 2 x – y

6 4 1 6 8 12 a 6 b
11. 108a5 12. 6 18x3 13. 14. 4 –2 a
2 25 b 2

6
15. 3 2x3 + 212 27x7 – 6
x5 16.
12
8.192 17. 5 + 3.125
6

3 4
18. 3 – 9 19. x–1 2 20. 6 x+2 x+3 2

Potencias y raíces 317

316-317.(2003) 317 25/11/02, 9:50 AM


6.6 Racionalización

Definición: El proceso de racionalización consiste en expresar


una fracción cuyo denominador es un término irracional,
es decir, tiene raíz irreductible, en otra fracción equivalente
cuyo denominador es un término racional, es decir, no
contiene raíz.

6.6.1 Técnicas de racionalización


Veremos aquí los casos más frecuentes de racionalización que
son:

a) Denominador irracional monomio:


A
n
, r<n
pr

n
En este caso amplificamos la fracción por: pn – r

n
A pn – r A n pn – r
y obtenemos: n

n
=
pr pn – r p

b) Denominador binomio (de índice 2).


A
a± b

En este caso la amplificación adecuada es por


±
a b

es decir, los mismos términos del binomio pero con la operación


opuesta. De este modo obtenemos del producto la diferencia de
cuadrados, con lo cual eliminamos las raíces:

A a– b A a– b
• =
a+ b a– b a–b

• Observación 1: Se pueden combinar ambas técnicas en algunos


casos.
• Observación 2: La segunda técnica se puede utilizar para casos
de sumas o diferencias de cubos, haciendo una
adecuada amplificación.

318 Potencias y raíces

318-319. 318 08/11/2001, 12:31


CAPÍTULO 6

a
1. Racionalicemos la expresión: 5
Ejercicios
b2
5 3
Amplificando por b obtenemos:
5 5
resueltos
a b3 a b3
5

5
=
b2 b3 b a–b
2. Racionalicemos la expresión: 7 3
a–b
7 4
Amplifiquemos por a–b
7 4 7 4
a–b a–b a–b a–b 7 4
• = = a–b
7
a–b 3 7
a–b 4 a–b
x–y
3. Racionalicemos la expresión: x+ y

Amplifiquemos por x – y:
x–y x– y x–y x– y
• = = x– y
x+ y x– y x–y
1
4. Racionalicemos la expresión:
2+ 3
En primer lugar amplifiquemos por 2 + 3 (caso de denominador
monomio).
1 2+ 3 2+ 3
• =
2+ 3 2+ 3 2+ 3

Ahora amplifiquemos por 2 – 3 (caso de denominador binomio).

2+ 3 2– 3 2+ 3 2– 3
• = = 2+ 3 2– 3
2+ 3 2– 3 4–3

Ejercicios
I. Racionalizar las siguientes expresiones fraccionarias:
2 a b
1. 2. 3 3. 4. 1 5. 6
2 3
3 b a 2 2 2 3
a mn ab a–b 2a2 b
6. 7. 5
8. 9. 10.
3 6 7
ab m2 n 7
a3 b2 a–b 5 6 a3 b3
6 3a 3
a– 2 2+ 3
11. 12. 13. 14.
2+ 2 a +3 2– 3
xy p–q
a–b 4
15. 16. x y – y x 17. ab–1 c–2 18.
p q–q p
a– b

19. 5a 20. m– n 21. 2


22. 3
6 1+ 2 + 3 2– 3+ 5
3 – 2a m– n

a 1 5 3 2
23. 24. 25. 26.
3
3
a+ b 2 2–3 3 3 5–5 3 4– 2

Potencias y raíces 319

318-319. 319 08/11/2001, 12:33


Ejercicios
27. Calcular el valor aproximado de la expresión x para x E N, 1< x < 10.
x
x+ 1
Observar los valores obtenidos. Comparar x con y determinar cuál
expresión es mayor. x x+ 1

x x+ 1
28. Demostrar que < I x E N.
x x+ 1
29. Calcular el valor aproximado de la expresión x para x E N, 1< x < 10.
x
x con x+ 1
Observar los valores obtenidos. Comparar y determinar cuál
x x +1
expresión es mayor.

x x+ 1
30. Demostrar que < I x E N.
x x+ 1

Soluciones
3 ab 2
1. 2 2. 9 3. 4. 4 5. 3
b
3
a2 b2 6 1 7
6. 07. 5
m3 n4 8.
7
a 4 • b5 9. a–b 10. 3 a 66a4b4
b
3
3a a – 3 a a –b a+ b
11. 6 – 3 2 12. 13. a 14. 7 + 4 3 15.
a–9 a2 – b
x y +y x 4
ab3 c2 p q +q p 5a 3– 2a • 3 + 2a
16. x–y 17. 18. pq 19.
bc 9 – 2a

6 5 1+ 2 – 3 • 2 3+ 5– 2 6
20. m– n • m+ n 21. 22.
2 4
3 3 3
a a2 – a • b +b a– b – 2 2 +3 3 – 3 5 +5 3
23. 24. 25.
19
a2 – b3 6
3 3
3 2 16 + 4 • 2 +2 4+ 8
26.
8
27. 1.414; 1.732; 2; 2.236; 2.449; 2.645; 2.828; 3; x < x+ 1
x x+ 1
28. Sug.: Racionalizar ambas expresiones
x x+ 1
29. 0.707; 0.577; 0.5; 0.447; 0.408; 0.377; 0.353; 0.3 ; >
1 x x+ 1
30. Sug.: 1 <
x2 (x + 1)2

6.7 Ecuaciones
irracionales

Definición: Son aquellas ecuaciones que presentan la


variable como cantidad subradical. Para resolverlas
debemos elevar a la potencia adecuada tantas veces sea
necesario hasta eliminar la raíz (o las raíces).

320 Potencias y raíces

320-321. 320 20/11/02, 1:51 PM


CAPÍTULO 6

1. Resolvamos la ecuación x+7 =5


Ejercicios
Vemos que elevando al cuadrado ambos miembros de la desigualdad
resueltos
la raíz se elimina.

x+7 =5 /( )2
x + 7 = 25 /–7
x = 18

2. Resolvamos la ecuación 1 + 5 + 3x + 4 = 2
Procedamos a elevar al cuadrado paso a paso:

1 + 5 + 3x + 4 = 2 /( )2

1+ 5 + 3x + 4 = 4 /–1

5 + 3x + 4 = 3 /( )2

5 + 3x + 4 = 9 /–5

3x + 4 = 4 /( )2

3x + 4 = 16 /– 4
1
3x = 12 /• 3
x = 4

3. Resolvamos la ecuación x +5 – x – 3 = 2
Elevamos al cuadrado ambos miembros de la igualdad.
Observamos que el primero es un binomio.

x+5 – x–3 = 2 /( )2

x+5– 2 x+5 x– 3 +x– 3 =4

2x + 2 – 2 x+5 x– 3 = 4
1
2x – 2 = 2 x+5 x– 3 /•
2
x–1 = x+5 x– 3 /( )2

x2 – 2x + 1 = x2 + 2x – 15 / –x2

– 2x + 1 = 2x –15

4x = 16

x =4

Potencias y raíces 321

320-321. 321 20/11/02, 1:52 PM


Ejercicios

I. Resuelva las siguientes ecuaciones:

1. x+3 =3 2. 2x – 7 = 13 3. 2 – 15x = 8

4. 1 + 2x = 2 5. 2 2x = 3 x 6. 1 + 7x = 2 2

7. 2 + 3x – 6 = 6 8. 5 2 3x = 15 9. 3+ 4+ x – 8 = 3

10. 2x + 13 – 4 = x – 5 11. x2 + 5 – 3 = x 12. x + 9 = x2 – 5

13. 2x2 + 3 = 5 x – 3 14.


3
2x = 2 15.
3
3x + 5 = 1

16. 4+5 x– 1 =3 17. 1+ 2 x + 7 = 3 18. 4 x+1 +2 =3 2

19. 3x + 1 + 2x – 1 = 7x + 2 20. 2 x – 3 + x + 1 = 3
10 5 2
x–2 x
21. x+2 = 22. x–5=
x+1 x+8
x–9 2x x 3x
23. 2x + 6 = 24. + =
x–5 5 3 5
25. 2x –1 = x + 3 – x – 2 26. 1 + x = x + 7

27. 2x – 1 + 2x+1=3 28. ax – bx = a b – b a

29. a x – b x = ab 30. x x 1
+ =
a b ab
1
31. 2x – 5 – 1 = 32. 2x – 7 – x – 3 = x + 4
2x – 5
3x
33. 2 – = x+5 34. 1 + 2x – 1 = 1 – 2x
x+5
35. m – 2n = mn 36. a – b x 2
= b–a x 2
x x

Soluciones
15 8
I. 1. x = 6 2. x = 88 3. x = – 2 4. x = 5. x1 = y x2 = 0
5 2 81
22 27
6. x = 7 7. x = 8. x = 9. x = 1.032
3 4
2 43
10. x1 = 6; x2 = 54 11. x = – 12. x = – 9
3

13. x1 = 13 ; x2 = 6 14. x = 4 15. x = – 4 16. x = 2


2 3

17. x = 9 18. x = 15 19. x1 = – 2; x2 = 1 20. x = 16


5
2
21. x = 22. x = 40 23. x1 = – 7 + 4 10; x2 = – 7 – 4 10
7 3

322 Potencias y raíces

322-323. 322 20/11/02, 2:00 PM


CAPITULO 6
CAPÍTULO

– 1 + 29 – 1 – 29
24. x = 0 25. x1 = 2
; x2 =
2
26. x = 9 27. x = 85
72
a2b2 1
28. x = ab 29. x = 2 30. x= 2
a–b a+b
13 + 5 13 – 5 – 1 + 113 – 1 – 113
31. x 1 = 4
; x2 =
4
32. x1 = ; x2 =
2 2

2
– 9 + 61 – 9 – 61 3
33. x1 = ; x2 = 34. x = 35. x = m – 2n 36. x = 1
8 8 8 2 2
m n

Prueba de selección múltiple


Potencias y Raíces (Marque la alternativa correcta).
–3
4. 2n–1 • 2n + 1 7. 1 –2
1. a5 • a3 • a–1 = a
2
A. a15 2–1 A. 8a6
A. 2n
B. a–15 8a5
B. 4n B.
C. a8 n2 1 6
a
C. 2 C. 2
D. a7
D. 42n D.
1 5
a
E. a–7 2 –1
2
E. 4n 1 –6
E. a
2. El valor de 5x2 8
5. El valor de (3x5y2z4)0
Si x = – 2 es: Si x = 2 y = – 1 z=1 8. 44 + 44 + 44 + 44
A. – 20 A. 96 A. 410
B. 20 B. – 96 B. 210
C. 100 C. –1 C. 25
D. – 100 D. 1 D. 216
E. 10 E. – 32 E. 416
3. El cuadrado de 6. 6m3n5 : – 2m2n3
9. 93 • 94 =
– 3m3 es: A. – 3mn2
A. – 9m6 A. 35
B. – 3m2n
B. – 9m9 B. 912
C. 9m6 C. 3mn2
C. 314
D. 9m9 D. 3m2n D. 312
E. 9m3
E. – 3mn3 E. 95

Potencias y raíces 323

322-323. 323 20/11/02, 2:01 PM


Prueba de selección múltiple
1
10. El valor de x en 14. 2 3 • 2 – 2 • 2 3 • 2 – 1 = E.
x 44x
2 3
= es: A. 83
3 2
B. 218 18. (0.5)x • (0.1)x • (40)x =
A. 1

B. –1 C. 8 A. 2x
D. 2–3 B. 4x
C. 0
3 E. 163 C. 20x
D.
2
15. Los valores de x e y en:
E. 2 D. 23x
3 ax + y =a
bx : by = b2 E. Otro
11. (3–1)–2 =
son respectivamente:
A. 9 19. (3–1)2 • (32)–1 • (3–1)–2 =
1 1 1
B. 1 A. ,
A.
2 2 3
9
C. 3 1 1
B. ,– B. 9
2 2
1 1
D. 3 1 C.
3 C. , 9
2 2
E. –9 D. –3
3 1
D. ,–
2 2
12. De las afirmaciones: E. –9

I an + an = a2n E. – 3 , 1
2 2 20. El valor de x en
II an • an = a2n 16. Los valores de y x2 2x 1 x
9 = 32 es:
III an • an = an para x = –1 son respec- 2

son verdaderas: tivamente: A. –5


1
A. Sólo I A. 1 , B. 5
2
1 1
B. I y II B. –1 , 2 C. 5
C. Sólo II C. 1 ,2 D. – 1
D. II y III 5
D. 1 , –2
E. Todas E. – 4
E. –1 , –2
13. El valor de x en 21. El valor de x en
3x + 1 = 9 x es: 17. 4x : 82x =
4 • 3x – 3 x = 27 es:
1
A. 3 A. x A. 1
2
B. –1 1
B. B. 2
24x
C. 2
C. 3
C. 1
D. 1 4x D. – 2
E. –3 D. 1
E. – 3
22x

324 Potencias y raíces

324-325. 324 08/11/2001, 12:53


CAPITULO 6
CAPÍTULO

1 1
22. x3 + x5 = D. – 1 D. (a – b)u – v
3
1 1
A. E. E. Ninguna.
x8 4
2
B. x5
x2 +1 26. Si x = 2 el valor de 30. El valor de
C.
x5 2x + 1 • 3x – 2 • 2 x – 1 es:
2
x +1
D. x2 • 2y • 3x • y3
x15 A. 8
si x = –1, y = –2 es:
E. Otro B. –8
2
A. –
C. 32 3
23. De las proposiciones: 2
B.
D. 16 3
I an • bn = (ab)n 1
C. –
E. 24 3
II (a + b)n = an + bn 1
D.
3
III an : a–m = an + m
27. La solución de x en E. Otro
Son falsas: (23x – 4)0 = 1 es:
3 12
31. =
A. I, II A. 2
4
B. Sólo II B. – 3 3
4 A.
4 2
C. I y III C. 6
3 B.
2
D. II y III D. No existe solución
C. 3
E. Ninguna E. Cualquier valor real
D. 6

24. (a + b)x + y : (a + b)x – y = 28. 23x – 4 • 3x – 5 • 2 2 – 3x • 33 – x =


E. 2 3
A. (a + b)2x 4
A. 9
32. 6• 3 =
B. (a + b)–2x 9
B.
4 A. 9
C. (a + b)–2y 1
C.
36 B. 9 2
D. (a + b)2y
D. 36
C. 2 3
E. (a + b) 1
E.
27 D. 3 2
25. El valor de x en
E. 18
29. (a + b)u • (a – b)v =
2x • 22x + 1 = 64x es:
A. (a2 – b2)uv
A. 3
B. (a2 – b2)u + v
B. –3

C. 1 C. (a + b)u + v
3

Potencias y raíces 325

324-325. 325 08/11/2001, 12:54


Prueba de selección múltiple
2
33.
3
64 = 37. 1 + 2 = 41. 2• 3• 6=

A. 3 A. 6
A. 2
3 B. 5 + 2 2 B. 6 6
B. 16
6
C. 3 + 2 2 C. 6
C. 2
6
D. 9 D. 30
D. 8
3 E. 2 6
E. 4 E. 5

38. 3– 2 3+ 2 = n
anm =
34. a b • a b = 42.
1
A. 1 A. a m
A. ab
1
B. 2 B. a n
B. ab2
m
C. 3– 2 C. a n
C. a2 b2
D. 5 D. an
D. a3 b
E. 5 E. am
E. a2 b

39. 3 2+ 3– 8 = 43.
3
25 • 5 =
3

35. a2 b3c4 =
A. 3 – 6 A.
6
5
A. abc2 b
B. 3 + 6 B. 6
25
B. a2 b c2 b
C. 3– 6 C. 5 5
3

C. abc c
6
D. 3+ 6 D. 5 5
D. abc2
E. 3 E. 5
E. abc2 c
2
40. Al racionalizar se 44. Si a = 3, b = 4, entonces
2
36. a a = obtiene: el valor de b2 – a2 es:
4 A. 2 2 A. 1
A. a

B.
4
a3 B. 2 B. 5
2
C. a C. C. 7
2
D. a a D. 4 2 D. 7
4
E. a a E. 2 E. –7

326 Potencias y raíces

326-327. 326 08/11/2001, 13:00


CAPITULO 6
CAPÍTULO

3
45. Al simplificar a15 b9 49. 3 2 • 2 18 = 52.
x
6x + 1 =
se obtiene:
A. 6 A. 6x
A. a12 b6 x
B. 36 B. 6
B. a5 b3 x
C. 12 C. 6 6
C. a 5 b3 D. 6 6 D. 6
D. a 12
b 6
E. 6 12 E. Otro
E. a12 b9
4 50. De las afirmaciones 53. La solución de la
46. Al racionalizar
5–1 siguientes: ecuación x + 2 = 5 es:
se obtiene:
I a 2 – b2 = a 2 – b2 A. 3
A. 5 +1
II a 2 + b2 = a + b B. 23
B. 5–1
III a b = b a C. 8
C. 4 5 +1
son verdaderas: D. 23
D. 4 5–1
5 +1 A. Sólo I E. 8
E.
4
B. Sólo II
54. La solución de
C. Sólo III
47. 3– 7 3+ 7 = 1 + 2x – 3 = 4 es:
4 D. Todas
A. 2 A. 0
E. Ninguna B. 3
B. 40

C. – 40 51. De las afirmaciones, C. 6


m n n m 3
D. 2 I a= a D.
2
m
E. 2 II a n = n am E. Otro

III an – m = an – am
2b
48. El valor de aa 2 si 55. La solución de
son falsas: 7 + 2 + x – 3 = 3 es:
a = 2 y b = 3 es:
3 A. Sólo III
A. 16 A. 4
3 B. I y III
B. 4 B. 3
6 C. II y III
C. 3 C. 5
6
D. Todas
D. 4 D. 7
E. Ninguna
E. Otro E. 9

Potencias y raíces 327

326-327. 327 08/11/2001, 13:02


Prueba de selección múltiple
3 3
56. En x+2 =2 , 58. Para que la expresión B. x < 5
el valor de x es: 2x – 3 sea real es
A. 0 necesario y suficiente C. x > 5
3
B. 6 que: 5
D. x < 3
C. 16 A. x ≥ 3
2 E. x > – 3
D. 62 B. x ≤ 5
3
E. 64 3
C. x ≥ 4 1
+
3 1
+
1
=
2 60. 81 27 9
x+1 2+ x– 1 2 =2 3
57. En D. x ≤ 1
2
A. 3
el valor de x es: 2
E. x ≥
3 1
A. ± 1 B. 9
B. ± 2 59. Para que la expresión
3 C. 3
C. 0 sea real es ne-
3 – 5x
D. ± 3 D. 1
cesario y suficiente que:
3
E. ± 4 A. x > E. Otro
5

Soluciones
Clave de Respuestas:
1. D 11. A 21. B 31. D 41. C 51. A
2. B 12. C 22. C 32. D 42. E 52. C
3. C 13. D 23. B 33. A 43. E 53. B
4. B 14. C 24. D 34. E 44. C 54. C
5. D 15. D 25. C 35. A 45. B 55. D
6. A 16. A 26. D 36. B 46. A 56. D
7. A 17. B 27. E 37. C 47. D 57. A
8. B 18. A 28. C 38. A 48. B 58. C
9. C 19. C 29. E 39. A 49. B 59. B
10. B 20. A 30. A 40. B 50. E 60. D

328 Potencias y raíces

328. 328 08/11/2001, 13:05


CAPÍTULO 7
L ogaritmos

Definición de logaritmo 7.1

Sean a, x E R+, a ≠ 1. Decimos que y es el logaritmo en base a de


x si y sólo si x = ay, lo que escribimos y = loga x.
NOTA: y es el exponente al que hay que elevar la base a
para obtener el número x.

• Observación 1: y = loga x es una función real cuyo dominio es


R+ y su rango o recorrido es R.

y = log2 x
2

11 1 1 2 3 4 5 6 7 8 9
84 2

–1

–2

–3

Solamente se puede calcular logaritmo de números reales positivos.


• Observación 2: Cada valor real positivo distinto de 1 que toma la base
a da origen a un sistema completo de logaritmos.
Si la base es 10 se acostumbra no escribirse y el sistema de
logaritmos de base 10 se llaman logaritmos vulgares, decimales
o logaritmos de Briggs.

Logaritmos 329

329. 329 8/11/01, 17:41


Si la base es e = 2,7128... entonces el sistema se denomina
de logaritmos naturales o neperianos y se acostumbra a anotar
por y = ln x.

• Observación 3: Si la base toma un valor entre 0 y 1, entonces


la gráfica queda como sigue:

11 1 1 2 3 4 5 6 7 8 9
84 2

-1

-2

-3
y = log 1 x
2

7.2 Propiedades

1. loga a = 1 El logaritmo de la base es 1.


2. loga 1 = 0 El logaritmo de 1 es 0.
3. loga M • N = loga M + loga N.
El logaritmo de un producto es igual a la suma de los logaritmos
de los factores.
M
4. loga = loga M – loga N.
N
El logaritmo de un cociente es igual al logaritmo del numerador
menos el logaritmo del denominador.
5. loga MP= p loga M.
El logaritmo de una potencia es igual al exponente multiplicado
por el logaritmo de la base.
logbN
6. loga N = Teorema de cambio de base.
logba
7. En el sistema de logaritmos en base 10, a la parte entera del
logaritmo de un número se le llama característica y a su parte
decimal se le llama mantisa.

330 Logaritmos

330-331. 330 8/11/01, 17:51


CAPÍTULO 7

1. Calcular: log2 128 Ejercicios


Solución: resueltos
Se debe encontrar el exponente al que hay que elevar la base 2 para que
dé 128. Como 27= 128, entonces:
log2 128 = 7

2. Calcular log 1 27
3

Solución:
1 3
1 2
= 3–1 y 27 = 27 2= 3
3
Aquí la pregunta es ¿a cuánto debemos elevar la base 3–1 para
3
2
que dé 3 ?
3
(3 – 1)x = 32
3
3 – x = 32

x = –3
2
Luego, log 1 27 = – 3
3 2
3. Calcular log5 3 125
Solución:
1
3
Como 125 = (53) 3 = 51 debemos encontrar x tal que 5x = 51 , de donde
3
x = 1. Luego, log5 125 = 1.

4. Calcular log27 1
9
Solución:
Debemos encontrar la forma de expresar la base del logaritmo y
el número al cual se le busca el logaritmo como potencias del
mismo número.
1
27 = 33 y = 9–1 = 3–2
9
Debemos hallar x tal que (33)x = 3–2, es decir, 3x = – 2 de donde
2
x=– .
3
1 2
Luego log27 =–
9 3

125
5. Calcular log 3
5 27
Solución:
125 53 5 3 3 –3
= = =
27 33 3 5

Logaritmos 331

330-331. 331 8/11/01, 17:53


x –3
Ejercicios Debemos hallar x tal que
3
=
3
. Es decir, x = – 3
5 5
resueltos Luego, log 3
125
=–3
5 27
6. Graficar la función y = log3 x. En base al gráfico responder las
siguientes preguntas.
a) ¿Cuál es su dominio?
b) ¿Cuál es su recorrido?
c) ¿Qué signo tiene el logaritmo en base 3 de los números menores
que 1?
d) ¿El logaritmo en base 3 de qué números está entre – 2 y 2?
e) Graficar la función y = log3 (x – 2) e indicar su dominio y su
recorrido.
Solución:
y
2

1 y = log3 x

1 1 1 2 3 4 5 6 7 8 9 x
9 3

–1

–2

a) Dom log3 x = R+
b) Rec log3 x = R

c) Negativo. Si observamos el gráfico, el logaritmo en base 3 de cual-


quier número menor que 1 es negativo. Por ejemplo, log3 1 = – 1
3
1
d) Observamos que log3 = – 2 y log3 9 = 2, luego el logaritmo en
9
base 3 de los números que están entre 1 y 9 están entre – 2 y 2
9
e)
y y = log3 (x – 2)
2

1 219 7 3 4 5 6 7 8 9 x
9 3

–1

–2
Dom log3 (x – 2) = {x E R / x > 2}
Rec log3 (x – 2) = R

332 Logaritmos

332-333. 332 8/11/01, 17:46


CAPITULO 7
CAPÍTULO

1
7. Calcular el valor de log a + log a
Solución:
1 1
log a + log a = log (a • a ) = log 1 = 0
N
8. Si log6 N = r. Determinar el logaritmo en base 6 de 216
Solución:
N
log6 = log6 N – log6 216 = r –3
216
3a b2
9. Desarrollar la expresión log – 1 . Escribirla en términos de
c
log a, log b, y log c.
Solución:
3a b2
log = log 3ab2 – log c–1
c– 1
= log 3 + log a + 2 log b + log c

10. Escribir como un solo logaritmo la expresión


1 1 1
2 log a – 2 log b – 2 log c
Solución:
1 1 1 1 1 1
log a – 2 log b – log c = log a 2 – log b 2 – log c 2
2 2
= log a – (log b + log c )

= log a – log b • c
a a
= log = log
b c bc

Ejercicios

1. Escriba como potencia del número que se indica los siguientes números:

a) 2, 4, 8, 16, 64, 256 de 2 1


c) 2, 4, 8, 16, 64, 256 de
2
b) 1, 1 , 1 , 1 , 1 , 1 1 1 1 1 1 1
de 2 d) , , , , , de 1
2 4 8 16 64 256 2 4 8 16 64 256 2
2. Escriba los siguientes números como potencia de 10.
1 1 1
a) 10; 100; 1.000.000 b) 10 ; 100; 1.000.000
c) 1; 0,1; 0,001; 0,0000001

3. Escriba como un logaritmo de base 3 los siguientes números:

a) 1; 2; 3; 4 b) – 1; – 2; – 3; – 4

Logaritmos 333

332-333. 333 8/11/01, 17:48


Ejercicios
4. Calcule los siguientes 6. Calcule los siguientes m) log 1 2
4
logaritmos: logaritmos:
3
n) log 81
a) log 1 a) log 1 1
2

b) log 10 b) log 1 2 o) log 1 4


2 2

c) log 100 c) log 1 4


2
p) log9 243
d) log 1.000 1
d) log 1
2 2 q) log0,3 0,0081
e) log 1 1
10 e) log 1 4
2 r) log50 1
f) log 1
100 f) log 2
1 8
2 s) log 3
g) log 0,01 2
27
3
g) log 2
h) log 0,0001 1
2
t) log 1 1.000
10
1
h) log 1 8. Encuentre entre qué poten-
i) log 10 2 8
cias de 10 está el número
i) log 1 8 cuyo logaritmo decimal es:
j) log 1.000 2
3 1 a) 0,5
5. Calcule los siguientes j) log 1 4
logaritmos: 2 b) 1,2
7. Calcule los siguientes c) 2,8
a) log 2 1 logaritmos:
d) 3,5
b) log 2 2 a) log7 343 e) – 0,5
b) log3.459 1 f) – 1,7
c) log 2 4
c) log 64 g) – 2,3
8
1 3 4 h) – 3,7
d) log 2 d) log5
2 6
e) log8 32 9. Encuentre entre qué núme-
1
e) log 2 ros enteros está el logaritmo
4
f) log2 32 decimal de: (bosqueje el
f) log 2 2 gráfico de y = log x)
g) log3 125
0.2
a) 7
3
g) log 2 2 h) log9 27
b) 9,27
4
1 i) log4 16 c) 12,58
h) log 2 8
j) log27 1 d) 83,025
3
i) log 2 8 k) log 128 e) 135
16
f) 2.992,16
3 1 1
j) log 2 4 l) log3 g) 0,27
9

334 Logaritmos

334-335. 334 20/11/02, 3:28 PM


CAPÍTULO 7

h) 0,349 b) 1, 2 c) log (a – b) (a2 – 2 ab + b2)= 2


i) 0,052 c) 2, 2
d) log 1 27 = – 3
d) 3, 2
j) 0,0116 3

e) 4, 2 18. Calcule el valor de la


k) 0,0098 incógnita:
f) 5, 2
l) 0,000145
a) log5 25 = x
14. Usando una calculadora
10. Escriba la característica científica determine el
de los logaritmos deci- b) log5 x = 3
número cuyo logaritmo
males de los números en base 2 es:
del ejercicio anterior. c) logx 27 = 3
a) 0, 2
d) log0,027 x = 1
11. Determine en qué base b) 1, 2 3
el logaritmo de: 1
c) 2, 2 e) log32 =x
2
a) 125 es 3
d) 3, 2 16
f) logx =2
3 36
b) 8 es
2 e) 4, 2 1
g) log0,008 x =
c) 16.384 es 7 3
f) 5, 2
d) 4 es 2 h) logx 1 = – 2
4
e) 9 es 2 15. Escriba alguna relación i) log5 x = 1
f) 16 es 2 entre los resultados del
ejercicio 13 y los del j) logx 8 = – 3
4
12. Escriba el logaritmo de ejercicio 14. 2
k) logx 4 = –
los siguientes números 3
en función de log 2. 1
16. Exprese x en forma de l) log3 x = –
3
logaritmo en cada igual 1
a) log 4 m) log2 =x
dad siguiente: 64

b) log 1 a) 4x = 1 n) log81
1
=x
16 3
1 b) 14x = 17
c) log o) log4 x = 3
32 2
c) ax = m • n
6
d) log 5 p) log2 x =
5
d) qx = a+b
e) log 125 q) logx 8 = – 2

f) log 0,5 17. Escriba en forma de r) logx 8 = – 3


13. Usando una calculadora potencia las siguientes
igualdades: s) logx 27 = – 3
científica determine el
número cuyo logaritmo
t) logx 9 = – 2
decimal es: a) loga b = p

a) 0, 2 b) log5 4 = p u) log 2 x = 0
3

Logaritmos 335

334-335. 335 20/11/02, 3:29 PM


Ejercicios
1 2
w) logx =– 24. Grafique y = log 1 x
4 3
2
1 2
v) log3 1 = x x) logx = – y = log x
1
3 3 3

19. Aplicando las propiedades de logaritmo, calcule: 25. Si la base del logaritmo es
mayor que 1, ¿qué signo
1
a) loga 1 + logb bn + logc tienen los lo ga rit mos
cn
de los números mayores
b) loga a2 + logb b3 que 1, y de los números
menores que 1?
c) loga ab + loga a
b 26. Si la base del logaritmo es
3 4
d) loga a + logb b + logc c menor que 1, ¿qué signo
tienen los loga ritmos
e) log1.000 – log3 92 de los números mayores
que 1, y de los números
f) log 0,1 – log 0,01 menores que 1?

g) log 1 1 + log 2 3 + log9 1 27. Encuentre usando una


4 3 2 3 calculadora:
h) log2 3 +log 1 3 (Sug.: escriba en notación de potencia) a) El logaritmo en base
2
3 de 4, 6, 8, 10.
20. Aplicando las propiedades escriba los siguientes logaritmos
b) El logaritmo en base
desarrollados:
1
de 4, 6, 8, 10.
4 a2 b3 –2
a3 b 3
a) log b c c) log e) log ¿Qué relación existe entre
a ac c2 a3
los resultados?
3
3 a2 b2
a2 b
b) log d) log 5 b 28. Demuestre que:
c
c
loga x = – log 1 x
a
21. Aplicando las propiedades, reduzca las expresiones
I x E R+
siguientes:

a) log a + log b – 2 log c 29. Demuestre que:

1 3 1 1 loga b • logb c • logc a = 1


b) log a + log b – log c – log d
2 2 2 2
30. Si loga p = 3 y
1 2 1
c) log a – log b + log c loga 36 p = 5, calcule a.
3 3 3
1 1 1 31. Calcule el valor de:
d) log x – log y – log z
2 3 4
1 1
1 loga + logb
22. Demuestre que: log9 x = log3 x a b
2
23. Grafique las funciones siguientes: 32. Calcule el valor numérico
de:
a) y = log2 (x + 1) c) y = 1 + log2 x logx x2
a) 3
b) y = log2 (x – 1) d) y = 1 – log2 x

336 Logaritmos

336-337. 336 20/11/02, 3:35 PM


CAPÍTULO 7

b) 4log x x 34. Demuestre que 36. Calcule el valor


1 numérico de:
c) log (log 1010) log + log y = 0
y 3 logb b + logb b2+ logb b–5
d) loga aa log a a 2
35. Si log x = y, encuentre
33. Si log 2 = 0,3010 y log 3
= 0,4771, encuentre sin log(10x 2) en función
calculadora log 120. de y.

Soluciones

1. a) 21, 22, 23, 24, 26, 28 b) 2–1, 2–2, 2–3, 2– 4, 2– 6, 2– 8

c) 1 –1
1 –2
1 –3
1 –4
1 –6
1 –
d) 1 1 1 2 1 3 1 4 1 6 1 8
, , , , , , , , , ,
2 2 2 2 2 2 2 2 2 2 2 2

2. a) 10; 102; 106 b) 10–1; 10–2; 10–6 c) 100; 10–1; 10–3; 10–7

1 1 1 1
3. a) log3 3; log3 9; log3 27; log3 81 b) log3 ; log3 ; log3 ; log3
3 9 27 81
4. a) 0 b) 1 c) 2 d) 3 e) –1

1 3
f) –2 g) –2 h) –4 i) j)
2 2
5. a) 0 b) 1 c) 2 d) –1 e) –2

1 1 3 3 2
f) g) h) – i) j) –
2 3 2 2 3
6. a) 0 b) –1 c) –2 d) 1 e) 2

1 1 3 3 2
f) – g) – h) i) – j)
2 3 2 2 3

5
7. a) 3 b) 0 c) 4 d) 0 e)
3
5 9 3 1 1
f) g) – h) i) j) –
2 2 4 2 3
7 4
k) l) –2 m) – 0,5 n) o) – 4
2 3
5 3
p) q) 4 r) 0 s) –3 t) –
2 2
8. a) entre 1 y 10 b) entre 10 y 102 c) entre 102 y 103 d) entre 103 y 104
e) entre 10–1 y 1 f) entre 10–2 y 10–1 g) entre 10–3 y 10–2 h) entre 10– 4 y 10–3

9. a) entre 0 y 1 b) entre 0 y 1 c) entre 1 y 2 d) entre 1 y 2

Logaritmos 337

336-337. 337 20/11/02, 3:35 PM


Soluciones

e) entre 2 y 3 f) entre 3 y 4 g) entre –1 y 0 h) entre –1 y 0


i) entre –2 y –1 j) entre –2 y –1 k) entre –3 y –2 l) entre – 4 y –3

10. a) 0 b) 0 c) 1 d) 1 e) 2 f) 3
g) –1 h) –1 i) –2 j) –2 k) –3 l) – 4

11. a) 5 b) 4 c) 4 d) 2 e) 3 f) 4

5
12. a) 2 log 2 b) – 4 log 2 c) – log 2
2
d) 1 – log 2 e) 3 – 3 log 2 f) – log 2

13. a) 1,58489319 b) 15,8489319 c) 158,489319


d) 1.584,89319 e) 15.848,9319 f) 158.489,319

14. a) 1,148698355 b) 2,29739671 c) 4,59479342


d) 9,18958684 e) 18,37917368 f) 36,75834736

15. En el ejercicio 13, cada número obtenido es igual al anterior multiplicado por 10. En el
ejercicio 14, cada número obtenido es igual al anterior multiplicado por 2.

16. a) x = log4 1 b) x = log14 17


c) x = loga m • n d) x = logq a + b

1 –3
17. a) ap = b b) 5p = 4 c) (a – b)2 = a2 – 2ab + b2 d) = 27
3

1
18. a) 2 b) 125 c) 3 d) 0,3 e) –
5
2 1
f) g) 0,2 h) 2 i) 5 j)
3 16
1 3 1 1
k) l) 3 m) – 6 n) – o) 8
8 4
5 2 1 1 1
p) 2 2 q) r) s) t)
4 2 3 3

u) 1 v) 0 w) 8 x) 3 3

13
19. a) 0 b) 5 c) 2 d)
12
3
e) –1 f) 1 g) – h) 0
2
3 3
20. a) 4 log b + 2 log c – 4 log a b) 6 log a + log b – log c
2 2

338 Logaritmos

338-339.(2003) 338 20/11/02, 3:44 PM


CAPITULO 7
CAPÍTULO

7 1
c) – 3 log a – 6 log b + 2 log c d) 2 log a + log b + log c
15 5
3 1
e) log a + log b – log c
4 4
ab ab3 3 ac x
21. a) log b) log c) log d) log
cd b2 4
c2 3
y z

23.
a) y = log2(x + 1) b) y = log2(x – 1)
3
3
2 2
1 1

–1– 12 1 2 3 4 5 6 7 2 3 4 5 6 7 8 9
–1 –1

–2
–2

c) d)
y = 1 + log2x
4

3
3
2 2
1
1 1
-1 21 2 4 5 6 7 8 16
1 -2
1 2 3 4 5 6 7 8
2 -3
-1 y = 1 – log2x

-2

-3
24.

3 3

2 2

1
1
1
1 3
1 2 3 4 5 6 7 8 9
1 2 3 4 5 6 7 8
2 -1
-1
-2
-2
-3
-3

25. Positivo, negativo


26. Negativo, positivo

Logaritmos 339

338-339.(2003) 339 20/11/02, 3:45 PM


Soluciones

27. 30. 6 33. 2,0791


a) 1,261859507 b) – 1,261859507
1,630929754 – 1,630929754 31. – 2 35. 1 + 2y
1,892789261 – 1,892789261
2,095903274 – 2,095903274 32. 36. 0
log3 x = – log 1 x x = 4, 6, 8, 10 a) 9 b) 2 c) 1 d) 2a
3

7.3 Ecuaciones exponenciales


y logarítmicas

Se llaman ecuaciones exponenciales aquellas ecuaciones que


presentan la incógnita en el exponente:

3x = 1 o 23x – 1 = 3x + 2
Se llaman ecuaciones logarítmicas aquellas ecuaciones que
presentan la incógnita como argumento de una función logarítmica:
log x = 2 o log (3x – 1) = log (x + 2)

Para resolver ecuaciones exponenciales podemos igualar las


bases y aplicar:

ax = ay ⇔ x = y

Ejemplo: 3x = 1
3x = 30

x=0
En ocasiones no es posible igualar las bases, y en estos casos
podemos aplicar el concepto de función logarítmica. Como ésta es
biyectiva en su rango se tiene:

log x = log y ⇔ x = y

Ejemplo: 23x – 1 = 3x + 2
log 23x – 1 = log 3x + 2
(3x – 1) log 2 = (x + 2) log 3
3x log 2 – log 2 = x log 3 + 2 log 3
x (3 log2 – log 3) = 2 log 3 + log 2
2 log 3 + log 2
x =
3 log 2 + log 3
340 Logaritmos

340-341. 340 8/11/01, 18:25


CAPÍTULO 7

Para resolver ecuaciones logarítmicas también debemos aplicar


el concepto de función biyectiva:
log x = log y ¤ x = y
Ejemplo:
log x = 2 2 = log 100
log x = log 100

x = 100
Ejemplo:
log (3x – 1) = log (x + 2)

3x – 1 = x + 2
2x = 3
2
x=
3

1. Resolver la ecuación 22x = 5 Ejercicios


Solución: Aplicando logaritmo. resueltos
log 22x = log 5 aplicando log ap = p log a
2x log 2 = log 5 despejando x
log 5
x= 2 log 2 con una calculadora

x= 0,69897
2•0,3010
x = 1,16096
2. Resolver la ecuación 5 • 52x – 1 = 2
Solución: Para aplicar logaritmo debemos efectuar primero la
multiplicación 5 • 52x – 1 = 52x

log 52x = log 2


2x log 5 = log 2

x= log 2
2 log 5
0,3010
x=
2•0,6989
x = 0,2154
3. Resolver la ecuación:
3x – 1 + 3x – 2 + 3x – 3 + 3x – 4 = 1.080

Logaritmos 341

340-341. 341 8/11/01, 18:25


Ejercicios Solución:

resueltos 3x (3–1 + 3–2 + 3–3 + 3– 4) = 1.080


1 1 1 1
3x 3 + 9 + 27 + 81 = 1.080

40
3x • = 1.080
81
1.080 • 81
3x =
40
3x = 2.187
3x = 37

x= 7

4. Resolver la ecuación
4x – 2 • 2 x + 3 + 64 = 0
Solución:
Haciendo 2x = y nos queda una ecuación de segundo grado.
4x – 2 • 2 x • 2 3+ 64 = 0
(2x)2 – 16 • 2x + 64 = 0
y2 – 16y2 + 64 = 0
(y – 8)2 = 0
y= 8
Luego 2x = 8 de donde x = 3

5. Resolver la ecuación
8x – 9 • 8–x = 8
Solución:
8x – 9 • 8–x = 8
1
8x – 9 • = 8 / • 8x
8x
82x – 9 = 8 • 8x
82x – 8 • 8x – 9 = 0 Sea 8x = y

y2 – 8y – 9 = 0
y1 = 9
(y – 9) (y + 1) = 0
y2 = –1

Si y1 = 9 fi 8x = 9 fi x = log 8 9 fi x = 0,94639
Si y2 = –1 fi 8x = –1 fi x no existe.

6. Resolver la ecuación 6x • 32x + 2 =20

342 Logaritmos

342-343 342 8/11/01, 18:38


CAPÍTULO 7
CAPITULO

Solución: 6x • 32x + 2 = 20
6x • 32x = 18 aplicando log
log 6x • 32x = log 18
log 6x + log 32x = log 3 + log 6
x log 6 + 2x log 3 = log 3 + log 2 + log 3
x (log 2 + log 3 + 2 log 3) = 2 log 3 + log 2

2 log 3 + log 2
x=
3 log 3 + log 2

7. Resolver la ecuación

log x + log (x + 1) = log 6


Solución: Aplicando propiedades de logaritmo el primer miembro
queda log x (x + 1).

log (x2 + x ) = log 6



x2 +x= 6
x2 +x–6= 0
(x + 3) (x – 2) = 0
cuyas soluciones son x1 = –3 y x2 = 2
Una vez obtenidas las soluciones de la ecuación de segundo grado
debemos comprobar que los valores obtenidos sean en realidad solución
de la ecuación logarítmica planteada. Recordemos que el dominio
de la función logarítmica es R+ y no R, por lo tanto, cualquier valor
obtenido para la incógnita que haga negativo o cero el argumento de algún
logaritmo, no es solución de la ecuación planteada. En este caso x = –3 no
es solución ya que log (–3) y log (–2) no están definidos.
x = 2 es solución ya que:

log 2 + log (2 + 1) = log 6


log 2 + log 3 = log 6
log 2 • 3 = log 6
log 6 ∫ log 6
8. Resolver la ecuación:
log x – log (x + 3) = –1

1 x
Solución: –1 = log y log x – log (x + 3) = log
10 x+3
x 1
luego log = log
x+3 10
x
= 1
x + 3 10

Logaritmos 343

342-343 343 8/11/01, 18:38


Ejercicios 10 x = x + 3
resueltos 9x=3
1
x=
3
que es solución de la ecuación planteada.
En efecto:
1 1
log – log + 3 = –1
3 3
10
log 1 – log 3 – log = –1
3
log 1 – log 3 – log 10 + log 3 = –1
0 – 1 = –1
–1 ∫ –1
9. Resolver la ecuación:
2log2 (x + 2) – log2 (x + 1) = 2
Solución:
x+2 2
log2 = log24
x+1

x+2 2
=4
x+1
(x + 2)2 = 4 (x + 1)
x2 + 4x + 4 = 4x + 4
x2 = 0
x =0
comprobando
2log2 2 – log21 = 2
{
{

1 0
2 •1 – 0 = 2
2∫2

10. Resolver la ecuación


log x3 = log2 x – 4
Solución:
3 log x = log2 x – 4 Sea u = log x
3u = u2 – 4
2
u – 3u – 4 = 0 u= 4
(u – 4) (u + 1) = 0
u = –1
Si log x = 4 fi x = 10.000
Si log x = –1 fi x = 0,1

Ambos valores satisfacen la ecuación.

344 Logaritmos

344-345. 344 8/11/01, 18:42


CAPITULO 7
CAPÍTULO

2x + 2–x
11. Despejar x en la expresión y =
2x – 2–x
Solución:
Sea u = 2x fi 2–x = 1 1
=
2x u
1
u+
u u
y= 1
/• u
u–
u
u2 + 1
y= 2
u –1
u2y – y = u2 + 1
u2y – u2 = y + 1
u2 (y – 1) = y + 1
y+1
u2 =
y–1
Como u = 2x entonces u2 = 22x

22x = y + 1 aplicando log


y–1
2x log 2 = log (y + 1) – log (y – 1)
log (y + 1) – log (y – 1)
x=
2 log 2
12. Resolver el siguiente sistema:
2x+y = 6
2x = 3 • 2y – 1

Solución: Aplicando logaritmo en ambas ecuaciones:


(x + y) log 2 = log 6
x log 2 = log 3 + (y – 1) log 2

x log 2 + y log 2 = log 6


x log 2 – y log 2 = log 3 – log 2

Sumando: 2 x log 2 = log 6 + log 3 – log 2


log 2 + log 3 + log 3 – log 2
x=
2 log 2
log 3
x= = log2 3 = 1,5849
log 2
Restando: 2 y log 2 = log 6 – log 3 + log 2
log 2 + log 3 – log 3 + log 2
y=
2 log 2
y=1
Luego la solución del sistema es (log2 3, 1)

Logaritmos 345

344-345. 345 8/11/01, 18:43


Ejercicios 13. Resolver el siguiente sistema:
log (x + 1) + log 2 = log y
resueltos
log x + log (y + 1) = log 5

Solución: Aplicando las propiedades de logaritmo:

log[(x + 1) • 2] = log y
log [x (y + 1)] = log 5

2x + 2 = y (1)
xy + x = 5 (2)

Reemplazando y en (2)

x ( 2x + 2) + x – 5 = 0

2x2 + 3x – 5 = 0
x1 = 1
x = –3 ±
9 + 40
=
4 x2 = – 10 = – 5
4 2
Si x1 = 1 fi y1 = 4
–5
Si x2 = fi y2 = – 3
2
La segunda solución obtenida no es solución del sistema, ya que log
[(x + 1) • 2] sería el logaritmo de un número negativo.
Por lo tanto, la solución es (1, 4).

14. Demostrar que:


log a2 + 1 + a + log a2 + 1 – a = 0 para todo a ≥ 0.

Solución:

log a2 + 1 + a + log a2 + 1 – a =

log a2 + 1 + a a2 + 1 – a =
log (a2 + 1 – a2) = log 1 = 0

15. Encontrar la base del sistema de logaritmos en que el logaritmo de 80


excede al logaritmo de 5 en 2 unidades.
Solución: Sea x la base buscada:

logx 80 – logx 5 = 2

logx 80 = 2
5
logx 16 = 2
luego, x = 4

346 Logaritmos

346-347. 346 8/11/01, 18:51


CAPITULO 7
CAPÍTULO

Ejercicios
1. Resuelva las siguientes ecuaciones:
1
a. 3 x = 12 c. 2 x + 2 = 5 x + 1 e. 2 • 2 x – 1 =
3
b. 2 x – 1 = 32x – 1 d. 3 2x – 3 – 2 4x –1 = 0 f. 3 3x + 1 = 3 • 2 x + 3
2. Resuelva las siguientes ecuaciones:

a. 2x – 1 + 2x + 2x + 1 = 14 e. 2 x + 2x + 1 + 2x + 2 + 2x + 3 = 15

b. 3x – 1 + 3x + 3x + 1 = 13 f. 3x + 3x + 1 + 3x + 2 = 39

c. 5x – 1 + 5x + 5x +1 = 31 g. 3x + 3x – 1 + 3x – 2 = 13

d. 5x – 1 + 5x + 5x + 1 = 62 h. 21 – x + 22 – x + 23 – x = 7
3. Resuelva las siguientes ecuaciones:

a. 4x – 9 • 2x + 1 + 81 = 0 d. 52x + 3 – 8 • 5x + 1 + 3 = 0

b. 9x + 3x – 12 = 0 e. 32x + 2 – 5 • 3x + 1 + 4 = 0
Nota: Reduzca las ecuaciones precedentes
c. 2 • 32x – 7 • 3x + 3 = 0 a ecuaciones de segundo grado.
4. Resuelva las siguientes ecuaciones:

a. 10x + 10–x = 2 d. ax + 24a–x – 11 = 0

b. 5x + 16 • 5–x – 8 = 0 e. 2x + 5 • 22 –x – 9 = 0
Nota: Reduzca las ecuaciones precedentes a
c. 5x + 15 • 5–x – 8 = 0
ecuaciones de segundo grado.
5. Resuelva las siguientes ecuaciones:

a. log x + log 3 = log 15 f. 2 log x = – 2

b. log 2 – log x = log 3 g. 2 log x + log 4 = 2

c. log x – 2 log 3 + log 2 = 0 h. log x3 = log 3 + log x2

d. 2 log x = 2 i. log x5 = 3 + log x2

e. 2 log 2 x = 4 j. 2 log 3 x – log 3 2 = 2

6. Resuelva las siguientes ecuaciones:

a. log (x + 3) = log (2x – 1)


b. log (x + 1) + log (x – 2) = log (x – 3) + log (x + 5)
c. 2 log (x + 1) – log (x – 1) = 1
d. log x = 1 + log (11 – x)
e. log (3x – 4) – log (2x + 1) = log (2x – 1) – log (3x + 4)

f. 2 log (x + 4) – log (x – 1)2 = log 3

Logaritmos 347

346-347. 347 8/11/01, 18:51


Ejercicios d)
– log x – 2 log y = 5
g) log (x + 1) = log 3 + log (x – 3) – 3 log x – log y = 3
h) 2 log2 (x + 2) – log2 (x + 1) = 2
i) 2 log3 (x + 2) = log3 9 e)

j) 3 log2 (x + 1) = log2 (x + 1)4 log2 (x + 2y) = 1


log3 (2x + y) = 0
7. Resuelva las siguientes ecuaciones:
11. Resuelva el siguiente sistema:
a) 2 log3 x = (log3 x)2 + 1
1 1 log2 (x + y) + log2 (x – y) = 6
b) log x 2 = (log x) 2
2x • 2y = 16
c) 2 log x = (log x )2

d) log x3 = log2 x + 2
12. Resuelva el siguiente sistema:

8. Resuelva los siguientes sistemas: (x + y)log (x – y) = 100


a) d) x2 – y2 = 1.000
2x – y = 3x 22x – 3 = 4y – 1
3 • 2x + 1 = 2y – 1 6x • 6 = 3y 13. Resuelva el siguiente sistema:
b) e)
log2 (x + y) – log2 (x – y) = –2
2x – 1 = 3y + 2 • 2 4x – y • 2 = 2x
3x • 3y = 81
3x – y =2 3 x + y : 3 = 3y

c) 14. Resuelva el siguiente sistema:


2x + y = 6
2x = 3 • 2y – 1 log3 (x + y) + log3 (x – y) = 5
ex : ey = e27
19. Si 3x + y = 2 y 2x – y = 3 pruebe que
x + y = log3 2 y que x – y = log2 3
15. Determine qué relación debe existir entre
p y q para que se cumplan las siguientes
10. Resuelva los siguientes sistemas: relaciones:
a) a) log p – log q = 1
log (x + 3) + log 2 = log (x + y) b) 2 log p + log q = 2
log y – log 3 = log (x + 1)
16. Determine dos números naturales x
b) e y tales que:
2 log x – log y = 3 y log x – x log y = 0
log x + log y = 1
x2 – y2 = 12

c)
17. Determine dos números tales que la suma
log (x + 1) – log 2 = log y
de sus cuadrados sea 10.100 y que la suma
log x – log (y + 1) = log 3
de sus logaritmos decimales sea 3.

348 Logaritmos

348-349. 348 20/11/02, 3:47 PM


CAPÍTULO 7

18. Determine dos números sabiendo que 21. Encuentre el sistema de logaritmos (su
la diferencia de sus logaritmos en base base) en que el logaritmo de 108 excede al
2 es 1 y que la suma de sus cuadrados logaritmo de 12 en 2 unidades.
es 1.280.
22. En las siguientes expresiones, despeje la
19. Determine dos números que estén en la incógnita que se indica:
razón 3:1, sabiendo que la suma de sus
logaritmos en base 3 es 7. a) log2 y = x + k ; y
b) log x – 2 log y = 0 ; x
20. Encuentre el sistema de logaritmos (su
base) en que el logaritmo de 324 excede al c) ln x = ln x0 – t ; x
logaritmo de 81 en 2 unidades. d) log3 k = log3 4 - 2 log3 x ; x
e) ln (30 – c) = ln 30 – 2 t ; c

Soluciones
2 log 2 log 2 – log 3 log 5 – 2 log 2
1. a) 1 + log 3 b) c)
log 2 – 2 log 3 log 2 – log 5
3 log 3 – log 2 log 3 3 log 2
d) e) – f)
2 log 3 – 4 log 2 log 2 3 log 3 – log 2
1
2. a) 2 b) 1 c) 1 d) e) 0 f) 1 g) 2 h) 1
log 5
2 log 3 log 2 log 3 – 2 log 5 log 4 – log 3
3. a) b) 1 c) 1, – d) –1, e) –1,
log 2 log 3 log 5 log 3
log 4 log 3 log 3 log 8 log 5
4. a) 0 b) c) 1, d) , e) 2,
log 5 log 5 log a log a log 2
2 9
5. a) 5 b) c) d) 10 e) 4 f) 0,1 g) 5 h) 3 i) 10 j) 3 2
3 2
13 7+5 3
6. a) 4 b) c) 4 ± 5 d) 10 e) 3 f) 2
g) 5 h) 0 i) 1 j) 0
3

7. a) 3 b) 1, 10.000 c) 1, 100 d) 10, 100


2 log 2 + log 3 2 log 2 + log 3 log 2 – log 3 2 log 3 + log 2 2 log 3 + log 2 log 2
8. a) , b) , –
– log 3 – log 2 log 3 log 2 – log 3 log 2 – log 3 log 3

log 3 + 2 log 6 3 log 6


c)
( log 3
log 2
,1 ) d) ,
2 (log 3 – log 6) – 2 log 6 + 2 log 3
e) (1, 1)

9. Sugerencia: resuelva el sistema formado con las dos condiciones dadas.


3 1
10. a) 3 15
, b) 10 10 , 3
2 2 10
c) No tiene solución porque los valores que resultan para x e y hacen que
algún argumento de logaritmo sea menor o igual que cero

d) 5 5 e) (0, 1)
0.1 , 0.01 0.01
11. (10, – 6) 12. (55, 45) 13. (10, – 6) 14. (18, – 9)

Logaritmos 349

348-349. 349 20/11/02, 3:47 PM


Soluciones

15. a) p – 10q = 0 18. 16 y 32 22. a) y = 2x + k

b) p2 q = 100 b) x = y2
19. 81 y 27
c) x = x0 e– t
16. 4 y 2 20. base 2 k
d) x = 2
k
17. 100 y 10 21. base 3 e) c = 30 (1 – e– 2 t)

Prueba de selección múltiple

1. Si log k = x, entonces 4. Si log x = a, entonces 7. log 27 =


3
log 100k = log x =
A. 1
A. 100 + k A. a B. 3
B. 100 + x C. 6
B. 2a
C. 2+k
C. 1 a D. 9
D. 2+x 2
E. 2x D. a E. 12
–1
E. a 2
2. Si log x = 0,3495, 8. log 81 9 =
entonces log x2 = 5. Si log x = y, entonces
A. 2
A. 0,3495 log 10x3 =
B. 1
B. (0,3495)2
A. 1 + 3x 1
C.
C. 2 • 0,3495 2
B. 1 + 3y
D. 4 • 0,3495 1
D. –
C. 10 + 3x 2
E. 4,3495
D. 10 + 3y E. –1
3. Si log x = a y
E. 30y
log y = b, entonces
1
log 3 xy = 9. log 27 =
6. Si e y = 3, entonces y = 3
A. log 3 A. 3
A. 3 a + 3 b
B. ln 3 B. 1
B. 3 a b
1 1 C. log 3 – log e 1
a+ b C.
C. 3 3 3
D. ln 3 – ln e 1
D. –
D. 1 a • b 3
3
E. 3
a+b E. ln 3 E. –1
e

350 Logaritmos

350-351. 350 8/11/01, 19:46


CAPITULO 7
CAPÍTULO

1 18. Si y = alog ax,


10. log + log x = 14. Si log x2 y = a
x
x entonces x vale:
y log = b,
1
A. log x y2
x A. loga y
B. log x entonces log y =
1 B. logy a
C. –1 A. (a – 2b)
3 C. y
1
D. 0 B. (2a + b)
5 D. 0
E. 1 1
C. (a + 2b) E. 1
3
1 19. La expresión
D. (2a – b)
11. El valor de 5 2 log a x log a y
1 a • a = 1 es
logq p • logp r •logr q es: E. (a – 2b)
5 equivalente a:
A. pq r 15. Si log x + log 3 = A. 2 loga x + loga y = 0
1 log 60 – log 20,
B.
pqr B. 2 loga x • loga y = 0
entonces x =
C. p + q + r
A. 0 C. 2 loga x • loga y = 1
D. 1 B. 1
D. 2 loga x + loga y = 1
E. 0 C. 3
D. 10 E. y = x2
12. La expresión loga b • logb c E. 33
es equivalente a: 20. De las siguientes expresio-
16. Si log n – log x =
nes; son equivalentes:
A. logb c 2 log y – 1, entonces n = I. blog b x • b2 log b y = blog b 1
B. logc b x2y II. logb x + logb y2 = 0
A.
C. loga c 10 III. xy2 = 1
xy2
D. loga bc B. A. Sólo I y II
10
E. logb ac C. 10 x2 y B. Sólo II y III
C. Sólo I y III
D. 10 x y2
a D. Todas
13. La expresión log es
b2c E. x (y – 1)2 E. Ninguna
equivalente a:
17. Si px –2 = qx + 1,
21. La expresión
A. log a – 2 log b + log c entonces x = 5 loga a – loga a4 + loga a–2
A. 2 log p vale:
B. log a – 2 log b + 2 log c
B. 2 log p + log q A. – 2
C. log a – 2 log b – log c C. 2 log p – log q B. – 1
2 log p + log q C. 0
D.
D. log a – 2 log b – 2 log c log p – log q D. 1
2 log p + log q
E. log a + 2 log b + log c E. E. 2
log q – log p

Logaritmos 351

350-351. 351 8/11/01, 19:46


Prueba de selección múltiple
22. La expresión 25. Si 2 x –2 + 2x + 2 = 17, 28. En la expresión
loga 5 + log 1 5 vale: entonces x vale: log 3 x = p + q, x vale:
a
A. – 2 A. 2 A. 3p + q
B. – 1 B. 1 B. 3q + p
C. 0 C. 0 C. 3p + q
D. 1 D. –1 D. 3p – q
E. 1 E. 3p + 3q
E. –2
1
23. El valor de log3 8+ log3
8 29. Si ln y = ln y0 – t,
es igual a: 26. Si 31– x – 3x– 1 = 8 • 3 –1,
entonces x vale: entonces y vale:
A. – 2
A. 2 A. e– t • y0
B. – 1
B. 1 B. e– t – y0
C. 0
D. 1 C. 0 C. e– t + y0

E. 2 D. –1 D. e– t – y0

E. – 2 E. et + y0
24. La expresión
1 1 log x 2 log y
log2 + log3 es: 27. Si log (x + 3) – log (x +2) 30. Si = ,
3 2 2 3
= log 2, entonces x vale: entonces :
A. log2 3 – log3 2
A. 2 A. x3 – y4 = 0
B. – log2 3 – log3 2
B. 1 B. x3 + y4 = 0
C. – log2 3 + log3 2 C. 3x – 4y = 0
C. 0
D. log2 3 + log3 2 D. –1 D. 3x + 4y = 0

E. – 2 E. x = 3 y
E. – log2 5

Soluciones

Clave de respuestas

1. D 6. B 11. D 16. B 21. B 26. C


2. D 7. C 12. C 17. D 22. C 27. D
3. C 8. C 13. C 18. C 23. C 28. C
4. C 9. D 14. E 19. A 24. B 29. A
5. B 10. D 15. B 20. D 25. A 30. A

352 Logaritmos

352. 352 8/11/01, 19:53


CAPÍTULO 8
T rigonometría

Sistemas de medición
de ángulos

Una unidad de medida para ángulos es el grado sexagesimal o sim-


plemente grado. El ángulo obtenido por una revolución completa en
sentido opuesto a las agujas del reloj mide 360 grados; por lo tanto,
1
un grado es 360 de una circunferencia.

Otra unidad de medida de ángulos es el radián.


Un radián es la medida del ángulo central de una circunferencia que
subtiende un arco de la misma longitud que su radio.

Relación entre ambos sistemas:

π
1º =
( )
180
rad
Nota: En general se omite la
1 rad =
( )
180 º
π palabra rad; así, un ángulo
puede medir 2π (en vez de
3
2π rad).
3

Trigonometría 353

353-354-355(2003) 353 20/11/02, 3:52 PM


Razones trigonométricas para
ángulos agudos
Sea a un ángulo agudo en el triángulo rectángulo ABC, de catetos
a y b y de hipotenusa c. Las razones trigonométricas son: seno,
coseno, tangente, cosecante, secante y cotangente y se definen:

B c
a cosec a = = hipotenusa
sen a = = cateto opuesto a cateto opuesto
c hipotenusa
c hipotenusa
b sec a = =
cos a = = cateto adyacente b cateto adyacente
c hipotenusa c
a
b
a cot a = = cateto adyacente
tg a = = cateto opuesto a cateto opuesto
b cateto adyacente

a
C b A

Identidades trigonométricas

Definición. Una identidad es una igualdad que se verifica


para todos los valores posibles de la variable.

Son identidades básicas:

1
1. cosec a =
sen a 5. ctg a = cos a
sen a
2. sec a = 1 6. sen2 a + cos2 a = 1
cos a
1 7. 1 + tg2 a = sec2 a
3. ctg a =
tg a
sen a 8. 1 + ctg2 a = cosec2 a
4. tg a =
cos a

Su demostración es consecuencia directa de la definición de razones


trigonométricas en 8.2 (ver ejercicio resuelto nº 10).

354 Trigonometría

353-354-355(2003) 354 20/11/02, 3:52 PM


CAPÍTULO 8

Funciones trigonométricas de un
ángulo cualquiera

Sea a un ángulo cualquiera en un sistema de coordenadas rectangulares


y sea P(x,y) un punto cualquiera de su lado terminal.

Si r = x2 + y2, definimos:

y
sen a = r II I
x
cos a = r
y P (x, y)
y
tg a = x
y

r a
csc a = y
o x
III IV
r
sec a = x

x
ctg a = y

Funciones trigonométricas de 60°, 30°


y 45°, 0°, 90°, 180° y 270°

función
ángulo sen cos tg cosec sec ctg
Los valores de las funciones
π 3 1 3 2 3 3
o 60º 2 trigonométricas de los ángulos
3 2 2 3 3
de 30º, 45º, 60º y de los ángu-
3 3 2 3 los cuadrangulares (su lado ter-
π 1 3
o 30º 2 3 2 3 minal coincide con un lado de
6 2
un cuadrante del sistema car-
π 2 2 2 2 tesiano) son usados frecuente-
o 45º 2 2 1 1 mente, y se presentan en la
4
tabla adjunta: (ver ejercicios
0 o 0º 0 1 0 indef. 1 indef. resueltos nos 12 y 13)

π
o 90º 1 0 indef. 1 indef. 0
2
π o 180º 0 -1 0 indef. -1 indef.


o 270º -1 0 indef. -1 indef. 0
6

Trigonometría 355

353-354-355(2003) 355 20/11/02, 3:52 PM


Funciones periódicas

Una función f es periódica si existe un número real positivo P tal que


f(x + P) = f(x) para todo valor de x en el dominio de f.
El número real P se llama período de la función f.
Las funciones seno y coseno, secante y cosecante son periódicas de
período 2π (360º).
Las funciones tangente y cotangente son periódicas de periodo π (180º).

Ejemplos:
a) sen a = sen (a + 2π) b) tg a = tg (a + 180º)
π 7π 3 3
sen = sen = tg 30º = tg 210º =
3 3 2 3

Funciones pares e impares


Una función f es par si f(–x) = f(x) para todo valor de x en el dominio de f.
Una función f es impar si f(–x) = – f(x), para todo valor de x en el dominio
de f.
Las funciones coseno y secante son funciones pares.
Las funciones seno, cosecante, tangente y cotangente son funciones
impares.
Ejemplos:
a) cos a = cos (– a) b) sen a = – sen (– a)
π π
cos
6
= cos –(6
)= 2
3
( 12 ) =
sen 30º = – sen (–30º) = – –
1
2

Ecuaciones trigonométricas

Definición. Una ecuación trigonométrica es aquella que con-


tiene la variable dentro de una expresión trigonométrica. Las
soluciones de estas ecuaciones son ángulos expresados en
grados o radianes.

Ejemplos:
1
a) sen x = b) 4tg x – 2cos x – 4 + 2 =0
2
c) tg (2x + π) = 1

Ver ejercicios resueltos nos 17 al 22.

356 Trigonometría

356-357(2003) 356 20/11/02, 3:54 PM


CAPÍTULO 8

Resolución de triángulos
no rectángulos

Se llama “resolver un triángulo” a determinar la medida de sus tres lados


y de sus tres ángulos interiores. Los siguientes teoremas se verifican en
todo tipo de triángulos.

8.9.1 Teorema del seno (o de los senos)


Sean a, b y g los ángulos interiores de un triángulo ABC cualquiera
y sean a, b y c los respectivos lados. Se cumple:
C
sen a sen b sen g g
= =
a b c
a
b
a b
A c B

8.9.2 Teorema del coseno


(o de los cosenos)
Sean a, b y g los ángulos interiores de un triángulo ABC cualquiera y
sean a, b y c los respectivos lados. Se cumple: C
a2 = b2 + c2 – 2bc cos a g

b2 = c2 + a2 – 2ac cos b
a b
c2 = a2 + b2 – 2ab cos g A B

8.9.3 Angulos de elevación y depresión


OBJETO
a es ángulo de elevación
Se llama ángulo de elevación al ángulo
formado por la horizontal y la recta que
une al observador con el objeto cuando el
objeto está sobre el observador.
a
OBSERVADOR

Se llama ángulo de depresión al ángulo


formado por la horizontal y la recta que OBSERVADOR b
une al observador con el objeto cuando el
objeto está bajo el observador.
b es ángulo de depresión. OBJETO

Ver ejercicios resueltos 14 al 16, 23 y 24

Trigonometría 357

356-357(2003) 357 20/11/02, 3:54 PM


Ejercicios 1. Exprese en radianes la medida de los ángulos:
resueltos ( )
π
a) 120º = 120º 180º rad = 2 rad
3
π
b) 54º = 54º
( ) π rad = 3 π rad
180º 10

2. Exprese en grados la medida de los ángulos siguientes:


π
a)
4
3
π=
4
3
π•
( ) 180º
π
= 240º b) 11
6
=
11
6
π•
( ) 180º
π
= 330º

2
3. Sea a un ángulo agudo y tg a = .
3
Determinemos las demás funciones.
B cateto opuesto
Solución: Como sabemos, tg a =
cateto adyacente
Dibujamos un triángulo ABC y a es uno de sus
ángulos agudos. Asignamos el valor de la tan-
2 gente, como en la figura, y luego determina-
mos la hipotenusa, aplicando el Teorema de
Pitágoras.

AB = 9+4 = 13
a
C 3 A
2 13
Así tenemos: sen a = csc a =
13 2
3 13
cos a = sec a =
13 3
2 3
tg a = ctg a =
3 2
4. Determine las funciones trigonométricas del ángulo b sabiendo que
2+b
sec b =
b
hipotenusa
Solución: Sabemos que sec b =
cat. adyacente
A
Aplicamos Teorema de Pitágo-
ras para determinar cateto AC
2+b
AC 2 + b2 = (2 + b)2

AC 2 = 4 + 4b + b2 – b2

b AC = 4 + 4b = 2 1+b
C b B
2 1+ b 2+ b
Por lo tanto: sen b = cosec b =
2+ b 2 1+ b
b 2+b
cos b = sec b =
2+ b b
2 1+ b b
tg b = ctg b =
b 2 1+ b

358 Trigonometría

358-359 358 09/11/2001, 10:43


CAPÍTULO 8

4
5. Determine las funciones trigonométricas del ángulo a si sen a =
3

cateto opuesto
Solución: Como sen a =
hipotenusa
Vemos que no es posible asignar dichos valores a los lados de un
triángulo rectángulo, pues un cateto no puede ser mayor (ni igual) a la
4
hipotenusa. Por lo tanto, no existe ningún ángulo cuyo seno sea .
3

Desafío: Averigüe qué son funciones acotadas y entre qué valores


están acotadas las funciones trigonométricas.

6. Demuestre que sen a • ctg a = cos a.

Analizamos el lado izquierdo de la igualdad y aplicamos las identida-


des que sean necesarias hasta obtener la expresión del lado derecho.

sen a • ctg a = sen a • cos a


sen a
= cos a

7. Demuestre que sen a (csc a – sen a) = cos2 a

Procediendo como en el caso anterior, tenemos que:

sen a (csc a - sen a ) = sen a ( 1 – sen a)


sen a

= sen a – sen2 a
sen a
= 1 – sen2 a
= cos2 a (identidad 6 pág. 354)

8. Demuestre que tg a = tg a – 1
1 – ctg a
En este caso analizaremos el lado derecho de la igualdad (es más
sencillo simplificar una expresión trigonométrica que amplificarla).
Aplicaremos las identidades que sean necesarias hasta obtener la
expresión del lado izquierdo.
tg a – 1 = tg a – 1
1 – ctg a 1– 1
tg a

= tg a – 1
tg a – 1
tg a

= tg a - 1 • tg a = tg a
tg a – 1

Trigonometría 359

358-359 359 09/11/2001, 10:43


Ejercicios 9. Demuestre que cos4 b + sen2 b = cos2 b + sen4 b
resueltos Procediendo como en los casos anteriores, analizamos el primer miem-
bro de la igualdad y aplicamos las identidades correspondientes:

cos4 b + sen2 b = (cos2 b)2 + sen2b

= (1 – sen2b)2 + sen2b

= 1 – 2 sen2b + sen4 b + sen2b

= 1 – sen2b + sen4 b

= cos2 b + sen4 b

10. Demuestre que sen2 a + cos2 a = 1

B Consideremos el M rec-
tángulo ABC y a uno de sus
ángulos agudos.
Por Teorema de Pitágoras se
c cumple:
a
a2 + b2 = c2 /:c2

a
C b A () ()
a 2+ b 2=1
c c

Aplicando las definiciones de razones trigonométricas, tenemos:

sen2 a + cos2 a = 1

11. Sea a un ángulo en posición estándar en un sistema de ejes coordena-


dos, esto es, el lado inicial de a coincide con la parte positiva del eje
x, el vértice del ángulo es el origen del sistema, y sea P(-4,3) el punto
del lado terminal de él. Determine todas las funciones trigonométricas
de a.

Solución: Primero deter-


minemos la longitud del P
segmento OP = r. 3

3
r= 32 + 42 fi r = 5
a
–4 0
4

360 Trigonometría

360-361 360 09/11/2001, 10:48


CAPÍTULO 8

y ahora aplicamos las definiciones de las funciones trigonométri-


cas de a: (x = – 4; y = 3; r = 5)

y 3 r 5
sen a = = cosec a = =
r 5 y 3
x 4 r 5
cos a = = – sec a = = –
r 5 x 4
y 3 x 4
tg a = = – cot a = = –
x 4 y 3

π
12. Encuentre las funciones trigonométricas para a = 60º ( o a = )
3
C
Solución: Consideremos
el M ABC equilátero de
lado 2 y sea CD su altura.
En M rectángulo ADC se 2 2
tiene que h2 + 1 = 4 . h

h= 3 a
y DAC = 60º A
D B
1
3
Entonces: sen 60º = cosec 60º = 2 = 2 3
2 3 3
1
cos 60º = sec 60º = 2
2
1 3
tg 60º = 3 cot 60º = =
3 3

Desarrolle el mismo ejercicio pero para un triángulo equilátero de lado


3, 5, a.
π
13. Encuentre las funciones trigonométricas para a = 90º (o )
2
Solución.

El punto (0,1) pertenece al (0, 1)


lado terminal de a.
a
Así: x = 0; y = 1; r = 1

y r
sen 90º = =1 cosec 90º = = 1
r y
x r
cos 90º = =0 sec 90º = = indefinida
r x
y x
tg 90º = = indefinida cot 90º = =0
x y

Trigonometría 361

360-361 361 09/11/2001, 10:48


Ejercicios 14.Desde un punto P situado a nivel el suelo, el ángulo de ele-
resueltos vación de la cima de una torre es de 30º. Si la distancia
entre el punto P y la base de la torre es 12 metros, deter-
mine la altura de ésta.

30º

P
12 m
La figura ilustra la situación planteada.
El triángulo determinado es rectángulo; un cateto es informa-
ción dada y el otro cateto es la incógnita. Una función que
relaciona los dos catetos es la tangente (la otra es la cotan-
gente)

h
Así tg 30º = 12

h
Pero tg 30º = 1 entonces = 1
3 12 3
despejando nos queda: h = 12
3
y racionalizando : h = 4 3

Solución: la torre mide 4 3 metros, aproximadamente 6,9


metros.

15. Desde un punto P situado a nivel del suelo se observa


la punta de una chimenea bajo un ángulo de elevación
de 30º y acercándose 20 metros desde otro punto Q el
ángulo de elevación es de 60º. Determine la altura de la
chimenea y la distancia desde ésta hasta el primer punto
de observación (P).

362 Trigonometría

362-363 362 20/11/02, 3:55 PM


CAPÍTULO 8

La figura muestra la situación


planteada.
Debemos determinar
h y d = x + 20. h
h
Tenemos tg 60º = x
60º 30º
h
y tg 30º = x + 20
x Q P
20 m
3 1
Como tg 60º = y tg 30º = se tiene
3
h 3 y (2) h 1
(1) x = x + 20 = 3
x 3 1
de (1) h = x 3 reemplazando en (2) =
x + 20 3
Así 3x = x + 20
2x = 20
x = 10
Por lo tanto, h = 10 3

Solución. La chimenea mide 10 3 metros (17,3m) y la distancia


desde ella al primer punto de observación es 30 metros.

16. Un niño eleva un volantín con


una cuerda tensa que forma un
ángulo de elevación de 60º con
la horizontal. ¿A qué altura se
encuentra el volantín del suelo
si la longitud de la cuerda es de
18 m x
18 metros y el niño mide 1.50
metros (o el niño tiene la cuerda
a 1.50 m del suelo)? d
60º

La figura representa la situación


planteada. En este caso, una 1.5 m
información es el largo de la
cuerda, lo que corresponde a la
hipotenusa del triángulo. Enton-
ces aplicamos la función seno.

x x 3
sen 60º = fi fi 3
18 18 = 2 x=9

La distancia desde el volantín al suelo es: d = x + 1,5


d = 9 3 + 1,5
d ª 17 m.

Desafío: Averigüe y construya los gráficos de las funciones trigonométricas.

Trigonometría 363

362-363 363 20/11/02, 3:55 PM


Ejercicios 1
17. Resuelva la ecuación: sen x =
resueltos 2

1
Solución: Sabemos que sen 30º = ; por lo tanto, una solución particular
2
es x1 = 30º.
Pero además en la circunferencia geométrica (de radio 1) la función
seno queda definida por el eje y, y también es positiva en el 2º cua-
drante; por lo tanto, ahí hay otra solución particular.
y
Observando la circunferencia
unitaria (de radio 1), vemos 1
que: 2
150º 30º
x2 = 150º también es solución x
de la ecuación
Nótese que 150º = 180º – 30º

Así, las soluciones particulares de la ecuación son:

x1 = 30º
( 6π )
o

y x2 = 150º
( 6π )
o 5

y para obtener las soluciones generales, agregamos a cada solución


particular, múltiplos del periodo.

π
Y las soluciones generales son: x1 = + 2kπ
6
π
x2 = 5 + 2kπ
6

y
2
18. Resolver la ecuación: cos x =
Solución: Sabemos que para
2 π
4
π
x1 = 45º (o )
4
2
el coseno vale
2
También sabemos que el co- 2 x
seno es positivo en el 1º y 4º 2
cuadrante (pues queda deter-
minado por la coordenada x),

entonces también hay una solu-
4
ción en el 4º cuadrante, que es
x2 = 315º (360º - 45º)
π
Y las soluciones generales son: x1 = + 2kπ
4
π
x2 = 7 + 2kπ
4

364 Trigonometría

364-365 364 20/11/02, 3:57 PM


CAPÍTULO 8

19. tg x = 3
Determinamos la solución particular primera, o el “ángulo de referen-
π
cia”, este es x = 60º (o )
3
Y luego determinamos el otro
cuadrante donde la tangente es
positiva, este es el tercer cua-
drante. En ese cuadrante está
la 2a solución particular y es
x1 = 60º
x2 = 240º = 180º + 60º
π
Las soluciones son: x1 = + kπ
3

x2 = + kπ
3
20. 2cos 2x = 3
3
Nos queda cos 2x =
2
3
Llamemos 2x = y, tenemos cos y =
2
De acuerdo con los análisis anteriores, obtenemos:
y1 = 30º e y2 = 330º
π π
y1 = + 2kπ y2 = 11 + 2kπ
6 6
pero y = 2x, entonces las soluciones son:
π
x1 = + kπ y2 = 11 π + kπ
12 12

21. 4 sen2 x tg x – tg x = 0
Factorizamos: tg x (4 sen2 x – 1) = 0
tg x (2 sen x – 1) (2 sen x + 1) = 0
Un producto de 3 factores es cero si cualquiera de ellos es cero. Así:
y
tg x = 0 (tg a = ) fi x1 = 0º
x
x2 = 180º
1
2 sen x = 1 fi sen x = fi x3 = 30º
2
(ángulo de referencia 30º) x4 = 150º

1
2 sen x = –1 Q sen x = – Q x5 = 210º
2
(ángulo de referencia 30º) x6 = 330º
π
6
π
Solución: {kπ, + 2kπ; 150º + 2kπ; 210º + 2kπ; 330º + 2kπ}
6

Trigonometría 365

364-365 365 20/11/02, 3:57 PM


Ejercicios 22. 1 – 2 sen2 x = cos x

resueltos Aplicamos identidades para expresar todas las funciones en términos de


coseno.

1 – 2(1 – cos2 x) = cos x


1 – 2 + 2 cos2 x = cos x
2 cos2 x – 1 = cos x

(esta es una ecuación de 2º grado en cos x)

2 cos2 x – cos x – 1 = 0

Factorizando, obtenemos:

(2 cos x + 1) (cos x – 1) = 0

Entonces:
1
2 cos x + 1 = 0 Q cos x = – Q x1 = 240º
2
(ángulo de referencia 60º) x2 = 300º

cos x – 1= 0 Q cos x = 1 Q x = 0º
(coordenada x)
60º 60º
π π
Solución: {4 + 2kπ, 5 + 2kπ, 2kπ}
3 3

23. Resolver el triángulo ABC, dados:


a = 36º b = 64º a = 12

Según el teorema del seno.


a b 12 b
= fi =
sen a sen b sen 36º sen 64º
Usando calculadora, obtenemos:
12 b
=
0,5878 0,8988
de donde b = 18,35

Ahora, como a = 36º y b = 64º, concluimos que g = 80º y aplicamos


nuevamente teorema del seno:
a c 12 c
= fi =
sen a sen g sen 36º sen 80º
Usamos la calculadora para obtener:
12 c
=
0,5878 0,9848
y tenemos c = 20,1

366 Trigonometría

366-367 366 10/11/2001, 16:41


CAPÍTULO 8

24. Resolvamos el triángulo ABC, dados


a = 18, b = 25 y c = 12.
Por teorema del coseno:
a2 = b2 + c2 – 2bc cos a

de donde:
b2 + c2 – a2
cos a =
2bc
625 + 144 – 324 445
cos a = = = 0,7417
600 600
Aquí obtenemos a = 42º10’

(en este caso, al resolver la ecuación resultante sólo debemos conside-


rar la solución entre 0º y 180º)

Para determinar b podemos aplicar nuevamente el teorema del coseno:

b2 = a2 + c2 – 2ac cos b
a 2 + c2 – b 2
cos b =
2ac
324 + 144 – 625 –157
cos b = = = –0,3634
432 324
De donde obtenemos b = 111º20’
Para determinar g, calculamos el suplemento de a + b.
g = 180º – (42º10’ + 111º20’)
g = 26º30’

Ejercicios
1. Demuestre la equivalencia entre ambas unidades de medición de ángulos.
2. Exprese en radianes la medida de los siguientes ángulos:
a) 45º e) 225º
b) 15º f) 210º
c) 150º g) – 60º
d) 300º h) –135º
3. Exprese en grados la medida de los siguientes ángulos:

a) 3 π d) π g) 1
4 16
b) 2 π e) – π h) 1
9 3 3
c) 7 π f) –5 π i) – 2
5 6

Trigonometría 367

366-367 367 10/11/2001, 16:41


Ejercicios

4. Determine las funciones trigonométricas del ángulo a sabiendo que sen a = 1


3
5. Determine las funciones trigonométricas del ángulo a sabiendo que sec a = 5
3
6. Determine las funciones trigonométricas del ángulo b sabiendo que ctg b = 5

7. Determine las funciones trigonométricas del ángulo g sabiendo que cosec g = b


1+b
8. Determine entre qué valores están acotadas las funciones seno y coseno.

9. Determine entre qué valores están acotadas las funciones tangente y cotangente.

10. Determine entre qué valores están acotadas las funciones secante y cosecante.

11.Verifique las identidades trigonométricas a partir de las funciones dadas.

a) (1 – sen2 a) sec2 a = 1

b) sen a • tg a = sec a - cos a

c) cos b • cot b = cosec b - sen b

2
d) sec2 j = cosec2 j
tg j
e) sen g (cosec g - sen g) = cos2 g

f) 1 + sec2 b = cosec b
tg b + sen b
g) tg2 a - sen2 a = tg2 a • sen2 a

h) cosec2 a • tg2 a - 1 = tg2 a

i) sen2 a • sec2 a = sec2 a -1

j) (1 + tg2 a) (1 – sen2 a) = 1

k) 1 + cos m + sen m = 2 cosec m


sen m 1 + cos m

l) 1 = cosec t – cot t
cosec t + cot t
m) sen2 a - cos2 a = sen4 a - cos4 a

n) 1 + cos a + sen a = cos a + 1


sen a cos a sen a • cos a

o) 1 – sen d = cos d
cos d 1 + sen d

368 Trigonometría

368-369 368 09/11/2001, 11:18


CAPÍTULO 8

12. Determine las


funciones trigonométricas 2
del ángulo a de la figura
a

5
13. Determine las funciones trigonométricas del 1
ángulo b de la figura.

b
14. Determine las funciones 2 3
trigonométricas del C
ángulo g de la figura.

M ABC isósceles con AB = 10,


AC = BC = 12.
Sugerencia : trace la altura hc.
g
A B

15. Si sen a = 0,3 determine las demás funciones trigonométricas de a.

16. Si cot b= 1,2 determine las demás funciones trigonométricas de b.

17. Si tg g = 1 determine el valor de: sen g – cos g.


b

18. Si sec d = 1 + a determine el valor de: cos2 d – 1.


1–a

19. Verifique que: 2 sen π • cos π • tg π = 1


4 4 4

20. Verifique que: sen 30º • cos 60º + cos 30º • sen 60º = tg 45º.

21. Determine las funciones trigonomé- y coincide con la recta de ecua-


tricas del ángulo b sabiendo que b ción x – 3y = 0.
está en posición estándar y que el
punto P(1,-2) pertenece al lado ter- 24. Determine las funciones trigonomé-
minal de él. tricas del ángulo b sabiendo que el
lado terminal está en el 4º cuadrante
22. Determine las funciones trigonomé- y es paralelo a la recta de ecuación
tricas del ángulo b sabiendo que el 3x + 2y + 3 = 0
punto Q(2,-5) pertenece al lado ter-
minal de él. 25. Determine las funciones trigonomé-
tricas del ángulo b sabiendo que
23. Determine las funciones trigonomé- el lado terminal está en el 2º cua-
tricas del ángulo b sabiendo que el drante y es perpendicular a la recta
lado terminal está en el 3er cuadrante de ecuación x – y = 3

Trigonometría 369

368-369 369 09/11/2001, 11:19


Ejercicios
Nota: Para resolver ejercicios que inclu-
26. A partir del ejercicio resuelto 12, verifi- yan ángulos cuyas funciones trigono-
que el valor de las funciones trigonomé- métricas se desconozcan se debe hacer
π uso de la calculadora.
tricas del ángulo de 30º o (
6 )
39. Desde lo alto de un edificio de 25m
27. Compruebe el valor de las funciones tri- de altura se obtiene una medición de
π
gonométricas del ángulo de 45º o (4
. ) 35º para el ángulo de depresión de un
Sugerencia: considere un triángulo rec- quiosco situado en el mismo plano del
edificio. ¿A qué distancia se encuentra
tángulo isósceles de lado (catetos) 1.
el quiosco del edificio?
28. Determine las funciones trigonométricas del
π 40. Desde lo alto de un acantilado se obser-
(
ángulo de 180º (o π) y de 270º o 3 .
2
) van dos botes bajo ángulos de depresión
de 20º y 30º, respectivamente. Deter-
29. Evalúe: sen 30º + cos 30º
mine la altura del acantilado sabiendo
que la distancia entre los botes es 35m.
30. Evalúe: 1 – tg2 45º
41. Un avión despega en un ángulo de 10º
1 + tg 60º m
31. Evalúe: y vuela con una velocidad de 75 .
1 – tg 60º seg
¿Cuánto tardará en alcanzar una altitud
32. Verifique la igualdad: de 15.000 metros?
ctg 30º – 1
= ctg 30º 42. Una escalera de 8 metros se encuentra
1 – tg 30º
apoyada en una pared y forma con ésta
2
33. Verifique 1 + sen 45º = sec2 45º + tg2 45º un ángulo de 40º. Calcule la distancia
cos2 45º entre la pared y el pie de la escalera.
π
π 2 tg
34. Verifique tg = 6 Desafíos.
3 1 – tg2 π
6
43. Grafica todas las funciones trigonomé-
π π π tricas asignando algunos valores y apli-
35. Verifique cos 3 = cos2 6 – sen2 6
cando las propiedades de periodicidad,
36. Un cohete es lanzado a nivel del suelo, paridad e imparidad y acotaciones.
en un ángulo constante de 60º hasta una
distancia de 3.000 metros. Determine a
qué altura se encuentra del suelo. 44. Resolver las siguientes ecuaciones
(Hallar todas las soluciones menores o
37. Sabiendo que el ángulo de elevación iguales a 2π):
del sol, a cierta hora del día es de 30º,
1) tg x = 1
determine la longitud de la sombra que
proyecta una persona que mide 1,6 m. 2) sen x = – 1
2
3) sen x + 1 = 0
38. Desde un punto P situado a 12 metros
de un edificio se observa un letrero 4) cos x – 1 = 0
luminoso que está en una ventana del
5) tg2 x – 1 = 0
edificio, bajo un ángulo de elevación
de 30º, y desde el mismo punto P se 6) sec y = 2
observa el techo del edificio bajo un 7) 2 cos t + 1 = 0
ángulo de elevación de 60º. Calcule la
altura del edificio. 8) 2 sen2 x – sen x = 0

370 Trigonometría

370-371 370 09/11/2001, 11:26


CAPÍTULO 8

9) 2 cos x = – 3 45. Resolver los triángulos ABC, dados:

10) sec2 t – 2 = 0 1) a = 30 b = 35 b = 35º

11) 2 sen x = csc x 2) a = 25 c = 94 a = 57º

12) 4 sen x = 3 csc x 3) a = 64 a = 28º b = 34º

13) 2 cos x = ctg x 4) a = 15º b = 55º c = 104

14) sec2 x = 3 tg2 x – 1 5) b = 58 b = 58º g = 18º


15) sen x = cos x 6) a = 100 b = 100 c = 40
16) tg x = 3 ctg x
7) a = 70 b = 15 g = 60º
17) tg2 t + tg t = 0
8) b = 10 c = 20 a = 40º
18) sen2 t + sen t cos t = 0
9) a = 14 b = 28 c = 50
19) 3 – tg2 y=0 (analizar)

20) 2 cos2 x + cos x = 0 46.Una pequeña embarcación debe diri-


girse desde una isla a un puerto en el
21) sen2 x – sen x – 6 = 0 continente, que se encuentra a 240 km
(indicación: analice soluciones) de la isla. Debido a la fuerte corriente,
después de navegar un tiempo, la embar-
22) cos x + 2 sec x + 3 = 0
cación se encuentra a 140 km de la isla
y a 35º dirección N.E. Determine a qué
23) sen2 x + 2 sen x + 1 = 0
distancia aproximada se encuentra del
24) 2 sen2 x – 3 sen x + 1 = 0 puerto y qué dirección debe tomar para
corregir el curso.
25) 2 cos 2 x – sen x – 1 = 0
47. Para llegar a casa Juanito debe cruzar un
26) cos 3x = 1 río de 30 m de ancho; él se encuentra
justo frente a su casa, pero en la orilla
27) 2 ctg2 x + csc2 x – 2 = 0 opuesta. La corriente lo desvía 28º río
arriba. ¿A qué distancia se encontrará de
28) 2 cos2 t + 3 cos t + 1 = 0 su casa cuando logre atravesar el río?

29) 1 – sen t = 3 cos t 48. Un avión vuela 250 km desde un punto


(indicación: eleve al cuadrado y ana- A en dirección 70º y luego 120 km en
lice soluciones) dirección 220º. ¿A qué distancia aproxi-
mada se encontrará del punto A?
30) 4 sen2 u – 1 = 0
49. Dos vehículos salen de una ciudad al
31) sen x + cos x = 1 mismo tiempo y circulan en carreteras
(indicación: eleve al cuadrado y ana- rectas que forman entre sí un ángulo
lice situaciones) de 70º. Si viajan a 90 km/h y 110
km/h respectivamente, ¿a qué distancia
32) 3 tg2 x – sec2 x – 5 = 0
se encontrarán después de 40 minutos?

Trigonometría 371

370-371 371 09/11/2001, 11:26


Ejercicios
50. Un bote pesquero utiliza un equipo de ondas sonoras para detectar un banco de peces
que se encuentra a 3 km de la embarcación y que se mueve a razón de 12 km/h en
dirección 35º N.E. Si el bote avanza a razón de 20 km/h, determine cuánto tiempo
tardará en alcanzar el banco de peces.

32
51. Al observar el sol desde la tierra se ve bajo un ángulo de 32’ (minutos, es decir, grados)
60

0,53º
TIERRA

SOL

Si la distancia entre la tierra y el sol es aproximadamente 150.000.000 km, calcule el


diámetro del sol.

Soluciones
π π
2. a) e) 5
4 4
π π
b) f) 7
12 6
π π
c) 5 g) –
6 3
π π
d) 5 h) – 3
3 4
3. a) 135º d) 11,25º g) 57,32º
b) 40º e) –60º h) 19,1º
c) 252º f) –150º i) 114,6º

1 2 2 2
4. sen a = cos a = tg a =
3 3 4
3 2
csc a = 3 sec a = ctg a = 2 2
4
4 3 3
5. sen a = cos a = tg a =
5 5 4
5 5 4
csc a = sec a = ctg a =
4 3 3

372 Trigonometría

372-373 372 09/11/2001, 11:29


CAPITULO 8
CAPÍTULO

5
6. sen a = 1 cos b = tg b = 1
6 6 5

6 6 5
csc b = sec b = ctg b =
5

7. No existe un ángulo agudo en tal triángulo (lo que no existe es ese triángulo
rectángulo)

8. – 1 £ sen a £ 1
– 1 £ cos a £ 1

9. La tangente y la cotangente varían entre menos infinito e infinito positivo. No tienen


restricciones.

10. sec a £ – 1 o sec a ≥ 1

csc a £ – 1 o sec a ≥ 1

11. Las demostraciones debe hacerlas el estudiante. Ver ejercicios resueltos nº 6 al 10.
21 2 21
12. sen a = cos a = tg a =
5 5 5
5 5 2
csc a = sec a = ctg a =
21 2 21

11 1
13. sen b = cos b = tg b = 11
2 3 2 3

2 3 1
csc b = sec b = 2 3 ctg b =
11 11

119 5 119
14. sen g = cos g = tg g =
12 12 5

3 91 3
15. sen a = cos a = tg a =
10 10 91

10 10 91
csc a = sec a = ctg a =
3 91 3

5 6 5
16. sen b = cos b = tg b =
61 61 6

61 61 6
csc b = sec b = ctg b =
5 6 5

Trigonometría 373

372-373 373 09/11/2001, 11:29


Soluciones
1–b
17.
1+ b2
18. –
4a
(1 + a)2
1 1
19. 2 • • =1
2 2
1 1 3 3
20. • + • =1
2 2 2 2
2 1
21. sen b = – cos b = tg b = –2
5 5
5 1
csc b = – sec b = 5 ctg b = –
2 2
5 2 5
22. sen b = – cos b = tg b = –
29 29 2
29 29 2
csc b = – sec b = ctg b=–
5 2 5
–1 –3 1
23. sen b = cos b = tg b =
10 10 3
10
csc b = – 10 sec b = – ctg b = 3
3
–3 2 3
24. sen b = cos b = tg b = –
13 13 2
13 13 2
csc b = – sec b = ctg b = –
3 2 3
1 1
25. sen b = cos b = – tg b = – 1
2 2
csc b = 2 sec b = – 2 ctg b = – 1
26. Ver ejercicio resuelto nº 12 y tabla de la página 343.
27. Ídem.
28. Ídem.

29. 1+ 3
2
30. 0

31. – (2 + 3 )
3–1 3–1
32. = = 3 = ctg 30º
1– 1 3–1
3 3
1
1+
33. Primer miembro: 2 =3 y Segundo miembro: 2 + 1 = 3
1
2 3 2
3 1 = 3 – 1
34. 3 = 35. 2 4 4
2
3
1 = 1
3 = 3 2 2

374 Trigonometría

374-375 374 20/11/02, 3:58 PM


CAPITULO 8
CAPÍTULO

36. 2.598 m es la altura a la que se encuentra el cohete del suelo.


37. La persona proyecta una sombra de 92,3 cm.
38. El edificio mide 20,78 m. (hay información de más en el enunciado).
39. 17,5 m
40. 34,47 m
41. 19 min 11,8 seg.

42. 5,14 m
3π π 3π 5π 7π
44. 1) 45º, 225º 2) 210º, 330º 3) 4) 2π 5) , , ,
2 4 4 4 4
π 5π 2π 4π π 3π π 11π
6) , 7) , 8) , , , 9) 150º, 210º
3 3 3 3 2 2 6 6
π 3π 5π 7π π 2π 4π 5π
10) , , , 11) 45º, 135º, 225º, 315º 12) , , ,
4 4 4 4 3 3 3 3
π 3π π 3π
13) , , , 14) 35,26º ; 144,74º ; 215,26º ; 324,74º
2 2 4 4
π 5π 3π 7π
15) , 16) 60º, 120º, 240º, 300º 17) 0, , π,
4 4 4 4
3π 7π π 2π 4π 5π π 2π 4π 3π
18) 0, , π, 19) , , , 20) , , ,
4 4 3 3 3 3 2 3 3 2
3π π π 3π 11π
21) No tiene 22) π 23) 24) , , ,
2 6 2 2 6
3π π 3π 5π 7π
25) 26) 0º, 120º, 240º, 360º 27) , , ,
2 4 4 4 4
2π 4π
28) , π, 29) 90º, 330º 30) 30º, 150º, 210º, 330º
3 3
31) 0º, 90º 32) 50,77º ; 129,23º ; 230,77º ; 309,23º

45. 1) a = 29,45º g = 115,55º c = 55,05

2) no existe un triángulo con esas medidas 3) g = 118º b = 76,23 c = 120

4) g = 110º a = 28,64 b = 90,66 5) a = 104º a = 66,36 c = 21,13

6) a = b = 78,465º ; g = 23,07º 7) a = 71,74º b = 48,26º c = 63,84

8) No existe un triángulo con esas medidas

9) No existe un triángulo con esas medidas (28 + 14 < 50)

46. 149 km 12,5º SE 47. 34 m

48. 158 km 49. 13,35 km


1
50. Si el barco avanza en dirección 33,6º NE alcanza al banco de peces en de hora.
4
1
Estudie usted otras posibilidades. ¿Se podrá demorar más de hora? ¿Cuál será el ángulo
2
necesario para alcanzar a los peces en el mínimo de tiempo?

51. 1.387.546,5 km.

Trigonometría 375

374-375 375 20/11/02, 3:59 PM


Prueba de selección múltiple

1. Exprese en radianes 5. Si sec a = 3 , 10. Si sen a = cos a


270º ¿cuál es el valor de cos2 a? entonces el valor de a es:
2π A. 0º y 180º
A.
2 A. 3 B. 45º y 225º
1 C. 135º y 315º
3π B.
B. 3 D. todo a E R
2
1 E. ninguno
3π C. 1
C. 9 11. Si cos x = entonces
4 2
D. 9
4π el valor de x es:
D. E. otro
3 π
A.
3
3
E. 6. Si tg a = 0,7 π
2 ¿cuánto vale ctg a? B.
6
2. Exprese en grados A. 0,3
π
C.
7π 4
6
radianes B. –0,7 3π
D.
4
A. 210º C. 7
10 E. otro
D. 10
B. 21º 7 12. Si ctg x = –1, entonces
E. otro ¿cuál es el valor de x?
C. 280º π
A.
7. Si 1 – tg b = 2, 4
D. 420º π
¿cuánto vale b? B.
2
E. 350º A. 90º y 270º 3π
C.
2 4
3. Si sen a = B. 0º y 180º 5π
5 D.
C. 45º y 225º 4
entonces tg a =

D. 135º y 315º E.
2 3
A.
21 E. 60º y 30º 13. ¿Cuál de las afirmaciones
2 siguientes es verdadera?
B. 8. Si cosec a = 1,
21 1
¿cuánto vale a? A. sen a =
21 cos a
C. A. 0º B. tg a - cos a = sec a
2
5 B. 90º C. 1 + sec2 a = tg2 a
D.
21 C. 180º D. sen2 90º + cos2 90º = tg2 45º
21 D. 270º
E. E. sec a - cosec a = 1
5 E. 45º
4. sen2 60º + cos2 30º = 14. ¿Cuál de las afirmaciones
3 9. Si tg2 a = 2 – 1, siguientes es falsa?
A. ¿cuánto vale sec2 a?
2 A. sen2a + cos2a = sec2a - tg2a
3 A. 1 + 2
B. B. cos b • sec b = 1
4 B. 1 – 2
C. 1 C. tg 30º = ctg 60º
C. 2
D. 3 1
D. 2 D. sen2 45º =
2
E. 1+ 3 E. otro E. sec 45º + cosec 45º = 2
4
376 Trigonometría

376-377 376 20/11/02, 4:00 PM


CAPÍTULO 8

15. A partir de la figura, 19. La expresión 2 cos2 a + sen2 a a b


¿qué relación es falsa? es equivalente a: A. =
sen 45º sen 35º

A. 2 – sen2 a sen 100º sen 45º


B. =
2 B. 2 + sen2 a c b
C. 2 sen 100º sen 45º
C. =
D. 2 – cos2 a c a
b E. 2 + cos2 a sen 35º c
D. =
sen 100º a
a 2 2 20. La expresión sen2 a • sec2 a
es equivalente a: a sen 100º
E. =
sen 35º c
A. tg a
A. a = b
B. tg2 a 24. En M ABC de la figura,
B. cos a = 2 cosec b =
2 C. ctg a
C. tg b = 1
D. ctg2 a
D. cosec b = 1
2 e) 1 b
E. tg a = cot a a
21. Si sen a = , entonces
a+1
cos a = r
16. Al expresar en radianes, p
a+1
un ángulo de 15º es A.
a
equivalente a:
2a + 1
B.
π a+1
A. a
8 a+1
C. q
π 2a + 1
B.
10 q
C.
π D. 2a+ 1 A.
a+1 r
12
π 2a+ 1 p
D. E. B.
15 a r
π
E. 22. Si ctg b = 2, entonces sen b = r
30 C.
p
17. Al expresar en grados un 1
5π A. r
ángulo de es equiva- 5 D.
12 q
lente a: 2
B.
A. 300º 5 p
E.
1 q
B. 150º C.
3
C. 75º 2 25. El valor de
D.
D. 30º 3 π π
1 sen + sen 3 es:
E. 15º E. 2 2
2
A. 1
18. La expresión cosec2 a - 1 23. A partir de la figura, ¿qué
es equivalente a: relación es correcta? B. 2
A. 1 + ctg2
a b
a C. 0
B. ctg2 a - 1
35º
C. 1 – tg2 a D. –1
D. ctg2 a
c 45º E. no está definido
E. tg2 a

Trigonometría 377

376-377 377 20/11/02, 4:00 PM


Prueba de selección múltiple

26. ¿Cuál es el valor de 28. Si sec b = 1,5 ¿cuál es el 30. ¿Cuál(es) de la(s) siguiente(s)
sen 30º • cos 30º • ctg 30º valor de sen b? relaciones es o son verdade-
ras?
1 3
A. A. π π
2 I) sen = cos
13 3 6
1 2
B.
4 B. π π π
13 II) tg – ctg 3 = csc
4 4 6
C. 2 5 π π
C. III) sen 2 • cos2 =1
3 4 4
D. 4
D. 5 A. Sólo I
3 2
E. B. Sólo II
4 2
e) C. Sólo III
3
27. ¿Cuál es el valor de
tg 60º • ctg 60º • sen2 60º? 29. ¿Cuál es el valor de D. Sólo I y II
π π E. I, II y III
3 tg + tg ?
A. 4 2
2
A. 1
9
B.
2 1
B.
9 2
C.
4 C. 0
3 1
D. D.
4 4
3 E. no está determinado
E.
2

Soluciones

1. B 11. B 21. D
2. A 12. C 22. A
3. B 13. D 23. B
4. A 14. E 24. D
5. B 15. D 25. C
6. D 16. C 26. E
7. D 17. C 27. D
8. B 18. D 28. C
9. D 19. A 29. E
10. B 20. E 30. A

378 Trigonometría

378 378 09/11/2001, 11:44


CAPÍTULO 9
N úmeros
complejos

Definiciones y
propiedades 9.1

Definición: Un número complejo es un par ordenado de números


reales.
El conjunto de los números complejos lo simbolizamos por k y
cada número complejo por la letra z.
k = {z = (a, b) / a E R, b E R}.
Si z = (a, b) E k, “a” se llama parte real del complejo z y se
denota por Re(z), y “b” se llama parte imaginaria del complejo
z y se denota por Im(z).

9.1.1 Igualdad
Dados dos complejos z1 = (a, b) y z2 = (c, d).

z1 = z2 si y sólo si { ab == cd
9.1.2 Representación geométrica
Geométricamente el conjunto de los números complejos representa el plano cartesiano.
Existe una relación biunívoca entre los elementos del conjunto k y el conjunto
de puntos del plano.
Sea z = (a, b) E k
• Si b = 0, entonces el complejo z es de la forma (a, 0) y se asimila al número real a.
Geométricamente se representa en el eje horizontal.
• Si a = 0, entonces el complejo z es de la forma (0, b) y decimos que es un imaginario
puro. Se representa geométricamente en el eje vertical.

Números complejos 379

379-381 379 10/11/2001, 16:44


9.1.3 Forma canónica de un complejo
El complejo z = (a, b) se puede escribir en su forma canónica como z = a + bi, donde i es
la unidad imaginaria. i = se cumple i2 = – 1.

9.1.4 Operaciones con números complejos


• SUMA.
Sean
z1 = (a, b) z2 = (c, d)
z1 + z2 = (a, b) + (c, d) = (a + c, b + d)
Sean
z1 = a + bi z2 = c + di
z1 + z2 = (a + bi) + (c + di) = a + c + ( b + d) i

• PRODUCTO.
Sean
z1 = (a, b) z2 = (c, d)
z1 • z2 = (a, b) • (c, d) = (ac – bd, ad + bc)
Sean
z1 = a + bi z2 = c + di
z1 • z2 = (a + bi) (c + di) = ac – bd + (ad + bc)i

Observación: Para efectuar las operaciones con números complejos


escritos en su forma canónica se procede como en el conjunto de
los números reales considerando que i2 = –1.

9.1.5 Estructura del conjunto (k, +, •)


El conjunto de los números complejos con las operaciones de
suma y producto tiene estructura de Campo.

• PROPIEDADES DE LA SUMA.
Sean z1, z2, z3, E k.
1. La suma es cerrada en k
Iz1, z2 E k, z1 + z2 E k.
2. La suma es asociativa en k
Iz1, z2 y z3 E k, (z1 + z2) + z3 = z1 + (z2 + z3).
3. La suma es conmutativa en k
Iz1, z2 E k, z1 + z2 = z2 + z1.
4. Existe un elemento neutro para la suma en k
Iz E k, H! w E k tal que z + w = z.

380 Números complejos

379-381 380 10/11/2001, 16:44


CAPÍTULO 9

5. Existe un elemento inverso para la suma en k


I z E k, H z` E k tal que z + z` = w (w neutro aditivo).
Nota: El neutro aditivo es el complejo (0, 0) o 0 + 0i = 0
El inverso aditivo de (a, b) es (– a, – b). Escrito en forma
canónica, el inverso aditivo de a + bi es – a – bi.
El inverso aditivo de z se denota por – z.
• PROPIEDADES DEL PRODUCTO.
Sean z1, z2, z3 E k.
1. El producto es cerrado en k
Iz1, z2 E k, z1 • z2 E k.
2. El producto es asociativo en k
Iz1, z2 y z3 E k, (z1 • z2) z3 = z1 (z2 • z3).
3. El producto es conmutativo en k
Iz1, z2 E k z1 • z2 = z2 • z1
4. Existe un elemento neutro para el producto en k
Iz E k, H! w E k tal que z • w = z.
5. Existe un elemento inverso para el producto en k
Iz E k, H z’ E k tal que z • z’ = w (neutro multiplicativo).
Nota: El neutro multiplicativo es el complejo (1, 0) o 1 + 0i = 1.
a , 2 –b 2
El inverso multiplicativo de (a, b) es ( 2
a +b 2 a +b )
Escrito en forma canónica, el inverso multiplicativo de
a b
a + bi es 2 – 2 i
a + b2 a + b2

El inverso multiplicativo de z se denota por z–1.


El elemento (0, 0) no tiene inverso multiplicativo.
• DISTRIBUTIVIDAD DEL PRODUCTO SOBRE LA SUMA
Sean z1, z2, z3 E k.
Iz1, z2 y z3 E k: z1 • (z2 + z3) = z1 • z2 + z1 • z3
Observación: Sean z y w dos complejos, definimos:
Resta: z – w = z + (– w)
División: z = z • w–1
w

9.1.6 Potencia de i
Las potencias de i son cíclicas y cada cuatro vuelven a repetir
su valor.
i =i
i2 = –1
i3 = i2 • i = –1 • i = – i
i4 = i2 • i2 = – 1 • – 1 = 1
i5 = i4 • i = i (i5 = i)

Números complejos 381

379-381 381 10/11/2001, 16:44


i6 = i5 • i = i • i = i2 =–1 (i6 = i2)
i7= i6 • i = – 1 • i = – i (i7 = i3)
i8= i7 • i = – i • i = – i2 = 1 (i8 = i4)

Ejercicios 1. Dados los complejos z1 = 2 + 3i, z2 = – 5 + i


z3 = 2i y z4 = – 5.
resueltos
Encontrar el valor de:
a) z1 + z2 – z3 b) z2 (z3 + z4)
1 1 z1 + z2
c) + d)
z2 z3 2(z3 – z4)
Solución:
a) z1 + z2 – z3 = (2 + 3i) + (– 5 + i) – (2i)
= 2 + 3i – 5 + i – 2i = – 3 + 2i
b) z2 (z3 + z4) = (– 5 + i) [(2i) + (– 5)] = (– 5 +i) (– 5 + 2i)
= 25 – 10i – 5i + 2i2 (i 2 = – 1)
= 25 – 10i – 5i – 2
= 23 – 15i
c) 1 + 1 = 1 + 1
z2 z3 –5 + i 2i
Para dividir por a + bi se amplifica la fracción por a – bi.

–5 – i –2i
= +
(–5 + i) (–5 – i) (2i) (–2i)
–5 – i –2i
= +
25 – i2 –4i2
–5 – i –2i –10 – 2i – 26i
= + +
26 4 52
10 28 7
= – – i=– 5 – i
52 52 26 23
z1 + z2
d) = (2 + 3i) + (–5 + i) = –3 + 4i =
2(z3 – z4) 2 ((2i) – (–5)) 10 + 4i
(–3 + 4i) (10 • 4i) –30 + 12i + 40i – 16i2
= =
(10 + 4i) (10 – 4i) 100 – 16i2
–14 + 52i 7 13 i
= = =
116 58 29
1 3
2. Dados los complejos z1 = 5 + i, z2 = – 3 – i,
2 4
z3 = – i y z4 = 1, encontrar:

a) Re (z1 – z3) b) Im
( z1
2
+ z3
)
382 Números complejos

382-383 ok 382 09/11/2001, 11:59


CAPÍTULO 9

c) Re ( zz )
4
1
d) 2 Im ( zz )
3
4
Solución:
(
a) Re (z1 – z3) = Re (5 + 1 i) – (– i) = Re 5 + 3 i = 5
2 2 ) ( )
(
b) Im 1 + z3 = Im
z2 ) 1
–3 – 3 i
4
[+ (–i)

3
]
[( ]
–3 + i
= Im 4 –i
3 3
–3 – i –3 + i
4 4 )( )
–3 + 3 i 3
= Im
[ 9–
4
9 2
16
i
–i = Im
–3
153
16] [
+ 4 i –i
153
16 ]
51(
= Im – 16 + 4 i–i
51 )
51(
= Im – 16 – 47 i
51
= – 47
51 )
5– 1 i
z
( )
c) Re z4 = Re
1
[ 5+ 1 i
2
1
] = Re
[( 2

5+ 1 i 5– 1 i
2 2 )( ) ]
5– 1 i 1
= Re 2

[
1 2
25 – i
2
= Re

20
5
101
4
– 2 i
101
4 ] [ ]
(
= Re 20 + 2 i =
101 101 101 )
z
( )
d) 2 Im z3 = 2 Im –i = 2 Im (– i) = 2 • (– 1) = – 2
4 1 ( )
3. Encontrar x e y para que se cumpla la siguiente igualdad:
2x – 3 + xi = x + y – 2yi – i – 1
Solución:
2x – 3 + xi = x + y – 1 – (2y + 1) i

2x – 3 = x + y – 1 x–y =2
x = – 2y – 1 x + 2y = – 1 x=1 e y=–1

4. Sabiendo que el neutro multiplicativo en k es z = 1 + 0i = 1,


demostrar que el inverso multiplicativo de a + bi es:
a b
– 2 i
a2 + b2 a + b2
Solución:
Sea z = (x + yi) el inverso pedido. Se debe cumplir que:

Números complejos 383

382-383 ok 383 09/11/2001, 11:59


z • (a + bi) = 1
(x + yi) (a + bi) = 1
ax – by + (ay + bx) i = 1 + 0 i

ax – by = 1
ay + bx = 0

resolviendo el sistema obtenemos:


a –b
x= 2 e y= 2 , luego
a + b2 a + b2
El inverso multiplicativo de (a + bi) es:
a b
(
a2 + b2
– 2
a + b2
i
)
5. Encontrar el valor de:
i5 – i7 i18 – i133
a = 11 9 b=
–i + i i24 + i111
Solución:
a) i5 = i 4 • i = 1 • i = i
i7 = i4 • i3 = 1 • i2 • i = – i
i11 = i8 • i3 = (i4)2 • i3 = 12 • (– i) = – i
i9 = i8 • i = i
i5 – i7 i – (–i) 2i
= = =1
–i11 + i9 i+ i 2i
b) i18 = (i4)4 • i2 = – 1
i135 = (i4)33 • i3 = – i
i24 = (i4)6 = 1
i111 = (i4)27 • i3 = – i
i18 – i135 –1 + i
= = (–1 + i) (1 + i)
i24 + i111 1– i (1 – i) (1 + i)
–1 – i + i + i 2 –2
= = = –1
1 – i2 2

6. Encontrar el valor de a) b) (1 + i)16


Solución:

2 3 + 2 i 2 3 + 2 i 3 2 + 4 3 i
a) = •
3 2 – 4 3 i 3 2 – 4 3 i 3 2 + 4 3 i

6 6 + 24 i + 6 i + 4 6 i2
=
18 – 48 i2

2 6 + 30i 6 5
= = + i
66 33 11

384 Números complejos

384-385 384 09/11/2001, 12:02


CAPÍTULO 9

b) (1 + i)16 = [(1 + i)2]8 = [1 + 2 i + i2]8 = (2i)8 = 256.


7. Sea 2x + (3y – 6) i + 3 un número complejo.
Determinar los valores de x e y para que la expresión dada sea:
a) un número imaginario puro
b) un número real
c) cero
d) igual a 2 – 5i

Solución:
2x + (3y – 6) i + 3 = (2x + 3) + (3y – 6) i
a) Para que sea imaginario puro, su parte real debe ser cero:
–3
2x + 3 = 0 fi x =
2
b) Para que sea un número real, su parte imaginaria debe
ser cero:
3y – 6 = 0 fi y = 2
c) Para que sea cero, la parte real y la parte imaginaria
deben ser ambas cero:
x=– 3 A y=2
2
d) Para que sea igual a 2 – 5i
1
2x + 3 = 2 fi x = –
2
1
3y – 6 = –5 fi y =
3

Ejercicios
1. Encuentre las raíces de los siguientes 4. Efectúe las siguientes operaciones:
números complejos: a) (2 + 3i) + (5 – 6i)
a) – 25 b) – 81 b) (3 – i) + (2 – 4i)

– 36 – 144 c) (5 + 4i) + (– 1 – i)
c) d)
d) (6 + i) – i
2. Efectúe las siguientes operaciones:
e) (8 – 4i) – (2 + i)
a) – 25 + – 4 –2 – 16 =
f) (3 – i) – (5 + 4i)
b) 3 – 49 – 2 – 25 + – 169 = g) – 2 – (6 – 2i)
3. Escriba los inversos aditivos de los h) (1 – i) – (1 + i)
siguientes números complejos: 5. Efectúe los siguientes productos:
a) 2 – 3i b) – 1 – i c) 6 – 2i d) – i a) (2 – 3i) (4 – i)

Números complejos 385

384-385 385 09/11/2001, 12:02


Ejercicios 10. Calcule el cuadrado de los siguientes
números complejos:
b) (5 + 2i) (– 1 – 6i)
a) 3 + 2i b) 5 – 3i c) 1 + i
c) (3 – 5i) (4 + i)
d) – 2 + i e) – 1 – i f) 2 – 2i
d) (– 3 – 2i) (– 1 + 6i) 11. Calcule:
e) (– 2 + i) (– 3 – i) a) (1 – i2)6 b) (i 22 + i30)4
f) (1 + 2i) (3 – i) c) (i5 + i – 12)2 d) (i – 3 – i – 5)– 2
g) (4 – 2i) (5 + i) 12. Verifique que los complejos 3 – i y
3 + i son solución de la ecuación
h) (3 + 2i) (7 – i) x2 – 6x + 10 = 0
6. Efectúe los siguientes productos: 13. Calcule:
a) (3 – 2i) (3 + 2i)
a)
b) (1 – 5i) (1 + 5i)
c) (– 6 + i) (– 6 – i) b)
d) (4 – 3 i) (4 + 3i)
14. Si z1 = ( 2, 3), z2 = ( 1, – 2), z3 = ( – 5,
e) (– 1 – i) (– 1 + i) 0) y z4 = ( 0, 4), encuentre:
f) (– 5 – 3i) (– 5 + 3i) a) z1 + z2 – z3
g) i • (– i) b) 2 z1 – 3 z2
h) 2i • (–2i) c) z4 (z1 + z2)
7. Calcule las siguientes divisiones: d) (z1 – z2) (z3 + z4)

a) (2 + 5i) : (3 – 2i) e)
b) (1 – 4i) : (6 – 2i)
f) (z1 + z4)
c) (3 – 2i) : (1 + i)
g)
d) (1 – i) : (2 – 4i)
e) (4 + 2i) : (5 – i) h) (z2 – z3 + z1 )
f) (2 + i) : (2 – i) 15. Si z1 = 3 – 5i, z2 = 6 + i, z3 = 4 – 9i
g) (1 – i) : (– i) y z4 = 5i, encuentre:
a) z1 – z2 + z4
h) (6 + 2i) : i
b) 2 – z1 + 5 z3
8. Calcule los inversos multiplicativos de
los siguientes números complejos: c) 4 z3 (z1 – z2)
d) 2 z1 z2 – 3 z3 z4
a) 1 – 2i b) – 1 + 2i c) 4 – i
e) (1 – z1) (1 + z2)
d) 3 + i e) – i f) 2i
f) 2 z1 ( z1 – z2 z3)
9. Calcule las siguientes potencias de i:
a) i – 1 b) i2 c) i 16 d) i 125 g)

e) i 1.003 f) i – 2 g) i – 3 h) i – 4 h)
i) i – 5 j) i – 6

386 Números complejos

386-387 386 20/11/02, 5:49 PM


CAPÍTULO 9

16. Efectúe las siguientes operaciones: cuánto gira cada uno.


a) (2 2 – 3i)(3 2 – i) 26. Pruebe que
(a + bi) i = – b + ai I a, b E R
b)
27. Calcule los siguientes productos:
17. Considerando que al complejo a) (3 – 2i) i2 b) (2 + i) i2
x + yi corresponde el par ordenado
(x, y) grafique los siguientes números c) (– 4 – 3i) i2 d) (– 1 + 3i) i2
complejos en el plano cartesiano: 28. Grafique el primer factor y el producto
en los ejercicios del problema anterior.
a) z1 = 2 + 3i b) z2 = – 5 – 2i Una los puntos con el origen de
c) z3 = – 8 + i d) z4 = 2 – 3i coordenadas y observe cuánto gira
cada uno.
e) z5 = – 5 + 2i f) z6 = – 8 – i
29. Encuentre a para que el producto
Compare z1 con z4, z2 con z5 y z3 (2a – 3i) (5 + i) sea un imaginario
con z6 puro.
18. Determine los números reales x e y 30. Determine un número complejo cuyo
que satisfagan la siguiente igualdad: cuadrado sea 8 – 6i.
31. Determine los números reales x e y que
a) 2x – 3i + y = xi – 2i + 2yi + 1
satisfagan la siguiente condición:
b) (2x – i) + (y – i) = (2 – 3i) – (x + 2yi) (2 + xi) : (1 – 2i) = y + i
c) (x + i) (y – 3i) = 1 – 7i 32. Calcule el valor de:
d) (2x – i) (–y + 2i) = – 10 + 11i z2 – 2z + 1 si z = 2 – 3i
19. Encuentre un número complejo cuyo 33. Calcule el valor de:
cuadrado sea – 3 – 4i z2 – 5z + 4 si z = 1 + i
34. Calcule el valor de:
20. Determine x para que el cociente 2 z2 – z – 3 si z = – 1 – 3i
35. Resuelva las siguientes ecuaciones:
sea imaginario puro.
a) (1 – z) (1 + i) = 2 – i

21. Encuentre x para que sea un b) z (1 – 2i) + 3 = 1 – 2z + i


número real.
c)
22. Determine x para que el producto
de (1 – 2i) (x – 5i) sea un número d)
real.
23. Determine x e y tales que: e) 3 + 4i
(x + yi)2 = – 16 – 30i
36. Si z = 4 – 3i, encuentre la parte
24. Calcule los productos siguientes:
real de
a) (2 – 3i) i b) (4 + 2i) i
c) (5 – 3i) i d) (– 2 + i) i 37. Calcule la raíz cuadrada de :
a) 3 + 4i b) 21 + 20i c) – 15 + 8i
e) (– 3 – 2i) i
d) 5 + 12i e) 8i f) 2i
25. Grafique el primer factor y el producto
del ejercicio anterior. Una el origen Sugerencia: plantee un sistema de
con el número complejo y observe ecuaciones.

Números complejos 387

386-387 387 20/11/02, 5:49 PM


Ejercicios 42. Calcule el valor de:
38. Determine z en la ecuación: 1 1 1 1
+ + + ..... +
2 3 100
z 1–z 5 i i i i
– =
3 + 4i 5i 3 – 4i
43. Demuestre que
39. Resuelva el sistema: z w
Re + Re =1
2wi + (1 – i) z= 3 z+w w + z
(1 – i) w + 4z = 2 + i
44. Demuestre que
40. Encuentre z E k tal que z + 1 = 0 Re (zw) = Re (z) Re (w) – Im (z) Im (w)
z
41. Calcule el valor de: 45. Demuestre que
1 1 1 1
+ + + ..... +
i2 i3 i50 Im (zw) = Re (z) Im (w) + Im (z) Re (w)
i

Soluciones

1. a) 5i b) 9i c) 6i d) 12i
2. a) –i b) 24i
3. a) – 2 + 3i b) 1+i c) – 6 + 2i d) i
4. a) 7 – 3i b) 5 – 5i c) 4 + 3i d) 6
e) 6 – 5i f) – 2 – 5i g) – 8 + 2i h) – 2i
5. a) 5 – 14i b) 7 – 32i c) 17 – 17i d) 15 – 16i
e) 7–i f) 5 + 5i g) 22 – 6i h) 23 + 11i
6. a) 13 b) 26 c) 37 d) 25 e) 2 f) 34 g) 1 h) 4
–4 19 7 11 1 5 3 1
7. a) 13
+
13
i b) 20 – 20 i c) 2 – 2 i d) 10 + 10 i
9 7 3 4
e) 13 + 13 i f) 5 + 5 i g) 1 + i h) 2 – 6i
1 2 1 2 4 1 3 1 1
8. a) 5 + 5 i b) – 5 – 5 i c) 17 + 17 i d) 10 – 10 i e) i f) – 2 i

9. a) –i b) – 1 c) 1 d) i e) – i f) – 1 g) i h) 1 i) – i j) – 1
10. a) 5 + 12i b) 16 – 30i c) 2i d) 3 – 4i e) 2i f) – 8i

1
11. a) 64 b) 16 c) 2i d) – 4
13. a) 0 b) 0
14. a) (8, 1) b) (1, 12) c) (– 4, 12) d) (– 25, – 21)

23 11 –2 7 3 41 1
e) , f) , – g) , h) , – 2
65 65 5 5 65 65 4

15. a) – 3 – i b) 19 – 40i c) – 264 + 12i d) – 89 – 114i

213 84 99 67
e) – 19 + 33i f) 270 + 570i g) 1.258 + 629 i h) 485 – 97 i

388 Números complejos

388-389 388 09/11/2001, 12:07


CAPITULO 9

16. a) 12 + 3 – 2 2 + 3 6 e)

10 3 6 -3 2
b) + i
11 11 –2
17. –3 – 2i
y –3 2 – 3i
3 z1
z5
z3
2 En todos los casos se observa que el
1
x
giro es de 90º.
–8 –5
–1
2 En general (a + bi) i = – b + ai
z6
–2
z2 27. a) – 3 + 2i b) – 2 – i c) 4 + 3i d) 1 – 3i
–3 z4

28.
5 1
18. a) x = 1, y = – 1 b) x = , y = –3 + 2i
6 2
2
c) x = 2, y = – 1 d) x = 2, y = 3
3 2+i
1
1 3 –3
x= , y=–6 x= , y=8 –2
–2
–1
2
3 4 3 – 2i –2 – i
19. – 1 + 2i; 1 – 2i
4 + 3i
1 –1 + 3i
20. x = 3
2
21. No existe, porque no cumple la condi-
–4
ción pedida. –1

–3
5 – 4 – 3i 1 – 3i
22. x = –
2
23. x = ± 3 y = ± 5 Se observa un giro de 180º.
24. a) 3 + 2i b) – 2 + 4i c) 3 + 5i
d) – 1 – 2i e) 2 – 3i –3
29. a = 10
25. a) b) 4

2
3+2i 30. 3 – i, –3 + i
4+2i
2
31. x = 1; y = 0
2 3
-2 4 32. – 8 – 6i
–3
2–3i
33. – 1 – 3i

34. – 18 + 15i

c) 3+5i d) 1 3 8 1
5
–2 + i 35. a) z = 2 + 2 i b) z = – 13 – 13 i
1

–2 –1 2
c) z = 3 + i d) z = – 1
–2
– 1 – 2i
9 13
3 5
e) – 5 + 5 i

–3 7
5–3i 36.
625

Números complejos 389

388-389 389 09/11/2001, 12:08


37) a) ± (2 + i) b) ± (5 + 2i) 1 9 7 1
39) w = 10 – 10 i; z =
10
+
2
i
c) ± (1 + 4i) d) ± (3 + 2i)
40) z = ± i
e) ± (2 + 2i) f) ± (1 + i)
38) z = – 1 + 2i 41) – 1 – i 42) 0

Conjugado y módulo
de un complejo 9.2

9.2.1 Conjugado de un complejo


Sea z = a + bi un número complejo.

Definición:
Se llama conjugado del complejo z al complejo
z = a – bi

• PROPIEDADES.
Sean z y w dos números complejos, entonces, se cumplen las
siguientes propiedades:
1. z = z El conjugado del conjugado de z es z.
El conjugado de una suma es igual
2. z + w = z + w a la suma de los conjugados de los
sumandos.

El conjugado de un producto es igual


3. z • w = z • w al producto de los conjugados de los
factores.

4. z z
, w π (0, 0) El conjugado de un cociente es igual al
=
w w cociente de los conjugados.
La suma de un complejo con su conju-
5. z + z = 2 Re (z) gado es igual a dos veces la parte real
del complejo.
La diferencia de un complejo con su
6. z – z = 2 Im (z) i conjugado es igual a dos veces la parte
imaginaria del complejo.
Un complejo es real si y sólo si es igual
7. z E R ¤ z = z
a su conjugado.
Para las demostraciones, ver ejercicios resueltos.

390 Números complejos

390-391 390 10/11/2001, 16:51


CAPÍTULO 9

9.2.2 Módulo de un complejo


Sea z = a + bi un número complejo.

Definición:
Se llama módulo o valor absoluto de z al número real
| z | definido por | z | = a2 + b 2

• PROPIEDADES.
Sean z y w dos números complejos, entonces, se cumplen las
siguientes propiedades:
El valor absoluto de la parte real de
1. | Re (z) | £ | z | un complejo es menor o igual al valor
absoluto del complejo.
El valor absoluto de la parte imaginaria
2. Im z ≤ z de un complejo es menor o igual al valor
absoluto del complejo.
Un complejo es cero si y sólo si su valor
3. z = 0 ⇔ z = 0
absoluto es cero.
El valor absoluto de un complejo es igual
4. z = – z = z al valor absoluto de su inverso aditivo y
de su conjugado.
El valor absoluto de un producto de
5. z • w = z • w complejos es igual al producto de los
valores absolutos de los factores.

z z El valor absoluto de un cociente de


6. = números complejos es igual al cociente de
w w
los valores absolutos de los números.
El valor absoluto de una suma de números
7. z + w ≤ z + w complejos es menor o igual a la suma
de los valores absolutos de los números
complejos.
La propiedad número 7 recibe el nombre de Desigualdad
Triangular.
Para las demostraciones, ver los ejercicios resueltos.

1. Dados los números complejos Ejercicios


z1 = 3 – 5i, z2 = – 6 + 3i, z3 = – 2i, z4 = 5. Encontrar: resueltos
a) el conjugado de cada uno b) el valor absoluto de cada uno

c) z1 + z 2 – 2 z 3 d) z1 ∑ z3

Números complejos 391

390-391 391 10/11/2001, 16:52


Solución:
a) Si z1 = 3 – 5i, entonces z1 = 3 + 5i
Si z2 = – 6 + 3i, entonces z2 = – 6 – 3i
Si z3 = – 2i, entonces z3 = 2i
Si z4 = 5, entonces z4 = 5
b) Si z1 = 3 – 5i, entonces z1 = 32 + – 5 2
= 34

Si z2 = – 6 + 3i, entonces z2 = – 6 2 + 32 = 45

2
Si z3 = – 2i, entonces z3 = –2 = 4 =2

Si z4 = 5, entonces z4 = 52 = 25 = 5
c) z1 + z2 – 2 z3 = 3 – 5i + – 6 + 3i – 2 – 2i

= – 3 + 2i = – 3 – 2i
d) z1 • z3 = 3 – 5i • – 2 i = – 10 – 6 i = 100 + 36 = 136

2. Demostrar que z = z. (Propiedad 1)


Solución: Sea z = a + bi E k
z = a + bi = a – bi = a + bi = z

3. Demostrar que z + w = z + w (Propiedad 2)


Solución: Sean z = a + bi, w = c + di E k

z + w = a + bi + c + di = a + c + b + d i

= (a + c) – (b + d)i = (a – bi) + (c – di) = z + w

4. Demostrar que z • w = z • w (Propiedad 3)


Solución: Sean z = a + bi, w = c + di E k

z • w = a + bi c + di = ac – bd + ad + bc i
= (ac – bd) – (ad + bc)i = ac – adi + bdi2 – bci
= a (c – di) – bi (c – di) = (a – bi) (c – di) = z • w

z
5. Demostrar que = z (propiedad 4)
w w
Solución: Sean z = a + bi, w = c + di E k

z a + bi bc – ad
w
= = ac + bd2 + 2
i
c + di c2 + d c2 + d
ac + bd bc – ad
= – i
2
c2 +d c2 + d2
ac – bdi2 – bci + adi
=
c2 + d2

392 Números complejos

392-393 392 10/11/2001, 16:57


CAPÍTULO 9

c a – bi + di a – bi
=
c + di c – di

a – bi c + di a – bi z
= = =
c + di c – di c – di w

6. Demostrar a) z + z = 2 Re (z) (Propiedades 5 y 6)


b) z – z = 2 Im (z)i
Solución: Sea z = a + bi
a) z + z = (a + bi) + (a – bi) = 2a = 2 Re (z)
b) z – z = (a + bi) – (a – bi) = 2bi = 2 Im (z) i

7. Demostrar z E R ¤ z = z (Propiedad 7)
Solución: Sea z = a + bi E k
fi) Hip: zER
Tesis: z=z
z E R fi a + bi E R fi b = 0 fi z = a fi z = a

\z=z
‹) Hip: z=z
Tesis: zER
z = z fi a + bi = a – bi fi a = a
y b = –b fi 2b = 0
b=0
\z=a luego z E R

8. Determinar un número complejo tal que su cuadrado sea


igual a su conjugado.
Solución: Sea x + yi el número complejo pedido.
(x + yi)2 = x – yi
x2 – y2 + 2xyi = x – yi

x2 – y2 = x
2xy = –y } cuya solución es x = – 1
2
y=± 3
2

luego los números buscados son

1 3 1 3
– + i y – – i Comprobarlo.
2 2 2 2

9. Probar que a) | Re (z) | £ |z|


b) | Im (z) | £ |z| (Propiedades 1 y 2)
Solución: Sea z = a + bi

Números complejos 393

392-393 393 10/11/2001, 16:58


a) |Re (z)| = |a| = a2 ≤ a2 + b2 = |z|
luego |Re (z)| £ |z|

b) |Im (z)| = |b| = b2 ≤ a2 + b2 = |z|


luego |Im (z)| £ |z|
10. Probar que z = 0 ¤ |z| = 0 (Propiedad 3)
Solución: Sea z = a + bi

fi) z = 0 fi a + bi = 0 fi a = 0 A b = 0

luego a2 + b2 = |z| = 0

‹) |z| = 0 fi a2 + b2 = 0 fi a2 + b2 = 0 fi a = 0 A b = 0
luego z = a + bi = 0
11. Probar que |z| = |– z| = | z | (Propiedad 4)
Solución: Sea z = a + bi

|z| =|a + bi| = a2 + b2

| – z| = |– a – bi| = –a 2+ –b 2
= a2 + b2

| z | =|a – bi| = a2 + (– b) 2 = a2 + b2
luego |z| = |– z| = | z |
12. Demostrar |z • w| = |z| • |w| (Propiedad 5)
Solución: Sean z = a + bi, w = c + di E k
|z • w|= |(a + bi) • (c + di)| = |(ac – bd) + (ad + bc) i|

= ac – bd 2 + ad + bc 2

= a2 c2 – 2 abcd + b2 d2 + a2 d2 + 2 abcd + b2 c2

= a 2 c 2 + d2 + b2 c 2 + d2 = a 2 + b2 c 2 + d2

= a 2 + b2 • c2 + d2 = |z| • |w|
z
13. Demostrar z = (Propiedad 6)
w w
Solución: Sean z = a + bi, w = c + di E k.
z a + bi ac + bd bc – ad
= = + i
w c + di c2 + d2 c2 + d2
2 2
= ac + bd bc – ad
+
c2 + d2 c2 + d2

= a 2 c 2 + 2 abcd + b2 d2 + b2 c 2 – 2 abcd + a 2 d2
2
c2 + d2

394 Números complejos

394-395 394 09/11/2001, 12:27


CAPÍTULO 9

a2 c2 + d2 + b2 c2 + d2 a2 + b2 c2 + d2
= =
2
c2 + d2 c2 + d2 2

a2 + b2 a2 + b2 z
= = =
c2 + d2 c2 + d2 w

14. Probar que z • z = |z|2


Solución: Sea z = a + bi
2
z • z = (a + bi) (a – bi) = a2 + b2 = a2 + b2 = |z|2
15. Probar que |z + w| £ |z| + |w| (Propiedad 7)
Solución: Sean z, w E k
|z + w|2 = (z + w) ( z + w ) (ver ejercicio 14)
= (z + w) ( z + w) (Prop. 2 de conjugado)
= zz + zw + wz + ww

= |z|2 + zw + zw + |w|2 wz = zw
= |z|2 + 2 Re zw + |w|2 (Prop. 5 de conjugado)
£ |z|2 + 2 |Re zw | + |w|2 (x £ |x| I x E R)
£ |z|2 + 2 |z w | + |w|2 (Prop. 1 de valor absoluto)
= |z|2 + 2 |z| | w | + |w|2 (Prop. 5 de valor absoluto)
= |z|2 + 2 |z| |w| + |w|2 (Prop. 4 de valor absoluto)
= (|z| + |w|)2
Luego:
|z + w|2 £ (|z| + |w|)2 extrayendo raíz cuadrada.
|z + w| £ |z| + |w|
16. Determinar el valor de a para que el valor absoluto del cociente
(3 – 2i) : (a + i) sea 3.
3 – 2i 13
Solución: = 3 = 3
a + i a2 + 1
3 – 2i a – i 13
= 3 = 9
a +i a – i a2 + 1

3a – 2 2a + 3 i
2
– = 3 9a2 + 9 = 13
a + 1 a2 + 1
2 2
3a – 2 – 2a + 3 4
+ = 3 a2 =
2 2 9
a2 + 1 a2 + 1

9a2 – 12a + 4 + 4a2 + 12a + 9


2
= 3 a=±2
a2 + 1 3

13 a2 + 1
= 3
2
2
a + 1

Números complejos 395

394-395 395 09/11/2001, 12:29


Ejercicios
1. Dados los siguientes números comple- 9. Calcule el conjugado y el valor
jos, encuentre su conjugado. absoluto de (i4 – i –11)–3.
a) 6 – 2i b) 4 – i c) 3 + 4i
10. Si z = a + bi, encuentre la parte real y
d) – 2i e) 5 f) 1 + 2 i 1 – z
2 la parte imaginaria de
2. Calcule el valor absoluto de los 1 + z
siguientes números complejos. 11. Determine los números complejos
a) 3 – 2i b) 9 – i c) 3 + 5i tales que su módulo sea 5 y la parte
real de su cuadrado sea 7.
d) 1 – i e) – i f) 3 – i
12. Encuentre z complejo tal que
3. Dados los números complejos z1 = 3 – 2i,
|z|2 = 37 y Im (z2) = 12.
z2 = 4 + i, z3 = – 2i y z4 = 1 – i,
encuentre: 13. Demuestre que I z, w E k, se
cumple que
a) |z3 • z2| f) z 1 – z 2 z + w 2
+ z – w 2
= |z|2 + |w|2
2
b) z1 – z3 g) z1 z2 + z3 z4 14. Demuestre que si dos complejos z
z1 – z2
y w tienen módulo 1, entonces
c) h) |z1 (z2 – z4 )| 1 – z w
z4 = 1
z – w
d) |(z2 – z3 ) z4| i) z1 + z2 z3 15. Demuestre que I z E k, z2 = z
2

z3 1 1 16. Encuentre z E k tal que:


e) j) –
z3 + z4 z1 z2 |1 – z| = 1 = |z|.
z
4. Calcule el valor absoluto de i8 17. Demuestre que i 4n + q = i q
i4 – i3 I n, q E N.
5. Determine el valor de a para que el 18. Determine un número z E k tal
valor absoluto de (a – 2i) sea 3. que su cuadrado sea el triple de
6. Encuentre x para que el valor absoluto su conjugado.
de (1 – xi) (1 + i) sea 10. 19. Encuentre los complejos que satis-
7. Encuentre el valor de b para que el fagan que la mitad de su cuadrado es
valor absoluto del cociente (b – 2i) : igual a un tercio de su conjugado.
(3 – i) sea 2. 1
20. Pruebe que si w + w es real,
8. Determine x para que el conjugado de
(x – i) (1 – 3i) sea igual a (–1 + 7i). entonces Im (w) = 0 V |w| = 1.

Soluciones
1
1. a) 6 + 2i b) 4 + i c) 3 – 4i d) 2i e) 5 f) + 2 i
2
2. a) 13 b) 82 c) 34 d) 2 e) 1 f) 10
2
3. a) 2 17 b) 3 c) 1 + 2i d) 5 2 e) f) 2
5
1 1
g) 153 h) 117 i) 5 + 10i j) –
13 17
1 1 1 2
4. 5. a = ± 5 6. x = ± 7 7. x = ± 6 8. x = 2 9. – – i;
2 4 4 4

396 Números complejos

396-397(2003) 396 20/11/02, 5:53 PM


CAPÍTULO 9

1 – a2 – b2 1 3
10. ;
2b 16. z1 = + i ; z2 = 1 – 3
i
2 2 2 2 2 2 2 2
1 + a + b 1 + a + b

11. (4 + 3i), (– 4 – 3i), (4 – 3i), (– 4 + 3i) 17. Recuerde que i4a + q = (i4)u • iq y i4 = 1
3 3 3
12. (6 + i), ( – 6 – i), (– 1 – 6i), (1 + 6i) 18. z1 = – + i; z2 = – 3 – 3 3 i
2 2 2 2
1 3
13. Use la propiedad |z|2 = z • z 19. z1 = – 1 + 3
i ; z2 = – – i
3 3 3 3
14. Eleve al cuadrado la igualdad y considere |z|2 = z • z

Representación trigonométrica o
forma polar de un número complejo 9.3

9.3.1 Definición de razones trigonométricas


Sea P (x, y) un punto del plano cartesiano Y
tal que OP forma un ángulo a con el

}
eje x. P (x, y)
Sea OP = r.
Recordemos que:
} }
0
a
r
y

X
y x
Seno a = = sen a
r
x
Coseno a = = cos a
r
y
Tangente a = = tg a
x

9.3.2 Representacón trigonométrica del complejo z = a + bi


El complejo z = a + bi geométricamente representa un punto
en el plano cartesiano.

z = a2 + b2 es la distancia del origen del plano al punto


representado por z. A esta distancia la llamamos r.

z = a + bi
r= z = a 2 + b2 (1) b

→ |z|
Sea a el ángulo definido por el vector oz y
el eje x. a
0 a
El ángulo a se llama argumento del complejo z.

a está definido para todo z π 0

Números complejos 397

396-397(2003) 397 20/11/02, 5:54 PM


En la figura se ve que
a = |z| cos a y b = |z| sen a (2)
Sea z = a + bi, aplicando las relaciones (1) y (2) se tiene que
z = |z| cos a + |z| sen a i
z = r (cos a + i sen a)

Para escribir un complejo z = a + bi en su forma trigonométrica


calculamos
1° r = a 2 + b2
2° Ubicamos a tal que cos a = ar y sen a = b
r
o bien ubicamos a tal que tg a = b , pero se debe cuidar de
a
determinar el cuadrante donde se encuentra el complejo z.
Ejemplo: Escribir z = 2 + 2i en forma trigonométrica.
r= 22 + 22 = 2 2

tg a = 2 = 1 fi a = 45°
2
luego z = 2 2 (cos 45° + i sen 45°)

9.3.3 Producto y cociente de complejos en forma polar


PRODUCTO.
Si z1 = r1 (cos a1 + i sen a1) y cos (a + b) = cos a cos b - sen a sen b
z2 = r2 (cos a2 + i sen a2) y sen (a + b) = sen a cos b - cos a sen b
entonces: z1 • z2 = r1 • r2 [cos (a1 + a2) + i sen (a1 + a2)]

COCIENTE.
Si z1 = r1 (cos a1 + i sen a1)
z2 = r2 (cos a2 + i sen a2) , z2 π 0
z1 r1
entonces: = [cos (a1 – a2) + i sen (a1 – a2)]
z2 r2

9.3.4 Potenciación de números complejos en forma polar


Si z = r (cos a + i sen a) y n E N,

entonces: zn = rn (cos n a + i sen n a)

Nota: Si r = 1 entonces z = cos a + i sen a

(cos a + i sen a)n = cos na + i sen na

(Demostración pág. 404 - ejercicios propuestos).

Esta fórmula se conoce con el nombre de fórmula de De Moivre


y también es válida si n es negativo.

398 Números complejos

398-399 398 10/11/2001, 17:01


CAPÍTULO 9

9.3.5 Radicación de números complejos en forma polar


Si z = r (cos a + i sen a) y n E N, entonces:
n n α + 2kπ α + 2kπ
z = r cos + i sen , k = 0, 1... n – 1
n n

NOTA: Se pueden obtener todas las raíces n-ésimas de un complejo z si se


multiplica una de estas raíces por todas las raíces n-ésimas de la unidad.

1. Encontrar la forma polar de los números complejos siguientes: Ejercicios


a) 1 – 3 i b) – 1 + i c) 2i d) 4 resueltos
Solución:

a) z = 1 – 3 i
2
r= 1+ 3 =2 1
– 3 – 3 z=1– 3
tg a = fi a = 300o
1
luego: z = 1 – 3 i = 2 (cos 300o + i sen 300º)

b) z = – 1 + i z = –1 + i 1
2 2
r= –1 +1 = 2
–1
1
tg a = = – 1 fi a = 135o
–1
luego: z = – 1 + i = 2 (cos 135o + i sen 135o)

c) z = 2i
2 z = 2i
2 2
r= 0 + 2 =2
2
tg a = no está definida,
0
significa que a vale 90o

luego: z = 2 (cos 90o + i sen 90o)

d) z = 4

r= 42 + 02 = 4 z=4
4
tg a = 0 = 0 fi a = 0o
4
luego: z = 4 (cos 0o + i sen 0o)

Números complejos 399

398-399 399 10/11/2001, 17:02


2. Escribir los siguientes complejos en su forma rectangular (a + bi).
a) 3 • (cos 30o + i sen 30o) b) 2 (cos 135o + i sen 135o)
o
c) 2 (cos 90 + i sen 90 ) o d) 3 (cos 180o + i sen 180o)
Solución:
Para transformar los números complejos de su forma polar en
su forma rectangular basta calcular los valores del seno y del
coseno del ángulo señalado.
Para resolver estos problemas es conveniente tener a mano la
siguiente tabla:
a 30º 45º 60º
1 1 3
sen a
2 2 2
3 1 1
cos a
2 2 2
1
tg a 1 3
3

a) z = 3 (cos 30o + i sen 30o) = 3


( 2
3
+ 1 i =
2 )
3 3
+ 3 i
2 2

b) z = 2 (cos 135o + i sen 135o) = 2 – 1 + 1 i = – 1 + i


( 2 2 )
cos 135o = cos (90o + 45o) = – cos 45o = – 1
2
2
sen 135o = sen (90o + 45o) = sen 45o = 1
2

c) z = 2 (cos 90o + i sen 90o) = 2 (0 + i) = 2 i


d) z = 3 (cos 180o + i sen 180o) = 3 (– 1 + 0 i) = – 3

3. Efectuar el producto de los siguientes números complejos:


a) z1 = 2 (cos 30o + i sen 30o) b) z1 = 2 (cos 60o + i sen 60o)
z2 = 3 (cos 15o + i sen 15o) z2 = 3 (cos 240o + i sen 240o)

Solución:
a) z1 • z2 = [2 (cos 30o + i sen 30o)] [3 (cos 15o + i sen 15o)]
= 6 (cos 45o + i sen 45o)
b) z1 • z2 = [ 2 (cos 60o + i sen 60o)] [ 3 (cos 240o + i sen 240o)]
= 6 (cos 300o + i sen 300o)

4. Efectuar el cociente de los siguientes números complejos:

a) z1 = 4 (cos 25o + i sen 25o) b) z1 = 2 (cos 120o + i sen 120o)

z2 = 2 (cos 80o + i sen 80o) z2 = 2 (cos 90o + i sen 90o)

400 Números complejos

400-401 400 10/11/2001, 17:08


CAPÍTULO 9

Solución:
z1 4 cos 25º + i sen 25º
a) = = 2 (cos (– 55o) + i sen (– 55o)
z2 2 cos 80º + i sen 80º
= 2 (cos 305o + i sen 305o)
z1 2 cos 120º + i sen 120º
b) = = cos 30o + i sen 30o
z2 2 cos 90º + i sen 90º
5) Encontrar la potencia indicada de cada número complejo:
a) (5 + 2i)6 b) (3 – 2i)4 c) (– 8 – i)5
Solución:
Es conveniente transformar primero los números complejos
a su forma polar.
a) z = 5 + 2i es un punto del primer cuadrante y por lo tanto el
argumento α será un ángulo menor que 90º.
Si tg a = 2 , entonces a = arctg 2 = 21,8o
5 5
r= 25 + 4 = 29

luego z = 5 + 2i = 29 (cos 21,8o + i sen 21,8o)

y z6 = ( 29 )6 (cos (6 • 21,8o) + i sen (6 • 21,8o))


= 24.389 (cos 130,8º + i sen 130,8o)
Pasando este complejo a su forma cartesiana
z6 = 24.389 (– 0.6534 + 0,7570 i) = – 15.935,8 + 18.462,5 i
\ (5 + 2i)6 ª – 15.935,8 + 18.462,5 i
b) z = 3 – 2i es un punto del cuarto cuadrante, luego 270o < a < 360º
–2
Si tg a = , entonces a = 326,3o
3
r = 9+4 = 13

z = 3 – 2i = 13 (cos 326,3o + i sen 326,3o)

z4 = (3 – 2i)4 = ( 13 )4 (cos (4 • 326,3o) + i sen (4 • 326,3o))

= 169 (cos 225,2o + i sen 225,2o)


que en la forma cartesiana es:
z4 = 169 (– 0,7046 – 0,7095 i) = – 119,1 – 119,9 i
\ (3 – 2i)4 = – 119,1 – 119,9 i ª – 119 – 120 i
c) z = 8 – i es un punto del tercer cuadrante y por lo tanto
180º < a < 270º
Si tg a = 1 , entonces a = arctg 1 = 187,1º
8 8

r= 64 + 1 = 65

Números complejos 401

400-401 401 10/11/2001, 17:09


z = – 8 – i = 65 (cos 187,1o + i sen 187,1o)

z5 = ( 65 )5 (cos (5 • 187,1o) + i sen (5 • 187,1o))


= 34.063 (cos 215,5o + i sen 215,5o)
que en la forma cartesiana es:
z5 = 34.063 (– 0,814 – 0,581 i) = – 27.727,3 – 19.790,6 i
\ (– 8 – i)5 = – 27.727 – 19.790 i
6. Calcular el valor de (1 + i)100
Solución: z = 1 + i = 2 (cos 45o + i sen 45o)

z100 = (1 + i)100 = ( 2 )100 (cos 4.500o + i sen 4.500o)


(1 + i)100 = 1,1•1015 (cos 180o + i sen 180o)
= 1,1•1015 (– 1 + 0i) = – 1,1•1015
\ (1 + i)100= – 1,1•1015
7. Encontrar todas las raíces indicadas de:
5 3
a) 1 b) 1 – i c) 4
3 + i y representarlas gráficamente.
Solución: a) z = 1 = (cos 0º + i sen 0o)
5 2 k π + i sen 2 k π
1 = cos
5 5
Si k = 0 : z1 = cos 0o + i sen 0o = 1

Si k = 1 : z2 = cos 2 π + i sen 2 π = 0,31 + 0,95 i


5 5
4π 4π
Si k = 2 : z3 = cos + i sen = – 0,81 + 0,59 i
5 5
6π 6π
Si k = 3 : z4 = cos + i sen = – 0,81 – 0,59 i
5 5

Si k = 4 : z5 = cos 8 π + i sen 8π
= 0,31 – 0,95 i
5 5

Gráficamente: 1 z2
0,9
0,8
0,7
z3 0,6
0,5
5 0,4
8p/
/5 0,3
6p
0,2 4
p/5

0,1
2p/

1 0,9 0,8 0,7 0,6 0,5 0,4 0,3 0,2 0,1


5

z1
– 0,1 0,1 0,2 0,3 0,4 0,5 0,6 0,7 0,8 0,9 1
– 0,2
– 0,3
– 0,4
– 0,5
z4 – 0,6
– 0,7
– 0,8
– 0,9
–1 z5

402 Números complejos

402-403 402 09/11/2001, 13:19


CAPÍTULO 9

b) z = 1 – i = 2 (cos 315o + i sen 315o)


3 3 315º + k 360º 315º + k 360º
1 – i = 2 cos + i sen
3 3

Si k = 0 : 1,1 (cos 105o + i sen 105o) = – 0,29 + 1,06 i


Si k = 1 : 1,1 (cos 225o + i sen 225o) = – 0,78 – 0,78 i
Si k = 2 : 1,1 (cos 345o + i sen 345o) = 1,06 – 0,29 i
Gráficamente: 1,0
0,9
0,8
0,7
0,6
0,5
0,4
º 0,3
225 1
0,2 05
º

0,1
-1,0 -0,9 -0,8 -0,7 -0,6 -0,5 -0,4 -0,3 -0,2 -0,1
0,1 0,2 0,3 0,4 0,5 0,6 0,7 0,8 0,9 1,0
-0,1
345º

-0,2
-0,3
-0,4
-0,5
-0,6
-0,7
-0,8
-0,9
-1,0

c) z = 3 + i = 2 (cos 30o + i sen 30o)

4 4 30º + k 360º 30º + k 360º


3 + i = 2 cos + i sen
4 4

Si k = 0 : 1,19 (cos 7,5o + i sen 7,5o) = 1,18 + 0,16 i


Si k = 1 : 1,19 (cos 97,5o + i sen 97,5o) = – 0,16 + 1,18 i
Si k = 2 : 1,19 (cos 187,5o + i sen 187,5o) = – 1,18 – 0,16 i
Si k = 3 : 1,19 (cos 277,5o + i sen 277,5o) = 0,16 – 1,18 i

Gráficamente: 1,2
1,1
1,0
0,9
0,8
0,7
0,6
0,5
5º 0,4
,
187

0,3 9
0,2
7,5º

-1,2 -1,1 -1,0 -0,9 -0,8 -0,7 -0,6 -0,5 -0,4 -0,3 -0,2 -0,1 0,1

-0,1 0,1 0,2 0,3 0,4 0,5 0,6 0,7 0,8 0,9 1,0 1,1 1,2
-0,2
-0,3
277 -0,4
º -0,5
-0,6
-0,7
-0,8
-0,9
-1,0
-1,1
-1,2

Números complejos 403

402-403 403 09/11/2001, 13:19


8. Usando la fórmula de De Moivre, hallar una expresión para
sen 5 a y para cos 5 a en función de sen a y cos a
Solución:
Se tiene, según la fórmula de De Moivre que (cos a + i sen
a)5 = cos 5 a + i sen 5 a
Desarrollando el primer miembro de esta ecuación de acuerdo al
teorema del binomio e igualando tenemos:
(cos a + i sen a)5 = cos 5 a + i sen 5 a
5 5 5
cos5a + cos4a i sen a + cos3a (i sen a)2 +
0 1 2

5 5 5
cos2a (i sen a)3 + cos a (i sen a)4 + (i sen a)5
3 4 5
= cos 5a + i sen 5a
cos5a + 5 i cos4a sena – 10 cos3a sen2a – 10i cos2a sen3a +
5 cosa sen4a + i sen5a = cos 5a + i sen 5a
cos5a – 10 cos3a sen2a + 5 cosa sen4a + (5 cos4a sena – 10
cos2a sen3a + sen5a) i = cos 5a + i sen 5a
De donde:
cos 5a = cos5a – 10 cos3a sen2a + 5 cosa sen4a
sen 5a = sen5a – 10 sen3a cos2a + 5 sena cos4a
9. Demostrar que si z1 = r1 (cos a1 + i sen a1 ) y
z2 = r2 (cos a2 + i sen a2 ), entonces
z1 z2 = r1 r2 [cos (a1 + a2 ) + i sen (a1 + a2 )]
Solución:
z1 • z2 = [r1 (cos a1 + i sen a1 )] [r2 (cos a2 + i sen a2 )]
= r1 r2 [cos a1 cos a2 – sen a1 sen a2 + i (sen a1 cos a2
+ cos a1 sen a2 )]
= r1 r2 [cos (a1 + a2 ) + i sen (a1 + a2 )]
10. Demostrar que si z1 = r1 (cos a1 + i sen a1 ) y
z2 = r2 (cos a2 + i sen a2 ), entonces
z1 r1
=
z2 r2
[cos (a1 – a2 ) + i sen (a1 – a2 )]
Solución:
z1 r1 cos α1 + i sen α1 cos α2 – i sen α2
= •
z2 r2 cos α2 + i sen α2 cos α2 – i sen α2

r1 cos α1 cos α2 + sen α1 sen α2 + i sen α1 cos α2 – cos α1 sen α2


=
r2 cos2 α2 + sen2 α2
r1
= [cos (a1 – a2) + i sen (a1 – a2)]
r2
11. Demostrar que si z = r (cos a + i sen a), entonces
zn = rn (cos n a + i sen n a) I n E N

404 Números complejos

404-405 404 20/11/02, 5:57 PM


CAPÍTULO 9

Solución:
La demostración de esta fórmula la haremos por inducción. Es
decir, la verificamos para n = 1, la suponemos verdadera para n = k
y con esta hipótesis la demostramos para n = k + 1.
Si n = 1 z = r (cos a + i sen a), verdadero.
Hip: si n = k
zk = rk (cos ka + i sen ka)
Tesis: si n = k + 1
zk + 1 = rk + 1 [cos (k + 1) a + i sen (k + 1) a]
Demostración:
Por hipótesis:
zk = rk (cos k a + i sen k a) /• z
z k + 1 k
= r (cos k a + i sen k a) • r (cos a + i sen a)
zk + 1 = rk • r [cos (k a + a) + i sen (k a + a)]
zk + 1 = rk + 1 [cos (k + 1) a + i sen (k + 1) a]
\ Se ha demostrado que:
zn = rn [cos (n a) + i sen (n a)]
12. Demostrar que si z = r (cos a + i sen a), entonces:

n n α + 2kπ α + 2k π
z = r cos + i sen
n n
k = 0, 1, ... n – 1
Solución:
n
Supongamos que r cos α + i sen α = ρ (cos b + i sen b)
y encontremos ρ y b
n
r cos α + i sen α = ρ (cos b + i sen b) ( )n
r (cos a + i sen a) = ρn [cos (n b) + i sen (n b)]

r cos a = ρ cos (n b)
n

r sen a = ρn sen (n b) ( )2

r2 cos2 a = ρ2n cos2 (n b)


r2 sen2 a = ρ2n sen2 (n b) sumando
n
r2 = ρ2n fi ρ = r
reemplazando ρ en la segunda ecuación del sistema:
n n
r sen a = r sen (n b) /:r
sen a = sen (n b)

n b = a + 2k p
b = α + 2kπ k = 0, 1, 2...
n

Números complejos 405

404-405 405 20/11/02, 5:58 PM


Luego,
n n α + 2kp α + 2kp
r (cos α + i sen α) = r cos + i sen
n n

con k = 0, 1, 2…, n – 1 ya que para valores de n enteros


menores que 0 o mayores que n – 1 los valores empiezan
a repetirse.

13. Resolver la ecuación x6 – 2x3 = – 2


Solución:
Haciendo u = x3 nos queda u2 – 2u + 2 = 0 cuyas
soluciones son: u1 = 1 + i y u2 = 1 – i
Como u = x3 debemos resolver
3
a) x3 = 1 + i fi x = 1 + i
3
b) x3 = 1 – i fi x = 1 – i
1 1
a) 1 + i 3 = 2 (cos 45º + i sen 45º) 3

1
3
45º + k • 360º 45º + k • 360º
= 2 cos + i sen
3 3
Si k = 0 : 1,12 (cos 15º + i sen 15º) = 1,08 + 0,29i
Si k = 1 : 1,12 (cos 135º + i sen 135º) = – 0,79 + 0,79i
Si k = 2 : 1,12 (cos 255º + i sen 255º) = – 0,29 – 1,08i
1 1
3
b) 1 – i 3 = 2 (cos 315º + i sen 315º)
1
3 315º + k • 360º 315º + k • 360º
= 2 cos + i sen
3 3
Si k = 0 : 1,12 (cos 105º + i sen 105º) = – 0,29 + 1,08i
Si k = 1 : 1,12 (cos 225º + i sen 225º) = 0,79 – 0,79i
Si k = 2 : 1,12 (cos 345º + i sen 345º) = 1,08 – 0,29i
Así vemos que las 6 soluciones de la ecuación x6 – 2x3 + 2 = 0 son
x1 = 1,08 + 0,29i x2 = – 0,79 + 0,79i x3 = – 0,29 – 1,08i
x4 = – 0,29 + 1,08i x5 = 0,79 – 0,79i x6 = 1,08 – 0,29i

Ejercicios
1. Escriba los siguientes complejos en su forma polar.
a) 1 – i b) 3 + i c) – 3 + 3 i d) 2 + i
e) – 2 – 2i f) – 7 + 3i g) 6 – 7i h) 1 + 9i
i) 10 + i j) – 6i k) – 5 l) 12i
2. Escriba los siguientes complejos en su forma rectangular (a + bi)
a) 4 (cos 45° + i sen 45°) b) 3 (cos 135° + i sen 135°)

406 Números complejos

406-407 406 09/11/2001, 13:25


CAPÍTULO 9

c) 3 (cos 30° + i sen 30°) b) z1 = 2 (cos 350° + i sen 350°)


d) cos 60° + i sen 60° z2 = 4 (cos 140° + i sen 140°)
e) 4 (cos 270° + i sen 270°) c) z1 = 8 (cos 10° + i sen 10°)
f) 2 (cos 210° + i sen 210°) z2 = 4 (cos 340° + i sen 340°)
g) 5 (cos 240° + i sen 240°) d) z1 = 6 (cos 345° + i sen 345°)
h) 3 (cos 330° + i sen 330°) z2 = 2 (cos 300° + i sen 300°)
z1
i) 12 (cos 180° + i sen 180°) 6. Efectúe los siguientes cocientes
z2
j) 1 (cos 315° + i sen 315°) a) z1 = 3 (cos 25° + i sen 25°)
2
k) 2 (cos 225° + i sen 225°) z2 = 2 (cos 30° + i sen 30°)
l) 5 (cos 270° + i sen 270°) b) z1 = 3 (cos 12° + i sen 12°)
z2 = 2 (cos 10° + i sen 10°)
3. Efectúe los siguientes productos z1 z2
c) z1 = 12 (cos 130° + i sen 130°)
a) z1 = 2 (cos 240° + i sen 240°)
z2 = 4 (cos 52° + i sen 52°)
z2 = 1 (cos 60° + i sen 60°)
2
d) z1 = 5 (cos 250° + i sen 250°)
b) z1 = 2 (cos 30° + i sen 30°)
z2 = 4 (cos 20° + i sen 20°)
z2 = 6 (cos 150° + i sen 150°)
e) z1 = 2 (cos 15° + i sen 15°)
c) z1 = 3 (cos 25° + i sen 25°) z2 = 1 (cos 350° + i sen 350°)
z2 = 2 (cos 65° + i sen 65°) 2

d) z1 = 2 (cos 130° + i sen 130°) 7. Calcule las siguientes potencias:


a) (3 + 2i)4 e) (–2 – i)5
z2 = 5 (cos 95° + i sen 95°)
b) (5 – i)5 f) (–3 + i)4
4. Efectúe los siguientes productos z1 z2 c) (–5 + 2i) g) (1 + 2i)6
6

a) z1 = 3 (cos 22° + i sen 22°) d) (1 – i)4 h) (5 – 2i)3

z2 = cos 35° + i sen 35° 8. Calcule el valor de (1 – i)50


b) z1 = 2 (cos 30° + i sen 30°) 9. Encuentre todas las raíces indicadas y
represéntelas gráficamente
z2 = 12 (cos 20° + i sen 20°)
5 4 3
c) z1 = 2 (cos 120° + i sen 120°) a) 2 b) 1+ i c) 8–i
z2 = 4 (cos 100° + i sen 100°) 10. Resuelva la ecuación x4 + 1 = 0
d) z1 = 3 (cos 220° + i sen 220°)
11. Resuelva la ecuación x3 + 2 + 2i = 0
z2 = 3 (cos 130° + i sen 130°)
12. Pruebe que:
e) z1 = 2 (cos 190° + i sen 190°)
a) sen 3x = 3 sen x – 4 sen3x
z2 = 2 (cos 330° + i sen 330°)
b) cos 3x = –3 cos x + 4 cos3x
z1 13. Resuelva la ecuación
5. Efectúe los siguientes cocientes
z2
a) z1 = 3 (cos 210° + i sen 210°) x6 + 4x3 + 5 = 0

z2 = 3 (cos 60° + i sen 60°) 14. Encuentre dos números que sumados
den 4 y multiplicados den 8.

Números complejos 407

406-407 407 09/11/2001, 13:26


Soluciones

1. a) 2 (cos 315º + i sen 315º) c) 3 +i d) 3 + 3 i


2 2
6. a) 1,5 – 0,13 i b) 1,22 + 0,04 i
b) 2 (cos 30º + i sen 30º)
c) 0,62 + 2,93 i d) – 0,80 – 0,96 i
c) 2 3 (cos 150º + i sen 150º)
e) 1,81 + 0,85 i
d) 5 (cos 26,5º + i sen 26,5º) 7. a) – 119 + 120 i b) 1.900 – 2.876 i
c) – 15.939 – 18.460 i d) – 4
e) 2 2 (cos 225º + i sen 225º)
e) – 38 – 41 i f) 28 – 96 i
f) 58 (cos 156,8º + i sen 156,8º)
g) 117 + 44 i h) 65 – 142 i
g) 85 (cos 310,6º + i sen 310,6º) 8. – 225 i

h) 82 (cos 83,7º + i sen 83,7º) 9. a) 1,15; (0,36 + 1,09 i); (– 0,93 + 0,68 i);
(– 0,93 – 0,68 i); (0,36 – 1,09 i)
i) 101 (cos 5,7º+ i sen 5,7º)
b) (3,92 + 0,78 i); (– 0,78 + 3,92 i);
j) 6 (cos 270º + i sen 270º)
(– 3,92 – 0,78 i); (0,78 – 3,92 i)
k) 5 (cos 180º + i sen 180º) c) (– 1,86 + 3,56 i); (– 2,15 – 3,39 i);

l) 12 (cos 90º + i sen 90º) (4,02 – 0,08 i)

1 1 1 1
2. a) 2 2 + 2 2 i b) – 3 + 3 i 10. + i ; – + i
2 2 2 2 2 2
3 3 1 3
c) + i d) + i
1 1 1 1
2 2 2 2
– – i ; – i
2 2 2 2
e) – 4 i f) – 3 – i
11. (0,37 + 1,37 i); (– 1,37 – 0,37 i); (1 – i)
5 5 3
g) – – i h) 3 3

3
i
2 2 2 2 12. Usar fórmula de De Moivre para
i) – 12 j) + 1

1
i
(cos α + i sen α)3
2 2 2 2
13. x1 = 1,29 + 0,20 i
k) – 1 – i l) – 5 i
x2 =– 0,82 + 1,02 i
3. a) 0.5 – 0.87 i b) – 2 3
x3 =– 0,47 – 1,22 i
c) 6i d) – 5 2 – 5 2 i
x4 =– 0,47 + 1,22 i
4. a) 1,63 + 2,52 i b) 10,9 + 13 i
x5 =– 0,82 – 1,02 i
c) – 6,13 – 5,14 i d) 5,12 – 0,9 i
x6 = 1,29 – 0,20 i
e) – 3,76 + 1,37 i
14. 2 + 2 i; 2 – 2i
1 3 3 1
5. a) – + i b) – – i
2 6 4 4

408 Números complejos

408-409 408 10/11/2001, 17:13


CAPÍTULO 9

Prueba de selección múltiple


1. El valor de 5. Si z1 = 3 – 2 i y 9. El valor de i– 13 es:
– 25 + 2 – 4 – – 36 z2 = 4 + i entonces
A. 0
es: z 1 • z2 =
B. 1
A. 3i A. 14 + 5 i
C. – 1
B. 4i B. 14 – 5 i
D. i
C. 5i C. – 14 – 5 i
D. – 14 + 5 i E. –i
D. 6 i
E. 5 – 14 i
E. – 6 i
6. Si z1 = 4 – 2 i y 10. El valor de
2. El inverso aditivo de z2 = – 3 + 6 i entonces (i11 + i– 5)6 es:
– 2 – 5 i es: z 1 : z2 = A. 64
8 2
A. – 2 + 5 i A. – + i B. – 64
15 5
B. 2–5i 8 2 C. 32
B. – i
15 5
C. 2+5i 8 2 D. – 32
C. + i
D. – 5 – 2 i 15 5 E. 16
8 2
E. 5+2i D. – – i
15 5
2 8
E. + i
5 15 11. El valor de
3. Si z1 = 4 – 2 i y
z2 = – 3 + 5 i entonces 7. El inverso multiplicativo (– i17 + i126)2 es:
z1 + z2 = de 1 + 2 i es:
A. 1
A. 1–3i 1 2 B. – 1
A. – 5 – 5 i
B. 7+3i C. i
1 2
B. – i
C. 1+3i 5 5 D. – i
D. – 1 + 3 i –
1
+
2
i
C. E. 2i
5 5
E. – 7 – 3 i
D. 1–2i
4. Si z1 = 2 – 5 i y 12. Si z = – 1 + 3 i
z2 = – 5 i entonces E. –1 – 2 i
entonces z2 es:
z1 – z2 =
8. El valor de i112 es: A. 8–6i
A. 2 + 10 i
A. 0 B. – 8 + 6 i
B. 2 – 10 i B. 1 C. – 8 – 6 i
C. – 2 + 10 i C. – 1
D. 6 + 8 i
D. – 2 – 10 i D. i
E. – 6 + 8 i
E. 2 E. –i

Números complejos 409

408-409 409 10/11/2001, 17:13


Prueba de selección múltiple
13. Si z = –3 + 5i, entonces 17. Son soluciones de la 21. En la igualdad
1 + z + z2 = ecuación 2x – 1 + i = 3 + i, x vale
A. 18 – 25i x2 – 2x + 5 = 0 A. 0
B. – 18 – 25i I) (1 + 2i) II) (1 – 2i) B. 1
C. 18 + 25i III) 2
C. 2
D. 20 + 25i
A. I y II D. – 1
E. – 20 + 25i
B. I y III E. i
14. El valor de C. II y III
1 1 1 1 1 D. sólo III 22. En la igualdad
+ 2+ 3+ 4+ 5 es: (x – 2yi) (1 – i) = 7 + i
i i i i i E. ninguna los valores de x e y
A. 0
respectivamente son:
B. 1 18. La diferencia entre los
complejos z1 y z2 es: A. 2 ; 3
C. – 1
3 + 6i, si z2 = 2 z1
B. 3 ; 2
D. i entonces z2 vale
E. – i C. 2 ; –3
A. – 3 – 6i
D. 3 ; –2
15. Si z1 = 2 – i , B. – 6 – 12i
E. – 2 ; – 3
z2 = – 2i y z3 = 4 + 2i, C. 3 – 6i
D. 6 – 12i 23. El número complejo
entonces 1 (z2 + z3) =
z1 E. 6 + 12i cuyo cuadrado es
8 3 – 4i es:
A. + 8 i
5 5 19. Si z = 1 – i y A • z2 = 1,
A. 2 – i
entonces A vale
B. – 8 – 4 i
5 5 B. 2 + i
A. – 1 i
4 2
C. + 8 i 1
C. – 2 – i
5 5 B. i
2 D. – 3 + i
4 8 C. 1 + 2i
D. – – i
5 5 E. 3 – i
4 D. 1 – 2i
E. – 8 i
5 5 E. – 1 – i x + i
24. Para que sea un
1 + i
16. Si z1 = 4 – 2i y imaginario puro, x debe
z2 = 5 + 6i, entonces 20. El valor de (i–2 – i–1 )–2 es:
valer:
Re (z1 z2) es: A. 2i
A. 9 A. 1
B. – 2i
B. 12 B. – 1
–1
C. 2i C. 0
C. 14
1
D. 20 D. D. 2
2i

E. 32 E. 1–i E. – 2

410 Números complejos

410-412 410 10/11/2001, 17:17


CAPÍTULO 9

1 – 2i 33. El conjugado de
25. Para que sea un 29. El valor de z E k que
3x – i (i–5 + i–12)–1 es:
número real, x debe satis face la ecuación
z– 1 =0 A. 1 + i
tomar el valor: z
B. 1 – i
A. cualquier complejo
A. 6
B. 1 y – 1 C. 1 + 1 i
B. – 6 2 2
1
C. 1 – 2i D. 1 – 1 i
C. 6 2 2
–1
D. – i
D. 6 E. – 1 – 1 i
E. – i 2 2
E. 1 34. Un complejo cuya parte
30. Si z = 1 – 3i, entonces
real es 3 y cuyo valor
z es:
26. Si z = 1 – i, entonces absoluto es 13 es:
2z2 – z + 1 vale: A. – 1 + 3i A. –3 + 2i
B. 1 + 3i B. –3 + 2i
A. 3
C. – 1 – 3i C. 3 – 2i
B. – 3 D. 3 – 3i
D. 1 + 1i
C. 3i 3 E. 3 + 3i
D. – 3i E. 1 – 1i
3 35. Un número complejo
E. 3 + 3i 31. Si z1 = 1 – 2i y z2 = 3i, tal que su cuadrado es la
z1 mitad de su conjugado
entonces es:
z2 es:
27. En la ecuación 5 1 3
z (1 – i) + 3 = 1 – 2i + 2z, A. A. + i
2 4 4
z vale: 3
B.
5
B. – 1 + i
3 4 4
A. 2 1 1 3
C. C. – i
3 4 4
B. – 2 1 3
D. + i
C. 2i D. 2 4 4
3 1 3
E. – i
D. – 2i E. 1 4 4

E. 1 – 2i 36. El complejo – 2 + 2i en
32. El valor absoluto de forma polar es:
i10
es: A. 2 2 (cos 45° + i sen
28. Si z = 2 – 3i, la parte i4 + i3
45°)
imaginaria de 1 es: A. 2
z2 B. 2 (cos 45° + i sen
1 45°)
A. 12 B.
2
C. 2 2 (cos 135° +
B. 12 • 13 C.
2
2 i sen 135°)
C. 12 • 132
D. 1 D. 2 2 (cos 225° +
D. 12 • 13–2 i sen 225°)
2
E. 13 E. E. 2 2 (cos 315° +
2
i sen 315°)

Números complejos 411

410-412 411 10/11/2001, 17:19


Prueba de selección múltiple
37. El complejo 5 (cos 180° 40. Si z1 = 4 (cos 100° + 43. Son raíces cuarta de –1:
+ i sen 180°) en su forma i sen 100°) y z2 = 2
1
(cos 55° + i sen 55°), I) + 1 i
cartesiana es: z1 2 2
A. 5 entonces z =
2
II) – 1 + 1 i
B. – 5 2 2
A. 2 + 2i 1 1
C. 5i III) – – i
2 2
D. – 5i B. 2 – 2i 1 1
IV) – i
E. – 5 + 5i C. 2 + 2 i 2 2
A. sólo I
38. Si z1 = 2 (cos 40° + D. 2 – 2 i
B. sólo I y IV
i sen 40°) y z2 = E. – 2 + 2 i
1
C. sólo II y III
(cos 20° + i sen 20°), D. todas
2 41. Si z1 = 6 (cos 40° +
entonces z1 • z2 = E. ninguna
i sen 40°) y z2 = 3 (cos
1 3
A. + i 280° + i sen 280°), enton- 44. Dos números cuya suma
2 2
z es 5 y su producto es
3 1 ces z 1 =
B. + i 2 25 son:
2 2
A. – 2 – 2 3 i 5+2 3 5–2 3
C. – 1
+ 3 i A.
2
i; i
2 2 2
3 B. – 1 – 3i 5+3 3 5–3 3
D. – + 1 i B.
2
i;
2
i
2 2
1 C. – 1 + 3i 5+4 3 5–4 3
E. – 3 i C. i; i
2 2 2 2
D. 1+ 3i 5–5 3
39. Si z1 = 2 (cos 290° + 5+5 3
D. i; i
–1 –1 2 2
i sen 290°) y z2 =3 (cos E. 3i
2 2 E. 2+ 3 i;3– 3 i
100° + i sen 100°),
entonces z1 • z2 = 42. El valor de (–2 + i)5 45. Es solución de la ecua-
ción x3 + 2 = – 2i
A. 3 3 – 3i A. – 38 + 41i
A. 1+i
B. 3 3 + 3i B. – 38 – 41i
B. –1+i
C. 3+3 3i C. 38 + 41i C. 2+i
D. 3–3 3i D. 32 + 41i D. –2+i
E. – 3 3 – 3i E. 1–i
E. 32 – 41i

Soluciones

1. A 8. B 15. A 22. D 29. B 36. C 43. D


2. C 9. E 16. E 23. A 30. B 37. B 44. D
3. C 10. B 17. A 24. B 31. B 38. A 45. E
4. E 11. E 18. B 25. C 32. C 39. B
5. B 12. C 19. B 26. D 33. D 40. C
6. D 13. B 20. C 27. A 34. C 41. C
7. B 14. E 21. C 28. D 35. B 42. C

412 Números complejos

410-412 412 10/11/2001, 17:21


CAPÍTULO 10
V ectores

Definiciones 10.1

Llamamos vector a un segmento dirigido. A su punto inicial


lo llamamos origen y a su punto final extremo. Distinguimos el
extremo porque en él dibujamos una punta de flecha.
Denotamos un vector: AB, PQ, RS , u, v.
B S
v
u
P Q
A R
Cada vector se caracteriza por tener magnitud, dirección y
sentido.
La magnitud o longitud es la distancia entre su origen y su
extremo y se llama valor absoluto, módulo o norma del vector. Se
designa por ||AB|| o ||v||.
La dirección es la dirección de la recta que contiene al vector y
de todas sus paralelas. Se representa por el ángulo θ, que se forma
entre la horizontal y la recta que contiene al vector.
Cada dirección admite dos sentidos, y éste está dado por la
punta de la flecha.
Todos los vectores que trasladados paralelamente coinciden,
constituyen el mismo vector. En un sistema de referencia
cartesiano, los asimilamos con el vector cuyo origen es el (0, 0) y
cuyo extremo es (u1, u2).

u2
u u1 y u2 se llaman coordenadas
q del vector u = (u1, u2).
u1

Vectores 413

413-421(2003).indd 413 01-02-2006 13:32:24


El valor absoluto, módulo o norma del vector u = (u1, u2) es:
|| u|| = u21 + u22 y corresponde a la longitud de la flecha que lo
representa.

Existen múltiples situaciones de la vida real que se representan


con vectores; éstas se llaman magnitudes vectoriales. Por ejemplo:
un avión se dirige a aterrizar a una velocidad constante de 160 Km/hr
en una trayectoria que forma un ángulo de 15° con la horizontal.
El sentido está dado hacia la pista.

15º
160

Otras situaciones como el número de hijos en una familia, la


edad de una persona o el valor de una casa quedan perfectamente
definidas con un número. No requieren de una dirección ni
de un sentido para precisarlas. Estas se llaman MAGNITUDES
ESCALARES.

10.2 Operaciones con Vectores

10.2.1 Suma de vectores

v2 Sean u = (u1, u2) y v = (v1, v2)


v vectores, entonces:
+
v u
u + v = (u1 + v1, u2 + v2)

u2 Geométricamente, el vector suma


u de dos vectores es la diagonal del
paralelogramo que se forma con
v1 u1
ambos vectores.

414 Vectores

413-421(2003).indd 414 01-02-2006 13:32:26


CAPÍTULO 10

También podemos decir que el vector


suma está representado por la flecha
+v
que resulta al unir el origen de u con el u
v
extremo de v, después de haber trasladado
paralelamente v hasta que su origen
coincida con el extremo de u. u
v

10.2.2 Producto por escalar

Sea u = (u1, u2) un vector y


k ∈ R un escalar, entonces:
k • u = (ku1, ku2)
3u2
Geométricamente, el vector
3u producto por escalar resulta de
u2 poner el vector a continuación
u
de sí mismo tantas veces como
u1 3u1 indica el escalar. En la figura
k = 3.

Si el escalar es negativo (k < 0),


entonces el vector producto por
escalar resulta de sentido inverso v2
–2v1 v
al vector u.
v v1
–2
Nota: El vector 0 = (0, 0)
El vector – u = –1 • u –2v2

10.2.3. Propiedades de la suma y


el producto por escalar
Sean u, v y w vectores y k1, k2 escalares en R.

1. u + v = v + u
2. u + ( v + w) = ( u + v) + w
3. u + 0 = u
4. u + (– u) = 0
5. k1 ( u + v) = k1 u + k1 v
6. (k1 + k2) u = k1 u + k2 u
7. (k1 k2) u = k1(k2 u)
8. 1•u=u

Ver demostraciones en ejercicios resueltos N°3.

Vectores 415

413-421(2003).indd 415 01-02-2006 13:32:27


10.2.4 Resta de vectores
Sean u = (u1, u2) y v = (v1, v2)
dos vectores, entonces:

u – v = u + (– v)

u
v

–v
Geométricamente, para
restar el vector v del vector u
u dibujamos desde el mismo origen
de u el vector – v y la diagonal del

u
paralelogramo así formado es el
–v
–v
vector u – v.
Observación: el vector que va
del extremo de v al extremo de u
también es u – v.

v
Además, podemos decir que
si a continuación del vector u
dibujamos – v, la flecha que une u
el origen de u con el extremo de –v
– v representa el vector u – v. u–v

10.3 Vector Unitario

10.3.1 Definición

Se llama VECTOR UNITARIO al vector cuyo valor absoluto o norma es 1.

Ejemplo: u =
( 12 , 5
13 13 ) || u|| =
144 25
+
169 169
=
169
169
=1

v=
( 2 , 5
3 3 ) || v|| =
4 5
+ =
9 9
9
9
=1

416 Vectores

413-421(2003).indd 416 01-02-2006 13:32:29


CAPÍTULO 10

Hay dos vectores unitarios especiales denotados por:


î = (1, 0) Ĵ = (0, 1)

Cualquier vector puede ser escrito como combinación lineal


(C.L) de ambos:
Ejemplo: (5, 2) = 5(1, 0) + 2(0, 1) = 5î+ 2Ĵ
Observamos que î indica la componente horizontal y Ĵ señala la
componente vertical del vector.

10.3.2 Normalizar un vector

Se llama normalizar un vector u al procedimiento utilizado para


conseguir otro vector û con la misma dirección y sentido que el
vector original pero de magnitud, módulo o norma 1.

Para ello, basta multiplicar el vector dado por el inverso de su


norma:

Ejemplo 1:

Sea u = (3,4); || u|| = 9 + 16 = 25 = 5

Entonces,

û= 1
|| u||
• u=
1
5
(3,4) =
( )
3 , 4
5 5

9 16
en efecto, ||u|| = + =1
25 25

Ejemplo 2:

Sea u = 2î + 4Ĵ ; || u|| = 4 + 16 = 20 = 2 5

Entonces,

1 1 1 2
û= u= (2î + 4Ĵ) = î+
5 Ĵ

|| u|| 2 5 5

1 4
en efecto, ||u|| = + =1
5 5

Vectores 417

413-421(2003).indd 417 01-02-2006 13:32:30


10.4 Descomposición de un vector

Sea u = (u 1 , u 2 ) un vector cualquiera. Este puede


descomponerse en su parte horizontal y su parte vertical como
sigue: llamamos α al ángulo medido desde el eje horizontal al
vector u , entonces:

u1 u2
cos α = y sen α = , luego
|| u|| || u|| u2

u1 = || u|| cos α y u2 = || u|| sen α


u
Es decir, el vector u = (u1, u2) puede escribirse:
α
u= || u|| cos α î + || u|| sen α Ĵ
u1

Ejercicios 1. Sean u = (2, 4), v = (–3, 1) y los escalares k1= 3 y k2 = –1


a) Hallar y graficar k1 u + v
resueltos
b) Calcular k2( u + v)
Solución:
a) k1 u + v = 3(2, 4) + (–3, 1)
= (6, 12) + (–3, 1)
= (3, 13)

Gráficamente: –

12 –


– u
8–
v


3u +


u

4–


u
v –
| | | –| | | | | | |
–3 –2 –1 2 4 6

418 Vectores

413-421(2003).indd 418 01-02-2006 13:32:34


CAPÍTULO 10

b) k2( u + v) = –1((2, 4) + (–3, 1))


= –1(–1, 5)
6
_
= (1, –5)
5
_
Gráficamente: _
_
_ u
v _

_
_
_
_
_

_
_
_
_
_
–3 _ 1 2
_
_
_ –1( u + v)
_
–6 _

2. Determinar la norma del vector u = (–5, 3)

Solución: || u|| = ||(–5, 3)|| = (–5)2 + 32 = 25 + 9 = 34

3. Demostrar las siguientes propiedades de la suma y el producto por


escalar.
a) u + v = v + u
Demostración:
u + v = (u1, u2) + (v1, v2) (sumando los vectores)
= (u1 + v1, u2 + v2) (aplicando conmutatividad de la
suma en R)
= (v1 + u1, v2 + u2) (Descomponiendo una suma
de vectores en sumandos)
= (v1,v2) + (u1 , u2)
=v+u
b) (k1 + k2) u = k1 u , k2 u

Demostración:
(k1 + k2) u = (k1 + k2) (u1, u2) (multiplicando por escalar)
(
= (k1 + k2)u1, (k1 + k2)u2 ) (aplicando distributividad
del producto sobre la
suma en R.)
= (k1u1 + k2u1, k1u2 + k2u2) (descomponiendo la suma
de vectores en sumandos)
= (k1u1, k1u2) + (k2u1, k2u2) (aplicando producto por
escalar)
= k1(u1, u2) + k2(u1, u2)
= k1 u + k2 u

Vectores 419

413-421(2003).indd 419 01-02-2006 13:32:37


4. Normalizar el vector u = 3î + 7Ĵ _
7
6
_
Solución:
1 5
_
u= • u
|| u|| 4
_
3_
|| u|| = 9 + 49 = 58
2_
1 3 7
u= (3î + 7Ĵ) = î+ Ĵ 17 _
58 58 58 58

_
_
_
3
u ≈ (0.4, 0.9) 58 1 2 3

5. Hallar la norma y la dirección del vector u = (–4, 3)


Solución:

a) || u|| = (–4)2+32 = 25 = 5
3_
u –4
2_ b) cos θ = 1 =
_ || u|| 5
u 1
θ
θ = arcos ( –45 ) = 143,1°
_
_
_
_

–4 –3 –2 –1

La norma del vector u es 5 y su dirección es de 143,1°.

6. Hallar las componentes horizontal y vertical del vector de norma


12 y dirección 54°.
Solución:
Sabemos que
u2 u1 = || u||cos θ y u2 = || u||sen θ
u Por lo tanto:
54º u1 = 12 cos 54° ≈ 3,4641 • 0,5878 = 2,0362
u1
u2 = 12 sen 54° ≈ 3,4641 • 0,8090 = 2,8025

3
7. Determinar el valor de m para que el vector u = î + 2mĴ sea
4
un vector unitario.
Solución: un vector unitario es aquel cuya norma es 1.
Por lo tanto:

( 34 ) + (2m) = 1
2
|| u|| = 2

9 2
16 + 4m = 1
7
4m2 =
16
2 7
m =
64
7
m=±
8

420 Vectores

413-421(2003).indd 420 01-02-2006 13:32:40


CAPÍTULO 10

Luego los vectores unitarios que resultan son:

3 3
u= î + 7 Ĵ y u= î – 7 Ĵ
4 8 4 8

8. Determinar el valor de m para que el vector u = 2î + mĴ sea un


vector unitario.
Solución: Para que u sea unitario debemos hacer || u|| = 1.
|| u|| = 22 + m2 = 4 + m2 = 1
4 + m2 = 1
m2 = –3
No existe un número real tal que su cuadrado sea –3. Esto se
debe a que ningún vector de R2 cuya primera coordenada sea 2
puede ser unitario.

1 Observe que el lugar


geométrico de todos
los vectores unitarios es
-1 1 2 circunferencia de radio 1.

-1

9. Dados los vectores: u v w

mostrar geométricamente que ( u + v) + w = u + ( v + w)

Solución: v w
v+w
u v
u+ )+ w
(u + v
+ w)
u + (v
10. Dado los vectores u = (3, –2) y v = (4, 5), hallar la norma del
vector u – v. Graficarlo.

Solución: || u – v|| = (3 – 4)2 + (–2 – 5)2 = 1 + 49 = 50

5–


v
– u–v


_
_
_
_

– 3 4
u
–2 –

Vectores 421

413-421(2003).indd 421 01-02-2006 13:32:43


11. En un punto A de un objeto se aplica una fuerza de 42N en
dirección 60°. Calcular la magnitud de la fuerza que actúa en
forma horizontal y de la fuerza que actúa en forma vertical.

ƒ2 N
42 Solución:
A 60º ƒ1 = 42 cos 60° = 21N
ƒ1
ƒ2 = 42 sen 60° = 21 3 N

La fuerza que actúa en dirección vertical es de 21 3 N y la


que actúa en forma horizontal es de 21N.

12. Un móvil se desliza en un plano inclinado en 32° con una


velocidad instantánea de 12 m/seg. Calcular la componente
horizontal y la componente vertical de dicha velocidad.

v1
32º
v2 v1 = –12 cos 32° = –10,18 m/seg
32º v2 = –12 sen 32° = –6,36 m/seg

Ejercicios
1. Determine cuáles de las siguientes 3. Determine la dirección y la norma
medidas se representan por vectores de los siguientes vectores.
y cuáles por escalares: a) v = 2î + 7Ĵ
a) 5 hijos b) v = –5î – 3Ĵ
b) 25 panes c) v = 2î – 9Ĵ
c) 18 km/hr d) v = –4î + 5Ĵ
d) $25.000 1 25
e) v = î +
e) La velocidad de aproximación 3 4 Ĵ
de un avión. 1
f) v = –3î + Ĵ
f) Desaceleración a 5 m/seg2. 2
4. Realice las siguientes operaciones
2. Determine la dirección y la norma entre los vectores u = 2î + 3Ĵ ,
de los siguientes vectores. Dibújelos v = 4î – Ĵ , w = 3î – 2Ĵ , p = –5î,
en el plano cartesiano. s = 6Ĵ y grafiquelos en el sistema
a) u = (5,2) cartesiano.
b) u = (2,–7) a) u + v
b) 2u – 3v
c) u = (–5,1)
c) u + v + w
d) u = (–6,–3) d) u – 5p
e) u = (5, 2) e) 3(u + s )
f) u = (– 5, 3 ) f) p – 2u + 3w

422 Vectores

422-431 422 25/11/02, 12:00 PM


CAPÍTULO 10

5. Escriba los siguientes vectores como 10. Encuentre la longitud y dirección


combinación lineal de los vectores de los siguientes vectores:
î = (1, 0) y Ĵ = (0, 1). a) u = (–5, 7)
a) u = (2, 5) b) v = (3, –2)
b) v = (3, –2) c) w= (10, 8)
c) w = (–8, –3) d) u = (–6, –3)
d) u = (–5, 0) e) v = (–5, 0)
e) v = (7, 0) f) w = (0, 3)
f) w = (0, –3)
11. Encuentre la longitud y dirección de
6. Encuentre un vector unitario que tenga los siguientes vectores:
la misma dirección que el vector dado. a) u = 2î – 5Ĵ
a) u = (4, 2) b) v = î + 6Ĵ
b) u = (–6, 1) c) w= 2 î + 2Ĵ
c) u = (5, –3) d) u = 5î – 4Ĵ
d) u = (–3, –4) e) v = 2î
e) u = (3, 8) f) w= –4Ĵ
f) u = (–5, 0)
12. Encuentre las componentes horizontal
7. Encuentre un vector unitario que y vertical de los vectores, conocida su
tenga la misma dirección que el magnitud y su dirección.
vector dado. a) 2, 45°
a) v = 3î + 5Ĵ b) 5, 30°
b) v = î + 4Ĵ 1
c) , 60°
c) v = –2î – 6Ĵ 3
d) v = 4î – Ĵ d) 10, 12°
π
e) v = –6î e) 12,
4
f) v = 4Ĵ f) 100, 115°
8. Normalice los siguientes vectores g) 25, 90°
a) u = (6, 2) h) 4, 135°
b) v = (–9, 3) i) 8, 212°
c) w = (5, –8) j) 3, 270°
d) u = (–1, –3) 3π
k) 18,
4
e) v = (1, 1) l) 5, π
f) w = (0, 5)
13. Dados los vectores u = 3î + 5Ĵ y
9. Normalice los siguientes vectores. v = 6î – 2Ĵ, encuentre la norma del
a) u = 3î – 5Ĵ vector 2u – 3v.
b) v = –2î + Ĵ 14. Para trasladar horizontalmente un
c) w = – 3Ĵ objeto desde un punto A hasta un
d) u = 2î + 2 Ĵ punto B se aplica una fuerza de
30N en dirección 30°. Calcular la
1 3
e) v = î– magnitud de la fuerza que actúa en
2 4 Ĵ
la dirección AB y la fuerza que actúa
f) w = – 5 î en dirección normal a AB.

Vectores 423

422-431 423 25/11/02, 12:00 PM


15. Un avión se desplaza en dirección 16. Una persona desea cruzar un río
60° NO a una velocidad de 800 en bote a una velocidad media de
km/hr y no hay viento. Al llegar 32 km/hr y sabe que el agua fluye
a cierto punto de su trayectoria se a 8 km/hr. ¿Qué velocidad y en
encuentra con un viento sobre él qué dirección debe imprimir a
de 60 km/hr en dirección 30° NO. su bote para mantener un rumbo
Calcule la velocidad real del avión al perpendicular a la orilla del río?
ser expuesto a ese viento.

Soluciones

1. a) escalar b) escalar c) vectorial


d) escalar e) vectorial f) vectorial

2 _
1 _ 6 5 4 3 2 1
2. a) ; 29; 21,8° d) ; 3 5; 206,6°
_
_
_
_
_

_
_
_
_
_
_
1 2 3 4 5
1 _
2 _
3 _


_
_

– 1 2
b) – ; 53; 285,9° _

– e) 2 _ ; 3 3 ; 15,8°


_
_
_
_
_

–2 – 1 2 3 4 5

c) ; 26; 168,7°
1 _ 3
_
_
_
_
_

_
–5 4 3 2 1 f) ; 2 2 ; 142,2°
_
_

– 5

3. a) 53; 74,05° b) 34; 210,96° c) 85; 282,5°


d) 41 ; 141,34° e) 6,26; 86,95° f) 3,0414; 189,5°

4. a) 6î + 2ĵ b) –8î + 9ĵ


9–

3_ –
2_ u
2u

6–
v
1_

u+
3v

– u
_
_
_
_
_
_

–1 _ 1 2 3 4 5 6 3–
v -v
-v – u
- v
| | | | | | | | | | | | –| | | | |
–12 –8 –4 –- v

424 Vectores

422-431 424 25/11/02, 12:01 PM


CAPÍTULO 10

c) 9î d) 27î + 3ĵ
3_
2_
u
1_ u+ v
u+v+ w
_
_
_
_
_
_
u – 5p
_ v 9
-1
w –5 p -p -p -p -p -p 25 27
-2 _
-3 _

e) 6î + 27ĵ f) –12ĵ

5. a) 2î + 5Ĵ d) –5î 10. a) 8,6; 125,50° d) 3 5; 206,57°


b) 3î – 2Ĵ e) 7Ĵ b) 3,6; 213,69° e) 5; 180°
c) –8î – 3Ĵ f) –3Ĵ c) 12,81; 38,66° f) 3, 90°

6. a) ( 2 5, 5
5 5 ) d) ( –3 , –4
5 5 ) 11. a) 5,39; 21,80°
b) 6,08; 80,54°
d) 6,40; 321,34°
e) 2; 0°
b) ( –6 , 1
37 37 ) e)
( 10 , 4 10
10 5 ) c) 2; 45° f) 4; 270°

12. a) (1,1) g) (0, 25)


c) ( 5 , –3
34 34 ) f) (–1,0)

7. a)
3
î+
5
ĵ d) 4
î–
1

b) ( 5 3, 5
2 2 ) h) (–2 2, 2 2)

34 34 17 17

b)
1
î+
4
ĵ e) –î
c) ( 1 , 3
2 6 ) i) (–6,78; –4,24)

17 17 d) (9, 78; 2,08) j) (0, –3)


1 3
c) –
10
î–
10
ĵ f) ĵ
e) ( 12 , 12
2 2 ) k) (9 2, –9 2)

8. a) ( 3 , 1
10 10 ) d) ( –1 , –3
10 10 ) f) (–42,26; 90,63) l) (–5, 0)

13. 20
b) (
–9 , 3
10 10 ) e) ( 2 , 2
2 2 ) 14. f1 = 15 3 f2 = 15

c) ( 5 , –8
93 93 ) f) (0,1)
15. El avión se desplaza a 852,47 km/hr en
dirección 147,98° NO.

5 2 16. 32,98 km/hr; 104° con la orilla.


9. a) 3 î – ĵ d) î+
1

34 34 6 3
–2 1 2 3
b) î+ ˆj e) î– ĵ
5 5 13 13
c) – Ĵ f) – î

Vectores 425

422-431 425 25/11/02, 12:01 PM


Producto Punto (o producto escalar) 10.5

10.5.1. Definición
Sean u = (u1, u2) = u1î + u2Ĵ y
v = (v1, v2) = v1î + v2Ĵ vectores.

Se llama PRODUCTO PUNTO entre los vectores u y v al


escalar que se obtiene de la siguiente forma:

u • v = u1v1 + u2v2

Ejemplo: u = (3, 2) = 3î + 2Ĵ


v = (5, –6) = 5î – 6Ĵ

u • v = 3 • 5 + 2 • (–6) = 15 – 12 = 3

Nota: El producto punto se llama también producto escalar (no


confundir con el producto por escalar) debido a que su resultado
es un escalar.

10.5.2. Propiedades
Sean u, v y w vectores y k un escalar en R, entonces:

1. (u + v) • w = u • w+v • w
2. u • v=v • u
3. (ku) • v = u • (kv) = k(u • v)
4. u • u= ||u||2 > 0; u • u=0¤u=0

10.5.3 Angulo entre vectores

Como sabemos, cualquier vector se puede


representar por un segmento dirigido con origen en
el origen del sistema cartesiano.
Llamamos q al ángulo formado por los vectores v
u y v . El ángulo 0 ≤ q ≤ π será cero si ambos
vectores tienen igual dirección y sentido y será π q u
si ambos vectores tienen igual dirección y sentidos
opuestos.

426 Vectores

422-431 426 25/11/02, 12:01 PM


CAPÍTULO 10

El ángulo q formado por los vectores u y v, ambos distintos de


cero, se obtiene a través de la expresión:

u•v
cos q = Ver ejercicio resuelto n°2
|| u || || v ||

Dos vectores se dicen PARALELOS si están contenidos en la


misma recta que pasa por el origen. Si u y v son vectores paralelos,
entonces existe k ∈ R tal que u = k v

Dos vectores se dicen ORTOGONALES si están contenidos en


rectas perpendiculares que pasan por el origen. Dos vectores u y
v son ortogonales si y sólo si u • v = 0.

Ejemplo:

6_
_
_ Sean u = (4, –2) = 4î – 2Ĵ
_ v v = (3, 6) = 3î + 6Ĵ
_
_
_
_
_
_

_ 3
u u • v = 12 + –12 = 0
–2 _

10.5.4 Proyección de un vector sobre otro

Sean u y v dos vectores distintos de


cero. Se llama:
• proyección vectorial de u sobre
v al vector:
u•v
pr(u , v) = v
|| v ||2

u
) u
pr( v ,

• proyección vectorial de v sobre


, v) v u al vector:
pr( u
v•u
pr(v , u) = u
|| u ||2

Ver ejercicio n°4

Vectores 427

422-431 427 25/11/02, 12:01 PM


Ejercicios 1. Dados los vectores u = 3î + 2Ĵ , v = –î + 5Ĵ y w = 6î – 3Ĵ.
resueltos Hallar (u + v) • w

Solución:
Primero sumamos u + v = 3î + 2Ĵ – î + 5Ĵ = 2î + 7Ĵ
Ahora hacemos (u + v) • w = (2î + 7Ĵ) • (6î – 3Ĵ)
= 2 • 6 + 7(–3) = –9

2. Determinar el ángulo formado por los vectores u = (2, 5) y


v = (–3, 2)
5_
_
_ Solución:
_ u Sabemos que el coseno del
ángulo entre dos vectores
θ_
está dado por:
_
_
_

_
_
_

–3 2

u•v 2 • –3 + 5 • 2 4
cos θ = = = ≈ 0,2060
|| u || || v || 29 • 13 377
θ = 78,1°

3. Hallar el valor de m para que los vectores u = mî + 5Ĵ y


v = 4î – (1 + m)Ĵ sean ortogonales.
Solución:
Para que dos vectores sean ortogonales, el ángulo q formado entre
ellos debe ser de 90°; por lo tanto, cos q = cos 90° = 0

u•v
Sabemos que cos θ =
|| u || || v ||
Para que este valor sea cero, u • v debe ser cero,
∴u • v = 4m – 5(1 + m) = 0
4m – 5 – 5m = 0
–5 – m = 5
m = –5

4. Determinar la proyección de u sobre v si u = 3î + 5Ĵ y


v = 6î + 2Ĵ.
Solución: Sabemos que pr(u, v) = u • v2 • v
|| v ||
3•6+5•2
Es decir, pr(u, v) = (6î + 2Ĵ)
40
28 21 7
= 40 (6î + 2Ĵ) = î+ .
5 5 Ĵ

428 Vectores

422-431 428 25/11/02, 12:01 PM


CAPÍTULO 10

Gráficamente:

5_
4_
3_
u
2_
1_ )
_ pr(u,v v
_
_
_
_
_
_
1 2 3 4 5 6

5. Determinar el ángulo obtuso del triángulo ABC sabiendo que


A = (3, 1), B = (6, 6) y C = (–2 ,2).

Solución:
Tenemos que: OA + AB = OB
∴ AB = OB – OA
6_ B AB = (6, 6) – (3, 1) = (3, 5)
5 _
y OA + AC = OC
4_
3_ ∴ AC = OC – OA
C 2_ u θ AC = (–2, 2) – (3, 1) = (–5, 1)
v 1_ A
θ
_
_
_
_
_

_
_
_
_
_
_

–5 –4 –3 –2 –1 1 2 3 4 5 6

Si llamamos u = AB = (3, 5) y v = AC = (–5, 1) el problema se


reduce a calcular el ángulo entre los vectores u y v.

u•v –15 + 5 –10


Luego cos θ = = = ≈ –0,3363
|| u || || v || 34 • 26 2 221
_1
Así θ = cos (–0,3363) = 109,65°

Ejercicios

1. Dados los vectores u = (3, –1), v = (5, 2) y w = (1, 6). Calcule.


a) u • v d) (u + v) • w g) 3u • 2v
b) u • w e) 3u • v h) u • u
c) v • w f) u • 3v i) w • w

Vectores 429

422-431 429 25/11/02, 12:01 PM


Ejercicios
2. Encuentre el ángulo formado por cada uno de los siguientes pares de vectores.
a) u = 2î + Ĵ v = –î + 2Ĵ
b) u = –3î – Ĵ v = 5î + Ĵ
c) u = î – 2Ĵ v = –î – 3Ĵ
d) u = 4î + Ĵ v = î + 5Ĵ
e) u = 3î + 5Ĵ v = –5î + 3Ĵ

3. Determine si los siguientes pares de vectores son paralelos. Si lo son, determine


si tienen igual o distinto sentido.
a) u = 3î + 2Ĵ v = 4,5î + 3Ĵ
b) u = î + 5Ĵ v = –î – 5Ĵ
c) u = 12î – 6Ĵ v = –16î + 8Ĵ
d) u = – 2î + 2Ĵ v = 2î + 2Ĵ
e) u = –2î – 4Ĵ v = –3î + 6Ĵ
f) u = 5î + Ĵ v = 6î + 2Ĵ

4. Determine cuáles de los siguientes pares de vectores son ortogonales y


cuáles no lo son.
a) u = 2î – 3Ĵ v = 4î + Ĵ
b) u = 6î – Ĵ v = 3î – 18Ĵ
c) u = 6î – Ĵ v = –3î – 18Ĵ
d) u = 4î + 2Ĵ v = –î – Ĵ
e) u = 5î v = –8Ĵ
f) u = 6î v = –12î

5. Encuentre el valor de m para que los siguientes pares de vectores sean


ortogonales.
a) u = 3î – mĴ v = î + 2Ĵ
b) u = mî + 2Ĵ v = 2î – 3Ĵ

c) u = 2î + Ĵ
3
v = mî + ( 1 + 3
2 )Ĵ

d) u = 1 î + v = î – mĴ
2 4 Ĵ
1 1
e) u = î– Ĵ v = (m + 1)î – Ĵ
2 2
f) u = 2mî – Ĵ v = 4î + 6mĴ

6. Encuentre el o los valores de m para que los siguientes pares de vectores sean
ortogonales. Analice la pertinencia de las soluciones.
a) u = 2mî – 3Ĵ v = mî – (m + 1)Ĵ
b) u = 4î – 2mĴ v = 5î – mĴ
c) u = (3m – 1)î – 2 v = mî + 6Ĵ
d) u = mî – 3mĴ v = 6î +(2m – 1)Ĵ
e) u = î + Ĵ v = mî – 2mĴ
f) u = –5î – mĴ v = –3î + (m + 2)Ĵ

430 Vectores

422-431 430 25/11/02, 12:02 PM


CAPÍTULO 10

7. Demuestre que u • v=v • u

8. Determine la proyección del vector u sobre el vector v y la proyección del


vector v sobre el vector u. Grafique.
a) u = –5î + ĵ v = 6ĵ
b) u = 5î + 2ĵ v = î + 8ĵ
c) u = –î + 3ĵ v = 6î + 5ĵ
3
d) u = î + 9ĵ v = 2î – 3ĵ
4
e) u = 6î v = 3î – 5ĵ

9. Dado el vector u = 5î + 2ĵ , encuentre su proyección sobre el eje x y sobre


el eje y.

10. Dado el vector u = 3î + 5ĵ


a) Encuentre u • î
b) Encuentre u • ĵ
c) Compare estas soluciones con las del ejercicio anterior. Establezca alguna
conclusión.

11. Pruebe que si u = u1î + u2ĵ , entonces:


i) u • î es la proyección de u sobre el eje x.
ii) u • ĵ es la proyección de u sobre el eje y.

12. Encuentre un vector cualquiera que sea ortogonal al vector dado:


a) u = (3, –6)
b) u = (–1, 15)
c) u = (2, –3)
d) u = (–5, –1)

e) u = ( )
1 1
,
2 3
f) u = (– 2, 3)
g) u = (2,5; –3)
h) u = (–0,5; –3,5)

13. Un vector de R2 es un par ordenado u = (u1, u2). Generalizando, un vector


de Rn es una n-upla ordenada.
u = (u1, u2, u3, …..un)
Pruebe que en Rn: u • v = v • u

14. Sean u = (3 3,3), v = (3, 3 3) y w = (–3, 3 3). Determine el ángulo formado


por u y v, el ángulo formado por u y w y el ángulo formado por v y w.

Vectores 431

422-431 431 25/11/02, 12:02 PM


Ejercicios
15. Determine el ángulo obtuso del triángulo formado por los puntos A=(2, 2),
B=(5, 1) y C=(1, 5)

16. Se tienen los puntos A=(2, 1), B=(0, 3) y C=(6, 0). Determine el ángulo obtuso
del triángulo ABC.

17. Sea ABC el triángulo formado por los vértices A=(–4, 2), B=(1, –4) y C=(3, 6).
Determine la medida de los segmentos en que hc divide al lado AB.

18. Sea ABC el triángulo formado por los vértices A=(–1, –1), B=(4, 2) y C=(–4, 3).
Calcule medida de los segmentos en que la altura hb divide al lado AC.

19. Sea ABC el triángulo formado por los vértices A=(–1, 3), B=(4, –2) y C=(8, 3).
Determine la medida de los segmentos que ha genera en el lado BC.

20. Encuentre ambas alturas del paralelogramo formado por los vectores u = (6, 2)
y v = (3, 4).

21. Sea ABCD el paralelogramo cuyos vértices son A=(0, 0), B=(6, –4), C=(9, –2),
D=(3, 2). Determine su área.

Soluciones

1. a) 13 b) -3 c) 17 d)14 e) 39 f) 39 g) 78 h) 10 i) 37

2. a) 90° b) 172,9° c) 45° d) 64,65° e) 90°

3. a) sí, igual. b) sí, distinto. c) sí, distinto. d) no. e) no. f) no.

4. a) no. b) no. c) sí. d) no. e) sí. f) no.

3 1 3
5. a) – b) 3 c) – – d) 2 e) –2 f) 0.
2 4 4 3

6. a) No existe. b) No existe.c) 1 ± 73
6
d) 1 ; si m = 0, u = 0 y no podemos hablar de ortogonalidad.
2
e) No existe. f) 3, –5.

432 Vectores

432-442 432 20/11/02, 6:06 PM


CAPÍTULO 10

7. u • v = (u1, u2) • (v1, v2) = u1v1 + u2v2 = v1u1 + v2u2 = (v1, v2) • (u1, u2) = v • u.

8. a) Ĵ b) 0,3î + 2,6Ĵ c) 1,08î + 0,9Ĵ d) –3,33î + 7,27Ĵ e)1,59î – 2,65Ĵ .

9. 5î ; 2Ĵ .

10. a) 3î b) 5Ĵ .

11. a) Representemos el eje x por el vector v = (x, 0) = xî, pr(u,v) = u1 x+ u2 0 • xî = u1î


que es la proyección del vector u sobre el eje x. x2
b) Idem considerando al eje y como el vector v = (0, y).

6 12
12. a) (2, 1) b) (15, 1) c) (6, 4) d) (2, –10) e) (2, –3) f) ( , 1) g) ( , 2) h) (–7, 1)
2 15
13. Ver respuesta ejercicio 7. 14. 30°, 90°, 60°.
15. 126,87° 16. 120,9°.
17. 1,42; 6,39. 18. 5,6; 0,6.
19. 0,78; 5,62. 20. 2,85; 3,6.
21. 24.

3
Vectores en el espacio IR 10.6

10.6.1 Definiciones
Un vector de R 3 es un segmento dirigido en el espacio
tridimensional. Análogamente a lo enunciado en el punto 10.1,
todos los vectores de R3 pueden considerarse con origen en
el origen del sistema cartesiano y extremo en cualquier punto
del espacio.
Z
7

5 Z
4
5
3
4
u2
–4

3
1
–3

2
–2

1 Y
1 u
–1

2 1 2 3 4
3 Y
4 1 2 3 4 5 6 7
5 X

u = (5, 2, 7) u = (–3, 5, –1)


X

Vectores 433

432-442 433 20/11/02, 6:06 PM


Llamamos:
î = (1, 0, 0) al vector unitario en la dirección x.
Ĵ = (0, 1, 0) al vector unitario en la dirección y.
kˆ= (0, 0, 1) al vector unitario en la dirección z.
Cualquier vector de R3 se puede escribir como combinación
lineal de los vectores î, Ĵ y k̂.
Las operaciones vectoriales mencionadas en el presente capítulo
se extienden todas a R3. Ver ejercicios resueltos.
Además:
î•î =1 î • Ĵ = 0
Ĵ • Ĵ = 1 î • kˆ = 0
kˆ• kˆ = 1 Ĵ • kˆ = 0

10.6.2. Producto vectorial o producto cruz


En forma especial se define el producto vectorial o producto cruz para vectores
de R3 de la siguiente forma:

Sean: u = u1î + u2Ĵ + u3kˆ y


v = v1î + v2Ĵ + v3kˆ vectores de R3

entonces el producto vectorial entre u y v es el vector:

w = u x v = (u2v3 – u3v2)î – (u3v1 – u1v3)Ĵ + (u1v2 – u2v1)kˆ

Obsérvese que los coeficientes de î, Ĵ y kˆ son el desarrollo de determinantes


2 x 2 (ver cap. 9.5), por lo tanto:

u2 u3 u1 u3 u1 u2
| | |
u x v = v v î – v v Ĵ + v v kˆ
2 3 1 3 1 2
| | | o

î Ĵ kˆ
|
u x v = u1 u2 u3
v1 v2 v3
|
Observaciones:
1. El vector w = u x v es ortogonal al vector u y al vector v.
2. El valor absoluto del producto “mixto” u • v x w representa el volumen del
paralelepípedo generado por los tres vectores.
u1 u2 u3
3. El valor de u • v x w se encuentra al calcular el determinante v1 v2 v3 , cuyo
w 1 w2 w3
valor absoluto corresponde al volumen del paralelepípedo formado por los vectores
u, v y w.
4. Sean u y v dos vectores de R3 que forman un ángulo θ, entonces,
||u x v|| = ||u|| ||v|| sen θ. ( Ver ejercicio resuelto nº 2).

434 Vectores

432-442 434 20/11/02, 6:07 PM


CAPÍTULO 10

Ejercicios
1. Dados u = (3, 2, 5) y v = (–3 ,1, 0), encuentre u x v y v x u.
resueltos
Solución:
î Ĵ kˆ
| |
u x v = 3 2 5 = –5î – 15Ĵ + 9kˆ
–3 1 0

î Ĵ kˆ
| |
v x u = –3 1 0 = 5î + 15Ĵ – 9kˆ
3 2 5

Observamos que el vector u x v = –(v x u), es decir, al conmutar


dos vectores en un producto cruz se generan vectores en
sentidos opuestos y con igual magnitud.

2. Dados los vectores u = (3, 1, 5) y v = (4, 2 ,–3).


a) Calcular el ángulo θ formado por ellos.
b) Calcular ||u|| ||v|| sen θ.
c) Calcular ||u x v||.
d) Comparar los valores de b) y c).

Solución:
u•v
a) Sabemos que cos θ = .
|| u || || v ||
3 • 4 + 1 • 2 + 5 • –3 –1
cos θ = = ≈ –0,0314
9 + 1 + 25 • 16 + 4 + 9 1.015

de donde θ = 91,7987°.

b) ||u|| ||v|| sen θ = 35 • 29 sen 91,7987° = 31,843.

î ĵ kˆ
c) u x v =
| |
3 1 5 = (–3 – 10)î – (–9 – 20)Ĵ + (6 – 4)kˆ
4 2 –3

u x v = (–13, 29, 2) y

||u x v|| = 169 + 841 + 4 = 1.014 ≈ 31,843

d) Los resultados de b) y c) son iguales. Con ello hemos dado


un ejemplo de que ||u x v|| = ||u|| ||v|| sen θ, donde θ es
el ángulo formado por los vectores u y v.

Vectores 435

432-442 435 20/11/02, 6:07 PM


3. Determinar el volumen del paralelepípedo formado por los
vectores u, v y u x v en R3 si u = (2a, 0, 0), v = (a, a 3, 0).

Solución: Veremos dos formas.

a)

Calculemos u x v = w .
w

Ĵ kˆ
| |
î
0 a 3
w= 2a 0 0 = 0î – 0Ĵ + 2 3a2kˆ
θ
a a 3 0
u v

2a
w = (0, 0, 2 3 a2)
B
A
Se trata entonces de un paralelepípedo
C cuya base es el paralelogramo ACBO y
su altura es ||w|| = OD

El área de las bases OA ¥ BA (base • altura), donde


OA = ||u|| y BA = ||v|| sen θ.

a 3
Sen θ = ∴ BA = a 3
|| v ||

||u|| = 4a2 = 2a ∴ AACBO = 2a x a 3 = 2a2 3.

El volumen del paralelepípedo es:

V = AACBD • OD = 2a2 3 • 2 3a2 = 12a4

b) Otra forma.
Sabemos que el volumen del paralelepípedo formado por los
vectores u, v y w es el valor absoluto del cálculo de u • v x w; y

| |
2a 0 0
u•vxw= a a 3 0 = 2a • a 3 • 2a2 3 = 12a4
0 0 2a2 3

Luego el volumen del paralelepípedo es 12a4.

436 Vectores

432-442 436 20/11/02, 6:07 PM


CAPÍTULO 10

Ejercicios
1. Grafique los siguientes vectores:
a) u = 3î + 2Ĵ + 5kˆ
b) u = 5î – 2Ĵ + kˆ
c) u = î + 6Ĵ – 3kˆ
d) u = –2î – 3Ĵ – 5kˆ
e) u = –3î + 2Ĵ – 4kˆ
f) u = –3î – 6Ĵ + 5kˆ
2. Dados los vectores u = (3, 2, 1) y v = (5, 4, 2), calcule u x v y v x u. Compare
ambos resultados.
3. Dadas las siguientes parejas de vectores en R3, calcule u x v y v x u en cada
caso y grafíquelos.
a) u = 2î + 3ĵ + 0kˆ v = 4î + Ĵ + 0kˆ
b) u = î + 2Ĵ + 0kˆ v = 5î – 3Ĵ + 0kˆ
c) u = –3î + 5Ĵ + 0k ˆ v = –2î – 3Ĵ + 0kˆ
4. Dadas las siguientes parejas de vectores de R3, calcule u x v y v x u en cada
caso y grafíquelos.
a) u = 2î + 3kˆ v= 4î – 5kˆ
b) u = î + 2kˆ v = –î + 2kˆ
c) u = 5î – kˆ v = 5î + 2kˆ
5. Dadas las siguientes parejas de vectores de R3, calcule u x v y v x u en cada
caso y grafíquelos.
a) u = 2Ĵ – 3kˆ v = Ĵ + 4kˆ
b) u = 3Ĵ – 6kˆ v = 2Ĵ + 2kˆ
c) u = –3Ĵ +kˆ v = 4Ĵ – 4kˆ
6. Calcule un vector normal a los vectores u = (2, 2, 1) y v = (1, 1, –5)
7. Calcule m para que el vector w = (5, m+1, –2) sea normal a los vectores
u = (5, 3, –1) y v = (4, 2, 1)
8. Sean u = (5, 2, 0) y v = (2, –5, 0) dos vectores :
a) Calcule w = u x v
b) Calcule u • v x w
c) Calcule v • u x w
d) Grafique el paralelepípedo generado por los vectores u, v y w.
e) Calcule el volumen de dicho paralelepípedo.
9. Sean u = (0, 3, 4) y v = (0, 4, –3) dos vectores
a) Calcule w 1 = u x v
b) Calcule w 2 = v x u
c) Grafique el paralelepípedo formado por los vectores u, v y w en ambos casos.
d) Calcule u • v x w en ambos casos.
e) Calcule el volumen de los paralelepípedos construidos en c.

Vectores 437

432-442 437 20/11/02, 6:07 PM


10. Sean u = 3î + kˆ y v = –2î + 6kˆ dos vectores
a) Calcule w 1 = u x v
b) Calcule w 2 = v x u
c) Grafique los paralelepípedos formados por los vectores u, v y w 1 y u, v y w 2.
d) Calcule u • v x w 1 y v • u x w 1
e) Calcule u • v x w 2 y v • u x w 2
f) Calcular el volumen de los paralelepípedos graficados en c.
11. Sean u = (1, –2, 3) y v = (–5, 1, 2). Calcule:
a) w 1 = u x v
b) w 2 = v x u
c) Calcule el volumen de los paralelepípedos generados por los vectores
u, v y w 1 y u, v y w 2.

12. Dados los puntos A(3, 2, 1) y B(6, 4, 5). Calcule las coordenadas del vector AB .

13. Las componentes del vector AB son (3, 0, 4) y las coordenadas del punto B son
(0, 3, 1). Halle las coordenadas del punto A.

14. Calcule el ángulo formado por los vectores u = (4, 3, 1) y el eje vertical
representado por el vector w = (0, 0, 2).

15. Calcule el ángulo formado por los vectores u = 5î + 2Ĵ – 3kˆ y v = î – 2Ĵ + kˆ.

Soluciones

2. u x v = (0, –1, 2); v x u = (0, 1, –2); u x v = –(v x u)


3. a) –10kˆ; 10kˆ b) –13kˆ; 13kˆ c) 19kˆ; –19kˆ
4. a) 22Ĵ , –22Ĵ b) 2Ĵ ; –2Ĵ c) –5Ĵ ; 5Ĵ
5. a) 11î , –11î b) 18î ; –18î c) 8î ; –8î
6. (–11,11,0)
7. m = –2.
8. a) w = –29kˆ b) 841 c) –841 e) 841
9. a) –21î b) 21î d) 525 e) Ambos tienen volumen 525
10. a) –20Ĵ b) 20Ĵ d) –400; 400 e) 400; –400.
f) Ambos tienen volumen 400.
11. a) (–7, 13, 11) b) (7, –13, –11) c) 369
12. (3, 2, 4)
13. (–3, 3, –3)
14. 78,69°
15. 97,61°

438 Vectores

432-442 438 20/11/02, 6:07 PM


CAPÍTULO 10

Prueba de selección múltiple


1. Si u = (2, 1) y v = (3, 2), 5. Los vectores u = (–1, 3, 2) C. ||u|| cos a
entonces u + v es: y v = (1, –3, –2) difieren
en: D. ||u|| sen a
A. (2, 3) E. u • u
A. Sólo magnitud.
B. (6, 2)
B. Sólo sentido. 9. El vector unitario en
C. (5, 3) la dirección y sentido
C. Sólo dirección. del vector u se expresa
D. (2, 3) por:
D. Dirección y sentido.
E. (–2, –1) A. u = ||u|| u
E. Magnitud y sentido.
2. Si u = (–1, 0) y v = (3, –1) u
B. u =
entonces 2u – 3v es: ||u||
6. Si u = (–1, 1, 1), entonces
A. (–10, –3) ||u|| vale: C. u = (1, 1)

B. (–10, 3) A. 0 D. u = ||u||

C. (10, –3) B. 1 1
E. u =
||u||
D. (–11, 3) C. 2
10. El vector unitario del
E. (11, 3) D. 2 vector u = (12,5) es:
12 5
3. El valor de k para que E. 3 A. ( , )
u = (k, –1, 1) y 13 13
7. Si u = (–3, –1), entonces –5 12
v = (1, 1) sean iguales es: B. ( , )
el vector unitario en la
13 13
A. 0 dirección de u es:
5 12
C. ( , )
B. 1 A. u = (–3, – 1) 13 13
12 5
C. 2 3 1 D. ( , )
B. u = ( , ,) 13 13
D. –2 10 10
5 12
–3 –1 E. ( , )
C. u = ( , ) 13 13
E. –1 10 10
3 –1 11. Si u = ( 3, 1), el ángulo
4. La longitud del vector D. u = ( , ) que forma con el eje x, es
u = (0, 1, 1) es: 10 10
decir, su dirección, es:
–3 1
A. 1 E. u = ( , ,)
10 10 A. 60°
B. 2 8. Si el vector u tiene B. 30°
dirección a, entonces
C. 2 C. 45°
la componente vertical
D. 3 de u es:
D. 120°
E. 3 A. u cos a
E. 150°
B. u sen a

Vectores 439

432-442 439 20/11/02, 6:07 PM


Prueba de selección múltiple
12. Sea u = mî + 2Ĵ. El valor 18. De los vectores
de m para que u sea B u = (2, –1); v = (–1, –2);
unitario es: A
w = (–4, 2) y o = (4,2)
son ortogonales:
1 60º 30º
A.
2 A. w y o

B. – 1 A. El sistema se des- B. u y w
2 plaza hacia el lado
1 C. v y o
C. de A.
4 D. u y v
B. El sistema se des-
D. – 1 plaza hacia el lado E. u y o
4 de B.
E. No existe. 19. Si u = 2î + 5Ĵ; v = î – 2Ĵ y
C. El sistema está en
w = 3î – Ĵ, entonces
equilibrio.
1 u + v • w es:
13. Si u = î + 3 Ĵ, el
m 2 D. No se puede deci-
A. –3
valor de m para que u dir.
sea unitario es: B. 3
E. En el desplazamiento
A. ±1 sólo interviene la incli- C. 6
B. 0 nación del plano.
D. –6
C. ±2 16. Si la magnitud del vector
E. 9
u es 2 y su dirección
D. ± 1 es 225°, entonces sus
2 20. El valor de m para que
componentes son:
u = î – mĴ y v = (1 – m)î
E. ± 1
3 A. (1, 1) + 2Ĵ sean ortogonales
es:

14. Si u = 2î – 5Ĵ y v = –3î B. (1, –1) A. 0


+ Ĵ, la norma del vector
u – v es: C. (–1, 1) B. 1

A. 11 C. –1
D. (–1, –1)
B. 25 1
D.
E. (0, 2) 3
C. 36 1
17. Si u = (3, 2) y v = (–5, 1), E. –
3
D. 61 entonces u • v es:
E. 1 21. Si u = 2î + 4Ĵ y v = 5î
A. 0
+ 3Ĵ, entonces el vector
15. En la figura, el cuerpo A B. 3 pr (u , v) es:
pesa 15 kg y B pesa 10
C. 5 5 3
kg. Prescindiendo de la A. î+ Ĵ
fuerza de roce podemos 17 17
D. 12 2 4
decir que: B. î+ Ĵ
E. 15 17 17

440 Vectores

432-442 440 20/11/02, 6:08 PM


CAPÍTULO 10

55 33 B. (–1, –2, –1) 28. Sean A = (2, 5, 1) y


C. î+ Ĵ
17 17 B =(–3, 2, 1) dos puntos,
C. (–1, –2 ,1)
33 55 entonces las coordena-
D. î+ Ĵ D. (–1, 2, –1)
17 17 das del vector AB son:
E. (–1, 2 ,1)
33 55 A. (5, –3, 1)
E. î+ Ĵ
17 17 25. Si u = 2î – 3Ĵ + kˆ y
B. (5, –3, 0)
v = î + Ĵ – kˆ entonces
22. Si u = 2î + 4Ĵ y v = 5î
u x v es: C. (–5, –3, 1)
+ 3Ĵ, entonces el vector
pr(v, u) es: A. (2, 5, 3) D. (–5, 3, 0)
3 B. (2, 5, –3) E. (–5, –3, 0)
A. î + Ĵ
5
C. (2, 3, 5)
29. Los componentes del
2 4
B. î+ Ĵ D. (2, –3, –5) vector AB son (5, 2, –1)
5 5
y las coordenadas del
22 11 E. (2, –3, 5) punto A son (0, 3, 2). Las
C. î+ Ĵ coordenadas del punto B
5 5
26. El vector normal a los son:
11 22 vectores u = (2, 1, 1) y
D. î+ Ĵ v = (1, –1, 3) es: A. (5, 5, 1)
5 5

11 22 A. 4î – 5Ĵ + 3kˆ B. (5, 1, –3)


E. î+ Ĵ
5 5 B. 4î – 5Ĵ – 3kˆ C. (5, 5, –1)

23. El vector (3, 4, –2) es C. –4î + 5Ĵ – 3kˆ D. (5, –1, 3)


igual a:
D. –4î – 5Ĵ + 3kˆ E. (5, –5, –1)
A. 3(1,0,0) – 4(0,1,0) –
E. 4î + 5Ĵ + 3kˆ 30. El ángulo formado por los
2(0,0,1)
vectores u = (1, 1, 1) y
27. Son normales al vector v = (1, –2, 1) es:
B. 3(1,0,0) + 4(0,1,0) +
u = 3î – Ĵ + kˆ los
2(0,0,1)
vectores: A. 30°
C. 3(1,0,0) – 4(0,1,0) – I) 3î – Ĵ – 10kˆ B. 45°
2(0,0,1)
II) –2î – 3Ĵ + 3kˆ C. 60°
D. 3(1,0,0) + 4(0,1,0) –
2(0,0,1) III) 2î + 3Ĵ – 3kˆ D. 90°

A. Sólo I E. 180°
E. –3(1,0,0) + 4(0,1,0)
+ 2(0,0,1) B. Sólo II

24. Si u = (1, 0, 1) y v = (2, C. Sólo III


1, 0) entonces u x v es:
D. Sólo I y II
A. (1, 2, 1)
E. I, II y III

Vectores 441

432-442 441 20/11/02, 6:08 PM


Soluciones

1) C 11) B 21) C

2) D 12) E 22) D

3) C 13) C 23) D

4) C 14) D 24) E

5) B 15) B 25) C

6) E 16) D 26) B

7) C 17) D 27) E

8) D 18) D 28) E

9) B 19) C 29) A

10) D 20) D 30) D

442 Vectores

432-442 442 20/11/02, 6:08 PM


CAPÍTULO 11
M
atrices
y determinantes

Conceptos básicos 11.1

Definición: Se denomina MATRIZ a una ordenación rectangular de


elementos. Estos elementos serán en general números reales.

En esta ordenación rectangular distinguimos las FILAS de la


matriz, que son las líneas de elementos ordenados horizontalmente,
y las COLUMNAS de la matriz, que son las líneas de elementos
ordenados verticalmente.
Si la matriz tiene m filas y n columnas se dice que es de orden
m ¥ n o su dimensión es m ¥ n.
Cada elemento de la matriz está entonces identificado por
la posición que ocupa, esto es, está en la intersección de una
fila y una columna.
Denotamos la matriz A de dimensión m ¥ n por A = [aij] m ¥ n,
donde cada aij representa el elemento situado en la fila i y en la
columna j. Es claro que 1 £ i £ m
1£j£n
Si la matriz consta de sólo 1 fila se llama matriz fila o vector
fila. Si consta de una sola columna, se llama matriz columna
o vector columna.
EJEMPLOS:
2 1 3
1. Sea A = 1 –1 2
es una matriz

de orden 2 ¥ 3 (2 filas y 3 columnas)

2. A = [1 2 2 – 3] es una matriz fila de orden 1 ¥ 4.

Matrices y determinantes 443

443-445(2003) 443 20/11/02, 6:19 PM


Ejercicios
junio julio agos. sept.
1. Dada la siguiente tabla de notas:
Detergente $448 $452 $452 $490
P1 P2 P3 P4
Colonia $820 $779 $790 $800
Altamira 6.1 6.5 4.9 3.1
Lavaloza $375 $375 $390 $410
Contreras 2.3 3.9 4.9 6.4
Limpiavid. $440 $430 $450 $460
Fernández 5.1 2.8 3.1 4.8
Lustramueb. $400 $410 $430 $470
Martínez 2.6 3.4 3.8 5.3
Valdés 6.3 6.6 3.7 6.9 a) Precio de lavaloza en los meses
Zamora 4.6 4.8 5.3 6.6 de agosto y septiembre.
b) Diferencia de precios de limpia-
Responda las siguientes preguntas: vidrio entre junio y septiembre.
a) ¿Qué nota obtuvo el 1er alumno c) Porcentaje de la colonia entre
en la 3a prueba? junio y julio.
d) Diferencia de precios entre lava-
b) ¿Qué nota obtuvo el 3er alumno loza y detergente en el mes de
en la 1a prueba? agosto.
c) ¿Cuál es la mejor nota del 4º 1 2 1 3
alumno? 5. Dada la matriz: 4 –1 –2 0
M= 3 2 –2 4
d) ¿Quién y en qué prueba sacó la
2 1 3 6
nota más baja?
5 2 –1 –1
e) ¿Quién y en qué prueba sacó la
nota más alta? a) Determine el orden de M.
b) Escriba los elementos m12; m53;
2. Determine el orden de las siguientes m24; m33.
matrices: c) Escriba los elementos de la 5a
fila.
2 –3 2 3 –1 d) Escriba los elementos de la 3a
a) 1 0 1
b) 1 3 columna.
4 –4
e) Calcule 2 m34 – 3 m43
1 2 1 1 1 1 3 f) Calcule m11 + m22 + m33 + m44
c) 4 1 0 d) –1 –1 –2 1
1 –2 0
2 3 –1 –3 6. Escriba con los siguientes elementos:
2 3 5
2
– 3, 2, 5, 0, 3, 1, 9, 12, – 3, – 5, 1, 0,
e) 101 f) 1 (se supone que están leídos por filas)
una matriz cuyo orden es:
3. Determine el número de elementos de a) 2 ¥ 6 b) 3 ¥ 4 c) 4 ¥ 3 d) 6 ¥ 2
una matriz si su orden es:
7. Escriba una matriz M de orden 3 ¥ 3 que
a) 2 ¥ 5
cumpla las siguientes condiciones:
b) 2 ¥ 2 a) los elementos de la 2a columna
c) 3 ¥ 4 son 1, 2, 3
d) 6 ¥ 3 b) los elementos de la 3a fila son 2, 3, 1
c) a11 = a22 + a33
e) m ¥ n d) a13 = a31
f) n ¥ p e) a12 = a21
4. Dada la siguiente tabla, señale: f) a23 = a21 + a22

444 Matrices y determinantes

443-445(2003) 444 20/11/02, 6:20 PM


CAPÍTULO 11

8. Determine en cada caso una matriz 9. Construya una matriz M (3 ¥ 3) de


que cumpla la condición indicada. modo que cumpla las siguientes
condiciones:
a) A [3 ¥ 4] ai = 0 si i = j
a) a22 = 0 d) a32 = a13
b) A [4 ¥ 3] aij = 1 si i > j
b) a23 = 2 e) a12 = a33 = –a32
c) A [2 ¥ 3] aij = i + j
c) a32 = a21 = 1 f) a11 = a31 = –a23
d) A [3 ¥ 3] aij = i – j2
e) A [4 ¥ 3] aij = i + j – 1

Soluciones
–3 2 5 0
–3 2 5 0 3 1
1. a) 4.9 b) 5.1 c) 5.3 6. a) b) 3 1 9 12
9 12 –3 –5 1 0
–3 –5 1 0
d) 2º alumno; 1a prueba.
e) 5º alumno, 4a prueba.
–3 2
2. a) 2 ¥ 3 b) 3 ¥ 2 c) 4 ¥ 3 –3 2 5 5 0
d) 3 ¥ 4 e) 1 ¥ 3 f) 2 ¥ 1 c) 0 3 1 d) 3 1
9 12 –3 9 12
3. a) 10 b) 4 c) 12 –5 1 0 –3 –5
1 0
d) 18 e) m n f) n p
7. – 8. –
4. a) $ 390 y $ 410 b) $ 20
c) El precio disminuyó un 5% d) $ 62
–2 –1 1
5. a) 5 ¥ 4 b) 2; – 1; 0; –2. c) 5 2 –1 –1 9. 1 0 2
d) 1 –2 –2 3 – 1 e) –1 f) 4 –2 1 –1

Igualdad y adición 111.2


de matrices
11.2.1 Matrices iguales
Dos matrices son iguales si se cumple:

a) Tienen el mismo orden, es decir, el mismo número de filas


y el mismo número de columnas.
b) Los elementos correspondientes a igual fila e igual columna
son iguales.

11.2.2 Adición de matrices


Sean A y B dos matrices de orden m ¥ n.
Definimos la ADICIÓN de A y B por:

Matrices y determinantes 445

443-445(2003) 445 20/11/02, 6:20 PM


A + B = C si y sólo si
aij + bij = cij donde 1 £ i £ m
1 £ j £ n.
Observación:
1. La matriz resultante C también es de orden m ¥ n.
2. No es posible sumar dos matrices que difieran ya sea
en el número de filas, en el número de columnas o
en ambos.

11.2.3 Propiedades de
la adición
Sean A, B y C matrices de orden m ¥ n. (escribimos
A E M m ¥ n, B E M m ¥ n,…), entonces:

1. La matriz A + B también es de orden m ¥ n, es decir:


A E Mm ¥ n y B E Mm ¥ n, entonces A + B E Mm ¥ n.
Ésta es la propiedad de la CLAUSURA.
2. A + B = B + A
La adición de matrices es conmutativa.
3. (A + B) + C = A + (B + C)
La adición de matrices es asociativa.
4. Existe la matriz nula de orden m ¥ n tal que:
A+0=A
donde la matriz 0 = [aij] con aij = 0 Ii, Ij
5. Existe la matriz inversa
I A E Mm ¥ n, H (– A) E Mm ¥ n fi A + (– A) = 0
El conjunto de las matrices con la operación ADICIÓN
recién definida y las propiedades que posee tiene estructura
de GRUPO ABELIANO O GRUPO CONMUTATIVO.

Ejercicios
1. Determinemos el valor de las incógnitas si A = B siendo:
resueltos 2–y a –2
A= y B= b 5
–3 x
Aplicando directamente la definición de igualdad de
matrices tenemos que:
a=2 y=2 b = –3 x=5
2. Determinemos el valor de x e y si A = B y
y
A = [2 – x] B =
–3
En este caso A tiene orden 1 ¥ 2 y B tiene orden 2 ¥ 1; por lo
tanto, NO puede haber igualdad.
3. Determinemos los valores de las incógnitas si A = B siendo

446 Matrices y determinantes

446-447 446 10/11/2001, 13:37


CAPÍTULO 11

u+v 2 x 4 u –v 3
A = w 4 B =
3 –y 1 4

Aquí tenemos u + v = –4
u – v = –2 } Q u=3 ; v=1
x = –3
y = –3
w = –1

4. Obtengamos la suma de A y B siendo:


1 –3 0 2
A = –1 2 y B = –2 5
4 –2 1 0
Ambas tienen el mismo orden; por lo tanto, se pueden sumar.
Sumamos los elementos correspondientes y obtenemos:
1+0 –3+2 1 –1
A + B = –1+–2 2+5 = –3 7
4+1 –2+0 5 –2
5. Aplicando las propiedades de la adición en las matrices
podemos resolver ecuaciones matriciales.
Resolvamos A + X = B siendo
– –3 4
A = 2 2 y B =
1 8 12 –1

Si A + X = B, entonces sumando la matriz opuesta de A, que


se denota (– A) a ambos lados, obtenemos:
X = B + (– A)
para obtener (–A) cambiamos el signo de cada elemento de A. Así:

–3 4 –2 2
X = +
12 –1 –1 –8

X = –5 6
y efectivamente, sumando A y X se obtiene B.
11 –9

Ejercicios
1. Determine el valor de la(s) variable(s) si en cada caso A = B siendo:
a) A = 1 0 B = x y d) A = a +b a–b B = 4 –2

4 –2
1 x 1 –3 y+1
b) A = B = 2x 3
y 1 e) A = B =
4 1 0 3 0 3
2 x y a 3 –4 x+y 1– y 3x 4
c) A = B = f) A = B =
1 0 1 b 0 1 0 –2 0 –2

Matrices y determinantes 447

446-447 447 10/11/2001, 13:38


Ejercicios
x2 5
g) A = 3y B= –2 x–y x2 – y2 x3
z+1 –1
i) A = x – 5y – 3x2 2 x3
u+v 2 2x 2
h) A = B= 2x – 3 – 2y x3
x 2–y 2x + y 2x2
u–x w 5 y+1 3

–z
2 2 –6z w 2x+y x2 + y2 – 2 x3
i) A = 3 B= 5 x–y B= – 5y 2x2 x3
x + y 12
x–3y 3x2 0
j) A = [a2 + b2 2a] B = [a2 – b2 9] 1+ 2+ 3 –1 + 2 2 – 3
j) A= 1– 2– 3 B == 1– 2+ 3
–1+ 2+ 3 1– 2+ 3
2. Dadas las matrices A y B, determine en
3. Dadas las matrices:
cada caso la suma A + B.
–2 0 3 3 –1 2
a) A = – 2 4 5 B = 06 60 33 A= B=
1 4 –1 0 1 4
1 –1 0
C= –3 1 –3
1
2 –2 1 – 2 5 –3
2
1 1 1 Obtenga g) A+B+C
b) A = B= –2 –
2 4 2 a) – A d) A + B h) B–A
1
–1 4
1 2 b) – B e) A – C i) C–A
3 2
c) – C f) A + B – C j) C–B–A
1 –2 1 2 0 –1
c) A = 2 1 2 B= –2 4 3
4. Determine el valor de las variables en
0 3 –3 1 5 –2
cada caso:
d) A = 2 2 3 3 5
1 +2x 3 5 3 2y –2
1 –1 2 a) +
2 2–y –3 z – 1 –3
3 2 –3 3 5 5
B= = 9 5 u
0 1 1 8 v w

a + 2b – 3a 2 a–b 2b a2 b + a2 –2 b 8 –3
b) =
e) A = 2b – a a + b B= 3 b – 2a –3a 2b c 0 4 d 6
b – 5 a 3a a + 4b b –7a
c) [6 + x 2 y 6 z] + [3 x 1 – y 4 – z]
f) A = [ x + y 2 x – y 3 x – 3y]
= [x –y z]
B = [ x + y x – 2 y 2 x + 4y] 1+a 4 2a 2 –3 x
d) 1+b 3 + 2b –3 = –6 y
2 a–b 3 a + 2b 1+c –3 2c 5 –9 z
g) A =
b – 5a b + 3a
5. Resuelva las siguientes ecuaciones
matriciales
a+b b
2a –
3 5 2 –3 +X= 1 0
B= a)
b – 3a 1 5 0 –1
3 a + 4b –2
5
2 3 1 –1
–2 2

[ ] +X= 2 –5
h) A = 1 + √2 b)
–1
B= 6 1 3 0
1 – √2 0
–1 5 –4 2

448 Matrices y determinantes

448-449 448 10/11/2001, 18:00


CAPÍTULO 11

c) X + [1 2 – a 3 + a] 6. Exprese la siguiente matriz:


= [a – 1 2 a a – 2] A=
1 0 –1 0 0 –3
d) X – 2 0 2 = 0 –3 0
1 3 –1 2 0 –1
Como:
1 2 0 2
e) 3 1 –X= 1 1 a) Suma de dos matrices cualesquiera.
7 2 2 –2
b) Suma de dos matrices tales que la
a –2b 3b –2a 3b –b
f) –X= diagonal principal de una de ellas esté
b–a 2b –b a+b – b – 2b
fomada sólo por 1.
1+ 2 1 c) Suma de dos matrices tales que los
g) 3 –X= 0 elementos que están bajo la diagonal
2– 3 1
principal de una de ellas sean todos
iguales a cero.
h) [2, 1 – 3, 7 0, 4] + X
d) Suma de dos matrices tales que los
= [– 3, 3 1, 7 2, 5]
elementos que están sobre la diagonal
2 4 3 –1 principal de una de ellas y los que están
3 3 5 2 bajo la diagonal principal de la otra
i) – X= sean todos iguales a cero.
–3 5 –1 2
4 2 7 9 e) Suma de dos matrices tales que la
1–m m+5 m m diagonal principal de una de ellas esté
j) X – =
2+3m 2m 0 1 formada sólo por ceros.

Soluciones

1. a) x = 1 y = 0 b) x = – 3 y=4 3 –2 0 4 2 – 3 8 5
c) 0 5 5 d)
c) a = 2 x = 3 d) a = 1 1 8 –5
1 0 3
b=1 y=–4 b=3
3a+b –3a+2b
2 –3 e)
e) x = f) x = 5b–3a b–2a
3 2 5b–4a b–4a
y=–3 y=–3 f) [2 x + 2 y 3x–3y 5 x + y]
–2
g) x = ± 5 y= z=–2 7a–5b 29 b
3 5a+ 2
g) 3 5 h)
h) x = – 3 y=–2 u=2 1– 2
21 a + 3 b
v=–8 w=–1 13 b – 2 a
5
17 –7 – 2 2
i) x = y= z= w=
2 2 3 9
3x 2x2 – x3
9
j) a = b=0 i) x – 10 y – x2 3x3
2
–5
3 x3
2 3x – 2 y 5 x2
–2 10 8 –3 –1 3
2. a) b)
7 –1 3 2 4 3 2
5 7 j) 2–2 2
6 2 2 3

Matrices y determinantes 449

448-449 449 10/11/2001, 18:01


2 0 –3 –3 1 –2 –1 –4
3. a) b) 0 –1 –4 –1 4 –7
–1 –4 1
5. a) X = 3 b) X = –3 –1
–1 –6
–3 –3
c) 3 –1 3 d) 1 –1 5
–2 –5 3 1 5 3
c) X = [a – 2 3a – 2 –5]
1 –1 6 4 –2 8
e) f)
–1 –1 2 –1 0 6 1 0 –4 1 0
d) X = 2 – 3 2 e) X = 2 0
5 –1 –1
g) – 2 0 2 h) –1 –3 5
3 3 –2 5 4
3 10 0
–5b 2
–1 1 –6 –4 2 –8 3a 4b
i) j) f) X = –2a 3b b
g) X = 3
1 1 –2 1 0 –6 1 – 3
5
4. a) x = z=6 v= 0 h) X = [ – 5,4 5,4 2,1]
2
y=1 u=3 w = –6
b) a = ± 2 c=2
1 11
b= 3 d=6 5 6
i) X = –17 41
c) x = – 2 y=– 1 z = –1 28 18
2
–4 –7 10
d) a = b= c= – 1 2m + 5
3 3 3 j) X =
2 + 3m 1 + 2m
x=6 y=0 z=2

11.3 Ponderación de una matriz


por un escalar

11.3.1 Definición
Ponderar una matriz A por un escalar α es multiplicar cada
elemento de la matriz A por el elemento α.
En general consideraremos a como un elemento del conjunto
de números reales.
El resultado de esta nueva operación nos da una nueva matriz
del mismo orden de la matriz original.

11.3.2 Propiedades
Notemos que la ponderación de una matriz por un escalar es una
operación entre elementos de diferentes conjuntos.
Sean A y B matrices de orden m ¥ n y sean a y b dos escalares.
(Consideremos a, b, E R).

450 Matrices y determinantes

450-451 450 20/11/02, 6:24 PM


CAPÍTULO 11

Se cumple:
1. a (A + B) = aA+aB
2. (a + b) A = aA+bA
3. (a b ) A = a (b A)
4. 1 ¥ A = A
Recordemos que (M m ¥ n , +) tenía estructura de Grupo Abeliano.
Si agregamos al conjunto esta nueva operación recién definida y
sus propiedades, ampliamos la estructura algebraica y obtenemos
un Espacio Vectorial.
Así, tienen estructura de Espacio Vectorial aquellos sistemas
formados por un conjunto dotado de una operación que tenga
estructura de Grupo Abeliano y de otra operación que tenga las
propiedades 1 a 4 indicadas anteriormente.

1. Sea A =
1 6 2
y a=4 Ejercicios
–2 0 3
resueltos
Obtengamos a A.
1 6 2 4 24 8
a A = 4A = 4 –2 0 3
=
–8 0 12
Para obtener a A simplemente multiplicamos cada elemento
de A por 4, que es el valor de a.
2. Determinemos el valor de las variables en:

1 0 4 y
x = z v–2
–2 3

Efectuemos la primera operación y luego planteemos las


igualdades correspondientes:
x 0 4 y
= z
– 2x 3x v–2
de donde: x=4
y=0
– 2x = z Q z=–8
3x = v – 2 Q v = 14

1 –2 –3 5
3. Dadas las matrices A= 3 1 y B= 2 –6
–3 4 5 –4

Resolvamos la ecuación X – 5A = – 3B

La solución de esta ecuación está dada por:

X = 5A – 3B es decir:

Matrices y determinantes 451

450-451 451 20/11/02, 6:24 PM


1 –2 –3 5
X= 5 3 1 + –3 2 –6
–3 4 5 –4

5 –10 9 –15
X= 15 5 + –6 18
–15 20 –15 12

14 –25
X= 9 23
–30 32

–12 9 27
4. Expresemos la matriz A = –3 –12 6

como el producto de un escalar por otra matriz.


Este problema tiene infinitas soluciones, pues podemos escoger
cualquier número real como el escalar, pero una simple inspección
de los elementos nos indica que podemos “factorizar” la matriz
por 3 y así mantener todos los elementos enteros.
– 12 9 27 –4 3 9
=3
– 3 – 12 6 –1 –4 2

5. La siguiente es una tabla de precios de artículos allí señalados.


pequeño mediano grande
óleo 1.100 1.600 2.000
látex 1.300 1.750 2.200
diluyente 700 1.000 1.300

Podemos considerar la información proveniente de una matriz


cuyas filas nos señalan el producto y cuyas columnas nos
señalan el tamaño.

Determinemos ahora la matriz correspondiente a los precios de


los artículos si ellos están rebajados en un 10%.

El nuevo precio se obtiene al multiplicar cada precio por 0.9, así


la matriz de los nuevos precios será:
1.100 1.600 2.000 990 1.440 1.800
0.9 1.300 1.750 2.200 = 1.170 1.575 1.980
700 1.000 1.300 630 900 1.170

Ejercicios
1. Calcule:

1 4
a) –2 1 4 b) –1 0 1 5 –1
1 2 3 –1 –3
e) 1
1 0 2 9
f)
1 6 –3 1 8 1 2 2 6 –3 4 2 2
c) 2 d) 5 –8 4 –3 –7
2 –1 2 –3 –2 –5 –4

452 Matrices y determinantes

452-453 452 10/11/2001, 13:47


CAPÍTULO 11

–2 –w + z
2 3 3 12 3
g) 2 h) e) x
4
= –1
–z
1 0 2 27 – 3 –3 –y
6 –12
a–b a+b
i) a + b a b j) 0 1 0 –1 2
5 3 15 –3 x x–1 9 p
–b –a
f) 2 y y+2 = – 3 11 q
2. Dadas las matrices:
z z–4 –4 r
1 –3 2 2 –2 4
A= 4 1 –3 B= 4. Resuelva las siguientes ecuacio-
3 1 5
–3 –3 4
nes matriciales:
C=
2 –1 5
a) X + 2 1 0 –1
= –3
2 –3 4
Determine: 2 1 –3 1 0 –3

a) 2A – B b) A – 2B + 3C 1 0 3 –3
b) 1
c) 3A – 3B d) 4 (A – B) 2 1 –X= –1 2 5
5 –3 2 4 –6
e) 2
A –
1
B f) 2 (A + B) – 2 (A – B)
5 2
c) 2 X – 3 1 4 1
= –3
–2 5 2
g) – 4 B h) B – 2 A –3 2 –3 2 –1 –5

1 –3 1
i) 1 C – 1 B + 1 A j) 3
2
A + B d) –2 1 0 –2
3 2 4 –3 –1 0 –4
3 2 3
3. Determine el valor de las incógnitas 4 2 –1
en cada caso: =X+ 5 –6 3

x y –1 5 5. Dadas las matrices:


a) 2 =3 u v
3 –2 1 2 –4 5 2 1
A= –1 3 ;B= 1 –1 yC= 1 0
b) –3 1 0 = 2 –3 – 5 w
u v
8
2 4 3 –2 –1 3
–2 x 5 4
resuelva las ecuaciones:
2x y x+9 y – 12 a) 3A + X = – B
c) –2 1 – y 2 = –u 2v
3 3+x w z b) 2A + C = B – X
c) 2X + A = – 3C
d) –3 xz uy = y + 1 x – 2 d) B – 2C – X = 3A
2z+1 2z

Soluciones
3
3
–2 –8 0 –1 2 6 2
1. a) b) g) h) –3
–2 –4 –1 0 2 0 9
2 2
2 12 –6 5 40 5 10
c) 4 –2 4 d) –15 –10 –25 –20
a 2 – b2 a2 + 2ab + b2
5

1 3
3
i) a2 + ab ab + b2
2 2 4 – ab – b2 – a2 – ab
9 –3 –9
1
e)
2
f)
4 4 j) 0 0 0 0
–3 3 3 0 0 0 0
3
2 2 2
–4 2
–9 –21 0 –4 0 –12 –8 6
4 4
2. a) b)
5 1 –11 4 –4 2

Matrices y determinantes 453

452-453 453 10/11/2001, 13:48


–3 –3 –6 –4 –4 –8
c) d) 4 0 –32
e) x = 2 y=–6 z=8 w=4
3 0 –24
–27 29 –33 29
–4 f) x = p= y= q=
0 0 2 3 2 3
5 8 –8 16
e) f)
1
1 – 11 12 4 20
z=6 r=– 4
5 5 3
16
–8 8 –16 0 4 0 –3
g) –12 –4 –20
h) 5
–5 –1 11 –8 9 –10 12 26
4. a) X =
–7 –2 15
b) X = 5 5
–5
–1 0 17 –28
i) 3 j) 8 –12 16 5 5
1 –1 –11 17 5 9 9 –3 –3
2 2 6 –5 –5 6
c) X = 2 2 2 d) X =
–3 –4 –15 9 –1 10 2
15 3
3. a) x = y= u=2 v= 2 2
2 2 3
–3 –1 1 –11 –8 0
b) u = 1 v= w= x = 12 5. a) X = 2 –8 b) X = 2 –7
5 5
–9 –10 0 –13
–9
c) x = y=4 u=–6 v=–2
5 –7 –5
–12 2 2
w=–6 z= –3
–11 –3
5 c) X = –1 d) X = 2 –10
–5 7 –1 2
2 –1 –20
d) x = y= z= u= 1 –13
8 8 5 15
2 2

11.4 Multiplicación de matrices

11.4.1 Procedimiento
Sean A y B dos matrices de orden 1 ¥ n y n ¥ 1, respectivamente.
El producto de A y B es una matriz de orden 1 ¥ 1 y se obtiene
sumando todos los productos de la forma a1i bi1 donde 1£ i £ j.
Así: A = a11 a12... a1n y B = b11
b21



bn
entonces AB = a1 b1 + a2 b2 + ... + an bn
Una ampliación de esta definición es la siguiente:
Sea A una matriz de orden m ¥ n y B una matriz de orden n ¥ p. El producto AB es una
matriz C de orden m ¥ p, donde cada elemento Cij se obtiene sumando los productos de
los elementos de la i-ésima fila de A con la j-ésima columna de B.
1 2
Ejemplo: A = 1 2 1 B = 1 3
3 1 2
3 –1

454 Matrices y determinantes

454-455 454 20/11/02, 6:32 PM


CAPÍTULO 11

A tiene orden 2 ¥ 3 y B tiene orden 3 ¥ 2, por lo tanto, la


matriz producto C tendrá orden 2 ¥ 2 (número de filas de A y
número de columnas de B).
El elemento C11 se obtiene multiplicando fila 1 de A por
columna 1 de B.
El elemento C12 se obtiene multiplicando fila 1 de A por
columna 2 de B.
El elemento C21 se obtiene multiplicando fila 2 de A por
columna 1 de B.
El elemento C22 se obtiene multiplicando fila 2 de A por
columna 2 de B.
Para cada elemento se suman los productos obtenidos.
1 •1 + 2 •1 + 1 •3 1 •2 + 2 •3 + 1 • – 1
AB = = 6 7
3 •1 + 1 •1 + 2 •3 3 •2 + 1 •3 + 2 • – 1 10 7

Observación: La multiplicación de matrices NO es una operación


conmutativa.
Si el número de columnas de una matriz no coincide con
el número de filas de la otra entonces el producto no se puede
efectuar.

11.4.2 Propiedades de la multiplicación

Como vimos anteriormente, no todas las matrices se pueden


multiplicar. Consideremos aquí el conjunto de matrices cuadradas,
de orden n ¥ n.
En este conjunto la multiplicación cumple las siguientes
propiedades:
1. Asociatividad.
Sean A, B, C matrices de orden n ¥ n, entonces
(A • B) • C = A • (B • C).
2. Existe neutro multiplicativo.
Sea A E M n ¥ n, entonces existe la matriz identidad
I E M n ¥ n tal que:
A • I = I • A = A.
Esta matriz, llamada también matriz unitaria, está formada por
1 en la diagonal principal y 0 fuera de ella.
3. El producto es distributivo respecto de la suma.
Sean A, B y C matrices de orden n ¥ n; se cumple:
A • (B + C) = AB + AC.
4. Existen divisores de cero.
Recordemos que en el conjunto de números reales R si el producto
de dos números es igual a cero, necesariamente uno de ellos es
cero. En el caso de las matrices, ello no ocurre y así podemos tener
dos matrices no nulas y tales que su producto es cero.
Así, afirmamos que H A, B E M n ¥ n, A π 0 A B π 0 tales
que AB = 0

Matrices y determinantes 455

454-455 455 20/11/02, 6:33 PM


Observación: El producto de matrices cuadradas de orden n
no es conmutativo.
–No todas las matrices son invertibles, es decir, dada una
matriz A de orden n ¥ n no siempre es posible encontrar una
matriz A–1 tal que A • A–1 = A–1 • A = I
Cuando es posible encontrar dicha matriz se dice que A es
invertible y A–1 se llama la matriz inversa de A.

11.4.3 Matrices inversas y


ecuaciones multiplicativas
Sea A una matriz cuadrada de orden 2 ¥ 2 dada por:

A= a b , donde a, b, c, d E R
c d
La matriz inversa de A viene dada por:
1 d –b
A–1 =
ad – bc –c a
La condición entonces para que la matriz A = a b sea
c d
invertible es que ad-bc sea distinta de cero.
La determinación de la matriz inversa de una matriz de orden
3 la dejaremos para el punto siguiente, pues con los determinantes
se facilita mucho su obtención.
Aplicando las matrices inversas podemos resolver ecuaciones
multiplicativas. Es decir, sean A y B dos matrices cuadradas.
Resolvamos: A • X = B
Si A es invertible, entonces existe A–1 tal que
A–1 • A = I = A • A–1 Así:
A•X = B /• A–1
A–1 • A • X = A–1 • B
Recordemos que la multiplicación no es conmutativa; por lo
tanto, si multiplicamos un lado de la igualdad por la izquierda,
debemos multiplicar el otro lado también por la izquierda.
Entonces la solución de la ecuación está dada por:
X = A–1 B

Ejercicios 2 3
1. Sea A = [ –1 2] y B= 1 –3
resueltos
a)¿Es posible efectuar el producto AB y BA?
b)¿Qué orden tiene(n) la(s) matriz(ces) resultante(s)?

456 Matrices y determinantes

456-457 456 10/11/2001, 18:13


CAPÍTULO 11

c) Obtengamos los productos.


• Soluciones:
a) A tiene orden 1 ¥ 2 y B tiene orden 2 ¥ 2; por lo tanto, se
puede obtener AB pero no BA.
b) AB tiene orden 1 ¥ 2 (número de filas de A y número de
columnas de B).
2 3
c) 1 –3

[–1 2] = [x y]

Notemos que la distribución así dispuesta nos entrega un


modo fácil de efectuar los productos, así:
AB = [–1 • 2 + 2 • 1 –1 • 3 + 2 • –3]
AB = [0 –9]

2. Dada A = 3 –2 y B = 2 –3
–1 5 3 4

Obtengamos AB y BA.

AB = 3 –2 2 –3 BA = 2 –3 3 –2
–1 5 3 4 3 4 –1 5

6 –6 –9–8 6 + 3 – 4 – 15
= =
– 2 + 15 3 + 20 9 – 4 – 6 + 20

0 – 17 9 – 19
AB = 13 23
BA =
5 14

Es claro que AB y BA son diferentes.


–2
3. Sea A = 2 –1 5 y B= 5
3 2 4
1
Obtengamos AB.
2 –1 5 –2 –4 + –5 + 5 –4
• = =
3 2 4 5 – 6 + 10 + 4 8
1

4. A= 2 –2 y B = 1 0 obtener AB y BA.
1 3 0 1

En este caso vemos que B es la matriz identidad y por lo


tanto es neutro multiplicativo, entonces AB = BA = A.

5. Obtengamos la matriz inversa de A = 4 –1


1 2
y comprobemos.
a b
Recordemos que para que exista la matriz inversa de c d
debe ocurrir que ad – bc π 0.

Matrices y determinantes 457

456-457 457 10/11/2001, 18:14


En este caso ad – bc = 8 + 1 = 9 π 0, por lo tanto, A-1 existe
y es: A– 1 = 1 2 1
9 –1 4
Comprobemos:
1 2 1 4 –1
A– 1 • A =
9 –1 4 1 2

1 8+1 –2+2
=
9 –4 +4 1+8

1 9 0
=
9 0 9

= 1 0 que es la matriz identidad.


0 1

Como el producto de matrices no es conmutativo debemos


comprobar que A • A–1 también es la matriz identidad.
4 –1 1 2 1
A • A– 1 = •
1 2 9 –1 4

2 1
4 –1 9 9
=
1 2 –1 4
9 9

8 1 4 4
+ –
= 9 9 9 9 = 1 0
2 2 1 8 0 1
– +
9 9 9 9

6. Determinemos si existe la matriz inversa de:


–3 5
A=
6 – 10

En este caso vemos que la condición para la existencia de la


matriz inversa no se cumple, pues ad – bc = 30 – 30 = 0 y,
por lo tanto, decimos que A no es invertible.
7. Sean A y B dos matrices dadas por:
4 –5 –3 –8
A= y B=
1 2 2 5

Resolvamos la ecuación B • X = A
Sabemos que si B es invertible, la ecuación tiene por solución
X = B–1 A.
B cumple la condición para tener matriz inversa y ésta es:
5 8
B– 1 =
–2 –3
–1
y así, la solución de BX = A se obtiene mediante B B X = B–1 A
{

I • X = B–1 A
X = B–1 A

458 Matrices y determinantes

458-459 458 10/11/2001, 13:55


CAPÍTULO 11

es decir X = 5 8 4 –5
–2 – 3 1 2

X = 20 + 8 – 25 + 16
–8–3 10 – 6
28 – 9
X=
– 11 4

Ejercicios
1. Calcule los siguientes productos: 3. Dadas las siguientes matrices:

2
A = 2a a B=
–a – 2a
a) 1 2 1 –1 – a 3a a 2a
0
2 –2 2 5 1
b) y 0 –a
3 4 1 –2 –1 C=
a 0
1 –1
c) 2 –1
1 4 calcule:
–1 2
3 –2
a 2a a) AB – AC b) A(B – C) c) 2A – 3AB
a b –a –b ab 2b
d) b2 – a
d) – 3 ABC e) (A – B)C f) (B – C) A
–a –b

2 3 2 3 6 g) 3 (A + B – C) h) (2A + B) C
e)
– 3 – 2 3 – 6 8
1 –a 4. Determine el valor de las variables
f) –2 – 2a a
3 – 3a
3a en cada caso:
a–b b–a
a+b a–b 2a –3 2
g) a+b –a+b 2b
a+b b+a
a) x y = – 9 13
–a b 5 3
9 4 4 8
h) 9 3 – 9 – 18

2. Encuentre la matriz inversa de las b) 2a – 3b 1 –2 = 4 8


siguientes matrices. Compruebe que 3 4

son inversas y en caso de no existir la


inversa explique por qué.
2 –5 x 9
a) A= 2 –2 b) A = –3 4 c) y
=
1 3 5 1 3 7 – 15

1
1
c) T = 2 d) M = 8 12
2a b
1 2 3 4 1
= 6 –3
2 d) x 3y –1 3 4 9
2
e) P = x 2x f) Q = 0.5 0.6
– x 3x 0.1 1
–3 4 x –y
2 – 3 e) = 1 –2
g) M = b – b h) N= 2 –1 2z u 0 1
3b 2b 3 2

Matrices y determinantes 459

458-459 459 10/11/2001, 13:57


Ejercicios
1 16 – 22
d) • • X= 1 0
3 2 2 4 –12 0 –1
f) x 2y –1 –1 3 = 4 –3
–2 4 1 2 2 = 1 1
e) •X•
3 5 1 1
a1 a2 a3 a4 4
–5
4 b1 b2 b3 b4 –3 6 5 12
g) –2 5 –3 6 = c c c c f) 2•
5 2
•3 X=
–2 1 2 3 4 –9 4
–1 d1 d2 d3 d4
8. Demuestre que en el conjunto
5. Sea A una matriz de orden 2 ¥ 3 de matrices cuadradas de orden 2
Sea B una matriz de orden 2 ¥ 4 el producto no es conmutativo.
Sea C una matriz de orden 3 ¥ 4 9. Encuentre dos matrices cuadradas
Sea D una matriz de orden 4 ¥ 3 de orden 2, A y B y ejemplifique
Sea E una matriz de orden 4 ¥ 2 la siguiente proposición:
Indique todos los productos que
A–1 • B • Aπ B
se puedan efectuar y dé un
ejemplo de cada uno de ellos. 10. Encuentre un par de matrices cua-
Indique el orden o dimensión de dradas de orden 2 cuyo producto
cada producto. sea conmutativo.
6. Encuentre un par de matrices Nota: No considere la matriz inversa
no nulas cuyo producto sea nulo. ni la matriz nula.
Haga el caso en que ambas son 11. Sea A = 1 a
0 1
matrices cuadradas, es decir, de
orden n • n y el caso en que Calcule A2, A3, A4, A5 ...
ambas no son cuadradas. ¿Puede hacer alguna generalización?
7. Resuelva las siguientes ecuacio-
Nota: A2= A • A ; A3= A2 • A ;
nes matriciales multiplicativas:
A4 = A3 • A
2 –3 2 2 1 –3
a) 1 6
• X=
–1 –6 12. Sea A = 2 3 yB=
1 4 2 5

–3 4 3 –4
b) X•
0 5
= Calcule A–1 , B–1 , AB y (AB)–1
5 6
Verifique que: (AB)–1 = B–1 • A–1
6 11 4 8
c) X • = (–2) ¿Ocurre siempre esto? Demuéstrelo.
9 –4 –6 12

Soluciones
–2 b2 2 ab
g) 4 b2 – 2a2
h) 0 0
–2 ab 9 18
1. a) 0 b) 2 14 4
10 7 –1 2. a) A–1 = 1 3 2
8 –1 2
2 2
c) 3 6 d) [a2 + ab 3a2 + 3b2] 1 1 –4
5 8 b) A–1 = –
23 –5 –3
5 6 – 3 2 2 3 + 2 6
e) 2 –
1
–2 6 –3 + 2 3 –3 2 – 4 4
c) T–1 = 2
7 1
a – 3a2 – 1
2
f) –2a – 6a2
3a – 9a2 d) M no es invertible. (Ver 11.5.1)

460 Matrices y determinantes

460-461 460 10/11/2001, 14:00


CAPÍTULO 11

1 3x –2x 5. Se pueden efectuar los siguientes


e) P–1 =
5x2 x x productos de 2 matrices:
–3
1 A•C de orden 2¥4
f) Q–1 = 25 5
11 –1 1 B•D de orden 2¥3
10 2
B•E de orden 2¥2
g) M–1 = 1 2 2b b
5b –3b b C•D de orden 3¥3
1 2 3
h) N–1 = C•E de orden 3¥2
5 – 3 2
E•B de orden 4¥4
–2a 2 0 –2a2 0
3. a) a2 7a 2
b) a2 7a2 E•A de orden 4¥3

4a + 3a2 2a + 6a2
D• C de orden 4¥4
c)
– 2a – 12a2 6a – 24a2
Algunos productos de 3 matrices son:
6a3 – 3a3
d)
–24a3 12a3 A C D de orden 2¥3
3a 2 –3a 2 – a2 – 4a 2 A C E de orden 2¥2
e) a2 2a 2
f) –2a 2 6a 2
– – –
3a 0 0 –3a2
g) h) 8a2 a2 2 1 5 –3
–3a 15a 6. A = y B=
–2 –1 – 10 6

4. a) x = 92 y= 21
Es claro que A π 0 y B π 0 y AB = 0
19 19
–2 –8
b) a= b=
5 15 –1 15 0
–12 –57 7. a) X = 1 9 –6 b) X =
c) x= y= 15 – 4 – 14 15 25 – 38
29 29
–18
d) a = 15 b=
26 13
2 – 88 4 1 –6 –11
21 32
c) X = d) X = –
x= y= 123 – 84 138 26 –2 –8
13 39
1
x= 1 1 –32 0
e) z= e) X = 2 0 f) –1 36
5 5 2 0 108
–2 –2
y= u=
5 5
11. A2 = 01 12 a A3 = 01 13 a A4 = 1 4 a
–1 0 1
f) x= 7 y=
11 22
Generalizando: An = 1 n a
g) a1 = 10 a2 = – 25 0 1
a3 = 15 a4 = – 30
1 4 –3 1 5 3
b1 = –8 b2 = 20 12. A– 1 = B– 1 =
5 –1 2 11 –2 1
b3 = – 12 b4 = 24
c1 = 4 c2 = – 10 AB = 8 9 (AB)–1 = 1 17 – 9
9 17 55 –9 8
c3 = 6 c4 = – 12
d1 = 2 d2 = –5 1 17 – 9
B–1 A–1 =
d3 = 3 d4 = –6 55 –9 8

Matrices y determinantes 461

460-461 461 10/11/2001, 14:02


11.5 Determinantes y sistemas
de ecuaciones

11.5.1 Determinantes y
sistemas lineales de orden 2
Dado el siguiente sistema de 2 ecuaciones:
ax + by = p
cx + dy = q

Las soluciones para x e y están dadas por:


pd – bq aq – pc
x = e y =
ad – bc ad – bc

La solución del sistema existirá sólo si el denominador común


x e y es distinto de cero. Esto es, ad – bc π 0.
Vemos que el sistema original también puede ser planteado
como ecuación matricial multiplicativa de la siguiente manera:
a b x = p
c d y q

Llamando A a la matriz de los coeficientes, X a la matriz de


las variables y B a la matriz de los términos constantes, tenemos la
ecuación A • X = B que tiene por solución X = A– 1 B, siempre que
A– 1 exista, es decir, siempre que A sea invertible y esta condición
esté dada por: ad – bc π 0.

La expresión ad – bc se llama DETERMINANTE de orden 2


asociado a la matriz a b y se denota por det. A = a b
c d c d
Vemos que las soluciones encontradas para x e y a partir
del sistema pueden ser expresadas en forma de determinantes
de la siguiente manera:
p b a p
q d c q
x = y =
a b a b
c d c d

Así, si llamamos Δ al determinante formado por los coeficientes


del sistema, Δx al determinante obtenido cambiando la columna
correspondiente a la variable x por la columna de términos
constantes y, Δy al determinante obtenido al cambiar la columna de
los coeficientes de y por la de términos constantes, podemos escribir
la solución del sistema de la siguiente manera:

Δx Δy
x = e y =
Δ Δ

462 Matrices y determinantes

462-463 462 10/11/2001, 14:03


CAPÍTULO 11

11.5.2 Determinantes y
sistemas lineales de orden 3

Consideraciones similares a las hechas en el punto anterior nos


llevan a definir un determinante de orden 3 a partir del sistema
de 3 ecuaciones lineales:
a1x + b1y + c1z = d1
a2x + b2y + c2z = d2
a3x + b3y + c3z = d3
como el valor común de los denominadores de x, y y z una
vez despejados éstos.

así a1 b1 c1
a2 b2 c2 = a1 b 2 c3 + a2 b 3 c1 + a3 b 1 c2
a3 b3 c3 – a3 b 2 c1 – a1 b 3 c2 – a2 b 1 c3

Si conmutamos los términos obtenidos y factorizamos obtenemos


que el valor del determinante puede ser expresado:
a1 (b2 c3 – b3 c2) – b1 (a2 c3 – a3 c2) + c1 (a2 b3 – a3 b2),
lo cual es equivalente a reducir el determinante de orden 3 a 3
determinantes de orden 2, es decir:
a1 b1 c1
b2 c2 a2 c2 a b2
a2 b2 c2 = a1 – b1 + c1 2
b3 c3 a3 c3 a3 b3
a3 b3 c3

Este proceso se llama DESARROLLO del Determinante por


MENORES.
Un proceso más simple para el cálculo de un determinante de
orden 3 es la llamada Regla de Sarrus, que consiste en “agregar”
al determinante las 2 primeras filas (en el mismo orden) o las dos
primeras columnas y luego formar todas las diagonales completas,
es decir, con 3 elementos. Los productos de las diagonales en
este sentido se suman y los productos de las diagonales en
el otro sentido se restan.
Ahora, para resolver un sistema de 3 ecuaciones lineales:
a1x + b1y + c1z = d1
a2x + b2y + c2z = d2
a3x + b3y + c3z = d3
formamos los siguientes determinantes:
Δ formado por los coeficientes de las variables.
Δx obtenido de Δ reemplazando la columna de los coeficientes
de x por la columna de los términos libres.
Δy reemplazando en Δ los coeficientes de y por los términos libres.
Δz reemplazando en Δ los coeficientes de z por los términos libres.
a1 b1 c1
Así: Δ = a2 b2 c2
a3 b2 c3

Matrices y determinantes 463

462-463 463 10/11/2001, 14:03


d1 b 1 c1 a1 d1 c1 a1 b 1 d1
Δx = d2 b 2 c2 Δy = a2 d2 c2 Δz = a2 b 2 d2
d3 b 3 c3 a3 d3 c3 a3 b 3 d3

y la solución del sistema está dada por:


Δy Δz
x = Δx y = z =
Δ Δ Δ
Este método para obtener soluciones es conocido como REGLA
DE CRAMER.
Observación: Claramente el sistema tiene solución si el valor
de Δ es distinto de cero.

Ejercicios 1. Calculemos el valor del determinante:


resueltos 2 –2
1 3

De inmediato: 2 • 3 – 1 • –2 = 6 + 2 = 8
2. Apliquemos determinantes para resolver el sistema:
3x – y =10
x + 2y = 8

Formemos los determinantes Δ, Δx y Δy:

Δ = 3 –1 = 6 – (–1) = 7 Como Δ es distinto de 0


1 2
el sistema tiene solución y
calculamos Δx y Δy
Δx = 10 – 1 = 20 – (–8) = 28
8 2

Δy = 3 10 = 24 – 10 = 14
1 8
y la solución del sistema es:

x = Δx = 28 = 4
Δ 7
Δy
y = = 14 = 2
Δ 7
3. Apliquemos determinantes para resolver el sistema:
2x – 7y = 3
– 4x + 14y = 1
Formemos y calculemos el determinante principal Δ
2 –7
Δ = – 4 14
= 28 – 28 = 0

Como Δ = 0 el sistema no tiene solución. (En este caso se trata


de dos rectas paralelas.)

464 Matrices y determinantes

464-465 464 10/11/2001, 14:05


CAPÍTULO 11

4. Calculemos el valor del determinante:


1 2 2
3 3 4
2 –1 5

Aplicando la regla de Sarrus, obtenemos:


1 2 2 1 2
3 3 4 3 3
2 –1 5 2 –1

= (15 + 16 – 6) – (12 – 4 + 30)


= 25 – 38
= – 13
5. Resolvamos el sistema de ecuaciones:
2x + y – z = 6
3x – 2y + 3z = 3
x – y + z = 0

Formemos y calculemos el determinante principal:


2 1 –1 2 1
Δ = 3 –2 3 3 –2 = (– 4 + 3 + 3) – (2 – 6 + 3) =
1 –1 1 1 –1 2+1=3
Como Δ π 0 el sistema tiene solución y calculamos:
6 1 –1 6 1
Δx = 3 –2 3 3 –2 = (– 12 + 3) – (– 18 + 3) =
0 –1 1 0 –1 – 9 + 15 = 6
2 6 –1 2 6
Δy = 3 3 3 3 3 = (6 + 18) – (– 3 + 18) =
1 0 1 1 0 24 – 15 = 9
2 1 6 2 1
Δz = 3 –2 3 3 –2 = (3 – 18) – (– 12 – 6) =
1 1 0 1 –1 – 15 + 18 = 3
y las soluciones son:

Δx
x = Qx = 2
Δ
Δy
y = Qy = 3
Δ
Δz
z = Qz = 1
Δ
6. Resolvamos el siguiente sistema aplicando determinantes.
(Regla de Cramer).
3x – 2y + 4z = 9
– 2x + 3y – z = 2
x + y + 3z = –1

Matrices y determinantes 465

464-465 465 10/11/2001, 14:06


Formemos y calculemos el determinante principal.

El valor del determinante principal es cero, por lo tanto, el


sistema no tiene solución.

Ejercicios
1. Calcule el valor de los siguientes c) –2 –6
d) 5 –2
determinantes: 1 3 10 –2

2 –3 3 2 h2 –2h 1 a
a) b) e) f)
1 2 1 –2 h –2 0 1
3 4 –2 3
c) d) –1 5 g) 1 a h) 2 0
5 2
a 1 1 2
a 1 2 b
e) 1 –a f) –b 3 4. Determine en cada caso el valor de la
incógnita para que los determinantes
g) h 1 h) 2 1 tengan el mismo valor:
2 h 3 2
–1 3 2 –2
1 a) y
0.2 5 x 1 3
2 –2 0.5
i) 1
j)
0.1 –3 0.3 y 2y 4 –1
4
b) y
3 –5 3 3

k) 0 –1
l) 1 3
5 3 2 x 3 – 2
2 2 c) y
3 1 1 2
2. Para cada uno de los siguientes
determinantes encuentre otro que 3 –3 h+1 2
d) y
3 3
h 2h
tenga el mismo valor:
2x x x–1 3x + 2
a) 2 3 b) –1 –3 e) 3 –2
y
1 5
1 4 5 6
5. Resuelva los siguientes sistemas de
5 –5 3 2
c) d) ecuaciones aplicando determinantes:
2 2 5 6
a –a a a2
e) f) a) x + y = 6 b) x – 5y = – 2
b b –a 2a
2x + y = 9 3x + 2y = 6
3. Dadas las siguientes matrices aplique
determinantes para establecer si ellas
son invertibles o no. c) 3x – 5y = 11 d) – 5x + y = 2
–3 1 2x + 4y = – 1 x – 4y = 3
a) 2 –1
b) 1 4
1 5

466 Matrices y determinantes

466-467 466 10/11/2001, 14:08


CAPÍTULO 11

1 1 g) 2x – y = –2 h) y – z=– 3
e) 2x – 3y = 6 f) x+y=
2 3
1 x + 2z = 1 2x + 3z = 1
x – 3y = 12 x– y=4
2 2y – 3z = 2 x – 2z = 4
g) 4x – 3y = 5 h) – 9x + y = – 1 8. Resuelva los siguientes sistemas
3 1
2x – y = –2 x– y=2 de ecuaciones aplicando deter-
2 10
minantes (Regla de Cramer).
i) –0.1x – 1 y=4
3 a) 2x – y – z = 0 b) x + 3y – z = –3
3x + 10y = –3 x + 3y + z = 5 2x – y + 4z = 14
3x – y + 2z = 4 x – 5y + z = 15
6. Calcule el valor de los siguientes
determinantes:
c) x + 3y = 9 d) 3x – 2y – 3z = 12
2 3 2 –1 2 1
a) 1 3 1 b) 3 1 0 2x – z = 8 x+ y+ z=3
2 1 3 2 1 –2
y + 4z = – 6 – 2x – y – 2z = –3
–2 1 5 2 –1 2
c) 1 2 3 d) 0 3 –1
–3 1 2 3 1 –2 e) – 4x + y + z = – 3 f) x + y – z = 2
0 3 –1 2 2 2
4x – y + z = 3 x – y + z = –2
e) 2 1 2 f) 1 1 –1
0 1 –2 3 3 –3 2x + y + 4z = 3 x+y+z=0
7. Escriba la ecuación matricial corres-
pondiente a cada uno de los siguientes g) x – 3y – 2z = 5
sistemas: 2x – y + 3z = 27
– 3x + 2y – z = – 22
a) 3x – y = 2 b) x + y = 7
9. Demuestre:
x + 4y = –3 2x – 6y = – 2
b+c a–b a
a) det c+a b–c b = 3 abc – a2 + b2 + c2
c) 2x + 3y = a d) – 5x + 2y = – 1 a+b c–a c

x – 6y = b 2x – 3y = 2 1 a 1+a
b)
det a 1 1 =a a–1 a+1
e) x – 2y – 3z = –3 f) 2x – y + z = – 4 1 a 1
2x + y + 4z = –1 x+y–z=2 a–b–c 2a 2a
c) det 2b b–c–a 2b = a+b+c 3
x – 3y – z = 4 2x – y – z = –3
2c 2c c–a–b

Soluciones

1. a) 7 b) – 8 c) – 14 d) – 7 e) No f) Sí g) Sí h) Sí
e) – a2 – 1 f) 6 + b2 g) h h – 2 –15
4. a) x = –23 b) y = c) x = – 2
11
h) 2 2 – 3 i) 0 j) 0.9
d) h = – 1 e) x = 7
k) 2 l) –4 3 2 9

3. a) Sí b) Sí c) No d) Sí 5. a) x = 3 y=3 b) x = 26 y=
12
17 17

Matrices y determinantes 467

466-467 467 10/11/2001, 14:09


39 –25 –11 –17 2 –1 1 x –4
c) x = y= d) x = y= f) 1 1 –1 y = 2
22 22 19 19 z
2 –1 –1 –3
–4
e) x = – 6 y = – 6 f) x = 10 y=
3 3 2 –1 0 x –2
g) No tiene solución h) x = 19 y = 170 g) 1 0 2 y = 1
0 2 –3 z 2
i) No tiene solución
6. a) 3 b) 15 c) 22 d) – 25 e) 10 f) 0 0 1 –1 x –3
h) 2 0 3 y = 1
1 0 –2 z 4
3 –1 x 2 1 1 x
7. a) y
= b) y
= 7
1 4 –3 2 –6 –2 8. a) x = 1 y= 1 z= 1
2 3 x
= a
b) x = 4 y=–2 z= 1
c) y b
1 –6
c) x = 3 y= 2 z =– 2
–5 2 x –1 d) x = 3 y= 3 z =– 3
d) y
=
2 –3 2
e) x = 1 y= 1 z= 0
1 –2 –3 x –3 f) x = 0 y= 1 z =– 1
e) 2 1 4 y = –1
1 –3 –1 z 4 g) x = 2 y=–5 z= 6

Prueba de selección múltiple


1. Sea A = (aij) 2 • 3 una A. –6 y 7 4. De las siguientes propo-
matriz tal que aij = i – j B. –6 y –4 siciones para A y B
entonces A es: C. –4 y –2 A y B E Ma • a:
I A•B=B•A
1 0 –1 D. –4 y 4
A. II Aπ0 y Bπ0
0 –1 –2 E. 6 y 8
entonces AB π 0
B. 0 –1 –2 3. Sea III A π 0 entonces
1 0 –1
existe A–1
0 +1 +2 1 –3
C. A. y Son falsas:
–1 0 1 3 –1
A. Sólo I
0 1 B. –2 1
D. –1 –2 1 1 B. Sólo II
–2 –1
A–B= C. II y III
0 –1 D. TODAS
E. 1 0 A.
–1 –4
E. NINGUNA
2 1 2 0
1 –3
2. ¿Qué valor deben tener x 3 –4 5.
B. Sea A = 5 –6
e y para que las matrices 2 0 2 –1
A y B sean iguales? 3 –4 –3 –3
C. y B = 0 –2
2 –2
A= 3 –5 1 4
2 x+2 –1 4 La matriz X que satisface
D.
2 –2
3 x+1
B – X = A es:
B= E. –3 –4 4 0
2 y 2 –2 A. 5 –4
1 –5
468 Matrices y determinantes

468-469 468 10/11/2001, 14:12


CAPITULO 11
CAPÍTULO

–2 0 A. k=0 B. I y III
B. 5 –4
C. II y III
1 –5 B. kπ0
C. kπ9 D. Todas
–4 0
C. –5 4 E. Ninguna
–1 5
D. k=9
E. A siempre es 12. La ecuación matricial
–2 –6
D. 5 –8 invertible 2 –1 0 x 2
1 3 1 0 –1 y = –3
z
–2 0 9. Si A = 1 x 0 –2 3 4
E. 5 –8 0 1
representa el sistema:
1 –5 entonces An =
2 –1 3
A. 2x + y = 2
6. Sea A =
– x – 2y = – 3
1 2 –2
1 nx
–1 2 –1 A. 0 1 y – 3z = 4
y B =
0 1 2
B. 1 xn B. 2x – y = 2
La matriz C que satisface 0 1 x–z= –3
C–A–B= 0 0 0 C. n nx – 2y + 3z = 4
0 0 0 0 1
1 1 4 C. 2x – y = 2
A. D. 1 nx
1 3 0 0 n x–y= –3
B. 1 1 2 E. Depende de n – 2x + 3y = 4
1 3 0
3 –3 2 10. Qué valor deben tener D. 2x + y = 2
C. 1 1 –4 – x – 2z = – 3
las variables a y b para
–3 3 –2 que se cumpla: – y + 3z = 4
D.
–1 –1 4 A = B siendo E. Ninguno de los
E. –1 –1 –4 1 –a –b anteriores.
–1 –3 0
A =
0 –2 –5
y
7. El sistema 3x – y = 2 1 a+2 b– 3
13. La matriz inversa de
B =
– 2x + 3y = – 1 0 –2 –5 A = 6 5 es
5 4
expresado en
A. a=–1 b= 3
forma matricial es: 2 A. A–1 = 4 –5
3 –5 6
A. 3 –1 2 = x B. a=–2 b=
y 2
–2 3 –1 B. A–1 = –4 5
C. a= 2 b= 3 5 –6
3 –1 2
B. xy =
D. a=–2 b= 3 –6 –5
–2 3 –1 C. A–1 =
E. a= 2 b = –3 –5 4
3 –1 x = 2
C. y
–2 3 –1
11. Dada la matriz D. A–1 = 1 –4 5
29 5 –6
D. 3 –2 x = 2 –1 3 4 2
–1 3 y –1 A= 2 –3 1 –5 E. A–1 = 1 4 –5
3 –3 6 –1 29 –5 6
3 –1
E. x y = 2 –1
14. La única proposición
–2 3 Se puede afirmar:
8. Para que la matriz I a11 = a43 verdadera es:
A =
–k–1 2k A. El conjunto de matri-
–5 9 II a22 = – 2 a33
ces con la multiplica-
no sea invertible III a13 – a31 = a23 ción forma un grupo
se debe cumplir: A. Sólo III abeliano.

Matrices y determinantes 469

468-469 469 10/11/2001, 14:13


Prueba de selección múltiple
2
B. El neutro multipli- A. 3
21. Determine los valores
cativo en Ma • a es 2 de a y b para que se
B. –
3 cumpla la igualdad
I = 1 1 3
1 1 C. 2 A – 3 B = C, siendo:
2
C. Si el producto de dos D. –3 α 2 –2 β
A= 1 –3 B=
matrices es 0, enton- 2 2 –5
ces al menos una de E. 0 –6 –5
y C= –4 9
ellas debe ser cero. 18. La expresión
D. Todas las matrices 1 a b A. a = 4 b= 3
b a
es igual a:
tienen inverso adi- a–b B. a =– 4 b= 3
tivo. A. a – b C. a =– 6 b= 3
E. Si M es de orden B. a + b D. a = 4 b=–3
3 • 4 y N es de orden 1 E. a = 6 b=–3
3 • 5, entonces MN C. a–b
es de orden 4 • 5. 1 22. El valor de
D. 1 m
a+b m
15. La condición que E. a2 – b2 det 0 m –1 –m es:
0 0 m+1
debe cumplir k para
que el sistema 2 3 A. 0
–3 1 B. m3
2kx+ y = 9 19. Calcule: =
–3 1
– 3x+ 2y = 5 2 3
C. m3 – 1
tenga solución es: A. 1 D. m3 – m
A. k = 3 E. m2 – 1
4 B. –1
B. k π – 3 23. La matriz principal (o
C. 11
4 matriz de los coeficien-
3 D. –11 tes) del sistema
C. k π
4
– x + y + z = 1 es:
D. k π 4 E. Otro
3 x – y – z = 2
E. k = – 4 20. El valor de m que x + y – z = –1
3 hace ver dadera la –1 1 1
2 –3 5 igualdad en: A. 1 – 1 +1
16. Sea A = 1 –2 1
,
m+2 1 –1 –1
entonces det (A) = [m – 2 m + 2 2m] m – 2 –1 1 1
A. 10 –m B. 1 –1 – 1
= [– 8 m] es: –1 1 1
B. –10 –1 1 1
A. 1
C. 0 C. 1 –1 – 1
B. –1 1 1 –1
D. Otro valor
C. 8 1 1 1
E. No existe D. 2 –1 – 1
D. – 8 –1 1 –1
17. El valor de m en : E. 0 –1 1 1
–m m+1 E. 1 2 –1
–2 5
= 0 es:
1 –1 –1

470 Matrices y determinantes

470-472 470 20/11/02, 6:41 PM


CAPÍTULO
CAPÍTULO 11

24. Si A = –1 –2 N una matriz de orden de 30. En la operación


1 –2
5 ¥ 3. Entonces el producto –2 1 –1 3 a a
1 –2 + = 11 12
B= , N ¥ M es de orden: 0 3 2 –1 a21 a22
–2 2
entonces A. 3 ¥ 3 el valor de a21 – a11 es:
2 (A + B) – 2 (A – B) = B. 4 ¥ 5 A. – 5

2 –4 C. 5 ¥ 4 B. 5
A. 4
•2
–4
D. 20 C. – 3
–2 –4
B. D. 3
2 –4 E. No se puede efectuar
–4 0 1 E. – 1
C. •
–6 0 2 28.Sea A una matriz
invertible y sea B = A–1 31. El valor de 2a en
D. 4 0 se puede afirmar:
6 4 –3 2a
4 5
= – 23 es:
–2 0 • –2 I. A = B–1
E. 3 –4 A. 1
II. Todos los eleme-
tos de A son B. 2
25. Sea A = 10 00
distintos de cero C. –1
0 –1
B= 0 0 III. det (A) π 0
D. –2
0 0 0 0 A. Sólo I
C= –1 0
D= 0 –1
, E. Otro
entonces la matriz B. I y II
–2 3 32. El valor de
M= se puede C. Sólo III
–5 4 2 –1 3 –1
• =
expresar por: D. I y III 1 3 2 1

A. – 2A + 3B + 5C – 4D E. Todos A. –1
B. – 35
B. – 2A – 3B + 5C – 4D
29. Los valores de las
C. – 25
C. 2A + 3B + 5C – 4D variables x, y, z para
que la igualdad se D. 35
D. 2A – 3B + 5C + 4D
cumpla son respec- E. 25
E. – 2A + 3B + 5C + 4D tivamente:
33. Sea A = (aij) dada
26. 1 m – m 1 = [3x – z y+2 x + y] =
1 –m 1 –m –3 2a – 2b
[8 –2y + 8 – 2x] por: A = 2b – a – 3b
A. (m + 1)2 –a b ab
A. – 2 2 – 10
B. (m – 1)2 3
Si x = a21 – a23 + 2a13 ,
8 entonces x =
C. m2 + 2m B. –2 8
3 A. 2a – b
D. – 2m – m2 –1 C. 2
–2 10
3 B. – b
E. – (m – 1)2
D. No se pueden deter- C. b
27. Sea M una matriz minar D. – 2a + b
de orden 3 ¥ 4 y E. Otros E. Otro

Matrices y determinantes 471

470-472 471 20/11/02, 6:42 PM


Prueba de selección múltiple
34. El valor de k en 38. Sea
3 – 2 = 2 es:
k+2 k–1 M= 0 2
N= a b
–2 0 0 1
A. –1 B. 1 3 –4
para que N • M =
33 1 –2 0
C. D.
5 5 los valores de a y b son
E. –5 respectivamente:
35. El valor de k para que la matriz A A. – 2 ; 3 B. 2 ; – 3
2 2
no sea invertible debe ser: 3 3
–3 0 0
C. – 2 ; – D. 2 ;
2 2
–1 3k 0 E. Otros
A=– 1
3 2k 1
2k
3 39. De las siguientes proposiciones,
A. 0 B. 1 si A, B y C son matrices de
C. –1 D. 3 orden n ¥ n, A y B invertibles, es
E. –3 verdadero:
A. (A + B) • C = C • (A + B)
36. A partir de la siguiente igualdad:
B. A + (B • C) = (A + B) •
u 2v 2 –2 3 –3
x 2y
= (A + C)
1 3 1 2
es falso: C. (A • B)– 1 = B– 1 • A– 1
A. u=4 B. v = 5 D. (A + B) (A – B) = A2 + B2
C. x=6 D. 2y = 3 E. A • B = B • A
E. 2u + v = 3
40. 5 3 –2 1 3 –2 1
37. El resultado de: + •
2 –1 6 2 –1 6 3
1 2 –1 –2 1 2
– •
–1 3 2 –2 2 –1 es –2
1
A.
10 0
B. 0 0 A. 3 B. 1 0
7 – 11 –1 0 1
3 0 2
3
C. – 6 – 2 D. 6 2
9 –6
1 3 1 –3 C. – 3 18
D. 1 1
1 1
–4 2
E. 3 –2
–3 9 E.
–1 6

Soluciones

1. B 6. B 11. A 16. E 21. C 26. B 31. B 36. B


2. B 7. C 12. B 17. A 22. D 27. C 32. D 37. D
3. C 8. D 13. B 18. B 23. C 28. D 33. C 38. C
4. D 9. A 14. D 19. B 24. A 29. A 34. D 39. C
5. C 10. A 15. B 20. A 25. B 30. B 35. A 40. E

472 Matrices y determinantes

470-472 472 20/11/02, 6:44 PM


CAPÍTULO 12
S umatoria
y progresiones

Sumatoria 12.1

Definición: Sea µ1, μ2, μ3, μ4..., μn E N los términos de una sucesión.

Definimos el símbolo μi de la siguiente forma:


1

i=1
Σ μi = μ1
n + 1 n
Σ
i=1
μi = Σ μi
i=1
+ μn +1
n
El símbolo Σ μi se lee: sumatoria de los μi desde μ1 hasta μn.
i=1

• Ejemplo:
5 4 3
Σ μi = iΣ
i=1 =1
μi + μ5 = Σ μi + μ4 + μ5
i=1
2
= Σ μi + μ3 + μ4 + μ5
i=1
1
= Σ μi + μ2 + μ3 + μ4 + μ5
i=1

= μ1 + μ2 + μ3 + μ4 + μ5

• Propiedades:
n n n
1. Σ (μi + vi) = iΣ
i=1 =1
μi + Σ vi
i=1
n
2. Σ k=n•k
i=1
k = constante

Sumatoria y progresiones 473

473 473 10/11/2001, 11:52


n n
3. Σ k μi = k iΣ= 1 μi
i=1
k = constante

n
4. Σ (μi – μi – 1) = μn – μo (propiedad telescópica)
i=1

5. Algunas sumas importantes y de uso frecuente:


n
a) Σ i = 1 + 2 + 3 + ... + n =
i=1
n(n +1)
2
n
n n + 1 2n + 1
b) Σ i2 = 1 + 4 + 9 + ... + n2 =
i=1 6
n 2

Σ i3 = 1 + 8 + 27 + ... + n3 =
n n + 1
c)
i=1 2

Observación:
Sean m y n números naturales tales que m £ n, entonces:
n n m – 1

Σ
i=m
μi = Σ
i=1
μi –
i=1
Σ μi

Ejercicios 1. Desarrollar las sumatorias:


n ∞
resueltos a) Σ (– 1)i – 1 (i2 + 1) b) Σ
k – 1
i=1 k=1 k + 1

Solución:
a) 2 – 5 + 10 – 17 + 26 – ... + (– 1)n – 1 (n2 + 1)
1 2 3 4
b) 0 + + + + +...
3 4 5 6
12
2. Calcular el valor de Σ (i – 1) (i + 1)
i=1

Solución:
(i – 1) (i + 1) = i2 – 1
12 12 12 12
Σ (i – 1) (i + 1) = iΣ= 1 (i2 – 1) = iΣ= 1 i2 – iΣ= 1 1
i=1

12 12 + 1 2 • 12 + 1
= – 1 • 12
6

= 12 13 25
• •
– 12 = 638
6
Se aplicó la fórmula
n
n n + 1 2n + 1
Σ i2 = 6
i=1

474 Sumatoria y progresiones

474-475 474 10/11/2001, 18:33


CAPÍTULO 12

3. Calcular la suma de los n primeros términos de:


1 • 6 + 2 • 7 + 3 • 8 + 4 • 9 +...
Solución:
Observando los términos de la suma nos damos cuenta que
el término general es ai = i (i + 5); por lo tanto, la suma se
expresa como:
n n
Σi
i=1
i+5 = Σ
i=1
i2 + 5 i

n n
= Σ i2 + 5 iΣ
i=1 =1
i

= n n+1 2n + 1 n n+1
+5
6 2
3 2
= n +9n +8n
3
12
4. Calcular el valor de:
k=5
Σ k+1 2k–3

Solución:
n n m– 1
Sabemos que: Σ μk = kΣ= 1 μk – kΣ= 1 μk
k=m

y (k + 1) (2 k – 3) = 2 k2 – k – 3, luego
12 12 4
Σ
k=5
k+1 2k–3 = Σ
k=1
2 k2 – k – 3 – Σ
k=1
2 k2 – k – 3

12 12 12 4 4 4
=2 Σ k2 – kΣ= 1 k – kΣ= 1 3 – 2 kΣ= 1 k2 + k=Σ1 k + kΣ= 1 3
k=1

12 • 13 • 25 12 • 13 4•5•9 4•5
= 2• – – 3 • 12 – 2 • + +3•4 =
6 2 6 2
= 1.300 – 78 – 36 – 60 + 10 + 12 = 1.148
12
\Σ k + 1 2 k – 3 = 1.148
k=5

n
n2 + 3 n
5. Si Σ μi =
i=1 2
, hallar μi

Solución:
n n–1
μn = Σ μi – iΣ
i=1 =1
μi

2
n2 + 3 n n–1 +3 n–1
= –
2 2
n2 + 3 n – n2 + 2 n – 1 – 3 n + 3 2n+2
= = =n+1
2 2
Como μn = n + 1, entonces μi = i + 1

Sumatoria y progresiones 475

474-475 475 10/11/2001, 18:33


n
6. Σ (ai – ai–1 ) = (a1 – a0) + (a2 – a1) + (a3 – a2) + .... + (an – an–1)
i=1

Eliminando el paréntesis y asociando en forma adecuada


obtenemos:

= an – a0

n n n
O bien Σ ai – ai – 1 = iΣ
i=1 =1
ai – Σ ai – 1
i=1

= (a1 + a2 + a3 + ... + an–1 + an) – (a0 + a1 + a2 + ... + an–1)

Se cancelan todos los elementos, excepto el último del


primer paréntesis y el primero del segundo, y nos queda:

n
Σ ai – ai – 1 = an – a0
i=1

7. Calcular Σ k2 – (k – 1)2
k=1

Al desarrollar nos queda: (1 – 0 ) + (22 – 1) + (32 – 22)


+ ... + (n2 – (n – 1)2)

Aplicando la propiedad telescópica, donde:


n
Σ
i=1
ai – ai – 1 = an – a0 nos queda inmediatamente:

Σ k2 – (k – 1)2 = n2 – 02
k=1

Comprobemos aplicando las propiedades anteriores.

n n

Σ k2 – (k – 1)2 = Σ k2 – (k2– 2k + 1) =
k=1 k=1

n n n

Σ 2k – 1 = 2 Σ k– Σ 1
k=1 k=1 k=1

2n (n + 1)
= –n
2

476 Sumatoria y progresiones

476-480 476 25/11/02, 12:32 PM


CAPÍTULO 12

= n2 + n – n

= n2

Obviamente el uso de la propiedad telescópica es mucho más


directo en este caso para calcular esa sumatoria.

n
1
8. Calculemos Σ k(k + 1)
k=1

Aplicaremos (y explicaremos) el método de fracciones parciales


para solucionar problemas de este tipo.

El método consiste en expresar la fracción original como suma


de 2 fracciones, como sigue:

1 A B
= +
k(k + 1) k k+1

Debemos determinar A y B.

1 A(k + 1) + Bk k(A + B) + A
= =
k(k + 1) k(k + 1) k(k + 1)

Debemos igualar los numeradores.

A=1
A+B=0 } fi B = –1

1 1 1
Entonces: = –
k(k + 1) k k+1

Y así la sumatoria pedida se transforma en:

Σ
k=1
( 1
k

1
k+1
= 1–
1
2
+
) ( ) (
1
2

1
3
+
)
( 1
3

1
4 )
+ ... +
1
n

1
n+1

Sumatoria y progresiones 477

476-480 477 25/11/02, 12:32 PM


Cada término de la sumatoria es la diferencia de dos elementos
consecutivos y vemos que al desarrollarla se cancelan todos, excepto
el primero y el último, por la propiedad telescópica.

Así:

n n
1 1 1 1 n
Σ k(k + 1)
= Σ k

k+1
=1–
n+1
=
n+1
k=1 k=1

9. Usemos el método anterior para calcular.

n
1
Σ (2k – 1)(2k + 1)
k=1

Primero expresamos en fracciones parciales:

1 A B
= +
(2k – 1)(2k + 1) 2k – 1 2k + 1

1 A(2k + 1) + B(2k – 1)
=
(2k – 1)(2k + 1) (2k – 1)(2k + 1)

1 2Ak + A + 2Bk –B
=
(2k – 1)(2k + 1) (2k – 1)(2k + 1)

1 = k(2A + 2B) + ( A – B) (igualando numeradores)

2A + 2B = 0 (Coeficiente de k no existe al lado izquierdo)

A–B=1

A–B=1 1 1
A+B=0 2A = 1 fi A = y B=
2 2

478 Sumatoria y progresiones

476-480 478 25/11/02, 12:33 PM


CAPÍTULO 12

Así, la suma pedida se transforma en:

n 1 1 n
1 1 1 1
Σ Σ
2 2
– = – =
k=1 2k – 1 2k + 1 k=1
2 (2k – 1) 2 (2k + 1)

n
1 1 1
2
Σ
k=1
( –
(2k – 1) (2k + 1) )
Y nuevamente podemos aplicar la propiedad telescópica, ya que
cada término de la sumatoria es la diferencia de dos elementos
consecutivos; por lo que, al desarrollar ésta se cancelan casi
todos los términos. Nos queda

n n
1 1 1 1
Σ
k=1
( (2k – 1)(2k + 1) ) =
2
Σ
k=1
( –
(2k – 1) (2k + 1) )
1 1 1 1
=
2 [( 1–
3 ) (
+
3

5 ) + ...

1 1
+
( 2n – 1

2n + 1 )]
1 1
=
2
1–
(
2n + 1 )
1 2n
=
2 ( 2n + 1 )
n
=
2n + 1

Sumatoria y progresiones 479

476-480 479 25/11/02, 12:34 PM


n

10. Calculemos Σ k3 – (k – 1)3


k=1

Podemos aplicar directamente la propiedad telescópica y


obtenemos:

Σ k3 – (k – 1)3 = n3
k=1

11. A partir de esta igualdad, podemos “deducir” la sumatoria de


los términos al cuadrado. Observa:
n

Σ k3 – (k3 – 3k2 + 3k – 1) = n3
k=1

Σ 3k2 – 3k + 1 = n3
k=1

n n n

3 Σ k2 – 3 Σ k+ Σ 1 = n3
k=1 k=1 k=1

n
n(n + 1)
3 Σ k2 = n3 + 3
2
–n
k=1

2n3 + 3n2 + 3n – 2n
=
2

n
n(2n2 + 3n + 1)
3 Σ k2 =
2
k=1

n
n(n + 1) (2n + 1)
Σ k2 =
6
k=1

Aquí “obtuvimos” la sumatoria de los cuadrados de los


números desde 1 hasta n.

480 Sumatoria y progresiones

476-480 480 25/11/02, 12:35 PM


CAPÍTULO 12

Ejercicios
20
1. Desarrollar las siguientes sumatorias: c) (i2 – i) =
i=1
6 5 15
1
a)
i=1 i
= b)
i=1
(2i – 1) = d) (i – 2i2 + i3) =
i=1
8 4
c) i2 = d) i3 = 5. Calcular el valor de las siguientes
i=1 i=1 sumas:
5 10
1
e) (-1)ii = f) (–1)i – 1 = 30
i=1 i=1 i a) k(k – 3)
n n k = 20

g) 2i = h) 1 = k = 36
i=1 i=1 2i – 1 b) (1 – 2k)
n n k = 12
1
i)
i=1
(-1)ii2 = j)
i=1
(–1)n (1+ =
i ) c)
k=8
(k3 – k2 + 1)
k=5
2. Escribir en forma de sumatoria las k = 10
siguientes series: d) 2k(1 – k)
k=8
2 1
a) 1 + + + 4 6. Calcular el término general y el valor
3 9 27
de los n primeros términos de las
1 2 3 4 5
b) + + + + siguientes sumas:
2 3 4 5 6
a) 2 • 3 + 3 • 4 + 4 • 5 + 5 • 6
1 1 1 1 1 1
c) – + + + + + + 6 • 7 + ...
2 3 4 5 6 7
1 1 1 1 b) 3 + 5 + 7 + 9 + ...
d) + + +
3 6 9 12 c) 1 • 4 + 2 • 5 + 3 • 6 + 4 • 7 + ...
3. Escribir en forma de sumatoria las d) 1 • 2 + 4 • 3 + 9 • 4 + 25 • 5 + 36
siguientes series: • 6 + 49 • 7 + ...
2 3 4 7. Dada la suma, hallar el término
a) 1 + + + + ...
3 5 7 general μi:
4 6 8 10 n
b) 2 +
3
+ + +
5 7 9
+ ... a) Σ μi =
i=1
2n3 + 3n2 – 5n
6
n
4. Calcular las siguientes sumas:
5
b)
i=1
Σ
μi = n3 + 2n
3
a) (2i3 – i + 3) = n
i=1
7
c) Σ μi
i=1
= n2 + 4n
b) (3i3 – 2i2 + i – 6) = n
Σ μi
2
i=1
d) = 3n + 7n
i=1 2

Soluciones
1 1 1 1 1 1 1 1 1
f) 1 – + – + – + – + –
1 1 1 1 1 2 3 4 5 6 7 8 9 10
1. a) 1 + + + + +
2 3 4 5 6 g) 2 + 4 + 6 + 8 + 10 + ... + 2n
b) 1 + 3 + 5 + 7 + 9
1 1 1 1
c) 1 + 4 + 9 + 16 + 25 + 36 + 49 + 64 h) 1 – + + + ... +
3 5 7 2n – 1
d) 1 + 8 + 27 + 64 i) –1 + 4 – 9 + 16 – 25 + ... + –1nn2
3 4 5 6 1
e) –1 + 2 – 3 + 4 – 5 j) –2 + + + + + ... + ( –1)n (1 + )
2 3 4 5 n

Sumatoria y progresiones 481

481-483 481 25/11/02, 12:37 PM


4 5 6 4
i i 1 1
2. a) b) c) (–1)i d)
i = 1 3i – 1 i=1 i+1 i=1 i+1 i=1 3i
• •
k 2k 4. a) 450 b) 2.058 c) 2.660 d) 12.040
3. a) b)
k=1 2k – 1 k=1 2k – 1
5. a) 6.160 b) –1.175 c) 1.026 d) –436
n
3 2
6. a) uk = (k + 1) (k + 2) ; (k + 1) (k + 2) = n + 6n + 11n
k=1
3
n
b) uk = 2k + 1 ; (2k + 1) = n2 + 2n
k=1
n
3 2
c) uk = k(k + 3) ; k(k + 3) = n + 6n + 5n
k=1
3
n
d) uk = k2(k + 1) ; k2(k + 1) = 3n4 + 10n3 + 9n2 + 2n
k=1
12
7. a) ui = i2 – 1 b) ui = i2 – i + 1 c) ui = 2i + 3 d) ui = 3i + 2

12.2 Sucesiones

12.2.1 Definición
Una sucesión es una función cuyo dominio es el conjunto de
números naturales.
En general, se denota de la siguiente manera:
{an} = a1, a2, a3, ..., an,....
El término general de la sucesión es an; el subíndice indica el lugar
que ocupa el término en la sucesión.

• Ejemplos.
Determinar los cinco primeros términos de la sucesión cuyo
término general es:
1. an = n
{an} = 1, 2, 3, 4, 5, ...

2. an = 1
n
{an} = 1, 1 , 1 , 1 , 1 , ...
2 3 4 5

482 Sumatoria y progresiones

481-483 482 25/11/02, 12:37 PM


CAPÍTULO 12

3. an = (–1)n
{an} = –1, 1, –1, 1, –1, ...
n
4. an = (–1)n
n+1
{an} = – 1 , 2 , – 3 , 4 , –5 , ...
2 3 4 5 6

Determinar el término general de las siguientes sucesiones:


Ejercicios
1) 1, 3, 5, 7, 9, ...
resueltos
an = 2n –1 (números impares)

1 1 1 1
2) 1, , , , , ...
4 9 16 25
1
an = (numerador 1, denominador son
n2
los números al cuadrado)

1 2 3 4 5
3) , , , , , ...
3 5 7 9 11
n
an = (el numerador es n, los
2n + 1
denominadores son impares, a
contar del 3)

1 1 1 1 1
4) ,– , ,– , , ...
2 4 8 16 32

1
an = (-1)n + 1 (los signos van intercalados,
2n
comenzando por +; los
numeradores son 1 y los
denominadores son potencias
de 2)
5) 2, 5, 8, 11, 14, ...
an = 3n –1 (la diferencia entre cada par de
términos consecutivos es 3, y
parten del 2, que es 3 – 1)

1 3 5 7 9
6) , , , , , ...
3 5 7 9 11
2n – 1
an = (los numeradores son impares
2n + 1
partiendo por el 1 y los
denominadores son impares
partiendo por el 3)

Sumatoria y progresiones 483

481-483 483 25/11/02, 12:37 PM


Ejercicios
1. Encuentra los cinco primeros términos 2. Encuentra el término general de las
de las sucesiones cuyos términos siguientes sucesiones:
generales son:
a) 3, 7, 11, 15, 19, ...
n–3
a) an =
n–2 b) –3, 9, –27, 81, –243, ...
n2
b) an = 3 5 7 9 11
n+1 c) , , , , , ...
4 7 10 13 16
2n + 3
c) an =
2n – 1
d) 1, –1, 1, –1, 1, ...
n2 – 1
d) an =
n2 + 1 –1 –1 –1 –1 –1
e) , , , , , ...
e) an = (–1)n
2n a a2 a3 a4 a5
2n + 1

Soluciones

1. a) 0 , 1 , 2 , 3 , 4 2. a) an = 4n –1
2 3 4 5
b) 1 , 4 , 9 , 16 , 25 b) an = (-1)n • 3n
2 3 4 5 6
2n + 1
c) 5 , 7 , 9 , 11 , 13 c) an =
3n + 1
1 3 5 7 9
d) 0 3
, , 8 , 15 , 24 d) an = (–1)n + 1
2 5 10 17 26
e) – 2 , 4 , –6 , 8 , –10 e) an = –1
3 5 7 9 11 an

12.2.2 Sucesiones convergentes


Una sucesión {an} es convergente si sus términos se van
acercando cada vez más a un cierto valor. Ese valor se llama
el límite de la sucesión y se dice que la sucesión converge
a ese límite.

Ejercicios 1. Tomemos la siguiente sucesión: an =


1
n
resueltos 1, 1 , 1 , 1 , 1 , ...
2 3 4 5
Los términos de esta sucesión se van acercando cada vez más a
cero, aunque nunca lo alcancen.

484 Sumatoria y progresiones

484-485 484 25/11/02, 12:38 PM


CAPÍTULO 12

1
La sucesión an = es convergente y su límite es cero.
n
n
2. Tomemos la siguiente sucesión: an =
n+1
1 , 2 , 3 , 4 , 5 , ...
2 3 4 5 6
Los términos de esta sucesión se van acercando cada vez más a
1, aunque nunca lo alcancen.
n
La sucesión an = es convergente y su límite es 1.
n+1
3. Consideremos la siguiente sucesión an = –2
–2, –2, –2, –2, –2, ...
Todos los términos son iguales a –2, esta sucesión es convergente
y su límite es –2.

12.2.3 Sucesiones divergentes


Si una sucesión no es convergente se dice que es divergente.
Una sucesión puede divergir porque sus términos oscilan o bien
porque sus términos crecen o decrecen sin medida.
Una sucesión oscilante también puede ser convergente.

1. Consideremos la siguiente sucesión an = (–1)n Ejercicios


–1, 1, –1, 1, –1, .... resueltos
En este caso los términos “oscilan” y no se acercan a ningún
valor; decimos que esta sucesión diverge.
2. Consideremos la sucesión an = 2n –1
1, 3, 5, 7, 9, ...
Los términos de esta sucesión se van haciendo cada vez más
grandes, no se acercan a ningún valor y por lo tanto esta
sucesión también diverge.
1
3. Consideremos la sucesión an = (-1)n + 1
2n
1 , – 1 , 1 , – 1 , 1 , ...
2 4 6 8 10
En este caso los términos “oscilan” pero se van acercando a cero;
esta sucesión converge y su límite es cero.

Sumatoria y progresiones 485

484-485 485 25/11/02, 12:38 PM


12.2.4 Sucesiones crecientes
y decrecientes
Una sucesión es creciente si cada término es mayor que el término
anterior; es decir an > an – 1
Una sucesión es decreciente si cada término es menor que el
anterior an < an – 1

Ejercicios
1
resueltos 1. La sucesión an = 2 –
n

1, 3 , 5 , 7 , 9 , 11 , ...
2 3 4 5 6

es una sucesión creciente, pero sus términos no crecen


indefinidamente; ninguno de ellos es mayor que 2.
Esta es una sucesión convergente y su límite es 2.

2n + 1
2. La sucesión an =
3
3 , 5 , 7 , 9 , 11 , 13 , ...
3 3 3 3 3 3

es una sucesión creciente y sus términos crecen indefinidamente.


Esta es una sucesión divergente.
3. La sucesión an = 1 – 3n
–2, –5, –8, –11, –14, ....
es una sucesión decreciente y sus términos decrecen sin
medida. No converge.

1
4. La sucesión an =
2n – 1

1, 1 , 1 , 1 , 1 , ...
3 5 7 9

es una sucesión decreciente, ya que el denominador “crece” sin


medida mientras que el numerador se mantiene constante. Esta
sucesión converge y su límite es cero.

Nota: En muchos casos, para determinar la convergencia o


divergencia de una situación es conveniente calcular los
términos de ella para valores muy grandes de n.

486 Sumatoria y progresiones

486-487 486 25/11/02, 12:40 PM


CAPÍTULO 12

Ejercicios
1. Analiza las siguientes sucesiones 2) a) Escribe 3 sucesiones crecientes
determinando cuáles de ellas son que sean convergentes.
crecientes, decrecientes, oscilantes,
convergentes o divergentes. b) Escribe 3 sucesiones crecientes
que sean divergentes.
a) an = n + 2
b) an = 6 – 12n c) Escribe 3 sucesiones decrecientes
que sean convergentes.
3
c) an =
n+3
d) Escribe 3 sucesiones decrecientes
3n – 1 que sean divergentes.
d) an =
n–1
1 e) Escribe 3 sucesiones oscilantes
e) an = 1 – que sean convergentes.
n
f) an = (–1)n • n f) Escribe 3 sucesiones oscilantes
n que sean divergentes.
g) an = (–1)2n •
n+1
(n + 1)2 3) Conjetura hipótesis sobre:
h) an = (–1)n + 1
n2
a) Condición necesaria (y suficiente)
n2
i) an = (–1)n para que una sucesión creciente
(n + 1)2
sea convergente.
1
j) an = 1 + (compare con ej. e)
n b) Condición necesaria (y suficiente)
n para que una sucesión decreciente
k) an = sea convergente.
n2 + 1

l) an =
{ n si n es par.
–n si n es impar.
c) Averigua el significado del con-
cepto de “sucesiones acotadas” y
da ejemplos de ella.

Soluciones

1) a) creciente, diverge h) oscila, converge


b) decreciente, diverge i) oscila, converge
c) decreciente, converge j) decrece, converge (al mismo valor
d) creciente, converge que sucesión e)

e) creciente, converge k) decreciente, converge

f) oscila, diverge l) oscila, diverge.

g) oscila, converge

Sumatoria y progresiones 487

486-487 487 25/11/02, 12:40 PM


12.3 Progresión aritmética

Definición: Una Progresión Aritmética (P.A.) es una sucesión


de términos a1, a2, a3, ..., an, ... tal que cada uno se obtiene
de sumar un valor constante al anterior.

a1 : primer término de la P.A.

d : diferencia de la P.A.

an : término enésimo de la P.A.

n : número de términos de la P.A.

Sn : suma de n términos de la P.A.

Fórmulas en una P.A.

an = a1 + (n – 1) d

d = an – an – 1

n n
Sn = (a + an) = [2a1 + (n – 1) d]
2 1 2

Los términos de una P.A. que se encuentran entre dos términos dados
se llaman medios aritméticos y el procedimiento para hallarlos se
denomina interpolación de medio aritméticos.

1. Calcular el término que ocupa el lugar 50 en una P. A. si el primero


Ejercicios es 5 y la diferencia es 2.
resueltos Solución:
a1 = 5 ; d=2 ; n = 50 ; an = ?
an = a1 + (n – 1) d
a50 = 5 + 49 • 2 = 103
a50 = 103

488 Sumatoria y progresiones

488-489 488 25/11/02, 12:42 PM


CAPÍTULO 12

2. El undécimo término de una P. A. es 49 y su diferencia es 4.


Encontrar el primer término.
Solución:
a11 = 49 ; d=4 ; a1 = ?
an = a1 + (n – 1) d
49 = a1 + (11 – 1) 4
a1 = 49 – 40
a1 = 9

3. El primer término de una P. A. es 5, su diferencia es 4 y el término


enésimo es 53. Hallar el número de términos.
Solución:
a1 = 5 ; d=4 ; an = 53 ; n=?
an = a1 + (n – 1) d
53 = 5 + (n – 1) 4
4n = 52
n = 13

4. Determinar la P. A. cuyo quinto término es 14 y cuyo décimo


término es 29.
Solución:
Conocer una P. A. significa determinar su primer término
y su diferencia.
a5 = 14 ; a10 = 29
como an = a1 + (n – 1) d
se tiene 14 = a1 + 4d
29 = a1 + 9d
Resolviendo el sistema:
d=3 y a1 = 2 luego la P. A. es:
2, 5, 8, 11, 14, 17, 20, 23, 26, 29, 32,...

5. En la P. A. tal que su sexto término es 15 y la diferencia es 3 ,


5
hallar el término del lugar 16.
Solución:
3
a6 = 15 ; d= ; a16 = ?
5
a6 = a1 + 5d fi 15 = a1 + 3 fi a1 = 12
a16= a1 + 15d fi a16 = 12 + 9 fi a16 = 21

Sumatoria y progresiones 489

488-489 489 25/11/02, 12:42 PM


6. Interpolar 5 medios aritméticos entre los números 3 y 6.
Solución:
Se está pidiendo formar una P. A. de 7 términos donde el primero
es 3 y el séptimo es 6. Se debe determinar la diferencia y con ello
calcular los 5 medios aritméticos pedidos:
a1 = 3 ; a7 = 6 ; a2, a3.... a6 = ?
a7 = a1 + 6d
1
6 = 3 + 6d fi d =
2
7
Luego: a2 = , a3 = 4, a4 = 9 , a5 = 5, a6 = 11
2 2 2

Son los cinco medios aritméticos pedidos.


7. Hallar tres números que están en P. A. y cuya suma sea 39.
Solución:
Sean a1 , a2 y a3 los números pedidos.
Hagamos a1 = a2 – d y a3 = a2 + d.
Luego: (a2 – d) + a2+ (a2 + d) = 39
3 a2 = 39
a2 = 13
a1 + a2 + a3 = 39
a1 + a3 = 26
Si a1 = 1, entonces a3 = 25 y los tres números pedidos son 1, 13 y 25.
Si a2 = 2, entonces a3 = 24 y los tres números pedidos son 2, 13 y 24.
Luego, este problema tiene infinitas soluciones. Los tríos de
números naturales que satisfacen la condición son:
1, 13 y 25 6, 13 y 20 11, 13 y 15
2, 13 y 24 7, 13 y 19 12, 13 y 14
3, 13 y 23 8, 13 y 18 13, 13 y 13
4, 13 y 22 9, 13 y 17
5, 13 y 21 10, 13 y 16
8. Determinar una P. A. sabiendo que la suma del primer
y tercer término es 44 y el producto del segundo por el
primero es 418.
Solución:
Sean a – d, a , a + d los tres términos de la P. A.

(a – d) + (a + d) = 44 (1)
a (a – d) = 418 (2)

De (1) 2a = 44 fi a = 22

490 Sumatoria y progresiones

490-491 490 25/11/02, 12:46 PM


CAPÍTULO 12

De (2) a2 – ad = 418
484 – 22 d = 418
d=3
Luego a – d = 22 – 3 = 19 y a + d = 22 + 3 = 25
La P.A. pedida es 19, 22, 25
9. Hallar la suma de los 100 primeros múltiplos de 3.

Solución:
Observamos que los 100 primeros múltiplos de 3 equivalen
a la P.A. cuyo primer término es 3, la diferencia es 3 y tiene
100 términos.
a1 = 3 ; d=3 ; n = 100
n
Sn = [2a1 + (n – 1)d]
2
S100 = 50 (6 + 99 • 3) = 15.150
Luego, la suma de los 100 primeros múltiplos de 3 es 15.150.
10. Calcular cuánto dinero tenía para vacaciones un joven si el
primer día gastó $ 7.000, fue disminuyendo el gasto en $200
diarios y el dinero le duró 30 días.

Solución:
Vemos que la situación planteada obecede a una P.A. donde el
primer término es 7.000, la diferencia es –200 y el número de
términos es 30. El dinero con que el joven contaba para vacaciones
es la suma de los 30 términos.
a1 = 7.000 ; d = –200 ; n = 30
n
Sn = [2a1 + (n – 1)d]
2
S30 = 15 (14.000 + 29 (–200)) = 123.000
El joven tenía $123.000 para vacaciones.
11. Para hacer un túnel se sabe que el primer metro tiene un
costo de $1.000.000 y por cada metro más se debe agregar
$80.000. ¿Cuál es el largo del túnel si se ha debido cancelar
$33.264.000.000?

Solución:
Podemos pensar esta obra como una P.A. en que el primer término
es 1.000.000, la diferencia es 80.000 y la suma del costo de los n
metros que hubo que hacer es 33.264.000.000.
a1 = 1.000.000 ; d = 80.000 ; Sn= 33.264.000.000 ; n=?
n
Sn = [2a1 + (n – 1)d]
2
33.264.000.000 = n [2.000.000 + 80.000 n – 80.000]
2

Sumatoria y progresiones 491

490-491 491 25/11/02, 12:46 PM


66.528.000.000 = 1.920.000 n + 80.000 n2
8n2 + 192 n – 6.652.800 = 0 /: 8
n2 + 24 n – 831.600 = 0
n1 = 900
n = –24 ± 3.326.976 =
2 n2 = –924

Luego el túnel tiene 900 m de largo.

Ejercicios 10. Hallar la suma de los 10 primeros


términos de las siguientes sucesio-
1. Dadas las progresiones aritméticas nes:
siguientes, hallar el término que se a) 1, 2, 3,...
indica:
b) 2, 4, 6,...
a) 5, 8, 11, 14... a10 c) 5, 2, –1,...
1
d) 1, , 0,...
b) 5, 2, –1, –4... a20 2
1 7 11. Encontrar la suma de una P.A. de
c) , 2, , 5... a12 20 términos si el primero es 5 y el
2 2
último es 62.
d) 3, 8, 13, 18... a15 12. Hallar la suma de los 15 primeros
términos de una P.A. si se sabe que
2. Determinar cuántos términos tiene
el quinto término es 17 y el séptimo
una P.A. si el primero es 5, el último
es 23.
es 50 y la diferencia es 3.
13. Calcular la suma de los múltiplos de
3. Hallar la P.A. tal que la suma de los siete que están entre 100 y 200.
20 primeros términos es 120 y su
diferencia es 2. 14. En una P.A. cuya diferencia es 4 el
término central vale 21 (tiene un
4. Determinar la diferencia en una P.A. número impar de términos). Si su
cuyo término de lugar 27 es 32 y suma es 189, encontrar el número de
cuyo término de lugar 18 es 5. Hallar términos y escribir la progresión.
también el primer término.
15. Hallar tres números enteros que estén
5. Calcular el término de lugar 100 en la en P.A., cuyo producto es 15.000 y
P.A. –6, –4, –2, 0, 2, .... cuya suma es 75.
6. En la P.A. 5, 8, 11, 14... determinar 16. Determinar cuántos términos de la
qué lugar ocupa el término de valor P.A., 2, 6, 10,... hay que sumar para
65. obtener 288.
7. Interpolar cinco medios aritméticos 17. Al sumar números pares consecutivos
entre 12 y 42. a partir del 10 se obtiene 580.
8. Interpolar tres medios aritméticos ¿Cuántos números se han sumado?
entre 12 y –12. 18. La suma de cinco números que están
9. Interpolar tres medios aritméticos en P.A. es 20 y su producto es 720.
entre 1 y 7. Hallar dichos números.

492 Sumatoria y progresiones

492-493 492 25/11/02, 12:48 PM


CAPÍTULO 12

19. Encontrar tres números sabiendo que 30. Para construir una vía elevada se
están en P.A., que su suma es 51 y levanta sobre una superficie hori-
el mayor es dos unidades menor que zontal una rampa de pendiente
el doble del menor. uniforme, la cual se sostiene sobre
12 soportes igualmente espaciados.
20. La suma de los nueve términos de La altura del primer soporte es 2
una P.A. es –9. La diferencia entre m y la del más alto es de 51,5 m.
el primer y el último es –16. Hallar Encontrar la altura de cada soporte.
dicha progresión.
31. Las medidas de los lados de un
21. Hallar una P.A. de 8 términos triángulo rectángulo forman una P.A.
sabiendo que los cuatro primeros y suman 45 m. Calcular cuánto mide
suman 40 y que los cuatro últimos cada uno.
suman 72. 32. Se deja caer una bola de goma, la
22. En una P.A. el segundo y el décimo cual en el primer bote se levanta 1
término suman 44 y el primero mñas m del suelo, en el segundo bote sólo
el noveno suman 34. Hallar los 95 cm, en el tercer bote 90 cm y así
términos mencionados. sucesivamente. Calcular cuánto ha
recorrido la bola desde que toca por
23. Si las expresiones 5x + 3, 3x + 2 primera vez el suelo hasta que llega
y 2x – 1 están en P.A., hallar el al punto más alto después del décimo
valor de x. bote. ¿Cuántos botes alcanza a dar
24. Interpolar dos medios aritméticos antes de detenerse?
entre a y b. Dar un ejemplo si 33. Las medidas de los ángulos de un
a = 5 y b = 8. triángulo forman una P.A. Si uno
de ellos mide 20o, ¿cuánto miden
25. Determinar d en una P.A. de n + 2
los otros dos?
términos si a1 = a y an+2 = b.
34. ¿Cuántos medios aritméticos se
26. Interpolar 5 medios aritméticos entre deben interpolar entre 4 y 40 para
–2 y 10. que la suma de la P.A. resultante sea
27. Encontrar la suma de los n primeros 220? ¿Cuáles son esos medios?
números naturales. 35. Un espectador de teatro ve bien a
28. Encontrar la suma de los p primeros 26 m del escenario. ¿En qué fila
números naturales pares. debe sentarse si la primera dista 8
m del escenario y la decimoctava,
29. Encontrar la suma de los q primeros que es la última, está a 42 m del
números naturales impares. escenario?

Soluciones

1. a) 32 b) –52 c) 17 d) 73 8. Los medios son 6, 0 y –6.

2. 16 3. –13, –11, –9, –7... 5 11


9. Los medios son ,4 y
2 2
4. d = 3 ; a1 = –46 5. 192 6. 21 10. a) 55 b) 110 c) –85 d) –12.5
7. Los medios son 17, 22, 27, 32 y 37 11. 670 12. 390 13. 2.107

Sumatoria y progresiones 493

492-493 493 25/11/02, 12:48 PM


14. n = 9; 5, 9, 13, 17, 21, 25, 29, 33, 37 n(n + 1)
27. Sn = 28. p + p2 29. q2
15. 20, 25 y 30 16. 12 17. 20 2
30. 2, 6.5,11, 15.5, 20, 24.5, 29, 33.5, 38,
18. 2, 3, 4, 5, 6 19. 12, 17 y 22
42.5, 47, 51.5 m.
20. –9, –7, –5, –3, –1, 1, 3, 5, 7, (a1 = –9 ; d = 2)
31. 11.25 m, 15 m y 18.75 m.
21. a1 = 7 d = 2
22. a1 = –3, a2 = 2, a9 = 37, a10 = 42, 32. 14.95 m alcanza a dar 20 botes

33. 60o y 100o


2a + b a + 2b
23. x = 2 24. , ; 6, 7
3 3 34. Se deben interpolar 8 medios y son 8,
12, 16, 20, 24, 28, 32 y 36.
b–a
25. d = 26. 0, 2, 4, 6, 8
n+1 35. Deben sentarse en la décima fila.

12.4 Progresión geométrica

12.4.1 Definición
Una Progresión geométrica (P.G.) es una sucesión de
términos a1, a2, a3, ..., an, ... tal que cada uno se obtiene de
multiplicar el antecesor por un valor constante r.
a1 : primer término de la P.G.

r : razón de la P.G.

an : término enésimo de la P.G.

n : número de términos de la P.G.

Sn : suma de n términos de la P.G.

Pn : producto de n términos de la P.G.

an = a1 rn – 1
a
r= a n
n–1

a1(1 – rn) Si r = 1 entonces


Sn = , r π1 S n = n • a1
1–r
Si n fi • y | r | < 1, entonces
a1
Sn =
1–r

494 Sumatoria y progresiones

494-495 494 25/11/02, 12:52 PM


CAPÍTULO 12

Pn = (a1 • an)n

Los términos de una P.G. que se encuentran entre dos términos dados
se llaman medios geométricos y el procedimiento para encontrarlos
se denomina interpolación de medios geométricos.

12.4.2 Cálculo de intereses


de capital
Una aplicación directa de las progresiones geométricas la
encontramos en los problemas financieros de intereses.

Un capital inicial C produce durante un año al i% de interés


C•i
anual los intereses de .
100
Se llama interés compuesto si al cabo del año ese dinero que
se obtuvo como interés de capital pasa a formar parte del capital y
produce interés para el año siguiente.
Entonces en t años el capital inicial C se convierte en un capital
final que llamaremos S y se calcula según la fórmula.

i t
S = C (1 + )
100
Por cada año que pasa el capital es un término de una P.G.,
donde el primer término es el capital inicial C y la razón es
i
1+ .
100
NOTA: t representa el período que se acuerda para el cálculo
y pago de los intereses. Puede ser anual, semestral,
trimestral o mensual.

1. Calcule el noveno término de una P.G. cuyo primer término Ejercicios


es 1 y la razón es 3.
Solución:
resueltos
an = a1 • r n–1 a1 = 1 ; r = 3 ; a9 = ?
a9 = 1 • 38 = 6.561
2. El quinto término de una P.G. es 162 y el primero es 2. Hallar
la razón.
Solución:
an = a1 • r n–1 a1 = 2 ; a5 = 162 ; r = ?
4 4
162 = 2 • r fi r = 81
r = 4√81
r =3

Sumatoria y progresiones 495

494-495 495 25/11/02, 12:52 PM


3. El séptimo término de una P.G. es 192 y la razón es 2. Hallar
el primer término:
Solución:
an = a1 • r n–1
a7 = 192 ; r = 2 ; a1 = ?
192
192 = a1 • 26 fi a1 = =3
64
1
4. En una P.G. el primer término es 32 y la razón es . Determinar
2
1
qué lugar ocupa el término que vale .
8
Solución:
an = a1 • r n–1
1 1
an = ; a1 = 32 ; r = ; n=?
8 2

( )
n–1
1 1
= 32 •
8 2
2–3 = 25 • 21 – n
2–3 = 26 – n

–3 = 6 – n

n=9
1
El término ocupa el noveno lugar de la P.G. dada.
8
5. Demostrar que la suma de n términos de una P.G. cuyo primer
término es a1 y cuya razón es r dada por la fórmula
a (1 – rn)
Sn = 1 • Si |r| < 1 y n tiende a infinito, entonces la
1–r a1
suma de infinitos términos de la P.G. es S =
1–r.
Solución:
Los términos de la P.G. son a1, a2 = a1 r
_
a3 = a1 r2, a4 = a1 r3 , ... an = a1 rn 1 , entonces
Sn = a1 + a1 r + a1 r2 + ... + a1 rn–1 /•r
2 n–1
r Sn = a1 r + a1 r + ... + a1 r + a1 r n

restando miembro a miembro.


Sn – r Sn = a1 – a1 rn
Sn (1 – r) = a1 (1 – rn)
a (1 – rn)
Sn = 1
1–r
Si |r| < 1 entonces rn se hace cada vez más chico a medida que
aumenta n. Por ejemplo si r = 0,5
r2 = (0,5)2 = 0,25 r7 = (0,5)7 = 0,0078125

r3 = (0,5)3 = 0,125 •

r4 = (0,5)4 = 0,0625
r15 = (0,5)15 = 0,0000305
5 5
r = (0,5) = 0,03125 •
r6 = (0,5)6 = 0,015625 •

496 Sumatoria y progresiones

496-497 496 10/11/2001, 12:22


CAPÍTULO 12

es decir, si n es muy grande, rn se hace muy pequeño y se puede


despreciar su valor por lo que en ese caso rn tiende a cero y:
a1
S=
1–r
6. Hallar la suma de los 10 primeros términos de la P.G. 4,
12, 36, 108...
Solución:
36
a1 = 4 ; r = = 3 ; n = 10
12
a1(1 – rn)
Sn =
1–r
4(1 – 310) 4(1 – 59.049)
S10 = = = 118.096
1–3 –2
7. Calcular la suma de infinitos términos de la P.G.
1, 1 , 1 , 1 , 1 , ...
2 4 8 16
Solución:
1
a1 = 1 ; r = <1 ; nfi•
2
a1
S=
1–r
1 1
S= = =2
1– 1 1
2 2
8. Demostrar que el producto de n términos de una P.G. es
Pn = √(a1 • an)n
Solución:
Sea la P.G. cuyo primer término es a1, y la razón es r, entonces
los términos siguientes son:
a2 = a1r
a3 = a1r2
a4 = a1r3


an = a1r n–1
Pn = a1 • a1 r • a1 r2 • .... • a1 rn–2 • a1 rn–1
Pn = a1 rn–1 • a1 rn–2 • a1 rn–3 • ... • a1 r • a1
Multiplicando término a término:
Pn2 = a1a1rn–1 • a1a1rn–1 • a1a1rn–1 • ... • a1a1rn–1 • a1a1rn–1
Pn2 = (a1 • an)n
Pn = (a1 • an)n
9. Calcular el producto de los 10 primeros términos de la P.G.
3 , 3 , 3 , ...
2 4

Sumatoria y progresiones 497

496-497 497 10/11/2001, 12:22


Solución.
Pn = (a1 • an)n
1 1 9 3
a1 = 3 ; r =
2
; a10 = 3 ( )
2
= 9
2
P10 = (3 • 3 • 2 ) = (3 • 3 • 2 )5 = 310 • 2–45
–9 10 –9

P10 = 310 • 2–45

10. Interpolar 5 términos entre 1 y 512 para que resulte una


8
progresión geométrica.
Solución:
1
Se trata de formar una P.G. donde el primer término sea y
8
el séptimo término sea 512.
a1 = 1 = 2 –3
8
a7 = 512 = 29

a7 = a1 • r6 fi 29 = 2 –3 • r6 fi r6 = 212
6
r = 4.096 fi r = 4

Con una calculadora podríamos aplicar logaritmo para calcular r.


1 1
a2 = •4=
8 2
1
a3 = • 42 = 2
8
1
a4 = • 43 = 8
8
1
a5 = • 44 = 32
8
a6 = 1 • 45 = 128
8
11. Probar que en una P.G. que tiene un número impar de términos,
el término central es igual a la raíz cuadrada del producto
de los extremos.
Solución:
Sea p = 2 n – 1 el número de términos de la P.G.
El primer término es a1.
El p-ésimo término es ap = a1 • r p–1
El término central ac es el de lugar p + 1 , es decir, es
p–1 2
ac = a1r 2 – 1
p–1
ac = a1r 2

ac = a1rn–1

a c = an

498 Sumatoria y progresiones

498-499 498 25/11/02, 12:53 PM


CAPÍTULO 12

Por demostrar que ac = a1 • ap = a1 • a1r p–1 =


a1 • a1r 2n – 2 = a12 • r 2(n – 1) = (a1 • r n – 1)2 = a1 • r n–1 = an
luego ac = a1 • ap p número impar.
12. Hallar el quinto término de una P.G. si se sabe que el primer
término es 2 y el noveno es 13.122.
Solución.
Como es una P.G de 9 términos, el quinto es el término central
y por lo tanto es la raíz del producto del primero por el último
(ver ejercicio anterior).
a5 = a1 • a9 = 2 • 13.122 = 162
13. Se toma una hoja de papel de 0,1 mm de grosor. Se corta en dos
y se pone una encima de la otra; queda un grosor de 0,2 mm.
Este fajo se corta en dos y se pone una parte encima de la otra;
queda de grosor de 0,4 mm. La siguiente vez que se efectúa
el mismo procedimiento, el fajo queda con un grosor de 0,8
mm. Suponiendo que el papel es de un tamaño tal que permite
efectuar el mismo procedimiento 30 veces, averiguar el grosor
que tendría el fajo formado.
Solución.
El grosor se duplica cada vez que se efectúa el procedimiento
descrito: 0,1; 0,2; 0,4; 0,8, ... estamos frente a una P.G.
cuyo primer término a1 = 0,1 mm y cuya razón r = 2. La
altura del fajo pedida es la suma de los 30 primeros términos
de esta P.G.
a (1 – r29) 0,1(1 – 229)
S30 = 1 = = 53.687.091 mm
1–r 1–2
Por lo tanto, el grosor del fajo obtenido luego de cortar 30 veces
el papel es de aproximadamente 53,68 km.
14. Se estima que el crecimiento de la población de una región
será de un 2% anual. Calcular el porcentaje en que habrá
crecido la población al cabo de 20 años. ¿A los cuántos años
se duplicará la población?
Solución.
Sea p la cantidad de población existente. Si el crecimiento se
estima en un 2% anual, entonces al cabo del:
1er año será 1,02 p
2do año será (1,02)2 p
3er año será (1,02)3 p


20o año será (1,02)20 p = 1,49 p.
Por lo tanto, en 20 años la población crecerá en un 49%.
Estamos frente a una P.G. cuyo primer término es p (población
existente en el momento de efectuar la medición) y cuya razón
es 1,02 (crecimiento de 2% anual).

Sumatoria y progresiones 499

498-499 499 25/11/02, 12:54 PM


Queremos averiguar en cuántos años se duplicará la población.
an = 2 p
an = a1 • rn – 1
2p = p • (1,02)n – 1
(1,02)n
2 =
1,02
(1,02)n = 2,04
log 2,04
n =
log 1,02
n = 36
Es decir, al cabo de 36 años se duplicará la población existente
si la tasa de crecimiento es de un 2% anual.
15. Expresar en forma racional los decimales infinitos siguientes:
a) 0,424242...
b) 025323232...
Solución:
42 42 42
a) 0,424242... = + + + ...
100 10.000 1.000.000
Es decir, se trata de una suma de infinitos términos de una P.G.
42 1
cuyo primer término a1 = y cuya razón r =
100 100
Como r = 1 < 1, entonces:
100
42 42
a1
= 99 = 42 = 14
100 100
S= =
1–r 1–
1 99 33
100 100

Así; 0,42 = 14
33
32 32
b) 0,25323232... = 0,25 + + +...
10.000 1.000.000

Se trata de sumar infinitos términos de una P.G. cuyo primer


32 1
término a1 = y su razón r = . Luego sumamos 0,25
10.000 100
y obtenemos la fracción pedida:
32 32
10.000 10.000 32
S= 1
= 99
=
1– 100 100
9.900
32
Así; 0,25323232... = 0,25 +
9.900
1 32 2.475 + 32
= + =
4 9.900 9.900
2.507
=
9.900
2.507
Luego 0,25323232... =
9.900

500 Sumatoria y progresiones

500-501 500 10/11/2001, 11:10


CAPÍTULO 12

16. Probar que dado un capital C, puesto a un interés compuesto del

i% en un período determinado, al cabo de t períodos iguales el


t
i
capital C se ha transformado en C 1 +
100
Solución.
Capital: C; interés: i%; t = períodos
Llamaremos S al capital acumulado al cabo de t períodos.
Al finalizar el primer período:
i
interés = C • i ; Capital = C + Ci = C 1 +
100 100 100
Al finalizar el segundo período:
i i i i i
interés = C 1 + ; Capital= C 1 + + C 1+
100 100 100 100 100
2
i
= C 1+
100
Al finalizar el tercer período:
2 2 2

interés = C 1 +
i

i ; Capital= C 1 + i + C 1+
i
i
100 100 100 100 100
3
i
= C 1+
100
Al finalizar el t-ésimo período:
t–1 t
i i i
interés = C 1 + • ; Capital = C 1 +
100 100 100
del t-ésimo período.
17. Se invierten $ 500.000 al 6% anual. Calcular el capital que se habrá
formado al cabo de 5 años si el interés es compuesto:
a) anual, b) semestral, c) cuatrimestral.
Solución.
C = 500.000 ; i = 6% anual ; t = 5 años
a) Al cabo de 5 años el capital se transforma en:
t 5
i 6
C 1+ = 500.000 1 + =
100 100

= 500.000 (1,06)5 = $ 669.111


b) Si los intereses se calculan semestralmente (dos veces al año).
C = 500.000 ; i = 3% semestral ; t = 2 • 5 = 10
En estas condiciones al cabo de 5 años el capital se transforma en:
t 10

C 1+
i = 500.000 1 + 3 =
100 100
= 500.000 (1,03)10 = $ 671.958
c) Si los intereses se calculan cuatrimestralmente (cuatro veces al año)
C = 500.000 ; i = 2% trimestral ; t = 3 • 5 = 15

Sumatoria y progresiones 501

500-501 501 10/11/2001, 11:11


Ahora el capital, al cabo de 5 años, se transforma en:
t 15
i 2
C 1+ = 500.000 1 + =
100 100

=500.000 (1,02)15 = $ 672.933


18. Calcular el capital que debe invertirse para que con un interés
compuesto del 10% anual, pagado semestralmente, al cabo de 3
años se logre un capital de $1.000.000.
Solución.
Nos preguntan por el capital inicial C. Llamaremos S al capital
logrado en los 3 años.
S = 1.000.000 ; i = 5% ; t=6
t
i
S = C 1+
100

1.000.000 = C (1,05)6
1.000.000
C= = $ 746.268
1,34
Luego, para lograr $ 1.000.000 en 3 años en las condiciones
señaladas, debe invertirse un capital inicial de aproximadamente
$ 746.268.
19. ¿A qué porcentaje de interés compuesto deberá ponerse un capital
para que se duplique en 5 años?
Solución:
Se desea que el capital inicial se duplique en 5 años: S = 2C
t
i
S= C 1+ ; i=?; t = 5 años
100
5
i
2C = C 1+
100
5
i
2 = 1+
100

5
100 2 = 100 + i
5
i = 100 2 – 100
i = 100 • 1,1487 – 100
i = 114,87 – 100 = 14,87
Es decir, un capital puesto a un interés compuesto del 15% anual
se duplica en 5 años (ver ejercicio siguiente).
20. Calcular el capital que se obtiene al depositar $ 400.000 a un
interés compuesto del 15% anual durante 5 años.
Solución.
C = 400.000 ; i = 15% ; t=5; S=?
t 5
i 15
S = C 1+ S = 400.000 1 +
100 100

502 Sumatoria y progresiones

502-503 502 25/11/02, 12:58 PM


CAPÍTULO 12

= 400.000 (1, 15)5


= 400.000 • 2,0113571 = $ 804.543
Vemos que en 5 años al 15% el capital aproximadamente se
duplica.
21. Un banco concede un préstamo de $ 1.000.000 pagadero en 3
años con un interés anual del 15%. ¿Cuánto se debe cancelar
mensualmente? ¿Cuánto se cancelará en total?
Solución:
El 15% anual corresponde al 1,25% de interés mensual y son 36
meses; por lo tanto, el capital de $ 1.000.000 se convertirá al
cabo de los tres años en S.
t
i C = 1.000.000
S = C 1 +
100 i = 1,25
t = 36
S = 1.000.000 (1,0125)36 = 1.563.942
lo que dividido en 36 cuotas nos da $ 43.443 cada mensualidad.
Se deben pagar 36 mensualidades de $ 43.443 cada una, lo que
da un total de $ 1.563.942.

Ejercicios

1. Determine si las siguientes sucesiones 4. Encuentre tres números que formen


son o no progresiones geométricas. Si una P. G. tales que su producto sea
lo son, hallar la razón. 1.728 y su suma sea 52.

a) 4, 12, 36, ... 5. En una P. G. el primer término es 23


y la razón es 2. ¿Cuántos términos se
b) 24, 18, 14, ... deben sumar para que el resultado
sea 1.449?
c) 3, – 9, 27, ...
1 1 1
6. Una P. G. consta de cuatro términos.
d) , , , ... Si la suma del primero y el último es
2 4 8
168 y la suma de los dos centrales es
e) q, 1, 1 ,
q 2
1
, ... 72, encuentre esta progresión.
q
1 1 1 3
f) r
+ r, + 2 + r2, + + 3 r + r3 ,
r
.... 7. Si un muchacho ganara $ 1 el primer
r2 r3
día de su trabajo, $ 2 el segundo
2. Encuentre el quinto término y la suma
día, $ 4, el tercer día, $ 8 el cuarto,
de los 10 primeros términos de la P.
G. 8, 4, 2, ... ¿cuánto habría ganado al cabo de
20 días de trabajo?
3. El segundo término de una P. G. es
32 8. Calcule la media geométrica entre
y el quinto es . Encuentre el
3 p y q.
octavo término.

Sumatoria y progresiones 503

502-503 503 25/11/02, 12:58 PM


9. Encuentre los 5 primeros términos de la razón y escribir los cuatro primeros
una P. G. de modo que el primero sea términos.
2 y el segundo sea 3.
22. Calcule la suma de:
10. Interpole 4 términos entre 1 y 243 de
3 + 0, 3 + 0, 03 + 0, 003 +...
modo que formen una P. G.
23. La suma de tres números en P. G. es 35.
La diferencia entre el mayor y el menor
11. Interpole tres términos entre 3 y 1
4 108 es 15. Encuentre la progresión.
de modo que se forme una P. G.
x–4
24. Encuentre x para que , x + 2
12. Interpole dos medios geométricos 2
entre a y b. y 2 (x – 2) estén en P. G. ¿Cuánto
vale la razón?
13. Interpole dos medios geométricos
entre 6 y 162 (ver ejercicio anterior).
25. Determine x para que x + 1, 2 x + 2 y
4 x + 4 formen una P. G.
14. Encuentre tres números en P. G. tales
que su producto sea 512 y que el
26. Determine x para que x – 1, x + 2
menor sea la décima parte del doble
y x + 5 estén en P. G.
del central.
27. Encuentre la suma de n términos de
15. Determine el producto de los once
la P. G. 1, – 1, 1, – 1,...
términos de una P. G. si el término
central es 3. 28. Si se sabe que una determinada
bacteria se reproduce por bipartición
16. Determine el producto de los 2n–1
cada 20 minutos, es decir, de cada
términos de una P. G. si su término
bacteria aparecen 2 cada 20 min,
central es q.
¿cuántas bacterias habrá pasadas
10 horas desde que se detectó la
17. Determine cuatro números que estén primera?
en P. G. tales que el primero y el
29. Se tiene un cuadrado de lado a. Se
cuarto sumen 91 y los dos centrales inscribe en él un cuadrado uniendo los
108
sumen 7 . puntos medios del cuadrado original
9
y así se van inscribiendo cuadrados
18. En la P. G. 4 , 2 , 1 ..., encuentre el cada vez más chicos. Calcule la suma
5 5 5 de las áreas y de los perímetros de los
valor de la razón y la suma de infinitos
infinitos cuadrados así obtenidos.
términos.
30. Calcule la suma de las áreas y de los
1 1 1 perímetros de todos los cuadrados que
19. Calcule la suma de 1+ + + + ...
3 9 27 se pueden inscribir sucesivamente
20. Determine cuántos términos tiene a partir de un cuadrado de 4 m de
la P. G. cuyo primer término es 2 lado.
y cuyo último término es 512 si su
suma es 682. 31. El crecimiento de una población es
del 4% anual. ¿Qué población habrá
21. La suma de una P. G. infinita es 6,25 dentro de 10 años si hoy hay 150.000
y el segundo término es 1. Encuentre habitantes?

504 Sumatoria y progresiones

504-505 504 10/11/2001, 11:20


CAPÍTULO 12

32. Una región tiene 1.000.000 de 36. Una persona decide ahorrar $ 500.000,
habitantes. ¿En cuántos años se los que ingresará al banco al inicio
duplicará esta población si la tasa de de cada año. Si el banco le paga un
crecimiento es del 5% anual? 12% de interés compuesto anual, ¿qué
capital tendrá al cabo de 6 años?
33. Calcule el capital final que se obten-
drá si se invierten $ 2.000.000 al 37. Para la adquisición de un departa-
16% anual al cabo de 3 años. mento una persona debe contraer
una deuda por $ 4.000.000. El banco
34. Determine el capital que se obtiene se la presta, pero le cobra un 15%
en 2 años colocando $ 500.000 a un anual. Si la persona decide pagar en
interés del 8% anual si los intereses 20 años, ¿cuánto deberá cancelar en
se liquidan trimestralmente. total al cabo de los 20 años? ¿Cuánto
le saldrá la cuota mensual si se fija
35. ¿Qué capital se debe invertir si
se quiere que al cabo de 5 años, el valor con el 15% anual?
colocado a un interés del 12% anual,
éste se convierta en $ 1.000.000?

Soluciones
20. n = 5 21. r = 1 ; 5, 1, 1 , 1
1. a) es P.G. r = 3 5 5 25
4
b) no es P.G. 14 ≠ 18 r = ; 1,25 ; 1 ; 0,8 ; 0,64
5
18 24
c) es P.G. r = –3 22. S = 10 23. 5, 10, 20.
3

d) es P.G. r = 1 24. x = 2 ; r = – 4
2 5 3
e) es P.G. r = 1 25. Cualquier x π – 1 hace de este trío
q
una P.G.
1
f) es P.G. razón = r
+r 26. No existe x E R tal que x – 1, x + 2 y
2. a5 = 1 ; S10 = 1.023 x + 5 estén en P.G.
2 64 27. Si n es par Sn = 0. Si n es impar Sn = 1.
3. 256 4. 4, 12 y 36
3 28. Habrá más de 1.073 millones de bacte-
5. Los seis primeros 6. 6, 18, 54, 162.
rias.
términos.
29. Sáreas = 2a2; Sperímetros = 8a
7. $ 1.048.575 8. pq = 4a 2 + 2
9 27 81 30. Sárea = 32 m2; 2– 2
9. 2, 3, , , 10. 3, 9, 27, 81
2 4 8
1 1 1 3 6
Sperímetros = 16 2 + 2 = 54,6 m.
11. , , 12. a2b , a2 b4
4 12 36 31. 222.033 habitantes.
13. 18 y 54 14. 1.6, 8, 40 32. En aproximadamente 15 años.
15. 177.147 16. q2n–1 33. $ 3.121.792 34. $ 585.526
1 1 4 16 35. $ 567.427 36. $ 4.544.508
17. , , ,
4 3 9 27
37. $ 65.466.128
18. r = 1 S=
8
19. S = 3
2 5 2 Cuota mensual fija: $ 272.776.

Sumatoria y progresiones 505

504-505 505 10/11/2001, 11:21


12.5 Progresión
armónica

Definición: Una Progresión Armónica (P.H.) es una


sucesión de términos a1, a2, a3,... an,...
tal que 1 1 1 1
, , , ... , ...
a1 a2 a3 an
están en progresión aritmética.

Nota: No disponemos de fórmulas elementales para el cálculo


de progresiones armónicas; por ello resolveremos los ejercicios
ocupando la definición y, por lo tanto, las fórmulas para
progresiones aritméticas.

1. Determinar si las siguientes sucesiones son o no progresiones


Ejercicios
armónicas (P.H.).
resueltos a) 1 1 1 1
, , , , ... b) 1, 2 1 2 1 2
, , , , , ...
2 4 6 8 3 2 5 3 7
Solución.
Debemos analizar si la sucesión de los términos recíprocos
es una P.A.
a) 2, 4, 6, 8,... es P.A. con a1 = y d = 2

luego 1 1 1 1 es P.H.
, , ,
2 4 6 8
3 5 7 1
b) 1, , 2, , 3, es P.A. con a1 = 1 y d =
2 2 2 2
2 1 2 1 2
luego 1, , , , , es P.H.
3 2 5 3 7
2. Hallar el término siguiente en cada P.H.:
1 1 1
a) , – 1, – , – b) 2 , 1, 3 , 3 , 1
2 4 7 3 4 5 2
Solución.
Debemos trabajar con los recíprocos de los términos dados y
buscando la diferencia de la P.A.
a) 2, – 1, – 4, – 7, es P.A. de d = –3, luego el término siguiente
es – 10.
Luego el término siguiente de la P.H. es – 1
10
b) 2 , 1, 4 , 5 , 2 es P.A. de d = 1 , luego el término siguiente es 7 .
3 3 3 3 3
Por lo tanto, el término siguiente de la P.H. es 3
7
3. Interpolar un medio armónico entre a y b.
Solución.
Sea x el medio armónico que debemos interpolar.

506 Sumatoria y progresiones

506-507 506 10/11/2001, 11:24


CAPÍTULO 12

Entonces a, x, b, deben estar en P.H., lo que significa que


1 1 1
, , deben ser P.A., luego
a x b
1 1 1 1
– = –
b x x a
2 1 1
= +
x b a
2 a+b
= fi x = 2ab
x ab a+b
4. Interpolar 2 medios armónicos entre 2 y 12.

Solución.
Sean x e y los medios armónicos que se desea interpolar.
Entonces 2, x, y, 12 deben ser P.H., por lo que 1 , 1 , 1 , 1
2 x y 12
deben ser P.A. y 1 – 1 = 1 – 1 = 1 – 1
12 y y x x 2
1 1
Sean x
=a , y
=b
1 1
–b=b–a=a–
12 2
Tomando dos igualdades:
1 1
–b=b–a fi a – 2b = –
12 12
1 1
b–a=a– fi b – 2a = –
2 2

2
de donde a = 13 y b=
36 9
13 2 2
Luego 1
x
= , 1y = y 1
, 13 , , 1
están en P.A.
36 9 2 36 9 12

Luego los medios armónicos pedidos son


9
x = 36 e y= así
13 2
36 9
2, , , 12 es una P.H.
13 2
5. Encontrar tres números que estén formando una P.A., una
P.G. y una P.H.
Solución.
Sean x, y, z los números buscados, entonces:

z–y =y–x (P.A.)


z y
=
y x
(P.G.)
1 1 1 1
– = –
z y y x
(P.H.)

La solución del problema planteado está en resolver este


sistema:

Sumatoria y progresiones 507

506-507 507 10/11/2001, 11:25


x – 2y + z = 0 x – 2y + z = 0 (1)
y2 – xz = 0 y2 – xz = 0 (2)
xy2 – 2xyz + zy2 = 0 /:y xy – 2xz + yz = 0 (3)

Reemplazando (2) en (3) y dividiendo por y obtenemos la


ecuación (1). De (2) y = x z

Reemplazando en (1)
x – 2 xz + z = 0 fi ( x – z )2 = 0 fi x – z =0
En (1) x + z = 2 xz Hagamos a = x y b = z
x – z = 0

a2 + b2 = 2ab
a–b= 0 fi a=b

2 a2 = 2ab
a= b fi x = z fi x=z
En cualquiera de las ecuaciones del sistema vemos que x = y = z.
Luego cualquier trío de números iguales están P.A. (d = 0), P.G.
(r = 1) y P.H. (sus recíprocos están en P.A.).

Ejercicios (Mezclado P.A., P.G. y P.H.)

1. En cada una de las sucesiones ¿Qué progresión se forma? Determine


siguientes, determine si son P.A., la diferencia. El primer término y el
P.G. o P.H. de lugar 52.

a) 2,
2 2 2
, , , ....
5. Encuentre tres números en P.G. sabiendo
3 9 27 que si se suma 2 al del medio se
b) 1, 0.1, 0.01, 0.001, ... forma una P.A. y que los tres números
sumados dan 28.
c) 3 , 1, 5 , 3 , ....
4 4 2 6. Encuentre tres números en P.A. sabiendo
d) 5, 5, 5, 5, ... que si se resta 2 al del medio se
transforman en P.G. y que los tres
e) a, a , a , a , .... aEN números sumados dan 51.
2 4 8
a–3 a–6 a – 9
f) 3 3 3
, 1, , , .... 7. Estudie la sucesión , ,
a a a
2 4 5
Encuentre el décimo término y la suma
2. En la sucesión x, x2, 4x2 determine
qué valor debe tomar x para que de los diez primeros términos.
ésta sea a) P.A., b) P.G., c) P.H. 8. En una plantación hay 50 filas de
3. Dada la función f(n) = 2n + 5, árboles, si cada fila tiene 1 árbol más
donde n E N, escriba los cinco que la anterior y la fila 12 tiene 38
primeros valores de f(n). ¿Forman árboles, calcule cuántos árboles hay
ellos alguna progresión? en la primera fila, en la última fila y
4. Escriba los cinco primeros términos cuántos árboles hay en la plantación
de la función f(n) = 3n – 2 con n E N. completa.

508 Sumatoria y progresiones

508-509(2003) 508 25/11/02, 1:03 PM


CAPÍTULO 12

9. Un refrigerador costó $ 154.000. toma una copa de 100 cc. y para que
Después de cuatro años se vendió no se note la reemplaza con agua.
en la mitad de su valor. Si esto se Otras personas realizan el mismo
repite cada cuatro años, ¿cuánto procedimiento, quedando cada vez
pagó por él la quinta persona que de inferior calidad el vino. ¿Cuántas
lo compró? personas deberán tomarse 100 cc. de
vino y reemplazarlo por agua para
10. Una garrafa de 5 litros contiene vino que la garrafa contenga 50% de agua
de buena calidad. Una persona se y 50% del vino original?

Soluciones

1 3. 7, 9, 11, 13, 15 P.A. d=2


1. a) P.G. r=
3
b) P.G. r=
1 4. 1, 4, 7, 10, 13 P.A. d=3
10

c) P.A. d= 1 a1 = 1 a52= 154


4
d) P.A. d = 0, P.G., r = 1 5. 4, 8 y 16 6. 9, 17 y 25
1 1 1 1 a–3
P.H. , , , es P.A. d = 0 7. Es una P.A. con a1 = a
, d=– 3
5 5 5 5 a
1 a – 30 10a – 165
e) P.G. r= a10 = a
S10 =
2 a

f) P.H. { 23 , 33 , 43 , 53 , es P.A. d = }
1
3
8. a1 = 27, a50 = 76 S50 = 2.575

2. a) – 1
b) 4 c) 7 9. $ 9.625 10. 6
2 4

Inducción 12.6
matemática

El principio de Inducción matemática es un método


de demostración válido para subconjuntos infinitos de
números naturales.
Se basa en los axiomas de Peano que afirman:

i) 1 E N (El uno es un número natural)

ii) k E N fi (k + 1) E N (si k es un número natural


entonces (k + 1) también lo es)

Sumatoria y progresiones 509

508-509(2003) 509 25/11/02, 1:04 PM


El principio de inducción es el siguiente:
Si P(n) es una proposición asociada al número natural n
y se verifica:
i) P(1) es verdadera, es decir, n = 1 hace verdadera la
proposición.
ii) Siempre que P(k) sea verdadera se cumple que P(k+1)
también lo es.
Entonces la proposición P(n) se cumple para todo número
natural n.

Ejercicios 1. Demostrar por inducción que la suma S de los n primeros


resueltos números naturales es S =
n(n + 1)
2
Solución:
n(n + 1)
Debemos demostrar que 1 + 2 + 3 + ... + n =
2
n
n(n + 1)
Sea P(n): i=
i=1 2
i) Verificamos si se cumple para n = 1
1
1•2
=1 = sí se cumple.
i=1 2
ii) Suponiendo que se cumple para n = k, verificamos si se
cumple para n = k + 1, es decir:
k
k(k + 1)
hip.: i =
i=1 2
k+1
(k + 1)(k + 2)
tesis: i =
i=1 2
k
k(k + 1)
Dem.: i = /sumamos a ambos lados
i=1 2
el término siguiente a k.
k
k(k + 1)
i + (k + 1) = + (k + 1)
i=1 2
k+1
k(k + 1)
i = + (k + 1)
i=1 2
k(k + 1) + 2(k + 1)
=
2
2
k + 2k + 2k + 2
=
2
2
k + 3k + 2
=
2
k+1
(k + 1) (k + 2)
i =
i=1 2

510 Sumatoria y progresiones

510-515(2003) 510 25/11/02, 1:07 PM


CAPÍTULO 12

que es lo que queríamos demostrar; por lo tanto, afirmamos


que para todo número natural n se cumple que la suma S es:
n(n + 1)
y P(n) es verdadera para todo n.
2
2. Demostrar que la suma de los n primeros números impares
es S = n2
Solución:
Por demostrar.
1 + 3 + 5 + ….+ (2n – 1) = n2
n
Sea P(n): (2j – 1) = n2
j=1

i) Verificamos si se cumple para n = 1


1
2j – 1 = 1 = 12 sí se cumple.
j=1

ii) Suponiendo que se cumple para n = k, verificamos si se


cumple para n = k + 1; es decir;
k
hip: 2j – 1 = k2
j=1
k+1
tesis: (2j – 1) = (k + 1)2
j=1

Dem.: Por la hipótesis, tenemos:


k
(2j – 1) = k2 sumamos a ambos lados el
j=1
impar siguiente a 2k – 1, que
es 2k + 1
k
(2j – 1) + (2k + 1) = k2 + (2k + 1)
j=1

k+1
(2j – 1) = k2 + 2k + 1
j=1

k+1
(2j – 1) = (k +1)2
j=1

que es nuestra tesis: entonces por el principio de inducción


afirmamos que la proposición es válida para todo número
natural n.

3. Demostrar que la expresión n2 + 5n es siempre par.

Solución:

Sea P(n): n2 + 5n = 2p para p E N.

i) Verificamos si se cumple para P(1)

n = 1; 12 + 5 • 1 = 6 = 2 • 3 (es par)

ii) Suponiendo que se verifica para n = k, demostremos que


también se cumple para n = k + 1:

Sumatoria y progresiones 511

510-515(2003) 511 25/11/02, 1:07 PM


hip.: k2 + 5k = 2p (para algún p E N)

tesis: (k + 1)2 + 5(k + 1) = 2q (para algún q E N)

En este caso vamos a trabajar con la tesis, de la siguiente


forma:
Debemos demostrar (k + 1)2 + 5(k + 1) = 2q
(k + 1)2 + 5(k + 1) = k2 + 2k + 1 + 5k + 5
= k2 + 5k + 2k + 6

{
= 2p + 2(k + 6) por hipótesis de
inducción
= 2(p + k + 6)
= 2q
Entonces si se cumple para k también se cumple para (k + 1) y
por lo tanto se cumple para todo natural n.
4. Demostrar que se cumple para todo número natural n el
siguiente enunciado.
1 1 1 1 n
+ + + ... + =
1•2 2•3 3•4 n (n + 1) n+1
Solución:
i) Verificamos primero que se cumple la igualdad para
n = 1, es decir,
1 1
= correcta
1•2 2
ii) Suponiendo que se verifica para n = k, debemos demostrar
que se verifica para n = k + 1

1 1 1 k
hip.: + = +…+
1•2 2•3 k (k + 1) k+1
1 1 1 1 k+1
tesis: + +…+ + =
1•2 2•3 k (k + 1) (k + 1)(k + 2) k+2
dem.: Por hipótesis tenemos:
1 1 1 k
+ +… = +
1•2 2•3 k (k + 1) k+1
Sumamos a ambos lados el término siguiente que
1
es :
(k + 1)(k + 2)

1 1 1 1 k
+ +…+ + = +
1•2 2•3 k (k + 1) (k + 1)(k + 2) k+1
1
(k + 1)(k + 2)

512 Sumatoria y progresiones

510-515(2003) 512 25/11/02, 1:07 PM


CAPÍTULO 12

1.Demostrar la inducción que la sumas de los n primeros


1
números naturales 1 1 k (k + 2) + 1
+ es S+=… …+ =
1•2 2•3 (k + 1)(k + 2) (k + 1)(k + 2)
Solución: Debemos demostrar que 1 + 2 + 3+ ….+ n = …..
(sacando
Sea P(n): ….. i = n(n + 1) denominador común)
i)Verificamos si se cumple
k2 + 2k + 1 para n = 1
=
….. = 1 (k=+11)(k x 2 + 2) si se cumple.
(k + 1)2
ii)Suponiendo
= que se cumple(factorizando)
para n = k, verificamos si se
(k + 1)(k
cumple para n = + 1, es decir: + 2)
k+1
hip. …..=i = … k(k(simplifi
+ 1) cando)
k+2
tesis
que es lo que…. debíamos
i = …. (k + 1) ( kpor
demostrar; + 2)lo tanto, afirmamos que
para todo
Dem. ….. i = número natural n se cumple
k(k + 1) /sumamos a ambos la proposición.
lados el término
5. Demostrar
siguiente a k. que para todo entero n, n ≥ 2, se verifi ca:
n
… i + (k + 1) = k(k (1+ 1) + (k>+1 1)
+ a) + na
siendo a un número real distinto de cero y mayor que -1.
…. i = k(k + 1) + (k + 1)
(a E R, a ≠ 0, a > –1)
= k(k + 1) + 2(k + 1)
Solución:
= k2
Sea P(n)+ la + 2k + 2 (1 + a)n > 1 + na
2kdesigualdad
= k2 + 3k
(notemos que+ para
2 n = 1 es falsa, sin embargo debemos verificar
que la +proposición
= (k 1) (k + 2) es verdadera a partir de n = 2)
…i)i n = 2 = (k + 1) (k + 2)
(1 lo
que es a)2 =queríamos
+ que 2 > 1 + 2a
1 + 2a + ademostrar, por lo pues
2
tantoaafi>rmamos
0, ya que
que
a ≠ 0)
para todo número natural n se cumple que la suma S es: n(n + 1)
y y P(n) es verdadera
entonces para todo n.se cumple para n = 2, es decir,
la proposición
P(2) es verdadera.
2.Demostrar que la suma de los n primeros números impares
es S =ii)
n2Suponiendo que se verifica para n = k, debemos demostrar
que se verifica para n = k + 1, es decir, si P(k) es verdadera,
Solución:
entoncesPor demostrar.
también debe serlo P(k + 1).
1 + 2Hip.:
+ 5 +(1….+
+ a) > 1– +
(2n
k 1)ka
= n2
Tesis:….
Sea P(n): + a)–k+1
(1 (2j 1) => n2
1 + (k + 1)a
Dem.: (1 + a) k+1 = (1 + a)k (1 + a) (prop. de exponente)
i)Verificamos si se cumple para n = 1
> (1 + ka) (1 + a) (hipótesis de inducción)
…. 2j – 1 = 1 = 12 si se cumple.
pero observamos que:
ii)Suponiendo que se cumple para n 2= k, verificamos si se
(1 + ka) (1 + a) = 1 + a + ka + ka (multiplicando)
cumple para n = k + 1; es decir
= 1 + a(k + 1) + ka 2 (factorizando)
hip……2j – 1 = k2
> 1 + a (k + 1) (pues ka2 > 0, k > 0
tesis …..(2j – 1) = (k + 1)2 y a ≠ 0)
Por Por
Dem. lo la
tanto, por propiedad
hipótesis, tenemos: transitiva de la desigualdad
concluimos que si
…(2j – 1) = k2 sumamos a ambos lados el impar siguiente
(1 + a)k > (1 + k)a entonces (1 + a)k+1 > 1 + (k + 1)a
a 2k – 1, que es 2k + 1
Y podemos afirmar que la proposición es válida para todo
…(2j – 1) + (2k + 1) = k2 + (2k + 1)
n ≥ 2 (a > –1, a ≠ 0)

Sumatoria y progresiones 513

510-515(2003) 513 25/11/02, 1:08 PM


Ejercicios

Demuestre que los siguientes enunciados son verdaderos para todo entero
positivo n:

1. 3 + 5 + 7 + … + (2n + 1) = n(n + 2)

(3n – 1)
2. 1 + 4 + 7 + … + (3n – 2) =
2
3. 3 + 7 + 11 + … + (4n – 1) = n(2n + 1)

4. 3 + 9 + 15 + … + (6n – 3) = 3n2

n(5n – 1)
5. 2 + 7 + 12 + … + (5n – 3) =
2
6n – 1
6. 1 + 6 + 62 + … + 6n–1 =
5

7. 2 + 6 + 18 + … + 2 • 3n–1 = 3n – 1

1 1 1 n (n + 3)
8. + + ... + =
1•2•3 2•3•4 n(n + 1) (n + 2) 4(n + 1) (n + 2)
n

9. i=1
2i = n(n + 1)

n
i(i + 1) n(n + 1) (n + 2)
10. =
i=1 2 6
n

11. i=1
2i = 2(2n – 1)

n
1 n
12. =
i=1 (2k – 1) (2k +1) 2n + 1
n
2k + 1 1
13. =1–
k=1 [k(k + 1)]2 (n + 1)2
n
n(n + 1) (2n + 1)
14. k2 =
k=1 6
n
n(n + 1)
15. k3 =
k=1 2
16. n3 – n + 3 es divisible por 3.

17. n2 + n es divisible por 2.

18. n2 – n + 2 es divisible por 2.

19. 4n – 1 es divisible por 3.

514 Sumatoria y progresiones

510-515(2003) 514 25/11/02, 1:08 PM


CAPÍTULO 12

20. 5n – 1 es divisible por 4.

21. 42n – 1 es divisible por 3.

22. n3 + 3n2 + 2n es divisible por 6.

23. 10n+1 + 3 • 10n + 5 es divisible por 9.

24. 23n – 1 es divisible por 7.

25. xn – yn es divisible por (x – y).


(sugerencia xk+1 – yk+1 =
xk(x – y) + (xk – yk) y )

26. x2n–1 + y2n–1 es divisible por x + y.

27. n < 2n

28. n2 < 2n para n ≥ 5

29. 2n + 1 < n2 para n ≥ 3.

30. 1 + 2n ≤ 3n

n(n + 1) (2n + 1)2


31. <
2 8
32. 3n ≤ 3 n

Prueba de selección múltiple


1. El término siguiente en A. sólo I D. – a
la P.A. 4, 1, – 2 es: B. sólo II E. 1 – a
A. –1 C. sólo III
B. –2 4. Si 1x , 1y , 1z están en P. H.
D. todas
C. – 3 E. ninguna podemos decir que x,
D. – 4 y, z están
E. – 5 3. En la P.A. siguiente la
diferencia es: A. en P.A.
2. De las siguientes suce-
a a2 2a 2 – a B. en P.G.
siones son P.G. , ,
a–1 a–1 a–1
1 C. en P.H.
III) 1, 1
,1 A.
a
2 4 D. en P.A. y P.G.
III) 1, 2, 4 B. – 1
a E. no forman ninguna
III) 1, – 1 , 1 C. a progresión.
2 4

Sumatoria y progresiones 515

510-515(2003) 515 25/11/02, 1:08 PM


Prueba de selección múltiple
5. La suma de todos los E. 18 E. 3 1 , 10, 30
3
números impares de dos
1 2
cifras es: 9. ,
3 3
y 1 están en P.A., 13. En la sucesión 1, 3 , 9 ,
4 16
A. 2.225 entonces están en P.H. 27
, 81 , ... el término
64 256
B. 2.350 A. 3 2 1 siguiente es:
C. 2.475 B. 3 1,5 1 108
A.
D. 2.525 192
C. 3 2 1,5
243
E. 2.550 B.
D. 2 1,5 1 1.024
6. Dados los números 3 E. 2 1 0,5 C.
324
y 12, ¿qué número se 768
debe intercalar entre 10. La suma de n números 162
D.
ambos para obtener una 192
impares consecutivos,
P.G.? partiendo del 1, es 2.500. 324
E.
1.024
Entonces n vale:
A. 4
14. La suma de los términos
B. 5 A. 10 1 1
de la sucesión 1, , ,
3 9
C. 6 B. 20 1
,... es:
27
D. 8 C. 25
A. 1
E. 10 D. 30 B. 1,5
1 E. 50 C. 2
7. Dados los números y
5 D. 2,5
1 11. El quincuagésimo múl-
, ¿qué número se debe
9 tiplo de 3 es: E. 3
intercalar entre ambos
para obtener una P.H.? A. 141 15. En una P.G. si a5 = 9
y a7 = 1, entonces a6
A. 7 B. 144
vale:
1 C. 147 A. 8
B.
7 D. 150 B. 5
C. 6
E. 153 C. 7
1
D.
6 12. Si el producto de tres D. 3
1 números que están en
E. E. 1
8
P.G. es 1.000 y la razón
8. Si el producto de tres es 3, los tres números 16. La suma de los cincuenta
números en P.G. es 27, son primeros números natu-
¿cuál es el término cen- rales es:
tral? 1
A. 1 , 4, 12 A. 1.025
3
A. 1 B. 2, 6, 18 B. 1.125
B. 3 1 C. 1.275
C. 2 , 7, 21
C. 6 3 D. 1.575
D. 9 D. 3, 9, 27
E. 1.750

516 Sumatoria y progresiones

516-517 516 10/11/2001, 11:43


CAPÍTULO
CAPITULO12
CAPÍTULO 12

1
17. La suma de los prime- C. (2n3 + 6n2 + 4n) B. 4
8
ros veinticinco números
C. 16
pares es: 1
D. (n3 + 3n2 + 2n) D. 64
8
A. 600
E. 81
1
B. 625 E. (2n3 + 6n2 – 4n)
6 25. La suma de infinitos tér-
i = 20
C. 650 21. El valor de Σ i2 + 5 minos de la progresión
es:
i = 10
5, 5 , 5 , 5
, ... es:
2 4 8
D. 1.125
A. 2.640 A. 5
E. 1.275 B. 8
B. 2.650
18. La suma de los prime- C. 9
C. 2.675
ros veinticinco números D. 10
impares es: D. 2.685
E. 12
A. 600 E. 2.690
26. La suma de tres números
22. El producto de los diez en P.A. es 45 . Si al del
B. 625
primeros términos de la 2
3
1 centro se le resta se
C. 650 progresión 1, , 1 , 1 , 2
2 4 8
... es: transforma en una P.G.
D. 1.125 Los números son:
A. 2–15
E. 1.275 A. 3, 15 25
,
B. 2–20 2 12

19. La expresión C. 2–25 15


B. 3, , 12
2
20 D. 2–30
Σ i–1 i + 1 =
i=1
E. 2– 45 C. 3, 6, 12
A. 2.250
23. En la P.A. 7, 10, 13, ... 25
D. 3, 6,
B. 2.500 el término de valor 37 2

ocupa el lugar: 7 15 23
C. 2.650 E. , ,
2 2 2
A. 8
D. 2.700 27. Para que el medio arit-
B. 9 mético sea igual al medio
E. 2.850 geométrico entre dos
C. 10 números a y b debe
20. La suma de los n prime- D. 11 cumplirse que:
ros términos de la serie
1 • 2 + 2 • 3 + 3 • 4 E. 12 A. a + b = 0
+ ... es:
24. El sexto término de una B. a – b = 0
P.G. es 1.024 y la razón
1 C. a + b > 0
A. (2n3 + 6n2 + 4n) es 4. Entonces el tercer
6
término es: D. a – b > 0
1 A. 1
B. (n3 + 3n2 + 2n) E. a – b < 0
6

Sumatoria y progresiones 517

516-517 517 10/11/2001, 11:44


Prueba de selección múltiple
28. Si tres números a, b y veces el medio aritmé- tercero, $ 8 el cuarto y así
c están en P.H. y P.G., tico, entonces: sucesivamente. ¿Cuánto
entonces: A. ab = 4 (a + b)2 habrá ganado al cabo
A. ac (a – 2b + c) = 0 de 25 días?
B. ab = 4a2 + b2
B. ac (a + 2b + c) = 0 A. $ 616.777.216
C. ab = 4 (a2 + b2)
C. ac (a – 2b – c) = 0 B. $ 633.554.431
D. ab = 2 (a + b)2
D. ac (a + 2b – c) = 0 C. $ 267.108.864
E. ab = 2a2 + 2b2
E. ac (– a – 2b – c) = 0 D. $ 134.217.728
30. Una persona gana $ 1
29. Si el medio geométrico el primer día de trabajo, E. $ 268.435.456
entre a y b es cuatro $ 2 el segundo, $ 4 el

Soluciones

1. E 6. C 11. D 16. C 21. D 26. B


2. D 7. B 12. E 17. C 22. E 27. B
3. C 8. B 13. B 18. B 23. D 28. A
4. A 9. B 14. B 19. E 24. C 29. A
5. C 10. E 15. D 20. A 25. D 30. B

518 Sumatoria y progresiones

518 518 10/11/2001, 13:00


CAPÍTULO

nálisis
combinatorio,
13
A
Teorema del binomio y
Elementos de probabilidades
Análisis combinatorio 13.1

13.1.1 Conceptos básicos


Sea n un número natural. Definimos el FACTORIAL de n,
denotado por n! en forma recursiva, por 1! = 1.
(n + 1) ! = n! (n + 1)
Definimos también 0! = 1
Sean n y k dos elementos de N0 tales que n > k. Se define el
n n n!
número por =
k k k! ( n – k) !

Se lee "n sobre k".


• PRINCIPIO DE LA MULTIPLICACIÓN
Supongamos que un suceso puede ocurrir de n maneras y
otro suceso puede ocurrir de m maneras, entonces ambos sucesos
pueden ocurrir de m • n maneras.

• PRINCIPIO DE LA SUMA
Supongamos que un suceso puede ocurrir de m maneras y otro
suceso puede ocurrir de n maneras, entonces hay m + n maneras
en que pueda ocurrir sólo uno de ellos.

13.1.2 Permutaciones
Una permutación de los elementos de un conjunto es
cualquier cambio en el orden de estos elementos sin repetirlos
ni omitirlos.

Análisis combinatorio, Teorema del binomio y Elementos de probabilidades 519

519 519 25/11/02, 1:12 PM


Según el principio multiplicativo, el número de permutaciones
que se pueden efectuar en un conjunto de n elementos es n!
Si denotamos por P (n) el número de permutaciones en un
conjunto de n elementos, tenemos:

(n) = n !P
Vemos que en las permutaciones importa la posición relativa
de los elementos entre sí, por lo tanto, si queremos permutar
en forma circular n elementos, el número de maneras en que
podemos hacerlo es:
Pc (n) = (n – 1) !
(puesto que rotar todos juntos en una dirección no constituye
una permutación diferente).

13.1.3 Arreglos o variaciones


Un arreglo o variación de k elementos tomados de un conjunto
de n elementos (k £ n) es cualquier ordenación que puede hacerse
con esos k elementos.
Notemos que dos arreglos o variaciones diferentes pueden
incluir los mismos elementos, sólo es necesaria una ordenación
distinta.
El número de arreglos o variaciones que pueden efectuarse de k
elementos tomados de un conjunto de n está dado por:

A n =
k
n!
(n – k) !
Ejemplo: En el conjunto {a, e, i, o, u} podemos formar:

V52 = A 52 = (5 5– !2) ! =
5!
3!
=
1
1


2
2


3
3
• 4 • 5
= 20

variaciones diferentes tomando 2 elementos cada vez.


Estos son:
ou, ae, ai, ao, au, ei, eo, eu, io, iu,
uo, ea, ia, oa, ua, ie, oe, ue, oi, ui
(au y ua son dos arreglos diferentes)

13.1.4 Combinaciones
Una combinación de k elementos tomados de un conjunto
de n elementos (k £ n) es cualquier subconjunto que se puede
formar con esos k elementos.
Notemos que al hablar de subconjunto no estamos considerando
el orden en que estén dispuestos los elementos.
Así, dos combinaciones serán distintas si al menos tienen
un elemento distinto.
El número de combinaciones que pueden formarse de k
elementos a partir de un conjunto de n elementos está dado
por:
n n!
Ck = n
k
=
k ! ( n – k) !

520 Análisis combinatorio, Teorema del binomio y Elementos de probabilidades

520-521 520 25/11/02, 1:14 PM


CAPÍTULO 13

Ejemplo: El número de combinaciones de 2 elementos que


pueden formarse a partir del conjunto M = {a, e, i, o, u} es:
• • • •
C 52 = 2 ! ( 55 !– 2) ! =
5!
2! 3!
1
1 •
2
2 •
3
1 •
4
2 •
5
3
= 10

y ellos son:
{a e}, {a i}, {a o}, {a u}, {e i}, {e o}, {e u}, {i o}, {i u}, {o u}
(en este caso {a u} y {u a} son la misma combinación).

1. Calculemos 5 ! y 8 ! Ejercicios
5!
Según la definición: resueltos
5! = 1 • 2 • 3 • 4 • 5 = 120
8! 1•2•3•4•5•6•7•8
= = 336
5! 1•2•3•4•5
2. Disponemos de 3 líneas de buses para viajar de la ciudad A a
la ciudad B y de 5 para viajar de B a C. ¿De cuántas maneras
podemos viajar de A a C pasando por B?
Vemos que cada una de las 3 líneas de A a B las podemos
combinar con cada una de las 5 que hay para viajar de B a C.
Por lo tanto, aplicamos el principio multiplicativo y el total
de posibilidades es 3 • 5 = 15.
3. ¿Cuántos “menús“ diferentes podemos escoger si en el restaurant
se dispone de 5 entradas diferentes, 4 platos de fondo y
6 postres?
Aplicando la regla del producto vemos que la elección del
menú que debe constar de 1 entrada, 1 plato de fondo y 1
postre se puede efectuar de:
5 x 4 x 6 = 120 maneras.
4. ¿De cuántas maneras puedo elegir 2 fichas de colores diferentes
si cuento con 3 fichas rojas, 4 azules y 7 amarillas?
Las fichas pueden ser: roja-azul; roja-amarilla y azul-amarilla.
1 roja y 1 azul se pueden escoger de 3 x 4 = 12 maneras
distintas.
1 roja y 1 amarilla, de 3 x 7 = 21 maneras, y 1 azul y 1 amarilla,
de 4 x 7 = 28 maneras.
En total, aplicando el principio de la suma puedo escoger 2 fichas
de colores diferentes de 12 + 21 + 28 = 61 maneras.
n n
5. Demostremos que k
=
n – k
(n > k)
Apliquemos la definición para obtener cada miembro de
la igualdad:
n n! (1)
=
k k ! ( n – k) !

n n! n! (2)
= =
n – k (n – k) ! (n – (n – k)) ! (n – k) ! k !
y comparando (1) y (2) vemos que la igualdad se cumple.

Análisis combinatorio, Teorema del binomio y Elementos de probabilidades 521

520-521 521 25/11/02, 1:15 PM


6. Determinemos el valor de x de modo que se cumpla:
x
= 10
2
Aplicando la definición tenemos:
x!
= 10
2 ! x –2 !
Esto es:
1 • 2 • 3… x – 3 x – 2 (x – 1) • x
= 10
1 • 2 • 1 • 2 • 3 … x– 3 x – 2
Simplificando nos queda:
x x – 1
= 10 es decir, x2 – x – 20 = 0
2
Las soluciones de esa ecuación son x1 = 5 y x2 = – 4,
pero en este caso no nos sirven las soluciones negativas, por lo
tanto, el valor de x es x = 5.

7. ¿Cuántas “palabras” no necesariamente pronunciables pueden


formarse con las letras de la palabra “VESTIDO”? (no pueden
repetirse las letras ni pueden omitirse).
Se trata de formar permutaciones con 7 elementos, y como
sabemos, el número de ellos está dado por 7 ! = 1 • 2 • 3 •
4 • 5 • 6 • 7 = 5.040
8. ¿Cuántas de las “palabras” obtenidas en el ejercicio anterior
empiezan con V y terminan con O?
Aquí, de las 7 letras de que disponemos, debemos dejar 2 fijas
(la primera y la última) y por lo tanto debemos permutar las otras
cinco restantes. El número de ellas es:
5 ! = 1 • 2 • 3 • 4 • 5 = 120
9. ¿Cuántos números diferentes de 3 cifras se pueden formar
con los dígitos del 1 al 9 si no se permite la repetición de
un dígito?
Nos piden el número de variaciones o arreglos de 3 elementos
tomados de un conjunto de 9. El número está dado por:
9! 9!
V 9
3
=
9 – 3 !
=
6!
= 7 • 8 • 9 = 504

El problema puede ser planteado también de la siguiente


manera:
Debemos “llenar” 3 casilleros y para ello disponemos de 9
elementos.
El primero puede ser llenado con cualquiera de los 9 números una
vez llenado éste, el segundo se puede ocupar con cualquiera de
los ocho restantes, pues no podemos repetir los números y una vez
llenado éste, para el tercero nos quedan 7 posibilidades.
Aplicando el principio de la multiplicación, la cantidad de
números que podemos formar es: 9 • 8 • 7 = 504.

522 Análisis combinatorio, Teorema del binomio y Elementos de probabilidades

522-523 522 10/11/01, 12:34 PM


CAPITULO 13

10. Resolvamos el problema anterior si se permite la repetición


de los dígitos.
Procedamos a “llenar” 3 casilleros con 9 elementos diferentes.
El primero se puede llenar con cualquiera de los 9 elementos;
como se permite la repetición, el segundo se puede llenar también
con cualquiera de los 9 y, por la misma razón, el tercer casillero
también se puede llenar de 9 maneras diferentes.
Por el principio multiplicativo la cantidad de números que se
puede formar está dada por 9 • 9 • 9 = 729.

11. ¿Cuántos subconjuntos de 4 elementos se pueden formar a


partir de un conjunto de 6?
Debemos encontrar el número de combinaciones de 4 elementos
tomados de un conjunto de 6. Este número está dado por:
6 6! 6! 1•2•3•4•5•6
C 4
=
4 ! (6 – 4) !
=
4! 2!
=
1•2•3•4•1•2
= 15

Ejemplo: Consideremos el conjunto A = {a, b, c, d, e, f}. Los


subconjuntos de 4 elementos son:

{a, b, c, d} {a, b, c, e} {a, b, c, f} {a, c, d, e} {a, c, d, f}


{b, c, d, e} {b, c, d, f} {b, d, e, f} {a, d, e, f} {c, d, e, f}
{a, b, d, e} {a, b, d, f} {a, b, e, f} {a, c, e, f} {b, c, e, f}

12. De un total de 15 niños y 6 niñas se desea escoger un grupo de 6.


¿De cuántas maneras puede hacerse esta elección?
En total contamos con 21 niños (entre hombres y mujeres). Como
no hay restricción de ningún tipo debemos formar conjuntos de 6
a partir de 21. El total de maneras está dado por:
21 21 21 ! 16 • 17 • 18 • 19 • 20 • 21
C 6
=
6
=
6 ! 21 – 6 !
=
1 • 2 • 3 • 4 • 5 • 6
= 54.264

13. Resolvamos el ejercicio anterior si el grupo que se desea formar


debe constar de 3 niños y de 3 niñas exactamente.
Debemos elegir 3 niños de un total de 15, esto es:
15 15 13 • 14 • 15
C3= 3
=
1• 2 • 3
= 455

y debemos elegir 3 niñas entre 6.


6 6 4•5•6
C3 = 3
=
1•2•3
= 20

En total el grupo se puede formar de acuerdo con el principio


multiplicativo de:

C 15
3 • C 6
3 = 455 • 20 = 9.100 maneras distintas.

14. Resolvamos el problema anterior con la condición de que en el


grupo debe haber siempre 2 niños fijos y una niña fija (y debe
constar de 3 niños y 3 niñas).

Análisis combinatorio, Teorema del binomio y Elementos de probabilidades 523

522-523 523 10/11/01, 12:34 PM


Como debe haber 2 niños fijos, debemos escoger 1 de un
total de 13.

C 13
1
= 13

Como debe haber 1 niña fija, debemos escoger las otras 2


de un total de 4.
4 3•4
C 4
2
=
2
=
1•2
= 6

y el total de maneras en que se puede formar el grupo con


esas condiciones es:

C 13
1
• C 42 = 13 • 6 = 78

Ejercicios
1. Calcule:
x x x
a) 4 ! b) 10 ! c) 7 ! c) + = 10 d) =3
2 3 2

d) 10 ! e) 15 ! f) 30 ! x x
7! 11 ! 28 ! e) = 15 f) = 28
2 x–2
n! k + 1 !
g) 5 ! 7 ! h) i) x
3! 4! n – 2 ! k – 1 ! x
g) + = 28
5 6
5 12
j) k) l) 10
4 9 3 x x x
h) –2 +3 =0
1 2 3
7 7 5 5
m) + n) +
2 3 3 2 x x x
i) +2 + = 20
1 2 3
6 4 6 5 5
o) + p) +2 –
4 2 3 3 2 x
j) = 36
x–2
6
7! + 8!
3 3. Compare:
n
q) n + 1 : r)
n n–1 6
7
4
• 6 5 5 6
a) + con
3 4 4
5
10 ! + • 9!
2
s) t) 3 ! + 4 ! + 5 ! 9 9 10
5 25 b) + con
8! •
3
• 9 3 4 4

n! + n+1 ! + n+2 ! 15 15 16
u) c) + con
2
n+2 6 7 7
2. Determine el valor de x de modo que
la igualdad se cumpla: 8 8 9
d) + con
4 5 5
x x
a) =7 b) =6
6 2

524 Análisis combinatorio, Teorema del binomio y Elementos de probabilidades

524-525 524 10/11/01, 12:39 PM


CAPITULO 13

4. Demuestre: 13. ¿De cuántas maneras se pueden


ordenar en un estante 12 libros?
n n n+1
+ = 14. ¿De cuántas maneras se pueden
k k+1 k+1
ordenar en un estante 12 libros si
5. Compare: tres de ellos deben estar siempre
juntos?
a) 2 ! + 3 ! + 4 ! con 2 ! 15. Hay 5 libros de química, 4 de física
16 y 6 de matemática. ¿De cuántas
b) 6! + 7! + 8! con 6 ! maneras se pueden ordenar en un
64 estante si deben quedar juntos los
c) 4! + 5! + 6! con 4! libros de cada tema?
36 16. ¿Cuántas “palabras” se pueden formar
d) 8! + 9! + 10! con 8! con las letras de la palabra LIBRO?
100 17. ¿Cuántas de las palabras del ejercicio
6. Demuestre: anterior empiezan con consonante?
n! + n + 1 ! + n + 2 !
18. ¿Cuántas de las palabras del ejercicio
= n! 16 terminan con o?
2
n + 2 19. ¿Cuántas palabras de 4 letras se
7. Compare: pueden formar con las letras de la
palabra CLAUDIO si no se pueden
5 4 5 repetir?
a)
3
: 2
con
3 20. ¿Cuántos números de 4 cifras se
pueden formar con los dígitos 0, 1, 2,
7 6 7 ..., 9 si deben empezar con 4 y no se
b) 5 : 4
con
5 pueden repetir los dígitos?
21. Ejercicio 19: ¿si se permite la repe-
c) tición?
3 2
1
: 0
con 3 22. ¿Cuántos números pares de 2 cifras
se pueden formar con los dígitos
{0, 1, 2, 3, 4} si no se permite la
d) 9 8 9 repetición?
7
: 6
con
7 (Recuerde, el número 04 no se
considera de 2 cifras).
8. Demuestre:
23. Ejercicio 20: ¿si se permite la repe-
n + 1 n n + 1 tición?
n – 1
: n – 2
=
n – 1 24. ¿Cuántas permutaciones pueden
hacerse con las letras de la palabra
9. ¿Cuántos números diferentes de 5 PINCEL?
cifras se pueden formar con los dígitos 25. ¿Cuántas de las permutaciones del
1, 2, ..., 9 sin repetir? ejercicio anterior comenzarán con P?
10. ¿De cuántas maneras se pueden ubicar 26. ¿Cuántas de las permutaciones del
6 personas en una fila? ejercicio anterior comenzarán con P
11. ¿De cuántas maneras se pueden ubicar y terminarán con L?
6 personas en una mesa redonda? 27. ¿Cuántas palabras pueden formarse
12. ¿De cuántas maneras se pueden sentar con 24 consonantes y 5 vocales si
en una mesa redonda 3 matrimonios cada una debe estar formada por
si cada esposo debe tener a su lado 3 consonantes y 3 vocales y no se
a su mujer? permite la repetición?

Análisis combinatorio, Teorema del binomio y Elementos de probabilidades 525

524-525 525 10/11/01, 12:39 PM


28. ¿Cuántas palabras pueden formarse 39. Ejercicio 38: ¿si del total de niños se
con 24 consonantes y 5 vocales si excluyen 8 niños y 5 niñas?
cada una debe estar formada por 3
consonantes y 3 vocales y deben ir 40. Ejercicio 38: ¿si deben estar incluidos
intercaladas? siempre 2 niños y 1 niña?

29. ¿Cuántas “patentes” se pueden formar 41. ¿Cuántos grupos de 5 personas se


si éstas constan de 2 letras seguidas de pueden formar entre 4 niños y 7
4 dígitos y no se permite la repetición? niñas si debe haber por lo menos 2
(Considere 27 letras y 10 dígitos) niñas incluidas?

30. Ejercicio 29: ¿si se permite la repeti- 42. Ejercicio 41: ¿si debe haber a lo más
ción? dos niñas incluidas?

31. ¿Cuántos números de 5 dígitos se 43. En un curso hay 19 niñas y 21 niños.


pueden formar si la cifra de las ¿De cuántas maneras se pueden
unidades es 5 y la de la unidad de mil formar grupos de 5 personas que
es 3 y no se permite la repetición? incluyan:
a) 2 niñas y 3 niños.
32. ¿De cuántas maneras se pueden
b) 3 niñas y 2 niños.
seleccionar 5 discos de un total de
15? c) 4 niñas.
d) 4 niños.
33. ¿De cuántas maneras se pueden
seleccionar 3 personas de un total 44. En un curso hay 15 niños y 16 niñas
de 8? y se desea seleccionar un grupo de
14 alumnos. De cuántas maneras se
34. ¿De cuántas maneras se pueden puede hacer esta selección si:
ordenar 8 hermanos en una fila si el a) no hay restricción.
menor debe estar en primer lugar y el
b) debe haber 7 niños y 7 niñas.
mayor en último lugar?
c) hay 5 niños y 5 niñas excluidos.
35. ¿De cuántas maneras se pueden d) hay 4 niños y 5 niñas incluidos.
ordenar 12 libros en un estante si
2 de ellos deben quedar siempre 45. La primera división de la Liga de
separados? Fútbol consta de 25 equipos. ¿Cuántos
partidos deben jugarse para completar
36. ¿Cuántos colores diferentes se pueden la primera rueda?
obtener mezclando 2 cada vez de
un total de 12? 46. En una sala de clases hay 22 sillas. ¿De
cuántas maneras se pueden ubicar
(Se supone que la mezcla se hace 15 alumnos?
siempre en la misma proporción).
47. ¿Cuántos números menores o iguales
37. De un total de 16 niños y 12 niñas de 3.000 y múltiplos de 5 se pueden
se desea escoger un grupo de 6. ¿De formar con los dígitos 0, 1, 2, 3,
cuántas maneras puede hacerse esta 4, 5?
elección?
48. ¿De cuántas maneras se puede formar
38. Ejercicio 37: ¿si el grupo debe constar un comité de 3 personas de un total
de 3 niños y 3 niñas? de 8?

526 Análisis combinatorio, Teorema del binomio y Elementos de probabilidades

526-527 526 25/11/02, 1:16 PM


CAPÍTULO 13

49. De un total de 8 personas, ¿de cuántas 55. ¿Cuántas diagonales tiene un octá-
maneras pueden ocupar 3 cargos gono?
diferentes?
56. ¿Cuántas diagonales tiene un polígono
50. De un total de 8 personas, ¿de cuántas de n lados?
maneras se puede formar un comité
de 4 personas si una de ellas debe 57. Se dispone de 10 puntos donde no
estar siempre incluida? hay 3 colineales. ¿Cuántos triángulos
51. En una liga de 12 equipos de fútbol, se pueden formar?
¿cuántos partidos deben jugarse si
58. De un total de x personas se pueden
cada equipo juega 2 veces con cada
formar 10 grupos de 2. Determine x.
uno de los restantes?
52. ¿De cuántas maneras se pueden 59. De un total de x personas se pueden
seleccionar 3 libros de un total de formar 21 grupos de 5. Determine x.
10?
60. ¿Cuántos números pares de 3 cifras,
53. ¿Cuántas diagonales tiene un pentá- entre 400 y 700, se pueden formar si
gono? no se permite la repetición?
54. ¿Cuántas diagonales tiene un hexá-
gono?

Soluciones

1. a) 24 b) 3.628.800 c) 5.040 9. 9 • 8 • 7 • 6 • 5 = 15.120 10. 6! = 720


d) 720 e) 32.760 f) 870
11. 5! = 120 12. 2! • 23 = 16
g) 4.200 h) n (n – 1) i) k (k + 1)
j) 5 k) 220 l) 120 13. 12! 14. 3! 10!
m) 56 n) 20 o) 21
15. 5! 4! 6! 3! 16. 5! = 120
p) 30 q) n + 1 r) 224
n
17. 3 • 4 • 3 • 2 • 1 = 72 18. 4! = 24
s) 2 t) 3! u) n!
19. 4 • 5 • 6 • 7 = 840 20. 9 • 8 • 7 = 504
2. a) x=7 b) x = 4 c) x = 4
d) x=3 e) x = 6 f) x = 8 21. 7 • 7 • 7 • 7 = 2.401
g) x=7 h) x = 3 i) x = 4
j) x=9 22. 10 (22 y 44 están excluidos)

3. a) ambos son iguales a 15 23. 1.000 24. 6! = 720


b) ambos son iguales a 210
25. 5! = 120 26. 4! = 24
c) ambos son iguales a 11.440
d) ambos son iguales a 126 27. 24 •
5
• 6! = 14.572. 800
3 3
5. a) son iguales b) son iguales
c) son iguales d) son iguales 28. 2 • (24 • 23 • 22 • 5 • 4 • 3) = 1.457.280

7. a) son iguales b) son iguales 29. 27 • 26 • 10 • 9 • 8 • 7 = 3.538.080


c) son iguales d) son iguales
30. 27 • 27 • 10 • 10 • 10 • 10 = 7.290.000

Análisis combinatorio, Teorema del binomio y Elementos de probabilidades 527

526-527 527 25/11/02, 1:16 PM


31.7 • 7 • 6 = 294 32. (154) = 3.003 33. ( 83 ) = 56 34. 6! = 720

36. ( ) = 66 (286) = 376.740


12
35.12! – 11! • 2 = 399.168.000 37.
2
38. (163) (123) = 123.200 39. ( 83 ) ( 73 ) = 1.960 40. ( ) ( ) = 770
14
1

11
2

41. 2 m y 3 h. Q ( ) ( ) = 84
7 4
2 3
3 m y 2 h. Q ( ) ( ) = 210
7 4
3 2
4 m y 1 h. Q ( ) ( ) = 140
7 4 84 + 210 + 140 + 21 = 455
4 1
Q ( )
7
5m = 21
5
42. 1 m y 4 h. Q ( ) ( ) =
7 4
7
1 4
7 + 84 = 91
2 m y 3 h. Q ( ) ( ) = 84
7 4
2 3

43. a) ( ) ( ) = 227.430
19 21
47. 1 dígito – 1
2 3
2 dígitos – 10
b) ( ) ( ) = 203.490
19 21 1 + 10 + 60 + 145 = 216
3 2 3 dígitos – 60
4 dígitos – 145
c) ( ) ( ) =81.396
19 21
4 1
48. ( ) = 56
8
49. 8 7 6 = 336
d) ( ) ( ) = 113.715
19 21 3
• •
1 4
50. ( ) = 35
7
51. 11 12 = 132
44. a) ( ) b) ( ) ( )
31 15 16 3

14 7 7
52. ( ) = 120
10
53. 5
c) ( ) d) ( )
21 22 3
14 5
25 • 24
54. 9 55. 20
45. = 300
2 n n–3

46. V 22
15
56.
2
57. (103) = 120
58. x = 5 59. x = 7 60. 104

13.2 Teorema
del binomio

13.2.1 Conceptos y
observaciones básicas
En el punto anterior nos familiarizamos con la expresión ( nk )
que leíamos "n sobre k"; nos será de gran utilidad en esta sección
donde desarrollaremos cualquier potencia de binomio sin hacer
multiplicaciones tediosas.

La expresión
n
k ( )
recibe el nombre de coeficiente binomial.
Desarrollando algunas potencias de binomios tenemos:
528 Análisis combinatorio, Teorema del binomio y Elementos de probabilidades

528-529 528 10/11/2001, 19:11


CAPITULO 13

(a + b)0 = 1
(a + b)1 = a + b
(a + b)2 = a2 + 2ab + b2
(a + b)3 = a3 + 3a2 b + 3 ab2 + b3
(a + b)4 = a4 + 4a3 b + 6 a2 b2 + 4 ab3 + b4

Observaciones.

1. El desarrollo de (a + b)n tiene n + 1 términos.


2. El primer término es an y el último es bn.
3. El exponente de a decrece mientras el de b aumenta en
1 unidad.
4. La suma de ambos exponentes es siempre constante, igual
a n.
5. Los coeficientes de los términos equidistantes del centro
son iguales.
6. Si comparamos los coeficientes de los términos obtenidos
en los desarrollos con los coeficientes binomiales, podemos
ver que son iguales.
Ejemplo:

n = n =1
0 n

n = n =n
1 n–1

n = n n n–1
2 n–2
= ... etc.
2

Con estas observaciones estamos en condiciones de formular


el teorema del binomio.

13.2.2 Teorema del binomio


El desarrollo de la potencia n–ésima del binomio (a + b)
corresponde a:
n
(a + b)n = Σ
k=0
n
k an – k bk

en forma extensiva;

(a + b)n = n an + n1 an – 1 b + n2 an – 2 b2 + ...
0

+ ... + n
n–1 abn – 1 + nn bn
Ejemplo:
Desarrollemos (a + b)4 aplicando el teorema:

(a + b)4 = 4 a4 + 4 a3 b + 4 a2 b2 + 4 ab3 + 44 b4
0 1 2 3

Análisis combinatorio, Teorema del binomio y Elementos de probabilidades 529

528-529 529 10/11/2001, 19:12


siendo: 4 = 4 = 1
0 4
4
1
= 4 =4
3
4 =6
2
entonces:
(a + b)4 = a4 + 4 a3 b + 6a2 b2 + 4 ab3 + b4

Si queremos determinar el término de orden r (o lugar r)


en el desarrollo de (a + b)n, ése está dado por:
n
tr = r–1
an – r + 1 br – 1
es decir, se obtiene haciendo k = r – 1
Por ejemplo, el tercer término de (a + b)6 está dado por:

t3 = 6 a4 b2 (k = 2)
2
Recordemos que en el desarrollo del binomio, (a + b)n
k varía desde 0 hasta n;
Así, el primer término se obtiene para k = 0
2º término se obtiene para k = 1
3º término se obtiene para k = 2
4º término se obtiene para k = 3
rº término se obtiene para k = r – 1
En forma más general, podemos expresar:
tk + 1 = n an – k bk
k

13.2.3 El triángulo de Pascal


Los coeficientes de los términos de un binomio también se
pueden obtener construyendo el siguiente ordenamiento, conocido
como triángulo de Pascal-Tartaglia:

n= 0
n= 1
n= 2
n= 3
n= 4
n= 5

La construcción de este triángulo se efectúa partiendo y terminando


con 1 y cada término se obtiene sumando en la fila anterior los dos
números más próximos.

530 Análisis combinatorio, Teorema del binomio y Elementos de probabilidades

530-531 530 25/11/02, 1:18 PM


CAPITULO 13

Así, para n = 6 hacemos:

1 6 15 20 15 6 1

y, por lo tanto, si queremos obtener (a + b)6, simplemente


completamos los factores literales de cada término tomando en
cuenta las consideraciones anteriores: Así:

(a + b)6 = a6 + 6a5b + 15a4b2 + 20a3b3 + 15a2b4 + 6ab5 + b6

1. Desarrollemos (2a + 3b)4 Ejercicios


Sabemos que el desarrollo tiene 5 términos y, de acuerdo al
triángulo de Pascal (o usando los coeficientes binomiales), los resueltos
coeficientes son 1 4 6 4 1.
Ahora, el primer término es 2a y el segundo es 3b
El desarrollo es:
(2a + 3b)4 = 1 (2 a)4 + 4 (2 a)3 • (3 b) + 6 (2 a)2 • (3 b)2 + 4
(2 a)1 • (3 b)3 + (3 b)4
completando el desarrollo, nos queda:
(2 a + 3 b)4 = 16 a4 + 96 a3 b + 216 a2 b2 + 216 a b3 + 81 b4
2. Determinemos el 6º término en el desarrollo de (a + 2b)11

Sabemos que: t6 = 11 a6 (2b)5


5
t6 = 462 • a6 • 32 b5
t6 = 14.784 a6 b5
3. Cuál es el coeficiente de x14 en el desarrollo de (x2 + x3)6.
Sabemos que el término general está dado por:

n
tk = k – 1 (x2)n – k + 1 (x3)k – 1

en este caso:

6
tk = k – 1 (x2)6 – k + 1 (x3)k – 1

6
= k – 1 x14 – 2k x3k – 3

6
= k – 1
xk + 11

queremos el coeficiente de x14, es decir, debemos igualar


exponentes:
xk + 11 = x14
o sea k=3
y el término que contiene a x14 en ese desarrollo es el tercero
y su coeficiente es:
6 = 15
2

Análisis combinatorio, Teorema del binomio y Elementos de probabilidades 531

530-531 531 25/11/02, 1:19 PM


4. Determinemos el exponente de x en el 8º término del desarrollo
de:
10
1
x3 +
x2
7
t8 = 10 3 1
x3 •
7 x2

= 10 x9 • 1
7 x14
= 120 x–5
y el exponente de x en el 8º término es – 5.
5. Determinemos el término independiente de x en el desarrollo
de:
6
1
– x4
x2

El término general es:


6 – k
1
t k + 1= 6 • – x4 k
k x2
6 – k
1
= 6 • – 1 k
• x4 k
k x2
1
= 6 – 1 k
• • x4 k
k x12 – 2 k

= 6 – 1 k
• x6 k – 12
k
para que el término sea independiente de x es necesario que el
exponente de x sea igual a 0.
es decir: 6k – 12 = 0
k=2
y para k = 2 tenemos:
4
6 1 • – x4 2
t3 = 2 x2

1
= 62 • – 1 2 • x8
x8

= 6
2

= 15
el término independiente de x en el desarrollo es el 3er término
y es igual a 15.

Ejercicios

1. Obtenga los siguientes desarrollos: c) (1 + 3a)7 d) (1 – b)11

a) (x + y)5 b) (x – 2y)5 e) (2a – 3b)4 f) (x – 1 )6


x

532 Análisis combinatorio, Teorema del binomio y Elementos de probabilidades

532-533 532 10/11/01, 12:49 PM


CAPITULO 13

7
h) penúltimo término de a2 – a
4 8
g) 12 + z2 h) m – 1 3
z 2 2
6
3 x3 6 i) último término de – 1)11(3x11
p 2q
i) – j) –
j) término central de (1 – 5x)6
2q p x4 4
2. Determine el término indicado en el 4. En el desarrollo de 1 5
deter-
3 x2 –
desarrollo correspondiente: x
mine:
a) 7º término en (x – y)11
a) el coeficiente numérico del cuarto
b) 5º término en (a + b)21 término
10
c) 10º término en 1 1 b)el término que contiene x4
a –b
2 7 c) el término independiente de x
d) 8º término en x2 y –
x y2 5. En el desarrollo de
e) 11º término en (2a – b)10 2
– 3 x y3
12

x2 y
determine:
4
1
f) 2º término en 1 – xyz
a) término general
11
g) 9º término en 2 2 +3 3
b) término que contiene x– 3
6
h) 4º término en 2 – 3 c) término que contiene y12
9
i) 6º término en 3 d) término independiente de x
a –a

x5 2
15 e) término independiente de y
j) 13º término en 2
– 5
x 6. Determine el término que contiene q9
3. Determine el coeficiente numérico en los siguientes desarrollos:
del término indicado:
a) (2p + q)11
a) 2ºtérmino en (2x – y)4
10
b) 3ertérmino en (3a + 4b)6 b) 1
q – pq
10
c) 9ºtérmino en x2 y2
y

x
c) (p2 – q3)7
d) 5ºtérmino en (– a + 12)5 1 3
d) 3 q5 – q
e) 8ºtérmino en (p2v2 – 1)14
f) término central (2x2 y + xy3 )8
17 7. Encuentre los 3 primeros términos en
2 1
g) último término – 10
a2 b2 ab el desarrollo de 2 x + 3

Soluciones

1. a) x5 + 5x4 y + 10x3 y2 + 10x2 y3 + 5xy4 + y5

b) x5 – 10x4 y + 40x3 y2 – 80x2 y3 + 80xy4 – 32y5

c) 1 + 21a + 189a2 + 945a3 + 2.835a4 + 5.103a5 + 5.103a6 + 2.187a7

d) 1 – 11b + 55b2 – 165b3 + 330b4 – 462b5 + 462b6 – 330b7 + 165b8 – 55b9 + 11b10 – b11

e) 16a4 – 96a3b + 216a2b2 – 216ab3 + 81b4

Análisis combinatorio, Teorema del binomio y Elementos de probabilidades 533

532-533 533 10/11/01, 12:50 PM


f) x6 – 6x4 + 15x2 – 20 + 15 – 6 + 1
2 4 6 x x x
g) 1 + 4 + 6 + 4z4 + z8
z8 z4
h) 1 (m8 – 8m7 + 28m6 – 56m5 + 70m4 – 56m3 + 28m2 – 8m + 1)
256
p6 3 p4 15 p2 q2 q4 q6
i) – + – 20 + 60 – 96 + 64
64 q6 8 q4 4 q2 p2 p4 p6
18
j) 729 – 729 + 1.215 – 135 + 135 x4 – 9 x11 + x
x24 2 x17 16 x10 16 x3 256 512 4.096

2. a) a7 = 462x5 y 6 h)
7
i) –1 j) – 2.500
729
b) a5 = 5.985a17 b4 4. a) – 90 b) a3 = 270x4
– 10 c) no existe
c) a10 =
a b9
– 128 5. a) ak + 1 = 12 212–k • (– 3)k • x3k – 24• y4k – 12
d) a8 = k
x7 y14
b) a8 = – 12 37 • 25 x–3 y16
e) a11 = b10 7
–4 c) a7 = 12 26 • 36 x– 6 y12
f) a2 = xyz 6
g) a9 = 2.640 • 312 2 d) a9 = 12 24 38 y20
8
h) a4 = – 120 6
e) a4 = – 12 29 33 x–15
3
i) a6 = – 30.618 6. a) a10 = 220 p2q9
a

j) a13 = 232.960 b) no hay


x45
3. a) –32 b) 19.440 c) 45 c) a4 = – 73 p6 q9

d) – 5 • 124 e) –3.432 d) a2 = – 27 q9

f) 1.120 g) –1 7. 32x10 + 160 6 x9 + 2.160 x8

13.3 Elementos de
probabilidades

13.3.1 Conceptos básicos


Se llama experimento determinístico a aquel en el cual el
resultado se puede predecir, es decir, siempre que se realice en
condiciones semejantes se obtendrá el mismo resultado.
Un experimento aleatorio es aquel en el cual no es posible
predecir el resultado aunque éste se realice en las mismas
condiciones.
Al conjunto de resultados posibles de obtener a partir de un
experimento aleatorio se le llama Espacio muestral.

534 Análisis combinatorio, Teorema del binomio y Elementos de probabilidades

534-535 534 10/11/01, 12:53 PM


CAPITULO 13

Cualquier subconjunto del Espacio muestral se denomina


Evento o suceso. Se pueden clasificar en:
a) evento cierto o seguro es aquel que está formado por
todo el Espacio muestral.
b) evento imposible es el subconjunto vacío del espacio
muestral.
c) eventos incompatibles o mutuamente excluyentes son
aquellos que no pueden suceder simultáneamente.
d) eventos complementarios son aquellos cuya unión es
el Espacio muestral y cuya intersección es el conjunto
vacío.

La probabilidad de que un evento o suceso ocurra es el


cociente entre el número de casos favorables y el número
de casos posibles.

Así, si la probabilidad de que ocurra el suceso A, designada


P (A), está dada por:
Nf
P A =
Np
donde Nf denota el número de casos favorables y
Np denota el número de casos posibles de ocurrencia
del experimento.
Observaciones.
1. La probabilidad de un evento cierto es 1.
2. La probabilidad de un evento imposible es 0.
3. Para cualquier evento A, se tiene que la probabilidad de
A, denotada P (A), es:
0 £ P (A) £ 1
4. Si A y B son eventos complementarios entonces:
P (A) = 1 – P (B)

13.3.2 Probabilidad de la unión y


de la intersección de dos eventos
Sean A y B dos eventos del Espacio muestral E.
Si A y B son mutuamente excluyentes, entonces la probabilidad
de que ocurra la unión de los dos está dada por:
P (A K B) = P (A) + P (B)
Si A y B no son mutuamente excluyentes entonces la
probabilidad de ocurrencia de A K B está dada por:
P (A K B) = P (A) + P (B) – P (A J B)

Análisis combinatorio, Teorema del binomio y Elementos de probabilidades 535

534-535 535 10/11/01, 12:53 PM


Ahora, si A y B son eventos independientes, la probabilidad de
ocurrencia de A J B está dada por:

P (A J B) = P (A) • P (B)

Si A y B no son eventos independientes, es decir, la ocurrencia


de uno de ellos, por ejemplo de A, influye sobre la ocurrencia del
otro, en este caso de B, entonces la probabilidad de ocurrencia
de A J B está dada por:

P (A J B) = P (A) • P (B | A)

donde P (B | A) es la probabilidad de ocurrencia de B condicionada


por la ocurrencia de A.

Ejercicios 1. Calculemos la probabilidad de que al lanzar una moneda


resueltos salga cara.
Al lanzar una moneda tenemos dos resultados posibles: CARA -
SELLO. Ese es entonces nuestro Espacio muestral. La cantidad de
elementos del Espacio muestral nos indica el número de casos
posibles Np. El número de casos favorables en este ejemplo
es uno solo: sale cara.
Así, la probabilidad pedida es:

1 nº casos favorables
P=
2 nº casos posibles
2. Calculemos la probabilidad de que al lanzar un dado nos salga
un número menor que 4.
Al lanzar el dado podemos obtener 6 resultados distintos. El
Espacio muestral es E = {1, 2, 3, 4, 5, 6}.
Los casos favorables son 3: {1, 2, 3}
La probabilidad pedida entonces es:
3 1
P= =
6 2

3. Al lanzar un dado, ¿cuál es la probabilidad de obtener un número


par o bien obtener un número menor que 3?
Sea E el Espacio muestral: E = {1, 2, 3, 4, 5, 6}
Es decir, el Nº de casos posibles es Np = 6
Sea A el evento: A = obtener número par.
Es decir, A = {2, 4, 6} Q NA = 3
Sea B el evento: B = obtener número menor que 3.
B = {1, 2} Q NB = 2
Vemos que A J B = {2} Q N (A J B) = 1

536 Análisis combinatorio, Teorema del binomio y Elementos de probabilidades

536-537 536 10/11/01, 12:54 PM


CAPITULO 13

Claramente A y B no son mutuamente excluyentes, entonces la


probabilidad de que ocurra A K B es:
P(AKB) = P(A) + P(B) – P(A J B)
3 + 2 – 1
=
6
P(AKB) = 2
3
4. Calculemos la probabilidad de que al lanzar 3 monedas,
obtengamos una cara y dos sellos.
Cada moneda origina 2 resultados posibles: cara o sello; como
son 3 monedas, el Espacio muestral consta de 8 elementos. Tres
de esos ocho elementos cumplen la condición pedida, así la
probabilidad buscada es 3 .
8

Ejercicios
1. Determine cuáles de los siguientes comprando 4 números, si hay 10 listas,
experimentos son predeterminados cada una con 20 números?
(o determinísticos) y cuáles son 4. ¿Cuál es la probabilidad de obtener
aleatorios. siete puntos en el lanzamiento de
a) jugar una cartilla de apuesta dos dados?
deportiva 5. ¿Cuál es la probabilidad de no obtener
b) mezclar azúcar y agua número par en el lanzamiento de
c) enfriar agua a 0º C un dado?

d) lanzar una piedra y medir 6. Al lanzar dos monedas, qué probabi-


su alcance lidad hay de:
e) comprar un número de rifa a) obtener dos caras
f) apostar en una carrera de caballos b) obtener 1 cara y 1 sello
g) preguntarle a un desconocido c) obtener lados iguales
si fuma 7. En el lanzamiento de 1 dado, cuál es
2. Señale el Espacio muestral de los la probabilidad de:
siguientes experimentos: a) obtener el Nº 5
a) lanzar una moneda b) no obtener el Nº 5
b) lanzar dos monedas c) obtener 3 o 5
c) lanzar 3 monedas d) obtener número menor que 5
d) lanzar 1 dado
8. En el lanzamiento de dos dados, cuál
e) lanzar 2 dados es la probabilidad de que:
f) asignar el premio en una rifa para a) la suma sea 11
la cual hay 10 listas con 10 números
b) la suma sea mayor que 9
cada una
c) la suma sea menor que 4
3. ¿Cuál es la probabilidad de ganar
uno de los 5 premios de una rifa d) no salgan números iguales

Análisis combinatorio, Teorema del binomio y Elementos de probabilidades 537

536-537 537 10/11/01, 12:55 PM


9. Al lanzar 1 dado dos veces consecu- 15. En una caja hay 10 bolitas rojas y 6
tivas, qué probabilidad hay de: azules. ¿Cuál es la probabilidad de
obtener 3 bolitas azules en 3 sacadas
a) obtener 2 ases
diferentes sin reposición?
b) obtener primero un 3 y luego un
número par 16. En un grupo de 30 personas todos
tienen edades diferentes menores
c) obtener primero un 3 y luego no
de 50 años. Si se integra una nueva
obtener 3
persona, también menor de 50, ¿cuál
d) obtener número par primero y el es la probabilidad de que su edad
3 después coincida con la de alguno de los
30?
10. En un naipe inglés (52 cartas) qué
probabilidad hay de: 17. Hay 150 números en una rifa. ¿Cuántos
a) obtener un trébol al sacar una habrá que comprar para tener un 8%
carta de probabilidad de ganarla?
b) obtener dos ases en una “entrega” 18. ¿Qué probabilidad hay de que al
(13 cartas) lanzar 2 dados se obtenga una suma
11. En una caja hay 12 bolas negras y 8 menor que 6?
rojas, qué probabilidad hay de:
19. Hay 16 monedas de $ 100, 22 monedas
a) sacar 1 negra de $ 50 y 12 monedas de $ 10. Al sacar
b) sacar 1 roja una moneda, ¿cuál es la probabilidad
de sacar una de $ 100 o de $ 50?
c) sacar 1 negra y, sin reponerla, sacar
luego una roja 20. En un curso la mitad de los alumnos
d) sacar 1 negra y luego de reponerla, son hombres. Si el 40% de los hombres
sacar una roja. sabe inglés y el 50% de las mujeres,
francés, ¿cuál es la probabilidad de
12. Se lanza un dado y sale 4. ¿Qué que al elegir un alumno sepa inglés
probabilidad hay de que al lanzarlo de o francés?
nuevo sume con el primer resultado
un número menor que 9? 21. En un grupo de 100 personas, 40
gustan de la música, 30 gustan del
13. Al comprar clavos, la probabilidad deporte, 10 gustan de la música y
de obtener 1 defectuoso es de 0,015. del deporte y los otros no gustan
¿Cuántos clavos “defectuosos” habrá ni de la música ni del deporte. Al
en un paquete que contiene 10 cajas y elegir una persona al azar, cuál es la
cada caja contiene aproximadamente probabilidad de que:
40?
a) guste sólo de la música
1 b) guste sólo del deporte
14. En un curso de 60 alumnos, de
3
los alumnos habla inglés, 1 habla c) guste de la música o del deporte
4
1
francés y habla los dos idiomas. d) no guste ni de la música ni del
10
deporte
¿Cuál es la probabilidad de que un
alumno elegido al azar hable sólo e) guste de ambos.
un idioma?

538 Análisis combinatorio, Teorema del binomio y Elementos de probabilidades

538-539 538 10/11/01, 12:56 PM


CAPÍTULO13
CAPITULO 13

Soluciones

1. a) aleatorio 1
b) predeterminado 4. o 16, 6 %
6
c) predeterminado 1
d) aleatorio 5. o 50%
2
e) aleatorio 6. a) 1 b) 1 c) 1
f) aleatorio 4 2 2
g) aleatorio 1 5
7. a) b)
6 6
2. a) E = {cara, sello} c) 1 d) 2
3 3
b) E = {cara cara; cara sello; sello cara; 1 1
sello sello} 8. a) b)
18 6
c) E = {cara cara cara; cara cara sello; c) 1
d) 5
cara sello cara; cara sello sello; sello 12 6
1
cara cara; sello cara sello; sello sello 9. a) b) 1
cara; sello sello sello} 36 12

d) E = {1, 2, 3, 4, 5, 6} c) 5 d) 1
36 12
e) E = {1-1, 1-2, 1-3, 1-4, 1-5, 1-6
4 : 48
2-1, 2-2, 2-3, 2-4, 2-5, 2-6 2 11
3-1, 3-2, 3-3, 3-4, 3-5, 3-6 10. a) 1 b)
4 52
4-1, 4-2, 4-3, 4-4, 4-5, 4-6
13
5-1, 5-2, 5-3, 5-4, 5-5, 5-6 11. a) 3 b) 2
6-1, 6-2, 6-3, 6-4, 6-5, 6-6} 5 5
24 6
f) E = {L1,1; L1,2; L1,3 ...; L1,10 c) d)
95 25
L2,1; L2,2; L2,3 ...; L2,10 2
12. 13. 6 14. 23
3 60

3
• 15. 1 16. 17. 12 números
28 5
• 5 19
18. 19.
L10,1; L10,2; L10,3 ...; L10,10} 18 25
4 196 20. 45% o 9
1 4 20
3. 21. a) 30% b) 20% c) 60%
200
5 d) 40% e) 10%

Prueba de selección múltiple


11! 7 + 7 =
1. Si 2x = 10, entonces el 2. = 3.
2! 9! 2 3
valor de x es: A. 2! B. 2 7 14
A. 5 B. 5
A. 5 B. 4
C. 55 D. 110 8
C. 3 D. 6 C. 4 D. 8
3
E. 10 E. 220 E. Otro

Análisis combinatorio, Teorema del binomio y Elementos de probabilidades 539

538-539 539 10/11/01, 12:58 PM


Prueba de selección múltiple
4. ¿De cuántas maneras se 8. El coeficiente numérico 11. El término indepen-
pueden escoger 3 libros del 2º término en el diente de x en el desa-
4
de un total de 9? desarrollo de (2 a + b)5 1
es: rrollo de x – es:
A. 27 x2
B. 504 A. 16 A. 2º

C. 84
B. 32 B. 3º
D. 12
C. 80
E. 729 C. 4º
D. 10
7 6 = D. Último
5.
5 : 4 E. 50
7
A. 9. El término central en el E. No hay término in-
5
y dependiente de x
7 desarrollo de (3x – )7
B. es:
2
6

7 12. ¿De cuántas maneras


C.
4 A. 2.835 x4 y3 se pueden seleccionar
8
3 personas de un total
6 de 7?
D. – 2.835
5 B. x4 y3
5 8
E. A. 35
4
C. 945 x3 y4
16 B. 40
6. El último término en
el desarrollo de (x – 3 – 945 3 4
y)5 es: D. x y C. 56
16
A. – 15 y5 D. 9
E. No hay término cen-
B. 15 y5 tral
E. 21
C. 243 y5 10. El término central en el
desarrollo de (2 x – y)6
D. – 243 y5 13. ¿De cuántas maneras
es: se pueden ordenar 5
E. – 243 x y5 libros en un estante si
A. – 60 x2 y4
2 de ellos deben estar
7. El coeficiente numérico siempre juntos?
del 8º término del desa- B. 60 x2 y4
rrollo de (2 – x)11 es: A. 4!
C. 160 x3 y3
A. 330 B. 2 • 4!
B. – 330 D. – 160 x3 y3
C. 5!
C. 5.280 E. No hay término D. 2 • 5!
central
D. – 5.280 5
E. !
2
E. Otro

540 Análisis combinatorio, Teorema del binomio y Elementos de probabilidades

540-544 540 25/11/02, 1:22 PM


CAPÍTULO 13

14. ¿De cuántas maneras 17. El valor de B. 18


se puede seleccionar
un grupo de 4 personas 5 + 2 5 + 5 es: C. 27
0 1 5
entre un total de 3 hom-
bres y 5 mujeres? A. 3 D. 120

B. 5 E. 216
A. 32
C. 10
B. 60 21. Una liga de fútbol consta
D.12 de 15 equipos. ¿Cuántos
C. 140 partidos deben jugarse
E. 125 para completar la pri-
D. 70 mera rueda?
18. n + 2 : n+1 =
n+1 n+1 A. 15
E. 144
A. n
B. 30
15. El coeficiente numérico
del tercer término del B. n + 1 C. 105
desarrollo de (1 + x)5
es: C. n + 2 D.150

A. 1 D.(n + 1) (n + 2) E. 225
B. 3 n+2
E.
C. 5 n+1 22. ¿De cuántas maneras
se puede escoger un
D.10 19. ¿Cuántas “palabras” de “menú” (1 entrada-plato
5 letras pueden formarse de fondo-postre) si se
E. Otro. dispone de 3 entradas,
con las letras de la pala-
bra TIGRE? (sin repetir) 3 platos de fondo y 5
16. De las siguientes afir-
postres?
maciones:
A. 60
n n A. 45
I =
k n–k B. 120
B. 15
II n + n = n+1 C. 240
k k+1 k+1 C. 11
D.600
n+1 n
III n–1
=
n–2 D.14
E. 720
son verdaderas: E. 125

A. Sólo I
20. ¿Cuántas “palabras” de 3
B. Sólo II letras pueden formarse 23. ¿Cuántas diagonales se
con las letras de la pueden trazar en un
C. I y II palabra FÚTBOL? (sin polígono de 10 lados?
repetir)
D. Todas A. 10
A. 6
E. Ninguna B. 30

Análisis combinatorio, Teorema del binomio y Elementos de probabilidades 541

540-544 541 25/11/02, 1:22 PM


Prueba de selección múltiple
C. 35 D. 60 D. 1
9
D. 70 E. 120
1
E.
3
E. Otro 27. ¿De cuántas maneras se
puede viajar de A a C 30. Al lanzar 2 dados, ¿cuál
pasando por B si hay 6 es la probabilidad de
24. De un grupo de n per- posibilidades de ir de que la suma sea mayor
sonas hay 45 maneras A a B y 4 para ir de B o igual a 10?
de formar parejas. ¿Cuál a C?
es el valor de n? 1
A. 24 A.
3
A. 9
B. 12 1
B.
B. 10 4
C. 10
C. 12 C.
1
D. 6 6
D. 15
E. 36 1
D.
E. 20 9
28. ¿Cuál es la probabilidad
de obtener un número 1
menor que 3 al lanzar E. 12
25. ¿Cuántas “palabras” se 1 dado?
pueden formar con las 31. La probabilidad de
letras de la palabra obtener 2 números igua-
A. 10%
LÁPIZ si la A y la P les al lanzar 2 dados
deben aparecer juntas? es:
B. 20%
A. 48
C. 30% 1
A.
2
B. 24
1 1
D. B.
C. 120 3 3
1
D. 60 E. 1
2 C.
4
E. 96 29. ¿Cuál es la probabilidad
de obtener 3 “caras” al D. 1
lanzar 3 monedas? 6
26. ¿De cuántas maneras se
pueden ubicar 6 perso- E.
1
nas en un auto si sólo 1 A. 1 12
4
de ellas sabe manejar? 32. La probabilidad de no
1 obtener el 5 o el 3 al
A. 5 B. lanzar 1 dado es:
8

B. 6 1
C.
1 A.
6 4
C. 30 1
B.
3

542 Análisis combinatorio, Teorema del binomio y Elementos de probabilidades

540-544 542 25/11/02, 1:23 PM


CAPÍTULO 13

2 1 38. ¿Cuál es la proba-


C. B.
3 18 bilidad de obtener
1 1
2 números pares al
D. C.
6 12 lanzar 2 dados?
5
E. 1
6 D. 9 A. 1
6
33. La probabilidad de no
1
obtener una carta de E. 1
corazón de un naipe
6 B.
2
inglés (52 cartas) es: 36. En una caja hay 5 mone-
das de $ 100 y 6 de $ 50. 1
C.
¿Cuál es la probabilidad 3
A. 1
4 de sacar 2 monedas de
$ 100 si se sacan 2 veces 1
3 D.
B. sin reponerlas? 12
4
1
C. 1 A.
3
E. 1
2 4

3 2 39. Al lanzar 3 dados, ¿cuán-


D. B.
8 5 tos resultados posibles se
1 2 pueden obtener?
E. C.
8 11
A. 18
34. Al lanzar 2 veces segui- 5
D. 11 B. 30
das 1 dado, ¿cuál es la
probabilidad de sacar
E.
4 C. 108
2 ases?
11

37. Una caja contiene 8 D. 196


1
A.
36 bolas rojas y 4 negras.
E. 216
¿Cuál es la probabilidad
1
B. de no sacar una bola
6 40. Al lanzar 1 dado, ¿cuál
negra?
1 es la probabilidad de
C. obtener un número im-
12
1
A. par o un número menor
1 3
D. que 4?
18
B. 2
1 3 1
E. A.
30 3
1
C. 2
35. Al lanzar 2 dados, ¿cuál 4 B.
3
es la probabilidad de
1
obtener una suma menor D. C.
1
6
que 3? 2

1 1 D.
A. E.
36 2

Análisis combinatorio, Teorema del binomio y Elementos de probabilidades 543

540-544 543 25/11/02, 1:26 PM


3 1
E. E.
4 40
41. Al lanzar 2 veces un dado, ¿cuál 43. En una carrera corren 8 caballos signados
es la probabilidad de obtener un 6 por números 1, 2, 3, …, 8. ¿Cuál es la
en el primer lanzamiento y un 2 en probabilidad de que gane el caballo
el segundo? 3 o el caballo 6 si todos tienen igual
probabilidad de ganar?
1
A.
6 1
A.
1
4
B. 3
1
B.
1 3
C.
36
1
C.
1 8
D. 18
1
D.
1 64
E.
12
E. Otro.
42. ¿Cuál es la probabilidad de ganar el
premio de una rifa para la cual se venden
20 listas y cada lista tiene 20 números, si
se compran 4 números?
1
A.
10

i
B.
100

1
C.
200

1
D.
50

Soluciones
1. A
2. C 9. E 16. C 23. C 30. C 37. B
3. D 10. D 17. D 24. B 31. D 38. E
4. C 11. E 18. C 25. A 32. C 39. E
5. A 12. A 19. B 26. E 33. B 40. B
6. D 13. B 20. D 27. A 34. A 41. C
7. D 14. D 21. C 28. D 35. A 42. B
8. C 15. D 22. A 29. B 36. C 43. A

544 Análisis combinatorio, Teorema del binomio y Elementos de probabilidades

540-544 544 25/11/02, 1:27 PM


CAPÍTULO 14
Problemas

Introducción
En esta sección aplicaremos los conocimientos adquiridos
en el planteamiento y resolución de problemas. Para esto es
necesario seguir algunos pasos:

1. Leer muy bien el enunciado y asegurarse de comprenderlo cabalmente.


2. Identificar los elementos conocidos y desconocidos que allí intervengan.
3. Asignar variables correspondientes a los elementos mencionados.
4. Plantear la o las ecuaciones necesarias de acuerdo con el problema.
5. Resolver la o las ecuaciones planteadas.
6. Entregar la solución del problema.
El último punto es importante porque muchas veces la solución del problema no
está dada por la solución directa de las ecuaciones. Muchas veces es conveniente
efectuar una comprobación de la solución.

Aplicación de ecuaciones
lineales enteras 14.1

Ejercicios 1. El doble de un número más 5 es igual al triple del mismo número


menos 2. ¿Cuál es el número?
resueltos
El enunciado es muy simple y hay sólo un elemento en cuestión.
Sea x el número pedido.
Escribamos los datos en forma de ecuación:

Problemas 545

545 545 25/11/02, 1:29 PM


2x + 5 = 3x – 2
resolviendo la ecuación, obtenemos:
x=7
que es la solución del problema.

2. El triple de un número disminuido en 18 es igual al mismo número


aumentado en 8. ¿Cuál es el número?
Sea x el número pedido:
Tenemos: 3x – 18 = x + 8
y resolviendo obtenemos:
2x = 26
x = 13
y ésa es la solución del problema.

3. Al doble de un número disminuido en 5 se le agrega el triple del


mismo número disminuido en 10 y se obtiene 25. Calcular el
cuádruple de dicho número.
Sea x el número en cuestión:
Tenemos: 2x – 5 + 3x – 10 = 25
resolviendo la ecuación:
5x – 15 = 25
5x = 40
x =8
el número es 8, pero se nos pide el cuádruple de él; por lo tanto,
la solución del problema es 4x = 32.

4. Encontrar tres enteros consecutivos cuya suma sea 54.


Como se trata de números consecutivos se pueden representar
por x, x + 1 y x + 2.
(También, y es más simple en este caso, por x – 1, x, x + 1).
Planteando la ecuación de acuerdo al problema, tenemos:
x+x+1+x+2 = 54
3x + 3 = 54
3x = 51
x = 17
y los números pedidos son 17, 18 y 19.

5. La suma de 3 números pares consecutivos es 402. ¿Cuáles


son los números?
Aunque formalmente la expresión de un número par está dada
por 2n (n E N), aquí podemos usar la variable simple x, dadas
las condiciones del problema. Como se trata de números pares,
la diferencia entre dos consecutivos de ellos es 2. Así, los
números son:
x, x + 2 , x + 4 o (x – 2 , x, x + 2).

546 Problemas

546-547 546 10/11/01, 1:33 PM


CAPITULO 14

Entonces la ecuación es:


x+x+2+x+4 = 402
3x + 6 = 402
3x = 396
x = 132
y los números son 132, 134 y 136.

6. La suma de tres números impares consecutivos es 297. ¿Cuál


es el doble del mayor?
Formalmente la expresión que identifica a un número impar es
2n + 1 (n E N), pero en este caso, dadas las condiciones del
problema, podemos identificarlo con x. Entonces los números
son x, x + 2 y x + 4.
Así: x+x+2+x+4 = 297
3x + 6 = 297
3x = 291
x = 97
Los números son 97, 99 y 101 y el doble del mayor es 202.

7. La suma de las edades de 2 hermanos es 31 años, si la diferencia


entre ellos es de 3 años, calculemos la edad de ambos.
Sea x la edad del mayor. Como la diferencia entre sus edades es de
3 años, el hermano menor tendrá entonces x – 3 años.
Así: x+x–3 = 31
2x – 3 = 31
2x = 34
x = 17
Entonces la edad del mayor es 17 años y la del menor es
14 años.

8. Dividir el número 550 en tres partes tales que la segunda exceda a


la primera en 25 y la tercera exceda a la segunda en 50.
Sea x la primera parte, entonces la segunda será x + 25 y la
tercera (x + 25) + 50 = x + 75
La ecuación correspondiente es:
x + x + 25 + x + 75 = 550
3x + 10 = 550
3x = 450
x = 150
y las partes serán, respectivamente: 150, 175 y 225.

9. La edad de Ana es el doble de la edad de María y hace 6 años era 4


veces la de María. Determinemos las edades actuales.
Sea x la edad actual de María. Entonces
2x es la edad actual de Ana.

Problemas 547

546-547 547 10/11/01, 1:33 PM


Hace 6 años la edad de María era
x – 6 y la edad de Ana era 2x – 6
y en ese momento la edad de Ana era el cuádruple de la edad
de María. Es decir:
2x – 6 = 4 (x – 6)
Resolviendo la ecuación, tenemos:
2x – 6 = 4x – 24
18 = 2x
9 =x
Entonces, actualmente María tiene 9 años y Ana tiene 18 años
(hace 6 años María tenía 3 años y Ana tenía 12, es decir, el
cuádruple de la edad de María).

Ejercicios
1. La suma de un número con su doble ¿Cuáles son los números?
es 18. 9. La suma de 3 números impares
¿Cuál es el número? consecutivos es – 279.
2. La diferencia de un número con su ¿Cuál es el mayor?
triple es – 16. 10. La suma de 4 números pares consecu-
¿Cuál es el número? tivos es 4.620.
3. El doble de un número aumentado ¿Cuál es el número menor?
en 1 es igual al triple del mismo 11. La suma de 4 números consecutivos
número disminuido en 4. es 546.
¿Cuál es el número? ¿Cuál es el doble del menor?
4. El triple de un número disminuido 12. Compro 1 kg de azúcar y 1 litro de
en 8 es igual al doble del mismo aceite. Si el kg de azúcar cuesta $ 30
número aumentado en 4. menos que el litro de aceite y en total
¿Cuál es el número? gasto $ 650.
5. Siete veces un número disminuido ¿Cuál es el precio de cada artículo?
en 1 es igual a 6 veces dicho número 13. Se compra una docena de pinceles y
aumentado en 3. media docena de cuadernos. Si cada
¿Cuál es el número? cuaderno es $ 60 más caro que cada
pincel y el total de la compra
6. La suma de tres números enteros es de $ 2.520, ¿cuál es el precio de
consecutivos es 636. cada artículo?
¿Cuáles son los números? 14. Se compran 2 kg de lentejas y 3 kg
7. La suma de tres números enteros de garbanzos. Si el kg de garbanzos
consecutivos es – 135. cuesta $ 60 más que el de lentejas y
¿Cuáles son los números? el total de la compra es de $ 1.930,
calcule el precio de cada artículo.
8. La suma de tres números impares
consecutivos es 369.

548 Problemas

548-549 548 10/11/01, 1:35 PM


CAPITULO 14

15. Pedro y Luis tienen en total $ 3.800. Si ¿cuántos hay de cada tipo?
Pedro gasta $ 600 y Luis gana $ 400, 27. En un corral hay conejos y gallinas,
Luis tendrá el doble de lo que tendrá si en total hay 40 patas y 14 cabezas.
Pedro. ¿Cuánto tiene cada uno? ¿Cuántos conejos hay? y ¿Cuántas
16. Hay 91 rosales rojos, blancos y gallinas?
amarillos. Si hay el doble de rosales 28. Un padre reparte una herencia a sus
rojos que amarillos y el doble de 3 hijos otorgándoles $ 2.000.000
rosales blancos que rojos, ¿cuántos por cada año que ellos tienen. Si el
rosales hay de cada color? hermano mayor tiene 3 años menos
17. ¿Cuántas parras se necesitan para que la suma de las edades de sus dos
producir 12.800 kg de uva si cada hermanos, el hermano del medio tiene
parra produce aproximadamente 160 el doble de la edad de su hermano
kg? menor y éste, el menor, tiene 4
18. Dividir el número 1.200 en dos partes años, determine cuál fue la cantidad
tales que el triple de la parte menor repartida.
exceda en 96 a la mayor. 29. Las edades de 2 hermanas suman
19. Dividir el número 164 en dos partes 26 años y el doble de la edad de
tales que el doble del mayor aumentado la menor disminuida en 1 es igual
en 40 sea igual al triple del menor a la edad de la mayor. Determine
disminuido en 7. ambas edades.

20. Dividir el número 750 en 3 partes tales 30. La edad actual de una madre es el
que la segunda sea igual al doble de la cuádruple de la edad de su hija más
primera y la tercera sea igual a la suma 3 años y hace 3 años la edad de la
de la primera y la segunda. madre era 8 veces la edad de la hija.
Determine ambas edades.
21. Dividir el número 196 en tres partes
tales que la segunda sea igual al doble 31. Las edades de dos hermanos suman 35
de la primera menos 3 unidades, la años. Dentro de 15 años la edad del
tercera sea igual al doble de la segunda menor será la edad actual del mayor.
más 2 unidades. Determine ambas edades.

22. El perímetro de un cuadrado es 52 cm. 32. Tengo en mi bolsillo 37 monedas de


¿Cuánto mide el lado? $ 50 y $ 100 y en total tengo $ 2.600.
¿Cuántas monedas de $ 50 y cuántas
23. El perímetro de un rectángulo es 64 cm de $ 100 tengo?
y su largo tiene 2 cm más que su ancho.
¿Cuáles son sus dimensiones? 33. Encontrar 3 números enteros consecuti-
vos tales que el doble del menor más el
24. Calcule las dimensiones de un rectán- triple del mediano menos el cuádruple
gulo sabiendo que el largo es el del mayor sea igual a 19.
triple del ancho y que su perímetro
es 72 cm. 34. La suma de dos números es 77 y la
diferencia entre el triple del menor y
25. Se desea cercar un terreno rectangular. el doble del mayor es 16. Determine
Si se necesitan 30 m de alambre y se dichos números.
sabe que el largo tiene 3 m más que
el ancho, determine las dimensiones 35. Dentro de 15 años la edad de un padre
del terreno. será el doble de la edad de su hijo.
Si las edades difieren en 30 años,
26. En un corral hay conejos y gallinas. determine las edades actuales.
Si en total hay 44 patas y 14 cabezas,

Problemas 549

548-549 549 10/11/01, 1:35 PM


Soluciones
(Ecuaciones enteras)

1. x = 6 2. x = 8 3. x = 5 4. x = 12 5. x = 4 6. 211 , 212 , 213


7. – 44, – 45, – 46 8. 121, 123, 125 9. – 91 10. 1.152
11. 270 12. $ 310 , $ 340 13. $ 120 pincel; $ 180 cuaderno.
14. $ 350 lentejas; $ 410 garbanzos
15. $ 1.800 Pedro; $ 2.000 Luis 16. 13 – 26 – 52. 17. 80.
18. 324 – 876. 19. 89 – 75 20. 125 – 250 – 375
21. 29 – 55 – 112 22. 13. 23. largo = 17 cm; ancho = 15 cm
24. l = 27 cm; a = 9 cm 25. a = 6 m; l = 9 m
26. 8 conejos; 6 gallinas 27. 6 conejos; 8 gallinas
28. $ 42.000.000 29. 9 – 17 30. h = 6 años; m = 27 años
31. 25 y 10 32. 22 de $ 50 y 15 de $ 100
33. 24 – 25 – 26 34. 34 – 43 35. P = 45 años; h = 15 años

Aplicación de ecuaciones
14.2 lineales fraccionarias

El procedimiento y los pasos a seguir son análogos al caso


anterior.
Los desarrollos para obtener las soluciones dependen de
las ecuaciones planteadas y, como se trata de expresiones
fraccionarias, deberán hacerse todas las amplificaciones
necesarias para resolver el problema.

1. Un número más su mitad, más su tercera parte y más su cuarta


Ejercicios parte es igual a 25. ¿Cuál es el número?
resueltos Denotemos por x el número pedido. Tenemos que el planteamiento
del problema nos queda:
x+ x
+ x
+ x = 25
2 3 4
Y resolvemos la ecuación amplificando por el m.c.m. entre los
denominadores, que es 12.
x+ x
+ x
+ x = 25 /• 12
2 3 4
12x + 6x + 4x + 3x = 300
1
25x = 300 /•
25
x = 12
El número pedido es 12.
2. De una pieza de tela de 60 m se venden los 2 . ¿Cuánto queda?
2 5
Quedan 60 – • 60 = 60 – 24
5
= 36 m

550 Problemas

550-551 550 10/11/01, 1:37 PM


CAPITULO 14

2
3. La edad de un hijo equivale a los de la edad de su padre y
7
1
hace 6 años la edad del hijo era de la del padre. Determine
6
las edades actuales.
Sea x la edad actual del padre. Entonces 2 x es la edad
7
actual del hijo.
Hace 6 años las edades eran x – 6 y 2 x – 6, respectivamente,
7
y la relación estaba dada por:
2 1
x–6 = (x – 6) /• 42
7 6
12x – 252 = 7x – 42
5x = 210
x = 42
La edad del padre es 42 años y la de su hijo es 12 años.

Ejercicios
1. Dividir el número 150 en dos partes, del número disminuido en 15. ¿Cuál
de modo que una de ellas sea la es el número?
mitad de la otra.
8. Se desea repartir $ 1.020 en tres partes,
2. Dividir el número 150 en tres partes, de modo que la primera sea igual a
de modo que la primera sea la mitad tres cuartos de la segunda más $ 180
de la segunda y la segunda sea igual a y la tercera sea igual a cinco sextos
los dos tercios de la tercera. de la primera más $ 120. ¿Cuánto
corresponde a cada parte?
3. Dividir el número 150 en dos partes,
de modo que la primera sea igual a los 9. En un curso la mitad de los alumnos
tres medios de la segunda. habla inglés y español, la sexta parte
habla francés y español, la octava
4. Dividir el número 150 en 4 partes,
parte habla alemán y español y los 25
de modo que la primera sea igual a
alumnos restantes hablan sólo español.
un tercio de la segunda, la segunda
¿Cuántos alumnos tiene el curso?
sea igual a un medio de la tercera
y la tercera, igual a la suma de la 10. Hace 15 años la edad de Juan era
cuarta y la primera. igual a los tres cuartos de la edad
de Pedro; sabiendo que Pedro tiene
5. Si pago los dos quintos de una deuda
10 años más que Juan, determine las
quedo debiendo $ 45.000 más los
edades actuales.
tres décimos de la deuda. ¿A cuánto
asciende ésta? 11. Un padre reparte una cantidad de
dinero entre sus hijos. El mayor recibió
6. La cuarta parte de un número dismi-
la mitad del total, el segundo recibió
nuido en 2 es igual a la sexta parte
la cuarta parte del total; el tercero
del mismo número aumentada en 1.
recibió la sexta parte del total y el
¿Cuál es el número?
cuarto recibió el dinero restante que
7. El doble de un número disminuido en eran $ 5.400. ¿Cuánto fue el dinero
la mitad del mismo es igual al triple repartido?

Problemas 551

550-551 551 10/11/01, 1:38 PM


12. Hace 10 años la edad de María era 2
23. Los de una cantidad de fruta son
igual a la décima parte de la edad de 5 3
Ana y actualmente es la cuarta parte. manzanas, los son naranjas y hay 12
7
¿Qué edades tienen Ana y María? duraznos. ¿Cuál es el total de fruta?
13. Dos números enteros consecutivos 3
son tales que la diferencia entre los tres 24. Los de los animales de una granja
8
cuartos del menor y los dos quintos son corderos y el resto son vacas.
del mayor equivalen al número mayor En total hay 64 animales. ¿Cuántos
disminuido en 17. corderos y cuántas vacas hay?
14. Tengo cierta cantidad de dinero. Gasto 25. Una mezcla de concreto se prepara con
1 arena y cemento en la proporción 3:4.
del total en un primer artículo;
5 Si se necesitan 105 kilos de concreto,
luego gasto la mitad de lo que ¿cuántos kilos de arena y de cemento
me queda en un segundo artículo se requieren?
y aún me quedan $ 4.800. ¿Cuánto 26. Si con 134 litros de agua se llena
dinero tenía? 2
un estanque sólo en sus 3
partes,
4 3
15. La diferencia entre los y los determine la capacidad del estanque.
5 4
de un número es 6. Determine el 27. La quinta parte de los asistentes a una
número. conferencia son alemanes, la mitad son
16. Un vehículo recorrió 210 kilómetros ingleses y hay 21 brasileños. ¿Cuántos
en 2 horas y 20 minutos. ¿Cuántos son los asistentes a la conferencia?
kilómetros recorrió en 1 minuto? 2 de los 3 de un número equivalen
28. Los
(Suponga una velocidad constante). 5 2
7 a las 3 partes del mismo número dismi-
17. Un hijo tiene de la edad de su 4
16
nuido en 6. Determine el número.
padre. Si ambas edades suman 92
años, determínelas. 29. Se compran 3 artículos A, B y C.
18. Un kilómetro corresponde aproxima- El artículo A costó $ 150.
damente a los ocho quintos de una El artículo A y el artículo B costaron las
milla. ¿Cuántos km hay en 150 tres cuartas partes del valor de C y el
millas? artículo C más el A costaron $ 50 más
19. De un depósito de agua se ocupan las que el doble del valor de B. ¿Cuánto
dos quintas partes; luego se ocupan costó cada uno?
los dos tercios de lo que queda, 1
30. Tengo una cantidad de dinero. Gasto
luego se ocupa un medio del resto 8
y aún quedan 144 litros. ¿Cuál es la de esa cantidad en un artículo y 5 de la
capacidad del depósito? 12
20. Un reloj se adelanta 90 segundos por misma en otro. Luego gasto 4
de lo
día. ¿En cuántos días el reloj marcará 5

un aumento de 12 minutos? que me queda y aún conservo $ 5.500.


¿Cuál era mi capital inicial?
21. Los 3 de los 2 de una deuda
4 5
31. Dos vehículos viajan en la misma
corresponden a $ 1.710. ¿A cuánto carretera acercándose en sentidos
asciende la deuda? opuestos. El primero mantiene una
22. El doble de un número más la mitad velocidad de 65 km/h y el segundo
del mismo número, más un cuarto de 55 km/h. Si se encuentran entre sí
del número más uno equivale a 100. a 520 km de distancia, ¿en cuántas
¿Cuál es el número? horas estarán a 40 km entre ellos, antes
de cruzarse?

552 Problemas

552-553 552 10/11/01, 1:40 PM


CAPITULO 14
Soluciones
(Ecuaciones fraccionarias)

1. 50 y 100 2. 25 - 50 - 75 3. 90 - 60 4. 10 - 30 - 60 - 50
5. $ 150.000 6. 36 7. 10 8. 360 - 240 - 420 9. 120
10. Juan: 45 años; Pedro: 55 años 11. $ 64.800
12. María tiene 15 años y Ana tiene 60 años 13. 24 ; 25 14. $ 12.000 15. 120
16. Recorrió 1,5 km 17. 28 y 64 años, respectivamente 18. 93,75 km 19. 1.440 litros.
20. 8 días 21. $ 5.700 22. 36 23. 70 24. 24 corderos y 40 vacas
25. 45 kg de arena y 60 kg de cemento 26. 201 litros 27. 70 28. 40
29. $ 150, $ 450 y $ 800, respectivamente 30. $ 60.000 31. 4 horas

Aplicación de sistemas
de ecuaciones lineales 14.3

Muchos problemas se pueden resolver en forma mucho más


simple si se plantean mediante un sistema de ecuaciones.
Esto implica el uso de 2 o más variables, cuyos valores deben
ser encontrados aplicando los métodos conocidos para la
solución de sistemas.

1. La suma de dos números es 17 y su diferencia es 7. Determine


Ejercicios los números.
resueltos Como se trata de dos números, éstos serán denotados por x
e y. Así tenemos:
x + y = 17
x–y = 5
Sumando ambas ecuaciones obtenemos:
2x = 22
x= 11
y sustituyendo el valor obtenido tenemos:
y=6
Entonces los números pedidos son 11 y 6.
2. La cuarta parte de la diferencia de dos números es igual a
9 y la diferencia entre el mayor y el triple del menor es 4.
Determine los números.
Sean x e y los números pedidos, con x > y.
Planteamos las ecuaciones de acuerdo con el enunciado:
x–y
=9
4
x – 3y= 4

Problemas 553

552-553 553 10/11/01, 1:40 PM


y resolvemos el sistema. Amplifiquemos la primera ecuación
por 4. Nos queda:
x – y = 36
x – 3y = 4
Restando ambas obtenemos:
2y = 32
y = 16
Y reemplazando el valor obtenido en cualquier ecuación
tenemos:
x = 52 y los números pedidos son 52 y 16.
1
3. Hace 6 años la edad de un hijo era de la edad de su padre y
5 2
dentro de 9 años la edad del hijo será los de la de su padre.
5
Determinar las edades actuales.
Sean x e y las edades actuales del padre y del hijo respectiva-
mente.
Hace 6 años, las edades eran x – 6 e y – 6, respectivamente.
Dentro de 9 años ambos tendrán x + 9 e y + 9, respectiva-
mente.
Planteamos entonces las ecuaciones de acuerdo con el enunciado.
1
y – 6 = (x – 6)
2
2
y + 9 = (x + 9)
5
Haciendo las amplificaciones adecuadas tenemos:
5y – 30 = x – 6
5y + 45 = 2x + 18
5y – x = 24
5y – 2x = – 27

Resolviendo el sistema obtenemos las soluciones x = 51


y = 15
lo que corresponde a las edades actuales de un padre y de
su hijo.

4. Dividir 90 en tres partes tales que la parte menor sea igual


1 9
a de la parte intermedia y la intermedia sea igual a
9 20
de la parte mayor.
Sean x, y, z las tres partes ordenadas de menor a mayor.
Tenemos: x + y + z = 90
1
x= y
9
9
y= z
20
20
de la 3ª ecuación: z = y
9
de la 2ª ecuación: y = 9x

554 Problemas

554-555 554 10/11/01, 1:41 PM


CAPITULO 14

20
de ambas igualdades obtenemos: z = •9•x
9
z = 20x
podemos expresar tanto y como z en términos de x.
Reemplazando en la 1ª ecuación:
x + 9x + 20x = 90 fi 30x = 90
x =3
y sustituyendo obtenemos: y = 27
z = 60
y así queda 90 dividido en esas tres partes.

Ejercicios
1. La suma de dos números es 12 y la 10. Si al doble de un número le resto
diferencia entre ellos es 6. Determí- el triple de otro obtengo 5 y si al
nelos. primer número le sumo el doble del
2. La suma de dos números es 6 y su segundo obtengo –1. ¿Cuáles son
diferencia es –17. Determínelos. los números?
3. La diferencia de dos números es 24 11. Si Pedro le da a Juan $ 400, ambos
y la suma del mayor con el doble quedan con la misma cantidad y si
del menor es –6. Determine ambos Juan le da a Pedro $ 300, entonces
números. Pedro tendrá exactamente el doble
4. Tengo $ 4.050 en monedas de $ 50 y de lo que tiene Juan. ¿Cuánto tiene
de $ 100. Si en total tengo 46 monedas, cada uno?
¿cuántas tengo de cada valor? 12. Si sumamos el dinero mío y el de mi
5. Dividir 60 en dos partes tales que la hermano hacemos $ 1.700 y si mi
parte menor sea igual a la tercera parte hermano me regala $ 155, tendremos
de la parte mayor. lo mismo cada uno. ¿Cuánto tiene
cada uno?
6. Tres kilos de un artículo A más un kilo
de un artículo B cuestan $ 955. Tres 13. Hace 10 años la edad de un hijo era
kilos del artículo B más un kilo del un séptimo de la edad de su madre y
artículo A cuestan $ 1.225. ¿Cuál es el dentro de 10 años la edad de la madre
precio de cada artículo por kilo? será el doble de la edad de su hijo.
Determine las edades actuales.
7. Determine dos números de modo que
la diferencia entre el mayor y el doble 14. Si a los dos términos de una fracción
del menor sea –1 y la suma del doble se le agrega 1, la fracción que resulta
del mayor con el menor sea 8. 1
es equivalente con y si a los
8. Tengo 18 aves entre patos y gallinas y 2
dos términos de la misma fracción
la diferencia entre el doble de patos
y el triple de gallinas es 1. ¿Cuántas se les resta 1, resulta una fracción
tengo de cada tipo? 1
equivalente a . Determine la
4
9. Si al triple de un número le agrego el fracción original.
doble de otro obtengo como resultado
8 y si al doble del segundo número le 15. La razón entre dos números es 2:3
5
agrego el primero obtengo 0. ¿Cuáles y su suma es . ¿Cuáles son los
4
son los números? números?

Problemas 555

554-555 555 10/11/01, 1:41 PM


16. Dos números están en la razón 3:1. 20. La suma de tres números es 27. Si
La diferencia entre el doble del mayor el mayor de ellos se divide por 5, se
y el menor es 15. ¿Cuáles son los obtiene el primero y si el primero se
números? multiplica por 3, se obtiene el segundo.
¿Cuáles son los números?
17. La edad de Ana es igual al triple de la
edad de Rosa. La diferencia entre la 21. La suma de dos ángulos de un triángulo
mitad de la edad de Ana y un tercio es igual a 86° y la diferencia entre
14 ellos es 30°. Determine los tres ángulos
de la edad de Rosa es . ¿Qué edad
3 del triángulo.
tiene cada una?
22. A, B y C reúnen en total $ 11.000.
18. La suma de tres números enteros
La diferencia entre el doble de lo que
positivos es 20. Si el menor se
tiene C y lo que tiene A es $ 400 y la
multiplica por 5, se obtiene el doble
diferencia entre el doble de lo que tiene
del mayor y si el mediano se multiplica
B y el triple de lo que tiene C es $ 570.
por 5, se obtiene el triple del mayor.
¿Cuánto tiene cada uno?
Determine los números.
19. La suma de tres números enteros 23. Compré tres artículos por $ 1.855. El
(distintos) es 3. La suma del menor primero y el tercero juntos costaron
con el mayor es 2 y la diferencia $ 205 más que el segundo, y el tercero
entre ellos es –14. ¿Cuáles son los costó $ 250 menos que el primero.
números? Determine el precio de cada uno.

Soluciones
1. x = 9 7. x = 3 13. hijo: 14 años 19. x = – 6
y=3 y=2 madre: 38 años y=1
z=8
11 2
2. x = – 8. x = 11 14.
2 5 20. a = 3
23 b=9
y = y =7
2 c = 15
1 21. a = 58
3. x = 14 19. x = 4 15. x =
2
3 b = 28
y = – 10 y = –2 y= d = 94
4
4. 11 monedas de $ 50 10. x = 1 16. x = 9 22. a = $ 4.540
35 monedas de $ 100 y = –1 y=3 b = $ 3.990
c = $ 2.470
5. 15 y 45 11. x = 2.500 17. Ana: 12 años
y = 1.700 Rosa: 4 años 23. a = $ 640
b = $ 825
6. a = $ 205 12. x = 1.005 18. x = 4 c = $ 390
b = $ 340 y = 695 y=6
z = 10

556 Problemas

556-559 556 10/11/2001, 19:15


CAPITULO 14

Problemas 14.4
misceláneos

1. Un trabajador tiene un contrato por 48 horas semanales


o 192 horas mensuales. Cada mes recibe un sueldo bruto
consistente en:
Sueldo base $330.240
Asignación zona $ 66.048
Antigüedad $ 39.625
Determine su valor hora base, su porcentaje de asignación de
zona y su porcentaje correspondiente a antigüedad.

2. Un alumno ha sacado las siguientes notas en álgebra: 5,5; 6,2;


5,8; 6,6 en una escala de 1 a 7. Debe dar una prueba más.
Averigüe qué nota debe obtener en esa prueba para lograr
un promedio final 6,5.

3. Se deben preparar 100 ml de un jarabe para la tos, con una


droga que viene con una concentración de 5 mg/ml y un elixir
saborizante de miel para distraer el sabor de la droga. ¿Cuánto
se debe usar de cada ingrediente si la solución debe quedar
con una concentración de 2mg/ml?

4. Dados los siguientes enunciados, expréselos como una


fórmula donde intervengan las variables y una constante
k. En cada caso estime el valor de la constante según las
condiciones dadas.
a) x es directamente proporcional a y. Si x es 22, entonces
y es 66.
b) r varía en forma directa respecto de t. Si r = 24; entonces
t = 6.
c) p es inversamente proporcional a la suma de r y t. Si
r = 0,4 y t = 0,8, entonces p = 1,4
d) La energía cinética E c varía en forma directamente
proporcional a la masa y al cuadrado de la velocidad con
que se mueve un cuerpo. Si m = 6, v = √10, entonces
Ec = 30.
e) La velocidad de desplazamiento v de un cuerpo varía en
forma directamente proporcional a la distancia recorrida
d, e inversamente proporcional al tiempo t que se
demora en recorrerla. Si d = 10 m y t = 2 seg, entonces
v = 5 m/seg.

5. Una empresa de distribución de correspondencia debe repartir


x cartas en 3 días. El primer día reparte a sobres, el segundo día

Problemas 557

556-559 557 10/11/2001, 19:15


reparte 6 sobre menos que el primer día. Determine cuántos
sobres le quedan por repartir el tercer día.

6. Sea A = x + y y b = x – y. Encuentre el valor de x2.


1 2 1 1
7. Se sabe que + =m y + = 2.
x x m m
Determine el valor de x.
x•x
8. Si x + 2x + 3x = 48, encuentre el valor de
x+x
9. Si t + 2t + 3t + 4t = 1, encuentre el valor de
1 2 3 4 5
+ + + +
t t t t t
10. Con a litros de bencina un automóvil alcanza a recorrer las
tres quintas partes del camino entre dos ciudades. Determine
cuántos litros se necesitan para recorrer el trayecto de
ida y vuelta.

11. De un total de n alumnos que rindió la prueba de ingreso


a la educación superior reprobaron b alumnos. Calcule el
porcentaje de alumnos que aprobó.

12. Si r es el 25% de A y t es el 75% de A, escriba r en


función de t.

13. Para preparar 120 kg. de concreto (cemento con arena), en


los primeros 40 kg se usa la proporción 3:2 y en el resto, se
usa la proporción 2:3 de cemento y arena, respectivamente.
Determine cuánta arena se usó.

14. Tres amigos reparten equitativamente entre ellos $960.000 en


billetes de 1.000, 5.000 y 10.000. Si cada uno recibe la misma
cantidad de billetes de cada denominación, ¿cuánto dinero
recibe cada amigo en billetes de 1.000?

15. En un mapa se lee: escala 1 : 120.000. Se mide con una regla


la distancia entre dos pueblos y se encuentra que es 5,4 cm.
Calcule la distancia real.

16. Se dispone de un presupuesto mensual de $10.000 para


comprar dulces o chocolates. Si cada dulce cuesta 10 pesos y
cada chocolate cuesta 50 pesos.
a) Escriba una fórmula que relacione la cantidad de dulces con
la cantidad de chocolates que se pueden comprar.
b) Grafique la función que resultó.
c) Compare las intersecciones con los ejes del gráfico con
la cantidad de dulces que podría comprar si no comprara
chocolates y con la cantidad de chocolates que se podría
comprar si no comprara dulces.
558 Problemas

556-559 558 10/11/2001, 19:15


CAPITULO 14

17. Una empresa invierte $3.000.000 en una maquinaria que


tiene una vida útil de 5 años; después quedará totalmente
desvalorizada. Escriba una fórmula que relacione el valor de
la maquinaria con el tiempo transcurrido.

18. La diagonal de un cuadrado mide 10 cm más que su lado.


Calcule la medida del lado.

19. Un paralelepípedo (sólido rectangular) mide 3 cm de largo,


2 de ancho y 1 de alto. Si duplico sus magnitudes lineales,
¿qué pasa con su volumen?

20. Una cañería de agua de 1 cm de radio transporta líquido a razón


de 1,2 m/s. ¿Cuánta agua sale en 1 hora?

Soluciones

1. 1.720, 20%, 12% 1


12. x = t
3
2. No puede subir a 6.5. Tendría que
obtener un 8.2 en la prueba que le 13. 64 kg
queda.
14. 20.000 pesos.
3. 40 ml de concentrado de droga.
15. 6,48 km.
60 ml de saborizante.
16. a) x + 5y = 1.000
4. a) x = ky ; k = 3
b)
b) r = kt ; k = 4
Chocolates
1
c) p = k ; k = 1.68
r+t
1
d) Ec = k mv2; k =
2
d 200
e) v = k ;k=1
t
Dulces
5. x – 2a + b 1.000

6. A • B – y2
17. y = 3.000.000 – 600.000x
7. 3 ( x = tiempo y = valor)
8. 4 18. 24,142 cm
9. 150
19. a) Se aumenta ocho veces.
10
10. a lt 20. 604.800 c.c.
3
n–b
11. • 100
n

Problemas 559

556-559 559 10/11/2001, 19:15


Indice
Capítulo 1
Álgebra en los Números Reales
1.1 LENGUAJE ALGEBRAICO .................................................................................. 7
1.2 VALORIZACIÓN DE EXPRESIONES ALGEBRAICAS ........................................... 12
1.3 REDUCCIÓN DE TÉRMINOS SEMEJANTES Y USO DE PARÉNTESIS .................. 14
1.4 MULTIPLICACIÓN ALGEBRAICA ....................................................................... 19
1.5 PRODUCTOS NOTABLES .................................................................................. 24
1.6 FACTORIZACIÓN .............................................................................................. 29
1.6.1 Factor común (Monomio y Polinomio) ............................................................... 29
1.6.2 Factor común compuesto ................................................................................... 32
1.6.3 Diferencia de cuadrados .................................................................................... 34
1.6.4 Trinomios ordenados .......................................................................................... 37
1.6.5 Sumas o diferencias de cubos ............................................................................ 41
1.7 FRACCIONES ALGEBRAICAS ............................................................................. 43
1.7.1 Simplificación .................................................................................................... 43
1.7.2 Multiplicación y División de fracciones algebraicas ........................................... 45
1.7.3 Adición y Sustracción de fracciones algebraicas ................................................. 50
PRUEBA DE SELECCIÓN MÚLTIPLE ................................................................... 56

Capítulo 2
Ecuaciones e inecuaciones de primer grado
2.1 ECUACIONES .................................................................................................... 60
2.1.1 Ecuaciones de primer grado con coeficientes enteros ......................................... 61
2.1.2 Ecuaciones de primer grado con coeficientes fraccionarios ................................ 65
2.1.3 Ecuaciones fraccionarias de primer grado .......................................................... 69
2.1.4 Ecuaciones literales de primer grado .................................................................. 73
2.1.5 Ecuaciones con valor absoluto ........................................................................... 79
2.2 PROBLEMAS ...................................................................................................... 80
2.3 DESIGUALDADES E INECUACIONES ................................................................ 89
2.3.1 Desigualdades .................................................................................................... 91
2.3.2 Inecuaciones ...................................................................................................... 94
2.3.3 Inecuaciones simultáneas ................................................................................... 97
2.3.4 Inecuaciones con valor absoluto ........................................................................ 100
PRUEBA DE SELECCIÓN MÚLTIPLE ................................................................... 104

Capítulo 3
Relaciones y funciones
3.1 LÓGICA ............................................................................................................. 111
3.2 CONJUNTOS ..................................................................................................... 123
3.2.1 Conceptos básicos ............................................................................................. 123
3.2.2 Operaciones entre conjuntos .............................................................................. 129
3.3 RELACIONES ..................................................................................................... 136
3.3.1 Conceptos básicos ............................................................................................. 136
3.3.2 Relación de equivalencia y de orden .................................................................. 144
3.4 FUNCIONES ...................................................................................................... 151
3.4.1 Conceptos básicos ............................................................................................. 151
3.4.2 La función de primer grado (Ecuación de la recta) .............................................. 162
3.4.3 Tipos de funciones. Función inversa ................................................................... 175
3.4.4 Funciones de primer grado simultáneas. Sistemas de ecuaciones de primer grado .............. 186

560 Índice

indice final 560-568 560 25/11/02, 1:33 PM


3.4.5 Inecuaciones con dos variables. Sistemas y problemas de programación lineal ......... 211
PRUEBA DE SELECCIÓN MÚLTIPLE ................................................................... 221

Capítulo 4
Ecuaciones e inecuaciones de segundo grado
4.1 ECUACIÓN CUADRÁTICA ................................................................................ 227
4.1.1 Solución de la ecuación por factorización ......................................................... 227
4.1.2 Solución de la ecuación cuadrática aplicando la fórmula general ...................... 230
4.1.3 Ecuaciones bicuadráticas ................................................................................... 233
4.1.4 Relación entre los coeficientes de una ecuación cuadrática
y sus raíces o soluciones y naturaleza de ellas .................................................... 235
4.2 LA FUNCIÓN CUADRÁTICA ............................................................................. 240
4.3 INECUACIONES DE SEGUNDO GRADO .......................................................... 246
4.4 SISTEMAS DE ECUACIONES DE SEGUNDO GRADO ....................................... 251
4.4.1 Sistemas que contienen una ecuación lineal y una ecuación cuadrática ............ 251
4.4.2 Sistemas en que ambas ecuaciones son de la forma ax2 ± by2 = c ...................... 253
4.4.3 Sistemas formados por una ecuación de la forma x2 ± y2 = a
y la otra ecuación, de la forma xy = b .............................................................. 256
4.4.4 Sistemas homogéneos formados por ecuaciones cuyos términos
son todos de segundo grado ............................................................................... 259
4.4.5 Otros sistemas y problemas ................................................................................ 262
PRUEBA DE SELECCIÓN MÚLTIPLE ................................................................... 267

Capítulo 5
Polinomios y teoría de ecuaciones
5.1 DEFINICIÓN Y OPERACIONES CON POLINOMIOS ......................................... 272
5.1.1 Suma .................................................................................................................. 273
5.1.2 Resta .................................................................................................................. 273
5.1.3 Producto ............................................................................................................ 273
5.1.4 División ............................................................................................................. 274
5.2 TEORÍA DE ECUACIONES ................................................................................. 283
5.2.1 Cálculo de las raíces de un polinomio. Factorización ........................................ 283
5.2.2 Relación entre los coeficientes de una ecuación P (x) = 0 y sus raíces ................ 284
PRUEBA DE SELECCIÓN MÚLTIPLE ................................................................... 291

Capítulo 6
Potencias y Raíces
6.1 POTENCIAS ....................................................................................................... 295
6.1.1 Potencias de exponente natural .......................................................................... 295
6.1.2 Potencias de exponente cero y exponente entero negativo ................................. 295
6.2 PROPIEDADES DE LAS POTENCIAS .................................................................. 299
6.2.1 Multiplicación de potencias de igual base .......................................................... 299
6.2.2 División de potencias de igual base ................................................................... 299
6.2.3 Elevación de potencia a potencia ....................................................................... 299
6.2.4 Multiplicación de potencias de igual exponente ................................................ 299
6.2.5 División de potencias de igual exponente .......................................................... 300
6.2.6 Potencia de un producto ................................................................................... 300
6.2.7 Potencia de un cociente ..................................................................................... 300
6.3 ECUACIONES EXPONENCIALES ........................................................................ 304
Índice 561

indice final 560-568 561 25/11/02, 1:33 PM


Indice
6.4 RAÍCES ............................................................................................................... 307
6.5 PROPIEDADES ................................................................................................... 307
6.5.1 Potencia de exponente fraccionario ................................................................... 307
6.5.2 Multiplicación de raíces de igual índice ............................................................. 308
6.5.3 División de raíces de igual índice ....................................................................... 308
6.5.4 Raíz de una raíz ................................................................................................. 308
6.6 RACIONALIZACIÓN .......................................................................................... 318
6.6.1 Técnicas de racionalización ............................................................................... 318
6.7 ECUACIONES IRRACIONALES ........................................................................... 320
PRUEBA DE SELECCIÓN MÚLTIPLE ................................................................... 323

Capítulo 7
Logaritmos
7.1 DEFINICIÓN DE LOGARITMO .......................................................................... 329
7.2 PROPIEDADES ................................................................................................... 330
7.3 ECUACIONES EXPONENCIALES Y LOGARÍTMICAS .......................................... 340
PRUEBA DE SELECCIÓN MÚLTIPLE ................................................................... 350

Capítulo 8
Trigonometría
8.1 SISTEMAS DE MEDICIÓN DE ÁNGULOS ........................................................... 353
8.2 RAZONES TRIGONOMÉTRICAS PARA ÁNGULOS AGUDOS ............................ 354
8.3 IDENTIDADES TRIGONOMÉTRICAS .................................................................. 354
8.4 FUNCIONES TRIGONOMÉTRICAS DE UN ÁNGULO CUALQUIERA ................ 355
8.5 FUNCIONES TRIGONOMÉTRICAS DE 60°, 30° y 45°, 0°, 90°, 180° y 270° ...... 355
8.6 FUNCIONES PERIÓDICAS.................................................................................. 356
8.7 FUNCIONES PARES E IMPARES ......................................................................... 356
8.8 ECUACIONES TRIGONOMÉTRICAS .................................................................. 356
8.9 RESOLUCIÓN DE TRIÁNGULOS NO RECTÁNGULOS ..................................... 357
8.9.1 Teorema del seno (o de los senos) ...................................................................... 357
8.9.2 Teorema del coseno (o de los cosenos) ............................................................... 357
8.9.3 Ángulos de elevación y depresión ...................................................................... 357
PRUEBA DE SELECCIÓN MÚLTIPLE ................................................................... 376

Capítulo 9
Números Complejos
9.1 DEFINICIONES Y PROPIEDADES ....................................................................... 379
9.1.1 Igualdad ............................................................................................................. 379
9.1.2 Representación geométrica ................................................................................ 379
9.1.3 Forma canónica de un complejo ........................................................................ 380
9.1.4 Operaciones con números complejos ................................................................ 380
9.1.5 Estructura del conjunto (k , + , • ) ..................................................................... 380
9.1.6 Potencias de i ..................................................................................................... 381
9.2 CONJUGADO Y MÓDULO DE UN COMPLEJO ............................................... 390
9.2.1 Conjugado de un complejo ................................................................................ 390
9.2.2 Módulo de un complejo ..................................................................................... 391
9.3 REPRESENTACIÓN TRIGONOMÉTRICA O FORMA POLAR DE UN
NÚMERO COMPLEJO ....................................................................................... 397
9.3.1 Definición de razones trigonométricas ............................................................... 397

562 Índice

indice final 560-568 562 25/11/02, 1:33 PM


9.3.2 Representación trigonométrica del complejo z = a + bi ...................................... 397
9.3.3 Producto y cociente de complejos en forma polar .............................................. 398
9.3.4 Potenciación de números complejos en forma polar .......................................... 398
9.3.5 Radicación de números complejos en forma polar ............................................. 399
PRUEBA DE SELECCIÓN MÚLTIPLE ................................................................... 409

Capítulo 10
Vectores
10.1 DEFINICIONES .................................................................................................. 413
10.2 OPERACIONES CON VECTORES ....................................................................... 414
10.2.1 Suma de vectores ............................................................................................... 414
10.2.2 Producto por escalar .......................................................................................... 415
10.2.3 Propiedades de la suma y el producto por escalar ............................................... 415
10.2.4 Resta de vectores ................................................................................................ 416
10.3 VECTOR UNITARIO ........................................................................................... 416
10.3.1 Definición .......................................................................................................... 416
10.3.2 Normalizar un vector ......................................................................................... 417
10.4 DESCOMPOSICIÓN DE UN VECTOR ................................................................ 418
10.5 PRODUCTO PUNTO (O PRODUCTO ESCALAR) .............................................. 426
10.5.1 Definición .......................................................................................................... 426
10.5.2 Propiedades ....................................................................................................... 426
10.5.3 Ángulo entre vectores ......................................................................................... 426
10.5.4 Proyección de un vector sobre otro .................................................................... 427
10.6 VECTORES EN EL ESPACIO R3 .......................................................................... 433
10.6.1 Definiciones ....................................................................................................... 433
10.6.2 Producto vectorial o producto cruz ..................................................................... 434
PRUEBA DE SELECCIÓN MÚLTIPLE ................................................................... 439

Capítulo 11
Matrices y determinantes
11.1 CONCEPTOS BÁSICOS ...................................................................................... 443
11.2 IGUALDAD Y ADICIÓN DE MATRICES ............................................................. 445
11.2.1 Matrices iguales ................................................................................................. 445
11.2.2 Adición de matrices ........................................................................................... 445
11.2.3 Propiedades de la adición .................................................................................. 446
11.3 PONDERACIÓN DE UNA MATRIZ POR UN ESCALAR ..................................... 450
11.3.1 Definición .......................................................................................................... 450
11.3.2 Propiedades ....................................................................................................... 450
11.4 MULTIPLICACIÓN DE MATRICES ...................................................................... 454
11.4.1 Procedimiento .................................................................................................... 454
11.4.2 Propiedades de la multiplicación ....................................................................... 455
11.4.3 Matrices inversas y ecuaciones multiplicativas ................................................... 456
11.5 DETERMINANTES Y SISTEMAS DE ECUACIONES .............................................. 462
11.5.1 Determinantes y Sistemas lineales de orden 2 .................................................... 462
11.5.2 Determinantes y Sistemas lineales de orden 3 .................................................... 463
PRUEBA DE SELECCIÓN MÚLTIPLE ................................................................... 468

Índice 563

indice final 560-568 563 25/11/02, 1:33 PM


Indice
Capítulo 12
Sumatoria y progresiones
12.1 SUMATORIA ...................................................................................................... 473
12.2 SUCESIONES ..................................................................................................... 482
12.2.1 Definición .......................................................................................................... 482
12.2.2 Sucesiones convergentes .................................................................................... 484
12.2.3 Sucesiones divergentes ....................................................................................... 485
12.2.4 Sucesiones crecientes y decrecientes ................................................................. 486
12.3 PROGRESIÓN ARITMÉTICA ............................................................................... 488
12.4 PROGRESIÓN GEOMÉTRICA ............................................................................ 494
12.4.1 Definición .......................................................................................................... 494
12.4.2 Cálculo de intereses de capital ........................................................................... 495
12.5 PROGRESIÓN ARMÓNICA ................................................................................ 506
12.6 INDUCCIÓN MATEMÁTICA .............................................................................. 509
PRUEBA DE SELECCIÓN MÚLTIPLE ................................................................... 515

Capítulo 13
Análisis combinatorio, Teorema del binomio y Elementos de probabilidades

13.1 ANÁLISIS COMBINATORIO ............................................................................... 519


13.1.1 Conceptos básicos ............................................................................................. 519
13.1.2 Permutaciones .................................................................................................... 519
13.1.3 Arreglos o variaciones ........................................................................................ 520
13.1.4 Combinaciones .................................................................................................. 520
13.2 TEOREMA DEL BINOMIO ................................................................................. 528
13.2.1 Conceptos y observaciones básicas .................................................................... 528
13.2.2 Teorema del binomio ......................................................................................... 529
13.2.3 El triángulo de Pascal ......................................................................................... 530
13.3 ELEMENTOS DE PROBABILIDADES ................................................................... 534
13.3.1 Conceptos básicos ............................................................................................. 534
13.3.2 Probabilidad de la unión y de la intersección de dos eventos ............................. 535
PRUEBA DE SELECCIÓN MÚLTIPLE ................................................................... 539

Capítulo 14
Problemas
14.1 APLICACIÓN DE ECUACIONES LINEALES ENTERAS ......................................... 545
14.2 APLICACIÓN DE ECUACIONES LINEALES FRACCIONARIAS ............................ 550
14.3 APLICACIÓN DE SISTEMAS DE ECUACIONES LINEALES .................................. 553
14.4 PROBLEMAS MISCELÁNEOS ............................................................................. 557

564 Índice

indice final 560-568 564 25/11/02, 1:33 PM


Indice Analítico
Álgebra, concepto de, 3 Logarítmica, 340
Análisis combinatorio, 519 Lineal, 545
Principio de la Lineal entera, 545
multiplicación en el, 519 Linealmente dependiente, 186
Principio de la suma en el, 519 Linealmente independiente, 186
Ángulo, 357 Literales, 73
Arreglos, 520 Sistema de, 186, 251, 553
de depresión, 357 Eliminación por igualación, 191
de elevación, 357 Eliminación por reducción, 189
Axioma, 123 Eliminación por sustitución, 190
Axioma de Peano, 509 Espacio muestral, 534
Coeficiente binomial, 528 Experimento aleatorio, 534
Combinaciones, 520 Experimento determinístico, 534
Conjunto, 123 Expresión algebraica, 11
Cardinalidad de un, 124 División de, 45
Clase de equivalencia entre, 144 Factorización de, 29
Conjunto universo,124 Multiplicación de, 19, 45
Conjunto vacío, 123 Término de una,11
Diferencia simétrica,130 Factor común compuesto, 32
Equivalencias de, 124 Factorial de un número, 519
Esquema de los Factorización, 29
conjuntos numéricos, 5 Fracciones algebraicas, 43
Idempotencia,117 Adición y sustracción de, 50
Igualdad de, 124 Multiplicación y división de, 45
Leyes de la asociatividad entre, 117 Función, 151
Leyes de la conmutatividad entre, 117 Composición de, 152
Leyes de la distributividad entre, 118 Dominio de una, 152
Leyes de Identidad, 117 Función biyectiva, 175
Leyes de Morgan, 117 Función constante, 152
Operaciones entre, 129 Función cuadrática, 240
Potencia de un, 124 Función epiyectiva, 175
Propiedades de la relación Función idéntica, 152
de inclusión entre, 130 Función inversa, 175
Subconjunto, 124 Función inyectiva, 175
Conjuntos numéricos, 4 Función preposicional o
Esquema de los , 4 preposición abierta, 111
Contradicción, 113 Funciones de primer grado, 186
Desigualdad, 89 Parte entera de una, 152
Determinantes, 462 Rango o recorrido de una, 152
Ecuación, 60 ,187, 227 Valor absoluto de una, 152
Bicuadrática, 233 Inducción matemática, 509
Con valor absoluto, 79 Inecuación, 89
Cuadrática, 230 con dos variables, 211
De primer grado, 61 con valor absoluto, 100
Exponencial, 304 de primer grado, 60
Fraccionaria, 550 de segundo grado, 246
Irracional, 320

Índice Analítico 565

indice final 560-568 565 25/11/02, 1:33 PM


Indice Analítico
simultáneas, 97 de la unión y de
Interés compuesto, 495 la intersección, 535
Intersecciones con los ejes, 240-241 de un evento cierto, 535
Lenguaje algebraico, 7 de un evento imposible, 535
Logaritmo, 329 Producto cartesiano, 136
Matrices, 443 Productos notables, 24
Adición de, 445 Programación lineal, 211
Determinante de orden 2, 462 Progresión aritmética, 488
Determinante de orden 3, 463 Progresión armónica, 506
Igualdad de, 445 Progresión geométrica, 494
Multiplicación de, 454 Propiedad telescópica, 474
Orden o dimensión de una, 443 Proposición, 111
Producto matriz-escalar, 450 Dominio o universo de una, 111
Regla de Cramer, 464 Negación de una, 111
Medios aritméticos, 488 Raíces, 307
Interpolación de, 488 Racionalización, 318
Medios geométricos, 495 Radián, 353
Interpolación de, 495 Recta, 162-163
Multiplicación algebraica, 19 Coeficiente de
Números complejos, 379-381 posición de una, 162
Conjugado y sus propiedades, 390 Ecuación de la recta
Módulo y sus propiedades, 391 dados dos puntos, 162
Propiedades de la suma de, 380 Intersección de la recta
Propiedades de producto de, 381 con los ejes, 165
Optimización, 211 Familia de, 166
Parábola, 240 Pendiente de la, 162
Concavidad de la, 240 Rectas paralelas, 163
Discriminante de la, 241 Rectas perpendiculares, 163
Vértice de la, 241 Regla de Cramer, 464
Permutaciones, 520 Relación, 137,144,149
Polinomio, 272-273 Dominio de una, 137
Definición de, 272 Gráfico cartesiano de una, 137
Grado de un, 272 Gráfico Sagital de una, 137
Operaciones con, 273 Propiedades de una, 144
Raíces complejas e Rango o recorrido de una, 137
irracionales de un, 284 Relación de equivalencia, 144
Raíces racionales de un, 284 Relación de orden, 145
Raíz de un, 283 Relación inversa, 137
Potencia, 295 Relaciones y funciones, 111
Potencia de exponente Sistema de ecuaciones
fraccionario, 307 de primer grado,186
Potencia de un número, 295 Sistema de ecuaciones
Propiedades de las, 299 de segundo grado, 251
Principio multiplicativo, 519 Sistema inconsistente, 186
Principio de la suma, 519 Sistema indeterminado, 186
Probabilidad, 534 Sistemas lineales en orden 2, 462
de eventos complementarios, 535 Sistemas lineales en orden 3, 463

566 Índice Analítico

indice final 560-568 566 25/11/02, 1:34 PM


Sucesiones, 482 Valoración de expresiones
Sumatoria, 473 algebraicas, 12
Tautología, 113 Valor absoluto, 79
Teorema, 123 Vector unitario, 416
Teorema del binomio, 529 Vectores, 413,427
Teorema del Coseno, 357 Descomposición de, 418
Teorema del Seno, 357 Magnitud, dirección
Teoría de ecuaciones, 283 y sentido de, 413
Término semejante, 14 Módulo o norma de, 417
Triángulo de Pascal, 530 Operaciones con, 414
Trigonometría, 353 Producto Punto o escalar, 426
Ecuación trigonométrica, 356 Vectores en R3, 433
Identidad trigonométrica, 354 Vectores ortogonales, 427
Trinomio ordenado, 37 Vectores paralelos, 427

Índice Analítico 567

indice final 560-568 567 25/11/02, 1:34 PM


indice final 560-568 568 25/11/02, 1:34 PM

También podría gustarte